diff --git "a/wiki/proofwiki/shard_1.txt" "b/wiki/proofwiki/shard_1.txt" new file mode 100644--- /dev/null +++ "b/wiki/proofwiki/shard_1.txt" @@ -0,0 +1,20229 @@ +\section{Conjugate of Subgroup is Subgroup} +Tags: Conjugacy, Subgroups, Conjugate of Subgroup is Subgroup + +\begin{theorem} +Let $G$ be a [[Definition:Group|group]]. +Let $H \le G$ be a [[Definition:Subgroup|subgroup]] of $G$. +Then the [[Definition:Conjugate of Group Subset|conjugate]] of $H$ by $a$ is a [[Definition:Subgroup|subgroup]] of $G$: +:$\forall H \le G, a \in G: H^a \le G$ +\end{theorem} + +\begin{proof} +Let $H \le G$. +First, we show that $x, y \in H^a \implies x \circ y \in H^a$: +{{begin-eqn}} +{{eqn | l = x, y + | o = \in + | r = H^a + | c = +}} +{{eqn | ll= \leadsto + | l = a x a^{-1}, a y a^{-1} + | o = \in + | r = H + | c = {{Defof|Conjugate of Group Subset}} +}} +{{eqn | ll= \leadsto + | l = \paren {a x a^{-1} } \paren {a y a^{-1} } + | o = \in + | r = H + | c = {{GroupAxiom|0}} +}} +{{eqn | ll= \leadsto + | l = a \paren {x y} a^{-1} + | o = \in + | r = H + | c = +}} +{{eqn | ll= \leadsto + | l = x y + | o = \in + | r = H^a + | c = {{Defof|Conjugate of Group Subset}} +}} +{{end-eqn}} +Next, we show that $x \in H^a \implies x^{-1} \in H^a$: +{{begin-eqn}} +{{eqn | l = x + | o = \in + | r = H^a + | c = +}} +{{eqn | ll= \leadsto + | l = a x a^{-1} + | o = \in + | r = H + | c = {{Defof|Conjugate of Group Subset}} +}} +{{eqn | ll= \leadsto + | l = \paren {a x a^{-1} }^{-1} = a x^{-1} a^{-1} + | o = \in + | r = H + | c = [[Power of Conjugate equals Conjugate of Power]] +}} +{{eqn | ll= \leadsto + | l = x^{-1} + | o = \in + | r = H^a + | c = {{Defof|Conjugate of Group Subset}} +}} +{{end-eqn}} +Thus by the [[Two-Step Subgroup Test]], $H^a \le G$. +{{qed}} +\end{proof} + +\begin{proof} +Let $*: G \times G / H \to G / H$ be the [[Definition:Group Action on Coset Space|group action on the (left) coset space]]: +:$\forall g \in G, \forall g' H \in G / H: g * \paren {g' H} := \paren {g g'} H$ +It is established in [[Action of Group on Coset Space is Group Action]] that $*$ is a [[Definition:Group Action|group action]]. +Then from [[Stabilizer of Coset under Group Action on Coset Space]]: +:$\Stab {a H} = a H a^{-1}$ +where $\Stab {a H}$ the [[Definition:Stabilizer|stabilizer]] of $a H$ under $*$. +The result follows from [[Stabilizer is Subgroup]]. +{{qed}} +\end{proof}<|endoftext|> +\section{Inner Automorphisms form Normal Subgroup of Automorphism Group} +Tags: Normal Subgroups, Automorphism Groups, Inner Automorphisms + +\begin{theorem} +Let $G$ be a [[Definition:Group|group]]. +Then the [[Definition:Set|set]] $\Inn G$ of all [[Definition:Inner Automorphism|inner automorphisms]] of $G$ is a [[Definition:Normal Subgroup|normal subgroup]] of the [[Definition:Automorphism Group of Group|automorphism group]] $\Aut G$ of $G$: +:$\Inn G \lhd \Aut G$ +\end{theorem} + +\begin{proof} +Let $G$ be a [[Definition:Group|group]] whose [[Definition:Identity Element|identity]] is $e$. +From [[Inner Automorphisms form Subgroup of Automorphism Group]], $\Inn G$ forms a [[Definition:Subgroup|subgroup]] of $\Aut G$. +It remains to be shown that $\Inn G$ is [[Definition:Normal Subgroup|normal in $\Aut G$]]. +Let $\kappa_x: G \to G$ be the [[Definition:Inner Automorphism|inner automorphism]] defined as: +:$\forall g \in G: \map {\kappa_x} g = x g x^{-1}$ +Let $\phi \in \Aut G$. +If we can show that: +:$\forall \phi \in \Aut G: \forall \kappa_x \in \Inn G: \phi \circ \kappa_x \circ \phi^{-1} \in \Inn G$ +then by the [[Normal Subgroup Test]]: +:$\Inn G \lhd \Aut G$ +Fix $\kappa_x \in \Inn G$. +We claim $\phi \circ \kappa_x \circ \phi^{-1} = \kappa_{\map \phi x}$. +Since $\phi \in \Aut G$ then $\phi$ is, in particular, a [[Definition:Group Homomorphism|homomorphism]]. +Therefore: +{{begin-eqn}} +{{eqn | ll= \forall g \in G: + | l = \map {\paren {\phi \circ \kappa_x \circ \phi^{-1} } } g + | r = \map {\paren {\phi \circ \kappa_x} } {\map {\phi^{-1} } g} + | c = +}} +{{eqn | r = \map \phi {x \phi^{-1} \paren g x^{-1} } + | c = +}} +{{eqn | r = \map \phi x \map \phi {\map {\phi^{-1} } g} \map \phi {x^{-1} } + | c = +}} +{{eqn | r = \map \phi x g \map \phi x^{-1} + | c = +}} +{{eqn | r = \map {\kappa_{\map \phi x} } g + | c = +}} +{{end-eqn}} +Therefore: +:$\phi \circ \kappa_g \circ \phi^{-1} = \kappa_{\map \phi g} \in \Inn G$ +Since $\kappa_x \in \Inn G$ and $\phi \in \Aut G$ were arbitrary: +:$\Inn G \lhd \Aut G$ +{{qed}} +\end{proof}<|endoftext|> +\section{Subgroup of Abelian Group is Normal} +Tags: Normal Subgroups, Abelian Groups + +\begin{theorem} +Every [[Definition:Subgroup|subgroup]] of an [[Definition:Abelian Group|abelian group]] is [[Definition:Normal Subgroup|normal]]. +\end{theorem} + +\begin{proof} +Let $H \le G$ where $G$ is [[Definition:Abelian Group|abelian]]. +Then: +{{begin-eqn}} +{{eqn | l = y + | o = \in + | r = H^a + | c = where $H^a$ is the [[Definition:Conjugate of Group Subset|conjugate of $H$ by $a$]] +}} +{{eqn | ll= \leadstoandfrom + | l = a y a^{-1} + | o = \in + | r = H + | c = {{Defof|Conjugate of Group Subset}} +}} +{{eqn | ll= \leadstoandfrom + | l = y + | o = \in + | r = H + | c = [[Conjugate of Commuting Elements|because $a y a^{-1} = y$]] as $G$ is [[Definition:Abelian Group|abelian]] +}} +{{end-eqn}} +{{Qed}} +\end{proof}<|endoftext|> +\section{Subgroup is Normal iff it contains Product of Inverses} +Tags: Normal Subgroups + +\begin{theorem} +A [[Definition:Subgroup|subgroup]] $H$ of a [[Definition:Group|group]] $G$ is [[Definition:Normal Subgroup|normal]] {{iff}}: +:$\forall a, b \in G: a b \in H \implies a^{-1} b^{-1} \in H$ +\end{theorem} + +\begin{proof} +=== Necessary Condition === +Suppose $H$ is [[Definition:Normal Subgroup|normal]]. +Let $a b \in H$. +{{begin-eqn}} +{{eqn | l = a b + | o = \in + | r = H + | c = +}} +{{eqn | ll= \leadsto + | l = a^{-1} \left({a b}\right) a + | o = \in + | r = H + | c = [[Subgroup is Normal iff Contains Conjugate Elements]] +}} +{{eqn | ll= \leadsto + | l = b a + | o = \in + | r = H + | c = [[Definition:Group|Group properties]] +}} +{{end-eqn}} +As $H$ is a [[Definition:Group|group]], then $x \in H \iff x^{-1} \in H$. +So $b a \in H \iff \left({b a}\right)^{-1} \in H \iff a^{-1} b^{-1} \in H$. +Thus $\forall a, b \in G: a b \in H \implies a^{-1} b^{-1} \in H$. +=== Sufficient Condition === +Suppose $a b \in H \implies a^{-1} b^{-1} \in H$. +Then (from the above) we have $\forall a, b \in H: a b \in H \implies b a \in H$. +Let $g \in G$, and let $h \in H$. Let $h = x g$ where $x \in G$. +Now $g h g^{-1} = g x g g^{-1} = g x$. +By hypothesis, $x g \in H \implies g x \in H$. +The result follows. +{{qed}} +\end{proof}<|endoftext|> +\section{Intersection with Normal Subgroup is Normal} +Tags: Normal Subgroups, Intersection with Normal Subgroup is Normal + +\begin{theorem} +Let $G$ be a [[Definition:Group|group]]. +Let $H$ be a [[Definition:Subgroup|subgroup]] of $G$, and let $N$ be a [[Definition:Normal Subgroup|normal subgroup]] of $G$. +Then $H \cap N$ is a [[Definition:Normal Subgroup|normal subgroup]] of $H$. +\end{theorem} + +\begin{proof} +By [[Intersection of Subgroups is Subgroup]], $H \cap N$ is a [[Definition:Subgroup|subgroup]] of $N$. +It remains to be shown that $H$ is [[Definition:Normal Subgroup|normal]] in $H$. +Because $N \lhd G$: +:$\forall n \in N: \forall g \in G: g n g^{-1} \in N$ +Let $x \in H \cap N$. +Because $H \le G$ and therefore [[Definition:Closed Algebraic Structure|closed]]: +:$\forall x \in H \cap N: \forall g \in H: g x g^{-1} \in H$ +But since $x \in N$ and $N \lhd G$, $g x g^{-1} \in N$. +The result follows. +{{qed}} +\end{proof}<|endoftext|> +\section{Kernel is Normal Subgroup of Domain} +Tags: Normal Subgroups, Kernels of Group Homomorphisms + +\begin{theorem} +Let $\phi$ be a [[Definition:Group Homomorphism|group homomorphism]]. +Then the [[Definition:Kernel of Group Homomorphism|kernel]] of $\phi$ is a [[Definition:Normal Subgroup|normal subgroup]] of the [[Definition:Domain of Mapping|domain]] of $\phi$: +:$\map \ker \phi \lhd \Dom \phi$ +\end{theorem} + +\begin{proof} +Let $\phi: G_1 \to G_2$ be a [[Definition:Group Homomorphism|group homomorphism]], where the [[Definition:Identity Element|identities]] of $G_1$ and $G_2$ are $e_{G_1}$ and $e_{G_2}$ respectively. +By [[Kernel of Group Homomorphism is Subgroup]]: +:$\map \ker \phi \le \Dom \phi$ +Let $k \in \map \ker \phi, x \in G_1$. +Then: +{{begin-eqn}} +{{eqn | l = \map \phi {x k x^{-1} } + | r = \map \phi x \map \phi k \paren {\map \phi x}^{-1} + | c = [[Homomorphism to Group Preserves Inverses]] +}} +{{eqn | r = \map \phi x e_{G_2} \paren {\map \phi x}^{-1} + | c = {{Defof|Kernel of Group Homomorphism}} +}} +{{eqn | r = \map \phi x \paren {\map \phi x}^{-1} + | c = {{Defof|Identity Element}} +}} +{{eqn | r = e_{G_2} + | c = {{Defof|Inverse Element}} +}} +{{end-eqn}} +So: +:$x k x^{-1} \in \map \ker \phi$ +This is true for all $k \in \map \ker \phi$ and $x \in G_1$. +From [[Subgroup is Normal iff Contains Conjugate Elements]], it follows that $\map \ker \phi$ is a [[Definition:Normal Subgroup|normal subgroup]] of $G_1$. +{{qed}} +\end{proof}<|endoftext|> +\section{Coset Product is Well-Defined} +Tags: Coset Product, Coset Product is Well-Defined + +\begin{theorem} +Let $\struct {G, \circ}$ be a [[Definition:Group|group]]. +Let $N$ be a [[Definition:Normal Subgroup|normal subgroup]] of $G$. +Let $a, b \in G$. +Then the [[Definition:Coset Product|coset product]]: +:$\paren {a \circ N} \circ \paren {b \circ N} = \paren {a \circ b} \circ N$ +is [[Definition:Well-Defined Operation|well-defined]]. +\end{theorem} + +\begin{proof} +Let $N \lhd G$ where $G$ is a [[Definition:Group|group]]. +Let $a, a', b, b' \in G: a \circ N = a' \circ N, b \circ N = b' \circ N$. +To show that the coset product is [[Definition:Well-Defined Operation|well-defined]], we need to demonstrate that $\paren {a \circ b} \circ N = \paren {a' \circ b'} \circ N$. +So: +{{begin-eqn}} +{{eqn | l = a \circ N + | r = a' \circ N + | c = +}} +{{eqn | ll= \leadsto + | l = a^{-1} \circ a' + | o = \in + | r = N + | c = [[Cosets are Equal iff Product with Inverse in Subgroup]] +}} +{{eqn | ll= \leadsto + | l = b^{-1} \circ a^{-1} \circ a' + | o = \in + | r = b^{-1} \circ N + | c = {{Defof|Subset Product}} +}} +{{eqn | ll= \leadsto + | l = b^{-1} \circ a^{-1} \circ a' + | o = \in + | r = N \circ b^{-1} + | c = $N$ is a [[Definition:Normal Subgroup|normal subgroup]] +}} +{{eqn | ll= \leadsto + | l = \exists n \in N: b^{-1} \circ a^{-1} \circ a' + | r = n \circ b^{-1} + | c = {{Defof|Subset Product}} +}} +{{eqn | ll= \leadsto + | l = \paren {a \circ b}^{-1} \circ \paren {a' \circ b'} + | r = n \circ b^{-1} \circ b' + | c = [[Definition:Group|Group Properties]] +}} +{{eqn | ll= \leadsto + | l = \paren {a \circ b}^{-1} \circ \paren {a' \circ b'} + | o = \in + | r = N + | c = {{Defof|Subset Product}} +}} +{{end-eqn}} +By [[Cosets are Equal iff Product with Inverse in Subgroup]]: +:$\paren {a \circ b}^{-1} \circ \paren {a' \circ b'} \in N \implies \paren {a \circ b} \circ N = \paren {a' \circ b'} \circ N$ +and the proof is complete. +{{qed}} +\end{proof} + +\begin{proof} +Let $N \lhd G$ where $G$ is a [[Definition:Group|group]]. +Consider $\paren {a \circ N} \circ \paren {b \circ N}$ as a [[Definition:Subset Product|subset product]]: +:$\paren {a \circ N} \circ \paren {b \circ N} = \set {a \circ n_1 \circ b \circ n_2: n_1, n_2 \in N}$ +This is justified by [[Coset Product of Normal Subgroup is Consistent with Subset Product Definition]]. +Since $N$ is [[Definition:Normal Subgroup|normal]], each [[Definition:Conjugate (Group Theory)|conjugate]] $b^{-1} \circ N \circ b$ is contained in $N$. +So for each $n_1 \in N$ there is some $n_3 \in N$ such that $b^{-1} \circ n_1 \circ b = n_3$. +So, if $a \circ n_1 \circ b \circ n_2 \in \paren {a \circ N} \circ \paren {b \circ N}$, it follows that: +{{begin-eqn}} +{{eqn | l = a \circ n_1 \circ b \circ n_2 + | r = a \circ b \circ b^{-1} \circ n_1 \circ b \circ n_2 + | c = +}} +{{eqn | r = a \circ b \circ n_3 \circ n_2 + | c = +}} +{{eqn | o = \in + | r = \paren {a \circ b} \circ N + | c = {{Defof|Subset Product}} +}} +{{eqn | o = \in + | r = N \circ b^{-1} + | c = {{Defof|Normal Subgroup}} +}} +{{end-eqn}} +That is: +:$\paren {a \circ N} \circ \paren {b \circ N} \subseteq \paren {a \circ b} \circ N$ +Then: +{{begin-eqn}} +{{eqn | l = a \circ b \circ n + | o = \in + | r = \paren {a \circ b} \circ N + | c = +}} +{{eqn | ll= \leadsto + | l = a \circ e \circ b \circ n + | o = \in + | r = \paren {a \circ N} \circ \paren {b \circ N} + | c = +}} +{{eqn | ll= \leadsto + | l = \paren {a \circ b} \circ N + | o = \subseteq + | r = \paren {a \circ N} \circ \paren {b \circ N} + | c = +}} +{{end-eqn}} +So: +:$\paren {a \circ N} \circ \paren {b \circ N} \subseteq \paren {a \circ b} \circ N$ +and +:$\paren {a \circ b} \circ N \subseteq \paren {a \circ N} \circ \paren {b \circ N}$ +The result follows by definition of [[Definition:Set Equality/Definition 2|set equality]]. +{{qed}} +\end{proof} + +\begin{proof} +Let $N \lhd G$ where $G$ is a [[Definition:Group|group]]. +Let $a, a', b, b' \in G: N \circ a = N \circ a', N \circ b = N \circ b'$. +We need to show that $N \circ a \circ b = N \circ a' \circ b'$. +So: +{{begin-eqn}} +{{eqn | l = N \circ a \circ b + | r = N \circ a' \circ b + | c = as $N \circ a = N \circ a'$ +}} +{{eqn | r = a' \circ N \circ b + | c = $N \circ a' = a' \circ N$ as $N$ is [[Definition:Normal Subgroup|normal]] +}} +{{eqn | r = a' \circ N \circ b' + | c = as $N \circ b = N \circ b'$ +}} +{{eqn | r = N \circ a' \circ b' + | c = $N \circ a' = a' \circ N$ as $N$ is [[Definition:Normal Subgroup|normal]] +}} +{{end-eqn}} +{{qed}} +\end{proof} + +\begin{proof} +Let $N \lhd G$ where $G$ is a [[Definition:Group|group]]. +Let $a, b \in G$. +We have: +{{begin-eqn}} +{{eqn | l = \paren {a \circ N} \circ \paren {b \circ N} + | r = a \circ N \circ b \circ N + | c = [[Subset Product within Semigroup is Associative]] +}} +{{eqn | r = a \circ b \circ N \circ N + | c = {{Defof|Normal Subgroup}} +}} +{{eqn | r = \paren {a \circ b} \circ N + | c = [[Product of Subgroup with Itself]] +}} +{{end-eqn}} +{{qed}} +\end{proof} + +\begin{proof} +Let $N \lhd G$ where $G$ is a [[Definition:Group|group]]. +Let $a, a', b, b' \in G: a \circ N = a' \circ N, b \circ N = b' \circ N$. +To show that the coset product is [[Definition:Well-Defined Operation|well-defined]], we need to demonstrate that: +:$\paren {a \circ b} \circ N = \paren {a' \circ b'} \circ N$ +So: +{{begin-eqn}} +{{eqn | l = a \circ N + | r = a' \circ N + | c = +}} +{{eqn | ll= \leadsto + | l = a + | o = \in + | r = a' \circ N + | c = {{Defof|Left Coset}} +}} +{{eqn | ll= \leadsto + | l = a + | r = a' \circ n_1 + | c = for some $n_1 \in N$ +}} +{{eqn-intertext|Similarly, $b' {{=}} b \circ n_2$ for some $n_2 \in N$.}} +{{eqn | ll= \leadsto + | l = a' \circ b' + | r = a \circ n_1 \circ b \circ n_2 + | c = +}} +{{eqn-intertext|But $N \circ b {{=}} b \circ N$, as $N$ is [[Definition:Normal Subgroup|normal]], and so:}} +{{eqn | ll= \leadsto + | l = a' \circ b' + | r = a \circ b \circ n_3 \circ n_2 + | c = as $n_1 \circ b = b \circ n_3$ for some $n_3 \in N$ +}} +{{eqn | ll= \leadsto + | l = a' \circ b' + | o = \in + | r = a \circ b \circ N + | c = as $n_3 \circ n_2 \in N$ +}} +{{eqn | ll= \leadsto + | l = a' \circ b' \circ N + | r = a \circ b \circ N + | c = {{Defof|Left Coset}} +}} +{{end-eqn}} +{{qed}} +\end{proof}<|endoftext|> +\section{Quotient Group of Abelian Group is Abelian} +Tags: Quotient Groups + +\begin{theorem} +Let $G$ be an [[Definition:Abelian Group|abelian group]]. +Let $N \le G$. +Then the [[Definition:Quotient Group|quotient group]] $G / N$ is [[Definition:Abelian Group|abelian]]. +\end{theorem} + +\begin{proof} +First we note that because $G$ is [[Definition:Abelian Group|abelian]], from [[Subgroup of Abelian Group is Normal]] we have $N \lhd G$. +Thus $G / N$ exists for ''all'' [[Definition:Subgroup|subgroups]] of $G$. +Let $X = x N, Y = y N$ where $x, y \in G$. +From the definition of [[Definition:Coset Product|coset product]]: +{{begin-eqn}} +{{eqn | l = X Y + | r = \paren {x N} \paren {y N} + | c = +}} +{{eqn | r = \paren {x y N} + | c = +}} +{{eqn | r = \paren {y x N} + | c = +}} +{{eqn | r = \paren {y N} \paren {x N} + | c = +}} +{{eqn | r = Y X + | c = +}} +{{end-eqn}} +Thus $G / N$ is [[Definition:Abelian Group|abelian]]. +{{qed}} +[[Category:Quotient Groups]] +meo3u1osmjyzur71i3fnrzfyink4s68 +\end{proof}<|endoftext|> +\section{Quotient Theorem for Group Epimorphisms} +Tags: Quotient Groups, Group Epimorphisms, Group Isomorphisms, Named Theorems + +\begin{theorem} +Let $\struct {G, \oplus}$ and $\struct {H, \odot}$ be [[Definition:Group|groups]]. +Let $\phi: \struct {G, \oplus} \to \struct {H, \odot}$ be a [[Definition:Group Epimorphism|group epimorphism]]. +Let $e_G$ and $e_H$ be the [[Definition:Identity Element|identities]] of $G$ and $H$ respectively. +Let $K = \map \ker \phi$ be the [[Definition:Kernel of Group Homomorphism|kernel]] of $\phi$. +There is one and only one [[Definition:Group Isomorphism|group isomorphism]] $\psi: G / K \to H$ satisfying: +:$\psi \circ q_K = \phi$ +where $q_K$ is the [[Definition:Quotient Epimorphism|quotient epimorphism]] from $G$ to $G / K$. +\end{theorem} + +\begin{proof} +Let $\mathcal R_\phi$ be the [[Definition:Equivalence Relation Induced by Mapping|equivalence on $G$ defined by $\phi$]]. +{{begin-eqn}} +{{eqn | l = \forall x \in G: e_G + | o = \mathcal R_\phi + | r = x + | c = +}} +{{eqn | ll= \leadstoandfrom + | l = \map \phi x + | r = \map \phi {e_G} + | c = Definition of $\mathcal R_\phi$ +}} +{{eqn | ll= \leadstoandfrom + | l = \map \phi x + | r = e_H + | c = [[Homomorphism to Group Preserves Identity]] +}} +{{end-eqn}} +Thus: +:$K = \eqclass {e_G} {\mathcal R_\phi}$ +From the [[Quotient Theorem for Epimorphisms]]: +:$\mathcal R_\phi$ is compatible with $\oplus$ +Thus from [[Kernel is Normal Subgroup of Domain]]: +:$K \lhd G$ +From [[Congruence Relation induces Normal Subgroup]], $\mathcal R_\phi$ is the [[Definition:Equivalence Relation Induced by Mapping|equivalence defined by $K$]]. +Thus, again by [[Quotient Theorem for Epimorphisms]], there is a unique [[Definition:Group Epimorphism|epimorphism]] $\psi: G / K \to H$ satisfying $\psi \circ q_K = \phi$. +{{qed}} +\end{proof}<|endoftext|> +\section{Congruence Relation induces Normal Subgroup} +Tags: Normal Subgroups, Congruence Relations + +\begin{theorem} +Let $\left({G, \circ}\right)$ be a [[Definition:Group|group]] whose [[Definition:Identity Element|identity]] is $e$. +Let $\mathcal R$ be a [[Definition:Congruence Relation|congruence relation]] for $\circ$. +Let $H = \left[\!\left[{e}\right]\!\right]_\mathcal R$, where $\left[\!\left[{e}\right]\!\right]_\mathcal R$ is the [[Definition:Equivalence Class|equivalence class]] of $e$ under $\mathcal R$. +Then: +: $(1): \quad \left({H, \circ \restriction_H}\right)$ is a [[Definition:Normal Subgroup|normal subgroup]] of $G$ +: $(2): \quad \mathcal R$ is the [[Congruence Modulo Subgroup is Equivalence Relation|equivalence relation $\mathcal R_H$ defined by $H$]] +: $(3): \quad \left({G / \mathcal R, \circ_\mathcal R}\right)$ is the [[Definition:Subgroup|subgroup]] $\left({G / H, \circ_H}\right)$ of the [[Definition:Semigroup|semigroup]] $\left({\mathcal P \left({G}\right), \circ_\mathcal P}\right)$. +\end{theorem} + +\begin{proof} +=== [[Congruence Relation on Group induces Normal Subgroup|Proof of Normal Subgroup]] === +{{:Congruence Relation on Group induces Normal Subgroup}} +=== [[Normal Subgroup induced by Congruence Relation defines that Congruence|Proof of Equality of Relations]] === +{{:Normal Subgroup induced by Congruence Relation defines that Congruence}} +=== [[Quotient Structure on Group defined by Congruence equals Quotient Group|Proof that Quotient Structure is Quotient Group]] === +{{:Quotient Structure on Group defined by Congruence equals Quotient Group}} +\end{proof}<|endoftext|> +\section{Preimage of Normal Subgroup of Quotient Group under Quotient Epimorphism is Normal} +Tags: Group Isomorphisms, Group Epimorphisms, Quotient Groups + +\begin{theorem} +Let $G$ be a [[Definition:Group|group]]. +Let $H \lhd G$ where $\lhd$ denotes that $H$ is a [[Definition:Normal Subgroup|normal subgroup]] of $G$. +Let $K \lhd G/H$ and $L = q_H^{-1} \left[{K}\right]$, where: +:$q_H: G \to G/H$ is the [[Definition:Quotient Group Epimorphism|quotient epimorphism]] from $G$ to the [[Definition:Quotient Group|quotient group]] $G/H$ +:$q_H^{-1} \left[{K}\right]$ is the [[Definition:Preimage of Subset under Mapping|preimage of $K$ under $q_H$]]. +Then: +:$L \lhd G$ +\end{theorem} + +\begin{proof} +By [[Quotient Mapping on Structure is Canonical Epimorphism]], both $q_K$ and $q_H$ are [[Definition:Group Epimorphism|epimorphisms]]. +From [[Composite of Group Epimorphisms is Epimorphism]] we have that $q_K \circ q_H: G \to \left({G / H}\right) / K$ is also an [[Definition:Group Epimorphism|epimorphism]]. +Now: +:$\forall x \in G: x \in \ker \left({q_K \circ q_H}\right) \iff q_K \left({q_H \left({x}\right)}\right) = K = e_{G/H}$ +This means the same as: +:$q_H \left({x}\right) \in \ker \left({q_K}\right) = K$ +But: +:$q_H \left({x}\right) \in K \iff x \in q_H^{-1} \left({K}\right) = L$ +Thus: +:$L = \ker \left({q_K \circ q_H}\right)$ +By [[Kernel is Normal Subgroup of Domain]]: +: $L \lhd G$ +{{qed}} +\end{proof}<|endoftext|> +\section{Trivial Quotient Group is Quotient Group} +Tags: Quotient Groups, Group Isomorphisms + +\begin{theorem} +Let $G$ be a [[Definition:Group|group]]. +Then the [[Definition:Trivial Quotient Group|trivial quotient group]]: +:$G / \set {e_G} \cong G$ +where: +:$\cong$ denotes [[Definition:Group Isomorphism|group isomorphism]] +:$e_G$ denotes the [[Definition:Identity Element|identity element]] of $G$ +is a [[Definition:Quotient Group|quotient group]]. +\end{theorem} + +\begin{proof} +From [[Trivial Subgroup is Normal]]: +:$\set {e_G} \lhd G$ +Let $x \in G$. +Then: +:$x \set {e_G} = \set {x e_G} = \set x$ +So each [[Definition:Left Coset|(left) coset]] of $G$ modulo $\set {e_G}$ has one [[Definition:Element|element]]. +Now we set up the [[Definition:Quotient Group Epimorphism|quotient epimorphism]] $\psi: G \to G / \set {e_G}$: +:$\forall x \in G: \map \phi x = x \set {e_G}$ +which is of course a [[Definition:Surjection|surjection]]. +We now need to establish that it is an [[Definition:Injection|injection]]. +Let $p, q \in G$. +{{begin-eqn}} +{{eqn | l = \map \phi p + | r = \map \phi q + | c = +}} +{{eqn | ll= \leadsto + | l = p \set {e_G} + | r = q \set {e_G} + | c = Definition of $\phi$ +}} +{{eqn | ll= \leadsto + | l = \set p + | r = \set q + | c = from above +}} +{{eqn | ll= \leadsto + | l = p + | r = q + | c = {{Defof|Set Equality}} +}} +{{end-eqn}} +So $\psi$ is a [[Definition:Group Isomorphism|group isomorphism]] and therefore: +:$G / \set {e_G} \cong G$ +{{qed}} +\end{proof}<|endoftext|> +\section{Correspondence Theorem (Group Theory)} +Tags: Normal Subgroups, Quotient Groups, Named Theorems + +\begin{theorem} +Let $G$ be a [[Definition:Group|group]]. +Let $N \lhd G$ be a [[Definition:Normal Subgroup|normal subgroup]] of $G$. +Then every [[Definition:Subgroup|subgroup]] of the [[Definition:Quotient Group|quotient group]] $G / N$ is of the form $H / N = \set {h N: h \in H}$, where $N \le H \le G$. +Conversely, if $N \le H \le G$ then $H / N \le G / N$. +The correspondence between [[Definition:Subgroup|subgroups]] of $G / N$ and [[Definition:Subgroup|subgroups]] of $G$ containing $N$ is a [[Definition:Bijection|bijection]]. +This [[Definition:Bijection|bijection]] maps [[Definition:Normal Subgroup|normal subgroups]] of $G / N$ onto [[Definition:Normal Subgroup|normal subgroups]] of $G$ which contain $N$. +\end{theorem} + +\begin{proof} +Let $H'$ be a [[Definition:Subgroup|subgroup]] of $G / N$, so that it consists of a certain [[Definition:Set|set]] $\set {h N}$ of [[Definition:Left Coset|left cosets]] of $N$ in $G$. +Let us define the [[Definition:Subset|subset]] $\map \beta {H'} \subseteq G$: +:$\map \beta {H'} = \set {g \in G: g N \in H'}$ +Then clearly: +:$N \subseteq \map \beta {H'}$ +Also: +:$e_G \in N$ +so: +:$e_G \in \map \beta {H'}$ +Let $x, y \in \map \beta {H'}$. Then: +{{begin-eqn}} +{{eqn | l = x, y + | o = \in + | r = \map \beta {H'} + | c = +}} +{{eqn | ll= \leadsto + | l = x N, y N + | o = \in + | r = H' + | c = Definition of $\beta$ +}} +{{eqn | ll= \leadsto + | l = \paren {x N} \paren {y N} + | r = x y N \in H' + | c = {{Defof|Quotient Group}}: $G / N$ and as $H'$ is a subgroup of $G / N$ +}} +{{eqn | ll= \leadsto + | l = x y + | o = \in + | r = \map \beta {H'} + | c = Definition of $\beta$ +}} +{{end-eqn}} +We also have, from [[Quotient Group is Group]]: +:$\paren {x N}^{-1} = x^{-1} N \implies x^{-1} \in \map \beta {H'}$ +Thus, by the [[Two-Step Subgroup Test]], $\map \beta {H'} \le G$ that contains $N$. +Conversely, let $H$ be such that $N \le H \le G$. +Let $\map \alpha H = \set {h N: h \in H} \subseteq G / N$. +It is easily checked that $\map \alpha H \le G / N$. +Now, let $X$ be the [[Definition:Set|set]] of [[Definition:Subgroup|subgroups]] of $G$ containing $N$ and $Y$ be the [[Definition:Set|set]] of all [[Definition:Subgroup|subgroups]] of $G / N$. +We now need to show that $\alpha: X \to Y$ is a [[Definition:Bijection|bijection]]. +We do this by checking that $\beta: Y \to X$ is the inverse of $\alpha$. +To do this, we show that $\alpha \circ \beta = I_Y$ and $\beta \circ \alpha = I_X$. +Suppose $N \le H \le G$. Then: +{{begin-eqn}} +{{eqn | l = \map {\paren {\beta \circ \alpha} } H + | r = \map \beta {H / N} + | c = Definition of $\alpha$ +}} +{{eqn | r = \set {g \in G: g N \in H / N} + | c = Definition of $\beta$ +}} +{{eqn | r = H + | c = {{Defof|Coset}} $H / N$ +}} +{{end-eqn}} +Thus $\beta \circ \alpha = I_X$. +Now let $H' \le G / N$. Then: +{{begin-eqn}} +{{eqn | l = \map {\paren {\alpha \circ \beta} } {H'} + | r = \map \alpha {\set {g \in G: g N \in H'} } + | c = Definition of $\beta$ +}} +{{eqn | r = \set {g N \in H'} + | c = Definition of $\alpha$ +}} +{{eqn | r = H' + | c = Definition of $H'$ +}} +{{end-eqn}} +Thus $\alpha \circ \beta = I_Y$. +So, by [[Bijection iff Inverse is Bijection]], $\alpha$ is a [[Definition:Bijection|bijection]]. +Now let $H \lhd G$ such that $N \le H$. +We show that $\map \alpha H = H / N \lhd G / N$. +This follows by [[Definition:Normal Subgroup/Definition 3|definition 3 of Normal Subgroup]] because: for any $h \in H, g \in G$ +:$\paren {g N} \paren {h N} \paren {g N}^{-1} = g h g^{-1} N \in H / N$ +:$\paren {g N}^{-1} \paren {h N} \paren {g N} = g^{-1} h g N \in H / N$ +Conversely, let $H' \lhd G / N$. +Recall: +:$\map \beta {H'} = \set {g \in G : g N \in H'}$ +Hence, for any $x \in G$ we have: +{{begin-eqn}} +{{eqn | l = x \map \beta {H'} x^{-1} + | r = \set {x g x^{-1} \in G: g N \in H'} + | c = +}} +{{eqn | r = \set {y \in G: x^{-1} y x N \in H'} + | c = +}} +{{end-eqn}} +Now for any $h' \in \map \beta {H'}$, we have: +:$h'N \in H'$ +For all $x \in G$: +:$\paren {x^{-1} N} \paren {h' N} \paren {x N} \in H'$ +From $H' \lhd G / N$: +:$x^{-1} h' x N \in H'$ +This implies: +:$h' \in x \map \beta {H'} x^{-1}$ +or: +:$\map \beta {H'} \subseteq x \map \beta {H'} x^{-1}$ +Similarly, we can also show: +:$\map \beta {H'} \subseteq x^{-1} \map \beta {H'} x$ +Hence by [[Definition:Normal Subgroup/Definition 4|definition 4 of Normal Subgroup]]: +:$\map \beta {H'} \lhd G$ +{{qed}} +\end{proof}<|endoftext|> +\section{Centralizer of Group Element is Subgroup} +Tags: Subgroups, Centralizers, Centralizer of Group Element is Subgroup + +\begin{theorem} +Let $\struct {G, \circ}$ be a [[Definition:Group|group]] and let $a \in G$. +Then $\map {C_G} a$, the [[Definition:Centralizer of Group Element|centralizer]] of $a$ in $G$, is a [[Definition:Subgroup|subgroup]] of $G$. +\end{theorem} + +\begin{proof} +Let $\struct {G, \circ}$ be a [[Definition:Group|group]]. +We have that: +: $\forall a \in G: e \circ a = a \circ e \implies e \in C_G \paren a$ +Thus $C_G \paren a \ne \O$. +Let $x, y \in C_G \paren a$. +Then: +{{begin-eqn}} +{{eqn | l = x \circ a + | r = a \circ x + | c = +}} +{{eqn | l = y \circ a + | r = a \circ y + | c = +}} +{{eqn | ll= \leadsto + | l = x \circ y \circ a + | r = x \circ a \circ y + | c = +}} +{{eqn | r = a \circ x \circ y + | c = +}} +{{eqn | ll= \leadsto + | l = x \circ y + | o = \in + | r = C_G \paren a + | c = +}} +{{end-eqn}} +Thus $C_G \paren a$ is closed under $\circ$. +Let $x \in C_G \paren a$. +Then: +{{begin-eqn}} +{{eqn | l = x \circ a + | r = a \circ x + | c = +}} +{{eqn | ll= \leadsto + | l = x^{-1} \circ x \circ a \circ x^{-1} + | r = x^{-1} \circ a \circ x \circ x^{-1} + | c = +}} +{{eqn | ll= \leadsto + | l = a \circ x^{-1} + | r = x^{-1} \circ a + | c = +}} +{{end-eqn}} +So: +: $x \in C_G \paren a \implies x^{-1} \in C_G \paren a$ +Thus, by the [[Two-Step Subgroup Test]], the result follows. +{{qed}} +\end{proof} + +\begin{proof} +Let $\struct {G, \circ}$ be a [[Definition:Group|group]]. +We have that: +:$\forall a \in G: e \circ a = a \circ e \implies e \in \map {C_G} a$ +Thus $\map {C_G} a \ne \O$. +Let $x, y \in \map {C_G} a$. +Then from [[Commutation with Group Elements implies Commuation with Product with Inverse]]: +:$a \circ x \circ y^{-1} = x \circ y^{-1} \circ a$ +so: +:$x \circ y^{-1} \in\map {C_G} a$ +The result follows by the [[One-Step Subgroup Test]], the result follows. +{{qed}} +\end{proof}<|endoftext|> +\section{Centralizer in Subgroup is Intersection} +Tags: Subgroups, Centralizers, Set Intersection + +\begin{theorem} +Let $G$ be a [[Definition:Group|group]]. +Let $H$ be a [[Definition:Subgroup|subgroup]] of $G$. +Then: +:$\forall x \in G: \map {C_H} x = \map {C_G} x \cap H$ +That is, the [[Definition:Centralizer of Group Element|centralizer]] of an [[Definition:Element|element]] in a [[Definition:Subgroup|subgroup]] is the [[Definition:Set Intersection|intersection]] of that [[Definition:Subgroup|subgroup]] with the [[Definition:Centralizer of Group Element|centralizer]] of the [[Definition:Element|element]] in the [[Definition:Group|group]]. +\end{theorem} + +\begin{proof} +It is clear that: +:$g \in \map {C_H} x \iff g \in \map {C_G} x \land g \in H$ +The result follows by definition of [[Definition:Set Intersection|set intersection]]. +{{qed}} +\end{proof}<|endoftext|> +\section{Kernel of Inner Automorphism Group is Center} +Tags: Inner Automorphisms, Centers of Groups + +\begin{theorem} +Let the [[Definition:Mapping|mapping]] $\kappa: G \to \Inn G$ from a [[Definition:Group|group]] $G$ to its [[Definition:Inner Automorphism Group|inner automorphism group]] $\Inn G$ be defined as: +:$\map \kappa a = \kappa_a$ +where $\kappa_a$ is the [[Definition:Inner Automorphism|inner automorphism]] of $G$ given by $a$. +Then $\kappa$ is a [[Definition:Group Epimorphism|group epimorphism]], and its [[Definition:Kernel of Group Homomorphism|kernel]] is the [[Definition:Center of Group|center]] of $G$: +:$\map \ker \kappa = \map Z G$ +\end{theorem} + +\begin{proof} +Let $\kappa: G \to \Aut G$ be a [[Definition:Mapping|mapping]] defined by $\map \kappa x = \kappa_x$. +It is clear that $\Img \kappa = \Inn G$. +It is also clear that $\kappa$ is a [[Definition:Group Homomorphism|homomorphism]]: +{{begin-eqn}} +{{eqn | l = \map \kappa x \map \kappa y + | r = \kappa_x \circ \kappa_y + | c = +}} +{{eqn | r = \kappa_{x y} + | c = [[Inner Automorphism is Automorphism]] +}} +{{eqn | r = \map \kappa {x y} + | c = +}} +{{end-eqn}} +Note that $\forall \kappa_x \in \Inn G: \exists x \in G: \map \kappa x = \kappa_x$. +Thus $\kappa: G \to \Inn G$ is a [[Definition:Surjection|surjection]] and therefore an [[Definition:Group Epimorphism|group epimorphism]]. +Now we investigate the [[Definition:Kernel of Group Homomorphism|kernel]] of $\kappa$: +{{begin-eqn}} +{{eqn | l = \map \ker \kappa + | r = \set {x \in G: \kappa_x = I_G} + | c = {{Defof|Kernel of Group Homomorphism}} +}} +{{eqn | r = \set {x \in G: \forall g \in G: \map {\kappa_x} g = \map {I_G} g} + | c = [[Equality of Mappings]] +}} +{{eqn | r = \set {x \in G: \forall g \in G: x g x^{-1} = g} + | c = Definition of $\kappa_x$ +}} +{{eqn | r = \set {x \in G: \forall g \in G: x g = g x} + | c = +}} +{{eqn | r = \map Z G + | c = {{Defof|Center of Group}} +}} +{{end-eqn}} +So the [[Definition:Kernel of Group Homomorphism|kernel]] of $\kappa$ is the [[Definition:Center of Group|center]] of $G$. +{{Qed}} +\end{proof}<|endoftext|> +\section{Group equals Center iff Abelian} +Tags: Abelian Groups, Centers of Groups + +\begin{theorem} +Let $G$ be a [[Definition:Group|group]]. +Then $G$ is [[Definition:Abelian Group|abelian]] {{iff}} $\map Z G = G$, that is, {{iff}} $G$ equals its [[Definition:Center of Group|center]]. +\end{theorem} + +\begin{proof} +=== Necessary Condition === +Let $G$ be [[Definition:Abelian Group|abelian]]. +Then: +:$\forall a \in G: \forall x \in G: a x = x a$ +Thus: +:$\forall a \in G: a \in \map Z G = G$ +{{qed|lemma}} +=== Sufficient Condition === +Let $\map Z G = G$. +Then by the definition of [[Definition:Center of Group|center]]: +:$\forall a \in G: \forall x \in G: a x = x a$ +and thus $G$ is [[Definition:Abelian Group|abelian]] by definition. +{{qed}} +\end{proof}<|endoftext|> +\section{Normalizer is Subgroup} +Tags: Normalizers + +\begin{theorem} +Let $G$ be a [[Definition:Group|group]]. +The [[Definition:Normalizer|normalizer]] of a [[Definition:Subset|subset]] $S \subseteq G$ is a [[Definition:Subgroup|subgroup]] of $G$. +:$S \subseteq G \implies \map {N_G} S \le G$ +\end{theorem} + +\begin{proof} +Let $a, b \in \map {N_G} S$. +Then: +{{begin-eqn}} +{{eqn | l = S^{a b} + | r = \paren {S^b}^a + | c = [[Conjugate of Set by Group Product]] +}} +{{eqn | r = S^a + | c = {{Defof|Normal Subgroup}} +}} +{{eqn | r = S + | c = {{Defof|Normal Subgroup}} +}} +{{end-eqn}} +Therefore $a b \in \map {N_G} S$. +Now let $a \in \map {N_G} S$: +:$a \in \map {N_G} S \implies S^{a^{-1} } = \paren {S^a}^{a^{-1} } = S^{a^{-1} a} = S$ +Therefore $a^{-1} \in \map {N_G} S$. +Thus, by the [[Two-Step Subgroup Test]], $\map {N_G} S \le G$. +{{qed}} +\end{proof}<|endoftext|> +\section{Subgroup is Normal Subgroup of Normalizer} +Tags: Normal Subgroups, Normalizers, Subgroups + +\begin{theorem} +Let $G$ be a [[Definition:Group|group]]. +A [[Definition:Subgroup|subgroup]] $H \le G$ is a [[Definition:Normal Subgroup|normal subgroup]] of its [[Definition:Normalizer|normalizer]]: +:$H \le G \implies H \lhd \map {N_G} H$ +\end{theorem} + +\begin{proof} +From [[Subgroup is Subgroup of Normalizer]] we have that $H \le \map {N_G} H$. +It remains to show that $H$ is [[Definition:Normal Subgroup|normal]] in $\map {N_G} H$. +Let $a \in H$ and $b \in \map {N_G} H$. +By the definition of [[Definition:Normalizer|normalizer]]: +: $b a b^{-1} \in H$ +Thus $H$ is [[Definition:Normal Subgroup|normal]] in $\map {N_G} H$. +{{qed}} +\end{proof}<|endoftext|> +\section{Normalizer of Subgroup is Largest Subgroup containing that Subgroup as Normal Subgroup} +Tags: Normalizers + +\begin{theorem} +Let $G$ be a [[Definition:Group|group]]. +Let $H$ be a [[Definition:Subgroup|subgroup]] of $G$. +Then $\map {N_G} H$, the [[Definition:Normalizer|normalizer]] of $H$ in $G$, is the largest [[Definition:Subgroup|subgroup]] of $G$ containing $H$ as a [[Definition:Normal Subgroup|normal subgroup]]. +\end{theorem} + +\begin{proof} +From [[Subgroup is Subgroup of Normalizer]], we have that $H \le \map {N_G} H$. +Now we need to show that $H \lhd \map {N_G} H$. +For $a \in \map {N_G} H$, the [[Definition:Conjugate of Group Subset|conjugate]] of $H$ by $a$ in $\map {N_G} H$ is: +{{begin-eqn}} +{{eqn | l = H^a + | r = \set {x \in \map {N_G} H: a x a^{-1} \in H} + | c = {{Defof|Conjugate of Group Subset}} +}} +{{eqn | r = H^a \cap \map {N_G} H + | c = {{Defof|Set Intersection}} +}} +{{eqn | r = H \cap \map {N_G} H + | c = {{Defof|Normalizer}} +}} +{{eqn | r = H + | c = [[Intersection with Subset is Subset]] +}} +{{end-eqn}} +so: +:$\forall a \in \map {N_G} H: H^a = H$ +and so by [[Definition:Normal Subgroup|definition of normal subgroup]]: +:$H \lhd \map {N_G} H$ +Now we need to show that $\map {N_G} H$ is the largest [[Definition:Subgroup|subgroup]] of $G$ containing $H$ such that $H \lhd \map {N_G} H$. +That is, to show that any [[Definition:Subgroup|subgroup]] of $G$ in which $H$ is [[Definition:Normal Subgroup|normal]] is also a [[Definition:Subset|subset]] of $\map {N_G} H$. +Take any $N$ such that $H \lhd N \le G$. +In $N$, the [[Definition:Conjugate of Group Subset|conjugate]] of $H$ by $a \in N$ is $N \cap H^a = H$. +Therefore: +:$H \subseteq H^a$ +Similarly, $H \subseteq H^{a^{-1} }$, so: +:$H^a \subseteq \paren {H^a}^{a^{-1} } = H$ +Thus: +:$\forall a \in N: H^a = H, a \in \map {N_G} H$ +That is: +:$N \subseteq \map {N_G} H$ +So what we have shown is that any [[Definition:Subgroup|subgroup]] of $G$ in which $H$ is [[Definition:Normal Subgroup|normal]] is a [[Definition:Subset|subset]] of $\map {N_G} H$, which is another way of saying that $\map {N_G} H$ is the largest such [[Definition:Subgroup|subgroup]]. +{{Qed}} +\end{proof}<|endoftext|> +\section{Normal Subgroup iff Normalizer is Group} +Tags: Normal Subgroups, Normalizers + +\begin{theorem} +Let $G$ be a [[Definition:Group|group]]. +Let $H$ be a [[Definition:Subgroup|subgroup]] of $G$. +Then $H$ is [[Definition:Normal Subgroup|normal]] in $G$ {{iff}} the [[Definition:Normalizer|normalizer]] of $H$ is equal to $G$: +:$H \lhd G \iff \map {N_G} H = G$ +\end{theorem} + +\begin{proof} +=== Sufficient Condition === +Let $H$ be [[Definition:Normal Subgroup|normal]] in $G$. +Then $G$ is trivially the largest [[Definition:Subgroup|subgroup]] of $G$ in which $H$ is [[Definition:Normal Subgroup|normal]]. +Thus from [[Normalizer of Subgroup is Largest Subgroup containing that Subgroup as Normal Subgroup]]: +:$\map {N_G} H = G$ +{{qed|lemma}} +=== Necessary Condition === +Let $\map {N_G} H = G$. +From [[Subgroup is Normal Subgroup of Normalizer]], $H$ is [[Definition:Normal Subgroup|normal]] in $\map {N_G} H$. +Hence $H$ is [[Definition:Normal Subgroup|normal]] in $G$. +{{qed}} +\end{proof}<|endoftext|> +\section{Normalizer of Conjugate is Conjugate of Normalizer} +Tags: Normalizers, Conjugacy + +\begin{theorem} +The [[Definition:Normalizer|normalizer]] of a [[Definition:Conjugate of Group Subset|conjugate]] is the [[Definition:Conjugate of Group Subset|conjugate]] of the [[Definition:Normalizer|normalizer]]: +:$S \subseteq G \implies \map {N_G} {S^a} = \paren {\map {N_G} S}^a$ +\end{theorem} + +\begin{proof} +From the definition of [[Definition:Conjugate of Group Subset|conjugate]]: +:$S^a = \set {y \in G: \exists x \in S: y = a x a^{-1} } = a S a^{-1}$ +From the definition of [[Definition:Normalizer|normalizer]]: +:$\map {N_G} S = \set {x \in G: S^x = S}$ +Thus: +{{begin-eqn}} +{{eqn | l = \map {N_G} {S^a} + | r = \set {x \in G: \paren {S^a}^x = S^a} + | c = +}} +{{eqn | r = \set {x \in G: x a S a^{-1} x^{-1} = a S a^{-1} } + | c = +}} +{{end-eqn}} +{{begin-eqn}} +{{eqn | l = \paren {\map {N_G} S}^a + | r = \set {x \in G: S^x = S}^a +}} +{{eqn | r = \set {x \in G: x S x^{-1} = S}^a +}} +{{eqn | r = \set {y \in G: \exists z \in \set {x \in G: x S x^{-1} = S}: y = a z a^{-1} } +}} +{{end-eqn}} +Suppose that $x \in \map {N_G} S$. +It is to be shown that: +: $a x a^{-1} \in \map {N_G} {S^a} = \map {N_G} {a S a^{-1} }$ +To this end, compute: +{{begin-eqn}} +{{eqn | l = \paren {a x a^{-1} } a S a^{-1} \paren {a x a^{-1} }^{-1} + | r = \paren {a x a^{-1} } a S a^{-1} \paren {a x^{-1} a^{-1} } + | c = [[Power of Conjugate equals Conjugate of Power]] +}} +{{eqn | r = a x S x^{-1} a^{-1} +}} +{{eqn | r = a S a^{-1} + | c = as $x \in \map {N_G} S$ +}} +{{end-eqn}} +Hence $a x a^{-1} \in \map {N_G} {S^a}$, and it follows that: +:$z \in \paren {\map {N_G} S}^a \implies z \in \map {N_G} {S^a}$ +Conversely, let $x \in \map {N_G} {S^a}$. +That is, let $x \in G$ such that $x a S a^{-1} x^{-1} = a S a^{-1}$. +Now if we can show that $a^{-1} x a \in \map {N_G} S$, then: +:$x = a \left({a^{-1} x a}\right) a^{-1} \in \paren {\map {N_G} S}^a$ +establishing the remaining inclusion. +Thus, we compute: +{{begin-eqn}} +{{eqn | l = \paren {a^{-1} x a} S \paren {a^{-1} x a}^{-1} + | r = a^{-1} x a S a^{-1} x^{-1} a + | c = [[Power of Conjugate equals Conjugate of Power]] +}} +{{eqn | r = a^{-1} a S a^{-1} a + | c = as $x \in \map {N_G} {S^a}$ +}} +{{eqn | r = S +}} +{{end-eqn}} +Combined with the above observation, this establishes that: +:$z \in \map {N_G} {S^a} \implies z \in \paren {\map {N_G} S}^a$ +Hence the result, by definition of [[Definition:Set Equality/Definition 2|set equality]]. +{{qed}} +\end{proof}<|endoftext|> +\section{Normalizer of Center is Group} +Tags: Normalizers, Centers of Groups + +\begin{theorem} +Let $G$ be a [[Definition:Group|group]]. +Let $\map Z G$ be the [[Definition:Center of Group|center]] of $G$. +Let $x \in G$. +Let $\map {N_G} x$ be the [[Definition:Normalizer|normalizer of $x$ in $G$]]. +Then: +:$\map Z G = \set {x \in G: \map {N_G} x = G}$ +That is, the [[Definition:Center of Group|center]] of a [[Definition:Group|group]] $G$ is the [[Definition:Set|set]] of [[Definition:Element|elements]] $x$ of $G$ such that the [[Definition:Normalizer|normalizer]] of $x$ is the whole of $G$. +Thus: +:$x \in \map Z G \iff \map {N_G} x = G$ +and so: +:$\index G {\map {N_G} x} = 1$ +where $\index G {\map {N_G} x}$ is the [[Definition:Index of Subgroup|index of $\map {N_G} x$ in $G$]]. +\end{theorem} + +\begin{proof} +$\map {N_G} x$ is the [[Definition:Normalizer|normalizer]] of the [[Definition:Set|set]] $\set x$. +Thus: +{{begin-eqn}} +{{eqn | l = \map {N_G} x + | r = G + | c = +}} +{{eqn | ll= \leadstoandfrom + | lo= \forall a \in G: + | l = \set x^a + | r = \set x + | c = {{Defof|Normalizer}} +}} +{{eqn | ll= \leadstoandfrom + | lo= \forall a \in G: + | l = a x a^{-1} + | r = x + | c = {{Defof|Conjugate of Group Element}} +}} +{{eqn | ll= \leadstoandfrom + | lo= \forall a \in G: + | l = a x + | r = x a + | c = +}} +{{eqn | ll= \leadstoandfrom + | l = x + | o = \in + | r = \map Z G + | c = {{Defof|Center of Group}} +}} +{{end-eqn}} +{{qed}} +\end{proof}<|endoftext|> +\section{Quotient Group of Ideal is Coset Space} +Tags: Ideal Theory, Quotient Groups, Cosets + +\begin{theorem} +Let $\struct {R, +, \circ}$ be a [[Definition:Ring (Abstract Algebra)|ring]]. +Let $J$ be an [[Definition:Ideal of Ring|ideal]] of $R$. +Let $\struct {R / J, +}$ be the [[Definition:Quotient Group|quotient group]] of $\struct {R, +}$ by $\struct {J, +}$. +Then each element of $\struct {R / J, +}$ is a [[Definition:Coset|coset]] of $J$ in $R$, that is, is of the form $x + J = \set {x + j: j \in J}$ for some $x \in R$. +The rule of addition of these [[Definition:Coset|cosets]] is: $\paren {x + J} + \paren {y + J} = \paren {x + y} + J$. +The [[Definition:Identity Element|identity]] of $\struct {R / J, +}$ is $J$ and for each $x \in R$, the [[Definition:Inverse Element|inverse]] of $x + J$ is $\paren {-x} + J$. +\end{theorem} + +\begin{proof} +From [[Ideal is Additive Normal Subgroup]] that $J$ is a [[Definition:Normal Subgroup|normal subgroup]] of $R$ under $+$. +From [[Quotient Ring Addition is Well-Defined]], $+$ is a [[Definition:Well-Defined Operation|well-defined operation]]. +The rest follows directly from the definition of [[Definition:Quotient Group|quotient group]]. +{{qed}} +\end{proof}<|endoftext|> +\section{Quotient Ring is Ring/Quotient Ring Product is Well-Defined} +Tags: Quotient Rings + +\begin{theorem} +Let $\struct {R, +, \circ}$ be a [[Definition:Ring (Abstract Algebra)|ring]] whose [[Definition:Ring Zero|zero]] is $0_R$ and whose [[Definition:Unity of Ring|unity]] is $1_R$. +Let $J$ be an [[Definition:Ideal of Ring|ideal]] of $R$. +Let $\struct {R / J, +, \circ}$ be the [[Definition:Quotient Ring|quotient ring]] of $R$ by $J$. +Then $\circ$ is [[Definition:Well-Defined Operation|well-defined]] on $R / J$, that is: +:$x_1 + J = x_2 + J, y_1 + J = y_2 + J \implies x_1 \circ y_1 + J = x_2 \circ y_2 + J$ +\end{theorem} + +\begin{proof} +From [[Left Cosets are Equal iff Product with Inverse in Subgroup]], we have: +{{begin-eqn}} +{{eqn | l = x_1 + J + | r = x_2 + J + | c = +}} +{{eqn | ll= \leadsto + | l = x_1 + \paren {-x_2} + | o = \in + | r = J + | c = +}} +{{end-eqn}} +and: +{{begin-eqn}} +{{eqn | l = y_1 + J + | r = y_2 + J + | c = +}} +{{eqn | ll= \leadsto + | l = y_1 + \paren {-y_2} + | o = \in + | r = J + | c = +}} +{{end-eqn}} +Hence from the [[Definition:Ideal of Ring|definition of ideal]]: +{{begin-eqn}} +{{eqn | l = \paren {x_1 + \paren {-x_2} } \circ y_1 + | o = \in + | r = J +}} +{{eqn | l = x_2 \circ \paren {y_1 + \paren {-y_2} } + | o = \in + | r = J +}} +{{end-eqn}} +Thus: +{{begin-eqn}} +{{eqn | l = \paren {x_1 + \paren {-x_2} } \circ y_1 + x_2 \circ \paren {y_1 + \paren {-y_2} } + | o = \in + | r = J + | c = [[Definition:Group|as $\struct {J, +}$ is a group]] +}} +{{eqn | ll= \leadsto + | l = x_1 \circ y_1 + \paren {-\paren {x_2 \circ y_2} } + | o = \in + | r = J + | c = [[Definition:Ring (Abstract Algebra)|Various ring properties]] +}} +{{eqn | ll= \leadsto + | l = x_1 \circ y_1 + J + | r = x_2 \circ y_2 + J + | c = [[Left Cosets are Equal iff Product with Inverse in Subgroup]] +}} +{{end-eqn}} +{{qed|lemma}} +\end{proof}<|endoftext|> +\section{Quotient Ring of Commutative Ring is Commutative} +Tags: Quotient Rings + +\begin{theorem} +Let $\struct {R, +, \circ}$ be a [[Definition:Ring (Abstract Algebra)|ring]] whose [[Definition:Ring Zero|zero]] is $0_R$ and whose [[Definition:Unity of Ring|unity]] is $1_R$. +Let $J$ be an [[Definition:Ideal of Ring|ideal]] of $R$. +Let $\struct {R / J, +, \circ}$ be the [[Definition:Quotient Ring|quotient ring]] defined by $J$. +If $\struct {R, +, \circ}$ is a [[Definition:Commutative Ring|commutative ring]], then so is $\struct {R / J, +, \circ}$. +\end{theorem} + +\begin{proof} +Let $\struct {R, +, \circ}$ be a [[Definition:Commutative Ring|commutative ring]] +That means $\circ$ is [[Definition:Commutative Operation|commutative]] on $R$. +Thus: +{{begin-eqn}} +{{eqn | ll= \forall x, y \in R: + | l = \paren {x + J} \circ \paren {y + J} + | r = x \circ y + J + | c = [[Definition:Quotient Ring|Definition of $\circ$]] in $R / J$ +}} +{{eqn | r = y \circ x + J + | c = [[Definition:Commutative Operation|Commutativity]] of $\circ$ +}} +{{eqn | r = \paren {y + J} \circ \paren {x + J} + | c = [[Definition:Quotient Ring|Definition of $\circ$]] in $R / J$ +}} +{{end-eqn}} +{{Qed}} +\end{proof}<|endoftext|> +\section{Quotient Ring of Ring with Unity is Ring with Unity} +Tags: Quotient Rings, Rings with Unity + +\begin{theorem} +Let $\struct {R, +, \circ}$ be a [[Definition:Ring with Unity|ring with unity]] whose [[Definition:Ring Zero|zero]] is $0_R$ and whose [[Definition:Unity of Ring|unity]] is $1_R$. +Let $J$ be an [[Definition:Ideal of Ring|ideal]] of $R$. +Let $\struct {R / J, +, \circ}$ be the [[Definition:Quotient Ring|quotient ring]] defined by $J$. +Then $\struct {R / J, +, \circ}$ is a [[Definition:Ring with Unity|ring with unity]], and its [[Definition:Unity of Ring|unity]] is $1_R + J$. +\end{theorem} + +\begin{proof} +Let $\struct {R, +, \circ}$ be a [[Definition:Ring with Unity|ring with unity]]. +First, let $J \subsetneq R$. +By [[Ideal of Unit is Whole Ring/Corollary|Ideal of Unit is Whole Ring: Corollary]]: +:$1_R \in J \implies J = R$ +So $1_R \notin J$. +Thus $1_R + J \ne J$, so $1_R + J \ne 0_{R/J}$. +Now let $x \in R$. +{{begin-eqn}} +{{eqn | l = \paren {1_R + J} \circ \paren {x + J} + | r = 1_R \circ x + J + | c = [[Definition:Quotient Ring|Definition of $\circ$]] in $R / J$ +}} +{{eqn | r = x + J + | c = {{Defof|Unity of Ring}} +}} +{{eqn | r = x \circ 1_R + J + | c = {{Defof|Unity of Ring}} +}} +{{eqn | r = \paren {x + J} \circ \paren {1_R + J} + | c = [[Definition:Quotient Ring|Definition of $\circ$]] in $R / J$ +}} +{{end-eqn}} +Thus $R / J$ has a [[Definition:Unity of Ring|unity]], and that [[Definition:Unity of Ring|unity]] is $1_R + J$. +Now suppose $J = R$. +Then $1_R + J = J$ and therefore $1_R = 0_R$. +The only ring to have $1_R = 0_R$ is the [[Definition:Null Ring|null ring]]. +This is appropriate, because: +:$R / J = R / R = \set {0_{R / R} }$ +which is the [[Definition:Null Ring|null ring]]. +{{Qed}} +\end{proof}<|endoftext|> +\section{Ring Epimorphism with Trivial Kernel is Isomorphism} +Tags: Ring Epimorphisms, Ring Isomorphisms + +\begin{theorem} +Let $\phi: \left({R_1, +_1, \circ_1}\right) \to \left({R_2, +_2, \circ_2}\right)$ be a [[Definition:Ring Epimorphism|ring epimorphism]]. +Then $\phi$ is a [[Definition:Ring Isomorphism|ring isomorphism]] iff $\ker \left({\phi}\right) = \left\{ {0_{R_1} }\right\}$. +\end{theorem} + +\begin{proof} +Follows directly from [[Kernel is Trivial iff Monomorphism]] and the definition of a [[Definition:Ring Epimorphism|ring epimorphism]]. +{{qed}} +[[Category:Ring Epimorphisms]] +[[Category:Ring Isomorphisms]] +7eicjqz42mvwfnab4mh5x2fqlui7ox6 +\end{proof}<|endoftext|> +\section{Quotient Epimorphism is Epimorphism/Ring} +Tags: Quotient Rings + +\begin{theorem} +Let $\struct {R, +, \circ}$ be a [[Definition:Ring (Abstract Algebra)|ring]] whose [[Definition:Ring Zero|zero]] is $0_R$ and whose [[Definition:Unity of Ring|unity]] is $1_R$. +Let $J$ be an [[Definition:Ideal of Ring|ideal]] of $R$. +Let $\struct {R / J, +, \circ}$ be the [[Definition:Quotient Ring|quotient ring]] defined by $J$. +Let $\phi: R \to R / J$ be the [[Definition:Quotient Ring Epimorphism|quotient (ring) epimorphism]] from $R$ to $R / J$: +:$x \in R: \map \phi x = x + J$ +Then $\phi$ is a [[Definition:Ring Epimorphism|ring epimorphism]] whose [[Definition:Kernel of Ring Homomorphism|kernel]] is $J$. +\end{theorem} + +\begin{proof} +Let $x, y \in R$. +Then: +{{begin-eqn}} +{{eqn | l = \map \phi {x + y} + | r = \paren {x + y} + J + | c = Definition of $\phi$ +}} +{{eqn | r = \paren {x + J} + \paren {y + J} + | c = {{Defof|Quotient Ring Addition}} +}} +{{eqn | r = \map \phi x + \map \phi y + | c = +}} +{{end-eqn}} +and: +{{begin-eqn}} +{{eqn | l = \map \phi {x \circ y} + | r = \paren {x \circ y} + J + | c = Definition of $\phi$ +}} +{{eqn | r = \paren {x + J} \circ \paren {y + J} + | c = {{Defof|Quotient Ring Product}} +}} +{{eqn | r = \map \phi x \, \map \phi y + | c = +}} +{{end-eqn}} +Thus $\phi$ is a [[Definition:Ring Homomorphism|homomorphism]]. +$\phi$ is [[Definition:Surjection|surjective]] because: +:$\forall x + J \in R / J: x + J = \map \phi x$ +Therefore $\phi$ is an [[Definition:Ring Epimorphism|epimorphism]]. +Let $x \in \map \ker \phi$. +Then: +{{begin-eqn}} +{{eqn | l = x + | o = \in + | r = \map \ker \phi + | c = +}} +{{eqn | ll= \leadstoandfrom + | l = \map \phi x + | r = 0_{R/J} + | c = {{Defof|Kernel of Ring Homomorphism}} +}} +{{eqn | ll= \leadstoandfrom + | l = x + J + | r = J + | c = [[Definition:Quotient Ring|$J$ is the zero of $\struct {R / J, +, \circ}$]] +}} +{{eqn | ll= \leadstoandfrom + | l = x + | o = \in + | r = J + | c = [[Left Coset Equals Subgroup iff Element in Subgroup]] +}} +{{end-eqn}} +Thus: +:$\map \ker \phi = J$ +{{Qed}} +\end{proof}<|endoftext|> +\section{Ring Homomorphism Preserves Subrings} +Tags: Ring Homomorphisms, Subrings, Ring Homomorphism Preserves Subrings + +\begin{theorem} +Let $\phi: \struct {R_1, +_1, \circ_1} \to \struct {R_2, +_2, \circ_2}$ be a [[Definition:Ring Homomorphism|ring homomorphism]]. +If $S$ is a [[Definition:Subring|subring]] of $R_1$, then $\phi \sqbrk S$ is a [[Definition:Subring|subring]] of $R_2$. +\end{theorem} + +\begin{proof} +Since $S \ne \O$, $\phi \sqbrk S \ne \O$. +From [[Group Homomorphism Preserves Subgroups]], $\struct {\phi \sqbrk S, +_2}$ is a [[Definition:Subgroup|subgroup]] of $\struct {R_2, +_2}$. +From [[Homomorphism Preserves Subsemigroups]], $\struct {\phi \sqbrk S, \circ_2}$ is a [[Definition:Subsemigroup|subsemigroup]] of $\struct {R_2, \circ_2}$. +Thus, as $\struct {R_2, +_2}$ is a [[Definition:Group|group]] and $\struct {R_2, \circ_2}$ is a [[Definition:Semigroup|semigroup]], the result follows. +{{qed}} +\end{proof} + +\begin{proof} +From [[Morphism Property Preserves Closure]], $\phi \sqbrk {R_1}$ is a [[Definition:Closed Algebraic Structure|closed algebraic structure]]. +From [[Epimorphism Preserves Rings]], $\phi \sqbrk S$ is a [[Definition:Ring (Abstract Algebra)|ring]]. +Hence the result, from the definition of [[Definition:Subring|subring]]. +{{qed}} +\end{proof} + +\begin{proof} +Let $S$ be a [[Definition:Subring|subring]] of $R_1$. +Since $S \ne \O$ it follows that $\phi \sqbrk S \ne \O$. +Let $x, y \in \phi \sqbrk S$. +Then: +:$\exists s, t \in S: x = \map \phi s, y = \map \phi t$ +So: +{{begin-eqn}} +{{eqn | l = x +_2 \paren {-y} + | r = \map \phi s +_2 \paren {-\map \phi t} + | c = +}} +{{eqn | r = \map \phi {s +_1 \paren {-t} } + | c = as $\phi$ is a [[Definition:Ring Homomorphism|homomorphism]] +}} +{{end-eqn}} +As $S$ is a [[Definition:Subring|subring]] of $R_1$, it is [[Definition:Closed Algebraic Structure|closed]] under $+_1$ and the taking of [[Definition:Ring Negative|negatives]]. +Thus $s +_1 \paren {-t} \in S$ and so $x +_2 \paren {-y} \in \phi \sqbrk S$. +Similarly: +{{begin-eqn}} +{{eqn | l = x \circ_2 y + | r = \map \phi s \circ_2 \map \phi t + | c = +}} +{{eqn | r = \map \phi {s \circ_1 t} + | c = as $\phi$ is a [[Definition:Ring Homomorphism|homomorphism]] +}} +{{end-eqn}} +Because $S$ is a [[Definition:Subring|subring]] of $R_1$, it is [[Definition:Closed Algebraic Structure|closed]] under $\circ_1$. +Thus $s \circ_1 t \in S$ and so $x \circ_2 y \in \phi \sqbrk S$. +The result follows from [[Subring Test]]. +{{qed}} +\end{proof}<|endoftext|> +\section{Kernel of Ring Homomorphism is Subring} +Tags: Ring Homomorphisms, Subrings + +\begin{theorem} +Let $\phi: \struct {R_1, +_1, \circ_1} \to \struct {R_2, +_2, \circ_2}$ be a [[Definition:Ring Homomorphism|ring homomorphism]]. +Then the [[Definition:Kernel of Ring Homomorphism|kernel]] of $\phi$ is a [[Definition:Subring|subring]] of $R_1$. +\end{theorem} + +\begin{proof} +From [[Ring Homomorphism of Addition is Group Homomorphism]] and [[Kernel of Group Homomorphism is Subgroup]]: +:$\struct {\map \ker \phi, +_1} \le \struct {R_1, +_1}$ +where $\le$ denotes [[Definition:Subgroup|subgroup]]. +Let $x, y \in \map \ker \phi$. +{{begin-eqn}} +{{eqn | l = \map \phi {x \circ_1 y} + | r = \map \phi x \circ_2 \map \phi y + | c = [[Definition:Morphism Property|Morphism Property]] +}} +{{eqn | r = 0_{R_2} \circ_2 0_{R_2} + | c = [[Definition:Kernel of Ring Homomorphism|Definition of Kernel]] +}} +{{eqn | r = 0_{R_2} + | c = +}} +{{end-eqn}} +Thus $x \circ_1 y \in \map \ker \phi$. +Thus the conditions for [[Subring Test]] are fulfilled, and $\map \ker \phi$ is a [[Definition:Subring|subring]] of $R_1$. +{{qed}} +\end{proof}<|endoftext|> +\section{Kernel of Ring Homomorphism is Ideal} +Tags: Ring Homomorphisms, Ideal Theory + +\begin{theorem} +Let $\phi: \struct {R_1, +_1, \circ_1} \to \struct {R_2, +_2, \circ_2}$ be a [[Definition:Ring Homomorphism|ring homomorphism]]. +Then the [[Definition:Kernel of Ring Homomorphism|kernel]] of $\phi$ is an [[Definition:Ideal of Ring|ideal]] of $R_1$. +\end{theorem} + +\begin{proof} +By [[Kernel of Ring Homomorphism is Subring]], $\map \ker \phi$ is a [[Definition:Subring|subring]] of $R_1$. +Let $s \in \map \ker \phi$, so $\map \phi s = 0_{R_2}$. +Suppose $x \in R_1$. Then: +{{begin-eqn}} +{{eqn | l = \map \phi {x \circ_1 s} + | r = \map \phi x \circ_2 \map \phi s + | c = {{Defof|Morphism Property}} +}} +{{eqn | r = \map \phi x \circ_2 0_{R_2} + | c = as $s \in \map \ker \phi$ +}} +{{eqn | r = 0_{R_2} + | c = Properties of $0_{R_2}$ +}} +{{end-eqn}} +and similarly for $\map \phi {s \circ_1 x}$. +The result follows. +{{Qed}} +\end{proof}<|endoftext|> +\section{Ideals of Division Ring} +Tags: Ideal Theory + +\begin{theorem} +Let $\struct {R, +, \circ}$ be a [[Definition:Division Ring|division ring]] whose [[Definition:Ring Zero|zero]] is $0_R$. +The only [[Definition:Ideal of Ring|ideals]] of $\struct {R, +, \circ}$ are $\set {0_R}$ and $R$ itself. +That is, $\struct {R, +, \circ}$ has no non-[[Definition:Null Ideal|null]] [[Definition:Proper Ideal|proper ideals]]. +\end{theorem} + +\begin{proof} +From [[Null Ring is Ideal]], $\set {0_R}$ is an [[Definition:Ideal of Ring|ideal]] of $\struct {R, +, \circ}$, as $\struct {R, +, \circ}$, being a [[Definition:Division Ring|division ring]], is also a [[Definition:Ring (Abstract Algebra)|ring]]. +By definition, every non-[[Definition:Ring Zero|zero]] [[Definition:Element|element]] of a [[Definition:Division Ring|division ring]] is a [[Definition:Unit of Ring|unit]]. +So $S \ne \set {0_R} \implies \exists x \in S: x \ne 0_R$ such that $r$ is a [[Definition:Unit of Ring|unit]] of $R$. +The result follows from [[Ideal of Unit is Whole Ring]]. +{{qed}} +\end{proof}<|endoftext|> +\section{Quotient Ring of Kernel of Ring Epimorphism} +Tags: Ring Epimorphisms, Ring Isomorphisms, Quotient Rings + +\begin{theorem} +Let $\phi: \struct {R_1, +_1, \circ_1} \to \struct {R_2, +_2, \circ_2}$ be a [[Definition:Ring Epimorphism|ring epimorphism]]. +Let $K = \map \ker \phi$. +Then there is a [[Definition:Unique|unique]] [[Definition:Ring Isomorphism|ring isomorphism]] $g: R_1 / K \to R_2$ such that: +:$g \circ q_K = \phi$ +$\phi$ is an [[Definition:Ring Isomorphism|isomorphism]] {{iff}} $K = \set {0_{R_1} }$. +\end{theorem} + +\begin{proof} +From the [[Quotient Theorem for Epimorphisms]], there is [[Definition:Unique|one and only one]] [[Definition:Isomorphism (Abstract Algebra)|isomorphism]] that satisfies the conditions for each of the operations on $R_1$. +So the first statement follows directly. +From [[Kernel is Trivial iff Monomorphism]], $\phi$ is a [[Definition:Ring Monomorphism|ring monomorphism]] {{iff}} $K = \set {0_{R_1} }$. +As $\phi$ is also an [[Definition:Ring Epimorphism|epimorphism]], the result follows. +\end{proof}<|endoftext|> +\section{Ring Epimorphism Preserves Ideals} +Tags: Ideal Theory, Ring Epimorphisms + +\begin{theorem} +Let $\phi: \struct {R_1, +_1, \circ_1} \to \struct {R_2, +_2, \circ_2}$ be a [[Definition:Ring Epimorphism|ring epimorphism]]. +Let $J$ be an [[Definition:Ideal of Ring|ideal]] of $R_1$. +Then $\phi \sqbrk J$ is an [[Definition:Ideal of Ring|ideal]] of $R_2$. +\end{theorem} + +\begin{proof} +$J$ is an [[Definition:Ideal of Ring|ideal]] of $R_1$, so it is also a [[Definition:Subring|subring]] of $R_1$. +From [[Ring Homomorphism Preserves Subrings]], it follows that $\phi \sqbrk J$ is a [[Definition:Subring|subring]] of $R_2$. +Now suppose $u \in \phi \sqbrk J$. +Let $v \in R_2$. +Then $\exists x \in J, y \in R_1$ such that $\map \phi x = u, \map \phi y = v$. +Thus, by the [[Definition:Morphism Property|morphism property]]: +:$u \circ_2 v = \map \phi x \circ_2 \map \phi y = \map \phi {x \circ_1 y}$ +So $u \circ_2 v \in \phi \sqbrk J$ because $x \circ_1 y \in J$. +Similarly $v \circ_2 u \in \phi \sqbrk J$ also. +The result follows. +{{qed}} +\end{proof}<|endoftext|> +\section{Preimage of Image of Subring under Ring Homomorphism} +Tags: Ring Homomorphisms, Subrings + +\begin{theorem} +Let $\phi: \left({R_1, +_1, \circ_1}\right) \to \left({R_2, +_2, \circ_2}\right)$ be a [[Definition:Ring Homomorphism|ring homomorphism]]. +Let $K = \ker \left({\phi}\right)$, where $\ker \left({\phi}\right)$ is the [[Definition:Kernel of Ring Homomorphism|kernel]] of $\phi$. +Let $J$ be a [[Definition:Subring|subring]] of $R_1$. +Then: +:$\phi^{-1} \left[{\phi \left[{J}\right]}\right] = J + K$ +\end{theorem} + +\begin{proof} +Let $x \in \phi^{-1} \left[{\phi \left[{J}\right]}\right]$. +Then: +{{begin-eqn}} +{{eqn | l=x + | o=\in + | r=\phi^{-1} \left[{\phi \left[{J}\right]}\right] + | c= +}} +{{eqn | ll=\implies + | l=\phi \left({x}\right) + | o=\in + | r=\phi \left[{J}\right] + | c=Definition of [[Definition:Preimage of Element under Mapping|preimage]] +}} +{{eqn | ll=\implies + | l=\exists b \in J: \phi \left({x}\right) + | r=\phi \left({b}\right) + | c=Definition of [[Definition:Image of Element under Mapping|image]] +}} +{{eqn | ll=\implies + | l=\phi \left({x}\right) + \left({- \phi \left({b}\right)}\right) + | r=0_{R_2} + | c=Definition of [[Definition:Ring Negative|ring negative]] on $R_2$ +}} +{{eqn | ll=\implies + | l=\phi \left({x + \left({-b}\right)}\right) + | r=0_{R_2} + | c=as $\phi$ is a [[Definition:Ring Homomorphism|homomorphism]] +}} +{{eqn | ll=\implies + | l=x + \left({-b}\right) + | o=\in + | r=K + | c=Definition of [[Definition:Kernel of Ring Homomorphism|kernel]] +}} +{{eqn | ll=\implies + | l=x + | o=\in + | r=J + K + | c=as $x = b + \left({x + \left({-b}\right)}\right)$ +}} +{{end-eqn}} +So we have shown that: +:$\phi^{-1} \left[{\phi \left[{J}\right]}\right] \subseteq J + K$ +Now suppose that $x \in J + K$. +Then: +{{begin-eqn}} +{{eqn | l=x + | o=\in + | r=J + K + | c= +}} +{{eqn | ll=\implies + | l=\exists b \in J, a \in K: x + | r=b + a + | c= +}} +{{eqn | ll=\implies + | l=\phi \left({x}\right) + | r=\phi \left({b}\right) + \phi \left({a}\right) + | c= +}} +{{eqn | r=\phi \left({b}\right) + 0_{R_2} + | c=as $a \in K$ +}} +{{eqn | r=\phi \left({b}\right) + | c= +}} +{{eqn | ll=\implies + | l=\phi \left({x}\right) + | o=\in + | r=\phi \left[{J}\right] + | c=as $b \in J$ +}} +{{eqn | ll=\implies + | l=x + | o=\in + | r=\phi^{-1} \left[{\phi \left[{J}\right]}\right] + | c=Definition of [[Definition:Preimage of Subset under Mapping|preimage]] +}} +{{end-eqn}} +So we have shown that: +:$J + K \subseteq \phi^{-1} \left[{\phi \left[{J}\right]}\right]$ +Hence the result by definition of [[Definition:Set Equality/Definition 2|set equality]]. +{{Qed}} +\end{proof}<|endoftext|> +\section{Preimage of Subring under Ring Homomorphism is Subring} +Tags: Ring Homomorphisms, Subrings + +\begin{theorem} +Let $\phi: \left({R_1, +_1, \circ_1}\right) \to \left({R_2, +_2, \circ_2}\right)$ be a [[Definition:Ring Homomorphism|ring homomorphism]]. +Let $S_2$ be a [[Definition:Subring|subring]] of $R_2$. +Then $S_1 = \phi^{-1} \left[{S_2}\right]$ is a [[Definition:Subring|subring]] of $R_1$ such that $\ker \left({\phi}\right) \subseteq S_1$. +\end{theorem} + +\begin{proof} +Let $K = \ker \left({\phi}\right)$ be the [[Definition:Kernel of Ring Homomorphism|kernel]] of $R_1$. +We have that $0_{R_2} \in S_2$ and so $\left\{{0_{R_2}}\right\} \subseteq S_2$. +From [[Subset Maps to Subset]]: +:$\phi^{-1} \left[{\left\{{0_{R_2}}\right\}}\right] \subseteq \phi^{-1} \left[{S_2}\right] = S_1$ +But by definition, $K = \phi^{-1} \left[{\left\{{0_{R_2}}\right\}}\right]$ +and so $S_1$ is a [[Definition:Subset|subset]] of $R_1$ containing $K$, that is: +:$K \subseteq S_1 \subseteq R_1$ +Now we need to show that $S_1$ is a [[Definition:Subring|subring]] of $R_1$. +Let $r, r' \in S_1$. +Then $\phi \left({r}\right), \phi \left({r'}\right) \in S_2$. +Hence: +{{begin-eqn}} +{{eqn | l=\phi \left({r +_1 r'}\right) + | r=\phi \left({r}\right) +_2 \phi \left({r'}\right) + | c=as $\phi$ is a [[Definition:Ring Homomorphism|ring homomorphism]] +}} +{{eqn | o=\in + | r=S_2 + | c=because $S_2$ is a [[Definition:Subring|subring]] +}} +{{end-eqn}} +So: +:$r + r' \in S_1$ +Then: +{{begin-eqn}} +{{eqn | l=\phi \left({-r}\right) + | r=-\phi \left({r}\right) + | c=[[Group Homomorphism Preserves Inverses]] +}} +{{eqn | o=\in + | r=S_2 + | c=because $S_2$ is a [[Definition:Subring|subring]] +}} +{{end-eqn}} +So: +:$-r \in \phi^{-1} S_1$ +Finally: +{{begin-eqn}} +{{eqn | l=\phi \left({r \circ_1 r'}\right) + | r=\phi \left({r}\right) \circ_2 \phi \left({r'}\right) + | c=as $\phi$ is a [[Definition:Ring Homomorphism|ring homomorphism]] +}} +{{eqn | o=\in + | r=S_2 + | c=because $S_2$ is a [[Definition:Subring|subring]] +}} +{{end-eqn}} +So: +:$r \circ_1 r' \in S_1$ +So from the [[Subring Test]] we have that $\phi^{-1} \left[{S_2}\right]$ is a [[Definition:Subring|subring]] of $R$ containing $K$. +{{qed}} +\end{proof}<|endoftext|> +\section{Preimage of Ideal under Ring Homomorphism is Ideal} +Tags: Ring Homomorphisms, Ideal Theory + +\begin{theorem} +Let $\phi: \left({R_1, +_1, \circ_1}\right) \to \left({R_2, +_2, \circ_2}\right)$ be a [[Definition:Ring Homomorphism|ring homomorphism]]. +Let $S_2$ be an [[Definition:Ideal of Ring|ideal]] of $R_2$. +Then $S_1 = \phi^{-1} \left[{S_2}\right]$ is an [[Definition:Ideal of Ring|ideal]] of $R_1$ such that $\ker \left({\phi}\right) \subseteq S_1$. +\end{theorem} + +\begin{proof} +From [[Preimage of Subring under Ring Homomorphism is Subring]] we have that $S_1 = \phi^{-1} \left[{S_2}\right]$ is a [[Definition:subring|subring]] of $R_1$ such that $\ker \left({\phi}\right) \subseteq S_1$. +We now need to show that $S_1$ is an [[Definition:Ideal of Ring|ideal]] of $R_1$. +Let $s_1 \in S_1, r_1 \in R_1$. +Then: +{{begin-eqn}} +{{eqn | l = \phi \left({r_1 \circ_1 s_1}\right) + | r = \phi \left({r_1}\right) \circ_2 \phi \left({s_1}\right) + | c = as $\phi$ is a [[Definition:Ring Homomorphism|homomorphism]] +}} +{{eqn | r = S_2 + | c = as $S_2$ is an [[Definition:Ideal of Ring|ideal]] of $R_2$ +}} +{{end-eqn}} +Thus: +:$r_1 \circ_1 s_1 \in \phi^{-1} \left[{S_2}\right]= S_1$ +Similarly for $s_1 \circ_1 r_1$. +So $S_1$ is an [[Definition:Ideal of Ring|ideal]] of $R_1$. +{{qed}} +\end{proof}<|endoftext|> +\section{Image of Preimage of Subring under Ring Epimorphism} +Tags: Ring Epimorphisms, Subrings + +\begin{theorem} +Let $\phi: \struct {R_1, +_1, \circ_1} \to \struct {R_2, +_2, \circ_2}$ be a [[Definition:Ring Epimorphism|ring epimorphism]]. +Let $S_2$ be a [[Definition:Subring|subring]] of $R_2$. +Then: +:$\phi \sqbrk {\phi^{-1} \sqbrk {S_2} } = S_2$ +\end{theorem} + +\begin{proof} +As $\phi$ is an [[Definition:Ring Epimorphism|epimorphism]], it is a [[Definition:Surjection|surjection]], and so $\Img \phi = R_2$. +So $S_2 \subseteq \Img {R_1}$. +The result then follows from [[Image of Preimage under Mapping]]. +{{qed}} +\end{proof}<|endoftext|> +\section{Diagonal Relation is Universally Congruent} +Tags: Congruence Relations + +\begin{theorem} +The [[Definition:Diagonal Relation|diagonal relation]] $\Delta_S$ on a [[Definition:Set|set]] $S$ is [[Definition:Universally Congruent|universally congruent]] with every [[Definition:Binary Operation|operation]] on $S$. +\end{theorem} + +\begin{proof} +We have that the [[Diagonal Relation is Equivalence|diagonal relation is an equivalence relation]]. +Let $\left({S, \circ}\right)$ be any [[Definition:Algebraic Structure|algebraic structure]]. +{{begin-eqn}} +{{eqn | o= + | r=x_1 \Delta_S x_2 \land y_1 \Delta_S y_2 + | c= +}} +{{eqn | o=\implies + | r=x_1 = x_2 \land y_1 = y_2 + | c=Definition of [[Definition:Diagonal Relation|Diagonal Relation]] +}} +{{eqn | o=\implies + | r=x_1 \circ y_1 = x_2 \circ y_2 + | c=(consequence of equality) +}} +{{eqn | o=\implies + | r=\left({x_1 \circ y_1}\right) \Delta_S \left({x_2 \circ y_2}\right) + | c=Definition of [[Definition:Diagonal Relation|Diagonal Relation]] +}} +{{end-eqn}} +{{qed}} +It can therefore be described as [[Definition:Universally Congruent|universally congruent]]. +{{Qed}} +\end{proof}<|endoftext|> +\section{Properties of Ordered Ring} +Tags: Ordered Rings + +\begin{theorem} +Let $\struct {R, +, \circ, \le}$ be an [[Definition:Ordered Ring|ordered ring]] whose [[Definition:Ring Zero|zero]] is $0_R$ and whose [[Definition:Unity of Ring|unity]] is $1_R$. +Let $U_R$ be the [[Definition:Group of Units of Ring|group of units]] of $R$. +Let $x, y, z \in \struct {R, +, \circ, \le}$. +Then the following properties hold: +: $(1): \quad x < y \iff x + z < y + z$. Hence $x \le y \iff x + z \le y + z$ (because $\struct {R, +, \le}$ is an [[Definition:Ordered_Group|ordered group]]). +: $(2): \quad x < y \iff 0 < y + \paren {-x}$. Hence $x \le y \iff 0 \le y + \paren {-x}$ +: $(3): \quad 0 < x \iff \paren {-x} < 0$. Hence $0 \le x \iff \paren {-x} \le 0$ +: $(4): \quad x < 0 \iff 0 < \paren {-x}$. Hence $x \le 0 \iff 0 \le \paren {-x}$ +: $(5): \quad \forall n \in \Z_{>0}: x > 0 \implies n \cdot x > 0$ +: $(6): \quad x \le y, 0 \le z: x \circ z \le y \circ z, z \circ x \le z \circ y$ +: $(7): \quad x \le y, z \le 0: y \circ z \le x \circ z, z \circ y \le z \circ x$ +\end{theorem}<|endoftext|> +\section{Positive Elements of Ordered Ring} +Tags: Ordered Rings + +\begin{theorem} +Let $\struct {R, +, \circ, \le}$ be an [[Definition:Ordered Ring|ordered ring]] [[Definition:Ring with Unity|with unity]] whose [[Definition:Ring Zero|zero]] is $0_R$ and whose [[Definition:Unity of Ring|unity]] is $1_R$. +Let $P$ be the set of [[Definition:Positive|positive]] elements of $R$ , that is, $P = R_{\ge 0}$. +Then: +: $(1): \quad P + P \subseteq P$ +: $(2): \quad P \cap \paren {-P} = \set {0_R}$ +: $(3): \quad P \circ P \subseteq P$ +If $\le$ is a [[Definition:Total Ordering|total ordering]], that is, if $\struct {R, +, \circ, \le}$ is a [[Definition:Totally Ordered Ring|totally ordered ring]], then: +: $(4): \quad P \cup \paren {-P} = R$ +The converse is also true: +Let $\struct {R, +, \circ}$ be a [[Definition:Ring (Abstract Algebra)|ring]]. +Let $P \subseteq R$ such that $(1)$, $(2)$ and $(3)$ are satisfied. +Then there is one and only one [[Definition:Ordering|ordering]] $\le$ [[Definition:Ordering Compatible with Ring Structure|compatible with the ring structure]] of $R$ such that $P = R_{\ge 0}$. +Also, if $(4)$ is also satisfied, then $\le$ is a [[Definition:Total Ordering|total ordering]]. +\end{theorem} + +\begin{proof} +=== Necessary Condition === +First, suppose that $\le$ is [[Definition:Ordering Compatible with Ring Structure|compatible with the ring structure]] of $R$. +Hence: +:$(OR1): \quad \le$ is [[Definition:Relation Compatible with Operation|compatible]] with $+$ +:$(OR2): \quad \forall x, y \in R: 0_R \le x, 0_R \le y \implies 0_R \le x \circ y$. +$(1)$: Let $x, y \in R: 0_R \le x, 0_R \le y$. +Then $0_R + 0_R \le x + y$ by the fact that $\le$ is [[Definition:Relation Compatible with Operation|compatible]] with $+$. +Thus $0_R \le x + y$ and thus $x + y \in P$. +$(2)$: By [[Properties of Ordered Ring]] item 4, $-P = \set {x \in R: x \le 0_R}$. +Let $x \in P \cap \paren {-P}$. +Then $x \le 0_R$ and $0_R \le x$. +So from the [[Definition:Ordering|antisymmetric nature]] of $\le$, $x = 0_R$. +$(3)$: This is equivalent to $0_R \le x, 0_R \le y \implies 0_R \le x \circ y$ which is one of the properties of being [[Definition:Ordering Compatible with Ring Structure|compatible with the ring structure]] of $R$. +$(4)$: Now if $\le$ is a [[Definition:Total Ordering|total ordering]], then $\forall x \in R$, either $x \le 0_R$ or $0_R \le x$, and the result follows. +=== Sufficient Condition === +Let $P \subseteq R$ such that $(1)$, $(2)$ and $(3)$ are satisfied. +By item $(OR2)$ of [[Properties of Ordered Ring]], we have: +:$x \le y \iff 0 \le y + \paren {-x}$ +so there is at most one ordering on $R$ compatible with the ring structure of $R$ such that $P = R_{\ge 0}$, namely, the one that satisfies: +:$x \le y \iff y + \left({-x}\right) \in P$ +Now we need to show that $\le$ thus defined has the required properties. +;[[Definition:Reflexive Relation|Reflexivity]]: +By $(2)$: +:$\forall x \in R: x \le x$ because $x + \paren {-x} = 0_R \in P$ +;[[Definition:Antisymmetric Relation|Antisymmetry]]: +Let $x \le y$ and $y \le x$. +Then $y + \paren {-x} \in P$ and $-\paren {y + \paren {-x} } = x + \paren {-y} \in P$. +Thus by $(2)$, $y + \paren {-x} = 0_R$ and thus $y = x$. +;[[Definition:Transitive Relation|Transitivity]]: +If $x \le y$ and $y \le z$, then $y + \paren {-x} \in P$ and $z + \paren {-y} \in P$. +But as $z + \paren {-x} = z + \paren {-y} + y + \paren {-x}$, we have that $z + \paren {-x} \in P$ from $(1)$. +Hence $x \le z$. +Let $x \le y$. +Then $z + x \le z + y$ since $\paren {z + y} + \paren {-\paren {x + x} } = y + \paren {-x} \in P$. +Finally, [[Definition:Ordering Compatible with Ring Structure|Ordering Compatible with Ring]] holds because of $(3)$. +If $(4)$ holds, then $\forall x, y \in R$, either $y + \paren {-x} \in P$ or $x + \paren {-y} = -\paren {y + \paren {-x} } \in P$, that is, either $x \le y$ or $y \le x$. +{{qed}} +\end{proof}<|endoftext|> +\section{Symmetric Difference with Intersection forms Ring} +Tags: Commutative Algebra, Set Intersection, Symmetric Difference, Power Set, Symmetric Difference with Intersection forms Ring, Examples of Commutative and Unitary Rings + +\begin{theorem} +Let $S$ be a [[Definition:Set|set]]. +Then $\struct {\powerset S, *, \cap}$ is a [[Definition:Commutative and Unitary Ring|commutative ring with unity]], in which the [[Definition:Unity of Ring|unity]] is $S$. +This ring is not an [[Definition:Integral Domain|integral domain]]. +\end{theorem} + +\begin{proof} +From [[Symmetric Difference on Power Set forms Abelian Group]], $\struct {\powerset S, *}$ is an [[Definition:Abelian Group|abelian group]], where $\O$ is the [[Definition:Identity Element|identity]] and each [[Definition:Element|element]] is [[Definition:Self-Inverse Element|self-inverse]]. +From [[Power Set with Intersection is Monoid]], $\struct {\powerset S, \cap}$ is a [[Definition:Commutative Monoid|commutative monoid]] whose [[Definition:Identity Element|identity]] is $S$. +Also [[Intersection Distributes over Symmetric Difference]]. +Thus $\struct {\powerset S, \cap}$ is a [[Definition:Commutative and Unitary Ring|commutative ring with a unity]] which is $S$. +From [[Intersection with Empty Set]]: +:$\forall A \in \powerset S: A \cap \O = \O = \O \cap A$ +Thus $\O$ is indeed the [[Definition:Ring Zero|zero]]. +However, from [[Set Intersection Not Cancellable]], it follows that $\struct {\powerset S, *, \cap}$ is not an [[Definition:Integral Domain|integral domain]]. +{{qed}} +\end{proof} + +\begin{proof} +From [[Power Set is Closed under Symmetric Difference]] and [[Power Set is Closed under Intersection]], we have that both $\struct {\powerset S, *}$ and $\struct {\powerset S, \cap}$ are [[Definition:Closed Algebraic Structure|closed]]. +Hence $\powerset S$ is a [[Definition:Ring of Sets|ring of sets]], and hence a [[Ring of Sets is Commutative Ring|commutative ring]]. +From [[Intersection with Subset is Subset]], we have $A \subseteq S \iff A \cap S = A$. Thus we see that $S$ is the [[Definition:Unity of Ring|unity]]. +Also during the proof of [[Power Set with Intersection is Monoid]], it was established that $S$ is the [[Definition:Identity Element|identity]] of $\struct {\powerset S, \cap}$. +We also note that [[Set Intersection Not Cancellable|set intersection is not cancellable]], so $\struct {\powerset S, *, \cap}$ is not an [[Definition:Integral Domain|integral domain]]. +The result follows. +{{qed}} +\end{proof}<|endoftext|> +\section{Subfield Test} +Tags: Subfields + +\begin{theorem} +Let $\struct {F, +, \circ}$ be a [[Definition:Field (Abstract Algebra)|field]] whose [[Definition:Field Zero|zero]] is $0_F$. +Let $K$ be a [[Definition:Subset|subset]] of $F$. +Then $\struct {K, +, \circ}$ is a [[Definition:Subfield|subfield]] of $\struct {F, +, \circ}$ {{iff}} these all hold: +:$(1): \quad K^* \ne \O$ +:$(2): \quad \forall x, y \in K: x + \paren {-y} \in K$ +:$(3): \quad \forall x, y \in K: x \circ y \in K$ +:$(4): \quad x \in K^* \implies x^{-1} \in K^*$ +where $K^*$ denotes $K \setminus \set {0_F}$. +\end{theorem} + +\begin{proof} +=== Necessary Condition === +Let $\struct {K, +, \circ}$ be a [[Definition:Subfield|subfield]] of $\struct {F, +, \circ}$. +Then the conditions $(1)$ to $(4)$ all hold by virtue of the [[Definition:Field Axioms|field axioms]]. +{{qed|lemma}} +=== Sufficient Condition === +Suppose the conditions $(1)$ to $(4)$ hold. +From the [[Division Subring Test]], it follows that $\struct {K, +, \circ}$ is a [[Definition:Division Ring|division ring]]. +As $\struct {F, +, \circ}$ is a [[Definition:Field (Abstract Algebra)|field]], then $\circ$ is [[Definition:Commutative Operation|commutative]] on all of $F$. +Therefore $\circ$ is [[Definition:Commutative Operation|commutative]] also on $K$ by [[Restriction of Commutative Operation is Commutative]]. +Thus $\struct {K, +, \circ}$ is a [[Definition:Field (Abstract Algebra)|field]]. +{{qed}} +\end{proof}<|endoftext|> +\section{Field of Quotients of Subdomain} +Tags: Fields of Quotients + +\begin{theorem} +Let $\struct {F, +, \circ}$ be a [[Definition:Field (Abstract Algebra)|field]] whose [[Definition:Unity of Ring|unity]] is $1_F$. +Let $\struct {D, +, \circ}$ be a [[Definition:Subdomain|subdomain]] of $\struct {F, +, \circ}$ whose [[Definition:Unity of Ring|unity]] is $1_D$. +Let: +:$K = \set {\dfrac x y: x \in D, y \in D^*}$ +where $\dfrac x y$ is the [[Definition:Division Product|division product]] of $x$ by $y$. +Then $\struct {K, +, \circ}$ is a [[Definition:Field of Quotients|field of quotients]] of $\struct {D, +, \circ}$. +\end{theorem} + +\begin{proof} +$1_D = 1_F$ by [[Subdomain Test]]. +The sum and product of two elements of $K$ are also in $K$ by [[Addition of Division Products]] and [[Product of Division Products]]. +The additive and product inverses of $K$ are also in $K$ by [[Negative of Division Product]] and [[Inverse of Division Product]]. +Thus by [[Subfield Test]], $\struct {K, +, \circ}$ is a [[Definition:Subfield|subfield]] of $\struct {F, +, \circ}$ which clearly contains $\struct {D, +, \circ}$. +Hence the result. +{{qed}} +\end{proof}<|endoftext|> +\section{Existence of Field of Quotients} +Tags: Fields of Quotients, Integral Domains + +\begin{theorem} +Let $\struct {D, +, \circ}$ be an [[Definition:Integral Domain|integral domain]]. +Then there exists a [[Definition:Field of Quotients|field of quotients]] of $\struct {D, +, \circ}$. +\end{theorem} + +\begin{proof} +Let $\struct {D, +, \circ}$ be an [[Definition:Integral Domain|integral domain]] whose [[Definition:Ring Zero|zero]] is $0_D$ and whose [[Definition:Unity of Ring|unity]] is $1_D$. +=== Inverse Completion is an Abelian Group === +By [[Inverse Completion of Integral Domain Exists]], we can define the [[Definition:Inverse Completion|inverse completion]] $\struct {K, \circ}$ of $\struct {D, \circ}$. +Thus $\struct {K, \circ}$ is a [[Definition:Commutative Semigroup|commutative semigroup]] such that: +:$(1): \quad$ The [[Definition:Identity Element|identity]] of $\struct {K, \circ}$ is $1_D$ +:$(2): \quad$ Every [[Definition:Element|element]] $x$ of $\struct {D^*, \circ}$ has an [[Definition:Inverse Element|inverse]] $\dfrac {1_D} x$ in $\struct {K, \circ}$ +:$(3): \quad$ Every [[Definition:Element|element]] of $\struct {K, \circ}$ is of the form $x \circ y^{-1}$ (which from the definition of [[Definition:Division Product|division product]], we can also denote $x / y$), where $x \in D, y \in D^*$. +It can also be noted that from [[Inverse Completion Less Zero of Integral Domain is Closed]], $\struct {K^*, \circ}$ is [[Definition:Closed Algebraic Structure|closed]]. +Hence $\struct {K^*, \circ}$ is an [[Definition:Abelian Group|abelian group]]. +=== Additive Operation on $K$ === +In what follows, we take for granted the rules of [[Definition:Associative Operation|associativity]], [[Definition:Commutative Operation|commutativity]] and [[Definition:Distributive Operation|distributivity]] of $+$ and $\circ$ in $D$. +We require to extend the [[Definition:Binary Operation|operation]] $+$ on $D$ to an [[Definition:Binary Operation|operation]] $+'$ on $K$, so that $\struct {K, +', \circ}$ is a [[Definition:Field (Abstract Algebra)|field]]. +By [[Addition of Division Products]], we define $+'$ as: +:$\forall a, c \in D, \forall b, d \in D^*: \dfrac a b +' \dfrac c d = \dfrac {a \circ d + b \circ c} {b \circ d}$ +where we have defined $\dfrac x y = x \circ y^{-1} = y^{-1} \circ x$ as [[Definition:Division Product|$x$ divided by $y$]]. +Next, we see that: +:$\forall a, b \in D: a +' b = \dfrac {a \circ 1_D + b \circ 1_D} {1_D \circ 1_D} = a + b$ +So $+$ [[Definition:Operation Induced by Restriction|induces the given operation $+$]] on its substructure $D$, and we are justified in using $+$ for both operations. +{{qed|lemma}} +=== Addition on $K$ makes an Abelian Group === +Now we verify that $\struct {K, +}$ is an [[Definition:Abelian Group|abelian group]] +Taking the [[Definition:Group Axioms|group axioms]] in turn: +==== $\text G 0$: Closure ==== +Let $\dfrac a b, \dfrac c d \in K$. +Then $a, c \in D$ and $b, d \in D^*$, and $\dfrac a b + \dfrac c d = \dfrac {a \circ d + b \circ c} {b \circ d}$. +As $b, d \in D^*$ it follows that $b \circ d \in D^*$ because $D$ is an [[Definition:Integral Domain|integral domain]]. +By the fact of [[Definition:Closed Algebraic Structure|closure]] of $+$ and $\circ$ in $D$, $a \circ d + b \circ c \in D$. +Hence $\dfrac a b + \dfrac c d \in K$ and $+$ is [[Definition:Closed Algebraic Structure|closed]]. +{{qed|lemma}} +==== $\text G 1$: Associativity ==== +{{begin-eqn}} +{{eqn | l = \paren {\frac a b + \frac c d} + \frac e f + | r = \frac {a \circ d + b \circ c} {b \circ d} + \frac e f + | c = +}} +{{eqn | r = \frac {\paren {a \circ d + b \circ c} \circ f + b \circ d \circ e} {b \circ d \circ f} + | c = +}} +{{eqn | r = \frac {a \circ d \circ f + b \circ c \circ f + b \circ d \circ e} {b \circ d \circ f} + | c = +}} +{{eqn | r = \frac {a \circ d \circ f + b \circ \paren {c \circ f + d \circ e} } {b \circ d \circ f} + | c = +}} +{{eqn | r = \frac a b + \frac {c \circ f + d \circ e} {d \circ f} + | c = +}} +{{eqn | r = \frac a b + \paren {\frac c d + \frac e f} + | c = +}} +{{end-eqn}} +Hence $\dfrac a b + \dfrac c d \in K$ and $+$ is [[Definition:Associative Operation|associative]]. +{{qed|lemma}} +==== $\text G 2$: Identity ==== +The [[Definition:Identity Element|identity]] for $+$ is $\dfrac 0 k$ where $k \in D^*$: +{{begin-eqn}} +{{eqn | l = \frac a b + \frac 0 k + | r = \frac {a \circ k + b \circ 0} {b \circ k} + | c = +}} +{{eqn | r = \frac {a \circ k} {b \circ k} + | c = +}} +{{eqn | r = \frac a b + | c = +}} +{{end-eqn}} +Similarly for $\dfrac 0 k + \dfrac a b$. +{{qed|lemma}} +==== $\text G 3$: Inverses ==== +The [[Definition:Inverse Element|inverse]] of $\dfrac a b$ for $+$ is $\dfrac {-a} b$: +{{begin-eqn}} +{{eqn | l = \frac a b + \frac {-a} b + | r = \frac {a \circ b + b \circ \paren {-a} } {b \circ b} + | c = +}} +{{eqn | r = \frac {b \circ \paren {a + \paren {-a} } } {b \circ b} + | c = +}} +{{eqn | r = \frac {b \circ 0} {b \circ b} + | c = +}} +{{eqn | r = \frac 0 {b \circ b} + | c = +}} +{{end-eqn}} +From [[Existence of Field of Quotients#G2: Identity|above]], this is the [[Definition:Identity Element|identity]] for $+$. +Similarly, $\dfrac {-a} b + \dfrac a b = \dfrac 0 {b \circ b}$. +Hence $\dfrac {-a} b$ is the [[Definition:Inverse Element|inverse]] of $\dfrac a b$ for $+$. +{{qed|lemma}} +==== $\text C$: Commutativity ==== +{{begin-eqn}} +{{eqn | l = \frac a b + \frac c d + | r = \frac {a \circ d + b \circ c} {b \circ d} + | c = +}} +{{eqn | r = \frac {c \circ b + d \circ a} {d \circ b} + | c = +}} +{{eqn | r = \frac c d + \frac a b + | c = +}} +{{end-eqn}} +Therefore, $\struct {K, +, \circ}$ is a [[Definition:Commutative and Unitary Ring|commutative ring with unity]]. +{{qed|lemma}} +=== Product Distributes over Addition === +From [[Extension Theorem for Distributive Operations]], it follows directly that $\circ$ [[Definition:Distributive Operation|distributes]] over $+$. +{{qed|lemma}} +=== Product Inverses in $K$ === +From [[Ring Product with Zero]], we note that: +:$\forall x \in D, y \in D^*: \dfrac x y \ne 0_D \implies x \ne 0_D$ +From [[Inverse of Division Product]]: +:$\forall x, y \in D^*: \paren {\dfrac x y}^{-1} = \dfrac y x$ +Thus $\dfrac x y \in K$ has the [[Definition:Ring Product Inverse|ring product inverse]] $\dfrac y x \in K$. +{{qed|lemma}} +=== Inverse Completion is a Field === +We have that: +:the [[Definition:Algebraic Structure|algebraic structure]] $\struct {K, +}$ is an [[Definition:Abelian Group|abelian group]] +:the [[Definition:Algebraic Structure|algebraic structure]] $\struct {K^*, \circ}$ is an [[Definition:Abelian Group|abelian group]] +:the operation $\circ$ [[Definition:Distributive Operation|distributes]] over $+$. +Hence $\struct {K, +, \circ}$ is a [[Definition:Field (Abstract Algebra)|field]]. +We also have that $\struct {K, +, \circ}$ [[Definition:Submagma|contains $\struct {D, +, \circ}$ algebraically]] such that: +:$\forall x \in K: \exists z \in D, y \in D^*: z = \dfrac x y$ +Thus $\struct {K, +, \circ}$ is a [[Definition:Field of Quotients|field of quotients]] of $\struct {D, +, \circ}$. +{{Qed}} +\end{proof}<|endoftext|> +\section{Quotient Theorem for Monomorphisms} +Tags: Fields of Quotients, Integral Domains, Ring Monomorphisms, Named Theorems + +\begin{theorem} +Let $K, L$ be [[Definition:Field of Quotients|fields of quotients]] of [[Definition:Integral Domain|integral domains]] $\struct {R, +_R, \circ_R}, \struct {S, +_S, \circ_S}$ respectively. +Let $\phi: R \to S$ be a [[Definition:Ring Monomorphism|monomorphism]]. +Then there is one and only one [[Definition:Ring Monomorphism|monomorphism]] $\psi: K \to L$ extending $\phi$, and: +:$\forall x \in R, y \in R^*: \map \psi {\dfrac x y} = \dfrac {\map \phi x} {\map \phi y}$ +Also, if $\phi$ is a [[Definition:Ring Isomorphism|ring isomorphism]], then so is $\psi$. +\end{theorem} + +\begin{proof} +By definition, $\struct {K, \circ_R}$ and $\struct {L, \circ_S}$ are [[Definition:Inverse Completion|inverse completions]] of $\struct {R, \circ_R}$ and $\struct {S, \circ_S}$ respectively. +{{Questionable|this is not the definition}} +So by the [[Extension Theorem for Homomorphisms]], there is one and only one [[Definition:Ring Monomorphism|monomorphism]] $\psi: \struct {K, \circ_R} \to \struct {L, \circ_S$ extending $\phi$. +Thus: +:$\forall x \in R, y \in R^*: \map \psi {\dfrac x y} = \dfrac {\map \phi x} {\map \phi y}$ +By the [[Extension Theorem for Isomorphisms]], $\psi$ is an [[Definition:Isomorphism (Abstract Algebra)|isomorphism]] if $\phi$ is. +Thus, $\forall x, y \in R, z, w \in R^*$: +{{begin-eqn}} +{{eqn | l = \map \psi {\frac x z +_R \frac y w} + | r = \map \psi {\frac {\paren {x \circ_R w} +_R \paren {y \circ_R z} } {z \circ_R w} } + | c = [[Addition of Division Products]] +}} +{{eqn | r = \frac {\map \phi {\paren {x \circ_R w} +_R \paren {y \circ_R z} } } {\map \phi {z \circ_R w} } + | c = Definition of $\psi$ +}} +{{eqn | r = \frac {\paren {\map \phi x \circ_S \map \phi w} +_S \paren {\map \phi y \circ_S \map \phi z} } {\map \phi z \circ_S \map \phi w} + | c = [[Definition:Morphism Property|Morphism Property]] +}} +{{eqn | r = \frac {\map \phi x} {\map \phi z} +_S \frac {\map \phi y} {\map \phi w} + | c = [[Addition of Division Products]] +}} +{{eqn | r = \map \psi {\frac x z} +_S \map \psi {\frac y w} + | c = Definition of $\psi$ +}} +{{end-eqn}} +Thus $\psi: K \to L$ is a [[Definition:Ring Monomorphism|monomorphism]]. +{{qed}} +\end{proof}<|endoftext|> +\section{Field of Quotients is Unique} +Tags: Fields of Quotients, Field Isomorphisms, Integral Domains + +\begin{theorem} +Let $\struct {D, +, \circ}$ be an [[Definition:Integral Domain|integral domain]]. +Let $K, L$ be [[Definition:Field of Quotients|field of quotients]] of $\struct {D, +, \circ}$. +Then there is [[Definition:Exactly One|one and only one]] [[Definition:Field Isomorphism|(field) isomorphism]] $\phi: K \to L$ satisfying: +:$\forall x \in D: \map \phi x = x$ +\end{theorem} + +\begin{proof} +Follows directly from the [[Quotient Theorem for Monomorphisms]]. +{{qed}} +\end{proof}<|endoftext|> +\section{Divided by Positive Element of Field of Quotients} +Tags: Fields of Quotients, Integral Domains + +\begin{theorem} +Let $\struct {K, +, \circ}$ be the [[Definition:Field of Quotients|field of quotients]] of a [[Definition:Totally Ordered Ring|totally ordered]] [[Definition:Integral Domain|integral domain]] $\struct {D, +, \circ, \le}$. +Then: +:$\forall z \in K: \exists x, y \in D: z = \dfrac x y, y \in D_{>0}$ +\end{theorem} + +\begin{proof} +By definition of [[Definition:Field of Quotients|field of quotients]]: +:$\forall z \in K: \exists x, y \in D: z = \dfrac x y, y \in D_{\ne 0}$ +Suppose $z = x' / y'$ such that $y' \notin D_{>0}$. +Then $y' < 0$ as $D$ is [[Definition:Totally Ordered Ring|totally ordered]]. +Then: +{{begin-eqn}} +{{eqn | l = x' / y' + | r = x' \circ \paren {y'}^{-1} + | c = {{Defof|Division}} +}} +{{eqn | r = \paren {-x'} \circ \paren {-y'}^{-1} + | c = [[Product of Ring Negatives]] +}} +{{eqn | r = \paren {-x'} / \paren {-y'} + | c = {Defof|Division}} +}} +{{end-eqn}} +{{explain|first to second step above assumes more than is stated: $-\paren {y'}^{-1} {{=}} \paren {-y'}^{-1}$ - add that step and link to that result}} +If $y' < 0$, then $\paren {-y'} > 0$ from [[Properties of Ordered Ring]] $(4)$. +So all we need to do is set $x = -x', y = -y'$ and the result follows. +{{qed}} +[[Category:Fields of Quotients]] +[[Category:Integral Domains]] +1i02g1dcijv3u3rs5199dhvjn1utptf +\end{proof}<|endoftext|> +\section{Total Ordering on Field of Quotients is Unique} +Tags: Fields of Quotients, Integral Domains + +\begin{theorem} +Let $\struct {K, +, \circ}$ be a [[Definition:Field of Quotients|field of quotients]] of an [[Definition:Ordered Integral Domain|ordered integral domain]] $\struct {D, +, \circ, \le}$. +Then there is one and only one [[Definition:Total Ordering|total ordering]] $\le'$ on $K$ which is [[Definition:Ordering Compatible with Ring Structure|compatible with its ring structure]] and induces on $D$ its given [[Definition:Total Ordering|total ordering]] $\le$. +That ordering is the one defined by: +:$P = \set {\dfrac x y \in K: x \in D_+, y \in D_+^*}$ +\end{theorem} + +\begin{proof} +First, note that from [[Divided by Positive Element of Field of Quotients]]: +:$\forall z \in K: \exists x, y \in R: z = \dfrac x y, y \in R_+^*$ +Now we show that $P$ satistfies conditions $(1)$ to $(4)$ of [[Positive Elements of Ordered Ring]]. +From [[Addition of Division Products]] and [[Product of Division Products]], it is clear that $P$ satisfies $(1)$ and $(3)$. +{{qed|lemma}} +Next we establish $(2)$. +Let $z \in P \cap \paren {-P}$. +Then $z \in P$ and $z \in \paren {-P}$. Thus: +:$\exists x_1, x_2 \in D_+, y_1, y_2 \in D_+^*: z = x_1 / y_1, -z = x_2 / y_2$ +So: +:$x_1 / y_1 = -\paren {x_2 / y_2} \implies x_1 \circ y_2 = -\paren {x_2 \circ y_1}$ +But $0 \le x_1 \circ y_2$ and $-\paren {x_2 \circ y_1} \le 0$. +So $x_1 \circ y_2 = 0$. +As $0 < y_2$, this means $x = 0$ and therefore $z = 0$. +Thus $(2)$ has been established. +{{qed|lemma}} +Now to show that $(4)$ holds. +Let $z = x / y$ where $x \in D, y \in D_+$. +Suppose $0 \le x$. Then $z \in P$. +However, suppose $x < 0$. Then $0 < \paren {-x}$ so $-z = \paren {-x} / y \in P$. +Thus: +:$z = -\paren {-z} \in -P$ +So: +:$P \cup \paren {-P} = K$ +So by [[Positive Elements of Ordered Ring]], the relation $\le'$ on $K$ defined by $P$ is a [[Definition:Total Ordering|total ordering]] on $K$ [[Definition:Ordering Compatible with Ring Structure|compatible with its ring structure]]. +{{qed|lemma}} +Now we need to show that the ordering induced on $D$ by $\le'$ is indeed $\le$. +Let $z \in D_+$. Then $z = z / 1_D \in P$, as $1_D \in D_+^*$. +Thus $D_+ \subseteq P$ and $D_+ \subseteq D$ so $D_+ \subseteq D \cap P$ from [[Intersection is Largest Subset]]. +Conversely, let $z \in D \cap P$. Then: +:$\exists x \in D_+, y \in D_+^*: z = x / y$ +If $x = 0$ then $z = 0$, and if $0 < x$ then as $z \circ y = x$ and $0 < y$, it follows that $0 < z$ by item 1 of [[Properties of Ordered Ring#Total Ordering|Properties of a Totally Ordered Ring]]. +So $\forall z \in D: 0 \le z \iff z \in P$. +Thus it follows that $z \in D \cap P \implies z \in D_+$, i.e. $D \cap P \subseteq D_+$. +Thus $D_+ = D \cap P$. +By item $(2)$ of [[Properties of Ordered Ring]], we have: +:$x \le y \iff 0 \le y + \left({-x}\right)$ +and thus it follows that the ordering induced on $D$ by $\le'$ is $\le$. +{{qed|lemma}} +Now we need to show uniqueness. +Let $\preceq$ be a total ordering on $K$ which is compatible with its ring structure and induces on $D$ the ordering $\le$. +Let $Q = \set {z \in K: 0 \preceq z}$. +We now show that $Q = P$. +Let $x \in D_+, y \in D_+^*$. +Then $0 \preceq x$ and $0 \prec 1 / y$ from item 4 of [[Properties of Ordered Ring#Total Ordering|Properties of a Totally Ordered Ring]]. +Thus by [[Definition:Ordering Compatible with Ring Structure|compatibility with ring structure]], $0 \preceq x / y$. +Hence $P \subseteq Q$. +Conversely, let $z \in Q$. +Let $z = x / y$ where $x \in D, y \in D_+^*$. +Then $x = z \circ y$ and by [[Definition:Ordering Compatible with Ring Structure|compatibility with ring structure]], $0 \preceq x$. +Thus $0 \le x$ and hence $z \in P$, and so $Q \subseteq P$. +So $Q = P$. +Therefore, by item $(2)$ of [[Properties of Ordered Ring]], it follows that $\preceq$ is the same as $\le$. +{{Qed}} +{{improve|Put this in the context of Ordered Integral Domain.}} +\end{proof}<|endoftext|> +\section{Order Embedding between Quotient Fields is Unique} +Tags: Ordered Rings, Fields of Quotients, Total Orderings, Order Embeddings + +\begin{theorem} +Let $\struct {R_1, +_1, \circ_1, \le_1}$ and $\struct {S, +_2, \circ_2, \le_2}$ be [[Definition:Totally Ordered Ring|totally ordered]] [[Definition:Integral Domain|integral domains]]. +Let $K, L$ be [[Definition:Totally Ordered Ring|totally ordered]] [[Definition:Field of Quotients|fields of quotients]] of $\struct {R_1, +_1, \circ_1, \le_1}$ and $\struct {S, +_2, \circ_2, \le_2}$ respectively. +Let $\phi: R \to S$ be a [[Definition:Order Embedding|order embedding]]. +Then there is one and only one [[Definition:Order Embedding|order embedding]] $\psi: K \to L$ extending $\phi$. +Also: +:$\forall x \in R, y \in R_{\ne 0}: \map \psi {\dfrac x y} = \dfrac {\map \phi x} {\map \phi y}$ +If $\phi: R \to S$ is an [[Definition:Order Isomorphism|order isomorphism]], then so is $\psi$. +\end{theorem} + +\begin{proof} +By [[Field of Quotients is Unique]], all we need to show is: +:$\forall x_1, x_2 \in R, y_1, y_2 \in R_{> 0}: \dfrac {x_1} {y_1} \le \dfrac {x_2} {y_2} \iff \dfrac {\map \phi {x_1}} {\map \phi {y_1} } \le \dfrac {\map \phi {x_2} } {\map \phi {y_2} }$ +Let $x_1 / y_1 \le x_2 / y_2$, where $y_1, y_2 \in R_{> 0}$. +As $y_1, y_2 \in R_{> 0}$, it follows that $0 < y_1 \circ_1 y_2$ and $0 < 1 / \paren {y_1 \circ_1 y_2}$. +We also have: +:$0 < \map \phi {y_1} \circ_2 \map \phi {y_2} = \map \phi {y_1 \circ_1 y_2}$ +Therefore: +:$x_1 \circ_1 y_2 = \frac {x_1} {y_1} \circ_1 \paren {y_1 \circ y_2} \le \dfrac {x_2} {y_2} \circ_1 \paren {y_1 \circ_1 y_2} = x_2 \circ_1 y_1$ +Conversely, let $x_1 \circ_1 y_2 \le x_2 \circ_1 y_1$. +Then: +:$\dfrac {x_1} {y_1} = x_1 \circ_1 y_2 \circ_1 \paren {\dfrac 1 {y_1 \circ_1 y_2} } \le x_2 \circ_1 y_1 \circ_1 \paren {\dfrac 1 {y_1 \circ_1 y_2} } = \dfrac {x_2} {y_2}$ +That is, we have: +:$\dfrac {x_1} {y_1} \le \dfrac {x_2} {y_2} \iff x_1 \circ_1 y_2 \le x_2 \circ_1 y_1$ +Similarly: +:$\dfrac {\map \phi {x_1} } {\map \phi {y_1} } \le \dfrac {\map \phi {x_2} } {\map \phi {y_2} } \iff \map \phi {x_1} \circ_2 \map \phi {y_2} \le \map \phi {x_2} \circ_2 \map \phi {y_1}$ +Now $\phi: R \to S$ is an [[Definition:Order Embedding|order embedding]]. +Therefore: +:$x_1 \circ_1 y_2 \le x_2 \circ_1 y_1 \iff \map \phi {x_1 \circ_1 y_2} \le \map \phi {x_2 \circ_1 y_1}$ +The result follows. +{{qed}} +\end{proof}<|endoftext|> +\section{Strict Ordering Preserved under Product with Cancellable Element} +Tags: Ordered Semigroups + +\begin{theorem} +Let $\left({S, \circ, \preceq}\right)$ be an [[Definition:Ordered Semigroup|ordered semigroup]]. +Let $x, y, z \in S$ be such that: +:$(1): \quad z$ is [[Definition:Cancellable Element|cancellable]] for $\circ$ +:$(2): \quad x \prec y$ +Then: +: $x \circ z \prec y \circ z$ +: $z \circ x \prec z \circ y$ +\end{theorem} + +\begin{proof} +Let $z$ be [[Definition:Cancellable Element|cancellable]] and $x \prec y$. +Then by the definition of [[Definition:Ordered Semigroup|ordered semigroup]]: +: $x \circ z \preceq y \circ z$ +From the fact that $z$ is [[Definition:Cancellable Element|cancellable]]: +: $x \circ z = y \circ z \iff x = y$ +Thus as $x \circ z \ne y \circ z$ it follows from [[Strictly Precedes is Strict Ordering]] that: +: $x \circ z \prec y \circ z$ +Similarly, $z \circ x \prec z \circ y$ follows from $z \circ x \preceq z \circ y$. +{{qed}} +\end{proof}<|endoftext|> +\section{Ordering of Inverses in Ordered Monoid} +Tags: Order Theory, Monoids + +\begin{theorem} +Let $\left({S, \circ, \preceq}\right)$ be an [[Definition:Ordered Monoid|ordered monoid]] whose [[Definition:Identity Element|identity]] is $e$. +Let $x, y \in S$ be [[Definition:Invertible Element|invertible]]. +Then: +:$x \prec y \iff y^{-1} \prec x^{-1}$ +\end{theorem} + +\begin{proof} +=== Necessary Condition === +{{begin-eqn}} +{{eqn | l = x + | o = \prec + | r = y + | c = +}} +{{eqn | ll= \implies + | l = e + | r = x^{-1} \circ x \prec x^{-1} \circ y + | c = [[Strict Ordering Preserved under Product with Cancellable Element]] +}} +{{eqn | ll= \implies + | l = y^{-1} + | r = e \circ y^{-1} \prec x^{-1} \circ y \circ y^{-1} = x^{-1} + | c = +}} +{{eqn | ll= \implies + | l = y^{-1} + | o = \prec + | r = x^{-1} + | c = +}} +{{end-eqn}} +{{qed|lemma}} +=== Sufficient Condition === +{{begin-eqn}} +{{eqn | l = y^{-1} + | o = \prec + | r = x^{-1} + | c = +}} +{{eqn | ll= \implies + | l = x + | r = \left({x^{-1} }\right)^{-1} \prec \left({y^{-1} }\right)^{-1} = y + | c = +}} +{{eqn | ll= \implies + | l = x + | o = \prec + | r = y + | c = +}} +{{end-eqn}} +{{qed}} +\end{proof}<|endoftext|> +\section{Ordered Group Equivalences} +Tags: Ordered Groups + +\begin{theorem} +Let $\left({S, \circ, \preceq}\right)$ be an [[Definition:Ordered Group|ordered group]] whose [[Definition:Identity Element|identity]] is $e$. +Let $x, y, z, \in S$. +Then the following are all equivalent: +: $(1): \quad x \prec y$ +: $(2): \quad x \circ z \prec y \circ z$ +: $(3): \quad z \circ x \prec z \circ y$ +: $(4): \quad y^{-1} \prec x^{-1}$ +: $(5): \quad e \prec y \circ x^{-1}$ +: $(6): \quad e \prec x^{-1} \circ y$ +\end{theorem} + +\begin{proof} +Since $\left({S, \circ, \preceq}\right)$ is an [[Definition:Ordered Group|ordered group]], we have, for all $x,y,z \in S$: +:$(R): \quad x \prec y \implies x \circ z \prec y \circ z$ +:$(L): \quad x \prec y \implies z \circ x \prec z \circ y$ +Using $(R)$ and $(L)$, let us demonstrate the equivalences. +=== $(1) \iff (2)$ and $(1) \iff (5)$ === +We have by applying $(R)$ twice: +{{begin-eqn}} +{{eqn|l = x + |o = \prec + |r = y +}} +{{eqn|ll= \implies + |l = x \circ z + |o = \prec + |r = y \circ z +}} +{{eqn|ll= \implies + |l = x \circ z \circ z^{-1} + |o = \prec + |r = y \circ z \circ z^{-1} +}} +{{end-eqn}} +This last identity comes down to $x \prec y$ by definition of [[Definition:Inverse (Abstract Algebra)|inverse]]. +Selecting $z = x^{-1}$ proves $(1) \iff (5)$ as well. +{{qed|lemma}} +=== $(1) \iff (3)$ and $(1) \iff (6)$ === +We have by applying $(L)$ twice: +{{begin-eqn}} +{{eqn|l = x + |o = \prec + |r = y +}} +{{eqn|ll= \implies + |l = z \circ x + |o = \prec + |r = z \circ y +}} +{{eqn|ll= \implies + |l = z^{-1} \circ z \circ x + |o = \prec + |r = z^{-1} \circ z \circ x +}} +{{end-eqn}} +This last identity comes down to $x \prec y$ by definition of [[Definition:Inverse (Abstract Algebra)|inverse]]. +Selecting $z = x^{-1}$ proves $(1) \iff (6)$ as well. +{{qed|lemma}} +=== $(4) \iff (5)$ === +Applying that we know $(1) \iff (2)$, we obtain: +{{begin-eqn}} +{{eqn|l = y^{-1} + |o = \prec + |r = x^{-1} +}} +{{eqn|ll= \iff + |l = e + |o = \prec + |r = y \circ x^{-1} +}} +{{end-eqn}} +since $y \circ y^{-1} = e$. +{{qed|lemma}} +The remaining equivalences follow from [[Material Equivalence is Equivalence]]. +{{qed}} +\end{proof}<|endoftext|> +\section{Monomorphism from Total Ordering} +Tags: Total Orderings, Monomorphisms + +\begin{theorem} +Let the following conditions hold: +: $(1): \quad$ Let $\left({S, \circ, \preceq}\right)$ and $\left({T, *, \preccurlyeq}\right)$ be [[Definition:Ordered Semigroup|ordered semigroups]]. +: $(2): \quad$ Let $\phi: S \to T$ be a [[Definition:Mapping|mapping]]. +: $(3): \quad$ Let $\preceq$ be a [[Definition:Total Ordering|total ordering]] on $S$. +Then $\phi \left({S, \circ, \preceq}\right) \to \left({T, *, \preccurlyeq}\right)$ is a [[Definition:Ordered Structure Monomorphism|(structure) monomorphism]] iff: +: $(1): \quad \phi$ is [[Definition:Strictly Increasing|strictly increasing]] from $\left({S, \preceq}\right)$ into $\left({T, \preccurlyeq}\right)$; +: $(2): \quad \phi$ is a [[Definition:Homomorphism (Abstract Algebra)|homomorphism]] from $\left({S, \circ}\right)$ into $\left({T, *}\right)$. +\end{theorem} + +\begin{proof} +This follows: +: $(1): \quad$ As a direct consequence of [[Mapping from Totally Ordered Set is Order Embedding iff Strictly Increasing]] +: $(2): \quad$ From the definition of [[Definition:Monomorphism (Abstract Algebra)|monomorphism]] as a [[Definition:Homomorphism (Abstract Algebra)|homomorphism]] which is an [[Definition:Injection|injection]]. +{{Qed}} +\end{proof}<|endoftext|> +\section{Extension Theorem for Total Orderings} +Tags: Total Orderings, Semigroups, Named Theorems + +\begin{theorem} +Let the following conditions be fulfilled: +:$(1):\quad$ Let $\struct {S, \circ, \preceq}$ be a [[Definition:Totally Ordered Commutative Semigroup|totally ordered commutative semigroup]] +:$(2):\quad$ Let all the [[Definition:Element|elements]] of $\struct {S, \circ, \preceq}$ be [[Definition:Cancellable Element|cancellable]] +:$(3):\quad$ Let $\struct {T, \circ}$ be an [[Definition:Inverse Completion|inverse completion]] of $\struct {S, \circ}$. +Then: +:$(1):\quad$ The [[Definition:Relation|relation]] $\preceq'$ on $T$ satisfying $\forall x_1, x_2, y_1, y_2 \in S: x_1 \circ \paren {y_1}^{-1} \preceq' x_2 \circ \paren {y_2}^{-1} \iff x_1 \circ y_2 \preceq x_2 \circ y_1$ is a [[Definition:Well-Defined Relation|well-defined relation]] +:$(2):\quad$ $\preceq'$ is the only [[Definition:Total Ordering|total ordering]] on $T$ [[Definition:Relation Compatible with Operation|compatible with $\circ$]] +:$(3):\quad$ $\preceq'$ is the only [[Definition:Total Ordering|total ordering]] on $T$ that induces the given [[Definition:Total Ordering|ordering]] $\preceq$ on $S$. +\end{theorem} + +\begin{proof} +By [[Inverse Completion is Commutative Semigroup]]: +:every [[Definition:Element|element]] of $T$ is of the form $x \circ y^{-1}$ where $x, y \in S$. +=== Proof that Relation is Well-Defined === +First we need to show that $\preceq'$ is [[Definition:Well-Defined Relation|well-defined]]. +So we need to show that if $x_1, x_2, y_1, y_2, z_1, z_2, w_1, w_2 \in S$ satisfy: +{{begin-eqn}} +{{eqn | l = x_1 \circ \paren {y_1}^{-1} + | r = x_2 \circ \paren {y_2}^{-1} + | c = +}} +{{eqn | l = z_1 \circ \paren {w_1}^{-1} + | r = z_2 \circ \paren {w_2}^{-1} + | c = +}} +{{eqn | l = x_1 \circ w_1 + | o = \preceq + | r = y_1 \circ z_1 + | c = +}} +{{end-eqn}} +then: +:$x_2 \circ \paren {w_2}^{-1} = y_2 \circ \paren {z_2}^{-1}$ +We have: +{{begin-eqn}} +{{eqn | l = x_1 \circ \paren {y_1}^{-1} + | r = x_2 \circ \paren {y_2}^{-1} + | c = +}} +{{eqn | ll= \leadstoandfrom + | l = x_1 \circ y_2 + | r = x_2 \circ y_1 + | c = +}} +{{end-eqn}} +and: +{{begin-eqn}} +{{eqn | l = z_1 \circ \paren {w_1}^{-1} + | r = z_2 \circ \paren {w_2}^{-1} + | c = +}} +{{eqn | ll= \leadstoandfrom + | l = z_1 \circ w_2 + | r = z_2 \circ w_1 + | c = +}} +{{end-eqn}} +So: +{{begin-eqn}} +{{eqn | l = x_2 \circ w_2 \circ y_1 \circ z_1 + | r = x_1 \circ w_1 \circ y_2 \circ z_2 + | c = +}} +{{eqn | o = \preceq + | r = y_1 \circ z_1 \circ y_2 \circ z_2 + | c = +}} +{{eqn | ll= \leadsto + | l = x_2 \circ w_2 + | o = \preceq + | r = y_2 \circ z_2 + | c = [[Strict Ordering Preserved under Product with Cancellable Element]] +}} +{{end-eqn}} +Thus $\preceq'$ is a [[Definition:Well-Defined Relation|well-defined relation]] on $T$. +{{qed|lemma}} +=== Proof that Relation is Transitive === +To show that $\preceq'$ is [[Definition:Transitive Relation|transitive]]: +{{begin-eqn}} +{{eqn | ll= \forall x, y, z, w, u, v \in S: + | l = x \circ y^{-1} + | o = \preceq' + | r = z \circ w^{-1} + | c = +}} +{{eqn | l = z \circ w^{-1} + | o = \preceq' + | r = u \circ v^{-1} + | c = +}} +{{eqn | ll= \leadsto + | l = x \circ w \circ v + | o = \preceq + | r = y \circ z \circ v + | c = +}} +{{eqn | o = \preceq + | r = y \circ w \circ u + | c = +}} +{{eqn | ll= \leadsto + | l = x \circ v + | o = \preceq + | r = y \circ u + | c = [[Strict Ordering Preserved under Product with Cancellable Element]] +}} +{{eqn | ll= \leadsto + | l = x \circ y^{-1} + | o = \preceq' + | r = u \circ v^{-1} + | c = +}} +{{end-eqn}} +Thus $\preceq'$ is [[Definition:Transitive Relation|transitive]]. +{{qed|lemma}} +=== Proof that Relation is Total Ordering === +To show that $\preceq'$ is a [[Definition:Total Ordering|total ordering]] on $T$ [[Definition:Relation Compatible with Operation|compatible with $\circ$]]: +Let $z_1, z_2 \in T$. +Then: +:$\exists x_1, x_2, y_1, y_2 \in S: z_1 = x_1 \circ y_1^{-1}, z_2 = x_2 \circ y_2^{-1}$ +Then: +:$z_1 \circ y_1 = x_1$ +:$z_2 \circ y_2 = x_2$ +As $\preceq$ is a total ordering on $S$, either: +:$z_1 \circ y_1 \circ y_2 \preceq z_2 \circ y_2 \circ y_1$ +or: +:$z_2 \circ y_2 \circ y_1 \preceq z_1 \circ y_1 \circ y_2$ +{{WLOG}}, suppose that: +:$z_1 \circ y_1 \circ y_2 \preceq z_2 \circ y_2 \circ y_1$ +So: +{{begin-eqn}} +{{eqn | l = z_1 \circ y_1 \circ y_2 + | o = \preceq + | r = z_2 \circ y_2 \circ y_1 + | c = +}} +{{eqn | ll= \leadsto + | l = x_1 \circ y_2 + | o = \preceq + | r = x_2 \circ y_1 + | c = +}} +{{eqn | ll= \leadsto + | l = x_1 \circ y_1^{-1} + | o = \preceq' + | r = x_2 \circ y_2^{-1} + | c = +}} +{{eqn | ll= \leadsto + | l = z_1 + | o = \preceq' + | r = z_2 + | c = +}} +{{end-eqn}} +and it is seen that $\preceq'$ is a [[Definition:Total Ordering|total ordering]] on $T$. +{{qed|lemma}} +=== Proof that Relation is Compatible with Operation === +Let $x_1, x_2, y_1, y_2 \in T$ such that $x_1 \preceq' x_2, y_1 \preceq' y_2$. +We need to show that $x_1 \circ y_1 \preceq' x_2 \circ y_2$. +Let: +: $x_1 = r_1 \circ s_1^{-1}, x_2 = r_2 \circ s_2^{-1}$ +: $y_1 = u_1 \circ v_1^{-1}, y_2 = u_2 \circ v_2^{-1}$ +We have: +{{begin-eqn}} +{{eqn | l = x_1 + | o = \preceq' + | r = x_2 + | c = +}} +{{eqn | ll= \leadsto + | l = r_1 \circ s_1^{-1} + | o = \preceq' + | r = r_2 \circ s_2^{-1} + | c = +}} +{{eqn | ll= \leadsto + | l = r_1 \circ s_2 + | o = \preceq + | r = r_2 \circ s_1 + | c = +}} +{{end-eqn}} +and +{{begin-eqn}} +{{eqn | l = y_1 + | o = \preceq' + | r = y_2 + | c = +}} +{{eqn | ll= \leadsto + | l = u_1 \circ v_1^{-1} + | o = \preceq' + | r = u_2 \circ v_2^{-1} + | c = +}} +{{eqn | ll= \leadsto + | l = u_1 \circ v_2 + | o = \preceq + | r = u_2 \circ v_1 + | c = +}} +{{end-eqn}} +Because of the [[Definition:Relation Compatible with Operation|compatibility of $\preceq$ with $\circ$]] on $S$: +{{begin-eqn}} +{{eqn | l = \paren {r_1 \circ s_2} \circ \paren {u_1 \circ v_2} + | o = \preceq + | r = \paren {r_2 \circ s_1} \circ \paren {u_2 \circ v_1} + | c = +}} +{{eqn | ll= \leadsto + | l = \paren {r_1 \circ u_1} \circ \paren {s_2 \circ v_2} + | o = \preceq' + | r = \paren {r_2 \circ u_2} \circ \paren {s_1 \circ v_1} + | c = +}} +{{eqn | ll= \leadsto + | l = \paren {r_1 \circ u_1} \circ \paren {s_1 \circ v_1}^{-1} + | o = \preceq' + | r = \paren {r_2 \circ u_2} \circ \paren {s_2 \circ v_2}^{-1} + | c = +}} +{{eqn | ll= \leadsto + | l = \paren {r_1 \circ s_1^{-1} } \circ \paren {u_1 \circ v_1^{-1} } + | o = \preceq' + | r = \paren {r_2 \circ s_2^{-1} } \circ \paren {u_2 \circ v_2^{-1} } + | c = +}} +{{eqn | ll= \leadsto + | l = x_1 \circ y_1 + | o = \preceq' + | r = x_2 \circ y_2 + | c = +}} +{{end-eqn}} +Thus [[Definition:Relation Compatible with Operation|compatibility]] is proved. +{{qed|lemma}} +=== Proof about Restriction of Relation === +To show that the restriction of $\preceq'$ to $S$ is $\preceq$: +{{begin-eqn}} +{{eqn | l = \forall x, y \in S: x + | o = \preceq + | r = y + | c = +}} +{{eqn | ll= \leadsto + | l = \forall a \in S: \paren {x \circ a} \circ a + | o = \preceq + | r = \paren {y \circ a} \circ a + | c = +}} +{{eqn | ll= \leadsto + | l = x + | r = \paren {x \circ a} \circ a^{-1} + | c = +}} +{{eqn | o = \preceq + | r = \paren {y \circ a} \circ a^{-1} + | c = +}} +{{eqn | r = y + | c = +}} +{{end-eqn}} +Conversely: +{{begin-eqn}} +{{eqn | l = \forall x, y \in S: x + | o = \preceq' + | r = y + | c = +}} +{{eqn | ll= \leadsto + | l = \exists u, v, z, w \in S: x + | r = u \circ v^{-1} + | c = +}} +{{eqn | l = y + | r = z \circ w^{-1} + | c = +}} +{{eqn | l = u \circ w + | o = \preceq + | r = z \circ v + | c = +}} +{{eqn | ll= \leadsto + | l = x \circ v \circ w + | r = u \circ w + | c = +}} +{{eqn | o = \preceq + | r = z \circ v + | c = +}} +{{eqn | r = y \circ v \circ w + | c = +}} +{{eqn | ll= \leadsto + | l = x + | o = \preceq + | r = y + | c = [[Strict Ordering Preserved under Product with Cancellable Element]] +}} +{{end-eqn}} +{{qed|lemma}} +=== Proof that Relation is Unique === +To show that $\preceq'$ is [[Definition:Unique|unique]]: +Let $\preceq^*$ be any [[Definition:Ordering|ordering]] on $T$ [[Definition:Relation Compatible with Operation|compatible with $\circ$]] that induces $\preceq$ on $S$. +Then: +{{begin-eqn}} +{{eqn | ll= \forall x, y, z, w \in S: + | l = x \circ y^{-1} + | o = \preceq^* + | r = z \circ w^{-1} + | c = +}} +{{eqn | ll= \leadstoandfrom + | l = x \circ w + | o = \preceq + | r = y \circ z + | c = +}} +{{end-eqn}} +Then: +{{begin-eqn}} +{{eqn | l = x \circ y^{-1} + | o = \preceq^* + | r = z \circ w^{-1} + | c = +}} +{{eqn | ll= \leadsto + | l = x \circ w + | r = \paren {x \circ y^{-1} } \circ \paren {y \circ w} + | c = +}} +{{eqn | o = \preceq + | r = \paren {z \circ w^{-1} } \circ \paren {y \circ w} + | c = +}} +{{eqn | r = y \circ z + | c = +}} +{{end-eqn}} +{{begin-eqn}} +{{eqn | l = x \circ w + | o = \preceq + | r = y \circ z + | c = +}} +{{eqn | ll= \leadsto + | l = x \circ y^{-1} + | r = \paren {x \circ w} \circ w^{-1} \circ y^{-1} + | c = +}} +{{eqn | o = \preceq^* + | r = \paren {y \circ z} \circ w^{-1} \circ y^{-1} + | c = +}} +{{eqn | r = z \circ w^{-1} + | c = +}} +{{end-eqn}} +So: +{{begin-eqn}} +{{eqn | ll= \forall x, y, z, w \in S: + | l = x \circ y^{-1} + | o = \preceq^* + | r = z \circ w^{-1} + | c = +}} +{{eqn | ll= \leadstoandfrom + | l = x \circ w + | o = \preceq + | r = y \circ z + | c = +}} +{{end-eqn}} +Hence as every [[Definition:Element|element]] of $T$ is of the form $x \circ y^{-1}$ where $x, y \in S$, the [[Definition:Total Ordering|orderings]] $\preceq^*$ and $\preceq'$ are identical. +{{qed}} +\end{proof}<|endoftext|> +\section{Cancellability in Naturally Ordered Semigroup} +Tags: Naturally Ordered Semigroup + +\begin{theorem} +Let $\left({S, \circ, \preceq}\right)$ be a [[Definition:Naturally Ordered Semigroup|naturally ordered semigroup]]. +Then: +\end{theorem}<|endoftext|> +\section{Strict Lower Closure of Sum with One} +Tags: Naturally Ordered Semigroup + +\begin{theorem} +Let $\left({S, \circ, \preceq}\right)$ be a [[Definition:Naturally Ordered Semigroup|naturally ordered semigroup]]. +Then: +:$\forall n \in \left({S, \circ, \preceq}\right): \left({n \circ 1}\right)^\prec = n^\prec \cup \left\{{n}\right\}$ +where $n^\prec$ is defined as the [[Definition:Strict Lower Closure of Element|strict lower closure]] of $n$, that is, the set of elements [[Definition:Strictly Precede|strictly preceding]] $n$. +\end{theorem} + +\begin{proof} +First note that as $\left({S, \circ, \preceq}\right)$ is [[Definition:Naturally Ordered Semigroup|well-ordered]] and hence [[Definition:Totally Ordered Set|totally ordered]], the [[Trichotomy Law (Ordering)|Trichotomy Law]] applies. +Thus: +{{begin-eqn}} +{{eqn | l = \forall m \in S: + | o = + | r = m \notin n^\prec + | c = +}} +{{eqn | o = \iff + | r = \neg \ m \prec n + | c = Definition of [[Definition:Strict Lower Closure of Element|Strict Lower Closure]] +}} +{{eqn | o = \iff + | r = m = n \lor n \prec m + | c = [[Trichotomy Law (Ordering)|Trichotomy Law]] +}} +{{eqn | o = \iff + | r = n \preceq m + | c = Definition of [[Definition:Strictly Precede|Strictly Precede]] +}} +{{end-eqn}} +So: +{{begin-eqn}} +{{eqn | l = \forall p \in S: + | o = + | r = p \notin \left({n \circ 1}\right)^\prec + | c = +}} +{{eqn | o = \iff + | r = n \circ 1 \preceq p + | c = from the above +}} +{{eqn | o = \iff + | r = n \prec p + | c = [[Sum with One is Immediate Successor in Naturally Ordered Semigroup]] +}} +{{end-eqn}} +Similarly: +{{begin-eqn}} +{{eqn | l = \forall p \in S: + | o = + | r = p \notin n^\prec \cup \left\{ {n}\right\} + | c = +}} +{{eqn | o = \iff + | r = n \preceq p \land n \ne p + | c = from the above +}} +{{eqn | o = \iff + | r = n \prec p + | c = Definition of [[Definition:Strictly Precede|Strictly Precede]] +}} +{{end-eqn}} +So: +:$p \notin n^\prec \cup \left\{{n}\right\} \iff p \notin \left({n \circ 1}\right)^\prec$ +Thus: +:$\complement_S \left({\left({n \circ 1}\right)^\prec}\right) = \complement_S \left({n^\prec \cup \left\{{n}\right\}}\right)$ +from the [[Definition:Relative Complement|definition of relative complement]]. +So: +{{begin-eqn}} +{{eqn | l = \left({n \circ 1}\right)^\prec + | r = \complement_S \left({\complement_S \left({\left({n \circ 1}\right)^\prec}\right)}\right) + | c = [[Relative Complement of Relative Complement]] +}} +{{eqn | r = \complement_S \left({\complement_S \left({n^\prec \cup \left\{ {n}\right\} }\right)}\right) + | c = from above +}} +{{eqn | r = n^\prec \cup \left\{ {n}\right\} + | c = [[Relative Complement of Relative Complement]] +}} +{{end-eqn}} +{{qed}} +\end{proof}<|endoftext|> +\section{Closed Interval of Naturally Ordered Semigroup with Successor equals Union with Successor} +Tags: Naturally Ordered Semigroup + +\begin{theorem} +Let $\left({S, \circ, \preceq}\right)$ be a [[Definition:Naturally Ordered Semigroup|naturally ordered semigroup]]. +Then: +:$\forall m, n \in \left({S, \circ, \preceq}\right): m \preceq n \implies \left[{m \,.\,.\, n \circ 1}\right] = \left[{m \,.\,.\, n}\right] \cup \left\{{n \circ 1}\right\}$ +where $\left[{m \,.\,.\, n}\right]$ is the [[Definition:Closed Interval|closed interval between $m$ and $n$]]. +\end{theorem} + +\begin{proof} +Let $m \preceq n$. Then: +{{begin-eqn}} +{{eqn | o = + | r = x \in \left[{m \,.\,.\, n \circ 1}\right] + | c = +}} +{{eqn | o = \iff + | r = m \preceq x \land x \preceq \left({n \circ 1}\right) + | c = Definition of [[Definition:Closed Interval|Closed Interval]] +}} +{{eqn | o = \iff + | r = m \preceq x \land \left({x \prec n \circ 1 \lor x = n \circ 1}\right) + | c = Definition of [[Definition:Strictly Precede|Strictly Precedes]] +}} +{{end-eqn}} +{{begin-eqn}} +{{eqn | o = + | r = x \in \left[{m \,.\,.\, n}\right] \cup \left\{ {n \circ 1}\right\} + | c = +}} +{{eqn | o = \iff + | r = m \preceq x \land \left({x \preceq n \lor x = n \circ 1}\right) + | c = Definitions of [[Definition:Closed Interval|Closed Interval]] and [[Definition:Set Union|Union]] +}} +{{eqn | o = \iff + | r = m \preceq x \land \left({x \prec n \lor x = n \lor x = n \circ 1}\right) + | c = Definition of [[Definition:Strictly Precede|Strictly Precedes]] +}} +{{eqn | o = \iff + | r = m \preceq x \land \left({x \prec n \circ 1 \lor x = n \circ 1}\right) + | c = Definition of [[Definition:Strictly Precede|Strictly Precedes]] +}} +{{end-eqn}} +Thus: +:$\left[{m \,.\,.\, n \circ 1}\right] = \left[{m \,.\,.\, n}\right] \cup \left\{{n \circ 1}\right\}$ +{{qed}} +\end{proof}<|endoftext|> +\section{Principle of Induction applied to Interval of Naturally Ordered Semigroup} +Tags: Naturally Ordered Semigroup + +\begin{theorem} +Let $\left({S, \circ, \preceq}\right)$ be a [[Definition:Naturally Ordered Semigroup|naturally ordered semigroup]]. +Let $\left[{p \,.\,.\, q}\right]$ be a [[Definition:Closed Interval|closed interval]] of $\left({S, \circ, \preceq}\right)$. +Let $T \subseteq \left[{p \,.\,.\, q}\right]$ such that the [[Definition:Minimal Element|minimal element]] of $\left[{p \,.\,.\, q}\right]$ is in $T$. +Let: +: $x \in T: x \prec q \implies x \circ 1 \in T$ +Then: +: $T = \left[{p \,.\,.\, q}\right]$ +\end{theorem} + +\begin{proof} +Let $T' = T \cup \left\{{x \in S: q \prec x}\right\}$. +Then $T'$ satisfies the conditions of the [[Principle of Mathematical Induction for Naturally Ordered Semigroup|Principle of Mathematical Induction]]. +Therefore: +:$T' = \left\{{x \in S: p \preceq x}\right\}$ +Therefore: +:$T = \left[{p \,.\,.\, q}\right]$ +{{Qed}} +\end{proof}<|endoftext|> +\section{Homomorphism of Powers} +Tags: Homomorphisms + +\begin{theorem} +Let $\struct {T_1, \odot}$ and $\struct {T_2, \oplus}$ be [[Definition:Semigroup|semigroups]]. +Let $\phi: \struct {T_1, \odot} \to \struct {T_2, \oplus}$ be a [[Definition:Semigroup Homomorphism|(semigroup) homomorphism]]. +\end{theorem}<|endoftext|> +\section{Naturally Ordered Semigroup is Unique} +Tags: Naturally Ordered Semigroup, Isomorphisms + +\begin{theorem} +Let $\struct {S, \circ, \preceq}$ and $\struct {S', \circ', \preceq'}$ be [[Definition:Naturally Ordered Semigroup|naturally ordered semigroups]]. +Let: +:$0'$ be the [[Definition:Smallest Element|smallest element]] of $S'$ +:$1'$ be the [[Definition:Smallest Element|smallest element]] of $S' \setminus \set {0'} = S'^*$. +Then the [[Definition:Mapping|mapping]] $g: S \to S'$ defined as: +:$\forall a \in S: \map g a = \circ'^a 1'$ +is an [[Definition:Isomorphism (Abstract Algebra)|isomorphism]] from $\struct {S, \circ, \preceq}$ to $\struct {S', \circ', \preceq'}$. +This [[Definition:Isomorphism (Abstract Algebra)|isomorphism]] is unique. +Thus, up to [[Definition:Isomorphism (Abstract Algebra)|isomorphism]], there is only one [[Definition:Naturally Ordered Semigroup|naturally ordered semigroup]]. +\end{theorem} + +\begin{proof} +=== Proof that Mapping is Isomorphism === +Let $T' = \Cdm g$, that is, the [[Definition:Codomain of Mapping|codomain]] of $g$. +By [[Zero is Identity in Naturally Ordered Semigroup]], $0'$ is the [[Definition:Identity Element|identity]] for $\circ'$. +Thus: +:$\map g 0 = \circ'^0 1' = 0'$ +and so $0' \in T'$ +Suppose $x' \in T'$. +Then: +:$\map g n = x'$ +and so: +{{begin-eqn}} +{{eqn | l = x' \circ' 1' + | r = \circ'^n 1' \circ' 1' + | c = +}} +{{eqn | r = \paren {\circ'^n 1'} \circ' \paren {\circ'^1 1'} + | c = +}} +{{eqn | r = \circ'^{\paren {n \circ 1} } 1' + | c = +}} +{{eqn | r = \map g {n \circ 1} + | c = +}} +{{end-eqn}} +So: +:$x' \in T' \implies x' \circ' 1' \in T'$ +Thus, by the [[Principle of Mathematical Induction for Naturally Ordered Semigroup|Principle of Mathematical Induction]] applied to $S'$: +:$T' = S'$ +So: +:$\forall a' \in S': \exists a \in S: \map g a = a'$ +and so $g$ is [[Definition:Surjection|surjective]] by definition. +From [[Index Laws for Semigroup/Sum of Indices|Index Laws for Semigroup: Sum of Indices]]: +:$\map g {a \circ b} = \map g a \circ' \map g b$ +and therefore $g$ is a [[Definition:Homomorphism (Abstract Algebra)|homomorphism]] from $\struct {S, \circ}$ to $\struct {S', \circ'}$. +Now: +{{begin-eqn}} +{{eqn | ll= \forall p \in S: + | l = \circ'^p 1' + | o = \prec' + | r = \circ'^p 1' \circ' 1' + | c = [[Sum with One is Immediate Successor in Naturally Ordered Semigroup]] +}} +{{eqn | r = \paren {\circ'^p 1'} \circ' \paren {\circ'^1 1'} + | c = +}} +{{eqn | r = \circ'^{\paren {p \circ 1} } 1' + | c = [[Index Laws for Semigroup/Sum of Indices|Index Laws for Semigroup: Sum of Indices]] +}} +{{eqn | ll= \leadsto + | l = \circ'^p 1' + | o = \prec' + | r = \circ'^{\paren {p \circ 1} } 1' + | c = +}} +{{end-eqn}} +For $p \in S$, let $S_p$ be the [[Definition:Initial Segment|initial segment]] of $S$: +:$S_p = \set {x \in S: x \prec p}$ +Let: +:$T = \set {p \in S: \forall a \in S_p: \circ'^a 1' \prec' \circ'^p 1'}$ +Now $S_0 = \O \implies 0 \in T$. +Suppose $p \in T$. +Then: +:$a \prec p \circ 1 \implies a \preceq p$ +By [[Strict Lower Closure of Sum with One]], either of these is the case: +:$(1): \quad a \prec p: p \in T \implies \circ'^a 1' \prec' \circ'^p 1' \prec' \circ'^{\paren {p \circ 1} } 1'$ +:$(2): \quad a = p: \circ'^a 1' = \circ'^p 1' \prec' \circ'^{\paren {p \circ 1} } 1'$ +In either case, we have: +:$p \in T \implies p \circ 1 \in T$, and by the[[Principle of Mathematical Induction for Naturally Ordered Semigroup|Principle of Mathematical Induction]]: +:$T = S$ +So $n \prec p \implies \circ'^n 1' \prec' \circ'^p 1'$. +Thus $g$ is a [[Definition:Surjection|surjective]] [[Definition:Monomorphism (Abstract Algebra)|monomorphism]] and therefore is an [[Definition:Isomorphism (Abstract Algebra)|isomorphism]] from $\struct {S, \circ, \preceq}$ to $\struct {S', \circ', \preceq'}$. +{{Qed}} +=== Proof that Isomorphism is Unique === +Now we need to show that the [[Definition:Isomorphism (Abstract Algebra)|isomorphism]] $g$ is unique. +Let $f: S \to S'$ be another [[Definition:Isomorphism (Abstract Algebra)|isomorphism]] different from $g$. +{{AimForCont}} $\map f 1 \ne 1'$. +We show by [[Principle of Mathematical Induction|induction]] that $1' \notin \Cdm f$. +{{finish}} +... Thus $1' \notin \Cdm f$ which is a contradiction. +Thus $\map f 1 = 1$ and it follows +{{finish}} +that $f = g$. +Thus the [[Definition:Isomorphism (Abstract Algebra)|isomorphism]] $g$ is unique. +{{Qed}} +\end{proof}<|endoftext|> +\section{Consecutive Subsets of N} +Tags: Natural Numbers + +\begin{theorem} +Let $\N_k$ denote the [[Definition:Initial Segment of Natural Numbers|initial segment of the natural numbers]] determined by $k$: +:$\N_k = \left\{{0, 1, 2, 3, \ldots, k - 1}\right\}$ +Then: +:$\N_k = \N_{k + 1} \setminus \left\{{k}\right\}$ +In particular: +:$\N_{k - 1} = \N_k \setminus \left\{{k - 1}\right\}$ +\end{theorem} + +\begin{proof} +The result follows as a direct application of [[Strict Lower Closure of Sum with One]]. +{{qed}} +[[Category:Natural Numbers]] +ebnlj9h3zjmkiqhic6nj6grj7lm6oxj +\end{proof}<|endoftext|> +\section{Well-Ordering Principle} +Tags: Number Theory, Natural Numbers, Named Theorems, Well-Orderings, Well-Ordering Principle, Ordering on Natural Numbers + +\begin{theorem} +Every [[Definition:Non-Empty Set|non-empty]] [[Definition:Subset|subset]] of $\N$ has a [[Definition:Smallest Element|smallest (or '''first''') element]]. +This is called the '''well-ordering principle'''. +The '''well-ordering principle''' also holds for $\N_{\ne 0}$. +\end{theorem} + +\begin{proof} +Consider the [[Definition:Natural Numbers|natural numbers]] $\N$ defined as the [[Definition:Naturally Ordered Semigroup|naturally ordered semigroup]] $\struct {S, \circ, \preceq}$. +From its definition, $\struct {S, \circ, \preceq}$ is [[Definition:Well-Ordered Set|well-ordered]] by $\preceq$. +The result follows. +As $\N_{\ne 0} = \N \setminus \set 0$, by [[Set Difference is Subset]] $\N_{\ne 0} \subseteq \N$. +As $\N$ is [[Definition:Well-Ordered Set|well-ordered]], by definition, every subset of $\N$ has a [[Definition:Smallest Element|smallest element]]. +{{qed}} +\end{proof} + +\begin{proof} +Let $S$ be a [[Definition:Non-Empty Set|non-empty]] [[Definition:Subset|subset]] of the [[Definition:Natural Numbers|set of natural numbers]] $\N$. +We take as [[Definition:Axiom|axiomatic]] that $\N$ is itself a [[Definition:Subset|subset]] of the [[Definition:Real Number|set of real numbers]] $\R$. +Thus $S \subseteq \R$. +By definition: +:$\forall n \in \N: n \ge 0$ +and so: +:$\forall n \in S: n \ge 0$ +Hence $0$ is a [[Definition:Lower Bound of Subset of Real Numbers|lower bound]] of $S$. +This establishes the fact that $S$ is [[Definition:Bounded Below Subset of Real Numbers|bounded below]]. +By the [[Continuum Property]], we have that $S$ admits an [[Definition:Infimum of Subset of Real Numbers|infimum]]. +Hence let $b = \inf S$. +Because $b$ is the [[Definition:Infimum of Subset of Real Numbers|infimum]] of $S$, it follows that $b + 1$ is not a [[Definition:Lower Bound of Subset of Real Numbers|lower bound]] of $S$. +So, for some $n \in S$: +:$n < b + 1$ +{{AimForCont}} $n$ is not the [[Definition:Smallest Element|smallest element]] of $S$. +Then there exists $m \in S$ such that: +:$b \le m < n < b + 1$ +from which it follows that: +:$0 < n - m < 1$ +But there exist no [[Definition:Natural Number|natural numbers]] $k$ such that $0 < k < 1$. +Hence $n = b$ is the [[Definition:Smallest Element|smallest element]] of $S$. +{{qed}} +\end{proof}<|endoftext|> +\section{Equality of Natural Numbers} +Tags: Natural Numbers + +\begin{theorem} +Let $m, n \in \N$. +Then: +: $\N_m \sim \N_n \iff m = n$ +where $\sim$ denotes [[Definition:Set Equivalence|set equivalence]] and $\N_n$ denotes the [[Definition:Initial Segment of Natural Numbers|set of all natural numbers less than $n$]]. +\end{theorem} + +\begin{proof} +By [[Set Equivalence is Equivalence Relation]], we have that: +: $m = n \implies \N_m \sim \N_n$ +It remains to show that: +: $m \ne n \implies \N_m \nsim \N_n$. +Since the [[Definition:Natural Number|naturals]] are [[Definition:Totally Ordered Set|totally ordered]], it will be sufficient to show that: +: $m \in \N_n \implies \N_m \nsim \N_n$ +Let $S = \left\{{n \in \N: \forall m \in \N_n: \N_m \nsim \N_n}\right\}$. +That is, $S$ is the [[Definition:Set|set]] of all the [[Definition:Natural Numbers|natural numbers]] $n$ such that $\N_m \nsim \N_n$ for all $m \in \N_n$. +We use [[Principle of Mathematical Induction|mathematical induction]] to prove that $S = \N$. +From [[Initial Segment of Natural Numbers determined by Zero is Empty]]: +:$\N_0 = \varnothing$ +Thus $0 \in S$. +This is the [[Principle of Mathematical Induction#Basis for the Induction|basis for the induction]]. +Now, assume the [[Principle of Mathematical Induction#Induction Hypothesis|induction hypothesis]] that $n \in S$. +We now complete the [[Principle of Mathematical Induction#Induction Step|induction step]], that is, to show that $n + 1 \in S$. +Let $m \in \N_{n+1}$. +If $m = 0$, then $\N_m \nsim \N_{n+1}$ because $\N_0 = \varnothing$ and $\N_{n+1} \ne \varnothing$. +[[Proof by Contradiction|Aiming for a contradiction]], suppose that $m \ge 1$ and $\N_m \sim \N_{n+1}$. +Then, by [[Set Equivalence Less One Element]], that means $\N_{m-1} \sim \N_n$. +But then $m - 1 \in \N_n$ which [[Definition:Contradiction|contradicts]] the [[Principle of Mathematical Induction#Induction Hypothesis|induction hypothesis]] that $n \in S$. +Thus $n + 1 \in S$. +The result follows from the fact that [[Set Equivalence is Equivalence Relation]], in particular the [[Definition:Symmetric Relation|symmetry]] clause. +{{qed}} +\end{proof}<|endoftext|> +\section{Principle of Counting} +Tags: Natural Numbers, Named Theorems + +\begin{theorem} +Let $T$ be a [[Definition:Set|set]] such that $T \sim \N_n$. +Then: +: $\forall m \in \N: n \ne m \implies T \nsim \N_m$ +\end{theorem} + +\begin{proof} +The result follows directly from [[Equality of Natural Numbers]] and the fact that [[Set Equivalence is Equivalence Relation|set equivalence is transitive]]. +{{qed}} +\end{proof}<|endoftext|> +\section{Cardinality Less One} +Tags: Set Theory + +\begin{theorem} +Let $S$ be a [[Definition:Finite Set|finite set]]. +Let: +:$\left\lvert{S}\right\rvert = n + 1$ +where $\left\lvert{S}\right\rvert$ is the [[Definition:Cardinality|cardinality]] of $S$. +Let $a \in S$. +Then: +:$\left\lvert{S \setminus \left\{ {a}\right\} }\right\rvert = n$ +where $\setminus$ denotes [[Definition:Set Difference|set difference]]. +\end{theorem} + +\begin{proof} +This follows as an immediate consequence of [[Set Equivalence Less One Element]]. +{{qed}} +\end{proof}<|endoftext|> +\section{Cardinality of Empty Set} +Tags: Empty Set, Cardinality, Finite Sets + +\begin{theorem} +:$\card S = 0 \iff S = \O$ +That is, the [[Definition:Empty Set|empty set]] is [[Definition:Finite Set|finite]], and has a [[Definition:Cardinality|cardinality]] of [[Definition:Zero (Number)|zero]]. +\end{theorem} + +\begin{proof} +[[Definition:Zero (Number)|Zero]] is defined as the [[Definition:Cardinal|cardinal]] of the [[Definition:Empty Set|empty set]]. +The result follows from [[Finite Cardinals and Ordinals are Equivalent]]. +\end{proof}<|endoftext|> +\section{Cardinality of Subset of Finite Set} +Tags: Set Theory, Subsets, Cardinality + +\begin{theorem} +Let $A$ and $B$ be [[Definition:Finite Set|finite sets]] such that $A \subseteq B$. +Let +:$\card B = n$ +where $\card {\, \cdot \,}$ denotes [[Definition:Cardinality|cardinality]]. +Then $\card A \le n$. +\end{theorem} + +\begin{proof} +Let $A \subseteq B$. +There are two cases: +$(1): \quad A \ne B$ +In this case: +:$A \subsetneqq B$ +and from [[Cardinality of Proper Subset of Finite Set]]: +:$\card A < n$ +$(2): \quad A = B$ +In this case: +:$\card A = \card B$ +and so: +:$\card A = n$ +In both cases: +:$\card A \le n$ +Hence the result. +{{qed}} +\end{proof}<|endoftext|> +\section{Cardinality of Surjection} +Tags: Surjections, Bijections, Cardinality + +\begin{theorem} +Let $S$ be a [[Definition:Set|set]]. +Let: +:$\card S = n$ +where $\card S$ denotes the [[Definition:Cardinality|cardinality]] of $S$. +Let $f: S \to T$ be a [[Definition:Surjection|surjection]]. +Then $\card T \le n$. +The equality: +:$\card T = n$ +occurs {{iff}} $f$ is a [[Definition:Bijection|bijection]]. +\end{theorem} + +\begin{proof} +We have that $\card S = n$. +Then by definition of [[Definition:Cardinality|cardinality]]: +:there is a [[Definition:Surjection|surjection]] from $S$ to $T$ +{{iff}}: +:there is a [[Definition:Surjection|surjection]] from $\N_n$ to $T$. +So we need consider the case only when $S = \N_n$. +By definition of [[Definition:Surjection|surjection]]: +:$\forall x \in T: f^{-1} \sqbrk {\set x} \ne \O$ +where $f^{-1} \sqbrk {\set x}$ denotes the [[Definition:Preimage of Subset under Mapping|preimage of $\set x$ under $f$]]. +From the [[Well-Ordering Principle]], $\N$ is [[Definition:Well-Ordered Set|well-ordered]]. +Therefore by [[Subset of Well-Ordered Set is Well-Ordered]], $S = \N_n$ is [[Definition:Well-Ordered Set|well-ordered]]. +We have that $f^{-1} \sqbrk {\set x} \subseteq S$. +Therefore $f^{-1} \sqbrk {\set x}$ is also [[Definition:Well-Ordered Set|well-ordered]]. +Therefore, by definition of [[Definition:Well-Ordered Set|well-ordered set]], $f^{-1} \sqbrk {\set x}$ has a [[Definition:Smallest Element|smallest element]]. +Let $g: T \to S$ be the [[Definition:Mapping|mapping]] defined as: +:$\forall x \in T: g \paren x =$ the [[Definition:Smallest Element|smallest element]] of $f^{-1} \sqbrk {\set x}$ +Let $x \in T$. +Then: +:$g \paren x \in f^{-1} \sqbrk {\set x}$ +so: +:$f \paren {g \paren x} = x$ +Thus: +:$f \circ g = I_T$ +and by [[Identity Mapping is Bijection]]: +:$I_T: T \to g \sqbrk T$ is a [[Definition:Bijection|bijection]]. +By [[Cardinality of Subset of Finite Set]]: +:$\card {g \sqbrk T} \le n$ +Let $\card {g \sqbrk T} = m$. +By [[Set Equivalence is Equivalence Relation]]: +:$\card T = m$ +[[Definition:Element|elements]]. +Suppose $m = n$. +Then by [[Cardinality of Subset of Finite Set]]: +:$g \sqbrk T = S$ +so $g: T \to S$ is a [[Definition:Bijection|bijection]]. +Therefore: +{{begin-eqn}} +{{eqn | l = f + | r = f \circ I_S + | c = +}} +{{eqn | r = f \circ g \circ g^{-1} + | c = +}} +{{eqn | r = I_S \circ g^{-1} + | c = +}} +{{eqn | r = g^{-1} + | c = +}} +{{end-eqn}} +So $f: S \to T$ is a [[Definition:Bijection|bijection]]. +{{Qed}} +\end{proof}<|endoftext|> +\section{Equivalence of Mappings between Sets of Same Cardinality} +Tags: Injections, Surjections, Bijections + +\begin{theorem} +Let $S$ and $T$ be [[Definition:Finite|finite]] sets such that $\card S = \card T$. +Let $f: S \to T$ be a [[Definition:Mapping|mapping]]. +Then the following statements are equivalent: +: $(1): \quad f$ is [[Definition:Bijection|bijective]] +: $(2): \quad f$ is [[Definition:Injection|injective]] +: $(3): \quad f$ is [[Definition:Surjection|surjective]]. +\end{theorem} + +\begin{proof} +$(2)$ implies $(3)$: +Let $f$ be an [[Definition:Injection|injection]]. +Then by [[Cardinality of Image of Injection]]: +:$\card S = \card {f \sqbrk S}$ +where $f \sqbrk S$ denotes the [[Definition:Image of Subset under Mapping|image]] of $S$ under $f$. +Therefore the [[Definition:Subset|subset]] $f \sqbrk S$ of $T$ has the same number of [[Definition:Element|elements]] as $T$. +Therefore: +: $f \sqbrk S = T$ +and so $f$ is a [[Definition:Surjection|surjection]]. +{{qed|lemma}} +$(3)$ implies $(1)$: +Let $f$ be [[Definition:Surjection|surjective]]. +We have that $\card S = \card T$. +By [[Cardinality of Surjection]], $f$ is [[Definition:Bijection|bijective]]. +{{qed|lemma}} +$(1)$ implies $(2)$: +Let $f$ be a [[Definition:Bijection|bijection]]. +By definition of [[Definition:Bijection|bijection]], $f$ is also an [[Definition:Injection|injection]]. +{{qed}} +\end{proof}<|endoftext|> +\section{Natural Numbers are Infinite} +Tags: Natural Numbers, Infinite Sets + +\begin{theorem} +The [[Definition:Set|set]] $\N$ of [[Definition:Natural Numbers|natural numbers]] is [[Definition:Infinite Set|infinite]]. +\end{theorem} + +\begin{proof} +Let the [[Definition:Mapping|mapping]] $s: \N \to \N$ be defined as: +:$\forall n \in \N: \map s n = n + 1$ +$s$ is clearly an [[Definition:Injection|injection]]. +{{AimForCont}} $\N$ were [[Definition:Finite Set|finite]]. +By [[Equivalence of Mappings between Sets of Same Cardinality]] it follows that $s$ is a [[Definition:Surjection|surjection]]. +But: +:$\forall n \in \N: \map s n \ge 0 + 1 > 0$ +So: +:$0 \notin s \sqbrk \N$ +and $s$ is not a [[Definition:Surjection|surjection]]. +From this [[Definition:Contradiction|contradiction]] it is seen that $\N$ cannot be [[Definition:Finite Set|finite]]. +So, by definition, $\N$ is [[Definition:Infinite Set|infinite]]. +{{Qed}} +\end{proof}<|endoftext|> +\section{Finite Non-Empty Subset of Totally Ordered Set has Smallest and Greatest Elements} +Tags: Total Orderings, Finite Non-Empty Subset of Totally Ordered Set has Smallest and Greatest Elements + +\begin{theorem} +Let $\left({S, \preceq}\right)$ be a [[Definition:Totally Ordered Set|totally ordered set]]. +Then every [[Definition:Finite Set|finite]] $T$ such that $\varnothing \subset T \subseteq S$ has both a [[Definition:Smallest Element|smallest]] and a [[Definition:Greatest Element|greatest]] element. +\end{theorem} + +\begin{proof} +Let $A \subseteq \N_{>0}$ such that every $B \subseteq S$ such that $\left|{B}\right| = n$ has a [[Definition:Greatest Element|greatest]] and a [[Definition:Smallest Element|smallest]] element. +Any $B \subseteq S$ such that $\left|{B}\right| = 1$ has $1$ element, $b \in B$ say. +Then $b$ is both the [[Definition:Greatest Element|greatest]] and [[Definition:Smallest Element|smallest]] element of $B$. +So $1 \in A$. +Let $n \in A$. +Let $B \subseteq S$ such that $\left|{B}\right| = n + 1$. +Then $\exists b \in B$, and $\left|B \setminus \left\{{b}\right\}\right| = n$ elements by [[Cardinality Less One]]. +So, by the [[Principle of Mathematical Induction#Induction Hypothesis|induction hypothesis]], $B \setminus \left\{{b}\right\}$ has a [[Definition:Greatest Element|greatest]] element $c$ and a [[Definition:Smallest Element|smallest]] element $a$, as $n \in A$. +Note that $b \ne c$ as $c \in B \setminus \left\{{b}\right\}$ but $b \notin B \setminus \left\{{b}\right\}$. +So as $\left({S, \preceq}\right)$ is totally ordered, either $b \prec c$ or $c \prec b$ as $b \ne c$. +If $b \prec c$ then $c$ is the [[Definition:Greatest Element|greatest]] element of $B$, otherwise it's $b$. +Similarly, either $b \prec a$ or $a \prec b$, and thus either $a$ or $b$ is the [[Definition:Smallest Element|smallest]] element of $B$. +Either way, $B$ has both a [[Definition:Greatest Element|greatest]] and a [[Definition:Smallest Element|smallest]] element, and therefore $n + 1 \in A$. +Therefore, by the [[Principle of Mathematical Induction]], $A = \N_{>0}$ and the proof is complete. +{{qed}} +\end{proof} + +\begin{proof} +The result follows from: +: [[Finite Non-Empty Subset of Ordered Set has Maximal and Minimal Elements]] +: [[Minimal Element of Chain is Smallest Element]] +: [[Maximal Element of Chain is Greatest Element]] +{{qed}} +\end{proof}<|endoftext|> +\section{Unique Isomorphism between Finite Totally Ordered Sets} +Tags: Total Orderings, Order Isomorphisms + +\begin{theorem} +Let $S$ and $T$ be [[Definition:Finite Set|finite sets]] such that: +:$\card S = \card T$ +Let $\struct {S, \preceq}$ and $\struct {T, \preccurlyeq}$ be [[Definition:Totally Ordered Set|totally ordered sets]]. +Then there is exactly one [[Definition:Order Isomorphism|order isomorphism]] from $\struct {S, \preceq}$ to $\struct {T, \preccurlyeq}$. +\end{theorem} + +\begin{proof} +It is sufficient to consider the case where $\struct {T, \preccurlyeq}$ is $\struct {\N_n, \le}$ for some $n \in \N$. +Let $A$ be the set of all $n \in \N$ such that if: +:$(1): \quad S$ is any set such that $\card S = n$, and +:$(2): \quad \preceq$ is any [[Definition:Total Ordering|total ordering]] on $S$ +then there is exactly one [[Definition:Order Isomorphism|isomorphism]] from $\struct {S, \preceq}$ to $\struct {\N_n, \le}$. +$\O \in A$ from [[Empty Mapping is Mapping]] and [[Equivalence of Mappings between Sets of Same Cardinality]]. +Now let $n \in A$. +Let $\struct {S, \preceq}$ be a [[Definition:Totally Ordered Set|totally ordered set]] with $n + 1$ elements. +By [[Finite Non-Empty Subset of Totally Ordered Set has Smallest and Greatest Elements]], $S$ has a [[Definition:Greatest Element|greatest element]] $b$. +Then $\card {S \setminus \set b} = n$ by [[Cardinality Less One]]. +The [[Definition:Total Ordering|total ordering]] on $S \setminus \set b$ is the one induced from that on $S$. +So as $n \in A$, there exists a unique [[Definition:Order Isomorphism|order isomorphism]] $f: S \setminus \set b \to \struct {\N_n, \le}$. +Let us define the mapping $g: S \to \N_{n + 1}$ as follows: +:$\forall x \in S: \map g x = \begin{cases} +\map f x: & x \in S \setminus \set b \\ +n: & x = b +\end{cases}$ +This is the desired [[Definition:Order Isomorphism|order isomorphism]] from $\struct {S, \preceq}$ to $\struct {\N_{n + 1}, \le}$. +Now let $h: \struct {S, \preceq}$ to $\struct {\N_{n + 1}, \le}$ be an [[Definition:Order Isomorphism|order isomorphism]]. +Then $\map h b = n$ (it has to be). +So the [[Definition:Restriction of Mapping|restriction]] of $h$ to $S \setminus \set b$ is an [[Definition:Order Isomorphism|isomorphism]] from $S \setminus \set b$ to $\struct {\N_n, \le}$, and hence $h = f$ (as $n \in A$, any such [[Definition:Order Isomorphism|order isomorphism]] is unique). +Thus: +:$\forall x \in S \setminus \set b: \map h x = \map f x = \map g x$ +and: +:$\map h b = n = \map g b$ +thus $h = g$. +Therefore $n + 1 \in A$. +The result follows from the [[Principle of Mathematical Induction]]. +{{qed}} +\end{proof}<|endoftext|> +\section{Isomorphism to Closed Interval} +Tags: Set Theory, Order Theory, Mapping Theory + +\begin{theorem} +Let $m, n \in \N$ such that $m < n$. +Then: +: $\left|{\left[{m + 1 \,.\,.\, n}\right]}\right| = n - m$ +Let $h: \N_{n - m} \to \left[{m + 1 \,.\,.\, n}\right]$ be the [[Definition:Mapping|mapping]] defined as: +:$\forall x \in \N_{n - m}: h \left({x}\right) = x + m + 1$ +Let the [[Definition:Ordering|orderings]] on $\left[{m + 1 \,.\,.\, n}\right]$ and $\N_{n - m}$ be those [[Definition:Restriction of Relation|induced]] by the [[Definition:Ordering|ordering]] of $\N$. +Then $h$ a unique [[Definition:Order Isomorphism|order isomorphism]]. +\end{theorem} + +\begin{proof} +From [[Unique Isomorphism between Finite Totally Ordered Sets]], it suffices to show that $h$ is an [[Definition:Order Isomorphism|order isomorphism]]. +To this end, remark that, for all $x, y \in \N_{n - m}$: +{{begin-eqn}} +{{eqn | l = h \left({x}\right) + | r = h \left({y}\right) +}} +{{eqn | ll= \iff + | l = x + m + 1 + | r = y + m + 1 +}} +{{eqn | ll = \iff + | l = x + | r = y + | c = [[Natural Number Addition is Cancellable]] +}} +{{end-eqn}} +proving $h$ is an [[Definition:Injection|injection]], and so a [[Definition:Bijection|bijection]], from [[Equivalence of Mappings between Sets of Same Cardinality]]. +By [[Ordering on Natural Numbers is Compatible with Addition]] and [[Natural Number Addition is Cancellable for Ordering]], it follows that: +:$x \le y \iff h \left({x}\right) \le h \left({y}\right)$ +so $h$ is an [[Definition:Order Isomorphism|order isomorphism]]. +{{qed}} +\end{proof}<|endoftext|> +\section{Regular Representation wrt Cancellable Element on Finite Semigroup is Bijection} +Tags: Semigroups, Regular Representations, Cancellability + +\begin{theorem} +Let $\left({S, \circ}\right)$ be a [[Definition:Finite|finite]] [[Definition:Semigroup|semigroup]]. +Let $a \in S$ be [[Definition:Cancellable Element|cancellable]]. +Then: +:the [[Definition:Left Regular Representation|left regular representation]] $\lambda_a$ +and: +:the [[Definition:Right Regular Representation|right regular representation]] $\rho_a$ +of $\left({S, \circ}\right)$ with respect to $a$ are both [[Definition:Bijection|bijections]]. +\end{theorem} + +\begin{proof} +By [[Cancellable iff Regular Representations Injective]], $\lambda_a$ and $\rho_a$ are [[Definition:Injection|injections]]. +We have that $S$ is [[Definition:Finite Set|finite]]. +From [[Injection from Finite Set to Itself is Surjection]], both $\lambda_a$ and $\rho_a$ are [[Definition:Surjection|surjections]]. +Thus $\lambda_a$ and $\rho_a$ are [[Definition:Injection|injective]] and [[Definition:Surjection|surjective]], and therefore [[Definition:Bijection|bijections]]. +{{qed}} +[[Category:Semigroups]] +[[Category:Regular Representations]] +[[Category:Cancellability]] +k62vgwldwv4np50anvdp0y1qep88fep +\end{proof}<|endoftext|> +\section{Power Set of Natural Numbers is not Countable} +Tags: Power Set, Natural Numbers, Uncountable Sets + +\begin{theorem} +The [[Definition:Power Set|power set]] $\powerset \N$ of the [[Definition:Natural Numbers|natural numbers]] $\N$ is not [[Definition:Countable Set|countable]]. +\end{theorem} + +\begin{proof} +There is [[No Bijection from Set to its Power Set|no bijection from a set to its power set]]. +From [[Injection from Set to Power Set]], we have that there exists an [[Definition:Injection|injection]] $f: \N \to \powerset \N$. +From the [[Cantor-Bernstein-Schröder Theorem]], there can be no [[Definition:Injection|injection]] $g: \powerset \N \to \N$. +So, by definition, $\powerset \N$ is not [[Definition:Countable Set|countable]]. +{{qed}} +\end{proof}<|endoftext|> +\section{Subset of Countably Infinite Set is Countable} +Tags: Countable Sets, Subsets + +\begin{theorem} +Every [[Definition:Subset|subset]] of a [[Definition:Countably Infinite Set|countably infinite set]] is [[Definition:Countable Set|countable]]. +\end{theorem} + +\begin{proof} +Let $S = \set {a_0, a_1, a_2, \ldots}$ be [[Definition:Countably Infinite Set|countably infinite]]. +Let $T \subseteq S = \set {a_{n_0}, a_{n_1}, a_{n_2}, \ldots}$, where $a_{n_0}, a_{n_1}, a_{n_2}, \ldots$ are the [[Definition:Element|elements]] of $S$ also in $T$. +If the set of numbers $\set {n_0, n_1, n_2, \ldots}$ has a largest number, then $T$ is [[Definition:Finite Set|finite]]. +Otherwise, consider the [[Definition:Bijection|bijection]] $i \leftrightarrow n_i$. +This leads to the [[Definition:Bijection|bijection]] $i \leftrightarrow a_{n_i}$ +This latter [[Definition:Bijection|bijection]] is the required [[Definition:Bijection|one-to-one correspondence]] between the [[Definition:Element|elements]] of $T$ and those of $\N$, showing that $T$ is indeed [[Definition:Countable Set|countable]]. +{{finish|formal justification}} +\end{proof}<|endoftext|> +\section{Infinite Set has Countably Infinite Subset} +Tags: Countable Sets, Infinite Sets, Subsets, Infinite Set has Countably Infinite Subset + +\begin{theorem} +Every [[Definition:Infinite Set|infinite set]] has a [[Definition:Countably Infinite Set|countably infinite]] [[Definition:Subset|subset]]. +\end{theorem} + +\begin{proof} +Let $S$ be an [[Definition:Infinite Set|infinite set]]. +We use [[Between Two Sets Exists Injection or Surjection]]. +Suppose that there exists an [[Definition:Injection|injection]] $\psi: \N \to S$. +Let $T$ be the [[Definition:Image of Mapping|image]] of $\psi$. +From [[Injection to Image is Bijection]], it follows that $\psi^{-1}: T \to \N$ is a [[Definition:Bijection|bijection]]. +Hence, $T$ is a [[Definition:Countably Infinite|countably infinite]] [[Definition:Subset|subset]] of $S$. +Now, suppose that that there exists a [[Definition:Surjection|surjection]] $\phi: \N \to S$. +From [[Surjection from Natural Numbers iff Countable]], it follows that $S$ is [[Definition:Countable Set|countably infinite]]. +So, from [[Set is Subset of Itself]], we have that $S$ is a [[Definition:Countable|countably infinite]] [[Definition:Subset|subset]] of $S$. +{{qed}} +\end{proof} + +\begin{proof} +Let $S$ be an [[Definition:Infinite Set|infinite set]]. +First an [[Definition:Injection|injection]] $f: \N \to S$ is constructed. +Let $g$ be a [[Definition:Choice Function|choice function]] on $\powerset S \setminus \set \O$. +Then define $f: \N \to S$ as follows: +:$\forall n \in \N: f \left({n}\right) = \begin{cases} +\map g S & : n = 0 \\ +\map g {S \setminus \set {\map f 0, \ldots, \map f {n - 1} } } & : n > 0 +\end{cases}$ +Since $S$ is [[Definition:Infinite Set|infinite]], $S \setminus \set {\map f 0, \ldots, \map f {n - 1} }$ is [[Definition:Non-Empty Set|non-empty]] for each $n \in \N$. +Therefore $f \sqbrk \N$ is [[Definition:Infinite Set|infinite]]. +To show that $f$ is injective, let $m, n \in \N$, say $m < n$. +Then: +:$\map f m \in \set {\map f 0, \ldots, \map f {n - 1} }$ +but: +:$\map f n \in S \setminus \set {\map f 0, \ldots, \map f {n - 1} }$ +Hence $\map f m \ne \map f n$. +Since $f$ is [[Definition:Injection|injective]], it follows from [[Injection to Image is Bijection]] that $f$ is a [[Definition:Bijection|bijection]] from $\N$ to $f \sqbrk \N$. +Thus $f \sqbrk \N$ is a [[Definition:Countable|countable]] subset of $S$. +{{Qed}} +\end{proof} + +\begin{proof} +Let $S$ be an [[Definition:Infinite Set|infinite set]]. +First an [[Definition:Injection|injection]] $f: \N \to S$ is constructed. +Let $f$ be a [[Definition:Choice Function|choice function]] on $\powerset S \setminus \set \O$. +That is: +:$\forall A \in \powerset S \setminus \set \O: \map f A \in A$ +This is justified only if the [[Axiom:Axiom of Choice|Axiom of Choice]] is accepted. +Let $\CC$ be the [[Definition:Set of Sets|set]] of all [[Definition:Finite Set|finite subsets]] of $S$. +Let $A \in \CC$. +Since $S$ is [[Definition:Infinite Set|infinite]] it follows that $S \setminus A \ne \O$. +So $S \setminus A \in \Dom f$. +Let $g: \CC \to \CC$ be the [[Definition:Mapping|mapping]] defined as: +:$\map g A = A \cup \set {\map f {S \setminus A} }$ +That is, $\map g A$ is constructed by joining $A$ with the element that $f$ chooses from $S \setminus A$. +Consider the [[Recursion Theorem]] applied to $g$, starting with the set $\O$. +We obtain a [[Definition:Mapping|mapping]] $U: \N \to \CC$ such that: +:$\map U x = \begin{cases} +\O & : x = 0 \\ \map U n \cup \set {\map f {S \setminus \map U n} } & : x = n^+ +\end{cases}$ +where here $\N$ is considered as the [[Definition:Set|set]] of [[Definition:Element|elements]] of the [[Definition:Von Neumann Construction of Natural Numbers|von Neumann construction of natural numbers]]. +Consider the mapping $v: \N \to S$, defined as: +:$\forall n \in \N: \map v n = \map f {S \setminus \map U n}$ +We have that, by definition of $v$: +:$(1): \quad \forall n \in \N: \map v n \notin \map U n$ +:$(2): \quad \forall n \in \N: \map v n \in \map U {n^+}$ +:$(3): \quad \forall m, n \in \N: n \le m \implies \map U n \subseteq \map U m$ +Then because $\map v n \in \map U m$ but $\map v m \notin \map U m$: +:$(4): \quad \forall m, n \in \N: n < m \implies \map v n \ne \map v m$ +So $(4)$ implies that $v$ maps [[Definition:Distinct Elements|distinct elements]] of $\N$ onto [[Definition:Distinct Elements|distinct elements]] of $S$. +Thus $v: \N \to S$ is an [[Definition:Injection|injection]]. +It follows from [[Injection to Image is Bijection]] that $v$ is a [[Definition:Bijection|bijection]] from $\N$ to $\map v \N$. +Thus $\map v \N$ is the [[Definition:Countable|countable]] subset of $S$ that was required. +{{Qed}} +\end{proof} + +\begin{proof} +Let $S$ be an [[Definition:Infinite Set|infinite set]]. +For all $n \in \N$, let: +:$\mathcal F_n = \left\{{T \subseteq S : \left\vert{T}\right\vert = n}\right\}$ +where $\left\vert{T}\right\vert$ denotes the [[Definition:Cardinality|cardinality]] of $T$. +From [[Set is Infinite iff exist Subsets of all Finite Cardinalities]]: +:$\mathcal F_n$ is [[Definition:Non-Empty Set|non-empty]]. +Using the [[Axiom:Axiom of Countable Choice|axiom of countable choice]], we can obtain a [[Definition:Sequence|sequence]] $\left\langle{S_n}\right\rangle_{n \in \N}$ such that $S_n \in \mathcal F_n$ for all $n \in \N$. +Define: +:$\displaystyle T = \bigcup_{n \mathop \in \N} S_n \subseteq S$ +For all $n \in \N$, $S_n$ is a [[Definition:Subset|subset]] of $T$ whose [[Definition:Cardinality|cardinality]] is $n$. +From [[Set is Infinite iff exist Subsets of all Finite Cardinalities]]: +:$T$ is infinite. +Because the [[Countable Union of Countable Sets is Countable|countable union of finite sets is countable]], $T$ is [[Definition:Countable Set|countable]]. +{{qed}} +\end{proof}<|endoftext|> +\section{No Bijection between Finite Set and Proper Subset} +Tags: Subsets, Set Theory, No Bijection between Finite Set and Proper Subset + +\begin{theorem} +A [[Definition:Finite Set|finite set]] can not be in [[Definition:Bijection|one-to-one correspondence]] with one of its [[Definition:Proper Subset|proper subsets]]. +That is, a [[Definition:Finite Set|finite set]] is not [[Definition:Dedekind-Infinite|Dedekind-infinite]]. +\end{theorem} + +\begin{proof} +Let $S$ be a [[Definition:Finite Set|finite set]], and let $T$ be a [[Definition:Proper Subset|proper subset]] of $S$. +Let $f: T \to S$ be an [[Definition:Injection|injection]]. +By [[Cardinality of Image of Injection]] and [[Cardinality of Subset of Finite Set]]: +:$\size {\Img f} = \size T < \size S$ +Here, $\Img f$ denotes the [[Definition:Image of Mapping|image]] of $f$. +Thus $\Img f \ne S$, and so $f$ is not a [[Definition:Bijection|bijection]]. +{{qed}} +\end{proof} + +\begin{proof} +Proof by [[Principle of Mathematical Induction|induction]]: +For all $n \in \N_{>0}$, let $P \left({n}\right)$ be the [[Definition:Proposition|proposition]]: +:The set $\N_n = \left\{{0, 1, \ldots, n-1}\right\}$ of [[Definition:Initial Segment of Natural Numbers|natural numbers less than $n$]] is [[Definition:Set Equivalence|equivalent]] to none of its [[Definition:Proper Subset|proper subsets]]. +Here we use the definition of [[Definition:Set Equivalence|set equivalence]] to mean that $S$ is [[Definition:Set Equivalence|equivalent]] to $T$ if there exists a [[Definition:Bijection|bijection]] between them. +=== Basis for the Induction === +$P \left({1}\right)$ is the case: +:The set $\N_1 = \left\{{0}\right\}$ is [[Definition:Set Equivalence|equivalent]] to none of its [[Definition:Proper Subset|proper subsets]]. +The only [[Definition:Proper Subset|proper subset]] of $\left\{{0}\right\}$ is the [[Definition:Empty Set|empty set]] $\varnothing$, to which $\left\{{0}\right\}$ is trivially not [[Definition:Set Equivalence|equivalent]]. +This is our [[Principle of Mathematical Induction#Basis for the Induction|basis for the induction]]. +=== Induction Hypothesis === +Now we need to show that, if $P \left({k}\right)$ is true, where $k \ge 1$, then it logically follows that $P \left({k+1}\right)$ is true. +So this is our [[Principle of Mathematical Induction#Induction Hypothesis|induction hypothesis]]: +:The set $\N_k = \left\{{0, 1, \ldots, k-1}\right\}$ is [[Definition:Set Equivalence|equivalent]] to none of its [[Definition:Proper Subset|proper subsets]]. +Then we need to show: +:The set $\N_{k+1} = \left\{{0, 1, \ldots, k}\right\}$ is [[Definition:Set Equivalence|equivalent]] to none of its [[Definition:Proper Subset|proper subsets]]. +=== Induction Step === +Suppose that $P \left({k+1}\right)$ is not true. +Let $\phi: N_{k+1} \to S$ be a [[Definition:Bijection|bijection]] where $S \subsetneq N_{k+1}$. +We aim to demonstrate that the existence of such a $\phi$ contradicts the [[No Bijection between Finite Set and Proper Subset/Proof 2#Induction Hypothesis|induction hypothesis]]. +We consider two cases: +;Case 1. +Let $S \subseteq N_k$. +Then we define the [[Definition:Composition of Mappings|composite mapping]] $\phi \circ \phi$ defined as: +:$\forall m \in \N_{k+1}: \phi \circ \phi \left({m}\right) = \phi \left({\phi \left({m}\right)}\right)$ +From [[Composite of Bijections is Bijection]], $\phi \circ \phi$ is a [[Definition:Bijection|bijection]]. +Now consider $T = \phi \circ \phi \left({\N_{k+1}}\right)$, the [[Definition:Image of Mapping|image set]] of $\N_{k+1}$ under $\phi \circ \phi$. +Let $m, t \in \N_{k+1}$ such that: +:$\phi \circ \phi \left({m}\right) = \phi \left({t}\right)$ +Since $\phi$ is an [[Definition:Injection|injection]] it follows that: +:$\phi \left({m}\right) = t$ +As $S$ is a [[Definition:Proper Subset|proper subset]] of $\N_{k+1}$ it follows from [[Set Difference with Proper Subset]] that $\N_{k+1} \setminus S \ne \varnothing$. +Let $x \in \N_{k+1} \setminus S$. +Suppose $\phi \left({x}\right) \in T$. +Then that would mean that $\exists m \in \N_{k+1}$ such that: +: $\phi \circ \phi \left({m}\right) = \phi \left({x}\right)$ +which from above it would follow that: +:$\phi \left({m}\right) = x$ +and so $x \in S$. +But this can not happen as $S \cap \left({\N_{k+1} \setminus S}\right) = \varnothing$ from [[Set Difference Intersection with Second Set is Empty Set]]. +So it must be the case that $\phi \left({x}\right) \notin T$. +Thus it can be concluded that $\phi \circ \phi$ is a [[Definition:Mapping|mapping]] from $\N_{k+1}$ onto a [[Definition:Proper Subset|proper subset]] $T$ of $S$. +Now, consider the mapping $\psi: \N_k \to U$ where $U$ is a [[Definition:Proper Subset|proper subset]] of $\N_k$. +We define $\psi$ as follows: +:$\forall a \in \N_k: \psi \left({a}\right) = \begin{cases} +a & : a \notin S = \phi \left({N_{k+1}}\right) \\ +\phi \left({a}\right) & : a \in S \end{cases}$ +The next step is to show that $\psi$ is an [[Definition:Injection|injection]]. +Consider $a, b \in \N_k$ such that $\psi \left({a}\right) = \psi \left({b}\right)$. +Either $\psi \left({a}\right) = \psi \left({b}\right) \in \N_k \setminus S$, in which case: +:$\psi \left({a}\right) = a = \psi \left({b}\right) = b$ +or $\psi \left({a}\right) = \psi \left({b}\right) \in S$, in which case: +:$\psi \left({a}\right) = \phi \left({a}\right) = \psi \left({b}\right) = \phi \left({b}\right)$ +As $\phi$ is a [[Definition:Injection|injection]] it follows that $a = b$. +So $\psi \left({a}\right) = \psi \left({b}\right) \implies a = b$ and so by definition $\psi$ is an [[Definition:Injection|injection]]. +Now consider $\psi \left({\N_k}\right)$. +We have that $\psi \left({\N_k}\right) = \N_k \setminus \left({S \setminus T}\right)$ so that $\psi$ is an [[Definition:Injection|injection]] from $\N_k$ to a [[Definition:Proper Subset|proper subset]] of itself. +This contradicts the [[No Bijection between Finite Set and Proper Subset/Proof 2#Induction Hypothesis|induction hypothesis]]. +;Case 2. +Let $S \not \subseteq N_k$. +This means that $k \in S$. +There exists $x \in \N_{k+1}$ such that $\phi \left({x}\right) = k$, as $\phi: \N_{k+1} \to S$ is a [[Definition:Surjection|surjection]]. +There also exists $y \in \N_{k+1} \setminus S$ as $\N_{k+1} \setminus S \ne \varnothing$ from [[Set Difference with Proper Subset]]. +Let $\phi': \N_{k+1} \to S$ be defined as: +:$\phi' \left({a}\right) = \begin{cases} +\phi \left({a}\right) : & a \ne x \\ +y : & a = x \end{cases}$ +As $\phi$ is a [[Definition:Bijection|bijection]] it follows that $\phi': \N_{k+1} \to S\,'$ is also a [[Definition:Bijection|bijection]], where: +:$S\,' = \left({S \setminus \left\{{k}\right\}}\right) \cup \left\{{y}\right\}$ +Proceeding as Case 1, we prove the same contradiction in the [[No Bijection between Finite Set and Proper Subset/Proof 2#Induction Hypothesis|induction hypothesis]]. +Thus by [[Proof by Contradiction]], the assumption that $\phi: N_{k+1} \to S$ can be a [[Definition:Bijection|bijection]] must be false. +So the truth of $P \left({m}\right)$ for all $m \le k$ implies the proof of $P \left({k+1}\right)$. +The result follows by the [[Second Principle of Mathematical Induction]]. +Therefore: +:For all $n \in \N$, the set $\N_n = \left\{{0, 1, \ldots, n-1}\right\}$ of [[Definition:Initial Segment of Natural Numbers|natural numbers less than $n$]] is [[Definition:Set Equivalence|equivalent]] to none of its [[Definition:Proper Subset|proper subsets]]. +{{qed}} +\end{proof}<|endoftext|> +\section{Infinite Set is Equivalent to Proper Subset} +Tags: Infinite Set is Equivalent to Proper Subset, Set Equivalence, Infinite Sets, Subsets + +\begin{theorem} +A [[Definition:Set|set]] is [[Definition:Infinite Set|infinite]] {{iff}} it is [[Definition:Set Equivalence|equivalent]] to one of its [[Definition:Proper Subset|proper subsets]]. +\end{theorem} + +\begin{proof} +Let $T$ be an [[Definition:Infinite Set|infinite set]]. +By [[Infinite Set has Countably Infinite Subset]], it is possible to construct a [[Definition:Countable|countably infinite]] [[Definition:Subset|subset]] of $T$. +Let $S = \set {a_1, a_2, a_3, \ldots}$ be such a [[Definition:Countable|countably infinite]] [[Definition:Subset|subset]] of $T$. +Create a [[Definition:Partition (Set Theory)|Partition]] of $S$ into: +:$S_1 = \set {a_1, a_3, a_5, \ldots}, S_2 = \set {a_2, a_4, a_6, \ldots}$ +Let a [[Definition:Bijection|bijection]] be established between $S$ and $S_1$, by letting $a_n \leftrightarrow a_{2 n - 1}$. +This is extended to a [[Definition:Bijection|bijection]] between $S \cup \paren {T \setminus S} = T$ and $S_1 \cup \paren {T \setminus S} = T \setminus S_2$ by assigning each [[Definition:Element|element]] in $T \setminus S$ to itself. +So a [[Definition:Bijection|bijection]] has been demonstrated between $T$ and one of its [[Definition:Proper Subset|proper subsets]] $T \setminus S_2$. +That is, if $T$ is [[Definition:Infinite Set|infinite]], it is [[Definition:Set Equivalence|equivalent]] to one of its [[Definition:Proper Subset|proper subsets]]. +Now, let $T_0 \subsetneq T$ be a proper subset of $T$, and $f: T \to T_0$ be a [[Definition:Bijection|bijection]]. +It follows from [[No Bijection between Finite Set and Proper Subset]] that $T$ must be [[Definition:Infinite Set|infinite]]. +{{qed}} +\end{proof} + +\begin{proof} +Let $S$ be a [[Definition:Set|set]]. +Suppose $S$ is [[Definition:Finite Set|finite]]. +From [[No Bijection between Finite Set and Proper Subset]] we have that $S$ can not be [[Definition:Set Equivalence|equivalent]] to one of its [[Definition:Proper Subset|proper subsets]]. +Suppose $S$ is [[Definition:Infinite Set|infinite]]. +From [[Infinite Set has Countably Infinite Subset]], we can construct $v: \N \to S$ such that $v$ is an [[Definition:Injection|injection]]. +We now construct the [[Definition:Mapping|mapping]] $h: S \to S$ as follows. +:$\map h x = \begin {cases} +\map v {n + 1} & : \exists n \in \N: x = \map v n \\ +x & : x \notin \Img v +\end {cases}$ +It is clear that $h$ is an [[Definition:Injection|injection]]. +But we have that $\map v 0 \notin \Img h$ and so $\Img h \subsetneq S$. +The result follows from [[Injection to Image is Bijection]]. +{{qed}} +\end{proof} + +\begin{proof} +Let $X$ be a [[Definition:Set|set]] which has a [[Definition:Proper Subset|proper subset]] $Y$ such that: +:$\card X = \card Y$ +where $\card X$ denotes the [[Definition:Cardinality|cardinality]] of $X$. +Then: +:$\exists \alpha \in \complement_X \paren Y$ +and +:$Y \subsetneqq Y \cup \set \alpha \subseteq X$ +The [[Definition:Inclusion Mapping|inclusion mappings]]: +:$i_Y: Y \to X: \forall y \in Y: i \paren y = y$ +:$i_{Y \cup \set \alpha}: Y \cup \set \alpha \to X: \forall y \in Y: i \paren y = y$ +give: +:$\card X = \card Y \le \card Y + \mathbf 1 \le \card X $ +from which: +:$\card X = \card Y + \mathbf 1 = \card X + \mathbf 1$ +So by definition $X$ is [[Definition:Infinite Set|infinite]]. +{{qed|lemma}} +Now suppose $X$ is [[Definition:Infinite Set|infinite]]. +That is: +:$\card X = \card X + \mathbf 1$ +Let $\alpha$ be any [[Definition:Object|object]] such that $\alpha \notin X$. +Then there is a [[Definition:Bijection|bijection]] $f: X \cup \set \alpha \to X$. +Let $f_{\restriction X}$ be the [[Definition:Restriction of Mapping|restriction]] of $f$ to $X$. +Then from [[Injection to Image is Bijection]]: +:$\image {f_{\restriction X} } = X \setminus \set {f \paren \alpha}$ +which is a [[Definition:Proper Subset|proper subset]] of $X$ which is [[Definition:Set Equivalence|equivalent]] to $X$. +{{qed}} +\end{proof}<|endoftext|> +\section{Cartesian Product of Countable Sets is Countable} +Tags: Set Theory, Cartesian Product, Countable Sets, Cartesian Product of Countable Sets is Countable + +\begin{theorem} +The [[Definition:Cartesian Product|cartesian product]] of two [[Definition:Countable Set|countable sets]] is [[Definition:Countable Set|countable]]. +\end{theorem} + +\begin{proof} +Let $S = \set {s_0, s_1, s_2, \dotsc}$ and $T = \set {t_0, t_1, t_2, \dotsc}$ be [[Definition:Countable Set|countable sets]]. +If both $S$ and $T$ are [[Definition:Finite Set|finite]], the result follows immediately. +Suppose either of $S$ or $T$ (or both) is [[Definition:Countably Infinite Set|countably infinite]]. +We can write the [[Definition:Element|elements]] of $S \times T$ in the form of an [[Definition:Infinite Set|infinite]] table: +:$\begin{array} {*{4}c} + \tuple {s_0, t_0} & \tuple {s_0, t_1} & \tuple {s_0, t_2} & \cdots \\ + \tuple {s_1, t_0} & \tuple {s_1, t_1} & \tuple {s_1, t_2} & \cdots \\ + \tuple {s_2, t_0} & \tuple {s_2, t_1} & \tuple {s_2, t_2} & \cdots \\ + \vdots & \vdots & \vdots & \ddots \\ +\end{array}$ +This table clearly contains all the [[Definition:Element|elements]] of $S \times T$. +Now we can count the [[Definition:Element|elements]] of $S \times T$ by processing the table diagonally. +First we pick $\tuple {s_0, t_0}$. +Then we pick $\tuple {s_0, t_1}, \tuple {s_1, t_0}$. Then we pick $\tuple {s_0, t_2}, \tuple {s_1, t_1}, \tuple {s_2, t_0}$. +We can see that all the [[Definition:Element|elements]] of $S \times T$ will (eventually) be listed, and there is a specific number ([[Definition:Element|element]] of $\N$) to index each of its [[Definition:Element|elements]] with. +Thus we have the required [[Definition:Bijection|one-to-one correspondence]] between $S \times T$ and $\N$, and our assertion is proved. +{{qed}} +\end{proof} + +\begin{proof} +Let $S, T$ be [[Definition:Countable Set|countable sets]]. +From the definition of [[Definition:Countable Set|countable]], there exists a [[Definition:Injection|injection]] from $S$ to $\N$, and similarly one from $T$ to $\N$. +Hence there exists an [[Definition:Injection|injection]] $g$ from $S \times T$ to $\N^2$. +Now let us investigate the [[Definition:Cardinality|cardinality]] of $\N^2$. +From the [[Fundamental Theorem of Arithmetic]], every [[Definition:Natural Number|natural number]] greater than $1$ has a unique [[Definition:Prime Decomposition|prime decomposition]]. +Thus, if a number can be written as $2^n 3^m$, it can be done thus in only one way. +So, consider the [[Definition:Function|function]] $f: \N^2 \to \N$ defined by: +:$\map f {n, m} = 2^n 3^m$. +Now suppose $\exists m, n, r, s \in \N$ such that $\map f {n, m} = \map f {r, s}$. +Then $2^n 3^m = 2^r 3^s$ so that $n = r$ and $m = s$. +Thus $f$ is an [[Definition:Injection|injection]], whence $\N^2$ is [[Definition:Countably Infinite Set|countably infinite]]. +Now we see that as $g$ and $f$ are [[Definition:Injection|injective]], it follows from [[Composite of Injections is Injection]] that $f \circ g: S \times T \to \N$ is also [[Definition:Injection|injective]]. +Hence the result. +{{qed}} +\end{proof} + +\begin{proof} +Let $S, T$ be [[Definition:Countable Set|countable sets]]. +Let $S = \set {s_1, s_2, s_3, \dotsc}$ and $T = \set {t_1, t_2, t_3, \dotsc}$ be [[Definition:Countably Infinite Enumeration|enumerations]] of $S$ and $T$ respectively. +Let $f: S \times T: \N$ be the [[Definition:Mapping|mapping]] defined as: +:$\forall \tuple {s_k, t_l} \in S \times T: \map f {s_k, t_l} = \dfrac {\paren {k + l - 1} \paren {k + l - 2} } 2 + \dfrac {l + \paren {-1}^{k + 1} } 2 k + \dfrac {1 + \paren {-1}^{k + l - 1} } 2 l$ +Then $f$ gives an [[Definition:Countably Infinite Enumeration|enumeration]] of $S \times T$. +This [[Definition:Countably Infinite Enumeration|enumeration]] can be depicted schematically as: +:$\begin {array} {} \tuple {s_1, t_1} & & \tuple {s_1, t_2} & \to & \tuple {s_1, t_3} & & \tuple {s_1, t_4} & \to & \dotsc \\ +\downarrow & \nearrow & & \swarrow & & \nearrow & \dotsc \\ +\tuple {s_2, t_1} & & \tuple {s_2, t_2} & & \tuple {s_2, t_3} & \dotsc \\ +& \swarrow & & \nearrow & \dotsc \\ +\tuple {s_3, t_1} & & \tuple {s_2, t_3} & \dotsc \\ +\downarrow & \nearrow & \dotsc \\ +\tuple {s_3, t_1} & \dotsc \\ +\vdots \\ +\end{array}$ +Hence the result. +{{qed}} +\end{proof}<|endoftext|> +\section{Composition of Sequence with Mapping} +Tags: Sequences + +\begin{theorem} +Let $\left \langle {a_j} \right \rangle_{j \in B}$ be a [[Definition:Sequence|sequence]]. +Let $\sigma: A \to B$ be a [[Definition:Mapping|mapping]], where $A \subseteq \N$. +Then $\left \langle {a_j} \right \rangle \circ \sigma$ is a [[Definition:Sequence|sequence]] whose value at each $k \in A$ is $a_{\sigma \left({k}\right)}$. +Thus $\left \langle {a_j} \right \rangle \circ \sigma$ is denoted $\left \langle {a_{\sigma \left({k}\right)}} \right \rangle_{k \in A}$. +\end{theorem} + +\begin{proof} +By definition, a [[Definition:Sequence|sequence]] is a [[Definition:Mapping|mapping]] whose [[Definition:Domain of Mapping|domain]] is a [[Definition:Subset|subset]] of $\N$. +Let the [[Definition:Range of Sequence|range]] of $\left \langle {a_j} \right \rangle_{j \in B}$ be $S$. +Thus $\left \langle {a_j} \right \rangle_{j \in B}$ can be expressed using the [[Definition:Mapping|mapping]] $f: B \to S$ as: +:$\forall j \in B: f \left({j}\right) = a_j$ +Let $k \in A$. +Then $\sigma \left({k}\right) \in B$. +By definition of [[Definition:Composition of Mappings|composition of mappings]]: +{{begin-eqn}} +{{eqn | l=f \circ \sigma \left({k}\right) + | r=f \left({\sigma \left({k}\right)}\right) + | c= +}} +{{eqn | ll=\implies + | l=\left \langle {a_j} \right \rangle_{j \in B} + | r=\left \langle {a_{\sigma \left({k}\right)} } \right \rangle_{\sigma \left({k}\right) \in B} + | c= +}} +{{eqn | r=\left \langle {a_{\sigma \left({k}\right)} } \right \rangle_{k \in A} + | c=as $\sigma \left({k}\right) \in B \implies k \in A$ +}} +{{end-eqn}} +{{qed}} +[[Category:Sequences]] +2wdusrdz15r73kktgzfr0g4a18i31vr +\end{proof}<|endoftext|> +\section{General Operation from Binary Operation} +Tags: Abstract Algebra + +\begin{theorem} +Let $\left({S, \oplus}\right)$ be a [[Definition:Magma|magma]]. +Then there a unique [[Definition:Sequence|sequence]] $\left \langle {\oplus_k} \right \rangle_{k \mathop \ge 1}$ such that: +:$(1): \quad \forall n \in \N_{>0}: \oplus_n$ is an [[Definition:Operation on Set|$n$-ary operation]] on $S$ such that: +:$(2): \quad \forall \left({a_1, \ldots, a_k}\right) \in S^k: \oplus_k \left({a_1, \ldots, a_k}\right) = \begin{cases} +a: & k = 1 \\ +\oplus_n \left({a_1, \ldots, a_n}\right) \oplus a_{n+1}: & k = n + 1 +\end{cases}$ +In particular, $\oplus_2$ is the same as the given [[Definition:Binary Operation|binary operation]] $\oplus$. +The $n$th [[Definition:Term of Sequence|term]] $\oplus_n$ of the [[Definition:Sequence|sequence]] $\left \langle {\oplus} \right \rangle$ is called the '''[[Definition:N-Ary Operation Defined by Binary Operation|$n$-ary operation defined by $\oplus$]]'''. +\end{theorem} + +\begin{proof} +Let $\Bbb S = \left\{{\odot:}\right.$ for some $n \in \N_{>0}$, $\odot$ is an [[Definition:Operation on Set|$n$-ary operation]] on $\left.{S}\right\}$. +Let $s: \Bbb S \to \Bbb S$ be the [[Definition:Mapping|mapping]] defined as follows. +Let $\odot$ be any [[Definition:Operation on Set|$n$-ary operation]] defined on $\Bbb S$. +Then $s \left({\odot}\right)$ is the $\left({n+1}\right)$-ary operation defined by: +:$\forall \left({a_1, \ldots, a_n, a_{n+1}}\right) \in S^{n+1}: s \left({\odot}\right) \left({a_1, \ldots, a_n, a_{n+1}}\right) = \odot \left({a_1, \ldots, a_n}\right) \oplus a_{n+1}$ +By the [[Principle of Recursive Definition]], there is a [[Definition:Unique|unique]] [[Definition:Sequence|sequence]] $\left \langle {\oplus_k} \right \rangle_{k \mathop \ge 1}$ such that: +: $\oplus_1$ is the [[Definition:Unary Operation|unary operation]] defined as $\oplus_1 \left({a}\right) = a$ +and: +: $\oplus_{n+1} = s \left({\oplus_n}\right)$ for each $n \in \N_{>0}$. +Let $A$ be the [[Definition:Set|set]] defined as the set of all $n$ such that: +:$\oplus_n$ is an [[Definition:Operation on Set|$n$-ary operation]] on $S$ +and: +:by the definition of $s$, $(2)$ holds for every [[Definition:Ordered Tuple|ordered $n + 1$-tuple]] in $S^{n+1}$. +It will be demonstrated by the [[Principle of Mathematical Induction]] that $A = \N$. +=== Basis for the Induction === +We have that: +:$\oplus_1$ is the unary operation defined as $\oplus_1 \left({a}\right) = a$ +By the [[Principle of Recursive Definition]]: +:$\oplus_2 = s \left({\oplus_1}\right)$ +is the [[Definition:Unique|unique]] [[Definition:Binary Operation|operation]] on $S$ such that: +{{begin-eqn}} +{{eqn | lo= \forall \left({a_1, a_2}\right) \in S^2: + | l = \oplus_2 \left({a_1, a_2}\right) + | r = s \left({\oplus_1}\right) \left({a_1, a_2}\right) + | c = Definition of $s$ +}} +{{eqn | r = \oplus_1 \left({a_1}\right) \oplus a_2 + | c = +}} +{{eqn | r = a_1 \oplus a_2 + | c = +}} +{{end-eqn}} +By the definition of a [[Definition:Magma|magma]], $\oplus$ is a [[Definition:Binary Operation|operation]] defined such that: +:$\forall \left({a_1, a_2}\right) \in S \times S: \oplus \left({a_1, a_2}\right) \in S$ +where $\oplus: S \times S \to S$ is a [[Definition:Mapping|mapping]]. +$a_1 \oplus a_2$ is defined as: +:$\forall \left({a_1, a_2}\right) \in S \times S: a_1 \oplus a_2 := \oplus \left({a_1, a_2}\right)$ +So $1 \in A$. +This is our [[Principle of Mathematical Induction#Basis for the Induction|basis for the induction]]. +=== Induction Hypothesis === +It is to be shown that, if $m \in A$ where $m \ge 1$, then it follows that $m + 1 \in A$. +This is the [[Principle of Mathematical Induction#Induction Hypothesis|induction hypothesis]]: +:$\oplus_m$ is an [[Definition:Operation on Set|$m$-ary operation]] on $S$ +and: +:by the definition of $s$, $(2)$ holds for every [[Definition:Ordered Tuple|ordered $m + 1$-tuple]] in $S^{m+1}$. +It is to be demonstrated that it follows that: +:$\oplus_{m+1}$ is an [[Definition:Operation on Set|$m+1$-ary operation]] on $S$ +and: +:by the definition of $s$, $(2)$ holds for every [[Definition:Ordered Tuple|ordered $m + 2$-tuple]] in $S^{m+2}$. +=== Induction Step === +This is our [[Principle of Mathematical Induction#Induction Step|induction step]]: +By the [[Principle of Recursive Definition]]: +:$\oplus_{m+1} = s \left({\oplus_m}\right)$ +is the [[Definition:Unique|unique]] [[Definition:Binary Operation|operation]] on $S$ such that: +{{begin-eqn}} +{{eqn | lo= \forall \left({a_1, a_2, \ldots, a_{m+1} }\right) \in S^{m+1}: + | l = \oplus_{m+1} \left({a_1, a_2, \ldots, a_{m+1} }\right) + | r = s \left({\oplus_m}\right) \left({a_1, a_2, \ldots, a_{m+1} }\right) + | c = Definition of $s$ +}} +{{eqn | r = \oplus_m \left({a_1, a_2, \ldots, a_m}\right) \oplus a_{m+1} + | c = +}} +{{eqn | r = \left({a_1 \oplus a_2 \oplus \cdots \oplus a_m}\right) \oplus a_{m+1} + | c = +}} +{{end-eqn}} +By the definition of a [[Definition:Magma|magma]], $\oplus$ is a [[Definition:Binary Operation|operation]] defined such that: +:$\forall \left({a_1, a_2}\right) \in S \times S: \oplus \left({a_1, a_2}\right) \in S$ +where $\oplus: S \times S \to S$ is a [[Definition:Mapping|mapping]]. +By the [[General Operation from Binary Operation#Induction Hypothesis|induction hypothesis]]: +:$\oplus_m \left({a_1, a_2, \ldots, a_m}\right) \oplus a_{m+1} \in S$ +So by definition: +$\oplus_{m+1} \left({a_1, a_2, \ldots, a_m, a_{m+1} }\right) \in S$ +Let $b := \oplus_{m+1} \left({a_1, a_2, \ldots, a_m, a_{m+1} }\right)$. +But then by the [[General Operation from Binary Operation#Basis of the Induction|basis of the induction]]: +:$b \oplus a_{m+2} \in S$ +That is: +:$\oplus_{m+1} \left({a_1, a_2, \ldots, a_m, a_{m+1} }\right) \oplus a_{m+2} = \oplus_{m+2} \left({a_1, a_2, \ldots, a_m, a_{m+1}, a_{m+2} }\right) \in S$ +So $m \in A \implies m + 1 \in A$ and the result follows by the [[Principle of Mathematical Induction]]: +{{Qed}} +\end{proof}<|endoftext|> +\section{Strictly Increasing Sequence induces Partition} +Tags: Sequences + +\begin{theorem} +Let $\left \langle {r_k} \right \rangle_{0 \mathop \le k \mathop \le n}$ be a [[Definition:Strictly Increasing Sequence|strictly increasing]] [[Definition:Finite Sequence|finite sequence]] of [[Definition:Natural Numbers|natural numbers]]. +Let: +:$\forall k \in \left[{1 \,.\,.\, n}\right]: A_k := \left[{r_{k-1} + 1 \,.\,.\, r_k}\right]$ +Then: +:$\left\{{A_k: k \in \left[{1 \,.\,.\, n}\right]}\right\}$ is a [[Definition:Partition (Set Theory)|partition]] of $\left[{r_0 + 1 \,.\,.\, r_n}\right]$. +\end{theorem} + +\begin{proof} +First we show that the [[Definition:Element|elements]] of $\left\{{A_k: k \in \left[{1 \,.\,.\, n}\right]}\right\}$ are [[Definition:Disjoint Sets|disjoint]]. +Let $j \in \left[{1 \,.\,.\, n}\right]$. +We have that: +: $\left \langle {r_k} \right \rangle_{0 \mathop \le k \mathop \le n}$ is [[Definition:Strictly Increasing Sequence|strictly increasing]] +and: +: $0 \le j - 1 < j \le n$ +Thus by [[Sum with One is Immediate Successor in Naturally Ordered Semigroup]]: +: $r_0 \le r_{j-1} < r_j \le r_n \implies \left({r_0 + 1}\right) \le \left({r_{j-1} + 1}\right) \le r_j \le r_n$ +So: +: $\varnothing \subset A_j \subseteq \left[{r_0 + 1 \,.\,.\, r_n}\right]$ +Because $\left \langle {r_k} \right \rangle_{0 \mathop \le k \mathop \le n}$ is [[Definition:Strictly Increasing Sequence|strictly increasing]]: +:$1 \le j < k \le n \implies A_j \cap A_k = \varnothing$ +So the elements of $\left\{{A_k: k \in \left[{1 \,.\,.\, n}\right]}\right\}$ are [[Definition:Disjoint Sets|disjoint]], as we were to show. +It remains to be established that: +:if $m \in \left[{r_0 + 1 \,.\,.\, r_n}\right]$ +then: +: $m \in A_k$ for some $k \in \left[{1\,.\,.\, n}\right]$. +Let $m \in \left[{r_0 + 1 \,.\,.\, r_n}\right]$. +Consider the set: +:$J = \left\{{j \in \left[{0 \,.\,.\, n}\right]: m \le r_j}\right\}$ +Clearly $j \ne \varnothing$ as $n \in J$. +Let $k$ be the [[Definition:Smallest Element|smallest]] element of $J$. +Then because $r_0 < m$: +: $k \ne 0$ +Thus: +: $k \in \left[{1 \,.\,.\, n}\right]$ +By its definition: +: $r_{k-1} < m \le r_k$ +Thus, by [[Sum with One is Immediate Successor in Naturally Ordered Semigroup]]: +: $r_{k-1} + 1 \le m \le r_k$ +Therefore $m \in A_k$. +{{qed}} +{{improve|A better link instead of [[Sum with One is Immediate Successor in Naturally Ordered Semigroup]]}} +\end{proof}<|endoftext|> +\section{Fundamental Principle of Counting} +Tags: Sequences, Fundamental Theorems, Set Theory, Combinatorics + +\begin{theorem} +Let $A$ be a [[Definition:Finite Set|finite set]]. +Let $\sequence {B_n}$ be a [[Definition:Sequence of Distinct Terms|sequence of distinct]] [[Definition:Finite Set|finite]] [[Definition:Subset|subsets]] of $A$ which form a [[Definition:Set Partition|partition]] of $A$. +Let $p_k = \size {B_k}$ for each $k \in \closedint 1 n$. +Then: +:$\displaystyle \size A = \sum_{k \mathop = 1}^n p_k$ +That is, the sum of the numbers of [[Definition:Element|elements]] in the [[Definition:Subset|subsets]] of a [[Definition:Set Partition|partition]] of a [[Definition:Set|set]] is equal to the total number of [[Definition:Element|elements]] in the [[Definition:Set|set]]. +\end{theorem} + +\begin{proof} +Let $r_0 = 0$, and let: +:$\displaystyle \forall k \in \closedint 1 n: r_k = \sum_{j \mathop = 1}^k {p_j}$ +Then: +:$r_{k - 1} + p_k = r_k$ +so: +:$r_{k - 1} < r_k$. +Thus by [[Isomorphism to Closed Interval]], $\closedint {r_{k - 1} } {r_k}$ has $r_k - r_{k - 1} = p_k$ [[Definition:Element|elements]]. +As a consequence, there exists a [[Definition:Bijection|bijection]] $\sigma_k: B_k \to \closedint {r_{k - 1} } {r_k}$ for each $k \in \closedint 1 n$. +Let $\sigma: A \to \N$ be the [[Definition:Mapping|mapping]] that satisfies: +:$\forall x \in B_k: \forall k \in \N: \map \sigma x = \map {\sigma_k} x$ +By [[Strictly Increasing Sequence on Ordered Set]], $\sequence {r_k}_{0 \mathop \le k \mathop \le n}$ is a [[Definition:Strictly Increasing Sequence|strictly increasing sequence]] of [[Definition:Natural Numbers|natural numbers]]. +Thus by [[Strictly Increasing Sequence induces Partition]], $\sigma: A \to \closedint 1 {r_n}$ is a [[Definition:Bijection|bijection]]. +By [[Isomorphism to Closed Interval]], $\closedint 1 {r_n}$ has $r_n$ [[Definition:Element|elements]]. +Hence the result. +{{Qed}} +\end{proof}<|endoftext|> +\section{Odd Number Theorem} +Tags: Square Numbers, Proofs by Induction, Sums of Sequences, Named Theorems, Odd Number Theorem + +\begin{theorem} +:$\displaystyle \sum_{j \mathop = 1}^n \paren {2 j - 1} = n^2$ +That is, the sum of the first $n$ [[Definition:Odd Integer|odd numbers]] is the $n$th [[Definition:Square Number|square number]]. +\end{theorem} + +\begin{proof} +Proof by [[Principle of Mathematical Induction|induction]]: +For all $n \in \N_{>0}$, let $\map P n$ be the [[Definition:Proposition|proposition]]: +:$\displaystyle n^2 = \sum_{j \mathop = 1}^n \paren {2 j - 1}$ +=== Basis for the Induction === +$\map P 1$ is true, as this just says $1^2 = 1$. +This is our [[Definition:Basis for the Induction|basis for the induction]]. +=== Induction Hypothesis === +Now we need to show that, if $\map P k$ is true, where $k \ge 1$, then it logically follows that $\map P {k + 1}$ is true. +So this is our [[Definition:Induction Hypothesis|induction hypothesis]]: +:$\displaystyle k^2 = \sum_{j \mathop = 1}^k \paren {2 j - 1}$ +Then we need to show: +:$\displaystyle \paren {k + 1}^2 = \sum_{j \mathop = 1}^{k + 1} \paren {2 j - 1}$ +=== Induction Step === +This is our [[Definition:Induction Step|induction step]]: +{{begin-eqn}} +{{eqn | l = \paren {k + 1}^2 + | r = k^2 + 2 k + 1 + | c = +}} +{{eqn | r = \sum_{j \mathop = 1}^k \paren {2 j - 1} + 2 k + 1 + | c = [[Odd Number Theorem#Induction Hypothesis|Induction Hypothesis]] +}} +{{eqn | r = \sum_{j \mathop = 1}^k \paren {2 j - 1} + 2 \paren {k + 1} - 1 + | c = +}} +{{eqn | r = \sum_{j \mathop = 1}^{k + 1} \paren {2 j - 1} + | c = +}} +{{end-eqn}} +So $\map P k \implies \map P {k + 1}$ and the result follows by the [[Principle of Mathematical Induction]]. +Therefore: +:$\displaystyle \forall n \in \N: n^2 = \sum_{j \mathop = 1}^n \paren {2 j - 1}$ +{{qed}} +\end{proof} + +\begin{proof} +{{begin-eqn}} +{{eqn | l = \sum_{j \mathop = 1}^n \paren {2 j - 1} + | r = n^2 + | c = [[Odd Number Theorem]] +}} +{{eqn | ll= \leadsto + | l = \sum_{j \mathop = 1}^n \paren {2 j - 1} + \sum_{j \mathop = 1}^n 1 + | r = n^2 + n + | c = +}} +{{eqn | ll= \leadsto + | l = \sum_{j \mathop = 1}^n \paren {2 j} + | r = n \paren {n + 1} + | c = +}} +{{eqn | ll= \leadsto + | l = \sum_{j \mathop = 1}^n j + | r = \frac {n \paren {n + 1} } 2 + | c = +}} +{{end-eqn}} +{{qed}} +\end{proof}<|endoftext|> +\section{General Associativity Theorem} +Tags: Associativity, Named Theorems, General Associativity Theorem + +\begin{theorem} +If an [[Definition:Operation|operation]] is [[Definition:Associative Operation|associative]] on $3$ entities, then it is [[Definition:Associative Operation|associative]] on any number of them. +\end{theorem}<|endoftext|> +\section{General Commutativity Theorem} +Tags: Named Theorems, Commutativity + +\begin{theorem} +Let $\struct {S, \circ}$ be a [[Definition:Semigroup|semigroup]]. +Let $\family {a_k}_{1 \mathop \le k \mathop \le n}$ be a [[Definition:Finite Sequence|sequence]] of [[Definition:Element|elements]] of $S$. +Suppose that: +:$\forall i, j \in \closedint 1 n: a_i \circ a_j = a_j \circ a_i$ +Then for every [[Definition:Permutation|permutation]] $\sigma: \N_n \to \N_n$: +:$a_{\map \sigma 1} \circ \cdots \circ a_{\map \sigma n} = a_1 \circ \cdots \circ a_n$ +where $\N_n$ is used here to denote the [[Definition:Initial Segment of One-Based Natural Numbers|initial segment]] of $\N_{>0}$: +:$\N_n = \set {1, 2, \ldots, n}$ +\end{theorem} + +\begin{proof} +The proof will proceed by the [[Principle of Mathematical Induction]] on $\N_{>0}$. +Let $T$ be the [[Definition:Set|set]] of all $n \in \N_{>0}$ such that: +:$a_{\map \sigma 1} \circ \cdots \circ a_{\map \sigma n} = a_1 \circ \cdots \circ a_n$ +holds for all [[Definition:Finite Sequence|sequences]] $\sequence {a_k}_{1 \mathop \le k \mathop \le n}$ of $n$ [[Definition:Element|elements]] of $S$ which satisfy: +:$\forall i, j \in \closedint 1 n: a_i \circ a_j = a_j \circ a_i$ +for every [[Definition:Permutation|permutation]] $\sigma: \N_n \to \N_n$. +=== Basis for the Induction === +We have that when $n = 1$: +:$\map \sigma 1 = 1$ +and so: +:$a_{\map \sigma 1} = a_1$ +for all $\sigma: \set 1 \to \set 1$. +Thus $1 \in T$. +This is our [[Definition:Basis for the Induction|basis for the induction]]. +=== Induction Hypothesis === +It is to be shown that, if $m \in T$ where $m \ge 1$, then it follows that $m + 1 \in T$. +This is the [[Definition:Induction Hypothesis|induction hypothesis]]: +:$a_{\map \sigma 1} \circ \cdots \circ a_{\map \sigma m} = a_1 \circ \cdots \circ a_m$ +holds for all [[Definition:Finite Sequence|sequences]] $\sequence {a_k}_{1 \mathop \le k \mathop \le m}$ of $m$ [[Definition:Element|elements]] of $S$ which satisfy: +:$\forall i, j \in \closedint 1 m: a_i \circ a_j = a_j \circ a_i$ +for every [[Definition:Permutation|permutation]] $\sigma: \N^*_m \to \N^*_m$. +It is to be demonstrated that it follows that: +:$a_{\map \sigma 1} \circ \cdots \circ a_{\map \sigma {m + 1} } = a_1 \circ \cdots \circ a_{m + 1}$ +holds for all [[Definition:Finite Sequence|sequences]] $\sequence {a_k}_{1 \mathop \le k \mathop \le m + 1}$ of $m + 1$ [[Definition:Element|elements]] of $S$ which satisfy: +:$\forall i, j \in \closedint 1 {m + 1}: a_i \circ a_j = a_j \circ a_i$ +for every [[Definition:Permutation|permutation]] $\sigma: \N_{m + 1} \to \N_{m + 1}$. +=== Induction Step === +This is our [[Definition:Induction Step|induction step]]: +Suppose $m \in T$. +Let $\sequence {a_k}_{1 \le k \le m + 1}$ be a [[Definition:Finite Sequence|sequence]] of $m+1$ [[Definition:Element|elements]] of $S$ which satisfy: +:$\forall i, j \in \closedint 1 {n + 1}: a_i \circ a_j = a_j \circ a_i$ +Let $\sigma: \N_{m + 1} \to \N_{m + 1}$ be a [[Definition:Permutation|permutation]] of $\closedint {1 {m + 1}$. +There are three cases to consider: +:$(1): \quad \map \sigma {m + 1} = m + 1$ +:$(2): \quad \map \sigma 1 = m + 1$ +:$(3): \quad \map \sigma r = m + 1$ for some $r \in \closedint 2 m$ +$(1): \quad$ Suppose $\map \sigma {m + 1} = m + 1$. +Then the [[Definition:Restriction of Mapping|restriction]] of $\sigma$ to $\N_m$ is then a [[Definition:Permutation|permutation]] of $\N_m$. +From the [[General Commutativity Theorem#Induction Hypothesis|induction hypothesis]]: +:$m \in T$: +Thus: +:$a_{\map \sigma 1} \circ \cdots \circ a_{\map \sigma m} = a_1 \circ \cdots \circ a_m$ +from which: +{{begin-eqn}} +{{eqn | l = a_{\map \sigma 1} \circ \cdots \circ a_{\map \sigma {m + 1} } + | r = \paren {a_{\map \sigma 1} \circ \cdots \circ a_{\map \sigma m} } \circ a_{\map \sigma {m + 1} } + | c = +}} +{{eqn | r = \paren {a_1 \circ \cdots \circ a_m} \circ a_{m + 1} + | c = +}} +{{eqn | r = a_1 \circ \cdots \circ a_{m + 1} + | c = +}} +{{end-eqn}} +$(2): \quad$ Suppose $\map \sigma 1 = m + 1$. +Let $\tau: \N^*_m \to \N^*_m$ be the mapping defined as: +:$\forall k \in \closedint 1 m: \map \tau k = \map \sigma {k + 1}$ +From [[Closed Interval of Naturally Ordered Semigroup with Successor equals Union with Successor]]: +:$\closedint 1 {m + 1} = \closedint 1 m \cup \set {m + 1}$ +Thus $\tau$ is clearly a [[Definition:Permutation|permutation]] on $\closedint 1 m$. +Thus, as $m \in T$: +:$a_{\map \tau 1} \circ \cdots \circ a_{\map \tau m} = a_1 \circ \cdots \circ a_m$ +So: +{{begin-eqn}} +{{eqn | l = a_{\map \sigma 1} \circ \cdots \circ a_{\map \sigma {m + 1} } + | r = a_{\map \sigma 1} \circ \paren {a_{\map \sigma 2} \circ \cdots \circ a_{\map \sigma {m + 1} } } + | c = +}} +{{eqn | r = a_{m + 1} \circ \paren {a_{\map \tau 1} \circ \cdots \circ a_{\map \tau {m + 1} } } + | c = +}} +{{eqn | r = a_{m + 1} \circ \paren {a_1 \circ \cdots \circ a_m} + | c = +}} +{{eqn | r = \paren {a_1 \circ \cdots \circ a_m} \circ a_{m + 1} + | c = [[Element Commutes with Product of Commuting Elements]] +}} +{{eqn | r = a_1 \circ \cdots \circ a_{m + 1} + | c = +}} +{{end-eqn}} +$(3): \quad$ Suppose $\map \sigma r = m + 1$ for some $r \in \closedint 2 m$. +Let $\tau: \N_{m + 1} \to \N_{m + 1}$ be a [[Definition:Mapping|mapping]] defined by: +:$\forall k \in \N_{m + 1}: \map \tau k = \begin{cases} +\map \sigma k & : k \in \closedint 1 {r - 1} \\ +\map \sigma {k + 1} & : k \in \closedint r m \\ +m + 1 & : k = m + 1 \end{cases}$ +It is seen that $\tau$ is a [[Definition:Permutation|permutation]] of $\N_{m + 1}$. +So, by the first case: +:$a_{\map \tau 1} \circ \cdots \circ a_{\map \tau {m + 1} } = a_1 \circ \cdots \circ a_{m + 1}$ +Thus: +{{begin-eqn}} +{{eqn | l = a_{\map \sigma 1} \circ \cdots \circ a_{\map \sigma {m + 1} } + | r = \paren {a_{\map \sigma 1} \circ \cdots \circ a_{\map \sigma {r - 1} } } \circ \paren {a_{\map \sigma r} \circ \paren {a_{\map \sigma {r + 1} } \circ \cdots \circ a_{\map \sigma {m + 1} } } } + | c = +}} +{{eqn | r = \paren {a_{\map \tau 1} \circ \cdots \circ a_{\map \tau {r - 1} } } \circ \paren {a_{\map \tau {m + 1} } \circ \paren {a_{\map \tau r} \circ \cdots \circ a_{\map \tau m} } } + | c = +}} +{{eqn | r = \paren {a_{\map \tau 1} \circ \cdots \circ a_{\map \tau {r - 1} } } \circ \paren {\paren {a_{\map \tau r} \circ \cdots \circ a_{\map \tau m} } \circ a_{\map \tau {m + 1} } } + | c = +}} +{{eqn | r = a_{\map \tau 1} \circ \cdots \circ a_{\map \tau {m + 1} } + | c = +}} +{{eqn | r = a_1 \circ \cdots \circ a_{m + 1} + | c = +}} +{{end-eqn}} +So in all cases, $m + 1 \in T$. +Thus by the [[Principle of Mathematical Induction]]: +:$T = \N_{>0}$ +The result follows. +{{qed}} +\end{proof}<|endoftext|> +\section{General Distributivity Theorem} +Tags: Abstract Algebra, Summations, Named Theorems, Distributive Operations, General Distributivity Theorem + +\begin{theorem} +:$\displaystyle \paren {\sum_{i \mathop = 1}^m a_i} * \paren {\sum_{j \mathop = 1}^n b_j} = \sum_{ {1 \mathop \le i \mathop \le m} \atop {1 \mathop \le j \mathop \le n} } \paren {a_i * b_j}$ +\end{theorem} + +\begin{proof} +Proof by [[Principle of Mathematical Induction|induction]]: +For all $n \in \Z_{> 0}$, let $\map P n$ be the [[Definition:Proposition|proposition]]: +:$\displaystyle \forall m \in \Z_{> 0}: \paren {\sum_{i \mathop = 1}^m a_i} * \paren {\sum_{j \mathop = 1}^n b_j} = \sum_{ {1 \mathop \le i \mathop \le m} \atop {1 \mathop \le j \mathop \le n} } \paren {a_i * b_j}$ +We have that $\struct {R, \circ, *}$ is a [[Definition:Ringoid (Abstract Algebra)|ringoid]], and so: +:$\forall a, b, c \in S: \paren {a \circ b} * c = \paren {a * c} \circ \paren {b * c}$ +:$\forall a, b, c \in R: a * \paren {b \circ c} = \paren {a * b} \circ \paren {a * c}$ +=== Basis for the Induction === +$\map P 1$ is the case: +:$\displaystyle \forall m \in \Z_{>0}: \paren {\sum_{i \mathop = 1}^m a_i} * b_1 = \sum_{1 \mathop \le i \mathop \le m} \paren {a_i * b_1}$ +This is demonstrated in [[General Distributivity Theorem/Lemma 1|Lemma 1]]. +This is our [[Definition:Basis for the Induction|basis for the induction]]. +=== Induction Hypothesis === +Now we need to show that, if $\map P k$ is true, where $k \ge 2$, then it logically follows that $\map P {k + 1}$ is true. +So this is our [[Definition:Induction Hypothesis|induction hypothesis]]: +:$\displaystyle \forall m \in \Z_{> 0}: \paren {\sum_{i \mathop = 1}^m a_i} * \paren {\sum_{j \mathop = 1}^k b_j} = \sum_{ {1 \mathop \le i \mathop \le m} \atop {1 \mathop \le j \mathop \le k} } \paren {a_i * b_j}$ +Then we need to show: +:$\displaystyle \forall m \in \Z_{> 0}: \paren {\sum_{i \mathop = 1}^m a_i} * \paren {\sum_{j \mathop = 1}^{k + 1} b_j} = \sum_{ {1 \mathop \le i \mathop \le m} \atop {1 \mathop \le j \mathop \le {k + 1} } } \paren {a_i * b_j}$ +=== Induction Step === +This is our [[Definition:Induction Step|induction step]]: +{{begin-eqn}} +{{eqn | l = \paren {\sum_{i \mathop = 1}^m a_i} * \paren {\sum_{j \mathop = 1}^{k + 1} b_j} + | r = \paren {\sum_{i \mathop = 1}^m a_i} * \paren {\paren {\sum_{j \mathop = 1}^k b_j} \circ b_{k + 1} } + | c = {{Defof|Summation}} +}} +{{eqn | r = \paren {\paren {\sum_{i \mathop = 1}^m a_i} * \paren {\sum_{j \mathop = 1}^k b_j} } \circ \paren {\paren {\sum_{i \mathop = 1}^m a_i} * b_{k + 1} } + | c = as $\struct {R, \circ, *}$ is a [[Definition:Ringoid (Abstract Algebra)|ringoid]] +}} +{{eqn | r = \paren {\paren {\sum_{i \mathop = 1}^m a_i} * \paren {\sum_{j \mathop = 1}^k b_j} } \circ \sum_{1 \mathop \le i \mathop \le m} \paren {a_i * b_{k + 1} } + | c = [[General Distributivity Theorem#Basis for the Induction|Basis for the Induction]] +}} +{{eqn | r = \sum_{ {1 \mathop \le i \mathop \le m} \atop {1 \mathop \le j \mathop \le k} } \paren {a_i * b_j} \circ \sum_{1 \mathop \le i \mathop \le m} \paren {a_i * b_{k + 1} } + | c = [[General Distributivity Theorem#Induction Hypothesis|Induction Hypothesis]] +}} +{{eqn | r = \sum_{ {1 \mathop \le i \mathop \le m} \atop {1 \mathop \le j \mathop \le {k + 1} } } \paren {a_i * b_j} + | c = [[Definition:Associative Operation|Associativity]] of $\circ$ in $\struct {R, \circ, *}$ +}} +{{end-eqn}} +So $\map P k \implies \map P {k + 1}$ and the result follows by the [[Principle of Mathematical Induction]]. +Therefore: +:$\displaystyle \forall m, n \in \Z_{> 0}: \paren {\sum_{i \mathop = 1}^m a_i} * \paren {\sum_{j \mathop = 1}^n b_j} = \sum_{ {1 \mathop \le i \mathop \le m} \atop {1 \mathop \le j \mathop \le n} } \paren {a_i * b_j}$ +{{qed}} +The same result can be obtained by fixing $n$ and using induction on $m$, which requires [[General Distributivity Theorem/Lemma 2|Lemma 2]] to be used for its [[Principle of Mathematical Induction#Basis for the Induction|base case]]. +\end{proof}<|endoftext|> +\section{Associativity on Indexing Set} +Tags: Indexed Families, Associativity + +\begin{theorem} +Let $\left({S, \circ}\right)$ be a [[Definition:Commutative Semigroup|commutative semigroup]]. +Let $\left \langle {x_\alpha} \right \rangle_{\alpha \mathop \in A}$ be a [[Definition:Indexed Family|family of terms of $S$ indexed by]] a [[Definition:Finite Set|finite]] [[Definition:Non-Empty Set|non-empty set]] $A$. +Let $\left \langle {B_k} \right \rangle_{1 \mathop \le k \mathop \le n}$ be a [[Definition:Family of Distinct Elements|family of distinct]] [[Definition:Subset|subsets]] of $A$ forming a [[Definition:Partition (Set Theory)|partition]] of $A$. +Then: +: $\displaystyle \prod_{k \mathop = 1}^n \left({\prod_{a \mathop \in B_k} x_\alpha}\right) = \prod_{\alpha \mathop \in A} x_\alpha$ +\end{theorem} + +\begin{proof} +For each $k \in \N_{>0}$, let $\left|{B_k}\right| = p_k$. +Let: +:$r_0 = 0$ +:$\displaystyle \forall k \in \N_{>0}: r_k = \sum_{j \mathop = 1}^k {p_j}$ +and: +:$p = r_n$ +Then: +: $r_k - r_{k-1} = p_k$ +So, by [[Isomorphism to Closed Interval]], both $\left[{1 \,.\,.\, p_k}\right]$ and $\left[{r_{k - 1} + 1 \,.\,.\, r_k}\right]$ have $p_k$ elements. +By [[Unique Isomorphism between Finite Totally Ordered Sets]], there is a unique [[Definition:Isomorphism (Abstract Algebra)|isomorphism]] $\tau_k: \left[{1 \,.\,.\, p_k}\right] \to \left[{r_{k - 1} + 1 \,.\,.\, r_k}\right]$ as both are [[Definition:Totally Ordered Set|totally ordered]]. +The [[Definition:Ordering|orderings]] on both of these are those induced by the ordering on $\N$. +It is clear that $\tau_k$ is defined as: +: $\forall j \in \left[{1 \,.\,.\, p_k}\right]: \tau_k \left({j}\right) = r_k + j$ +For each $k \in \left[{1 \,.\,.\, n}\right]$, let $\rho_k: \left[{1 \,.\,.\, p_k}\right] \to B_k$ be a [[Definition:Bijection|bijection]]. +By [[Strictly Increasing Sequence induces Partition]], the [[Definition:Mapping|mapping]] $\sigma: \left[{1 \,.\,.\, p}\right] \to A$ defined by: +: $\displaystyle \forall j \in \left[{r_{k-1}+1 \,.\,.\, r_k}\right]: \forall k \in \left[{1 \,.\,.\, n}\right]: \sigma \left({j}\right) = \rho_k \left({\tau_k^{-1} \left({j}\right)}\right)$ +is a [[Definition:Bijection|bijection]]. +Let $\forall j \in \left[{1 \,.\,.\, p}\right]: y_j = x_{\sigma \left({j}\right)}$. +By definition: +: $\displaystyle \prod_{\alpha \mathop \in A} {x_\alpha} = \prod_{j \mathop = 1}^p {x_{\sigma \left({j}\right)}} = \prod_{j \mathop = 1}^p {y_j}$ +Also: +: $\displaystyle \forall k \in \left[{1 \,.\,.\, n}\right]: \prod_{\alpha \mathop \in B_k} {x_\alpha} = \prod_{i \mathop = 1}^{p^k} {x_{\rho_k \left({i}\right)}}$ +Also by definition: +: $\displaystyle \prod_{j \mathop = r_{k - 1} + 1}^{r_k} y_j = \prod_{i \mathop = 1}^{p_k} y_{\tau_k \left({i}\right)} = \prod_{i \mathop = 1}^{p_k} x_{\sigma \left({\tau_k \left({i}\right)}\right)} = \prod_{i \mathop = 1}^{p_k} x_{\rho_k \left({i}\right)}$ +So by the [[General Associativity Theorem]]: +{{begin-eqn}} +{{eqn | l = \prod_{a \mathop \in A} {x_a} + | r = \prod_{j \mathop = 1}^p y_j + | c = +}} +{{eqn | r = \prod_{k \mathop = 1}^n \left({\prod_{j \mathop = r_{k - 1} \mathop + 1}^{r_k} {y_j} }\right) + | c = +}} +{{eqn | r = \prod_{k \mathop = 1}^n \left({\prod_{i \mathop = 1}^{p_k} x_{\rho_k \left({i}\right)} }\right) + | c = +}} +{{eqn | r = \prod_{k \mathop = 1}^n \left({\prod_{a \mathop \in B_k} x_\alpha}\right) + | c = +}} +{{end-eqn}} +{{qed}} +\end{proof}<|endoftext|> +\section{External Direct Product of Groups is Group} +Tags: Group Theory, Direct Products, Group Direct Products + +\begin{theorem} +Let $\struct {G_1, \circ_1}$ and $\struct {G_2, \circ_2}$ be [[Definition:Group|groups]] whose [[Definition:Identity Element|identity elements]] are $e_1$ and $e_2$ respectively. +Let $\struct {G_1 \times G_2, \circ}$ be the [[Definition:Group Direct Product|external direct product]] of $G_1$ and $G_2$. +Then $\struct {G_1 \times G_2, \circ}$ is a [[Definition:Group|group]] whose [[Definition:Identity Element|identity element]] is $\tuple {e_1, e_2}$. +\end{theorem} + +\begin{proof} +Taking the [[Definition:Group Axioms|group axioms]] in turn: +=== G0: Closure === +From [[External Direct Product Closure]] it follows that $\struct {G_1 \times G_2, \circ}$ is [[Definition:Closed Algebraic Structure|closed]]. +{{qed|lemma}} +=== G1: Associativity === +From [[External Direct Product Associativity]] it follows that $\circ$ is [[Definition:Associative Operation|associative]] on $G_1 \times G_2$. +{{qed|lemma}} +=== G2: Identity === +From [[External Direct Product Identity]] it follows that $\tuple {e_1, e_2}$ is the [[Definition:Identity Element|identity element]] of $\struct {G_1 \times G_2, \circ}$. +{{qed|lemma}} +=== G3: Inverses === +From [[External Direct Product Inverses]] it follows that $\tuple {g_1^{-1}, g_2^{-1} }$ is the [[Definition:Inverse Element|inverse element]] of $\tuple {g_1, g_2}$ in $\struct {G_1 \times G_2, \circ}$. +{{qed|lemma}} +All [[Definition:Group Axioms|group axioms]] are fulfilled, hence the result. +{{qed}} +\end{proof}<|endoftext|> +\section{External Direct Product of Projection with Canonical Injection} +Tags: Abstract Algebra, Direct Products, Cartesian Product, Projections + +\begin{theorem} +Let $\struct {S_1, \circ_1}$ and $\struct {S_2, \circ_2}$ be [[Definition:Algebraic Structure|algebraic structures]] with [[Definition:Identity Element|identity elements]] $e_1$ and $e_2$ respectively. +Let $\struct {S_1 \times S_2, \circ}$ be the [[Definition:External Direct Product|external direct product]] of $\struct {S_1, \circ_1}$ and $\struct {S_2, \circ_2}$ +Let: +:$\pr_1: \struct {S_1 \times S_2, \circ} \to \struct {S_1, \circ_1}$ be the [[Definition:First Projection|first projection]] from $\struct {S_1 \times S_2, \circ}$ to $\struct {S_1, \circ_1}$ +:$\pr_2: \struct {S_1 \times S_2, \circ} \to \struct {S_2, \circ_2}$ be the [[Definition:Second Projection|second projection]] from $\struct {S_1 \times S_2, \circ}$ to $\struct {S_2, \circ_2}$. +Let: +:$\inj_1: \struct {S_1, \circ_1} \to \struct {S_1 \times S_2, \circ}$ be the [[Definition:Canonical Injection (Abstract Algebra)|canonical injection]] from $\struct {S_1, \circ_1}$ to $\struct {S_1 \times S_2, \circ}$ +:$\inj_2: \struct {S_2, \circ_2} \to \struct {S_1 \times S_2, \circ}$ be the [[Definition:Canonical Injection (Abstract Algebra)|canonical injection]] from $\struct {S_2, \circ_2}$ to $\struct {S_1 \times S_2, \circ}$. +Then: +:$(1): \quad \pr_1 \circ \inj_1 = I_{S_1}$ +:$(2): \quad \pr_2 \circ \inj_2 = I_{S_2}$ +where $I_{S_1}$ and $I_{S_2}$ are the [[Definition:Identity Mapping|identity mappings]] on $S_1$ and $S_2$ respectively. +\end{theorem} + +\begin{proof} +Let $\tuple {s_1, s_2} \in S_1 \times S_2$. +So, $s_1 \in S_1$ and $s_2 \in S_2$. +From the definition of the [[Definition:Canonical Injection (Abstract Algebra)|canonical injection]], we have: +:$\map {\inj_1} {s_1} = \tuple {s_1, e_2}$ +:$\map {\inj_2} {s_2} = \tuple {e_1, s_2}$ +From the definition of the [[Definition:First Projection|first projection]]: +:$\map {\pr_1} {s_1, e_2} = s_1$ +and similarly from the definition of the [[Definition:Second Projection|second projection]]: +:$\map {\pr_2} {e_1, s_2} = s_2$ +Thus: +:$\map {\pr_1 \circ \inj_1} {s_1} = s_1$ +:$\map {\pr_2 \circ \inj_2} {s_2} = s_2$ +and the result follows from the definition of the [[Definition:Identity Mapping|identity mapping]]. +{{qed}} +\end{proof}<|endoftext|> +\section{Inverse Completion of Natural Numbers} +Tags: Inverse Completions, Natural Numbers + +\begin{theorem} +There exists an [[Definition:Inverse Completion|inverse completion]] of the [[Definition:Natural Numbers|natural numbers]] under [[Definition:Natural Number Addition|addition]]. +\end{theorem} + +\begin{proof} +The set of [[Definition:Natural Numbers|natural numbers]] under [[Definition:Natural Number Addition|addition]] can be denoted $\left ({\N, +}\right)$. +From [[Natural Numbers under Addition form Commutative Monoid]], the [[Definition:Algebraic Structure|algebraic structure]] $\left ({\N, +}\right)$ is a [[Definition:Commutative Monoid|commutative monoid]]. +Therefore by definition of [[Definition:Commutative Monoid|commutative monoid]], $\left ({\N, +}\right)$ is a [[Definition:Commutative Semigroup|commutative semigroup]]. +From [[Natural Number Addition is Cancellable]], all of the elements of $\left ({\N, +}\right)$ are [[Definition:Cancellable Element|cancellable]]. +The result follows from the [[Inverse Completion Theorem]]. +{{qed}} +[[Category:Inverse Completions]] +[[Category:Natural Numbers]] +q7fkui5qs8edlrc74tqw2t2cnac9r61 +\end{proof}<|endoftext|> +\section{L'Hôpital's Rule} +Tags: Differential Calculus, Limits of Functions, L'Hôpital's Rule + +\begin{theorem} +Let $f$ and $g$ be [[Definition:Real Function|real functions]] which are [[Definition:Continuous on Interval|continuous]] on the [[Definition:Closed Real Interval|closed interval]] $\closedint a b$ and [[Definition:Differentiable on Interval|differentiable]] on the [[Definition:Open Real Interval|open interval]] $\openint a b$. +Let: +:$\forall x \in \openint a b: \map {g'} x \ne 0$ +where $g'$ denotes the [[Definition:Derivative|derivative]] of $g$ {{WRT|Differentiation}} $x$. +Let: +:$\map f a = \map g a = 0$ +Then: +:$\displaystyle \lim_{x \mathop \to a^+} \frac {\map f x} {\map g x} = \lim_{x \mathop \to a^+} \frac {\map {f'} x} {\map {g'} x}$ +provided that the second limit exists. +\end{theorem} + +\begin{proof} +Let $l = \displaystyle \lim_{x \mathop \to a^+} \frac{f' \left({x}\right)}{g' \left({x}\right)}$. +Let $\epsilon > 0$. +By the definition of [[Definition:Limit of Real Function|limit]], we ought to find a $\delta > 0$ such that: +:$\forall x: \left\vert{x - a}\right\vert < \delta \implies \left\vert{\dfrac {f \left({x}\right)} {g \left({x}\right)} - l}\right\vert < \epsilon$ +Fix $\delta$ such that: +:$\forall x: \left\vert{x - a}\right\vert < \delta \implies \left\vert{\dfrac {f' \left({x}\right)} {g' \left({x}\right)} - l}\right\vert < \epsilon$ +which is possible by the definition of [[Definition:Limit of Real Function|limit]]. +Let $x$ be such that $\left\vert{x - a}\right\vert < \delta$. +By the [[Cauchy Mean Value Theorem]] with $b = x$: +: $\exists \xi \in \left({a \,.\,.\, x}\right): \dfrac {f' \left({\xi}\right)} {g' \left({\xi}\right)} = \dfrac {f \left({x}\right) - f \left({a}\right)} {g \left({x}\right) - g \left({a}\right)}$ +Since $f \left({a}\right) = g \left({a}\right) = 0$, we have: +: $\exists \xi \in \left({a \,.\,.\, x}\right): \dfrac {f' \left({\xi}\right)} {g' \left({\xi}\right)} = \dfrac {f \left({x}\right)} {g \left({x}\right)}$ +Now, as $a < \xi < x$, it follows that $\left\vert{\xi - a}\right\vert < \delta$ as well. +Therefore: +:$\left\vert{\dfrac {f \left({x}\right)} {g \left({x}\right)} - l }\right\vert = \left\vert{ \dfrac {f' \left({\xi}\right)} {g' \left({\xi}\right)} - l}\right\vert < \epsilon$ +which leads us to the desired conclusion that: +:$\displaystyle \lim_{x \mathop \to a^+} \frac {f \left({x}\right)} {g \left({x}\right)} = \lim_{x \mathop \to a^+} \frac {f' \left({x}\right)} {g' \left({x}\right)}$ +{{qed}} +\end{proof} + +\begin{proof} +Take the [[Cauchy Mean Value Theorem]] with $b = x$: +:$\exists \xi \in \openint a x: \dfrac {\map {f'} \xi} {\map {g'} \xi} = \dfrac {\map f x - \map f a} {\map g x - \map g a}$ +Then if $\map f a = \map g a = 0$ we have: +:$\exists \xi \in \openint a x: \dfrac {\map {f'} \xi} {\map {g'} \xi} = \dfrac {\map f x} {\map g x}$ +Note that $\xi$ depends on $x$; that is, different values of $x$ may require different values of $\xi$ to make the above statement valid. +It follows from [[Limit of Function in Interval]] that $\xi \to a$ as $x \to a$. +Also, $\xi \ne a$ when $x > a$. +So from '''Hypothesis $2$''' of [[Limit of Composite Function]], it follows that: +:$\displaystyle \lim_{x \mathop \to a^+} \dfrac {\map {f'} \xi} {\map {g'} \xi} = \lim_{x \mathop \to a^+} \dfrac {\map {f'} x} {\map {g'} x}$ +Hence the result. +{{qed}} +\end{proof}<|endoftext|> +\section{Natural Numbers under Addition form Commutative Monoid} +Tags: Natural Numbers, Examples of Monoids + +\begin{theorem} +The [[Definition:Algebraic Structure|algebraic structure]] $\struct {\N, +}$ consisting of the [[Definition:Set|set]] of [[Definition:Natural Numbers|natural numbers]] $\N$ under [[Definition:Natural Number Addition|addition]] $+$ is a [[Definition:Commutative Monoid|commutative monoid]] whose [[Definition:Identity Element|identity]] is [[Definition:Zero (Number)|zero]]. +\end{theorem} + +\begin{proof} +Consider the [[Definition:Natural Numbers|natural numbers]] $\N$ defined as the [[Definition:Naturally Ordered Semigroup|naturally ordered semigroup]]. +From the definition of the [[Definition:Naturally Ordered Semigroup|naturally ordered semigroup]], it follows that $\struct {\N, +}$ is a [[Definition:Commutative Semigroup|commutative semigroup]]. +From the definition of [[Definition:Zero (Number)|zero]], $\struct {\N, +}$ has $0 \in \N$ as the [[Definition:Identity Element|identity]], hence is a [[Definition:Monoid|monoid]]. +{{qed}} +\end{proof}<|endoftext|> +\section{Natural Numbers form Commutative Semiring} +Tags: Natural Numbers, Semirings + +\begin{theorem} +The [[Definition:Semiring of Natural Numbers|semiring of natural numbers]] $\struct {\N, +, \times}$ forms a [[Definition:Commutative Semiring|commutative semiring]]. +\end{theorem} + +\begin{proof} +The [[Definition:Algebraic Structure|algebraic structure]] $\struct {\N, +}$ is a [[Definition:Commutative Monoid|commutative monoid]] from [[Natural Numbers under Addition form Commutative Monoid]]. +Then we have: +:$(1): \quad$ $\times$ is [[Definition:Closed Operation|closed]] +:$(2): \quad$ [[Natural Number Multiplication is Associative]] +:$(3): \quad$ [[Natural Number Multiplication is Commutative]] +:$(4): \quad$ [[Natural Number Multiplication Distributes over Addition]] +Thus $\struct {\N, \times}$ forms an [[Definition:Algebraic Structure|algebraic structure]] which is [[Definition:Closure (Abstract Algebra)|closed]] such that $\times$ is [[Definition:Associative Operation|associative]] and [[Definition:Commutative Operation|commutative]]. +So by definition, $\struct {\N, \times}$ is a [[Definition:Commutative Semigroup|commutative semigroup]]. +The result follows from definition of [[Definition:Commutative Semiring|commutative semiring]] and the [[Definition:Distributive Operation|distributivity]] of $\times$ over $+$. +{{qed}} +[[Category:Natural Numbers]] +[[Category:Semirings]] +3dw345e0hzrnc5uqio3ha4k7kqjsxnm +\end{proof}<|endoftext|> +\section{Integer Multiplication is Well-Defined} +Tags: Integer Multiplication, Integer Multiplication is Well-Defined + +\begin{theorem} +[[Definition:Integer Multiplication|Integer multiplication]] is [[Definition:Well-Defined Operation|well-defined]]. +\end{theorem} + +\begin{proof} +Consider the [[Definition:Integer/Formal Definition|formal definition of the integers]]: $x = \eqclass {a, b} {}$ is an [[Definition:Equivalence Class|equivalence class]] of [[Definition:Ordered Pair|ordered pairs]] of [[Definition:Natural Numbers|natural numbers]]. +Consider the [[Definition:Mapping|mapping]] $\phi: \N_{>0} \to \Z_{>0}$ defined as: +:$\forall u \in \N_{>0}: \map \phi u = u'$ +where $u' \in \Z$ be the [[Definition:Strictly Positive Integer|(strictly) positive integer]] $\eqclass {b + u, b} {}$. +Let $v' = \eqclass {c + v, c} {}$. +Then: +{{begin-eqn}} +{{eqn | l = u' v' + | r = \eqclass {b + u, b} {} \times \eqclass {c + v, c} {} + | c = +}} +{{eqn | r = \eqclass {\paren {b + u} \paren {c + v} + b c, \paren {b + u} c + b \paren {c + v} } {} + | c = +}} +{{eqn | r = \eqclass {b c + b v + c u + u v + b c, b c + u c + b c + b v} {} + | c = +}} +{{eqn | r = \eqclass {b c + u v, b c} {} + | c = +}} +{{eqn | r = \eqclass {b + u v, b} {} + | c = +}} +{{eqn | r = \paren {u v}' + | c = +}} +{{end-eqn}} +{{qed}} +\end{proof}<|endoftext|> +\section{Integer Multiplication is Closed} +Tags: Integer Multiplication, Algebraic Closure + +\begin{theorem} +The [[Definition:Set|set]] of [[Definition:Integer|integers]] is [[Definition:Closed Algebraic Structure|closed]] under [[Definition:Integer Multiplication|multiplication]]: +:$\forall a, b \in \Z: a \times b \in \Z$ +\end{theorem} + +\begin{proof} +Let us define $\eqclass {\tuple {a, b} } \boxminus$ as in the [[Definition:Integer/Formal Definition|formal definition of integers]]. +That is, $\eqclass {\tuple {a, b} } \boxminus$ is an [[Definition:Equivalence Class|equivalence class]] of [[Definition:Ordered Pair|ordered pairs]] of [[Definition:Natural Numbers|natural numbers]] under the [[Definition:Congruence Relation|congruence relation]] $\boxminus$. +$\boxminus$ is the [[Definition:Congruence Relation|congruence relation]] defined on $\N \times \N$ by: +:$\tuple {x_1, y_1} \boxminus \tuple {x_2, y_2} \iff x_1 + y_2 = x_2 + y_1$ +In order to streamline the notation, we will use $\eqclass {a, b} {}$ to mean $\eqclass {\tuple {a, b} } \boxminus$, [[Definition:Integer/Formal Definition/Notation|as suggested]]. +[[Definition:Integer Multiplication|Integer multiplication]] is defined as: +:$\forall a, b, c, d \in \N: \eqclass {a, b} {} \times \eqclass {c, d} {} = \eqclass {a c + b d, a d + b c} {}$ +We have that: +:$\forall a, b, c, d \in \N: \eqclass {a, b} {} \in \Z, \eqclass {c, d} {} \in \Z$ +Also: +:$\forall a, b, c, d \in \N: \eqclass {a, b} {} \times \eqclass {c, d} {} = \eqclass {a c + b d, a d + b c} {}$ +But: +:$a c + b d \in \N, a d + b c \in \N$ +So: +:$\forall a, b, c, d \in \N: \eqclass {a c + b d, a d + b c} {} \in \Z$ +Therefore [[Definition:Integer Multiplication|integer multiplication]] is [[Definition:Closed Algebraic Structure|closed]]. +{{qed}} +\end{proof}<|endoftext|> +\section{Integer Multiplication is Commutative} +Tags: Integer Multiplication, Commutativity + +\begin{theorem} +The operation of [[Definition:Integer Multiplication|multiplication]] on the [[Definition:Set|set]] of [[Definition:Integer|integers]] $\Z$ is [[Definition:Commutative Operation|commutative]]: +:$\forall x, y \in \Z: x \times y = y \times x$ +\end{theorem} + +\begin{proof} +From the [[Definition:Integer/Formal Definition|formal definition of integers]], $\eqclass {a, b} {}$ is an [[Definition:Equivalence Class|equivalence class]] of [[Definition:Ordered Pair|ordered pairs]] of [[Definition:Natural Numbers|natural numbers]]. +Let $x = \eqclass {a, b} {}$ and $y = \eqclass {c, d} {}$ for some $x, y \in \Z$. +Then: +{{begin-eqn}} +{{eqn | l = x \times y + | r = \eqclass {a, b} {}\times \eqclass {c, d} {} + | c = {{Defof|Integer|subdef = Formal Definition}} +}} +{{eqn | r = \eqclass {a c + b d, a d + b c} {} + | c = {{Defof|Integer Multiplication}} +}} +{{eqn | r = \eqclass {c a + d b, d a + c b} {} + | c = [[Natural Number Multiplication is Commutative]] +}} +{{eqn | r = \eqclass {c a + d b, c b + d a} {} + | c = [[Natural Number Addition is Commutative]] +}} +{{eqn | r = \eqclass {c, d} {} \times \eqclass {a, b} {} + | c = {{Defof|Integer Multiplication}} +}} +{{eqn | r = y \times x + | c = {{Defof|Integer|subdef = Formal Definition}} +}} +{{end-eqn}} +{{qed}} +\end{proof}<|endoftext|> +\section{Integer Multiplication is Associative} +Tags: Integer Multiplication, Associativity + +\begin{theorem} +The operation of [[Definition:Integer Multiplication|multiplication]] on the [[Definition:Set|set]] of [[Definition:Integer|integers]] $\Z$ is [[Definition:Associative Operation|associative]]: +:$\forall x, y, z \in \Z: x \times \paren {y \times z} = \paren {x \times y} \times z$ +\end{theorem} + +\begin{proof} +From the [[Definition:Integer/Formal Definition|formal definition of integers]], $\eqclass {a, b} {}$ is an [[Definition:Equivalence Class|equivalence class]] of [[Definition:Ordered Pair|ordered pairs]] of [[Definition:Natural Numbers|natural numbers]]. +Let $x = \eqclass {a, b} {}$, $y = \eqclass {c, d} {}$ and $z = \eqclass {e, f} {}$ for some $x, y, z \in \Z$. +Then: +{{begin-eqn}} +{{eqn | l = x \times \paren {y \times z} + | r = \eqclass {a, b} {} \times \paren {\eqclass {c, d} {} \times \eqclass {e, f} {} } + | c = {{Defof|Integer|subdef = Formal Definition}} +}} +{{eqn | r = \eqclass {a, b} {} \times \eqclass {c e + d f, c f + d e} {} + | c = {{Defof|Integer Multiplication}} +}} +{{eqn | r = \eqclass {a \paren {c e + d f} + b \paren {c f + d e}, a \paren {c f + d e} + b \paren {c e + d f} } {} + | c = {{Defof|Integer Multiplication}} +}} +{{eqn | r = \eqclass {a c e + a d f + b c f + b d e, a c f + a d e + b c e + b d f} {} + | c = [[Natural Number Multiplication Distributes over Addition]] +}} +{{eqn | r = \eqclass {a c e + b d e + a d f + b c f, a c f + b d f + a d e + b c e} {} + | c = [[Natural Number Addition is Commutative]] and [[Natural Number Addition is Associative|Associative]] +}} +{{eqn | r = \eqclass {\paren {a c + b d} e + \paren {a d + b c} f, \paren {a c + b d} f + \paren {a d + b c} e} {} + | c = [[Natural Number Multiplication Distributes over Addition]] +}} +{{eqn | r = \eqclass {a c + b d, a d + b c} {} \times \eqclass {e, f} {} + | c = {{Defof|Integer Multiplication}} +}} +{{eqn | r = \paren {\eqclass {a, b} {} \times \eqclass {c, d} {} } \times \eqclass {e, f} {} + | c = {{Defof|Integer Multiplication}} +}} +{{eqn | r = \paren {x \times y} \times z + | c = {{Defof|Integer|subdef = Formal Definition}} +}} +{{end-eqn}} +{{qed}} +\end{proof}<|endoftext|> +\section{Integer Multiplication Distributes over Addition} +Tags: Integer Addition, Integer Multiplication, Distributive Operations + +\begin{theorem} +The operation of [[Definition:Integer Multiplication|multiplication]] on the [[Definition:Set|set]] of [[Definition:Integer|integers]] $\Z$ is [[Definition:Distributive Operation|distributive]] over [[Definition:Integer Addition|addition]]: +:$\forall x, y, z \in \Z: x \times \paren {y + z} = \paren {x \times y} + \paren {x \times z}$ +:$\forall x, y, z \in \Z: \paren {y + z} \times x = \paren {y \times x} + \paren {z \times x}$ +\end{theorem} + +\begin{proof} +Let us define $\Z$ as in the [[Definition:Integer/Formal Definition|formal definition of integers]]. +That is, $\Z$ is an [[Definition:Inverse Completion|inverse completion]] of $\N$. +From [[Natural Numbers form Commutative Semiring]], we have that: +:All [[Definition:Element|elements]] of $\N$ are [[Definition:Cancellable Element|cancellable]] for [[Definition:Natural Number Addition|addition]] +:[[Definition:Natural Number Addition|Addition]] and [[Definition:Natural Number Multiplication|multiplication]] are [[Definition:Commutative Operation|commutative]] and [[Definition:Associative Operation|associative]] on the [[Definition:Natural Numbers|natural numbers]] $\N$ +:[[Definition:Natural Number Multiplication|Natural number multiplication]] is [[Definition:Distributive Operation|distributive]] over [[Definition:Natural Number Addition|natural number addition]]. +The result follows from the [[Extension Theorem for Distributive Operations]]. +{{qed}} +\end{proof}<|endoftext|> +\section{Construction of Inverse Completion} +Tags: Inverse Completions + +\begin{theorem} +This page consists of a series of linked theorems, each of which builds towards one result. +To access the proofs for the individual theorems, click on the links which form the titles of each major section. +\end{theorem}<|endoftext|> +\section{Integer Multiplication Identity is One} +Tags: Integer Multiplication + +\begin{theorem} +The [[Definition:Identity Element|identity]] of [[Definition:Integer Multiplication|integer multiplication]] is $1$: +:$\exists 1 \in \Z: \forall a \in \Z: a \times 1 = a = 1 \times a$ +\end{theorem} + +\begin{proof} +Let us define $\eqclass {\tuple {a, b} } \boxtimes$ as in the [[Definition:Integer/Formal Definition|formal definition of integers]]. +That is, $\eqclass {\tuple {a, b} } \boxtimes$ is an [[Definition:Equivalence Class|equivalence class]] of [[Definition:Ordered Pair|ordered pairs]] of [[Definition:Natural Numbers|natural numbers]] under the [[Definition:Congruence Relation|congruence relation]] $\boxtimes$. +$\boxtimes$ is the [[Definition:Congruence Relation|congruence relation]] defined on $\N \times \N$ by $\tuple {x_1, y_1} \boxtimes \tuple {x_2, y_2} \iff x_1 + y_2 = x_2 + y_1$. +In order to streamline the notation, we will use $\eqclass {a, b} {}$ to mean $\eqclass {\tuple {a, b} } \boxtimes$, [[Definition:Integer/Formal Definition/Notation|as suggested]]. +From [[Construction of Inverse Completion|the method of construction]], $\eqclass {c + 1, c} {}$, where $c$ is any element of the [[Definition:Natural Numbers|natural numbers]] $\N$, is [[Definition:Isomorphic Copy|the isomorphic copy]] of $1 \in \N$. +To ease the algebra, we will take $\eqclass {1, 0} {}$ as a canonical instance of this [[Definition:Equivalence Class|equivalence class]]. +Thus it is to be shown that: +:$\forall a, b \in \N: \eqclass {a, b} {} \times \eqclass {1, 0} {} = \eqclass {a, b} {} = \eqclass {1, 0} {} \times \eqclass {a, b} {}$ +From [[Natural Numbers form Commutative Semiring]], we take it for granted that: +:[[Definition:Natural Number Addition|addition]] and [[Definition:Natural Number Multiplication|multiplication]] are [[Definition:Commutative Operation|commutative]] and [[Definition:Associative Operation|associative]] on the [[Definition:Natural Numbers|natural numbers]] $\N$ +:[[Definition:Natural Number Multiplication|natural number multiplication]] is [[Definition:Distributive Operation|distributive]] over [[Definition:Natural Number Addition|natural number addition]]. +So: +{{begin-eqn}} +{{eqn | l = \eqclass {a, b} {} \times \eqclass {1, 0} {} + | r = \eqclass {1 \times a + 0 \times b, 0 \times a + 1 \times b} {} + | c = +}} +{{eqn | r = \eqclass {a, b} {} + | c = from [[Construction of Inverse Completion/Equivalence Relation/Members of Equivalence Classes|Construction of Inverse Completion: Members of Equivalence Classes]] +}} +{{end-eqn}} +So: +:$\eqclass {a, b} {} \times \eqclass {1, 0} {} = \eqclass {a, b} {}$ +The identity $\eqclass {a, b} {} = \eqclass {1, 0} {} \times \eqclass {a, b} {}$ is demonstrated similarly. +{{qed}} +\end{proof}<|endoftext|> +\section{Integer Multiplication has Zero} +Tags: Integer Multiplication + +\begin{theorem} +The [[Definition:Set|set]] of [[Definition:Integer|integers]] under [[Definition:Integer Multiplication|multiplication]] $\struct {\Z, \times}$ has a [[Definition:Zero Element|zero element]], which is $0$. +\end{theorem} + +\begin{proof} +Let us define $\eqclass {\tuple {a, b} } \boxminus$ as in the [[Definition:Integer/Formal Definition|formal definition of integers]]. +That is, $\eqclass {\tuple {a, b} } \boxminus$ is an [[Definition:Equivalence Class|equivalence class]] of [[Definition:Ordered Pair|ordered pairs]] of [[Definition:Natural Numbers|natural numbers]] under the [[Definition:Congruence Relation|congruence relation]] $\boxminus$. +$\boxminus$ is the [[Definition:Congruence Relation|congruence relation]] defined on $\N \times \N$ by $\tuple {x_1, y_1} \boxminus \tuple {x_2, y_2} \iff x_1 + y_2 = x_2 + y_1$. +In order to streamline the notation, we will use $\eqclass {a, b} {}$ to mean $\eqclass {\tuple {a, b} } \boxminus$, [[Definition:Integer/Formal Definition/Notation|as suggested]]. +From [[Construction of Inverse Completion/Identity of Quotient Structure|the method of construction]], $\eqclass {c, c} {}$, where $c$ is any element of the [[Definition:Natural Numbers|natural numbers]] $\N$, is the [[Definition:Identity (Abstract Algebra)|identity]] of $\struct {\Z, +}$. +To ease the algebra, we will take $\eqclass {0, 0} {}$ as a canonical instance of this equivalence class. +We need to show that: +$\forall a, b, c \in \N: \eqclass {a, b} {} \times \eqclass {0, 0} {} = \eqclass {0, 0} {} = \eqclass {0, 0} {} \times \eqclass {a, b} {}$. +From [[Natural Numbers form Commutative Semiring]], we can take it for granted that [[Definition:Natural Number Addition|addition]] and [[Definition:Natural Number Multiplication|multiplication]] are [[Definition:Commutative Operation|commutative]] on the [[Definition:Natural Numbers|natural numbers]] $\N$. +{{begin-eqn}} +{{eqn | l = \eqclass {a, b} {} \times \eqclass {0, 0} {} + | r = \eqclass {a \times 0 + b \times 0, a \times 0 + b \times 0} {} + | c = +}} +{{eqn | r = \eqclass {0, 0} {} + | c = [[Construction of Inverse Completion/Equivalence Relation/Equivalence Class of Equal Elements|Construction of Inverse Completion: Equivalence Class of Equal Elements]] +}} +{{eqn | r = \eqclass {0 \times a + 0 \times b, 0 \times a + 0 \times b} {} + | c = +}} +{{eqn | r = \eqclass {0, 0} {} \times \eqclass {a, b} {} + | c = +}} +{{end-eqn}} +{{qed}} +\end{proof}<|endoftext|> +\section{Ring of Integers has no Zero Divisors} +Tags: Integers + +\begin{theorem} +The [[Definition:Integer|integers]] have no [[Definition:Zero Divisor of Ring|zero divisors]]: +:$\forall x, y, \in \Z: x \times y = 0 \implies x = 0 \lor y = 0$ +\end{theorem} + +\begin{proof} +Let us define $\eqclass {\tuple {a, b} } \boxminus$ as in the [[Definition:Integer/Formal Definition|formal definition of integers]]. +That is, $\eqclass {\tuple {a, b} } \boxminus$ is an [[Definition:Equivalence Class|equivalence class]] of [[Definition:Ordered Pair|ordered pairs]] of [[Definition:Natural Numbers|natural numbers]] under the [[Definition:Congruence Relation|congruence relation]] $\boxminus$. +$\boxminus$ is the [[Definition:Congruence Relation|congruence relation]] defined on $\N \times \N$ by $\tuple {x_1, y_1} \boxminus \tuple {x_2, y_2} \iff x_1 + y_2 = x_2 + y_1$. +In order to streamline the notation, we will use $\eqclass {a, b} {}$ to mean $\eqclass {\tuple {a, b} } \boxminus$, [[Definition:Integer/Formal Definition/Notation|as suggested]]. +From [[Construction of Inverse Completion/Identity of Quotient Structure|the method of construction]], $\eqclass {c, c} {}$, where $c$ is any element of the [[Definition:Natural Numbers|natural numbers]] $\N$, is the [[Definition:Identity Element|identity]] of $\struct {\Z, +}$. +To ease the algebra, we will take $\eqclass {0, 0} {}$ as a canonical instance of this [[Definition:Equivalence Class|equivalence class]]. +We need to show that: +:$\forall a, b, c, d \in \N: \eqclass {a, b} {} \times \eqclass {c, d} {} = \eqclass {0, 0} {} \implies \eqclass {a, b} {} = \eqclass {0, 0} {} \lor \eqclass {c, d} {} = \eqclass {0, 0} {}$ +From [[Natural Numbers form Commutative Semiring]], we can take it for granted that: +:[[Definition:Natural Number Addition|addition]] and [[Definition:Natural Number Multiplication|multiplication]] are [[Definition:Commutative Operation|commutative]] and [[Definition:Associative Operation|associative]] on the [[Definition:Natural Numbers|natural numbers]] $\N$ +:[[Definition:Natural Number Multiplication|natural number multiplication]] is [[Definition:Distributive Operation|distributive]] over [[Definition:Natural Number Addition|natural number addition]]. +So: +{{begin-eqn}} +{{eqn | l = \eqclass {a, b} {} \times \eqclass {c, d} {} + | r = \eqclass {0, 0} {} + | c = +}} +{{eqn | ll= \leadsto + | l = \eqclass {a c + b d, a d + b c} {} + | r = \eqclass {0, 0} {} + | c = +}} +{{eqn | ll= \leadsto + | l = a c + b d + 0 + | r = a d + b c + 0 + | c = +}} +{{eqn | ll= \leadsto + | l = a c + b d + | r = a d + b c + | c = +}} +{{end-eqn}} +We have to be careful here, and bear in mind that $a, b, c, d$ are [[Definition:Natural Numbers|natural numbers]], and we have not defined (and, at this stage, will not define) subtraction on such entities. +{{WLOG}}, suppose that $\eqclass {c, d} {} \ne \eqclass {0, 0} {}$. +Then $c \ne d$. +{{WLOG}}, suppose also that $c > d$. +{{wtd|[[Ordering in terms of Addition]] should have a version for $\N$}} +From [[Ordering in terms of Addition]], $\exists p \in \N: d + p = c$ where $p > 0$. +Then: +{{begin-eqn}} +{{eqn | l = a c + b d + | r = a d + b c + | c = +}} +{{eqn | ll= \leadsto + | l = a \paren {d + p} + b d + | r = a d + b \paren {d + p} + | c = +}} +{{eqn | ll= \leadsto + | l = a d + a p + b d + | r = a d + b d + b p + | c = +}} +{{eqn | ll= \leadsto + | l = a p + | r = b p + | c = +}} +{{eqn | ll= \leadsto + | l = a + | r = b + | c = +}} +{{eqn | ll= \leadsto + | l = \eqclass {a, b} {} + | r = \eqclass {0, 0} {} + | c = [[Construction of Inverse Completion/Equivalence Relation/Equivalence Class of Equal Elements|Construction of Inverse Completion: Equivalence Class of Equal Elements]] +}} +{{end-eqn}} +Similarly for when $c < d$. +Thus: +:$\eqclass {c, d} {} \ne \eqclass {0, 0} {} \implies \eqclass {a, b} {} = \eqclass {0, 0} {}$ +A similar argument shows that: +:$\eqclass {a, b} {} \ne \eqclass {0, 0} {} \implies \eqclass {c, d} {} = \eqclass {0, 0} {}$ +The equivalence between the two forms of the statement of this theorem follows from [[De Morgan's Laws (Logic)/Conjunction of Negations|De Morgan's Laws: Conjunction of Negations]] and the [[Rule of Transposition]]. +{{qed}} +\end{proof}<|endoftext|> +\section{Integers form Integral Domain} +Tags: Integers, Integral Domains + +\begin{theorem} +The [[Definition:Integer|integers]] $\Z$ form an [[Definition:Integral Domain|integral domain]] under [[Definition:Integer Addition|addition]] and [[Definition:Integer Multiplication|multiplication]]. +\end{theorem} + +\begin{proof} +First we note that the [[Integers form Commutative Ring with Unity|integers form a commutative ring with unity]] whose [[Integer Multiplication has Zero|zero is $0$]] and whose [[Integer Multiplication Identity is One|unity is $1$]]. +Next we see that the [[Ring of Integers has no Zero Divisors|$\struct {\Z, +, \times}$ has no divisors of zero]]. +So, by definition, the [[Definition:Algebraic Structure|algebraic structure]] $\struct {\Z, +, \times}$ is an [[Definition:Integral Domain|integral domain]] whose [[Definition:Ring Zero|zero]] is $0$ and whose [[Definition:Unity of Ring|unity]] is $1$. +{{Qed}} +\end{proof}<|endoftext|> +\section{Positive Integer is Well-Defined} +Tags: Integers + +\begin{theorem} +"[[Definition:Positive Integer|Positive]]" as applied to an [[Definition:Integer|integer]] is [[Definition:Well-Defined Relation|well-defined]]. +\end{theorem} + +\begin{proof} +Let us define $\eqclass {\tuple {a, b} } \boxminus$ as in the [[Definition:Integer/Formal Definition|formal definition of integers]]. +That is, $\eqclass {\tuple {a, b} } \boxminus$ is an [[Definition:Equivalence Class|equivalence class]] of [[Definition:Ordered Pair|ordered pairs]] of [[Definition:Natural Numbers|natural numbers]] under the [[Definition:Congruence Relation|congruence relation]] $\boxminus$. +$\boxminus$ is the [[Definition:Congruence Relation|congruence relation]] defined on $\N \times \N$ by $\tuple {x_1, y_1} \boxminus \tuple {x_2, y_2} \iff x_1 + y_2 = x_2 + y_1$. +In order to streamline the notation, we will use $\eqclass {a, b} {}$ to mean $\eqclass {\tuple {a, b} } \boxminus$, [[Definition:Integer/Formal Definition/Notation|as suggested]]. +Thus, what we are trying to prove is: +:$\eqclass {a, b} {} = \eqclass {c, d} {} \land b < a \implies d < c$ +By definition: +:$\eqclass {a, b} {} = \eqclass {c, d} {} \iff a + d = b + c$ +So: +{{begin-eqn}} +{{eqn | l = b + | o = < + | r = a +}} +{{eqn | ll= \leadsto + | l = \exists p \in \N: a + | r = b + p +}} +{{eqn | ll= \leadsto + | l = b + p + d + | r = b + c +}} +{{eqn | ll= \leadsto + | l = p + d + | r = c +}} +{{eqn | ll= \leadsto + | l = d + | o = < + | r = c +}} +{{end-eqn}} +{{qed}} +[[Category:Integers]] +tfoqxrhyoxppc1d4o3kos1sbvkcow8n +\end{proof}<|endoftext|> +\section{Natural Numbers are Non-Negative Integers} +Tags: Integers, Natural Numbers, Natural Numbers are Non-Negative Integers + +\begin{theorem} +Let $m \in \Z$. Then: +:$(1): \quad m \in \N \iff 0 \le m$ +:$(2): \quad m \in \N_{> 0} \iff 0 < m$ +:$(3): \quad m \notin \N \iff -m \in \N_{> 0}$ +That is, the [[Definition:Natural Numbers|natural numbers]] are precisely those [[Definition:Integer|integers]] which are greater than or equal to [[Definition:Zero (Algebra)|zero]]. +\end{theorem} + +\begin{proof} +Consider the [[Definition:Integer/Formal Definition|formal definition of the integers]]: $x = \eqclass {a, b} {}$ is an [[Definition:Equivalence Class|equivalence class]] of [[Definition:Ordered Pair|ordered pairs]] of [[Definition:Natural Numbers|natural numbers]]. +Let $x \in \Z_{>0}$ be a [[Definition:Strictly Positive Integer|(strictly) positive integer]]. +Thus by definition: +:$x > 0$ +This is equivalent to the condition that $a > b$. +Hence: +:$x = \eqclass {b + u, b} {}$ +for some $u \in \N_{>0}$. +It is immediate that: +:$\forall c \in \N: \eqclass {b + u, b} {} = \eqclass {c + u, c} {}$ +Consider the [[Definition:Mapping|mapping]] $\phi: \N_{>0} \to \Z_{>0}$ defined as: +:$\forall u \in \N_{>0}: \map \phi u = u'$ +where $u' \in \Z$ be the [[Definition:Strictly Positive Integer|(strictly) positive integer]] $\eqclass {b + u, b} {}$. +From the above we have that $\phi$ is [[Definition:Well-Defined Mapping|well-defined]]. +Let $\eqclass {b + u, b} {} = \eqclass {c + v, c} {}$. +Then: +:$b + u + c = b + c + v$ +and so: +:$u = v$ +Hence $\phi$ is an [[Definition:Injection|injection]]. +Next note that all $u' \in \Z_{>0}$ can be expressed in the form: +:$u' = \eqclass {b + u, b} {}$ +for arbitrary $b$. +Hence $\phi$ is a [[Definition:Surjection|surjection]]. +Hence $\phi$ is an [[Definition:Isomorphism|isomorphism]]. +By defining $\map \phi 0 = \eqclass {b, b} {}$ we see that $\N$ and $\Z_{\ge 0}$ are [[Definition:Isomorphism|isomorphic]]. +{{qed}} +\end{proof}<|endoftext|> +\section{Subtraction on Integers is Extension of Natural Numbers} +Tags: Integers, Subtraction + +\begin{theorem} +[[Definition:Integer Subtraction|Integer subtraction]] is an extension of the definition of [[Definition:Natural Number Subtraction|subtraction]] on the [[Definition:Natural Numbers|natural numbers]]. +\end{theorem} + +\begin{proof} +Let $m, n \in \N: m \le n$. +From [[Definition:Natural Number Subtraction|natural number subtraction]], $\exists p \in \N: m + p = n$ such that $n - m = p$. +As $m, n, p \in \N$, it follows that $m, n, p \in \Z$ as well. +However, as $\Z$ is the [[Definition:Inverse Completion|inverse completion]] of $\N$, it follows that $-m \in \Z$ as well, so it makes sense to express the following: +{{begin-eqn}} +{{eqn | l = \paren {n + \paren {-m} } + m + | r = n + \paren {\paren {-m} + m} + | c = +}} +{{eqn | r = n + | c = +}} +{{eqn | r = p + m + | c = +}} +{{eqn | r = \paren {n - m} + m + | c = +}} +{{end-eqn}} +Thus, as all elements of $\Z$ are [[Definition:Cancellable Element|cancellable]], it follows that $n + \paren {-m} = n - m$. +So: +: $\forall m, n \in \Z, m \le n: n + \paren {-m} = n - m = n -_\N m$ +and the result follows. +{{qed}} +[[Category:Integers]] +[[Category:Subtraction]] +7wy2x7ny7d5h653ngmzazw9eqd8qcub +\end{proof}<|endoftext|> +\section{Non-Zero Integers are Cancellable for Multiplication} +Tags: Integers, Non-Zero Integers are Cancellable for Multiplication + +\begin{theorem} +Every non-zero [[Definition:Integer|integer]] is [[Definition:Cancellable Element|cancellable]] for [[Definition:Integer Multiplication|multiplication]]. +That is: +:$\forall x, y, z \in \Z, x \ne 0: x y = x z \iff y = z$ +\end{theorem} + +\begin{proof} +Let $x y = x z$. +There are two cases to investigate: $x > 0$ and $x < 0$. +Let $x > 0$. +From [[Natural Numbers are Non-Negative Integers]], $x \in \N_{> 0}$. +By the [[Extension Theorem for Distributive Operations]] and [[Ordering on Natural Numbers is Compatible with Multiplication]], $x$ is [[Definition:Cancellable Element|cancellable]] for multiplication. +{{qed|lemma} +Let $x < 0$. +We know that the [[Integers form Integral Domain]] and are thus a [[Definition:Ring (Abstract Algebra)|ring]]. +Then $-x > 0$ and so: +{{begin-eqn}} +{{eqn | l = x y + | r = x z + | c = +}} +{{eqn | ll= \leadsto + | l = \paren {-\paren {-x} } y + | r = \paren {-\paren {-x} } z + | c = [[Negative of Ring Negative]] +}} +{{eqn | ll= \leadsto + | l = -\paren {\paren {-x} y} + | r = -\paren {\paren {-x} z} + | c = [[Product with Ring Negative]] +}} +{{eqn | ll= \leadsto + | l = -y + | r = -z + | c = as $-x$ is [[Definition:Strictly Positive Integer|(strictly) positive]], the above result holds +}} +{{eqn | ll= \leadsto + | l = y + | r = z + | c = $\struct {\Z, +}$ is a [[Definition:Group|group]]: {{GroupAxiom|3}} +}} +{{end-eqn}} +{{qed|lemma}} +So whatever non-zero value $x$ takes, it is [[Definition:Cancellable Element|cancellable]] for multiplication. +{{Qed}} +\end{proof} + +\begin{proof} +Let $y, z \in \Z: y \ne z$. +{{begin-eqn}} +{{eqn | l = y + | o = \ne + | r = z + | c = +}} +{{eqn | ll= \leadstoandfrom + | l = y - z + | o = \ne + | r = 0 + | c = +}} +{{eqn | ll= \leadstoandfrom + | l = x \paren {y - z} + | o = \ne + | r = 0 + | c = [[Ring of Integers has no Zero Divisors]] +}} +{{eqn | ll= \leadstoandfrom + | l = x y - x z + | o = \ne + | r = 0 + | c = [[Integer Multiplication Distributes over Subtraction]] +}} +{{end-eqn}} +The result follows by [[Rule of Transposition|transposition]]. +{{qed}} +\end{proof} + +\begin{proof} +Let $x y = x z$. +There are two cases to investigate: $x > 0$ and $x < 0$. +Let $x > 0$. +From [[Natural Numbers are Non-Negative Integers]], $x \in \N_{> 0}$. +By the [[Extension Theorem for Distributive Operations]] and [[Ordering on Natural Numbers is Compatible with Multiplication]], $x$ is [[Definition:Cancellable Element|cancellable]] for [[Definition:Integer Multiplication|multiplication]]. +{{qed|lemma}} +Let $x < 0$. +We know that the [[Integers form Integral Domain]] and are thus a [[Definition:Ring (Abstract Algebra)|ring]]. +Then $-x > 0$ and so: +{{begin-eqn}} +{{eqn | l = \paren {-x} y + | r = -\paren {x y} + | c = [[Product with Ring Negative]] +}} +{{eqn | r = -\paren {x z} + | c = $\struct {\Z, +}$ is a [[Definition:Group|group]]: {{GroupAxiom|3}} +}} +{{eqn | r = \paren {-x} z + | c = [[Product with Ring Negative]] +}} +{{eqn | ll= \leadsto + | l = y + | r = z + | c = from above: case where $x > 0$ +}} +{{end-eqn}} +{{qed|lemma}} +So whatever non-zero value $x$ takes, it is [[Definition:Cancellable Element|cancellable]] for [[Definition:Integer Multiplication|multiplication]]. +{{Qed}} +\end{proof}<|endoftext|> +\section{Multiplicative Ordering on Integers} +Tags: Integer Multiplication, Orderings on Integers + +\begin{theorem} +Let $x, y, z \in \Z$ such that $z > 0$. +Then: +:$x < y \iff z x < z y$ +:$x \le y \iff z x \le z y$ +\end{theorem} + +\begin{proof} +Let $z > 0$. +Let $M_z: \Z \to \Z$ be the mapping defined as: +:$\forall x \in \Z: \map {M_z} x = z x$ +All we need to do is show that $M_z$ is an [[Definition:Order Embedding|order embedding]] from $\struct {\Z, +, \le}$ to itself. +By [[Monomorphism from Total Ordering]], we just need to show that: +:$x < y \implies z x < z y$ +If $x < y$, then $0 < y - x$, so $z \in \N$ and $y - x \in \N$ by [[Natural Numbers are Non-Negative Integers]]. +Thus by [[Ordering on Natural Numbers is Compatible with Multiplication]]: +:$z \paren {y - x} \in \N$ +Therefore +:$0 < z \paren {y - x} = z y - z x$ +That is: +:$z x < z y$ +{{qed}} +\end{proof}<|endoftext|> +\section{Invertible Integers under Multiplication} +Tags: Integer Multiplication, Units of Rings + +\begin{theorem} +The only [[Definition:Invertible Element|invertible elements]] of $\Z$ for [[Definition:Integer Multiplication|multiplication]] (that is, [[Definition:Unit of Ring|units of $\Z$]]) are $1$ and $-1$. +\end{theorem} + +\begin{proof} +Let $x > 0$ and $x y > 0$. +Suppose $y \le 0$. +Then from [[Multiplicative Ordering on Integers]] and [[Ring Product with Zero]]: +:$x y \le x \, 0 = 0$ +From this contradiction we deduce that $y > 0$. +Now, if we have $x > 0$ and $x y = 1$, then $y > 0$ and hence $y \in \N$ by [[Natural Numbers are Non-Negative Integers]]. +Hence by [[Invertible Elements under Natural Number Multiplication]] it follows that $x = 1$. +Thus $1$ is the only element of $\N$ that is [[Definition:Invertible Element|invertible]] for [[Definition:Natural Number Multiplication|multiplication]]. +Therefore by [[Natural Numbers are Non-Negative Integers]] and [[Product with Ring Negative]], the result follows. +{{qed}} +\end{proof}<|endoftext|> +\section{Index Laws for Monoids} +Tags: Monoids + +\begin{theorem} +These results are an extension of the results in [[Index Laws for Semigroup]] in which the [[Definition:Domain of Mapping|domain]] of the indices is extended to include all [[Definition:Integer|integers]]. +Let $\struct {S, \circ}$ be a [[Definition:Monoid|monoid]] whose [[Definition:Identity Element|identity]] is $e$. +Let $a \in S$ be [[Definition:Invertible Element|invertible]] for $\circ$. +Let $n \in \N$. +Let $a^n$ be the [[Definition:Power of Element/Monoid|$n$th power of $a$]]: +:$a^n = \begin{cases} +e : & n = 0 \\ +a^{n - 1} \circ a : & n > 0 \\ +\paren {a^{-n}}^{-1} : & n < 0 +\end{cases}$ +Then we have the following results: +\end{theorem}<|endoftext|> +\section{Totally Ordered Abelian Group Isomorphism} +Tags: Isomorphisms, Abelian Groups + +\begin{theorem} +Let $\left({\Z', +', \le'}\right)$ be a [[Definition:Totally Ordered Structure|totally ordered]] [[Definition:Abelian Group|abelian group]]. +Let $0'$ be the [[Definition:Identity Element|identity]] of $\left({\Z', +', \le'}\right)$. +Let $\N' = \left\{{x \in \Z': x \ge' 0'}\right\}$. +Let $\Z'$ contain at least two elements. +Let $\N'$ be [[Definition:Well-Ordered Set|well-ordered]] for the ordering induced on $\N'$ by $\le'$. +Then the [[Definition:Mapping|mapping]] $g: \Z \to \Z'$ defined by: +:$\forall n \in \Z: g \left({n}\right) = \left({+'}\right)^n 1'$ +is an [[Definition:Group Isomorphism|isomorphism]] from $\left({\Z, +, \le}\right)$ onto $\left({\Z', +', \le'}\right)$, where $1'$ is the [[Definition:Smallest Element|smallest element]] of $\N' \setminus \left\{{0'}\right\}$. +\end{theorem} + +\begin{proof} +First we establish that $g$ is a [[Definition:Homomorphism (Abstract Algebra)|homomorphism]]. +Suppose $z \in \Z'$ such that $z \ne 0'$. +Then by [[Ordering of Inverses in Ordered Monoid]], either $z >' 0'$ or $-z >' 0'$. +Thus either: +:$z \in \N' \setminus \left\{{0'}\right\}$ +or: +:$-z \in \N' \setminus \left\{{0'}\right\}$ +and thus $\N' \setminus \left\{{0'}\right\}$ is not [[Definition:Empty Set|empty]]. +Therefore $\N' \setminus \left\{{0'}\right\}$ has a [[Definition:Minimal Element|minimal element]]. +Call this [[Definition:Minimal Element|minimal element]] $1'$. +It is clear that $\N'$ is an ordered [[Definition:Semigroup|semigroup]] satisfying $(NO 1)$, $(NO 2)$ and $(NO 4)$ of the [[Definition:Naturally Ordered Semigroup Axioms|naturally ordered semigroup axioms]]. +Also: +{{begin-eqn}} +{{eqn | o = + | r = 0' \le' x \le' y + | c = +}} +{{eqn | o = \implies + | r = 0' \le' y - x + | c = +}} +{{eqn | o = \implies + | r = y - x \in \N' \land x +' \left({y - x}\right) = y + | c = +}} +{{end-eqn}} +Thus $\N'$ also satisfies $(NO 3)$ of the [[Definition:Naturally Ordered Semigroup Axioms|naturally ordered semigroup axioms]]. +So $\left({\N', +', \le'}\right)$ is a [[Definition:Naturally Ordered Semigroup|naturally ordered semigroup]]. +So, by [[Naturally Ordered Semigroup is Unique]], the [[Definition:Restriction of Mapping|restriction]] to $\N$ of $g$ is an [[Definition:Group Isomorphism|isomorphism]] from $\left({\N, +, \le}\right)$ to $\left({\N', +', \le'}\right)$. +By [[Index Laws for Monoids/Sum of Indices|Index Law for Sum of Indices]], $g$ is a [[Definition:Homomorphism (Abstract Algebra)|homomorphism]] from $\left({\Z, +}\right)$ into $\left({\Z', +'}\right)$. +Next it is established that $g$ is [[Definition:Surjection|surjective]]. +Let $y \in \Z': y <' 0'$. +{{begin-eqn}} +{{eqn | o = + | r = y <' 0' + | c = +}} +{{eqn | o = \implies + | r = -y >' 0' + | c = +}} +{{eqn | o = \implies + | r = \exists n \in \N: -y = g \left({n}\right) + | c = +}} +{{eqn | o = \implies + | r = y = - g \left({n}\right) = g \left({-n}\right) + | c = [[Homomorphism with Identity Preserves Inverses]] +}} +{{end-eqn}} +Therefore $g$ is a [[Definition:Surjection|surjection]]. +Now to show that $g$ is a [[Definition:Group Monomorphism|monomorphism]], that is, it is [[Definition:Injection|injective]]. +Let $n < m$. +{{begin-eqn}} +{{eqn | o = + | r = n < m + | c = +}} +{{eqn | o = \implies + | r = m - n \in \N_{>0} + | c = +}} +{{eqn | o = \implies + | r = g \left({m}\right) - g \left({n}\right) - g \left({m - n}\right) \in \N' \setminus \left\{ {0'}\right\} + | c = +}} +{{eqn | o = \implies + | r = g \left({n}\right) <' \left({g \left({m}\right) - g \left({n}\right)}\right) +' g \left({n}\right) = g \left({m}\right) + | c = [[Strict Ordering Preserved under Product with Cancellable Element]] +}} +{{end-eqn}} +Therefore it can be seen that $g$ is [[Definition:Strictly Increasing|strictly increasing]]. +It follows from [[Monomorphism from Total Ordering]] that $g$ is a [[Definition:Group Monomorphism|monomorphism]] from $\left({\Z, +, \le}\right)$ to $\left({\Z', +', \le'}\right)$. +A [[Definition:Surjection|surjective]] [[Definition:Group Monomorphism|monomorphism]] is an [[Definition:Group Isomorphism|isomorphism]], and the result follows. +{{Qed}} +\end{proof}<|endoftext|> +\section{Integers under Addition form Totally Ordered Group} +Tags: Integer Addition, Additive Group of Integers + +\begin{theorem} +Then the [[Definition:Ordered Structure|ordered structure]] $\struct {\Z, +, \le}$ is a [[Definition:Totally Ordered Group|totally ordered group]]. +\end{theorem} + +\begin{proof} +=== $\struct {\Z, +, \le}$ is an Ordered Structure === +==== $(1)$ ==== +By [[Integer Addition is Closed]], $\struct {\Z, +}$ is an [[Definition:Algebraic Structure|algebraic structure]]. +==== $(2)$ ==== +$\le$ is an [[Definition:Ordering|ordering]] on $\Z$. +Thus, $\struct {\Z, \le}$ is an [[Definition:Ordered Set|ordered set]]. +==== $(3)$ ==== +By [[Ordering is Preserved on Integers by Addition]] and [[Integer Addition is Commutative]], $\le$ is [[Definition:Relation Compatible with Operation|compatible]] with $+$. +Thus, $\struct {\Z, +, \le}$ is an [[Definition:Ordered Structure|ordered structure]]. +{{qed|lemma}} +=== $\struct {\Z, +, \le}$ is a Totally Ordered Structure === +By definition, the [[Definition:Ordered Structure|ordered structure]] $\struct {\Z, +, \le}$ is a [[Definition:Totally Ordered Structure|totally ordered structure]] {{iff}} $\le$ is a [[Definition:Total Ordering|total ordering]]. +This follows from [[Ordering on Integers is Total Ordering]]. +{{qed|lemma}} +=== $\struct {\Z, +, \le}$ is a Totally Ordered Group === +By definition, the [[Definition:Totally Ordered Structure|totally ordered structure]] $\struct {\Z, +, \le}$ is a [[Definition:Totally Ordered Group|totally ordered group]] {{iff}} $\struct {\Z, +}$ is a [[Definition:Group|group]]. +This follows from [[Integers under Addition form Abelian Group]]. +{{qed}} +[[Category:Integer Addition]] +[[Category:Additive Group of Integers]] +15qysqr4r51uk7597b8j8vjkp15ng6w +\end{proof}<|endoftext|> +\section{Integers form Totally Ordered Ring} +Tags: Integers, Totally Ordered Rings + +\begin{theorem} +The [[Definition:Algebraic Structure|structure]] $\struct {\Z, +, \times, \le}$ is a [[Definition:Totally Ordered Ring|totally ordered ring]]. +\end{theorem} + +\begin{proof} +From [[Integers form Commutative Ring]], $\struct {\Z, +, \times}$ is a [[Definition:Commutative Ring|commutative ring]]. +We need to show that $\le$ is a [[Definition:Ordering Compatible with Ring Structure|compatible ordering]] on $\Z$. +That is, that: +:$(1): \quad \le$ is [[Definition:Relation Compatible with Operation|compatible]] with $+$ +:$(2): \quad \forall x, y \in \Z: 0 \le x, 0 \le y \implies 0 \le x \times y$ +The first one follows from the fact that [[Integers under Addition form Totally Ordered Group]]. +Now, from [[Multiplicative Ordering on Integers]], we have: +:$\forall z, x, y \in \Z, 0 < y: z \le x \iff y \times z \le y \times x$ +Let $z = 0, 0 \le x$. +Then: +:$0 = x \times z \le x \times y = y \times x$ +So if $y \ne 0$, the condition $0 \le x, 0 \le y \implies 0 \le x \times y$ holds. +Now if $y = 0$, we have $x \times y = 0$ and thus $0 \le x \times y$. +Thus $\le$ is [[Definition:Relation Compatible with Operation|compatible]] with $\Z$, and the result follows. +{{qed}} +\end{proof}<|endoftext|> +\section{Congruences on Rational Numbers} +Tags: Congruence Relations, Rational Numbers + +\begin{theorem} +There are only two [[Definition:Congruence Relation|congruence relations]] on the [[Definition:Field (Abstract Algebra)|field]] of [[Definition:Rational Number|rational numbers]] $\left({\Q, +, \times}\right)$: +:$(1): \quad$ The [[Definition:Diagonal Relation|diagonal relation]] $\Delta_\Q$ +:$(2): \quad$ The [[Definition:Trivial Relation|trivial relation]] $\Q \times \Q$. +\end{theorem} + +\begin{proof} +From: +:[[Diagonal Relation is Universally Compatible]] and +:[[Trivial Relation is Universally Congruent]] +we know that both these [[Definition:Relation|relations]] are [[Definition:Relation Compatible with Operation|compatible]] with both [[Definition:Rational Addition|addition]] and [[Definition:Rational Multiplication|multiplication]] on $\Q$. +Now we need to show that these are the ''only'' such [[Definition:Relation|relations]]. +Let $\RR$ be a [[Definition:Congruence Relation|congruence]] on $\Q$, such that $\RR \ne \Delta_\Q$. +{{begin-eqn}} +{{eqn | l = \RR + | o = \ne + | r = \Delta_\Q + | c = +}} +{{eqn | ll= \leadsto + | l = \exists r, s \in \Q: r + | o = \ne + | r = s \land r \RR s + | c = +}} +{{eqn | ll= \leadsto + | l = \exists h \in \Q, h \ne 0: h + | r = r - s + | c = +}} +{{eqn | ll= \leadsto + | l = h + | o = \RR + | r = 0 + | c = as $\RR$ is a [[Definition:Congruence Relation|congruence relation]] compatible with $+$ +}} +{{end-eqn}} +Then: +{{begin-eqn}} +{{eqn | l = \forall x \in \Q: \paren {x / h} + | o = \RR + | r = \paren {x / h} + | c = +}} +{{eqn | ll= \leadsto + | l = \paren {\paren {x / h} \times h} + | o = \RR + | r = \paren {\paren {x / h} \times 0} + | c = as $\RR$ is a [[Definition:Congruence Relation|congruence relation]] compatible with $\times$ +}} +{{eqn | ll= \leadsto + | l = x + | o = \RR + | r = 0 + | c = +}} +{{eqn | ll= \leadsto + | l = \eqclass 0 \RR + | r = \Q + | c = +}} +{{eqn | ll= \leadsto + | l = \RR + | r = \Q \times \Q + | c = +}} +{{end-eqn}} +{{qed}} +\end{proof}<|endoftext|> +\section{Number of Elements in Partition} +Tags: Combinatorics + +\begin{theorem} +Let $S$ be a [[Definition:Set|set]]. +Let there be a [[Definition:Set Partition|partition]] on $S$ of $n$ [[Definition:Subset|subsets]], each of which has $m$ [[Definition:Element|elements]]. +Then: +:$\card S = n m$ +\end{theorem} + +\begin{proof} +Let the [[Definition:Set Partition|partition]] of $S$ be $S_1, S_2, \ldots, S_n$. +Then: +:$\forall k \in \set {1, 2, \ldots, n}: \card {S_k} = m$ +By definition of [[Definition:Integer Multiplication|multiplication]]: +:$\displaystyle \sum_{k \mathop = 1}^n \card {S_k} = n m$ +and the result follows from the [[Fundamental Principle of Counting]]. +{{qed}} +\end{proof}<|endoftext|> +\section{Cardinality of Complement} +Tags: Combinatorics, Set Difference, Cardinality + +\begin{theorem} +Let $T \subseteq S$ such that $\card S = n, \card T = m$. +Then: +:$\card {\relcomp S T} = \card {S \setminus T} = n - m$ +where: +:$\relcomp S T$ denotes the [[Definition:Relative Complement|complement of $T$ relative to $S$]] +:$S \setminus T$ denotes the [[Definition:Set Difference|difference between $S$ and $T$]]. +\end{theorem} + +\begin{proof} +The result is obvious for $S = T$ or $T = \O$. +Otherwise, $\set {T, S \setminus T}$ is a [[Definition:Set Partition|partition]] of $S$. +Let $\card {S \setminus T} = p$. +Then by the [[Fundamental Principle of Counting]]: +: $m + p = n$ +and the result follows. +{{qed}} +\end{proof}<|endoftext|> +\section{Cardinality of Cartesian Product} +Tags: Combinatorics, Cartesian Product, Cardinality, Cardinality of Cartesian Product + +\begin{theorem} +Let $S \times T$ be the [[Definition:Cartesian Product|cartesian product]] of two [[Definition:Finite Set|finite sets]] $S$ and $T$. +Then: +:$\card {S \times T} = \card S \times \card T$ +where $\card S$ denotes [[Definition:Cardinality|cardinality]]. +\end{theorem} + +\begin{proof} +Let $\card S = n$ and $\card T = m$. +If either $n = 0$ or $m = 0$, then from [[Cartesian Product is Empty iff Factor is Empty]]: +:$S \times T = \O$ +and the result holds, as $n m = 0 = \card \O$ from [[Cardinality of Empty Set]]. +So, we assume that $n > 0$ and $m > 0$. +For each $a \in S$, we define the [[Definition:Mapping|mapping]] $g_a: T \to \set a \times T$ such that: +:$\forall y \in T: \map {g_a} y = \tuple {a, y}$ +The mapping $g_a$ is a [[Definition:Bijection|bijection]], so: +:$\card {\set a \times T} = m$ +Now let: +:$\mathbb T = \set {\set a \times T: a \in S}$ +We define the [[Definition:Mapping|mapping]] $h: S \to \mathbb T$: +:$\forall a \in S: \map h a = \set a \times T$ +The mapping $h$ is a [[Definition:Bijection|bijection]], so $\card {\mathbb T} = n$. +Thus $\mathbb T$ is a [[Definition:Partition (Set Theory)|partition]] of $S \times T$ containing $n$ [[Definition:Set|sets]]. +Hence from [[Number of Elements in Partition]]: +:$\card {S \times T} = n m$ +and the result follows. +{{qed}} +\end{proof}<|endoftext|> +\section{Cardinality of Set of All Mappings} +Tags: Combinatorics, Mapping Theory, Cardinality, Cardinality of Set of All Mappings + +\begin{theorem} +Let $S$ and $T$ be [[Definition:Set|sets]]. +The [[Definition:Cardinality|cardinality]] of the [[Definition:Set of All Mappings|set of all mappings]] from $S$ to $T$ (that is, the total number of [[Definition:Mapping|mappings]] from $S$ to $T$) is: +:$\card {T^S} = \card T^{\card S}$ +\end{theorem}<|endoftext|> +\section{Cardinality of Power Set of Finite Set} +Tags: Power Set, Combinatorics, Cardinality, Cardinality of Power Set + +\begin{theorem} +Let $S$ be a [[Definition:Set|set]] such that: +:$\card S = n$ +where $\card S$ denotes the [[Definition:Cardinality|cardinality]] of $S$, +Then: +:$\card {\powerset S} = 2^n$ +where $\powerset S$ denotes the [[Definition:Power Set|power set]] of $S$. +\end{theorem} + +\begin{proof} +Let $T = \set {0, 1}$. +For each $A \in \powerset S$, we consider the [[Definition:Characteristic Function of Set|characteristic function]] $\chi_A: S \to T$ defined as: +:$\forall x \in S: \map {\chi_A} x = \begin{cases} +1 & : x \in A \\ +0 & : x \notin A +\end{cases}$ +Now consider the [[Definition:Mapping|mapping]] $f: \powerset S \to T^S$: +:$\forall A \in \powerset S: \map f A = \chi_A$ +where $T^S$ is the [[Definition:Set of All Mappings|set of all mappings]] from $S$ to $T$. +Also, consider the [[Definition:Mapping|mapping]] $g: T^S \to \powerset S$: +:$\forall \phi \in T^S: \map g \phi = \phi^{-1} \sqbrk {\set 1}$ +where $\phi^{-1} \sqbrk {\set 1}$ is the [[Definition:Preimage of Subset under Mapping|preimage]] of $\set 1$ under $\phi$. +Consider the [[Definition:Characteristic Function of Set|characteristic function]] of $\phi^{-1} \sqbrk {\set 1}$, denoted $\chi_{\phi^{-1} \sqbrk {\set 1} }$. +We have: +{{begin-eqn}} +{{eqn | l = \forall x \in S: \map {\chi_{\phi^{-1} \sqbrk {\set 1} } } x + | r = \begin{cases} 1 & : x \in \phi^{-1} \sqbrk {\set 1} \\ 0 & : x \notin \phi^{-1} \sqbrk {\set 1} \end{cases} + | c = +}} +{{eqn | r = \begin{cases} 1 & : \map \phi x = 1 \\ 0 & : \map \phi x = 0 \end{cases} + | c = +}} +{{eqn | r = \map \phi x + | c = +}} +{{end-eqn}} +So: +{{begin-eqn}} +{{eqn | l = \forall \phi \in T^S: \map {\paren {f \circ g} } \phi + | r = \map f {\phi^{-1} } {\sqbrk {\set 1} } + | c = +}} +{{eqn | r = \chi_{\phi^{-1} \sqbrk {\set 1} } + | c = +}} +{{eqn | r = \phi + | c = +}} +{{end-eqn}} +So $f \circ g = I_{T^S}$, that is, the [[Definition:Identity Mapping|identity mapping]] on $T^S$. +So far so good. Now we consider the [[Definition:Preimage of Subset under Mapping|preimage]] of $\set 1$ under $\chi_A$: +: $\chi_A^{-1} \sqbrk {\set 1} = A$ +from the definition of the [[Definition:Characteristic Function of Set|characteristic function]] $\chi_A$ above. +Thus: +{{begin-eqn}} +{{eqn | l = \forall A \in \powerset S: \map {\paren {g \circ f} } A + | r = \map g {\map f A} + | c = +}} +{{eqn | r = \map g {\chi_A} + | c = +}} +{{eqn | r = \chi_A^{-1} \sqbrk {\set 1} + | c = +}} +{{eqn | r = A + | c = +}} +{{end-eqn}} +So $g \circ f = I_{\powerset S}$, that is, the [[Definition:Identity Mapping|identity mapping]] on $\powerset S$. +It follows from [[Bijection iff Left and Right Inverse]] that $f$ and $g$ are [[Definition:Bijection|bijections]]. +Thus by [[Cardinality of Set of All Mappings]] the result follows. +{{Qed}} +\end{proof} + +\begin{proof} +Enumerating the [[Definition:Subset|subsets]] of $S$ is equivalent to counting all of the ways of selecting $k$ out of the $n$ elements of $S$ with $k = 0, 1, \ldots, n$. +So, from [[Cardinality of Set of Subsets]], the number we are looking for is: +:$\displaystyle \card {\powerset S} = \sum_{k \mathop = 0}^n \binom n k$ +But from the [[Binomial Theorem|binomial theorem]]: +:$\displaystyle \paren {x + y}^n = \sum_{k \mathop = 0}^n \binom n k x^{n - k} y^k$ +It follows that: +:$2^n = \displaystyle \paren {1 + 1}^n = \sum_{k \mathop = 0}^n \binom n k \paren 1^{n - k} \paren 1^k = \sum_{k \mathop = 0}^n \binom n k = \card {\powerset S}$ +{{Qed}} +\end{proof} + +\begin{proof} +Given an element $x$ of $S$, each [[Definition:Subset|subset]] of $S$ either includes $x$ or does not include $x$ (this follows directly from the [[Definition:Set|definition of a set]]), which gives us two possibilities. +The same reasoning holds for any [[Definition:Element|element]] of $S$. +One can intuitively see that this means that there are $\displaystyle \underbrace {2 \times 2 \times \ldots \times 2}_{\card S} = 2^{\card S}$ total possible combinations of [[Definition:Element|elements]] of $S$. +This is exactly $\card {\powerset S}$. +{{qed}} +\end{proof} + +\begin{proof} +Proof by [[Principle of Mathematical Induction|induction]]: +For all $n \in \N$, let $P \left({n}\right)$ be the [[Definition:Proposition|proposition]]: +:$\left|{S}\right| = n \implies \left|{\mathcal P \left({S}\right)}\right| = 2^n$ +Do not confuse $P \left({n}\right)$, which is a [[Definition:Propositional Function|propositional function]] on $\N$, with $\mathcal P \left({S}\right)$, the [[Definition:Power Set|power set]] of $S$. +=== Basis for the Induction === +From [[Cardinality of Empty Set]], we have: +: $S = \varnothing \iff \left|{S}\right| = 0$ +Then: +: $\mathcal P \left({\varnothing}\right) = \left\{{\varnothing}\right\}$ +has one element, that is, $\varnothing$. +So: +: $\left|{\mathcal P \left({\varnothing}\right)}\right| = \left|{\left\{{\varnothing}\right\}}\right| = 1 = 2^0$ +thus confirming that $P \left({0}\right)$ holds. +This is our [[Principle of Mathematical Induction#Basis for the Induction|basis for the induction]]. +=== Induction Hypothesis === +Now we need to show that, if $P \left({k}\right)$ is true, where $k \ge 2$, then it logically follows that $P \left({k+1}\right)$ is true. +So this is our [[Principle of Mathematical Induction#Induction Hypothesis|induction hypothesis]]: +:$\left|{S}\right| = k \implies \left|{\mathcal P \left({S}\right)}\right| = 2^k$ +Then we need to show: +:$\left|{S}\right| = k + 1 \implies \left|{\mathcal P \left({S}\right)}\right| = 2^{k + 1}$ +=== Induction Step === +This is our [[Principle of Mathematical Induction#Induction Step|induction step]]: +Let $\left|{S}\right| = k + 1$. +Let $x \in S$. +Consider the set $S' = S \setminus \left\{{x}\right\}$ where $x$ is any element of $S$. +We see that $\left|{S'}\right| = k$. +Now adjoin $x$ to all the [[Definition:Subset|subsets]] of $S'$. +Counting the [[Definition:Subset|subsets]] of $S$, we have: +: all the [[Definition:Subset|subsets]] of $S'$ +and: +: all the [[Definition:Subset|subsets]] of $S'$ with $x$ adjoined to them. +From the [[Cardinality of Power Set of Finite Set/Proof 3#Induction Hypothesis|induction hypothesis]], there are $2^k$ subsets of $S'$. +Adding $x$ to each of these does not change their number, so there are another $2^k$ [[Definition:Subset|subsets]] of $S$ consisting of all the [[Definition:Subset|subsets]] of $S'$ with $x$ adjoined to them. +In total that makes $2^k + 2^k = 2 \times 2^k = 2^{k + 1}$ [[Definition:Subset|subsets]] of $S$. +So $P \left({k}\right) \implies P \left({k + 1}\right)$ and the result follows by the [[Principle of Mathematical Induction]]. +Therefore: +:$\forall n \in \N: \left|{S}\right| = n \implies \left|{\mathcal P \left({S}\right)}\right| = 2^n$ +{{qed}} +\end{proof}<|endoftext|> +\section{Cardinality of Set of Injections} +Tags: Injections, Combinatorics, Counting Arguments, Cardinality, Cardinality of Set of Injections + +\begin{theorem} +Let $S$ and $T$ be [[Definition:Finite Set|finite sets]]. +The number of [[Definition:Injection|injections]] from $S$ to $T$, where $\card S = m, \card T = n$ is often denoted ${}^m P_n$, and is: +:${}^m P_n = \begin{cases} +\dfrac {n!} {\paren {n - m}!} & : m \le n \\ +0 & : m > n \end{cases}$ +\end{theorem} + +\begin{proof} +Let $m > n$. +By the [[Pigeonhole Principle]], there can be no [[Definition:Injection|injection]] from $S$ to $T$ when $\left|{S}\right| > \left|{T}\right|$. +Once $\left|{T}\right|$ elements of $S$ have been used up, there is no element of $T$ left for the remaining elements of $S$ to be mapped to such that they all still map to different elements of $T$. +Let $m = 0$. +The only [[Definition:Injection|injection]] from $\varnothing \to T$ is $\varnothing \times T$ which is $\varnothing$. +So if $m = 0$ there is $1 = n! / n!$ [[Definition:Injection|injection]]. +Let $0 < m \le n$. +As in the proof of [[Cardinality of Set of All Mappings]], we can assume that $S = \N_m$ and $T = \N_n$. +For each $k \in \left[{1 \,.\,.\, n}\right]$, let $\mathbb H \left({k, n}\right)$ be the set of all injections from $\N_k$ to $\N_n$. +The proof now proceeds by [[Principle of Mathematical Induction|induction]]. +Let: +:$\displaystyle \mathbb S = \left\{{k \in \left[{1 \,.\,.\, n}\right]: \left|{\mathbb H \left({k, n}\right)}\right| = \frac {n!} {\left({n - k}\right)!}}\right\}$ +=== Basis for the Induction === +Let $k = 1$. +From [[Cardinality of Set of All Mappings]], there are $n^1 = n$ different mappings from $S$ to $T$. +From [[Mapping from Singleton is Injection]], each one of these $n$ mappings is an [[Definition:Injection|injection]]. +Thus: +:$\displaystyle \left|{\mathbb H \left({1, n}\right)}\right| = n = \frac {n!} {\left({n - 1}\right)!}$ +and so it follows that: +:$1 \in \mathbb S$. +This is the [[Principle of Mathematical Induction#Basis for the Induction|basis for the induction]]. +=== Induction Hypothesis === +We suppose that: +:$\displaystyle \left|{\mathbb H \left({k, n}\right)}\right| = \frac {n!} {\left({n - k}\right)!}$. +This is the [[Principle of Mathematical Induction#Induction Hypothesis|induction hypothesis]]. +We need to show that: +:$\displaystyle \left|{\mathbb H \left({k+1, n}\right)}\right| = \frac {n!} {\left({n - \left({k+1}\right)}\right)!}$. +=== Induction Step === +This is the [[Principle of Mathematical Induction#Induction Step|induction step]]: +Let $k \in \mathbb S$ such that $k < n$. +Let $\rho: \mathbb H \left({k + 1, n}\right) \to \mathbb H \left({k, n}\right)$ be the [[Definition:Mapping|mapping]] defined by: +:$\forall f \in \mathbb H \left({k + 1, n}\right): \rho \left({f}\right) =$ the [[Definition:Restriction of Mapping|restriction]] of $f$ to $\N_k$ +Given that $g \in \mathbb H \left({k, n}\right)$ and $a \in \N_n - g \left({\N_k}\right)$, let $g_a: \N_{k + 1} \to \N_n$ be the [[Definition:Mapping|mapping]] defined as: +:$g_a \left({x}\right) = \begin{cases} +g \left({x}\right) & : x \in \N_k \\ +a & : x = k \end{cases}$ +Now $g$ is an [[Definition:Injection|injection]] as $g \in \mathbb H \left({k, n}\right)$, and as $g_a \left({a}\right) \notin g \left({\N_k}\right)$ it follows that $g_a$ is also an [[Definition:Injection|injection]]. +Hence: +:$g_a \in \mathbb H \left({k + 1, n}\right)$ +It follows from the definition of $\rho$ that: +:$\rho^{-1} \left({\left\{{g}\right\}}\right) = \left\{{g_a: a \in \N_n - g \left({\N_k}\right)}\right\}$ +Since $g$ is an [[Definition:Injection|injection]], $g \left({\N_k}\right)$ has $k$ elements. +Therefore $\N_n - g \left({\N_k}\right)$ has $n - k$ elements by [[Cardinality of Complement]]. +As $G: a \to g_a$ is clearly a [[Definition:Bijection|bijection]] from $\N_n - g \left({\N_k}\right)$ onto $\rho^{-1} \left({\left\{{g}\right\}}\right)$, that [[Definition:Set|set]] has $n - k$ elements. +Clearly: +:$\left\{{\rho^{-1} \left({\left\{{g}\right\}}\right): g \in \mathbb H \left({k, n}\right)}\right\}$ +is a [[Definition:Partition (Set Theory)|partition]] of $\mathbb H \left({k + 1, n}\right)$. +Therefore by [[Number of Elements in Partition]]: +:$\left|{\mathbb H \left({k + 1, n}\right)}\right| = \left({n - k}\right) \dfrac {n!} {\left({n - k}\right)!} = \dfrac {n!} {\left({\left({n - k}\right) - 1}\right)!}$ +as $k \in \mathbb S$. +But: +:$\left({n - k}\right) - 1 = n - \left({k + 1}\right)$ +So: +:$k + 1 \in \mathbb S$ +By [[Principle of Mathematical Induction|induction]]: +:$\mathbb S = \left[{1 \,.\,.\, n}\right]$ +and in particular: +:$m \in \mathbb S$ +{{qed}} +\end{proof}<|endoftext|> +\section{Cardinality of Set of Bijections} +Tags: Combinatorics, Bijections, Cardinality, Cardinality of Set of Bijections + +\begin{theorem} +Let $S$ and $T$ be sets such that $\size S = \size T = n$. +Then there are $n!$ [[Definition:Bijection|bijections]] from $S$ to $T$. +\end{theorem} + +\begin{proof} +Follows directly from [[Cardinality of Set of Injections]] and [[Equivalence of Mappings between Sets of Same Cardinality]]. +{{Qed}} +\end{proof}<|endoftext|> +\section{Cardinality of Set of Subsets} +Tags: Combinations, Combinatorics, Cardinality of Set of Subsets + +\begin{theorem} +Let $S$ be a [[Definition:Set|set]] such that $\card S = n$. +Let $m \le n$. +Then the number of [[Definition:Subset|subsets]] $T$ of $S$ such that $\card T = m$ is: +: ${}^m C_n = \dfrac {n!} {m! \paren {n - m}!}$ +\end{theorem} + +\begin{proof} +For each $X \subseteq \N_n$ and $Y \subseteq S$, let $B \left({X, Y}\right)$ be the set of all [[Definition:Bijection|bijections]] from $X$ onto $Y$. +Let $\Bbb S$ be the [[Definition:Set of Sets|set]] of all [[Definition:Subset|subsets]] of $S$ with $m$ [[Definition:Element|elements]]. +By [[Cardinality of Power Set of Finite Set]] and [[Cardinality of Subset of Finite Set]], $\Bbb S$ is [[Definition:Finite Set|finite]], so let $s = \left|{\Bbb S}\right|$. +Let $\beta: B \left({\N_n, S}\right) \to \Bbb S$ be the [[Definition:Mapping|mapping]] defined as $\forall f \in B \left({\N_n, S}\right): \beta \left({f}\right) = f \left({\N_m}\right)$. +For each $Y \in \Bbb S$, the [[Definition:Mapping|mapping]]: +:$\Phi_Y: \beta^{-1} \left({Y}\right) \to B \left({\N_m, Y}\right) \times B \left({\N_n - \N_m, S - Y}\right)$ +defined as: +:$\Phi_Y \left({f}\right) = \left({f_{\N_m}, f_{\N_n - \N_m}}\right)$ +is also (clearly) a bijection. +By [[Cardinality of Set of Bijections]]: +:$\left|{B \left({\N_m, Y}\right)}\right| = m!$ +and: +:$\left|{B \left({\N_n - \N_m, S - Y}\right)}\right| = \left({n - m}\right)!$ +So by [[Cardinality of Cartesian Product]]: +:$\left|{\beta^{-1} \left({Y}\right)}\right| = m! \left({n - m}\right)!$ +It is clear that $\left\{{\beta^{-1} \left({Y}\right): Y \in \Bbb S}\right\}$ is a [[Definition:Partition (Set Theory)|partition]] of $B \left({\N_n, S}\right)$. +Therefore by [[Number of Elements in Partition]]: +:$\left|{B \left({\N_n, S}\right)}\right| = m! \left({n - m}\right)! s$ +Consequently, as $\left|{B \left({\N_n, S}\right)}\right| = n!$ by [[Cardinality of Set of Bijections]], it follows that: +:$m! \left({n - m}\right)! s = n!$ +and the result follows. +{{qed}} +\end{proof} + +\begin{proof} +Let ${}^m C_n$ be the number of [[Definition:Subset|subsets]] of $m$ [[Definition:Element|elements]] of $S$. +From [[Number of Permutations]], the number of [[Definition:Permutation (Ordered Selection)|$m$-permutations of $S$]] is: +:${}^m P_n = \dfrac {n!} {\paren {n - m}!}$ +Consider the way ${}^m P_n$ can be calculated. +First one makes the selection of which $m$ elements of $S$ are to be arranged. +This number is ${}^m C_n$. +Then for each selection, the number of different arrangements of these is $m!$, from [[Number of Permutations]]. +So: +{{begin-eqn}} +{{eqn | l = m! \cdot {}^m C_n + | r = {}^m P_n + | c = [[Product Rule for Counting]] +}} +{{eqn | r = \frac {n!} {\paren {n - m}!} + | c = [[Number of Permutations]] +}} +{{eqn | ll= \leadsto + | l = {}^m C_n + | r = \frac {n!} {m! \paren {n - m}!} + | c = [[Product Rule for Counting]] +}} +{{end-eqn}} +{{qed}} +\end{proof} + +\begin{proof} +Let $\N_n$ denote the [[Definition:Set|set]] $\set {1, 2, \ldots, n}$. +Let $\struct {S_n, \circ}$ denote the [[Definition:Symmetric Group on n Letters|symmetric group]] on $\N_n$. +Let $r \in \N: 0 < r \le n$. +Let $B_r$ denote the [[Definition:Set|set]] of all [[Definition:Subset|subsets]] of $\N_n$ of [[Definition:Cardinality|cardinality]] $r$: +:$B_r := \set {S \subseteq \N_n: \card S = r}$ +Let $*$ be the [[Definition:Mapping|mapping]] $*: S_n \times B_r \to B_r$ defined as: +:$\forall \pi \in S_n, \forall S \in B_r: \pi * B_r = \pi \sqbrk S$ +where $\pi \sqbrk S$ denotes the [[Definition:Image of Subset under Mapping|image of $S$ under $\pi$]]. +From [[Group Action of Symmetric Group on Subset]] it is established that $*$ is a [[Definition:Group Action|group action]]. +The [[Definition:Stabilizer|stabilizer]] of any $U \in B_r$ is the [[Definition:Set|set]] of [[Definition:Permutation|permutations]] on $\N_n$ that fix $U$. +Let $U = \set {1, 2, \ldots, r}$. +So: +:$\tuple {a_1, a_2, \ldots, a_r}$ can be any one of the $r!$ [[Definition:Permutation|permutations]] of $1, 2, \ldots, r$ +:$\tuple {a_{r + 1}, a_{r + 2}, \ldots, _n}$ can be any one of the $\paren {n - r}!$ [[Definition:Permutation|permutations]] of $r + 1, r + 2, \ldots, n$. +Thus: +:$\order {\Stab U} = r! \paren {n - r}!$ +From [[Group Action of Symmetric Group on Subset is Transitive]]: +:$B_r = \Orb U$ +and so: +:$\card {B_r} = \card {\Orb U}$ +From the [[Orbit-Stabilizer Theorem]]: +:$\card {\Orb U} = \dfrac {\order {S_n} } {\order {\Stab U} } = \dfrac {n!} {r! \paren {n - r}!}$ +But $\card {B_r}$ is the number of [[Definition:Subset|subsets]] of $\N_n$ of [[Definition:Cardinality|cardinality]] $r$. +Hence the result. +{{qed}} +\end{proof} + +\begin{proof} +The proof proceeds by [[Principle of Mathematical Induction|induction]]. +For all $n \in \Z_{\ge 0}$, let $\map P n$ be the [[Definition:Proposition|proposition]]: +:${}^m C_n = \dfrac {n!} {m! \paren {n - m}!}$ +=== Basis for the Induction === +$\map P 1$ is the case: +:${}^m C_n = \dfrac {n!} {m! \paren {n - m}!}$ +{{begin-eqn}} +{{eqn | l = {}^m C_1 + | r = \begin {cases} 1 & : m = 0 \text { or } m = 1 \\ 0 & : \text {otherwise} \end {cases} + | c = +}} +{{eqn | r = \dfrac {1!} {0! \paren {1 - 0}!} + | c = for $m = 0$ +}} +{{eqn | r = \dfrac {1!} {1! \paren {1 - 1}!} + | c = for $m = 1$ +}} +{{end-eqn}} +Thus $\map P 1$ is seen to hold. +This is the [[Principle of Mathematical Induction#Basis for the Induction|basis for the induction]]. +=== Induction Hypothesis === +Now it needs to be shown that, if $\map P k$ is true, where $k \ge 1$, then it logically follows that $\map P {k + 1}$ is true. +So this is the [[Principle of Mathematical Induction#Induction Hypothesis|induction hypothesis]]: +:${}^m C_k = \dfrac {k!} {m! \paren {k - m}!}$ +from which it is to be shown that: +:${}^m C_{k + 1} = \dfrac {\paren {k + 1}!} {m! \paren {k + 1 - m}!}$ +=== Induction Step === +This is the [[Principle of Mathematical Induction#Induction Step|induction step]]: +The number of ways to choose $m$ [[Definition:Element|elements]] from $k + 1$ [[Definition:Element|elements]] is: +:the number of ways to choose $m$ elements [[Definition:Element|elements]] from $k$ [[Definition:Element|elements]] (deciding not to select the $k + 1$th [[Definition:Element|element]]) +added to: +:the number of ways to choose $m - 1$ elements [[Definition:Element|elements]] from $k$ [[Definition:Element|elements]] (after having selected the $k + 1$th [[Definition:Element|element]] for the $n$th selection) +{{begin-eqn}} +{{eqn | l = {}^m C_{k + 1} + | r = {}^m C_k + {}^{m - 1} C_k + | c = +}} +{{eqn | r = \binom k m + \binom k {m - 1} + | c = [[Cardinality of Set of Subsets/Proof 4#Induction Hypothesis|Induction Hypothesis]] +}} +{{eqn | r = \binom {k + 1} m + | c = [[Pascal's Rule]] +}} +{{end-eqn}} +So $\map P k \implies \map P {k + 1}$ and the result follows by the [[Principle of Mathematical Induction]]. +Therefore: +:$\forall n \in \Z_{>0}: {}^m C_n = \dfrac {n!} {m! \paren {n - m}!}$ +\end{proof}<|endoftext|> +\section{Set of Integers Bounded Below by Integer has Smallest Element} +Tags: Set of Integers Bounded Below has Smallest Element + +\begin{theorem} +Let $\Z$ be the [[Definition:Integer|set of integers]]. +Let $\le$ be the [[Definition:Ordering on Integers|ordering on the integers]]. +Let $\O \subset S \subseteq \Z$ such that $S$ is [[Definition:Bounded Below Set|bounded below]] in $\struct {\Z, \le}$. +Then $S$ has a [[Definition:Smallest Element|smallest element]]. +\end{theorem} + +\begin{proof} +We have that $S$ is [[Definition:Bounded Below Set|bounded below]] in $\Z$. +So: +: $\exists m \in \Z: \forall s \in S: m \le s$ +Hence: +: $\forall s \in S: 0 \le s - m$ +Thus: +:$T = \set {s - m: s \in S} \subseteq \N$ +The [[Well-Ordering Principle]] gives that $T$ has a [[Definition:Smallest Element|smallest element]], which we can call $b_T \in T$. +Hence: +:$\paren {\forall s \in S: b_T \le s - m} \land \paren {\exists b_S \in S: b_T = b_S - m}$ +So: +{{begin-eqn}} +{{eqn | o = + | r = s \in S: b_S - m \le s - m + | c = +}} +{{eqn | o = \leadsto + | r = \forall s \in S: b_S \le s + | c = Cancellability of elements of $\Z$ +}} +{{eqn | o = \leadsto + | r = b_S \in S \land \paren {\forall s \in S: b_S \le s} + | c = {{Defof|Smallest Element}} +}} +{{end-eqn}} +So $b_S$ is the [[Definition:Smallest Element|smallest element]] of $S$. +{{Qed}} +\end{proof}<|endoftext|> +\section{Set of Integers Bounded Above by Integer has Greatest Element} +Tags: Number Theory, Integers, Set of Integers Bounded Above has Greatest Element + +\begin{theorem} +Let $\Z$ be the [[Definition:Integer|set of integers]]. +Let $\le$ be the [[Definition:Ordering on Integers|ordering on the integers]]. +Let $\O \subset S \subseteq \Z$ such that $S$ is [[Definition:Bounded Above Set|bounded above]] in $\struct {\Z, \le}$. +Then $S$ has a [[Definition:Greatest Element|greatest element]]. +\end{theorem} + +\begin{proof} +$S$ is [[Definition:Bounded Above Set|bounded above]], so $\exists M \in \Z: \forall s \in S: s \le M$. +Hence $\forall s \in S: 0 \le M - s$. +Thus the set $T = \set {M - s: s \in S} \subseteq \N$. +The [[Well-Ordering Principle]] gives that $T$ has a [[Definition:Smallest Element|smallest element]], which we can call $b_T \in T$. +Hence: +:$\paren {\forall s \in S: b_T \le M - s} \land \paren {\exists g_S \in S: b_T = M - g_S}$ +So: +{{begin-eqn}} +{{eqn | o = + | r = \forall s \in S: M - g_S \le M - s + | c = +}} +{{eqn | o = \leadsto + | r = \forall s \in S: -g_S \le -s + | c = Cancellability of elements of $\Z$ +}} +{{eqn | o = \leadsto + | r = \forall s \in S: g_S \ge s + | c = +}} +{{eqn | o = \leadsto + | r = g_S \in S \land \paren {\forall s \in S: g_S \ge s} + | c = which is how the [[Definition:Greatest Element|greatest element]] is defined. +}} +{{end-eqn}} + +So $g_S$ is the [[Definition:Greatest Element|greatest element]] of $S$. +{{qed}} +\end{proof} + +\begin{proof} +Since $S$ is [[Definition:Bounded Above Set|bounded above]], $\exists M \in \Z: \forall s \in S: s \le M$. +Hence we can define the set $S' = \set {-s: s \in S}$. +$S'$ is [[Definition:Bounded Below Set|bounded below]] by $-M$. +So from [[Set of Integers Bounded Below by Integer has Smallest Element]], $S'$ has a [[Definition:Smallest Element|smallest element]], $-g_S$, say, where $\forall s \in S: -g_S \le -s$. +Therefore $g_S \in S$ (by definition of $S'$) and $\forall s \in S: s \le g_S$. +So $g_S$ is the [[Definition:Greatest Element|greatest element]] of $S$. +{{qed}} +\end{proof}<|endoftext|> +\section{Principle of Least Counterexample} +Tags: Proof Techniques + +\begin{theorem} +Suppose $P \paren n$ is a [[Definition:Propositional Function|condition]] on $n \in \set {x \in \Z: x \ge m \in \Z}$. +Suppose next that: $\neg \paren {\forall n \ge m: P \paren n}$. +(That is, not all $n \ge m$ satisfy $P \paren n$.) +Then there is a '''least counterexample''', that is a [[Definition:Smallest Element|smallest]] [[Definition:Integer|integral]] value of $n$ for which $\neg P \paren n$. +\end{theorem} + +\begin{proof} +Let $S = \set {n \in \Z: n \ge m \in \Z: \neg P \paren n}$. +That is, $S$ is the set of all elements in $\Z$ not less than $m$ for which the [[Definition:Propositional Function|condition]] is false. +Since: +:$\neg \paren {\forall n \ge m: P \paren n}$ +it follows that: +:$S \ne \O$ +Also, $S \subseteq \Z$ and is [[Definition:Bounded Below Set|bounded below]] (by $m$). +Therefore [[Set of Integers Bounded Below by Integer has Smallest Element|$S$ has a smallest element]], which proves the result. +\end{proof}<|endoftext|> +\section{Absolute Value is Bounded Below by Zero} +Tags: Absolute Value Function + +\begin{theorem} +Let $x \in \R$ be a [[Definition:Real Number|real number]]. +Then the [[Definition:Absolute Value|absolute value]] $\size x$ of $x$ is [[Definition:Bounded Below Mapping|bounded below]] by $0$. +\end{theorem} + +\begin{proof} +Let $x \ge 0$. +Then $\size x = x \ge 0$. +Let $x < 0$. +Then $\size x = -x > 0$. +The result follows. +{{qed}} +\end{proof}<|endoftext|> +\section{Division Theorem} +Tags: Division Theorem, Named Theorems, Divisors + +\begin{theorem} +For every pair of [[Definition:Integer|integers]] $a, b$ where $b \ne 0$, there exist [[Definition:Unique|unique]] [[Definition:Integer|integers]] $q, r$ such that $a = q b + r$ and $0 \le r < \size b$: +:$\forall a, b \in \Z, b \ne 0: \exists! q, r \in \Z: a = q b + r, 0 \le r < \size b$ +\end{theorem} + +\begin{proof} +From [[Division Theorem/Positive Divisor|Division Theorem: Positive Divisor]]: +:$\forall a, b \in \Z, b > 0: \exists! q, r \in \Z: a = q b + r, 0 \le r < b$ +That is, the result holds for [[Definition:Positive Integer|positive]] $b$. +{{qed|lemma}} +It remains to show that the result also holds for [[Definition:Negative Integer|negative]] values of $b$. +Let $b < 0$. +Consider: +:$\size b = -b > 0$ +where $\size b$ denotes the [[Definition:Absolute Value|absolute value]] of $b$: by definition $\size b > 0$. +From [[Division Theorem/Positive Divisor|Division Theorem: Positive Divisor]], we have the existence of $\tilde q, \tilde r \in \Z$ such that: +:$a = \tilde q \size b + \tilde r, 0 \le \tilde r < \size b$ +Since $\size b = -b$: +:$a = \tilde q \paren {-b} + \paren {\tilde r} = \paren {-\tilde q} b + \tilde r$ +Taking: +:$q = -\tilde q, r = \tilde r$ +the existence has been proved of [[Definition:Integer|integers]] $q$ and $r$ that satisfy the requirements. +The proof that they are [[Definition:Unique|unique]] is the same as that for the [[Division Theorem/Positive Divisor/Uniqueness|proof for positive $b$]], but with $\size b$ replacing $b$. +{{qed}} +\end{proof} + +\begin{proof} +=== Existence === +Consider the [[Definition:Arithmetic Sequence|arithmetic sequence]]: +:$\ldots, a - 3 b, a - 2 b, a - b, a, a + b, a + 2 b, a + 3 b, \ldots$ +which extends in both directions. +Then by the [[Well-Ordering Principle]], there must exist a smallest [[Definition:Positive Integer|non-negative]] element, denoted by $r$. +So $r = a - q b$ for some $q \in \Z$. +$r$ must be in the interval $\hointr 0 b$ because otherwise $r - b$ would be smaller than $r$ and a [[Definition:Positive Integer|non-negative]] element in the [[Definition:Arithmetic Sequence|sequence]]. +{{qed|lemma}} +=== Uniqueness === +Suppose we have another pair $q_0$ and $r_0$ such that $a = b q_0 + r_0$, with $0 \le r_0 < b$. +Then: +:$b q + r = b q_0 + r_0$ +Factoring we see that: +:$r - r_0 = b \paren {q_0 - q}$ +and so: +:$b \divides \paren {r - r_0}$ +Since $0 \le r < b$ and $0 \le r_0 < b$, we have that: +:$-b < r - r_0 < b$ +Hence: +:$r - r_0 = 0 \implies r = r_0$ +So now: +:$r - r_0 = 0 = b \paren {q_0 - q}$ +which implies that: +:$q = q_0$ +Therefore the solution is [[Definition:Unique|unique]]. +{{Qed}} +\end{proof} + +\begin{proof} +From the equation $0_F = 0_F \circ d + 0_F$, the theorem is true for the trivial case $f = 0_F$. +So, if there is a [[Definition:Counterexample|counterexample]] to be found, it will have a [[Definition:Degree of Polynomial over Field|degree]]. +{{AimForCont}} there exists at least one [[Definition:Counterexample|counterexample]]. +By a version of the [[Well-Ordering Principle]], we can assign a number $m$ to the lowest [[Definition:Degree of Polynomial over Field|degree]] possessed by any [[Definition:Counterexample|counterexample]]. +So, let $f$ denote a [[Definition:Counterexample|counterexample]] which has that minimum [[Definition:Degree of Polynomial over Field|degree]] $m$. +If $m < n$, the equation $f = 0_F \circ d + f$ would show that $f$ was not a [[Definition:Counterexample|counterexample]]. +Therefore $m \ge n$. +Suppose $d \divides f$ in $F \sqbrk X$. +Then: +:$\exists q \in F \sqbrk X: f = q \circ d + 0_F$ +and $f$ would not be a [[Definition:Counterexample|counterexample]]. +So $d \nmid f$ in $F \sqbrk X$. +So, suppose that: +{{begin-eqn}} +{{eqn | l = f + | r = \sum_{k \mathop = 0}^m {a_k \circ X^k} +}} +{{eqn | l = d + | r = \sum_{k \mathop = 0}^n {b_k \circ X^k} +}} +{{eqn | l = m + | o = \ge + | r = n +}} +{{end-eqn}} +We can create the [[Definition:Polynomial in Ring Element|polynomial]] $\paren {a_m \circ b_n^{-1} \circ X^{m - n} } \circ d$ which has the same [[Definition:Degree of Polynomial over Field|degree]] and [[Definition:Leading Coefficient of Polynomial|leading coefficient]] as $f$. +Thus $f_1 = f - \paren {a_m \circ b_n^{-1} \circ X^{m - n} } \circ d$ is a [[Definition:Polynomial in Ring Element|polynomial]] of [[Definition:Degree of Polynomial over Field|degree]] less than $m$. +Since $d \nmid f$, $f_1$ is a non-[[Definition:Null Polynomial over Ring|zero polynomial]]. +There is no [[Definition:Counterexample|counterexample]] of [[Definition:Degree of Polynomial over Field|degree]] less than $m$. +Therefore: +:$f_1 = q_1 \circ d + r$ +for some $q_1, r \in F \sqbrk X$, where either: +:$r = 0_F$ +or: +:$r$ is non-[[Definition:Null Polynomial over Ring|zero]] with [[Definition:Degree of Polynomial over Field|degree]] strictly less than $n$. +Hence: +{{begin-eqn}} +{{eqn | l = f + | r = f_1 + \paren {a_m \circ b_n^{-1} \circ X^{m - n} } \circ d + | c = +}} +{{eqn | r = \paren {q_1 + a_m \circ b_n^{-1} \circ X^{m - n} } \circ d + r + | c = +}} +{{end-eqn}} +Thus $f$ is not a [[Definition:Counterexample|counterexample]]. +From this [[Proof by Contradiction|contradiction]] follows the result. +{{qed}} +\end{proof} + +\begin{proof} +Suppose $\map \deg f < \map \deg d$. +Then we take $\map q X = 0$ and $\map r X = \map a X$ and the result holds. +Otherwise, $\map \deg f \ge \map \deg d$. +Let: +:$\map f X = a_0 + a_1 X + a_2 x^2 + \cdots + a_m X^m$ +:$\map d X = b_0 + b_1 X + b_2 x^2 + \cdots + b_n X^n$ +We can subtract from $f$ a suitable multiple of $d$ so as to eliminate the highest term in $f$: +:$\map f X - \map d X \cdot \dfrac {a_m} {b_n} x^{m - n} = \map p X$ +where $\map p X$ is some polynomial whose [[Definition:Degree of Polynomial over Field|degree]] is less than that of $f$. +If $\map p X$ still has [[Definition:Degree of Polynomial over Field|degree]] higher than that of $d$, we do the same thing again. +Eventually we reach: +:$\map f X - \map d X \cdot \paren {\dfrac {a_m} {b_n} x^{m - n} + \dotsb} = \map r X$ +where either $r = 0_F$ or $r$ has [[Definition:Degree of Polynomial over Field|degree]] that is less than $n$. +This approach can be formalised using the [[Principle of Complete Induction]]. +{{qed}} +\end{proof} + +\begin{proof} +Proof by [[Principle of Mathematical Induction|induction]]: +For all $n \in \N_{> 0}$, let $\map P n$ be the [[Definition:Proposition|proposition]]: +:$\forall f \in F \sqbrk X: \exists q, r \in F \sqbrk X: f = q \circ d + r$ provided that $\map \deg f < n$ +=== Basis for the Induction === +$\map P 0$ is the statement that $q$ and $r$ exist when $f = 0$. +This is shown trivially to be true by taking $q = r = 0$. +This is our [[Definition:Basis for the Induction|basis for the induction]]. +=== Induction Hypothesis === +Now we need to show that, if $\map P k$ is true, where $k \ge 0$, then it logically follows that $\map P {k + 1}$ is true. +So this is our [[Definition:Induction Hypothesis|induction hypothesis]]: +:$\forall f \in F \sqbrk X: \exists q, r \in F \sqbrk X: f = q \circ d + r$ provided that $\map \deg f < k$ +Then we need to show: +:$\forall f \in F \sqbrk X: \exists q, r \in F \sqbrk X: f = q \circ d + r$ provided that $\map \deg f < k + 1$ +=== Induction Step === +This is our [[Definition:Induction Step|induction step]]: +Let $f$ be such that $\map \deg f = n$. +Let: +:$f = a_0 + a_1 \circ x + a_2 \circ x^2 + \cdots + a_n \circ x^n$ where $a_n \ne 0$ +Let: +:$d = b_0 + b_1 \circ x + b_2 \circ x^2 + \cdots + b_j \circ x^j$ where $b_j \ne 0$ +If $n < l$ then take $q = 0, r = f$. +If $n \ge l$, consider: +:$c := f - a_n b_j^{-1} x^{n - j} d$ +This has been carefully arranged so that the [[Definition:Polynomial Coefficient|coefficient]] of $x^n$ in $c$ is zero. +Thus $\map \deg c < n$. +Therefore, by the [[Division Theorem for Polynomial Forms over Field/Proof 3#Induction Hypothesis|induction hypothesis]]: +:$c = d q_0 + r$ +where $\map \deg r < \map \deg d$. +Therefore: +{{begin-eqn}} +{{eqn | l = f + | r = d \paren {q_0 + a_n b_j^{-1} x^{n - j} } + r + | c = +}} +{{eqn | r = d q + r + | c = where $\map \deg r < \map \deg d$ and $q = q_0 + a_n b_j^{-1} x^{n - j}$ +}} +{{end-eqn}} +Thus the existence of $q$ and $r$ have been established. +As for uniqueness, assume: +:$d q + r = d q' + r'$ +with $\map \deg r < \map \deg d, \map \deg {r'} < \map \deg d$ +Then: +:$d \paren {q - q'} = r' - r$ +By [[Degree of Sum of Polynomials]]: +:$\map \deg {r' - r} \le \max \set {\map \deg {r'}, \map \deg r} < \map \deg d$ +and by [[Degree of Product of Polynomials over Integral Domain]]: +:$\map \deg {d \paren {q - q'} } = \map \deg d + \map \deg {q - q'}$ +That is: +:$\map \deg d < \map \deg d + \map \deg {q - q'}$ +and the only way for that to happen is for: +:$\map \deg {q - q'} = -\infty$ +that is, for $q - q'$ to be the [[Definition:Null Polynomial|null polynomial]]. +That is, $q - q' = 0_F$ and by a similar argument $r' - r = 0_F$, demonstrating the uniqueness of $q$ and $r$. +So $\map P k \implies \map P {k + 1}$ and the result follows by the [[Principle of Mathematical Induction]]. +Therefore: +:$\forall f \in F \sqbrk X: \exists q, r \in F \sqbrk X: f = q \circ d + r$ such that either: +::$(1): \quad r = 0_F$ +:or: +::$(2): \quad r \ne 0_F$ and $r$ has [[Definition:Degree of Polynomial over Field|degree]] that is less than $n$. +{{qed}} +\end{proof} + +\begin{proof} +Consider the set of [[Definition:Divisor of Integer|integer multiples]] $x \size b$ of $\size b$ less than or equal to $a$: +:$M := \set {k \in \Z: \exists x \in \Z: k = x \size b, k \le a}$ +We have that: +:$-\size a \size b \le -\size a \le a$ +and so $M \ne \O$. +From [[Set of Integers Bounded Above by Integer has Greatest Element]], $M$ has a [[Definition:Greatest Element|greatest element]] $h \size b$. +Then $h \size b \le a$ and so: +:$a = h \size b + r$ +where $r \ge 0$. +On the other hand: +:$\paren {h + 1} \size b = h \size b + \size b > h \size b$ +So: +:$\paren {h + 1} \size b > a$ +and: +:$h \size b + \size b > h \size b + r$ +Thus: +:$r \le b$ +Setting: +:$q = h$ when $b > 0$ +:$q = -h$ when $b < 0$ +it follows that: +:$h \size b = q b$ +and so: +:$a = q b + r$ +as required. +{{Qed}} +\end{proof}<|endoftext|> +\section{Odd Integer 2n + 1} +Tags: Odd Integers + +\begin{theorem} +Let $m$ be an [[Definition:Odd Integer|odd integer]]. +Then there exists [[Definition:Exactly One|exactly one]] [[Definition:Integer|integer]] $n$ such that $2 n + 1 = m$. +\end{theorem} + +\begin{proof} +Follows directly from the [[Division Theorem]]. +{{qed}} +[[Category:Odd Integers]] +mpi9q6vy7fvjpowo6jxu7u75i7r217j +\end{proof}<|endoftext|> +\section{Integer Divisor Results} +Tags: Divisors, Integers, Integer Divisor Results + +\begin{theorem} +Let $m, n \in \Z$ be [[Definition:Integer|integers]]. +Let $m \divides n$ denote that $m$ is a [[Definition:Divisor of Integer|divisor]] of $n$. +The following results all hold: +\end{theorem}<|endoftext|> +\section{Zero Divides Zero} +Tags: Divisors + +\begin{theorem} +Let $n \in \Z$ be an [[Definition:Integer|integer]]. +Then: +:$0 \divides n \implies n = 0$ +That is, [[Definition:Zero (Number)|zero]] is the only [[Definition:Integer|integer]] [[Definition:Divisor of Integer|divisible]] by [[Definition:Zero (Number)|zero]]. +\end{theorem} + +\begin{proof} +{{begin-eqn}} +{{eqn | l = 0 + | o = \divides + | r = n + | c = +}} +{{eqn | ll= \leadsto + | l = \exists q \in \Z: n + | r = q \times 0 + | c = {{Defof|Divisor of Integer}} +}} +{{eqn | ll= \leadsto + | l = n + | r = 0 + | c = Integers have no [[Definition:Zero Divisor of Ring|zero divisors]], as [[Integers form Integral Domain]]. +}} +{{end-eqn}} +{{Qed}} +\end{proof}<|endoftext|> +\section{Absolute Value of Integer is not less than Divisors} +Tags: Number Theory, Divisors + +\begin{theorem} +A (non-[[Definition:Zero (Number)|zero]]) [[Definition:Integer|integer]] is greater than or equal to its [[Definition:Divisor of Integer|divisors]] in [[Definition:Absolute Value|magnitude]]: +:$\forall c \in \Z_{\ne 0}: a \divides c \implies a \le \size a \le \size c$ +\end{theorem} + +\begin{proof} +Suppose $a \divides c$ for some $c \ne 0$. +From [[Negative of Absolute Value]]: +: $a \le \size a$ +Then: +{{begin-eqn}} +{{eqn | l = a + | o = \divides + | r = c + | c = +}} +{{eqn | lll=\leadsto + | ll= \exists q \in \Z: + | l = c + | r = a q + | c = {{Defof|Divisor of Integer}} +}} +{{eqn | lll=\leadsto + | l = \size c + | r = \size a \size q + | c = +}} +{{eqn | lll=\leadsto + | l = \size a \size q \ge \size a \times 1 + | r = \size a + | c = +}} +{{eqn | lll=\leadsto + | l = a \le \size a + | o = \le + | r = \size c + | c = +}} +{{end-eqn}} +{{qed}} +\end{proof}<|endoftext|> +\section{Divisor Relation on Positive Integers is Partial Ordering} +Tags: Divisors, Integers, Order Theory + +\begin{theorem} +The [[Definition:Divisor of Integer|divisor relation]] is a [[Definition:Partial Ordering|partial ordering]] of $\Z_{>0}$. +\end{theorem} + +\begin{proof} +Checking in turn each of the criteria for an [[Definition:Ordering|ordering]]: +=== [[Integer Divisor Results/Integer Divides Itself/Proof 1|Divisor Relation is Reflexive]] === +{{:Integer Divisor Results/Integer Divides Itself/Proof 1}} +=== [[Divisor Relation is Transitive]] === +:$\forall x, y, z \in \Z: x \divides y \land y \divides z \implies x \divides z$: +{{:Divisor Relation is Transitive/Proof 2}} +=== [[Divisor Relation is Antisymmetric]] === +{{:Divisor Relation is Antisymmetric}} +=== Divisor Ordering is Partial === +Let $a = 2$ and $b = 3$. +Then neither $a \divides b$ nor $b \divides a$. +Thus, while the [[Definition:Divisor of Integer|divisor relation]] is an [[Definition:Ordering|ordering]], it is specifically a [[Definition:Partial Ordering|partial ordering]] +{{qed}} +\end{proof}<|endoftext|> +\section{Common Divisor in Integral Domain Divides Linear Combination} +Tags: Divisibility + +\begin{theorem} +Let $\struct {D, +, \times}$ be an [[Definition:Integral Domain|integral domain]]. +Let $c$ be a [[Definition:Common Divisor of Ring Elements|common divisor]] of two elements $a$ and $b$ of $D$. +That is: +:$a, b, c \in D: c \divides a \land c \divides b$ +Then: +:$\forall p, q \in D: c \divides \paren {p \times a + q \times b}$ +\end{theorem} + +\begin{proof} +{{begin-eqn}} +{{eqn | l = c + | o = \divides + | r = a +}} +{{eqn | ll= \leadsto + | lo= \exists x \in D: + | l = a + | r = x \times c + | c = {{Defof|Divisor of Ring Element}} +}} +{{eqn | l = c + | o = \divides + | r = b +}} +{{eqn | ll= \leadsto + | lo= \exists y \in D: + | l = b + | r = y \times c + | c = {{Defof|Divisor of Ring Element}} +}} +{{eqn | ll= \leadsto + | lo= \forall p, q \in D: + | l = p \times a + q \times b + | r = p \times x \times c + q \times y \times c + | c = substituting for $a$ and $b$ +}} +{{eqn | r = \paren {p \times x + q \times y} c + | c = as $\times$ is [[Definition:Distributive Operation|distributive]] over $+$ +}} +{{eqn | ll= \leadsto + | lo= \exists z \in D: + | l = p \times a + q \times b + | r = z \times c + | c = where $z = p \times x + q \times y$ +}} +{{eqn | ll= \leadsto + | l = c + | o = \divides + | r = \paren {p \times a + q \times b} + | c = {{Defof|Divisor of Ring Element}} +}} +{{end-eqn}} +{{qed}} +\end{proof}<|endoftext|> +\section{Existence of Greatest Common Divisor} +Tags: Greatest Common Divisor, Existence of Greatest Common Divisor + +\begin{theorem} +Let $a, b \in \Z$ be [[Definition:Integer|integers]] such that $a \ne 0$ or $b \ne 0$. +Then the [[Definition:Greatest Common Divisor of Integers|greatest common divisor]] of $a$ and $b$ exists. +\end{theorem} + +\begin{proof} +=== Proof of Existence === +We have that: +:$\forall a, b \in \Z: 1 \divides a \land 1 \divides b$ +so $1$ is always a [[Definition:Common Divisor of Integers|common divisor]] of any two [[Definition:Integer|integers]]. +{{qed|lemma}} +=== Proof of there being a Largest === +As the definition of $\gcd$ shows that it is symmetric, we can assume without loss of generality that $a \ne 0$. +First we note that from [[Absolute Value of Integer is not less than Divisors]]: +:$\forall c \in \Z: \forall a \in \Z_{>0}: c \divides a \implies c \le \size c \le \size a$ +The same applies for $c \divides b$. +Now we have three different results depending on $a$ and $b$: +{{begin-eqn}} +{{eqn | l = a \ne 0 \land b \ne 0 + | o = \implies + | r = \gcd \set {a, b} \le \min \set {\size a, \size b} + | c = +}} +{{eqn | l = a = 0 \lor b = 0 + | o = \implies + | r = \gcd \set {a, b} = \max \set {\size a, \size b} + | c = +}} +{{eqn | l = a = b = 0 + | o = \implies + | r = \forall x \in \Z: x \divides a \land x \divides b + | c = +}} +{{end-eqn}} +So if $a$ and $b$ are ''both'' [[Definition:Zero (Number)|zero]], then ''any'' $n \in \Z$ divides both, and there is no [[Definition:Greatest Common Divisor|greatest common divisor]]. +This is why the proviso that $a \ne 0 \lor b \ne 0$. +So we have proved that [[Definition:Common Divisor|common divisors]] exist and are [[Definition:Bounded Above Set|bounded above]]. +Therefore, from [[Set of Integers Bounded Above by Integer has Greatest Element]] there is always a [[Definition:Greatest Common Divisor|'''greatest''' common divisor]]. +{{qed}} +\end{proof} + +\begin{proof} +By definition of [[Definition:Greatest Common Divisor|greatest common divisor]], we aim to show that there exists $c \in \Z_{>0}$ such that: +{{begin-eqn}} +{{eqn | l = c + | o = \divides + | r = a +}} +{{eqn | l = c + | o = \divides + | r = b +}} +{{end-eqn}} +and: +:$d \divides a, d \divides b \implies d \divides c$ +Consider the [[Definition:Set|set]] $S$: +:$S = \set {s \in \Z_0: \exists x, y \in \Z: s = a x + b y}$ +$S$ is not [[Definition:Empty Set|empty]], because by setting $x = 1$ and $y = 0$ we have at least that $a \in S$. +From the [[Well-Ordering Principle]], there exists a smallest $c \in S$. +So, by definition, we have $c > 0$ is the smallest such that $c = a x + b y$ for some $x, y \in \Z$. +Let $d$ be such that $d \divides a$ and $d \divides b$. +Then from [[Common Divisor Divides Integer Combination]]: +:$d \divides a x + b y$ +That is: +:$d \divides c$ +We have that: +{{begin-eqn}} +{{eqn | lo= \exists t, u \in \Z: + | l = a + | r = c t + u: + | rr= 0 \le u < c + | c = [[Division Theorem]] +}} +{{eqn | ll= \leadsto + | l = a + | r = a x t + b y t + u + | c = Definition of $c$ +}} +{{eqn | ll= \leadsto + | l = u + | r = r \paren {1 - x t} + b \paren {-y t} + | c = rearranging +}} +{{eqn | ll= \leadsto + | l = u + | r = 0 + | c = as $u < c$ and the definition of $c$ +}} +{{eqn | ll= \leadsto + | l = c + | o = \divides + | r = a + | c = {{Defof|Divisor of Integer}} +}} +{{end-eqn}} +[[Definition:Mutatis Mutandis|Mutatis mutandis]]: +:$c \divides b$ +Now suppose $c'$ is such that: +{{begin-eqn}} +{{eqn | l = c' + | o = \divides + | r = a +}} +{{eqn | l = c' + | o = \divides + | r = b +}} +{{end-eqn}} +and: +:$d \divides a, d \divides b \implies d \divides c'$ +Then we have immediately that: +:$c' \divides c$ +and by the same coin: +$c \divides c'$ +and so: +:$c = c'$ +demonstrating that $c$ is [[Definition:Unique|unique]]. +{{qed}} +\end{proof}<|endoftext|> +\section{Greatest Common Divisor is at least 1} +Tags: Greatest Common Divisor + +\begin{theorem} +Let $a, b \in \Z$ be [[Definition:Integer|integers]]. +The [[Definition:Greatest Common Divisor of Integers|greatest common divisor]] of $a$ and $b$ is at least $1$: +:$\forall a, b \in \Z_{\ne 0}: \gcd \set {a, b} \ge 1$ +\end{theorem} + +\begin{proof} +From [[One Divides all Integers]]: +:$\forall a, b \in \Z: 1 \divides a \land 1 \divides b$ +and so: +:$1 \le \gcd \set {a, b}$ +as required. +{{qed}} +[[Category:Greatest Common Divisor]] +2nrji97dz25wb80mslueng8xuvr7oza +\end{proof}<|endoftext|> +\section{GCD of Integer and Divisor} +Tags: Greatest Common Divisor + +\begin{theorem} +Let $a, b \in \Z_{>0}$, i.e. [[Definition:Integer|integers]] such that $a, b > 0$. +Then: +: $a \divides b \implies \gcd \set {a, b} = a$ +\end{theorem} + +\begin{proof} +$a \divides b$ by hypothesis, $a \divides a$ from [[Integer Divides Itself]]. +Thus $a$ is a [[Definition:Common Divisor of Integers|common divisor]] of $a$ and $b$. +Note that from [[Absolute Value of Integer is not less than Divisors]]: +:$\forall x \in \Z: x \divides a \implies x \le \size a$. +As $a$ and $b$ are both [[Definition:Positive Integer|positive]], the result follows. +{{qed}} +\end{proof}<|endoftext|> +\section{GCD for Negative Integers} +Tags: Greatest Common Divisor + +\begin{theorem} +:$\gcd \set {a, b} = \gcd \set {\size a, b} = \gcd \set {a, \size b} = \gcd \set {\size a, \size b}$ +Alternatively, this can be put: +:$\gcd \set {a, b} = \gcd \set {-a, b} = \gcd \set {a, -b} = \gcd \set {-a, -b}$ + +which follows directly from the above. +\end{theorem} + +\begin{proof} +Note that $\size a = \pm a$. +Suppose that: +:$u \divides a$ +where $\divides$ denotes [[Definition:Divisor of Integer|divisibility]]. +Then: +:$\exists q \in \Z: a = q u$ +Then: +:$\size a = \pm q u = \paren {\pm q} u \implies u \divides \size a$ +So every [[Definition:Divisor of Integer|divisor]] of $a$ is a [[Definition:Divisor of Integer|divisor]] of $\size a$. +Similarly, note that: +:$a = \pm \size a$ +so every [[Definition:Divisor of Integer|divisor]] of $\size a$ is a [[Definition:Divisor of Integer|divisor]] of $a$. +So it follows that the [[Definition:Common Divisor of Integers|common divisors]] of $a$ and $b$ are the same as those of $a$ and $\size b$, and so on. +In particular: +:$\gcd \set {a, b} = \gcd \set {a, \size b}$ +and so on. +{{Qed}} +[[Category:Greatest Common Divisor]] +le9bxzi7pir8rzelwcgd7h3vqlvza11 +\end{proof}<|endoftext|> +\section{GCD with Zero} +Tags: Greatest Common Divisor + +\begin{theorem} +Let $a \in \Z$ be an [[Definition:Integer|integer]] such that $a \ne 0$. +Then: +:$\gcd \left\{{a, 0}\right\} = \left\lvert{a}\right\rvert$ +where $\gcd$ denotes [[Definition:Greatest Common Divisor of Integers|greatest common divisor (GCD)]]. +\end{theorem} + +\begin{proof} +Follows from: +: [[Integer Divides Zero]] +: [[GCD for Negative Integers]]. +{{qed}} +\end{proof}<|endoftext|> +\section{Set of Integer Combinations equals Set of Multiples of GCD} +Tags: Integer Combinations, Greatest Common Divisor + +\begin{theorem} +The [[Definition:Set|set]] of all [[Definition:Integer Combination|integer combinations]] of $a$ and $b$ is precisely the set of all [[Definition:Divisor of Integer|integer multiples]] of the [[Definition:Greatest Common Divisor of Integers|GCD]] of $a$ and $b$: +:$\gcd \set {a, b} \divides c \iff \exists x, y \in \Z: c = x a + y b$ +\end{theorem} + +\begin{proof} +=== Necessary Condition === +Let $d = \gcd \set {a, b}$. +Then: +:$d \divides c \implies \exists m \in \Z: c = m d$ +So: +{{begin-eqn}} +{{eqn | l = \exists p, q \in \Z: d + | r = p a + q b + | c = [[Bézout's Lemma]] +}} +{{eqn | ll= \leadsto + | l = c + | r = m d + | c = +}} +{{eqn | r = m p a + m q b + | c = +}} +{{eqn | r = \paren {m p} a + \paren {m q} b + | c = +}} +{{eqn | ll= \leadsto + | l = \exists x, y \in \Z: c + | r = x a + y b + | c = +}} +{{end-eqn}} +Thus: +:$\gcd \set {a, b} \divides c \implies \exists x, y \in \Z: c = x a + y b$ +{{qed|lemma}} +=== Sufficient Condition === +Suppose $\exists x, y \in \Z: c = x a + y b$. +From [[Common Divisor Divides Integer Combination]], we have: +:$\gcd \set {a, b} \divides \paren {x a + y b}$ +It follows directly that $\gcd \set {a, b} \divides c$ and the proof is finished. +{{qed}} +\end{proof}<|endoftext|> +\section{GCD with Remainder} +Tags: Greatest Common Divisor + +\begin{theorem} +Let $a, b \in \Z$. +Let $q, r \in \Z$ such that $a = q b + r$. +Then: +:$\gcd \set {a, b} = \gcd \set {b, r}$ +where $\gcd \set {a, b}$ is the [[Definition:Greatest Common Divisor of Integers|greatest common divisor]] of $a$ and $b$. +\end{theorem} + +\begin{proof} +{{begin-eqn}} +{{eqn | o = + | r = \gcd \set {a, b} \divides a \land \gcd \set {a, b} \divides b + | c = {{Defof|Greatest Common Divisor of Integers}} +}} +{{eqn | ll= \leadsto + | o = + | r = \gcd \set {a, b} \divides \paren {a - q b} + | c = [[Common Divisor Divides Integer Combination]] +}} +{{eqn | ll= \leadsto + | o = + | r = \gcd \set {a, b} \divides r + | c = as $r = a - q b$ +}} +{{eqn | ll= \leadsto + | o = + | r = \gcd \set {a, b} \le \gcd \set {b, r} + | c = {{Defof|Greatest Common Divisor of Integers}} +}} +{{end-eqn}} +The argument works the other way about: +{{begin-eqn}} +{{eqn | o = + | r = \gcd \set {b, r} \divides b \land \gcd \set {b, r} \divides r + | c = {{Defof|Greatest Common Divisor of Integers}} +}} +{{eqn | ll= \leadsto + | o = + | r = \gcd \set {b, r} \divides \paren {q b + r} + | c = [[Common Divisor Divides Integer Combination]] +}} +{{eqn | ll= \leadsto + | o = + | r = \gcd \set {b, r} \divides a + | c = as $a = q b + r$ +}} +{{eqn | ll= \leadsto + | o = + | r = \gcd \set {b, r} \le \gcd \set {a, b} + | c = {{Defof|Greatest Common Divisor of Integers}} +}} +{{end-eqn}} +Thus: +:$\gcd \set {a, b} = \gcd \set {b, r}$ +{{qed}} +[[Category:Greatest Common Divisor]] +52xifv9uor3uhps3z23ov9u5wtec8hn +\end{proof}<|endoftext|> +\section{Integer Combination of Coprime Integers} +Tags: Coprime Integers, Integer Combinations, Integer Combination of Coprime Integers + +\begin{theorem} +Two [[Definition:Integer|integers]] are [[Definition:Coprime Integers|coprime]] {{iff}} there exists an [[Definition:Integer Combination|integer combination]] of them equal to $1$: +:$\forall a, b \in \Z: a \perp b \iff \exists m, n \in \Z: m a + n b = 1$ +\end{theorem} + +\begin{proof} +{{begin-eqn}} +{{eqn | l = a + | o = \perp + | r = b +}} +{{eqn | ll= \leadsto + | l = \gcd \set {a, b} + | r = 1 + | c = {{Defof|Coprime Integers}} +}} +{{eqn | ll= \leadsto + | l = \exists m, n \in \Z: m a + n b + | r = 1 + | c = [[Bézout's Lemma]] +}} +{{end-eqn}} +Then we have: +{{begin-eqn}} +{{eqn | l = \exists m, n \in \Z: m a + n b + | r = 1 + | c = +}} +{{eqn | ll= \leadsto + | l = \gcd \set {a, b} + | o = \divides + | r = 1 + | c = [[Set of Integer Combinations equals Set of Multiples of GCD]] +}} +{{eqn | ll= \leadsto + | l = \gcd \set {a, b} + | r = 1 + | c = +}} +{{eqn | ll= \leadsto + | l = a + | o = \perp + | r = b + | c = {{Defof|Coprime Integers}} +}} +{{end-eqn}} +{{qed}} +\end{proof} + +\begin{proof} +=== Sufficient Condition === +Let $a, b \in \Z$ be such that $\exists m, n \in \Z: m a + n b = 1$. +Let $d$ be a [[Definition:Divisor of Integer|divisor]] of both $a$ and $b$. +Then: +:$d \divides m a + n b$ +and so: +:$d \divides 1$ +Thus: +:$d = \pm 1$ +and so: +:$\gcd \set {a, b} = 1$ +Thus, by definition, $a$ and $b$ are [[Definition:Coprime Integers|coprime]]. +{{qed|lemma}} +=== Necessary Condition === +Let $a \perp b$. +Thus they are not both $0$. +Let $S$ be defined as: +:$S = \set {a m + b n: m, n \in \Z}$ +$S$ contains at least one [[Definition:Strictly Positive Integer|strictly positive integer]], because for example $a^2 + b^2 \in S$. +By [[Set of Integers Bounded Below has Smallest Element]], let $d$ be the [[Definition:Smallest Element|smallest element]] of $S$ which is [[Definition:Strictly Positive Integer|strictly positive]]. +Let $d = a x + b y$. +It remains to be shown that $d = 1$. +By the [[Division Theorem]]: +:$a = d q + r$ where $0 \le r < d$ +Then: +{{begin-eqn}} +{{eqn | l = r + | r = a - d q + | c = +}} +{{eqn | r = a - \paren {a x + b y} q + | c = +}} +{{eqn | r = a \paren {1 - x q} + b \paren {- y q} + | c = +}} +{{eqn | o = \in + | r = S + | c = +}} +{{end-eqn}} +But we have that $0 \le r < d$. +We have defined $d$ as the [[Definition:Smallest Element|smallest element]] of $S$ which is [[Definition:Strictly Positive Integer|strictly positive]] +Hence it follows that $r$ cannot therefore be [[Definition:Strictly Positive Integer|strictly positive]] itself. +Hence $r = 0$ and so $a = d q$. +That is: +:$d \divides a$ +By a similar argument: +:$d \divides b$ +and so $d$ is a [[Definition:Common Divisor of Integers|common divisor]] of both $a$ and $b$. +But the [[Definition:Greatest Common Divisor|GCD]] of $a$ and $b$ is $1$. +Thus it follows that, as $d \in S$: +:$\exists m, n \in \Z: m a + n b = 1$ +{{qed}} +\end{proof} + +\begin{proof} +=== Sufficient Condition === +Let $a, b \in \Z$ be such that $\exists m, n \in \Z: m a + n b = 1$. +Let $d$ be a [[Definition:Divisor of Integer|divisor]] of both $a$ and $b$. +Then: +:$d \mathrel \backslash m a + n b$ +and so: +:$d \mathrel \backslash 1$ +Thus: +: $d = \pm 1$ +and so: +: $\gcd \left\{ {a, b}\right\} = 1$ +Thus, by definition, $a$ and $b$ are [[Definition:Coprime Integers|coprime]]. +{{qed|lemma}} +=== Necessary Condition === +Let $a \perp b$. +Thus they are not both $0$. +Let $S$ be defined as: +:$S = \left\{ {\lambda a + \mu b: \lambda, \mu \in \Z}\right\}$ +$S$ contains at least one [[Definition:Strictly Positive Integer|strictly positive integer]], because for example: +: $a \in S$ (setting $\lambda = 1$ and $\mu = 0$) +: $b \in S$ (setting $\lambda = 0$ and $\mu = 1$) +By [[Set of Integers Bounded Below has Smallest Element]], let $d$ be the [[Definition:Smallest Element|smallest element]] of $S$ which is [[Definition:Strictly Positive Integer|strictly positive]]. +Let $d = \alpha a + \beta b$. +Let $c \in S$, such that $\lambda_0 a + \mu_0 b = c$ for some $\lambda_0, \mu_0 \in \Z$. +By the [[Division Algorithm]]: +:$\exists \gamma, \delta \in \Z: c = \gamma d + \delta$ +where $0 \le \delta < d$ +Then: +{{begin-eqn}} +{{eqn | l = \delta + | r = c - \gamma d + | c = +}} +{{eqn | r = \left({\lambda_0 a + \mu_0 b}\right) - \gamma \left({\alpha a + \beta b}\right) + | c = +}} +{{eqn | r = \left({\lambda_0 - \gamma \alpha}\right) a + \left({\mu_0 - \gamma \beta}\right) b + | c = +}} +{{eqn | o = \in + | r = S + | c = +}} +{{end-eqn}} +But we have that $0 \le \delta < d$. +We have defined $d$ as the [[Definition:Smallest Element|smallest element]] of $S$ which is [[Definition:Strictly Positive Integer|strictly positive]] +Hence it follows that $\delta$ cannot therefore be [[Definition:Strictly Positive Integer|strictly positive]] itself. +Hence $\delta = 0$ and so $c = \gamma d$. +That is: +:$d \mathrel \backslash c$ +and so the [[Definition:Smallest Element|smallest element]] of $S$ which is [[Definition:Strictly Positive Integer|strictly positive]] is a [[Definition:Divisor of Integer|divisor]] of both $a$ and $b$. +But $a$ and $b$ are [[Definition:Coprime Integers|coprime]]. +Thus it follows that, as $d \mathrel \backslash 1$: +:$d = 1$ +and so, by definition of $S$: +:$\exists m, n \in \Z: m a + n b = 1$ +{{qed}} +\end{proof}<|endoftext|> +\section{Divisor of Sum of Coprime Integers} +Tags: Number Theory, Divisors, Coprime Integers + +\begin{theorem} +Let $a, b, c \in \Z_{>0}$ such that: +:$a \perp b$ and $c \divides \paren {a + b}$. +where: +:$a \perp b$ denotes $a$ and $b$ are [[Definition:Coprime Integers|coprime]] +:$c \divides \paren {a + b}$ denotes that $c$ is a [[Definition:Divisor of Integer|divisor]] of $a + b$. +Then $a \perp c$ and $b \perp c$. +That is, a [[Definition:Divisor of Integer|divisor]] of the sum of two [[Definition:Coprime Integers|coprime integers]] is [[Definition:Coprime Integers|coprime]] to both. +\end{theorem} + +\begin{proof} +Let $d \in \Z_{>0}: d \divides c \land d \divides a$. +Then: +{{begin-eqn}} +{{eqn | l = d + | o = \divides + | r = \paren {a + b} + | c = as $c \divides \paren {a + b}$ +}} +{{eqn | ll= \leadsto + | l = d + | o = \divides + | r = \paren {a + b - a} + | c = +}} +{{eqn | ll= \leadsto + | l = d + | o = \divides + | r = b + | c = +}} +{{eqn | ll= \leadsto + | l = d + | r = 1 + | c = as $d \divides a$ and $d \divides b$ which are [[Definition:Coprime Integers|coprime]] +}} +{{end-eqn}} +A similar argument shows that if $d \divides c \land d \divides b$ then $d \divides a$. +It follows that: +: $\gcd \set {a, c} = \gcd \set {b, c} = 1$ +Hence the result. +{{qed}} +\end{proof}<|endoftext|> +\section{Integers Divided by GCD are Coprime} +Tags: Greatest Common Divisor, Coprime Integers + +\begin{theorem} +Any pair of [[Definition:Integer|integers]], not both zero, can be reduced to a pair of [[Definition:Coprime Integers|coprime]] ones by dividing them by their [[Definition:Greatest Common Divisor of Integers|GCD]]: +:$\gcd \set {a, b} = d \iff \dfrac a d, \dfrac b d \in \Z \land \gcd \set {\dfrac a d, \dfrac b d} = 1$ +That is: +:$\dfrac a {\gcd \set {a, b} } \perp \dfrac b {\gcd \set {a, b} }$ +\end{theorem} + +\begin{proof} +Let $d = \gcd \set {a, b}$. +We have: +:$d \divides a \iff \exists s \in \Z: a = d s$ +:$d \divides b \iff \exists t \in \Z: b = d t$ +So: +{{begin-eqn}} +{{eqn | l = \exists m, n \in \Z: d + | r = m a + n b + | c = [[Bézout's Identity]] +}} +{{eqn | ll= \leadstoandfrom + | l = d + | r = m d s + n d t + | c = Definition of $s$ and $t$ +}} +{{eqn | ll= \leadstoandfrom + | l = 1 + | r = m s + n t + | c = dividing through by $d$ +}} +{{eqn | ll= \leadstoandfrom + | l = \gcd \set {s, t} + | r = 1 + | c = [[Bézout's Identity]] +}} +{{eqn | ll= \leadstoandfrom + | l = \gcd \set {\frac a d, \frac b d} + | r = 1 + | c = Definition of $s$ and $t$ +}} +{{end-eqn}} +{{qed}} +\end{proof}<|endoftext|> +\section{Product of Coprime Factors} +Tags: Coprime Integers + +\begin{theorem} +Let $a, b, c \in \Z$ such that $a$ and $b$ are [[Definition:Coprime Integers|coprime]]. +Let both $a$ and $b$ be [[Definition:Divisor of Integer|divisors]] of $c$. +Then $a b$ is also a [[Definition:Divisor of Integer|divisor]] of $c$. +That is: +:$a \perp b \land a \divides c \land b \divides c \implies a b \divides c$ +\end{theorem} + +\begin{proof} +We have: +:$a \divides c \implies \exists r \in \Z: c = a r$ +:$b \divides c \implies \exists s \in \Z: c = b s$ +So: +{{begin-eqn}} +{{eqn | l = a + | o = \perp + | r = b + | c = +}} +{{eqn | ll= \leadsto + | l = \exists m, n \in \Z: m a + n b + | r = 1 + | c = [[Integer Combination of Coprime Integers]] +}} +{{eqn | ll= \leadsto + | l = c m a + c n b + | r = c + | c = +}} +{{eqn | ll= \leadsto + | l = b s m a + a r n b + | r = c + | c = +}} +{{eqn | ll= \leadsto + | l = a b \paren {s m + r n} + | r = c + | c = +}} +{{eqn | ll= \leadsto + | l = a b + | o = \divides + | r = c + | c = +}} +{{end-eqn}} +{{qed}} +\end{proof}<|endoftext|> +\section{Existence of Lowest Common Multiple} +Tags: Lowest Common Multiple, Existence of Lowest Common Multiple + +\begin{theorem} +Let $a, b \in \Z: a b \ne 0$. +The [[Definition:Lowest Common Multiple of Integers|lowest common multiple]] of $a$ and $b$, denoted $\lcm \set {a, b}$, always exists. +\end{theorem} + +\begin{proof} +We prove its existence thus: +$a b \ne 0 \implies \size {a b} \ne 0$ +Also $\size {a b} = \pm a b = a \paren {\pm b} = \paren {\pm a} b$. +So it definitely exists, and we can say that: +:$0 < \lcm \set {a, b} \le \size {a b}$ +Now we prove it is the lowest. That is: +:$a \divides n \land b \divides n \implies \lcm \set {a, b} \divides n$ +Let $a, b \in \Z: a b \ne 0, m = \lcm \set {a, b}$. +Let $n \in \Z: a \divides n \land b \divides n$. +We have: +: $n = x_1 a = y_1 b$ +: $m = x_2 a = y_2 b$ +As $m > 0$, we have: +{{begin-eqn}} +{{eqn | l = n + | r = m q + r: 0 \le r < \size m = m + | c = +}} +{{eqn | lll=\leadsto + | l = r + | r = n - m q + | c = +}} +{{eqn | r = 1 \times n + \paren {-q} \times m + | c = +}} +{{eqn | lll=\leadsto + | l = r + | r = x_1 a + \paren {-q} x_2 a + | c = +}} +{{eqn | r = y_1 b + \paren {-q} y_2 b + | c = +}} +{{eqn | lll=\leadsto + | l = a + | o = \divides + | r = r + | c = +}} +{{eqn | ll= \land + | l = b + | o = \divides + | r = r + | c = +}} +{{end-eqn}} +Since $r < m$, and $m$ is the smallest ''positive'' [[Definition:Common Multiple|common multiple]] of $a$ and $b$, it follows that $r = 0$. +So: +: $\forall n \in \Z: a \divides n \land b \divides n: \lcm \set {a, b} \divides n$ +That is, $\lcm \set {a, b}$ divides any [[Definition:Common Multiple|common multiple]] of $a$ and $b$. +{{qed}} +\end{proof} + +\begin{proof} +Either $a$ and $b$ are [[Definition:Coprime Integers|coprime]] or they are not. +Let: +:$a \perp b$ +where $a \perp b$ denotes that $a$ and $b$ are [[Definition:Coprime Integers|coprime]]. +Let $a b = c$. +Then: +:$a \divides c, b \divides c$ +where $a \divides c$ denotes that $a$ is a [[Definition:Divisor of Integer|divisor]] of $c$. +Suppose both $a \divides d, b \divides d$ for some $d \in \N_{> 0}: d < c$. +Then: +:$\exists e \in \N_{> 0}: a e = d$ +:$\exists f \in \N_{> 0}: b f = d$ +Therefore: +:$a e = b f$ +and from {{EuclidPropLink|book = VII|prop = 19|title = Relation of Ratios to Products}}: +:$a : b = f : e$ +But $a$ and $b$ are [[Definition:Coprime Integers|coprime]]. +From: +:{{EuclidPropLink|prop = 21|title = Coprime Numbers form Fraction in Lowest Terms}} +and: +:{{EuclidPropLink|prop = 20|title = Ratios of Fractions in Lowest Terms}} +it follows that $b \divides e$ +Since: +:$a b = c$ +and: +:$a e = d$ +it follows from {{EuclidPropLink|prop = 17|title = Multiples of Ratios of Numbers}} that: +:$b : e = c : d$ +But $b \divides e$ and therefore: +:$c \divides d$ +But $c > d$ which is impossible. +Therefore $a$ and $b$ are both the [[Definition:Divisor of Integer|divisor]] of no number less than $c$. +Now suppose $a$ and $b$ are not [[Definition:Coprime Integers|coprime]]. +Let $f$ and $e$ be the least numbers of those which have the same [[Definition:Ratio|ratio]] with $a$ and $b$. +Then from {{EuclidPropLink|prop = 19|title = Relation of Ratios to Products}}: +:$a e = b f$ +Let $a e = c$. +Then $b f = c$. +Hence: +:$a \divides c$ +:$b \divides c$ +Suppose $a$ and $b$ are both the [[Definition:Divisor of Integer|divisor]] of some number $d$ which is less than $c$. +Let: +:$a g = d$ +and: +:$b h = d$ +Therefore: +:$a g = b h$ +and so by {{EuclidPropLink|prop = 19|title = Relation of Ratios to Products}}: +:$a : b = f : e$ +Also: +:$f : e = h : g$ +But $f, e$ are the least such. +From {{EuclidPropLink|prop = 20|title = Ratios of Fractions in Lowest Terms}}: +:$e \divides g$ +Since $a e = c$ and $a g = d$, from {{EuclidPropLink|prop = 17|title = Multiples of Ratios of Numbers}}: +:$e : g = c : d$ +But: +:$e \divides g$ +So $c \divides d$ +But $c > d$ which is impossible. +Therefore $a$ and $b$ are both the [[Definition:Divisor of Integer|divisor]] of no number less than $c$. +{{qed}} +\end{proof}<|endoftext|> +\section{Product of GCD and LCM} +Tags: Greatest Common Divisor, Lowest Common Multiple, Product of GCD and LCM + +\begin{theorem} +:$\lcm \set {a, b} \times \gcd \set {a, b} = \size {a b}$ +where: +:$\lcm \set {a, b}$ denotes the [[Definition:Lowest Common Multiple of Integers|lowest common multiple]] of $a$ and $b$ +:$\gcd \set {a, b}$ denotes the [[Definition:Greatest Common Divisor of Integers|greatest common divisor]] of $a$ and $b$. +\end{theorem} + +\begin{proof} +It is sufficient to prove that $\lcm \set {a, b} \times \gcd \set {a, b} = a b$, where $a, b \in \Z_{>0}$. +{{begin-eqn}} +{{eqn | l = d + | r = \gcd \set {a, b} + | c = +}} +{{eqn | ll= \leadsto + | l = d + | o = \divides + | r = a b + | c = +}} +{{eqn | ll= \leadsto + | lo= \exists n \in \Z_{>0}: + | l = a b + | r = d n + | c= +}} +{{end-eqn}} +{{begin-eqn}} +{{eqn | l = d \divides a + | o = \land + | r = d \divides b + | c = +}} +{{eqn | ll= \leadsto + | lo= \exists u, v \in \Z: + | l = a = d u + | o = \land + | r = b = d v + | c = +}} +{{eqn | ll= \leadsto + | l = d u b = d n + | o = \land + | r = a d v = d n + | c = +}} +{{eqn | ll= \leadsto + | l = n = b u + | o = \land + | r = n = a v + | c = +}} +{{eqn | ll= \leadsto + | l = a \divides n + | o = \land + | r = b \divides n + | c = +}} +{{end-eqn}} +Now we have $a \divides m \land b \divides m \implies m = a r = b s$. +Also, by [[Bézout's Lemma]] we have $d = a x + b y$. +So: +{{begin-eqn}} +{{eqn | l = m d + | r = a x m + b y m + | c = +}} +{{eqn | r = b s a x + a r b y + | c = +}} +{{eqn | r = a b \paren {s x + r y} + | c = +}} +{{eqn | r = d n \paren {s x + r y} + | c = +}} +{{end-eqn}} +So: +:$m = n \paren {s x + r y}$ +Thus: +:$n \divides m \implies n \le \size m$ +while: +:$a b = d n = \gcd \set {a, b} \times \lcm \set {a, b}$ +as required. +{{qed}} +\end{proof} + +\begin{proof} +Let $a = g m$ and $b = g n$, where $g = \gcd \set {a, b}$ and $m$ and $n$ are [[Definition:Coprime Integers|coprime]]. +The existence of $m$ and $n$ are proved by [[Integers Divided by GCD are Coprime]]. +Since $a = g m \divides g m n$ and $b = g n \divides g m n$, $g m n$ is the [[Definition:Lowest Common Multiple of Integers|LCM]] of $a$ and $b$. +{{explain|why?}} +Then it follows that: +:$\lcm \set {a, b} \times \gcd \set {a, b} = g m n \times g = g m \times g n = \size {a b}$ +{{qed}} +\end{proof} + +\begin{proof} +Let $d := \gcd \set {a, b}$. +Then by definition of the [[Definition:Greatest Common Divisor of Integers|GCD]], there exist $j_1, j_2 \in \Z$ such that $a = d j_1$ and $b = d j_2$. +Because $d$ [[Definition:Divisor of Integer|divides]] both $a$ and $b$, it must [[Definition:Divisor of Integer|divide]] their [[Definition:Integer Multiplication|product]]: +:$\exists l \in \Z$ such that $a b = d l$ +Then we have: +{{begin-eqn}} +{{eqn | l = d l + | m = \paren {d j_1} b + | mo= = + | r = a \paren {d j_2} +}} +{{eqn | ll= \leadsto + | l = l + | m = j_1 b + | mo= = + | r = a j_2 +}} +{{end-eqn}} +showing that $a \divides l$ and $b \divides l$. +That is, $l$ is a [[Definition:Common Multiple|common multiple]] of $a$ and $b$. +Now it must be shown that $l$ is the least such number. +Let $m$ be any [[Definition:Common Multiple|common multiple]] of $a$ and $b$. +Then there exist $k_1, k_2 \in \Z$ such that $m = a k_1 = b k_2$. +By [[Bézout's Lemma]]: +:$\exists x, y \in \Z: d = a x + b y$ +So: +{{begin-eqn}} +{{eqn | l = m d + | r = m a x + m b y +}} +{{eqn | r = \paren {b k_2} a x + \paren {a k_1} b y +}} +{{eqn | r = a b \paren {b k_2 + a k_1} +}} +{{eqn | r = d l \paren {b k_2 + a k_1} +}} +{{end-eqn}} +Thus: +:$m = l \paren {b k_2 + a k_1}$ +that is, $l \divides m$. +Hence by definition of the [[Definition:Lowest Common Multiple of Integers|LCM]]: +:$\lcm \set {a, b} = l$ +In conclusion: +:$a b = d l = \gcd \set {a, b} \cdot \lcm \set {a, b}$ +{{qed}} +\end{proof} + +\begin{proof} +Let: +{{begin-eqn}} +{{eqn | l = m + | r = {p_1}^{k_1} {p_2}^{k_2} \dotsm {p_r}^{k_r} +}} +{{eqn | l = n + | r = {p_1}^{l_1} {p_2}^{l_2} \dotsm {p_r}r^{l_r} + | c = +}} +{{end-eqn}} +From [[LCM from Prime Decomposition]]: +:$\lcm \set {m, n} = p_1^{\max \set {k_1, l_1} } p_2^{\max \set {k_2, l_2} } \dotsm p_r^{\max \set {k_r, l_r} }$ +From [[GCD from Prime Decomposition]]: +:$\gcd \set {m, n} = p_1^{\min \set {k_1, l_1} } p_2^{\min \set {k_2, l_2} } \dotsm p_r^{\min \set {k_r, l_r} }$ +From [[Sum of Maximum and Minimum]], for all $i \in \set {1, 2, \ldots, r}$: +:$\min \set {k_i, l_i} + \max \set {k_i, l_i} = k_i + l_i$ +Hence: +{{begin-eqn}} +{{eqn | l = \gcd \set {m, n} \times \lcm \set {m, n} + | r = p_1^{k_1 + l_1} p_2^{k_2 + l_2} \dotsm p_r^{k_r + l_r} + | c = +}} +{{eqn | r = p_1^{k_1} p_1^{l_1} p_2^{k_2} p_2^{l_2} \dotsm p_r^{k_r} p_r^{l_r} + | c = +}} +{{eqn | r = p_1^{k_1} p_2^{k_2} \dotsm p_r^{k_r} \times p_1^{l_1} p_2^{l_2} \dotsm p_r^{l_r} + | c = +}} +{{eqn | r = m n + | c = +}} +{{end-eqn}} +{{qed}} +\end{proof}<|endoftext|> +\section{Congruent to Zero if Modulo is Divisor} +Tags: Modulo Arithmetic + +\begin{theorem} +Let $a, z \in \R$. +Then $a$ is [[Definition:Congruence (Number Theory)|congruent]] to $0$ modulo $z$ {{iff}} $a$ is an [[Definition:Integer Multiple|integer multiple]] of $z$. +:$\exists k \in \Z: k z = a \iff a \equiv 0 \pmod z$ +If $z \in \Z$, then further: +:$z \divides a \iff a \equiv 0 \pmod z$ +\end{theorem} + +\begin{proof} +{{begin-eqn}} +{{eqn | l = \exists k \in \Z: a + | r = k z + | c = +}} +{{eqn | ll= \leadstoandfrom + | l = \exists k \in \Z: a + | r = 0 + k z + | c = +}} +{{end-eqn}} +Thus by definition of [[Definition:Congruence (Number Theory)|congruence]], $a \equiv 0 \pmod z$ and the result is proved. +If $z$ is an [[Definition:Integer|integer]], then by definition of [[Definition:Divisor of Integer|divisor]]: +:$z \divides a \iff \exists k \in \Z: a = k z$ +Hence the result for [[Definition:Integer|integer]] $z$. +{{qed}} +[[Category:Modulo Arithmetic]] +87apiiualuplsor2u54hkqv8t4i4x69 +\end{proof}<|endoftext|> +\section{Integer is Congruent Modulo Divisor to Remainder} +Tags: Modulo Arithmetic + +\begin{theorem} +Let $a \in \Z$. +Let $a$ have a [[Definition:Remainder|remainder]] $r$ on [[Definition:Integer Division|division]] by $m$. +Then: +: $a \equiv r \pmod m$ +where the notation denotes that $a$ and $r$ are [[Definition:Congruence Modulo Integer|congruent modulo $m$]]. +\end{theorem} + +\begin{proof} +Let $a$ have a [[Definition:Remainder|remainder]] $r$ on [[Definition:Integer Division|division]] by $m$. +Then: +: $\exists q \in \Z: a = q m + r$ +Hence by definition of [[Definition:Congruence Modulo Integer|congruence modulo $m$]]: +:$a \equiv r \pmod m$ +{{qed}} +\end{proof}<|endoftext|> +\section{Integer is Congruent to Integer less than Modulus} +Tags: Modulo Arithmetic + +\begin{theorem} +Let $m \in \Z$. +Then each [[Definition:Integer|integer]] is [[Definition:Congruence Modulo Integer|congruent (modulo $m$)]] to precisely one of the [[Definition:Integer|integers]] $0, 1, \ldots, m - 1$. +\end{theorem} + +\begin{proof} +=== Proof of Existence === +Let $a \in \Z$. +Then from the [[Division Theorem]]: $\exists r \in \set {0, 1, \ldots, m - 1}: a \equiv r \pmod m$. +=== Proof of Uniqueness === +Suppose that: +:$\exists r_1, r_2 \in \set {0, 1, \ldots, m - 1}: a \equiv r_1 \pmod m \land a \equiv r_2 \pmod m$ +Then: +:$\exists r_1, r_2 \in \Z: a = q_1 m + r_1 = q_2 m + r_2$ +This contradicts the uniqueness clause in the [[Division Theorem]]. +{{qed}} +\end{proof}<|endoftext|> +\section{Modulo Addition is Well-Defined} +Tags: Modulo Addition, Modulo Addition is Well-Defined + +\begin{theorem} +Let $m \in \Z$ be an [[Definition:Integer|integer]]. +Let $\Z_m$ be the [[Definition:Integers Modulo m|set of integers modulo $m$]]. +The [[Definition:Modulo Addition|modulo addition]] operation on $\Z_m$, defined by the rule: +:$\eqclass a m +_m \eqclass b m = \eqclass {a + b} m$ +is a [[Definition:Well-Defined Operation|well-defined operation]]. +That is: +:If $a \equiv b \pmod m$ and $x \equiv y \pmod m$, then $a + x \equiv b + y \pmod m$. +\end{theorem} + +\begin{proof} +The equivalence class $\eqclass a m$ is defined as: +:$\eqclass a m = \set {x \in \Z: x = a + k m: k \in \Z}$ +That is, the set of all [[Definition:Integer|integers]] which differ from $a$ by an [[Definition:Integer Multiple|integer multiple]] of $m$. +Thus the notation for addition of two [[Definition:Integers Modulo m|set of integers modulo $m$]] is not usually $\eqclass a m +_m \eqclass b m$. +What is more normally seen is $a + b \pmod m$. +Using this notation: +{{begin-eqn}} +{{eqn | l = a + | o = \equiv + | r = b + | rr= \pmod m + | c = +}} +{{eqn | lo= \land + | l = c + | o = \equiv + | r = d + | rr= \pmod m + | c = +}} +{{eqn | ll= \leadsto + | l = a \bmod m + | r = b \bmod m + | c = {{Defof|Congruence Modulo Integer}} +}} +{{eqn | lo= \land + | l = c \bmod m + | r = d \bmod m + | c = +}} +{{eqn | ll= \leadsto + | l = a + | r = b + k_1 m + | c = for some $k_1 \in \Z$ +}} +{{eqn | lo= \land + | l = c + | r = d + k_2 m + | c = for some $k_2 \in \Z$ +}} +{{eqn | ll= \leadsto + | l = a + c + | r = b + d + \paren {k_1 + k_2} m + | c = {{Defof|Integer Addition}} +}} +{{eqn | ll= \leadsto + | l = a + c + | o = \equiv + | r = b + d + | rr= \pmod m + | c = {{Defof|Modulo Addition}} +}} +{{end-eqn}} +{{qed}} +\end{proof} + +\begin{proof} +We need to show that if: +:$\eqclass {x'} m = \eqclass x m$ +:$\eqclass {y'} m = \eqclass y m$ +then: +:$\eqclass {x' + y'} m = \eqclass {x + y} m$ +Since: +:$\eqclass {x'} m = \eqclass x m$ +and: +:$\eqclass {y'} m = \eqclass y m$ +it follows from the definition of [[Definition:Integers Modulo m|set of integers modulo $m$]] that: +:$x \equiv x' \pmod m$ +and: +:$y \equiv y' \pmod m$ +By definition, we have: +:$x \equiv x' \pmod m \implies \exists k_1 \in \Z: x = x' + k_1 m$ +:$y \equiv y' \pmod m \implies \exists k_2 \in \Z: y = y' + k_2 m$ +which gives us: +:$x + y = x' + k_1 m + y' + k_2 m = x' + y' + \left({k_1 + k_2}\right) m$ +As $k_1 + k_2$ is an [[Definition:Integer|integer]], it follows that, by definition: +:$x + y \equiv \left({x' + y'}\right) \pmod m$ +Therefore, by the definition of [[Definition:Integers Modulo m|integers modulo $m$]]: +:$\eqclass {x' + y'} m = \eqclass {x + y} m$ +{{qed}} +\end{proof}<|endoftext|> +\section{Modulo Multiplication is Well-Defined} +Tags: Modulo Multiplication, Modulo Multiplication is Well-Defined + +\begin{theorem} +The [[Definition:Modulo Multiplication|multiplication modulo $m$]] operation on $\Z_m$, the set of [[Definition:Integers Modulo m|integers modulo $m$]], defined by the rule: +:$\eqclass x m \times_m \eqclass y m = \eqclass {x y} m$ +is a [[Definition:Well-Defined Operation|well-defined operation]]. +That is: +:If $a \equiv b \pmod m$ and $x \equiv y \pmod m$, then $a x \equiv b y \pmod m$. +\end{theorem} + +\begin{proof} +We need to show that if: +:$\eqclass {x'} m = \eqclass x m$ +and: +:$\eqclass {y'} m = \eqclass y m$ +then: +:$\eqclass {x' y'} m = \eqclass {x y} m$ +We have that: +:$\eqclass {x'} m = \eqclass x m$ +and: +:$\eqclass {y'} m = \eqclass y m$ +It follows from the definition of [[Definition:Residue Class|residue class modulo $m$]] that: +:$x \equiv x' \pmod m$ +and: +:$y \equiv y' \pmod m$ +By definition, we have: +:$x \equiv x' \pmod m \implies \exists k_1 \in \Z: x = x' + k_1 m$ +:$y \equiv y' \pmod m \implies \exists k_2 \in \Z: y = y' + k_2 m$ +which gives us: +:$x y = \paren {x' + k_1 m} \paren {y' + k_2 m} = x' y' + \paren {x' k_2 + y' k_1} m + k_1 k_2 m^2$ +Thus by definition: +:$x y \equiv \paren {x' y'} \pmod m$ +Therefore, by the definition of [[Definition:Residue Class|residue class modulo $m$]]: +:$\eqclass {x' y'} m = \eqclass {x y} m$ +{{qed}} +\end{proof} + +\begin{proof} +The equivalence class $\eqclass a m$ is defined as: +:$\eqclass a m = \set {x \in \Z: x = a + k m: k \in \Z}$ +that is, the set of all [[Definition:Integer|integers]] which differ from $a$ by an [[Definition:Integer Multiple|integer multiple]] of $m$. +Thus the notation for addition of two [[Definition:Residue Class|residue classes modulo $z$]] is not usually $\eqclass a m +_m \eqclass b m$. +What is more normally seen is: +: $a b \pmod m$ +Using this notation: +{{begin-eqn}} +{{eqn | l = a + | o = \equiv + | r = b + | rr= \pmod m + | c = +}} +{{eqn | lo= \land + | l = c + | o = \equiv + | r = d + | rr= \pmod m + | c = +}} +{{eqn | ll= \leadsto + | l = a \bmod m + | r = b \bmod m + | c = {{Defof|Congruence Modulo Integer}} +}} +{{eqn | lo= \land + | l = c \bmod m + | r = d \bmod m + | c = +}} +{{eqn | ll= \leadsto + | l = a + | r = b + k_1 m + | c = for some $k_1 \in \Z$ +}} +{{eqn | lo= \land + | l = c + | r = d + k_2 m + | c = for some $k_2 \in \Z$ +}} +{{eqn | ll= \leadsto + | l = a c + | r = \paren {b + k_1 m} \paren {d + k_2 m} + | c = {{Defof|Multiplication}} +}} +{{eqn | r = b d + b k_2 m + d k_1 m + k_1 k_2 m^2 + | c = [[Integer Multiplication Distributes over Addition]] +}} +{{eqn | r = b d + \paren {b k_2 + d k_1 + k_1 k_2 m} m + | c = +}} +{{eqn | ll= \leadsto + | l = a c + | o = \equiv + | r = b d + | rr= \pmod m + | c = {{Defof|Modulo Multiplication}} +}} +{{end-eqn}} +{{qed}} +\end{proof}<|endoftext|> +\section{Congruence of Product} +Tags: Modulo Arithmetic + +\begin{theorem} +Let $a, b, z \in \R$. +Let $a$ be [[Definition:Congruence (Number Theory)|congruent to $b$ modulo $z$]], that is: +:$a \equiv b \pmod z$ +Then: +:$\forall m \in \Z: m a \equiv m b \pmod z$ +\end{theorem} + +\begin{proof} +Let $m \in \Z$ and $a \equiv b \pmod z$. +Suppose $m = 0$. +Then the {{RHS}} of the assertion degenerates to $0 \equiv 0 \pmod z$ which is trivially true. +Otherwise, from [[Congruence by Product of Moduli]], we have: +:$a \equiv b \iff m a \equiv m b \pmod z$ +As $m \in \Z$, it follows that $m z$ is an [[Definition:Integer Multiple|integer multiple]] of $z$. +Hence from [[Congruence by Divisor of Modulus]], it follows that: +:$m a \equiv m b \implies m a \equiv m b \pmod z$ +{{qed}} +\end{proof}<|endoftext|> +\section{Congruence of Powers} +Tags: Modulo Arithmetic + +\begin{theorem} +Let $a, b \in \R$ and $m \in \Z$. +Let $a$ be [[Definition:Congruence Modulo Integer|congruent to $b$ modulo $m$]], that is: +:$a \equiv b \pmod m$ +Then: +:$\forall n \in \Z_{\ge 0}: a^n \equiv b^n \pmod m$ +\end{theorem} + +\begin{proof} +Proof by [[Principle of Mathematical Induction|induction]]: +For all $n \in \Z_{\ge 0}$, let $\map P n$ be the [[Definition:Proposition|proposition]]: +:$a \equiv b \implies a^k \equiv b^k \pmod m$ +$\map P 0$ is trivially true, as $a^0 = b^0 = 1$. +$\map P 1$ is true, as this just says: +:$a \equiv b \pmod m$ +=== Basis for the Induction === +$\map P 2$ is the case: +:$a^2 \equiv b^2 \pmod m$ +which follows directly from the fact that [[Modulo Multiplication is Well-Defined]]. +This is our [[Principle of Mathematical Induction#Basis for the Induction|basis for the induction]]. +=== Induction Hypothesis === +Now we need to show that, if $\map P k$ is true, where $k \ge 2$, then it logically follows that $\map P {k + 1}$ is true. +So this is our [[Principle of Mathematical Induction#Induction Hypothesis|induction hypothesis]]: +:$a \equiv b \implies a^k \equiv b^k \pmod m$ +Then we need to show: +:$a \equiv b \implies a^{k + 1} \equiv b^{k + 1} \pmod m$ +=== Induction Step === +This is our [[Principle of Mathematical Induction#Induction Step|induction step]]: +Suppose $a^k \equiv b^k \pmod m$. +Then $a^k a \equiv b^k b \pmod m$ by definition of [[Definition:Modulo Multiplication|modulo multiplication]]. +Thus $a^{k + 1} \equiv b^{k + 1} \pmod m$. +So $\map P k \implies \map P {k + 1}$ and the result follows by the [[Principle of Mathematical Induction]]. +Therefore: +:$\forall n \in \Z_+: a \equiv b \implies a^n \equiv b^n \pmod m$ +{{qed}} +\end{proof}<|endoftext|> +\section{Modulo Addition is Associative} +Tags: Modulo Addition + +\begin{theorem} +[[Definition:Modulo Addition|Addition modulo $m$]] is [[Definition:Associative|associative]]: +:$\forall \eqclass x m, \eqclass y m, \eqclass z m \in \Z_m: \paren {\eqclass x m +_m \eqclass y m} +_m \eqclass z m = \eqclass x m +_m \paren {\eqclass y m +_m \eqclass z m}$ +where $\Z_m$ is the [[Definition:Integers Modulo m|set of integers modulo $m$]]. +That is: +:$\forall x, y, z \in \Z: \paren {x + y} + z \equiv x + \paren {y + z} \pmod m$ +\end{theorem} + +\begin{proof} +{{begin-eqn}} +{{eqn | l = \paren {\eqclass x m +_m \eqclass y m} +_m \eqclass z m + | r = \eqclass {x + y} m +_m \eqclass z m + | c = {{Defof|Modulo Addition}} +}} +{{eqn | r = \eqclass {\paren {x + y} + z} m + | c = {{Defof|Modulo Addition}} +}} +{{eqn | r = \eqclass {x + \paren {y + z} } m + | c = [[Associative Law of Addition]] +}} +{{eqn | r = \eqclass x m +_m \eqclass {y + z} m + | c = {{Defof|Modulo Addition}} +}} +{{eqn | r = \eqclass x m +_m \paren {\eqclass y m +_m \eqclass z m} + | c = {{Defof|Modulo Addition}} +}} +{{end-eqn}} +{{qed}} +\end{proof}<|endoftext|> +\section{Modulo Addition is Commutative} +Tags: Modulo Addition + +\begin{theorem} +[[Definition:Modulo Addition|Modulo addition]] is [[Definition:Commutative Operation|commutative]]: +:$\forall x, y, z \in \Z: x + y \pmod m = y + x \pmod m$ +\end{theorem} + +\begin{proof} +From the definition of [[Definition:Modulo Addition|modulo addition]], this is also written: +:$\forall m \in \Z: \forall \eqclass x m, \eqclass y m \in \Z_m: \eqclass x m +_m \eqclass y m = \eqclass y m +_m \eqclass x m$ +Hence: +{{begin-eqn}} +{{eqn | l = \eqclass x m +_m \eqclass y m + | r = \eqclass {x + y} m + | c = {{Defof|Modulo Addition}} +}} +{{eqn | r = \eqclass {y + x} m + | c = [[Commutative Law of Addition]] +}} +{{eqn | r = \eqclass y m +_m \eqclass x m + | c = {{Defof|Modulo Addition}} +}} +{{end-eqn}} +{{qed}} +\end{proof}<|endoftext|> +\section{Modulo Addition has Identity} +Tags: Modulo Addition + +\begin{theorem} +Let $m \in \Z$ be an [[Definition:Integer|integer]]. +Then [[Definition:Modulo Addition|addition modulo $m$]] has an [[Definition:Identity Element|identity]]: +:$\forall \eqclass x m \in \Z_m: \eqclass x m +_m \eqclass 0 m = \eqclass x m = \eqclass 0 m +_m \eqclass x m$ +That is: +:$\forall a \in \Z: a + 0 \equiv a \equiv 0 + a \pmod m$ +\end{theorem} + +\begin{proof} +{{begin-eqn}} +{{eqn | l = \eqclass x m +_m \eqclass 0 m + | r = \eqclass {x + 0} m + | c = {{Defof|Modulo Addition}} +}} +{{eqn | r = \eqclass x m + | c = +}} +{{eqn | r = \eqclass {0 + x} m + | c = +}} +{{eqn | r = \eqclass 0 m +_m \eqclass x m + | c = {{Defof|Modulo Addition}} +}} +{{end-eqn}} +Thus $\eqclass 0 m$ is the [[Definition:Identity Element|identity]] for [[Definition:Modulo Addition|addition modulo $m$]]. +{{Qed}} +\end{proof}<|endoftext|> +\section{Modulo Addition has Inverses} +Tags: Modulo Addition + +\begin{theorem} +Let $m \in \Z$ be an [[Definition:Integer|integer]]. +Then [[Definition:Modulo Addition|addition modulo $m$]] has [[Definition:Inverse Element|inverses]]: +For each [[Definition:Element|element]] $\eqclass x m \in \Z_m$, there exists the element $\eqclass {-x} m \in \Z_m$ with the property: +:$\eqclass x m +_m \eqclass {-x} m = \eqclass 0 m = \eqclass {-x} m +_m \eqclass x m$ +where $\Z_m$ is the [[Definition:Integers Modulo m|set of integers modulo $m$]]. +That is: +:$\forall a \in \Z: a + \paren {-a} \equiv 0 \equiv \paren {-a} + a \pmod m$ +\end{theorem} + +\begin{proof} +{{begin-eqn}} +{{eqn | l = \eqclass x m +_m \eqclass {-x} m + | r = \eqclass {x + \paren {-x} } m + | c = {{Defof|Modulo Addition}} +}} +{{eqn | r = \eqclass 0 m + | c = +}} +{{eqn | r = \eqclass {\paren {-x} + x} m + | c = +}} +{{eqn | r = \eqclass {-x} m +_m \eqclass x m + | c = {{Defof|Modulo Addition}} +}} +{{end-eqn}} +As $-x$ is a perfectly good [[Definition:Integer|integer]], $\eqclass {-x} m \in \Z_m$, whatever $x$ may be. +{{qed}} +\end{proof}<|endoftext|> +\section{Modulo Multiplication is Closed} +Tags: Modulo Multiplication + +\begin{theorem} +[[Definition:Modulo Multiplication|Multiplication modulo $m$]] is [[Definition:Closed Algebraic Structure|closed]] on the set of [[Definition:Integers Modulo m|integers modulo $m$]]: +:$\forall \eqclass x m, \eqclass y m \in \Z_m: \eqclass x m \times_m \eqclass y m \in \Z_m$. +\end{theorem} + +\begin{proof} +From the definition of [[Definition:Modulo Multiplication|multiplication modulo $m$]], we have: +:$\eqclass x m \times_m \eqclass y m = \eqclass {x y} m$ +By the [[Division Theorem]]: +:$x y = q m + r$ where $0 \le r < m$ +Therefore for all $0 \le r < m$: +:$\eqclass {x y} m = \eqclass r m$ +Therefore from the definition of [[Definition:Integers Modulo m|integers modulo $m$]]: +:$\eqclass {x y} m \in \Z_m$ +{{qed}} +\end{proof}<|endoftext|> +\section{Modulo Multiplication is Associative} +Tags: Modulo Multiplication, Modulo Multiplication is Associative + +\begin{theorem} +[[Definition:Modulo Multiplication|Multiplication modulo $m$]] is [[Definition:Associative|associative]]: +:$\forall \eqclass x m, \eqclass y m, \eqclass z m \in \Z_m: \paren {\eqclass x m \times_m \eqclass y m} \times_m \eqclass z m = \eqclass x m \times_m \paren {\eqclass y m \times_m \eqclass z m}$ +That is: +:$\forall x, y, z \in \Z_m: \paren {x \cdot_m y} \cdot_m z = x \cdot_m \paren {y \cdot_m z}$ +\end{theorem} + +\begin{proof} +{{begin-eqn}} +{{eqn | l = \paren {\eqclass x m \times_m \eqclass y m} \times_m \eqclass z m + | r = \eqclass {x y} m \times_m \eqclass z m + | c = {{Defof|Modulo Multiplication}} +}} +{{eqn | r = \eqclass {\paren {x y} z} m + | c = {{Defof|Modulo Multiplication}} +}} +{{eqn | r = \eqclass {x \paren {y z} } m + | c = [[Integer Multiplication is Associative]] +}} +{{eqn | r = \eqclass x m \times_m \eqclass {y z} m + | c = {{Defof|Modulo Multiplication}} +}} +{{eqn | r = \eqclass x m \times_m \paren {\eqclass y m \times_m \eqclass z m} + | c = {{Defof|Modulo Multiplication}} +}} +{{end-eqn}} +{{qed}} +\end{proof} + +\begin{proof} +Let $j$ be the largest [[Definition:Integer|integer]] such that: +:$j m \le x y$ +Let $p$ be the largest [[Definition:Integer|integer]] such that: +:$p m \le y z$ +By definition of [[Definition:Modulo Multiplication|multiplication modulo $m$]]: +:$x \cdot_m y = x y - j m$ +:$y \cdot_m z = y z - p m$ +Let $k$ be the largest [[Definition:Integer|integer]] such that: +:$k m \le \paren {x y - j m} z$ +Let $q$ be the largest [[Definition:Integer|integer]] such that: +:$q m \le x \paren {y z - p m}$ +Then: +:$\paren {j z + k} m \le \paren {x y} z$ +:$\paren {q + x p} m \le x \paren {y z}$ +Thus: +{{begin-eqn}} +{{eqn | l = \paren {x \cdot_m y} \cdot_m z + | r = \paren {x y - j m} z - k m + | c = {{Defof|Modulo Multiplication}} +}} +{{eqn | l = x \cdot_m \paren {y \cdot_m z} + | r = x \left({y z - p m}\right) - q m + | c = {{Defof|Modulo Multiplication}} +}} +{{end-eqn}} +But suppose that there exists an [[Definition:Integer|integer]] $s$ such that: +:$s m \le \paren {x y} z$ +and: +:$j z + k < s$ +Then: +:$\paren {j z + k + 1} m \le \paren {x y} z$ +from which: +:$\paren {k + 1} m \le \paren {x y - j m} z$ +But this contradicts the definition of $k$. +Thus $j z + k$ is the largest of those [[Definition:Integer|integers]] $i$ such that $i m \le \paren {x y} z$. +Similarly, $q + x p$ is the largest of those [[Definition:Integer|integers]] $i$ such that $i m \le x \paren {y z}$. +From [[Integer Multiplication is Associative]]: +:$\paren {x y} z = x \paren {y z}$ +Thus $j z + k = q + x p$ and so: +{{begin-eqn}} +{{eqn | l = \paren {x \cdot_m y} \cdot_m z + | r = \paren {x y - j m} z - k m + | c = {{Defof|Modulo Multiplication}} +}} +{{eqn | r = x y z - \paren {j z + k} m + | c = +}} +{{eqn | r = x y z - \paren {q + x p} m + | c = +}} +{{eqn | r = x \paren {y z - p m} - q m + | c = +}} +{{eqn | r = x \cdot_m \paren {y \cdot_m z} + | c = {{Defof|Modulo Multiplication}} +}} +{{end-eqn}} +{{qed}} +\end{proof}<|endoftext|> +\section{Modulo Multiplication is Commutative} +Tags: Modulo Multiplication + +\begin{theorem} +[[Definition:Modulo Multiplication|Multiplication modulo $m$]] is [[Definition:Commutative Operation|commutative]]: +:$\forall \eqclass x m, \eqclass y m \in \Z_m: \eqclass x m \times_m \eqclass y m = \eqclass y m \times_m \eqclass x m$ +\end{theorem} + +\begin{proof} +{{begin-eqn}} +{{eqn | l = \eqclass x m \times_m \eqclass y m + | r = \eqclass {x y} m + | c = {{Defof|Modulo Multiplication}} +}} +{{eqn | r = \eqclass {y x} m + | c = [[Integer Multiplication is Commutative]] +}} +{{eqn | r = \eqclass y m \times_m \eqclass x m + | c = {{Defof|Modulo Multiplication}} +}} +{{end-eqn}} +{{qed}} +\end{proof}<|endoftext|> +\section{Modulo Multiplication has Identity} +Tags: Modulo Multiplication + +\begin{theorem} +[[Definition:Modulo Multiplication|Multiplication modulo $m$]] has an [[Definition:Identity Element|identity]]: +:$\forall \eqclass x m \in \Z_m: \eqclass x m \times_m \eqclass 1 m = \eqclass x m = \eqclass 1 m \times_m \eqclass x m$ +\end{theorem} + +\begin{proof} +Follows directly from the definition of [[Definition:Modulo Multiplication|multiplication modulo $m$]]: +{{begin-eqn}} +{{eqn | l = \eqclass x m \times_m \eqclass 1 m + | r = \eqclass {x \times 1} m + | c = +}} +{{eqn | r = \eqclass x m + | c = +}} +{{eqn | r = \eqclass {1 \times x} m + | c = +}} +{{eqn | r = \eqclass 1 m \times_m \eqclass x m + | c = +}} +{{end-eqn}} +Thus $\eqclass 1 m$ is the [[Definition:Identity Element|identity]] for [[Definition:Modulo Multiplication|multiplication modulo $m$]]. +{{qed}} +\end{proof}<|endoftext|> +\section{Modulo Multiplication Distributes over Modulo Addition} +Tags: Modulo Multiplication, Modulo Addition, Distributive Operations + +\begin{theorem} +[[Definition:Modulo Multiplication|Multiplication modulo $m$]] is [[Definition:Distributive Operation|distributive]] over [[Definition:Modulo Addition|addition modulo $m$]]: +:$\forall \eqclass x m, \eqclass y m, \eqclass z m \in \Z_m$: +:: $\eqclass x m \times_m \paren {\eqclass y m +_m \eqclass z m} = \paren {\eqclass x m \times_m \eqclass y m} +_m \paren {\eqclass x m \times_m \eqclass z m}$ +:: $\paren {\eqclass x m +_m \eqclass y m} \times_m \eqclass z m = \paren {\eqclass x m \times_m \eqclass z m} +_m \paren {\eqclass y m \times_m \eqclass z m}$ +where $\Z_m$ is the [[Definition:Integers Modulo m|set of integers modulo $m$]]. +That is, $\forall x, y, z, m \in \Z$: +: $x \paren {y + z} \equiv x y + x z \pmod m$ +: $\paren {x + y} z \equiv x z + y z \pmod m$ +\end{theorem} + +\begin{proof} +Follows directly from the definition of [[Definition:Modulo Multiplication|multiplication modulo $m$]] and [[Definition:Modulo Addition|addition modulo $m$]]: +{{begin-eqn}} +{{eqn | l = \eqclass x m \times_m \paren {\eqclass y m +_m \eqclass z m} + | r = \eqclass x m \times_m \eqclass {y + z} m + | c = +}} +{{eqn | r = \eqclass {x \paren {y + z} } m + | c = +}} +{{eqn | r = \eqclass {\paren {x y} + \paren {x z} } m + | c = +}} +{{eqn | r = \eqclass {x y} m +_m \eqclass {x z} m + | c = +}} +{{eqn | r = \paren {\eqclass x m \times_m \eqclass y m} +_m \paren {\eqclass x m \times_m \eqclass z m} + | c = +}} +{{end-eqn}} +And the second is like it, namely this: +{{begin-eqn}} +{{eqn | l = \paren {\eqclass x m +_m \eqclass y m} \times_m \eqclass z m + | r = \eqclass {x + y} m \times_m \eqclass z m + | c = +}} +{{eqn | r = \eqclass {\paren {x + y} z} m + | c = +}} +{{eqn | r = \eqclass {\paren {x z} + \paren {y z} } m + | c = +}} +{{eqn | r = \eqclass {x z} m +_m \eqclass {y z} m + | c = +}} +{{eqn | r = \paren {\eqclass x m \times_m \eqclass z m} +_m \paren {\eqclass y m \times_m \eqclass z m} + | c = +}} +{{end-eqn}} +{{qed}} +\end{proof}<|endoftext|> +\section{Intersection of Congruence Classes} +Tags: Modulo Arithmetic, Set Intersection, Lowest Common Multiple, Intersection of Congruence Classes + +\begin{theorem} +Let $\mathcal R_m$ denote [[Definition:Congruence Modulo Integer|congruence modulo $m$]] on the [[Definition:Set|set]] of [[Definition:Integer|integers]] $\Z$. +Then: +:$\mathcal R_m \cap \mathcal R_n = \mathcal R_{\lcm \set {m, n} }$ +where $\lcm \set {m, n}$ is the [[Definition:Lowest Common Multiple of Integers|lowest common multiple]] of $m$ and $n$. +In the language of [[Definition:Modulo Arithmetic|modulo arithmetic]], this is equivalent to: +:$a \equiv b \pmod m, a \equiv b \pmod n \implies a \equiv b \pmod {\lcm \set {m, n} }$ +\end{theorem} + +\begin{proof} +Let $\tuple {a, b} \in \mathcal R_m \cap \mathcal R_n$. +That is, let $\tuple {a, b} \in \mathcal R_m$ and $\tuple {a, b} \in \mathcal R_n$. +That means, by definition of [[Definition:Congruence Modulo Integer|congruence]]: +:$a \equiv b \pmod m$ +:$a \equiv b \pmod n$ +Thus by definition of [[Definition:Congruence (Number Theory)/Integers/Integer Multiple|congruence]]: +:$\exists r, s \in \Z: a - b = r m, a - b = s n$ +Let $d = \gcd \set {m, n}$ so that $m = d m', n = d n', m' \perp n'$. +Substituting for $m$ and $n$: +:$r d m' = s d n'$ and so $r m' = s n'$. +So $n' \divides r m'$ and $m' \perp n'$ so by [[Euclid's Lemma]] $n' \divides r$. +So we can put $r = k n'$ and get: +:$a - b = r m = k m n' = k m \dfrac n d = k \dfrac {m n} d$ +But: +:$\dfrac {m n} d = \dfrac {m n} {\gcd \set {m, n} }$ +So by [[Product of GCD and LCM]]: +:$a - b = k \lcm \set {m, n}$ +So: +:$a \equiv b \pmod {\lcm \set {m, n} }$ +and hence the result. +{{qed}} +\end{proof}<|endoftext|> +\section{Mappings Between Residue Classes} +Tags: Residue Classes + +\begin{theorem} +Let $\eqclass a m$ be the [[Definition:Residue Class|residue class of $a$ (modulo $m$)]]. +Let $\phi: \Z_m \to \Z_n$ be a [[Definition:Mapping|mapping]] given by: +:$\map \phi {\eqclass x m} = \eqclass x n$ +Then $\phi$ is [[Definition:Well-Defined Mapping|well defined]] {{iff}} $m$ is a [[Definition:Divisor of Integer|divisor]] of $n$. +\end{theorem} + +\begin{proof} +For $\phi$ to be [[Definition:Well-Defined Mapping|well defined]], we require that: +:$\forall x, y \in \Z_m: \eqclass x m = \eqclass y m \implies \map \phi {\eqclass x m} = \map \phi {\eqclass y m}$ +Now: +:$\eqclass x m = \eqclass y m \implies x - y \divides m$ +For $\map \phi {\eqclass x m} = \map \phi {\eqclass y m}$ we require that: +:$\eqclass x n = \eqclass y n \implies x - y \divides n$ +Thus $\phi$ is [[Definition:Well-Defined Mapping|well defined]] {{iff}}: +:$x - y \divides m \implies x - y \divides n$ +That is, {{iff}} $m \divides n$. +{{qed}} +\end{proof}<|endoftext|> +\section{GCD from Congruence Modulo m} +Tags: Modulo Arithmetic, Greatest Common Divisor + +\begin{theorem} +Let $a, b \in \Z, m \in \N$. + +Let $a$ be [[Definition:Congruence Modulo Integer|congruent]] to $b$ modulo $m$. +Then the [[Definition:Greatest Common Divisor of Integers|GCD]] of $a$ and $m$ is equal to the [[Definition:Greatest Common Divisor of Integers|GCD]] of $b$ and $m$. +That is: +:$a \equiv b \pmod m \implies \gcd \set {a, m} = \gcd \set {b, m}$ +\end{theorem} + +\begin{proof} +We have: +:$a \equiv b \pmod m \implies \exists k \in \Z: a = b + k m$ +Thus: +:$a = b + k m$ +and the result follows directly from [[GCD with Remainder]]. +{{qed}} +\end{proof}<|endoftext|> +\section{Prime Number has 4 Integral Divisors} +Tags: Prime Numbers + +\begin{theorem} +Let $p$ be an [[Definition:Integer|integer]]. +Then $p$ is a [[Definition:Prime Number|prime number]] {{iff}} $p$ has exactly four [[Definition:Integer|integral]] [[Definition:Divisor of Integer|divisors]]: $1, -1, p, -p$. +\end{theorem} + +\begin{proof} +=== Necessary Condition === +Let $p$ be a [[Definition:Prime Number/Definition 1|prime number]] from the definition that $p$ has exactly $2$ [[Definition:Divisor of Integer|divisors]] which are [[Definition:Positive Integer|positive integers]]. +From [[One Divides all Integers]] and [[Integer Divides Itself]] those [[Definition:Positive Integer|positive integers]] are $1$ and $p$. +Also, we have $-1 \divides p$ and $-p \divides p$ from [[One Divides all Integers]] and [[Integer Divides its Negative]]. +{{AimForCont}}: +: $\exists x < 0: x \divides p$ +where $x \ne -1$ and $x \ne -p$. +Then: +: $\size x \divides x \divides p$ +and so $\size x$ is therefore a [[Definition:Positive Integer|positive integer]] other than $1$ and $p$ that [[Definition:Divisor of Integer|divides]] $p$. +This is a [[Proof by Contradiction|contradiction]] of the condition for $p$ to be [[Definition:Prime Number/Definition 1|prime]]. +So $-1$ and $-p$ are the only [[Definition:Negative Integer|negative integers]] that [[Definition:Divisor of Integer|divide]] $p$. +It follows that $p$ has exactly those four [[Definition:Divisor of Integer|divisors]]. +{{qed|lemma}} +=== Sufficient Condition === +Suppose $p$ has the [[Definition:Divisor of Integer|divisors]] $1, -1, p, -p$. +It follows that $1$ and $p$ are the only [[Definition:Positive Integer|positive integers]] that [[Definition:Divisor of Integer|divide]] $p$. +Thus $p$ has exactly two [[Definition:Divisor of Integer|divisors]] which are [[Definition:Positive Integer|positive integers]]. +{{qed}} +\end{proof}<|endoftext|> +\section{Prime not Divisor implies Coprime} +Tags: Prime Numbers, Coprime Integers, Prime not Divisor implies Coprime + +\begin{theorem} +Let $p, a \in \Z$. +If $p$ is a [[Definition:Prime Number|prime number]] then: +: $p \nmid a \implies p \perp a$ +where: +: $p \nmid a$ denotes that $p$ does not [[Definition:Divisor of Integer|divide]] $a$ +: $p \perp a$ denotes that $p$ and $a$ are [[Definition:Coprime Integers|coprime]]. +It follows directly that if $p$ and $q$ are [[Definition:Prime Number|primes]], then: +: $p \divides q \implies p = q$ +: $p \ne q \implies p \perp q$. +\end{theorem} + +\begin{proof} +Let $p \in \Bbb P, p \nmid a$. +We need to show that $\gcd \set {a, p} = 1$. +Let $\gcd \set {a, p} = d$. +As $d \divides p$, we must have $d = 1$ or $d = p$ by [[GCD with Prime]]. +But if $d = p$, then $p \divides a$ by definition of [[Definition:Greatest Common Divisor of Integers|greatest common divisor]]. +So $d \ne p$ and therefore $d = 1$. +{{qed}} +\end{proof} + +\begin{proof} +Let $p$ be a [[Definition:Prime Number|prime number]]. +Let $a \in \Z$ be such that $p$ is not a [[Definition:Divisor of Integer|divisor]] of $a$. +{{AimForCont}} $p$ and $a$ are not [[Definition:Coprime Integers|coprime]]. +Then: +:$\exists c \in \Z_{>1}: c \divides p, c \divides a$ +where $\divides$ denotes [[Definition:Divisor of Integer|divisibility]]. +But then by definition of [[Definition:Prime Number|prime]]: +:$c = p$ +Thus: +:$p \divides a$ +The result follows by [[Proof by Contradiction]]. +{{qed}} +\end{proof}<|endoftext|> +\section{Composite Number has Two Divisors Less Than It} +Tags: Divisors + +\begin{theorem} +Let $n \in \Z_{> 1}$ such that $n \notin \mathbb P$. +Then: +:$\exists a, b \in \Z: 1 < a < n, 1 < b < n: n = a b$ +That is, a non-[[Definition:Prime Number|prime]] number greater than $1$ can be expressed as the product of two [[Definition:Positive Integer|positive integers]] strictly greater than $1$ and less than $n$. +Note that these two numbers are not necessarily [[Definition:Distinct Elements|distinct]]. +\end{theorem} + +\begin{proof} +Since $n \notin \mathbb P$, it has a [[Definition:Positive Integer|positive]] [[Definition:Divisor of Integer|factor]] $a$ such that $a \ne 1$ and $a \ne n$. +Hence $\exists b \in \Z: n = a b$. +Thus by definition of [[Definition:Divisor of Integer|factor]]: +:$a \divides n$ +where $\divides$ denotes [[Definition:Divisor of Integer|divisibility]]. +From [[Divisor Relation on Positive Integers is Partial Ordering]]: +:$a \le n$ +As $a \ne n$, it follows that $a < n$. +From [[One Divides all Integers]]: +:$1 \divides a$ +Thus from [[Divisor Relation on Positive Integers is Partial Ordering]]: +:$1 \le a$ +Similarly, as $1 \ne a$ it follows that $1 < a$. +Since $a \ne n$, it follows that $b \ne 1$. +Similarly, since $a \ne 1$, it follows that $b \ne n$. +Thus: +:$b \divides n: 1 \ne b \ne n$ +Arguing as above, we show that $1 < b < n$ and the result follows. +{{qed}} +Note that we have not shown (and it is not necessarily the case) that $a \ne b$. +[[Category:Divisors]] +197jd22ea9wbyx8phcgp1vaw454jncw +\end{proof}<|endoftext|> +\section{Condition for Divisibility of Powers of Prime} +Tags: Prime Numbers + +\begin{theorem} +Let $p$ be a [[Definition:Prime Number|prime]]. +Let $k, l \in \Z_{>0}$. +Then: +:$p^k \divides p^l \iff k \le l$ +\end{theorem} + +\begin{proof} +=== Necessary Condition === +Let $k \le l$. +Then: +:$l - k \ge 0$ +Thus $p^k, p^{l - k} \in \Z$ such that $p^l = p^k p^{l - k}$. +Thus: +:$p^k \divides p^l$ +{{qed|lemma}} +=== Sufficient Condition === +Let $p^k \divides p^l$. +Then: +:$\exists b \in \Z_{>0}: p^l = p^k b$ +By the [[Fundamental Theorem of Arithmetic]], $b$ has a unique [[Definition:Prime Decomposition|prime decomposition]]. +Either $b = 1$ (in which case $k = l$) or $b$ has a [[Definition:Prime Decomposition|prime decomposition]] consisting entirely of $p$'s. +In this case: +:$\exists m \in \Z: b = p^m$ +Hence: +:$p^{l - k} = p^m$ +Thus from the [[Fundamental Theorem of Arithmetic]]: +:$l - k = m > 0$ +Thus: +:$l > k$ +{{qed|lemma}} +The result follows from combining the two results. +{{qed}} +[[Category:Prime Numbers]] +tan97zsd81ac6y8j52yd6eemo9afhp9 +\end{proof}<|endoftext|> +\section{Exponents of Primes in Prime Decomposition are Less iff Divisor} +Tags: Prime Numbers + +\begin{theorem} +Let $a, b \in \Z_{>0}$. +Then $a \divides b$ {{iff}}: +:$(1): \quad$ every [[Definition:Prime Number|prime]] $p_i$ in the [[Definition:Prime Decomposition|prime decomposition]] of $a$ appears in the [[Definition:Prime Decomposition|prime decomposition]] of $b$ +and: +:$(2): \quad$ the [[Definition:Exponent|exponent]] of each $p_i$ in $a$ is less than or equal to its [[Definition:Exponent|exponent]] in $b$. +\end{theorem} + +\begin{proof} +Let $a, b \in \Z_{>0}$. +Let their [[Definition:Prime Decomposition|prime decomposition]] be: +:$a = p_1^{k_1} p_2^{k_2} \dotsm p_n^{k_n}$ +:$b = q_1^{l_1} q_2^{l_2} \dotsm q_n^{l_n}$ +=== Necessary Condition === +Let: +:$(1): \quad$ [[Definition:Prime Number|prime]] in the [[Definition:Prime Decomposition|prime decomposition]] of $a$ appear in the [[Definition:Prime Decomposition|prime decomposition]] of $b$ +and: +:$(2): \quad$ its [[Definition:Exponent|exponent]] in $a$ be less than or equal to its [[Definition:Exponent|exponent]] in $b$. +Then: +{{begin-eqn}} +{{eqn | l = a + | r = p_1^{k_1} p_2^{k_2} \dotsm p_r^{k_r} + | c = +}} +{{eqn | l = b + | r = p_1^{l_1} p_2^{l_2} \dotsm p_r^{l_r} \dotsm p_s^{l_s} + | c = +}} +{{end-eqn}} +where: +:$k_1 \le l_1, k_2 \le l_2, \dotsc, k_r \le l_r, r \le s$ +Thus: +:$d = p_1^{l_1 - k_1} p_2^{l_2 - k_2} \dotsm p_r^{l_r - k_r} \in \Z$ +and so: +:$b = a d$ +So $a \divides b$. +{{qed|lemma}} +=== Sufficient Condition === +Let $a \divides b$. +Let $a = p_1^{k_1} p_2^{k_2} \dotsm p_r^{k_r}$ be the [[Definition:Prime Decomposition|prime decomposition]] of $a$. +Then: +:$\forall i \in \N_r: p_i^{k_i} \divides a$ +Hence by [[Divisor Relation on Positive Integers is Partial Ordering]] each $p_i^{k_i}$ also [[Definition:Divisor of Integer|divides]] $b$. +Thus: +:$\exists c \in \Z: b = p_i^{k_i} c$ +The [[Definition:Prime Decomposition|prime decomposition]] of $b$ is therefore: +:$b = p_i^{k_i} \times \paren {\text {prime decomposition of $c$} }$ +which may need to be rearranged. +So $p_i$ must occur in the [[Definition:Prime Decomposition|prime decomposition]] of $b$ with an [[Definition:Exponent|exponent]] at least as big as $k_i$. +The result follows. +{{qed}} +[[Category:Prime Numbers]] +eovrd4xpmf3wv3snuvq7s6yuwm49psk +\end{proof}<|endoftext|> +\section{Not Coprime means Common Prime Factor} +Tags: Coprime Integers, Prime Numbers + +\begin{theorem} +Let $a, b \in \Z$. +If $d \divides a$ and $d \divides b$ such that $d > 1$, then $a$ and $b$ have a [[Definition:Common Divisor of Integers|common divisor]] which is [[Definition:Prime Number|prime]]. +\end{theorem} + +\begin{proof} +As $d > 1$, it has a [[Definition:Prime Decomposition|prime decomposition]]. +Thus there exists a prime $p$ such that $p \divides d$. +From [[Divisor Relation on Positive Integers is Partial Ordering]], we have $p \divides d, d \divides a \implies p \divides a$, and similarly for $b$. +The result follows. +{{qed}} +[[Category:Coprime Integers]] +[[Category:Prime Numbers]] +1dbh22tlukqincvbw8rkczuduc9vuvj +\end{proof}<|endoftext|> +\section{Set of Divisors of Integer} +Tags: Number Theory + +\begin{theorem} +Let $n \in \Z_{>1}$. +Let $n$ be expressed in its [[Definition:Prime Decomposition|prime decomposition]]: +:$n = p_1^{k_1} p_2^{k_2} \dotsm p_r^{k_r}$ +where $p_1 < p_2 < \dotsb < p_r$ are distinct [[Definition:Prime Number|primes]] and $k_1, k_2, \ldots, k_r$ are [[Definition:Positive Integer|positive integers]]. +The [[Definition:Set|set]] of [[Definition:Divisor of Integer|divisors]] of $n$ is: +:$\set {p_1^{h_1} p_2^{h_2} \dotsm p_r^{h_r}: 0 \le h_i \le k_i, i = 1, 2, \ldots, r}$ +\end{theorem} + +\begin{proof} +Each [[Definition:Integer|integer]] in the given set is a [[Definition:Divisor of Integer|divisor]] of $n$ because: +:$(1): \quad \forall i: k_i - h_i \ge 0$ +:$(2): \quad n = \paren {p_1^{h_1} p_2^{h_2} \dotsm p_r^{h_r} } p_1^{k_1 - h_1} p_2^{k_2 - h_2} \ldots p_r^{k_r - h_r}$ +from [[Exponents of Primes in Prime Decomposition are Less iff Divisor]]. +By the [[Fundamental Theorem of Arithmetic]], these [[Definition:Integer|integers]] are distinct. +It is necessary to show that the [[Definition:Integer|integers]] in this [[Definition:Set|set]] are the ''only'' [[Definition:Divisor of Integer|divisors]] of $n$. +Let $d > 1$ and let $p \in \mathbb P: p \divides d$. +{{begin-eqn}} +{{eqn | o = + | r = p \divides d \land d \divides n + | c = +}} +{{eqn | o = \leadsto + | r = p \divides n + | c = [[Divisor Relation on Positive Integers is Partial Ordering]] +}} +{{eqn | o = \leadsto + | r = \exists i: p = p_i, 1 \le i \le r + | c = +}} +{{eqn | o = \leadsto + | r = p \in \set {p_i: 1 \le i \le r} + | c = +}} +{{eqn | o = \leadsto + | r = d = p_1^{h_1} p_2^{h_2} \dotsm p_r^{h_r}: 0 \le h_i + | c = +}} +{{end-eqn}} +It remains to be shown that: +:$\forall i: h_1 \le k_i$ +First note that: +:$d \divides n \implies \forall i: p_i^{k_i} \divides n$ +From above, all the primes $p_i$ are [[Definition:Distinct|distinct]]. +Therefore by [[Prime not Divisor implies Coprime]]: +:$p_1 \nmid p_2^{k_2} p_3^{k_3} \dotsm p_r^{k_r} \implies \gcd \set {p_1, p_2^{k_2} p_3^{k_3} \ldots p_r^{k_r} } = 1$ +So: +:$p_1^{h_1} \divides n \implies n = p_1^{k_1} \paren {p_2^{k_2} p_3^{k_3} \dotsm p_r^{k_r} }$ + +By [[Euclid's Lemma]]: +:$p_1^{h_1} \divides p_1^{k_1} \implies h_1 \le k_1$ +and the same argument applies to each of the other [[Definition:Prime Factor|prime factors]] of $n$. +The result follows. +{{qed}} +[[Category:Number Theory]] +rzbwqi92fhcc2ar5kuqb694dh6aozep +\end{proof}<|endoftext|> +\section{Sum Less Minimum is Maximum} +Tags: Algebra + +\begin{theorem} +For all [[Definition:Number|numbers]] $a, b$ where $a, b$ in $\N, \Z, \Q$ or $\R$: +:$a + b - \min \left({a, b}\right) = \max \left({a, b}\right)$ +\end{theorem} + +\begin{proof} +From [[Sum of Maximum and Minimum]] we have that $a + b = \max \left({a, b}\right) + \min \left({a, b}\right)$. +Thus $a + b - \min \left({a, b}\right) = \max \left({a, b}\right)$ follows by subtracting $\min \left({a, b}\right)$ from both sides. +It is clear that this result applies when $a, b$ in $\Z, \Q$ or $\R$, as [[Definition:Subtraction|subtraction]] is well-defined throughout those number sets. +However, it can be seen to apply in $\N$ as well, despite the fact that $n - m$ is defined on $\N$ only when $n \ge m$. +This is because the fact that $a + b \ge \min \left({a, b}\right)$ follows immediately again from $a + b = \max \left({a, b}\right) + \min \left({a, b}\right)$. +{{qed}} +[[Category:Algebra]] +qfqfes3zbhckirbxe25ex8hvfroraqj +\end{proof}<|endoftext|> +\section{Sum Less Maximum is Minimum} +Tags: Algebra + +\begin{theorem} +For all [[Definition:Number|numbers]] $a, b$ where $a, b$ in $\N, \Z, \Q$ or $\R$: +:$a + b - \max \left({a, b}\right) = \min \left({a, b}\right)$. +\end{theorem} + +\begin{proof} +From [[Sum of Maximum and Minimum]] we have that $a + b = \max \left({a, b}\right) + \min \left({a, b}\right)$. +Thus $a + b - \max \left({a, b}\right) = \min \left({a, b}\right)$ follows by subtracting $\max \left({a, b}\right)$ from both sides. +It is clear that this result applies when $a, b$ in $\Z, \Q$ or $\R$, as [[Definition:Subtraction|subtraction]] is well-defined throughout those number sets. +However, it can be seen to apply in $\N$ as well, despite the fact that $n - m$ is defined on $\N$ only when $n \ge m$. +This is because the fact that $a + b \ge \max \left({a, b}\right)$ follows immediately again from $a + b = \max \left({a, b}\right) + \min \left({a, b}\right)$. +{{qed}} +[[Category:Algebra]] +ef4y60wxp42ltsv2eex2nzjibeles2j +\end{proof}<|endoftext|> +\section{GCD and LCM from Prime Decomposition} +Tags: Greatest Common Divisor, Lowest Common Multiple, Prime Numbers + +\begin{theorem} +Let $m, n \in \Z$. +Let: +:$m = p_1^{k_1} p_2^{k_2} \dotsm p_r^{k_r}$ +:$n = p_1^{l_1} p_2^{l_2} \dotsm p_r^{l_r}$ +:$p_i \divides m \lor p_i \divides n, 1 \le i \le r$. +That is, the [[Definition:Prime Number|primes]] given in these [[Definition:Prime Decomposition|prime decompositions]] may be [[Definition:Divisor of Integer|divisors]] of ''either'' of the numbers $m$ or $n$. +Note that if one of the [[Definition:Prime Number|primes]] $p_i$ does not appear in the [[Definition:Prime Decomposition|prime decompositions]] of either one of $m$ or $n$, then its corresponding index $k_i$ or $l_i$ will be zero. +Then the following results apply: +:$\gcd \set {m, n} = p_1^{\min \set {k_1, l_1} } p_2^{\min \set {k_2, l_2} } \ldots p_r^{\min \set {k_r, l_r} }$ +:$\lcm \set {m, n} = p_1^{\max \set {k_1, l_1} } p_2^{\max \set {k_2, l_2} } \ldots p_r^{\max \set {k_r, l_r} }$ +\end{theorem} + +\begin{proof} +The proof of these results can be found in: +:[[GCD from Prime Decomposition]] +:[[LCM from Prime Decomposition]] +{{Qed}} +[[Category:Greatest Common Divisor]] +[[Category:Lowest Common Multiple]] +[[Category:Prime Numbers]] +iur6iagonkv1hmri44ycxc6lbtsc4ux +\end{proof}<|endoftext|> +\section{GCD and LCM Distribute Over Each Other} +Tags: Greatest Common Divisor, Lowest Common Multiple, Distributive Operations + +\begin{theorem} +Let $a, b, c \in \Z$. +Then: +: $\lcm \set {a, \gcd \set {b, c} } = \gcd \set {\lcm \set {a, b}, \lcm \set {a, c} }$ +: $\gcd \set {a, \lcm \set {b, c} } = \lcm \set {\gcd \set {a, b}, \gcd \set {a, c} }$ +That is, [[Definition:Greatest Common Divisor of Integers|greatest common divisor]] and [[Definition:Lowest Common Multiple of Integers|lowest common multiple]] are [[Definition:Distributive Operation|distributive]] over each other. +\end{theorem} + +\begin{proof} +=== LCM Distributive over GCD === +Let $p_s$ be any of the prime divisors of $a, b$ or $c$, and let $s_a, s_b$ and $s_c$ be its exponent in each of those numbers. +Let $x = \lcm \set {a, \gcd \set {b, c} }$. +Then from [[GCD and LCM from Prime Decomposition]], the exponent of $p_s$ in $x$ is $\max \set {s_a, \min \set {s_b, s_c} }$. +From [[Max and Min Operations are Distributive over Each Other]], $\max$ distributes over $\min$. +Therefore: +: $\max \set {s_a, \min \set {s_b, s_c} } = \min \set {\max \set {s_a, s_b}, \max \set {s_a, s_c} }$ +Hence it follows that [[Definition:Lowest Common Multiple of Integers|LCM]] is [[Definition:Distributive Operation|distributive]] over [[Definition:Greatest Common Divisor of Integers|GCD]]. +{{qed|lemma}} +=== GCD Distributive over LCM === +Let $p_s$ be any of the prime divisors of $a, b$ or $c$, and let $s_a, s_b$ and $s_c$ be its exponent in each of those numbers. +Let $x = \gcd \set {a, \lcm \set {b, c} }$. +Then from [[GCD and LCM from Prime Decomposition]], the exponent of $p_s$ in $x$ is $\min \set {s_a, \max \set {s_b, s_c} }$. +From [[Max and Min Operations are Distributive over Each Other]], $\min$ distributes over $\max$. +Therefore: +: $\min \set {s_a, \max \set {s_b, s_c} } = \max \set {\min \set {s_a, s_b}, \min \set {s_a, s_c} }$ +and the result follows. +Hence it follows that [[Definition:Greatest Common Divisor of Integers|GCD]] is [[Definition:Distributive Operation|distributive]] over [[Definition:Lowest Common Multiple of Integers|LCM]]. +{{qed}} +\end{proof}<|endoftext|> +\section{Tau Function from Prime Decomposition} +Tags: Tau Function, Tau Function from Prime Decomposition + +\begin{theorem} +Let $n$ be an [[Definition:Integer|integer]] such that $n \ge 2$. +Let the [[Definition:Prime Decomposition|prime decomposition]] of $n$ be: +:$n = p_1^{k_1} p_2^{k_2} \cdots p_r^{k_r}$ +Let $\map \tau n$ be the [[Definition:Tau Function|tau function]] of $n$. +Then: +:$\displaystyle \map \tau n = \prod_{j \mathop = 1}^r \paren {k_j + 1}$ +\end{theorem} + +\begin{proof} +We have: +:$d \divides n \implies \forall i: 1 \le i \le r: d = p_1^{l_1} p_2^{l_2} \ldots p_1^{l_1}, 0 \le l_i \le k_i$ +For each $i$, there are $k_i + 1$ choices for $l_i$, making $\paren {k_1 + 1} \paren {k_2 + 1} \cdots \paren {k_r + 1}$ choices in all. +By the [[Fundamental Theorem of Arithmetic]] and hence the uniqueness of [[Definition:Prime Decomposition|prime decomposition]], each of these choices results in a different number, therefore a distinct [[Definition:Divisor of Integer|divisor]]. +{{qed}} +\end{proof} + +\begin{proof} +From [[Tau of Power of Prime]] we have: +:$\forall j \in \closedint 1 r: \map \tau {p_j^{k_j} } = k_j + 1$ +The result follows immediately from [[Tau Function is Multiplicative]]. +{{qed}} +\end{proof}<|endoftext|> +\section{N less than M to the N} +Tags: Algebra + +\begin{theorem} +:$\forall m, n \in \Z_{>0}: m > 1 \implies n < m^n$ +\end{theorem} + +\begin{proof} +{{begin-eqn}} +{{eqn | l = n + | r = \underbrace {1 + 1 + \cdots + 1}_{\text {$n$ times} } + | c = +}} +{{eqn | o = < + | r = 1 + m + m^2 + \cdots + m^{n - 1} + | c = as $m > 1$ +}} +{{eqn | r = \frac {m^n - 1} {m - 1} + | c = [[Sum of Geometric Sequence]] +}} +{{eqn | o = \le + | r = m^n - 1 + | c = as $m - 1 \ge 1$ +}} +{{eqn | o = < + | r = m^n + | c = +}} +{{end-eqn}} +{{qed}} +\end{proof}<|endoftext|> +\section{Basis Representation Theorem} +Tags: Number Bases, Named Theorems + +\begin{theorem} +Let $b \in \Z: b > 1$. +For every $n \in \Z_{> 0}$, there exists [[Definition:Exactly One|one and only one]] [[Definition:Sequence|sequence]] $\sequence {r_j}_{0 \mathop \le j \mathop \le t}$ such that: +: $(1): \quad \displaystyle n = \sum_{k \mathop = 0}^t r_k b^k$ +: $(2): \quad \displaystyle \forall k \in \closedint 0 t: r_k \in \N_b$ +: $(3): \quad r_t \ne 0$ +This unique sequence is called the '''representation of $n$ to the base $b$''', or, informally, we can say '''$n$ is (written) in base $b$'''. +\end{theorem} + +\begin{proof} +Let $\map {s_b} n$ be the number of ways of representing $n$ to the base $b$. +We need to show that $\map {s_b} n = 1$ always. +Now, it is possible that some of the $r_k = 0$ in a particular representation. +So we may exclude these terms, and it won't affect the representation. +So, suppose: +:$n = r_k b^k + r_{k - 1} b^{k - 1} + \cdots + r_t b^t$ +where $r_k \ne 0, r_t \ne 0$. +Then: +{{begin-eqn}} +{{eqn | l = n - 1 + | r = r_k b^k + r_{k - 1} b^{k - 1} + \cdots + r_t b^t - 1 + | c = +}} +{{eqn | r = r_k b^k + r_{k - 1} b^{k - 1} + \cdots + \paren {r_t - 1} b^t + b^t - 1 + | c = +}} +{{eqn | r = r_k b^k + r_{k - 1} b^{k - 1} + \cdots + \paren {r_t - 1} b^t + \sum_{j \mathop = 0}^{t - 1} {\paren {b - 1} b^j} + | c = [[Sum of Geometric Sequence]] +}} +{{end-eqn}} +Note that we have already specified that $b > 1$. +So for each representation of $n$ to the base $b$, we can find a representation of $n - 1$. +If $n$ has another representation to the base $b$, then the same procedure will generate a new representation of $n - 1$. +Thus: +:$(1): \quad \map {s_b} n \le \map {s_b} {n - 1}$ +Note that this holds even if $n$ has no representation at all, because if this is the case, then $\map {s_b} n = 0 \le \map {s_b} {n - 1}$. +So for $m, n \in \N$ such that $m \ge n$, the inequality $(1)$ implies the following: +:$\forall m, n: \map {s_b} m \le \map {s_b} {m - 1} \le \ldots \le \map {s_b} {n + 1} \le \map {s_b} n$ +From [[N less than M to the N]] and the fact that $b^n$ has at least one representation (itself), we see: +:$1 \le \map {s_b} {b^n} \le \map {s_b} n \le \map {s_b} 1 = 1$ +The entries at either end of this inequality are $1$, so all the intermediate entries must also be $1$. +So $\map {s_b} n = 1$ and the theorem has been proved. +{{qed}} +\end{proof}<|endoftext|> +\section{Sum of Geometric Sequence} +Tags: Sums of Sequences, Geometric Sequences, Sum of Geometric Sequence + +\begin{theorem} +Let $x$ be an element of one of the [[Definition:Standard Number Field|standard number fields]]: $\Q, \R, \C$ such that $x \ne 1$. +Let $n \in \N_{>0}$. +Then: +:$\displaystyle \sum_{j \mathop = 0}^{n - 1} x^j = \frac {x^n - 1} {x - 1}$ +\end{theorem} + +\begin{proof} +For all $n \in \N_{> 0}$, let $\map P n$ be the [[Definition:Proposition|proposition]]: +:$\displaystyle \sum_{j \mathop = 0}^{n - 1} x^j = \frac {x^n - 1} {x - 1}$ +=== Basis for the Induction === +$\map P 1$ is the case: +{{begin-eqn}} +{{eqn | l = \dfrac {x^1 - 1} {x - 1} + | r = 1 + | c = +}} +{{eqn | r = 2^0 + | c = +}} +{{eqn | r = \sum_{j \mathop = 0}^{1 - 1} x^j + | c = +}} +{{end-eqn}} +so $\map P 1$ holds. +This is our [[Definition:Basis for the Induction|basis for the induction]]. +=== Induction Hypothesis === +Now we need to show that, if $\map P k$ is true, where $k \ge 1$, then it logically follows that $\map P {k + 1}$ is true. +So this is our [[Definition:Induction Hypothesis|induction hypothesis]]: +:$\displaystyle \sum_{j \mathop = 0}^{k - 1} x^j = \frac {x^k - 1} {x - 1}$ +Then we need to show: +:$\displaystyle \sum_{j \mathop = 0}^k x^j = \frac {x^{k + 1} - 1} {x - 1}$ +=== Induction Step === +This is our [[Definition:Induction Step|induction step]]: +{{begin-eqn}} +{{eqn | l = \sum_{j \mathop = 0}^k x^j + | r = \sum_{j \mathop = 0}^{k - 1} x^j + x^k + | c = +}} +{{eqn | r = \frac {x^k - 1} {x - 1} + x^k + | c = +}} +{{eqn | r = \frac {x^k - 1 + \paren {x - 1} x^k} {x - 1} + | c = +}} +{{eqn | r = \frac {x^k - 1 + x^{k + 1} - x^k} {x - 1} + | c = +}} +{{eqn | r = \frac {x^{k + 1} - 1} {x - 1} + | c = +}} +{{end-eqn}} +So $\map P k \implies \map P {k + 1}$ and the result follows by the [[Principle of Mathematical Induction]]. +Therefore: +:$\displaystyle \forall n \in \N_{> 0}: \sum_{j \mathop = 0}^{n - 1} x^j = \frac {x^n - 1} {x - 1}$ +{{qed}} +\end{proof} + +\begin{proof} +Let $\displaystyle S_n = \sum_{j \mathop = 0}^{n - 1} x^j$. +Then: +{{begin-eqn}} +{{eqn | l = \paren {x - 1} S_n + | r = x S_n - S_n + | c = +}} +{{eqn | r = x \sum_{j \mathop = 0}^{n - 1} x^j - \sum_{j \mathop = 0}^{n - 1} x^j + | c = +}} +{{eqn | r = \sum_{j \mathop = 1}^n x^j - \sum_{j \mathop = 0}^{n - 1} x^j + | c = +}} +{{eqn | r = x^n + \sum_{j \mathop = 1}^{n - 1} x^j - \paren {x^0 + \sum_{j \mathop = 1}^{n - 1} x^j} + | c = +}} +{{eqn | r = x^n - x^0 + | c = +}} +{{eqn | r = x^n - 1 + | c = +}} +{{end-eqn}} +The result follows. +{{qed}} +\end{proof} + +\begin{proof} +From [[Difference of Two Powers]]: +:$\displaystyle a^n - b^n = \paren {a - b} \paren {a^{n - 1} + a^{n - 2} b + a^{n - 3} b^2 + \ldots + a b^{n - 2} + b^{n - 1} } = \paren {a - b} \sum_{j \mathop = 0}^{n - 1} a^{n - j - 1} b^j$ +Set $a = x$ and $b = 1$: +:$\displaystyle x^n - 1 = \paren {x - 1} \paren {x^{n - 1} + x^{n - 2} + \cdots + x + 1} = \paren {x - 1} \sum_{j \mathop = 0}^{n - 1} x^j$ +from which the result follows directly. +{{qed}} +\end{proof} + +\begin{proof} +=== [[Sum of Geometric Sequence/Proof 4/Lemma|Lemma]] === +{{:Sum of Geometric Sequence/Proof 4/Lemma}}{{qed|lemma}} +Then by the [[Sum of Geometric Sequence/Proof 4/Lemma|lemma]]: +{{begin-eqn}} +{{eqn | l = \paren {1 - x} \sum_{i \mathop = 0}^{n - 1} x^i + | r = 1 - x^n +}} +{{eqn | ll= \leadsto + | l = \sum_{i \mathop = 0}^{n - 1} x^i = \frac {1 - x^n} {1 - x} + | r = \frac {x^n - 1} {x - 1} +}} +{{end-eqn}} +{{qed}} +\end{proof}<|endoftext|> +\section{Congruence of Sum of Digits to Base Less 1} +Tags: Number Theory + +\begin{theorem} +Let $x \in \Z$, and $b \in \N, b > 1$. +Let $x$ be [[Basis Representation Theorem|written in base $b$]]: +:$x = \sqbrk {r_m r_{m - 1} \ldots r_2 r_1 r_0}_b$ +Then: +:$\displaystyle \map {s_b} x = \sum_{j \mathop = 0}^m r_j \equiv x \pmod {b - 1}$ +where $\map {s_b} x$ is the [[Definition:Digit Sum|digit sum]] of $x$ [[Basis Representation Theorem|in base $b$ notation]]. +That is, the [[Definition:Digit Sum|digit sum]] of any [[Definition:Integer|integer]] $x$ [[Basis Representation Theorem|in base $b$ notation]] is [[Definition:Congruence Modulo Integer|congruent to $x$ modulo $b - 1$]]. +\end{theorem} + +\begin{proof} +Let $x \in \Z, x > 0$, and $b \in \N, b > 1$. +Then from the [[Basis Representation Theorem]], $x$ can be expressed uniquely as: +:$\displaystyle x = \sum_{j \mathop = 0}^m r_j b^j, r_0, r_1, \ldots, r_m \in \set {0, 1, \ldots, b - 1}$ +Proof by [[Principle of Mathematical Induction|induction]]: +For all $n \in \N_{>0}$, let $\map P n$ be the [[Definition:Proposition|proposition]] $\displaystyle \sum_{j \mathop = 0}^n r_j \equiv x \pmod {b - 1}$. +=== Basis for the Induction === +$\map P 1$ is trivially true: +:$\forall x: 0 \le x \le b: x \equiv x \pmod {b - 1}$ +This is our [[Definition:Basis for the Induction|basis for the induction]]. +=== Induction Hypothesis === +Now we need to show that, if $\map P k$ is true, where $k \ge 2$, then it logically follows that $\map P {k + 1}$ is true. +So this is our [[Definition:Induction Hypothesis|induction hypothesis]]: +:$\displaystyle \sum_{j \mathop = 0}^k r_j \equiv \sum_{j \mathop = 0}^k r_j b^j \pmod {b - 1}$ +Then we need to show: +:$\displaystyle \sum_{j \mathop = 0}^{k \mathop + 1} r_j \equiv \sum_{j \mathop = 0}^{k \mathop + 1} r_j b^j \pmod {b - 1}$ +=== Induction Step === +This is our [[Definition:Induction Step|induction step]]: +Let $y$ be expressed as: +:$\displaystyle y = \sum_{j \mathop = 0}^{k \mathop + 1} {r_j b^j}, r_0, r_1, \ldots, r_{k + 1} \in \set {0, 1, \ldots, b}$ +Then: +:$\displaystyle y = \sum_{j \mathop = 0}^k r_j b^j + r_{k + 1} b^{k + 1}$ +Now from [[Congruence of Powers]]: +:$b \equiv 1 \pmod {b - 1} \leadsto b^n \equiv 1^n \pmod {b - 1} \leadsto b^n \equiv 1 \pmod {b - 1}$ +So by [[Definition:Modulo Multiplication|modulo multiplication]]: +:$r_{k + 1} b^{k + 1} \equiv r_{k + 1} \pmod {b - 1}$ +From the [[Congruence of Sum of Digits to Base Less 1#Induction Hypothesis|induction hypothesis]]: +:$\displaystyle \sum_{j \mathop = 0}^{k \mathop + 1} r_j \equiv y \pmod {b - 1}$ +Thus by [[Definition:Modulo Addition|modulo addition]]: +:$\displaystyle \sum_{j \mathop = 0}^{k \mathop + 1} r_j \equiv \sum_{j \mathop = 0}^k r_j + r_{k + 1} \pmod {b - 1}$ +Hence $\map P k \implies \map P {k + 1}$ and the result follows by the [[Principle of Mathematical Induction]]: +:$\displaystyle \sum_{j \mathop = 0}^n r_j \equiv \sum_{j \mathop = 0}^n r_j b^j \pmod {b - 1}$ +{{Qed}} +[[Category:Number Theory]] +9i4cwx5p7k5bks1ofs6jy82lm78qvu2 +\end{proof}<|endoftext|> +\section{Euler Phi Function of Integer} +Tags: Euler Phi Function + +\begin{theorem} +Let $n \in \Z_{>0}$, that is, a [[Definition:Strictly Positive Integer|(strictly) positive integer]]. +Let $\phi: \Z_{>0} \to \Z_{>0}$ be the [[Definition:Euler Phi Function|Euler $\phi$-function]]. +Then for any $n \in \Z_{>0}$, we have: +:$\map \phi n = n \paren {1 - \dfrac 1 {p_1} } \paren {1 - \dfrac 1 {p_2} } \cdots \paren {1 - \dfrac 1 {p_r} }$ +where $p_1, p_2, \ldots, p_r$ are the distinct [[Definition:Prime Number|primes]] dividing $n$. +Or, more compactly: +:$\displaystyle \map \phi n = n \prod_{p \mathop \divides n} \paren {1 - \frac 1 p}$ +where $p \divides n$ denotes the [[Definition:Prime Number|primes]] which [[Definition:Divisor of Integer|divide]] $n$. +\end{theorem} + +\begin{proof} +If $n = 1$ the result holds by inspection. +Let $n \ge 2$. +We express $n$ in its [[Definition:Prime Decomposition|prime decomposition]]: +:$n = p_1^{k_1} p_2^{k_2} \cdots p_r^{k_r}, p_1 < p_2 < \cdots < p_r$ +as it is always possible to do. +By definition, all primes are [[Definition:Coprime Integers|coprime]] to each other. +Hence from [[Euler Phi Function is Multiplicative]]: +:$\map \phi n = \map \phi {p_1^{k_1} } \map \phi {p_2^{k_2} } \cdots \map \phi {p_r^{k_r} }$ +and from [[Euler Phi Function of Prime Power]]: +:$\map \phi {p^k} = p^k \paren {1 - \dfrac 1 p}$ +So: +:$\map \phi n = p_1^{k_1} \paren {1 - \dfrac 1 {p_1} } p_2^{k_2} \paren {1 - \dfrac 1 {p_2} } \cdots p_r^{k_r} \paren {1 - \dfrac 1 {p_r} }$ +and the result follows directly from: +:$n = p_1^{k_1} p_2^{k_2} \cdots p_r^{k_r}$ +{{qed}} +\end{proof}<|endoftext|> +\section{Euler Phi Function is Multiplicative} +Tags: Multiplicative Functions, Euler Phi Function + +\begin{theorem} +The [[Definition:Euler Phi Function|Euler $\phi$ function]] is a [[Definition:Multiplicative Arithmetic Function|multiplicative function]]: +:$m \perp n \implies \map \phi {m n} = \map \phi m \, \map \phi n$ +where $m, n \in \Z_{>0}$. +\end{theorem} + +\begin{proof} +Let $R = \set {r_1, r_2, \ldots, r_{\map \phi m} }$ and $S = \set {s_1, s_2, \ldots, s_{\map \phi n} }$ be the [[Definition:Reduced Residue System|reduced residue systems]] for the respective moduli $m$ and $n$. +We are to show that the set of $\map \phi m \map \phi n$ [[Definition:Integer|integers]]: +:$T = \set {n r + m s: r \in R, s \in S}$ +is a [[Definition:Reduced Residue System|reduced residue system]] for modulus $m n$. +We need to establish the following: +: Each integer in $T$ is [[Definition:Coprime Integers|prime to]] $m n$ +: No two integers in $T$ is [[Definition:Congruence (Number Theory)|congruent modulo $m n$]] +: Each integer [[Definition:Coprime Integers|prime to]] $m n$ is [[Definition:Congruence (Number Theory)|congruent modulo $m n$]] to one of these integers in $T$. +We prove each in turn: +Let $p$ be a [[Definition:Prime Factor|prime divisor]] of $\gcd \set {n r + m s, m n}$ where $r \in R, s \in S$. +As $p$ divides $m n$ but $m \perp n$, $p$ either divides $m$ or $n$ but not both, from [[Divisors of Product of Coprime Integers]]. +{{WLOG}}, suppose $p \divides m$. +Then as $p \divides n r + m s$, we have $p \divides n r$ and hence $p \divides r$. +But then $p \divides \gcd \set {m, r} = 1$ which is a contradiction. +Similarly if $p \divides n$. +So there is no such prime and hence $n r + m s \perp m n$. +{{qed|lemma}} +Let $n r + m s = n r' + m s' \pmod {m n}$, where $r, r' \in R, s, s' \in S$. +Then: +:$n \paren {r - r'} + m \paren {s - s'} = k \paren {m n}$ for some $k \in \Z$. +As $m$ divides two of these terms it must divide the third, so: +:$m \divides n \paren {r - r'}$ +Now $m \perp n$ so by [[Euclid's Lemma]]: +:$m \divides \paren {r - r'}$ +or: +:$r \equiv r' \pmod m$ +But $r$ and $r'$ are part of the same [[Definition:Reduced Residue System|reduced residue system]] modulo $m$, so: +:$r = r'$ +Similarly for $n$: we get: +:$s = s'$ +Hence distinct elements of $T$ can not be [[Definition:Congruence (Number Theory)|congruent modulo $m n$]]. +{{qed|lemma}} +Let $k \in \Z: k \perp m n$. +Since $m \perp n$, from [[Set of Integer Combinations equals Set of Multiples of GCD]] we can write $k = n r' + m s'$ for some $r', s' \in \Z$. +Suppose there exists some [[Definition:Prime Number|prime number]] $p$ such that $p \divides m$ and $p \divides r'$. +Such a prime would be a common divisor of both $k$ and $m n$, contradicting $k \perp m n$. +Hence $r' \perp m$ and so is [[Definition:Congruence (Number Theory)|congruent modulo $m $]] to one of these integers in $R$. +By the same argument, $s' \perp n$ and so is [[Definition:Congruence (Number Theory)|congruent modulo $n$]] to one of these integers in $S$. +Writing $r' = r + a m, \, s' = s + b n$ we have: +:$k = n r' + m s' = n r + m s + m n \paren {a + b} \equiv n r + m s \pmod {m n}$ +Hence the result. +{{qed}} +\end{proof}<|endoftext|> +\section{Euler Phi Function of Prime Power} +Tags: Euler Phi Function, Prime Numbers + +\begin{theorem} +Let $p^n$ be a [[Definition:Prime Power|prime power]] for some [[Definition:Prime Number|prime number]] $p > 1$. +Then: +:$\map \phi {p^n} = p^n \paren {1 - \dfrac 1 p} = \paren {p - 1} p^{n - 1}$ +where $\phi: \Z_{>0} \to \Z_{>0}$ is the [[Definition:Euler Phi Function|Euler $\phi$ function]]. +\end{theorem} + +\begin{proof} +From [[Euler Phi Function of Prime]]: +:$\map \phi p = p - 1$ +From [[Prime not Divisor implies Coprime]]: +:$k \perp p^n \iff p \nmid k$ +There are $p^{n - 1}$ [[Definition:Integer|integers]] $k$ such that $1 \le k \le p^n$ which are [[Definition:Divisor of Integer|divisible]] by $p$: +:$k \in \set {p, 2 p, 3 p, \ldots, \paren {p^{n - 1} } p}$ +Therefore: +:$\map \phi {p^n} = p^n - p^{n - 1} = p^n \paren {1 - \dfrac 1 p} = \paren {p - 1} p^{n - 1}$ +{{qed}} +\end{proof}<|endoftext|> +\section{Sum of Euler Phi Function over Divisors} +Tags: Euler Phi Function + +\begin{theorem} +Let $n \in \Z_{>0}$ be a [[Definition:Strictly Positive Integer|strictly positive integer]]. +Then $\displaystyle \sum_{d \mathop \divides n} \map \phi d = n$ +where: +:$\displaystyle \sum_{d \mathop \divides n}$ denotes the [[Definition:Sum Over Divisors|sum over all of the divisors]] of $n$ +:$\map \phi d$ is the [[Definition:Euler Phi Function|Euler $\phi$ function]], the number of [[Definition:Integer|integers]] less than $d$ that are [[Definition:Coprime Integers|prime to]] $d$. +That is, the total of all the [[Definition:Euler Phi Function|totients]] of all [[Definition:Divisor of Integer|divisors]] of a number equals that number. +\end{theorem} + +\begin{proof} +Let us define: +:$S_d = \set {m \in \Z: 1 \le m \le n, \gcd \set {m, n} = d}$. +That is, $S_d$ is all the numbers less than or equal to $n$ whose [[Definition:Greatest Common Divisor of Integers|GCD]] with $n$ is $d$. +Now from [[Integers Divided by GCD are Coprime]] we have: +: $\gcd \set {m, n} = d \iff \dfrac m d, \dfrac n d \in \Z: \dfrac m d \perp \dfrac n d$ +So the number of integers in $S_d$ equals the number of [[Definition:Positive Integer|positive integers]] no bigger than $\dfrac n d$ which are [[Definition:Coprime Integers|prime to]] $\dfrac n d$. +That is, by definition of the [[Definition:Euler Phi Function|Euler phi function]]: +:$\card {S_d} = \map \phi {\dfrac n d}$ +From the definition of the $S_d$, it follows that for all $1 \le m \le n$: +:$\exists d \divides n: m \in S_d$ +Therefore: +:$\displaystyle \set {1, \ldots, n} = \bigcup_{d \mathop \divides n} S_d$ +Moreover, it follows from the definition of the $S_d$ that they are [[Definition:Pairwise Disjoint Family|pairwise disjoint]]. +Now from [[Cardinality of Set Union/Corollary|Corollary to Cardinality of Set Union]], it follows that: +{{begin-eqn}} +{{eqn | l = n + | r = \sum_{d \mathop \divides n} \card {S_d} +}} +{{eqn | r = \sum_{d \mathop \divides n} \map \phi {\dfrac n d} +}} +{{end-eqn}} +But from [[Sum Over Divisors Equals Sum Over Quotients]]: +:$\displaystyle \sum_{d \mathop \divides n} \map \phi {\dfrac n d} = \sum_{d \mathop \divides n} \map \phi d$ +and hence the result. +{{qed}} +\end{proof}<|endoftext|> +\section{Möbius Function is Multiplicative} +Tags: Multiplicative Functions, Möbius Function + +\begin{theorem} +The [[Definition:Möbius Function|Möbius function]] $\mu$ is a [[Definition:Multiplicative Arithmetic Function|multiplicative function]]: +:$m \perp n \implies \map \mu {m n} = \map \mu m \map \mu n$ +where $m, n \in \Z_{>0}$. +\end{theorem} + +\begin{proof} +First note that we have $\map \mu 1 = 1$, which agrees with [[Value of Multiplicative Function at One]]. +Let $m, n \in \Z_{>0}$ such that $m \perp n$. +First, suppose that either $\map \mu m = 0$ or $\map \mu n = 0$. +Then either $m$ or $n$ has a factor $p^2$ where $p$ is [[Definition:Prime Number|prime]]. +Thus it will follow that $m n$ will also have a factor $p^2$ and hence $\map \mu {m n} = 0$. +So the result holds when $\map \mu m = 0$ or $\map \mu n = 0$. +Now suppose that $\map \mu m \ne 0$ and $\map \mu n \ne 0$. +Let $m = p_1 p_2 \ldots p_r$, $n = q_1 q_2 \ldots q_s$ where all the $p_i, q_j$ are [[Definition:Prime Number|prime]]. +Then: +:$m n = p_1 p_2 \ldots p_r q_1 q_2 \ldots q_s$ +As $m \perp n$ it follows that: +:$\forall i, j: p_i \ne q_j$ +Hence there is no prime in $m n$ whose power is higher than $1$, which means that $\map \mu {m n} \ne 0$. +So: +:$\map \mu {m n} = \paren {-1}^{r + s} = \paren {-1}^r \paren {-1}^s = \map \mu m \map \mu n$ +Hence the result. +{{qed}} +\end{proof}<|endoftext|> +\section{Sum of Möbius Function over Divisors} +Tags: Euler Phi Function, Möbius Function + +\begin{theorem} +Let $n \in \Z_{>0}$ be a [[Definition:Strictly Positive Integer|strictly positive integer]]. +Then: +:$\displaystyle \sum_{d \mathop \divides n} \map \mu d \frac n d = \map \phi n$ +where: +:$\displaystyle \sum_{d \mathop \divides n}$ denotes the [[Definition:Sum Over Divisors|sum over all of the divisors]] of $n$ +:$\map \phi n$ is the [[Definition:Euler Phi Function|Euler $\phi$ function]], the number of [[Definition:Integer|integers]] less than $n$ that are [[Definition:Coprime Integers|prime to]] $n$ +:$\map \mu d$ is the [[Definition:Möbius Function|Möbius function]]. +Equivalently, this says that: +:$\phi = \mu * I_{\Z_{>0} }$ +where: +:$*$ denotes [[Definition:Dirichlet Convolution|Dirichlet convolution]] +:$I_{\Z_{>0} }$ denotes the [[Definition:Identity Mapping|identity mapping]] on $\Z_{>0}$, that is: +::$\forall n \in \Z_{>0}: I_{\Z_{>0} }: n \mapsto n$ +{{wtd|Add a link to a page proving this equivalence.}} +\end{theorem} + +\begin{proof} +=== [[Sum of Möbius Function over Divisors/Lemma|Lemma]] === +{{:Sum of Möbius Function over Divisors/Lemma}} +Let $\map 1 k = 1$ be the [[Definition:Constant Mapping|constant mapping]]. +Then $\phi$ is defined as: +:$\displaystyle \map \phi n = \sum_{k \mathop \perp n} \map 1 k$ +We have that $\gcd \set {n, k}$ is $1$ if $k \perp n$ and $0$ otherwise. +{{explain|How can that be?}} +Thus we can rewrite the above sum as: +:$\displaystyle \sum_{k \mathop = 1}^n \floor {\frac 1 {\gcd \set {n, k} } }$ +Now we may use the lemma, with $\gcd \set {n, k}$ replacing $n$, to get: +{{begin-eqn}} +{{eqn | l = \map \phi n + | r = \sum_{k \mathop = 1}^n \paren {\sum_{d \mathop \divides \gcd \set {n, k} } \map \mu d} + | c = +}} +{{eqn | r = \sum_{k \mathop = 1}^n \sum_{\binom {d \mathop \divides n} {d \mathop \divides k} } \map \mu d + | c = +}} +{{end-eqn}} +For a fixed [[Definition:Divisor of Integer|divisor]] $d$ of $n$, we must sum over all those $k$ in the range $1 \le k \le n$ which are multiples of $d$. +If we write $k = q d$, then $1 \le k \le n$ {{iff}} $1 \le q \le \dfrac n d$. +Hence the last sum for $\map \phi n$ can be written as: +{{begin-eqn}} +{{eqn | l = \phi \left({n}\right) + | r = \sum_{d \mathop \divides n} \paren {\sum_{q \mathop = 1}^{\tfrac n d} \map \mu d} + | c = +}} +{{eqn | r = \sum_{d \mathop \divides n} \map \mu d \sum_{q \mathop = 1}^{\tfrac n d} \map 1 q + | c = +}} +{{eqn | r = \sum_{d \mathop \divides n} \map \mu d \frac n d + | c = +}} +{{end-eqn}} +{{qed}} +\end{proof}<|endoftext|> +\section{Binomial Coefficient of Prime} +Tags: Prime Numbers, Binomial Coefficients, Binomial Coefficient of Prime + +\begin{theorem} +Let $p$ be a [[Definition:Prime Number|prime number]]. +Then: +:$\forall k \in \Z: 0 < k < p: \dbinom p k \equiv 0 \pmod p$ +where $\dbinom p k$ is defined as a [[Definition:Binomial Coefficient|binomial coefficient]]. +\end{theorem} + +\begin{proof} +Because: +:$\dbinom p k = \dfrac {p \left({p - 1}\right) \left({p - 2}\right) \cdots \left({p - k + 1}\right)} {k!}$ +is an [[Definition:Integer|integer]], we have that: +:$k! \mathrel \backslash p \left({p - 1}\right) \left({p - 2}\right) \cdots \left({p - k + 1}\right)$ +But because $k < p$ it follows that: +:$k! \mathop \perp p$ +that is, that: +:$\gcd \left\{{k!, p}\right\} = 1$ +So by [[Euclid's Lemma]]: +:$k! \mathrel \backslash \left({p - 1}\right) \left({p - 2}\right) \cdots \left({p - k + 1}\right)$ +Hence: +:$\dbinom p k = p \dfrac {\left({p - 1}\right) \left({p - 2}\right) \cdots \left({p - k + 1}\right)} {k!}$ +Hence the result. +{{qed}} +\end{proof} + +\begin{proof} +[[Lucas' Theorem]] gives: +:$\dbinom n k \equiv \dbinom {\left \lfloor {n / p} \right \rfloor} {\left \lfloor {k / p} \right \rfloor} \dbinom {n \bmod p} {k \bmod p} \pmod p$ +So, substituting $p$ for $n$: +:$\dbinom p k \equiv \dbinom {\left \lfloor {p / p} \right \rfloor} {\left \lfloor {k / p} \right \rfloor} \dbinom {p \bmod p} {k \bmod p} \pmod p$ +But $p \bmod p = 0$ by [[Definition:Modulo Operation|definition]]. +Hence, if $0 < k < p$, we have that: +:$k \bmod p \ne 0$ +and so: +:$\dbinom {p \bmod p} {k \bmod p} = \dbinom 0 {k \bmod p} = 0$ +by [[Definition:Binomial Coefficient|definition of binomial coefficients]]. +The result follows immediately. +{{qed}} +\end{proof} + +\begin{proof} +By the definition of [[Definition:Binomial Coefficient|binomial coefficient]]: +{{begin-eqn}} +{{eqn | l = \binom p k + | r = \frac {p!} {k! \paren {n - k}!} +}} +{{eqn | ll= \leadstoandfrom + | l = p! + | r = k! \paren {p - k}! \binom p k +}} +{{end-eqn}} +Now, $p$ [[Definition:Divisor of Integer|divides]] the {{LHS}} by [[Divisors of Factorial]]. +So $p$ must also [[Definition:Divisor of Integer|divide]] the {{RHS}}. +[[Definition:By Hypothesis|By hypothesis]] $k < p$. +We have that: +:$k! = k \paren {k - 1} \dotsm \paren 2 \paren 1$ +We also have that $p$ is [[Definition:Prime Number|prime]], and each factor is less than $p$. +Thus $p$ is not a [[Definition:Factor (Algebra)|factor]] of $k!$. +Similarly: +:$\paren {p - k}! = \paren {p - k} \paren {p - k - 1} \dotsm \paren 2 \paren 1$ +It follows, by the same reasoning as above, that $p$ is also not a [[Definition:Factor (Algebra)|factor]] of $\paren {p - k}!$. +The result then follows from [[Euclid's Lemma]]. +{{qed}} +\end{proof}<|endoftext|> +\section{Prime Power of Sum Modulo Prime} +Tags: Number Theory, Combinatorics, Prime Numbers, Proofs by Induction + +\begin{theorem} +Let $p$ be a [[Definition:Prime Number|prime number]]. +Then: +:$\forall n \in \N_{> 0}: \paren {a + b}^{p^n} \equiv a^{p^n} + b^{p^n} \pmod p$ +\end{theorem} + +\begin{proof} +Proof by [[Principle of Mathematical Induction|induction]]: +For all $n \in \N_{> 0}$, let $\map P n$ be the [[Definition:Proposition|proposition]]: +:$\paren {a + b}^{p^n} \equiv a^{p^n} + b^{p^n} \pmod p$ +=== Basis for the Induction === +First from [[Power of Sum Modulo Prime]] we have that $\map P 1$ is true: +:$\paren {a + b}^p \equiv a^p + b^p \pmod p$ +This is our [[Definition:Basis for the Induction|basis for the induction]]. +=== Induction Hypothesis === +Now we need to show that, if $\map P k$ is true, where $k \ge 1$, then it logically follows that $\map P {k + 1}$ is true. +So this is our [[Definition:Induction Hypothesis|induction hypothesis]]: +:$\paren {a + b}^{p^k} \equiv a^{p^k} + b^{p^k} \pmod p$ +Then we need to show: +:$\paren {a + b}^{p^{k + 1} } \equiv a^{p^{k + 1} } + b^{p^{k + 1} } \pmod p$ +=== Induction Step === +This is our [[Definition:Induction Step|induction step]]: +{{begin-eqn}} +{{eqn | l = \paren {a + b}^{p^k} + | o = \equiv + | r = a^{p^k} + b^{p^k} + | rr= \pmod p + | c = [[Prime Power of Sum Modulo Prime#Induction Hypothesis|Induction Hypothesis]] +}} +{{eqn | ll= \leadsto + | l = \paren {\paren {a + b}^{p^k} }^p + | o = \equiv + | r = \paren {a^{p^k} + b^{p^k} }^p + | rr= \pmod p + | c = [[Congruence of Powers]] +}} +{{eqn | ll= \leadsto + | l = \paren {\paren {a + b}^{p^k} }^p + | o = \equiv + | r = \paren {a^{p^k} }^p + \paren {b^{p^k} }^p + | rr= \pmod p + | c = [[Prime Power of Sum Modulo Prime#Basis for the Induction|Basis for the Induction]] +}} +{{eqn | ll= \leadsto + | l = \paren {a + b}^{p^{k + 1} } + | o = \equiv + | r = a^{p^{k + 1} } + b^{p^{k + 1} } + | rr= \pmod p + | c = +}} +{{end-eqn}} +So $\map P k \implies \map P {k + 1}$ and the result follows by the [[Principle of Mathematical Induction]]. +Therefore: +:$\forall n \in \N_{> 0}: \paren {a + b}^{p^n} \equiv a^{p^n} + b^{p^n} \pmod p$ +{{qed}} +\end{proof}<|endoftext|> +\section{Binomial Coefficient involving Power of Prime} +Tags: Prime Numbers, Binomial Coefficients, Binomial Coefficient involving Power of Prime + +\begin{theorem} +: $\dbinom {p^n k} {p^n} \equiv k \pmod p$ +where $\dbinom {p^n k} {p^n}$ is a [[Definition:Binomial Coefficient|binomial coefficient]]. +\end{theorem} + +\begin{proof} +From [[Prime Power of Sum Modulo Prime]] we have: +:$(1): \quad \paren {a + b}^{p^n} \equiv \paren {a^{p^n} + b^{p^n} } \pmod p$ +We can write this: +:$\paren {a + b}^{p^n k} = \paren {\paren {a + b}^{p^n} }^k$ +By $(1)$ and [[Congruence of Powers]], we therefore have: +:$\paren {a + b}^{p^n k} \equiv \paren {a^{p^n} + b^{p^n} }^k \pmod p$ +The coefficient $\dbinom {p^n k} {p^n}$ is the [[Definition:Binomial Coefficient|binomial coefficient]] of $b^{p^n}$ in $\paren {a + b}^{p^n k} = \paren {\paren {a + b}^{p^n} }^k$. +Expanding $\paren {a^{p^n} + b^{p^n} }^k$ using the [[Binomial Theorem]], we find that the coefficient of $b^{p^n}$, the second term, is $\dbinom k 1 = k$. +So: +:$\dbinom {p^n k} {p^n} \equiv k \pmod p$ +{{qed}} +\end{proof} + +\begin{proof} +[[Lucas' Theorem]] states that for $n, k, p \in \Z$ and $p$ be a [[Definition:Prime Number|prime number]], such that: +:$n = a_r p^r + \cdots + a_1 p + a_0$ +:$k = b_r p^r + \cdots + b_1 p + b_0$ +then: +:$\displaystyle \binom n k \equiv \prod_{j \mathop = 0}^r \binom {a_j}{b_j} \pmod p$ +Therefore: +{{begin-eqn}} +{{eqn | l = \binom {p^n k} {p^n} + | o = \equiv + | r = \binom k 1 \prod_{j \mathop = 0}^{n - 1} \binom 0 0 \pmod p + | c = [[Lucas' Theorem]] +}} +{{eqn | o = \equiv + | r = k \prod_{j \mathop = 0}^{n - 1} 1 \pmod p + | c = [[Binomial Coefficient with One]], [[Binomial Coefficient with Zero]] +}} +{{eqn | o = \equiv + | r = k \pmod p + | c = +}} +{{end-eqn}} +{{qed}} +\end{proof}<|endoftext|> +\section{Cassini's Identity} +Tags: Fibonacci Numbers, Cassini's Identity + +\begin{theorem} +:$F_{n + 1} F_{n - 1} - F_n^2 = \paren {-1}^n$ +\end{theorem} + +\begin{proof} +We see that: +: $F_2 F_0 - F_1^2 = 1 \times 0 - 1 = -1 = \left({-1}\right)^1$ +so the proposition holds for $n = 1$. +We also see that: +: $F_3 F_1 - F_2^2 = 2 \times 1 - 1 = \left({-1}\right)^2$ +so the proposition holds for $n = 2$. +Suppose the proposition is true for $n = k$, that is: +: $F_{k + 1} F_{k - 1} - F_k^2 = \left({-1}\right)^k$ +It remains to be shown that it follows from this that the proposition is true for $n = k + 1$, that is: +: $F_{k + 2} F_k - F_{k + 1}^2 = \left({-1}\right)^{k + 1}$ +So: +{{begin-eqn}} +{{eqn | l = F_{k + 2} F_k - F_{k + 1}^2 + | r = \left({F_k + F_{k + 1} }\right) F_k - F_{k + 1}^2 +}} +{{eqn | r = F_k^2 + F_k F_{k + 1} - F_{k + 1}^2 +}} +{{eqn | r = F_k^2 + F_k F_{k + 1} - F_{k + 1} \left({F_k + F_{k - 1} }\right) +}} +{{eqn | r = F_k^2 + F_k F_{k + 1} - F_k F_{k + 1} - F_{k + 1} F_{k - 1} +}} +{{eqn | r = F_k^2 - F_{k + 1} F_{k - 1} +}} +{{eqn | r = \left({-1}\right) \left({F_{k + 1} F_{k - 1} - F_k^2}\right) +}} +{{eqn | r = \left({-1}\right) \left({-1}\right)^k +}} +{{eqn | r = \left({-1}\right)^{k + 1} +}} +{{end-eqn}} +By the [[Principle of Mathematical Induction]], the proof is complete. +{{qed}} +Note that from the above we have that: +: $F_{k + 2} F_k - F_{k + 1}^2 = \left({-1}\right)^{k + 1}$ +from which: +: $F_{n + 1}^2 - F_n F_{n + 2} = \left({-1}\right)^n$ +follows immediately. +\end{proof} + +\begin{proof} +First we use this [[Cassini's Identity/Lemma|lemma]]: +{{:Cassini's Identity/Lemma}} +Then the [[Definition:Determinant of Matrix|determinant]] of both sides is taken. +The {{LHS}} follows directly from the [[Definition:Determinant of Order 2|order 2 determinant]]: +:$\begin{bmatrix} +F_{n + 1} & F_n \\ +F_n & F_{n - 1} +\end{bmatrix} = F_{n + 1} F_{n - 1} - F_n^2$ +Now for the {{RHS}}: +==== Basis for the Induction ==== +:$\begin{vmatrix} +1 & 1 \\ +1 & 0 +\end{vmatrix} = 1 \times 0 - 1 \times 1 = -1 = \left({-1}\right)^1$ +==== Induction Hypothesis ==== +For $k \in \Z_{>0}$, it is assumed that: +:$\begin{vmatrix} +1 & 1 \\ +1 & 0 +\end{vmatrix}^k = \left({-1}\right)^k$ +It remains to be shown that: +:$\begin{vmatrix} +1 & 1 \\ +1 & 0 +\end{vmatrix}^{k + 1} = \left({-1}\right)^{k + 1}$ +==== Induction Step ==== +The induction step follows from [[Determinant of Matrix Product]]: +:$\begin{vmatrix} +1 & 1 \\ +1 & 0 +\end{vmatrix}^{k+1} = \begin{vmatrix} +1 & 1 \\ +1 & 0 +\end{vmatrix}^k \begin{vmatrix} +1 & 1 \\ +1 & 0 +\end{vmatrix} = \left({-1}\right)^k \left({-1}\right) = \left({-1}\right)^{k + 1}$ +Hence by [[Principle of Mathematical Induction|induction]]: +:$\forall n \in \Z_{>0}: \begin{vmatrix} +1 & 1 \\ +1 & 0 +\end{vmatrix}^n = \left({-1}\right)^n$ +{{qed}} +\end{proof}<|endoftext|> +\section{Rational Numbers are Countably Infinite} +Tags: Rational Numbers, Countable Sets, Rational Numbers are Countably Infinite + +\begin{theorem} +The [[Definition:Set|set]] $\Q$ of [[Definition:Rational Number|rational numbers]] is [[Definition:Countably Infinite|countably infinite]]. +\end{theorem} + +\begin{proof} +The [[Definition:Rational Number|rational numbers]] are arranged thus: +:$\dfrac 0 1, \dfrac 1 1, \dfrac {-1} 1, \dfrac 1 2, \dfrac {-1} 2, \dfrac 2 1, \dfrac {-2} 1, \dfrac 1 3, \dfrac 2 3, \dfrac {-1} 3, \dfrac {-2} 3, \dfrac 3 1, \dfrac 3 2, \dfrac {-3} 1, \dfrac {-3} 2, \dfrac 1 4, \dfrac 3 4, \dfrac {-1} 4, \dfrac {-3} 4, \dfrac 4 1, \dfrac 4 3, \dfrac {-4} 1, \dfrac {-4} 3 \ldots$ +It is clear that every [[Definition:Rational Number|rational number]] will appear somewhere in this list. +Thus it is possible to set up a [[Definition:Bijection|bijection]] between each [[Definition:Rational Number|rational number]] and its position in the list, which is an element of $\N$. +{{qed}} +\end{proof} + +\begin{proof} +Let us define the mapping $\phi: \Q \to \Z \times \N$ as follows: +:$\forall \dfrac p q \in \Q: \phi \left({\dfrac p q}\right) = \left({p, q}\right)$ +where $\dfrac p q$ is [[Definition:Canonical Form of Rational Number|in canonical form]]. +Then $\phi$ is clearly [[Definition:Injection|injective]]. +From [[Cartesian Product of Countable Sets is Countable]], we have that $\Z \times \N$ is [[Definition:Countable|countably infinite]]. +The result follows directly from [[Domain of Injection to Countable Set is Countable]]. +{{qed}} +\end{proof} + +\begin{proof} +For each $n \in \N$, define $S_n$ to be the [[Definition:Set|set]]: +:$S_n := \left\{{\dfrac m n: m \in \Z}\right\}$ +By [[Integers are Countably Infinite]], each $S_n$ is [[Definition:Countably Infinite|countably infinite]]. +Because each [[Definition:Rational Number|rational number]] can be written down with a [[Definition:Positive Integer|positive]] [[Definition:Denominator|denominator]], it follows that: +:$\forall q \in \Q: \exists n \in \N: q \in S_n$ +which is to say: +:$\displaystyle \bigcup_{n \mathop \in \N} S_n = \Q$ +By [[Countable Union of Countable Sets is Countable]], it follows that $\Q$ is [[Definition:Countable|countable]]. +Since $\Q$ is manifestly [[Definition:Infinite Set|infinite]], it is [[Definition:Countably Infinite|countably infinite]]. +{{qed}} +\end{proof} + +\begin{proof} +Let $Q_\pm = \set {q \in \Q: \pm q > 0}$. +For every $q \in Q_+$, there exists at least one pair $\tuple {m, n} \in \N \times \N$ such that $q = \dfrac m n$. +Therefore, we can find an [[Definition:Injection|injection]] $i: Q_+ \to \N \times \N$. +By [[Cartesian Product of Natural Numbers with Itself is Countable]], $\N \times \N$ is [[Definition:Countable|countable]]. +Hence $Q_+$ is [[Definition:Countable|countable]], by [[Domain of Injection to Countable Set is Countable]]. +The map $-: q \mapsto -q$ provides a [[Definition:Bijection|bijection]] from $Q_-$ to $Q_+$, hence $Q_-$ is also [[Definition:Countable|countable]]. +Hence $\Q$ is [[Definition:Countable Set|countable]]. +{{qed}} +\end{proof}<|endoftext|> +\section{Real Numbers form Ordered Field} +Tags: Examples of Fields, Ordered Fields, Real Numbers + +\begin{theorem} +The [[Definition:Real Number|set of real numbers]] $\R$ forms an [[Definition:Ordered Field|ordered field]] under [[Definition:Real Addition|addition]] and [[Definition:Real Multiplication|multiplication]]: $\struct {\R, +, \times, \le}$. +\end{theorem} + +\begin{proof} +From [[Real Numbers form Field]], we have that $\struct {\R, +, \times}$ forms a [[Definition:Field (Abstract Algebra)|field]]. +From [[Ordering Properties of Real Numbers]] we have that $\struct {\R, +, \times, \le}$ is a [[Definition:Ordered Field|ordered field]]. +{{qed}} +\end{proof}<|endoftext|> +\section{Rational Numbers form Subfield of Real Numbers} +Tags: Rational Numbers, Real Numbers + +\begin{theorem} +The [[Definition:Ordered Field|(ordered) field]] $\struct {\Q, +, \times, \le}$ of [[Definition:Rational Number|rational numbers]] forms a [[Definition:Subfield|subfield]] of the [[Definition:Field of Real Numbers|field of real numbers]] $\struct {\R, +, \times, \le}$. +That is, the [[Definition:Field of Real Numbers|field of real numbers]] $\struct {\R, +, \times, \le}$ is an [[Definition:Extension of Operation|extension]] of the [[Definition:Rational Number|rational numbers]] $\struct {\Q, +, \times, \le}$. +\end{theorem} + +\begin{proof} +Recall that [[Rational Numbers form Ordered Field]]. +Then from [[Rational Numbers form Subset of Real Numbers]]: +:$\Q \subseteq \R$ +Hence the result by definition of [[Definition:Subfield|subfield]]. +{{qed}} +\end{proof}<|endoftext|> +\section{Definition:Rational Number/Canonical Form} +Tags: Definitions: Rational Numbers, Definitions: Fractions + +\begin{theorem} +Let $r \in \Q$ be a [[Definition:Rational Number|rational number]]. +The '''canonical form of $r$''' is the expression $\dfrac p q$, where: +:$r = \dfrac p q: p \in \Z, q \in \Z_{>0}, p \perp q$ +where $p \perp q$ denotes that $p$ and $q$ have no [[Definition:Common Divisor of Integers|common divisor]] except $1$. +\end{theorem}<|endoftext|> +\section{Ordering Properties of Real Numbers} +Tags: Real Numbers, Inequalities + +\begin{theorem} +=== [[Trichotomy Law for Real Numbers|Trichotomy Law]] === +{{:Trichotomy Law for Real Numbers}} +=== [[Real Number Ordering is Transitive|Ordering is Transitive]] === +{{:Real Number Ordering is Transitive}} +=== [[Real Number Ordering is Compatible with Addition|Ordering is Compatible with Addition]] === +{{:Real Number Ordering is Compatible with Addition}} +=== [[Real Number Ordering is Compatible with Multiplication|Ordering is Compatible with Multiplication]] === +{{:Real Number Ordering is Compatible with Multiplication}} +\end{theorem}<|endoftext|> +\section{Order is Preserved on Positive Reals by Squaring} +Tags: Analysis, Inequalities, Order is Preserved on Positive Reals by Squaring + +\begin{theorem} +:$x < y \iff x^2 < y^2$ +\end{theorem} + +\begin{proof} +=== Necessary Condition === +Assume $x < y$. +Then: +{{begin-eqn}} +{{eqn | l = x < y + | o = \implies + | r = x \times x < x \times y + | c = [[Real Number Ordering is Compatible with Multiplication]] +}} +{{eqn | l = x < y + | o = \implies + | r = x \times y < y \times y + | c = [[Real Number Ordering is Compatible with Multiplication]] +}} +{{eqn | o = \leadsto + | r = x^2 < y^2 + | c = [[Real Number Ordering is Transitive]] +}} +{{end-eqn}} +So: +:$x < y \implies x^2 < y^2$ +{{qed|lemma}} +=== Sufficient Condition === +Assume $x^2 < y^2$. +Then: +{{begin-eqn}} +{{eqn | l = x^2 + | o = < + | r = y^2 + | c = +}} +{{eqn | ll= \leadsto + | l = 0 + | o = < + | r = y^2 - x^2 + | c = [[Real Number Ordering is Compatible with Addition]] +}} +{{eqn | ll=\leadsto + | l = \paren {y - x} \paren {y + x} + | o = > + | r = 0 + | c = [[Difference of Two Squares]] +}} +{{eqn | ll= \leadsto + | l = \paren {y - x} \paren {y + x} \paren {y + x}^{-1} + | o = > + | r = 0 \times \paren {y + x}^{-1} + | c = as $x + y > 0$ +}} +{{eqn | ll= \leadsto + | l = y - x + | o = > + | r = 0 + | c = +}} +{{eqn | ll= \leadsto + | l = x + | o = < + | r = y + | c = +}} +{{end-eqn}} +So: +:$x^2 < y^2 \implies x < y$ +{{qed}} +\end{proof} + +\begin{proof} +From [[Real Numbers form Ordered Field]], the [[Definition:Real Number|real numbers]] form an [[Definition:Ordered Field|ordered field]]. +The result follows from [[Order of Squares in Ordered Field]]. +{{qed}} +\end{proof} + +\begin{proof} +From [[Real Numbers form Ordered Field]], the [[Definition:Real Number|real numbers]] form an [[Definition:Ordered Field|ordered field]]. +By definition, an [[Definition:Ordered Field|ordered field]] is a [[Definition:Totally Ordered Ring|totally ordered ring]] without [[Definition:Proper Zero Divisor|proper zero divisors]]. +The result follows from [[Order of Squares in Totally Ordered Ring without Proper Zero Divisors]]. +{{qed}} +\end{proof} + +\begin{proof} +=== Necessary Condition === +Let $x < y$. +Then: +{{begin-eqn}} +{{eqn | l = x < y + | o = \implies + | r = x \times x < x \times y + | c = [[Real Number Ordering is Compatible with Multiplication]] +}} +{{eqn | l = x < y + | o = \implies + | r = x \times y < y \times y + | c = [[Real Number Ordering is Compatible with Multiplication]] +}} +{{eqn | o = \leadsto + | r = x^2 < y^2 + | c = [[Real Number Ordering is Transitive]] +}} +{{end-eqn}} +So: +:$x < y \implies x^2 < y^2$ +{{qed|lemma}} +=== Sufficient Condition === +Let $x^2 < y^2$. +{{AimForCont}} $x \ge y$. +Then: +{{begin-eqn}} +{{eqn | l = x \ge y + | o = \implies + | r = x \times x \ge x \times y + | c = [[Real Number Ordering is Compatible with Multiplication]] +}} +{{eqn | l = x \ge y + | o = \implies + | r = x \times y \ge y \times y + | c = [[Real Number Ordering is Compatible with Multiplication]] +}} +{{eqn | o = \leadsto + | r = x^2 \ge y^2 + | c = [[Real Number Ordering is Transitive]] +}} +{{end-eqn}} +But this [[Definition:Contradiction|contradicts]] our assertion that $x^2 < y^2$. +Hence by [[Proof by Contradiction]] it follows that: +:$x < y$ +{{qed}} +\end{proof}<|endoftext|> +\section{Real Plus Epsilon} +Tags: Real Analysis, Inequalities + +\begin{theorem} +Let $a, b \in \R$, such that: +:$\forall \epsilon \in \R_{>0}: a < b + \epsilon$ +where $\R_{>0}$ is the set of [[Definition:Strictly Positive|strictly positive]] [[Definition:Real Number|real numbers]]. +That is: +:$\epsilon > 0$ +Then: +:$a \le b$ +\end{theorem} + +\begin{proof} +{{AimForCont}} $a > b$. +Then: +:$a - b > 0$ +[[Definition:By Hypothesis|By hypothesis]], we have: +:$\forall \epsilon > 0: a < b + \epsilon$ +Let $\epsilon = a - b$. +Then: +:$a < b + \paren {a - b} \implies a < a$ +The result follows by [[Proof by Contradiction]]. +{{qed}} +\end{proof}<|endoftext|> +\section{Difference of Two Squares} +Tags: Commutative Rings, Square Function, Polynomial Theory, Difference of Two Squares + +\begin{theorem} +Let $\struct {R, +, \circ}$ be a [[Definition:Commutative Ring|commutative ring]] whose [[Definition:Ring Zero|zero]] is $0_R$. +Let $x, y \in R$. +Then: +: $x \circ x + \paren {- \paren {y \circ y} } = \paren {x + y} \circ \paren {x + \paren {-y} }$ +When $R$ is one of the standard [[Definition:Number|sets of numbers]], that is $\Z, \Q, \R$, and so on, then this translates into: +:$x^2 - y^2 = \paren {x + y} \paren {x - y}$ +\end{theorem} + +\begin{proof} +{{begin-eqn}} +{{eqn | l = \paren {x + y} \circ \paren {x + \paren {-y} } + | r = x \circ x + y \circ x + x \circ \paren {-y} + y \circ \paren {-y} + | c = [[Definition:Distributive Operation|Distributivity]] of $\circ$ over $+$ in a [[Definition:Ring (Abstract Algebra)|ring]] +}} +{{eqn | r = x \circ x + x \circ y + x \circ \paren {-y} + y \circ \paren {-y} + | c = $R$ is a [[Definition:Commutative Ring|commutative ring]] +}} +{{eqn | r = x \circ x + x \circ \paren {y + \paren {-y} } + \paren {-\paren {y \circ y} } + | c = various [[Definition:Ring (Abstract Algebra)|ring]] properties +}} +{{eqn | r = x \circ x + x \circ 0_R + \paren {-\paren {y \circ y} } + | c = +}} +{{eqn | r = x \circ x + \paren {-\paren {y \circ y} } + | c = +}} +{{end-eqn}} +{{qed}} +\end{proof} + +\begin{proof} +This is a special case of [[Difference of Two Powers]]: +:$\displaystyle a^n - b^n = \left({a - b}\right) \left({a^{n - 1} + a^{n - 2} b + a^{n - 3} b^2 + \cdots + a b^{n - 2} + b^{n - 1} }\right) = \left({a - b}\right) \sum_{j \mathop = 0}^{n - 1} a^{n - j - 1} b^j$ +The result follows by setting $n = 2$. +{{qed}} +\end{proof} + +\begin{proof} +{{:Euclid:Proposition/II/5}} +:[[File:Euclid-II-5.png|400px]] +Let $AB$ be cut into equal segments at $C$ and unequal segments at $D$. +Then the [[Definition:Containment of Rectangle|rectangle contained]] by $AD$ and $DB$ together with the square on $CD$ equals the square on $BC$. +(That is, let $x = AC, y = CD$. Then $\paren {x + y} \paren {x - y} + y^2 = x^2$.) +This is proved as follows. +[[Construction of Square on Given Straight Line|Construct the square $CBFE$]] on $CB$, and join $BE$. +[[Construction of Parallel Line|Construct $DG$ parallel]] to $CE$ through $G$, and let $DG$ cross $BE$ at $H$. +[[Construction of Parallel Line|Construct $KM$ parallel]] to $AB$ through $H$. +[[Construction of Parallel Line|Construct $AK$ parallel]] to $BF$ through $A$. +From [[Complements of Parallelograms are Equal]]: +:$\Box CDHL = \Box FGHM$. +Add the square $DBMH$ to each. +Then $\Box CBML = \Box DBFG$. +But as $AC = CB$, from [[Parallelograms with Equal Base and Same Height have Equal Area]] we have that: +:$\Box ACLK = \Box CBML$ +Add $\Box CDHL$ to each. +Then $\Box ADHK$ is equal in area to the [[Definition:Gnomon|gnomon]] $CBFGHL$. +But $\Box ADHK$ is the [[Definition:Containment of Rectangle|rectangle contained]] by $AD$ and $DB$, because $DB = DH$. +So the [[Definition:Gnomon|gnomon]] $CBFGHL$ is equal in area to the [[Definition:Containment of Rectangle|rectangle contained]] by $AD$ and $DB$. +Now $\Box LHGE$ is equal to the square on $CD$. +Add $\Box LHGE$ to each of the [[Definition:Gnomon|gnomon]] $CBFGHL$ and $\Box ADHK$. +Then the [[Definition:Gnomon|gnomon]] $CBFGHL$ together with $\Box LHGE$ equals the [[Definition:Containment of Rectangle|rectangle contained]] by $AD$ and $DB$ and the square on $CD$. +But the [[Definition:Gnomon|gnomon]] $CBFGHL$ together with $\Box LHGE$ is the square $CBFE$. +Hence the result. +{{qed}} +\end{proof}<|endoftext|> +\section{Mediant is Between} +Tags: Real Analysis + +\begin{theorem} +Let $a, b, c, d$ be ''any'' [[Definition:Real Number|real numbers]] such that $b > 0, d > 0$. +Let $r = \dfrac a b < \dfrac c d = s$. +Then: +:$r < \dfrac {a + c} {b + d} < s$ +\end{theorem} + +\begin{proof} +Let $r, s \in \R$ be such that: +:$r < s$ +and: +:$r = \dfrac a b, s = \dfrac c d$ +where $a, b, c, d$ are [[Definition:Real Number|real numbers]] such that $b > 0, d > 0$. +Because $b, d > 0$, it follows from [[Real Number Ordering is Compatible with Multiplication]] that: +:$b d > 0$ +Thus: +{{begin-eqn}} +{{eqn | l = \frac a b + | o = < + | r = \frac c d + | c = +}} +{{eqn | ll= \leadsto + | l = \frac a b \paren {b d} + | o = < + | r = \frac c d \paren {b d} + | c = [[Real Number Ordering is Compatible with Multiplication]] +}} +{{eqn | ll= \leadsto + | l = a d + | o = < + | r = b c + | c = +}} +{{end-eqn}} +Then: +{{begin-eqn}} +{{eqn | l = a \paren {b + d} + | r = a b + a d + | c = +}} +{{eqn | o = < + | r = a b + b c + | c = [[Real Number Ordering is Compatible with Addition]], as $a d < b c$ from above +}} +{{eqn | r = \paren {a + c} b + | c = +}} +{{end-eqn}} +From [[Reciprocal of Strictly Positive Real Number is Strictly Positive]]: +:$\paren {a + c}^{-1} > 0$ +and: +:$b^{-1} > 0$ +It follows from [[Real Number Ordering is Compatible with Multiplication]] that: +:$\dfrac a b < \dfrac {a + c} {b + d}$ +The other half is proved similarly. +{{Qed}} +\end{proof}<|endoftext|> +\section{Real Numbers Between Epsilons} +Tags: Real Analysis + +\begin{theorem} +Let $a, b \in \R$ such that $\forall \epsilon \in \R_{>0}: a - \epsilon < b < a + \epsilon$. +Then $a = b$. +\end{theorem} + +\begin{proof} +From [[Real Plus Epsilon]]: +:$b < a + \epsilon \implies b \le a$ +From [[Real Number Ordering is Compatible with Addition]]: +:$a - \epsilon < b \implies a < b + \epsilon$ +Then from [[Real Plus Epsilon]]: +:$a < b + \epsilon \implies a \le b$ +The result follows. +{{qed}} +\end{proof}<|endoftext|> +\section{Exists Integer Below Any Real Number} +Tags: Analysis + +\begin{theorem} +Let $x$ be a [[Definition:Real Number|real number]]. +Then there exists an [[Definition:Integer|integer]] less than $x$: +:$\forall x \in \R: \exists n \in \Z: n < x$ +\end{theorem} + +\begin{proof} +Clearly we may assume without loss of generality that $x < 0$. +From the [[Archimedean Principle]]: +:$\exists m \in \N: m > -x$ +By [[Real Numbers form Ordered Field]], we have that $\R$ is an [[Definition:Ordered Field|ordered field]]. +Therefore by property $(3)$ of [[Properties of Ordered Field]], $\Z \owns -m < x$. +{{qed}} +[[Category:Analysis]] +279b5qb10xgfmugmysigl3ksxrndsf3 +\end{proof}<|endoftext|> +\section{Archimedean Principle} +Tags: Real Analysis, Number Theory + +\begin{theorem} +Let $x$ be a [[Definition:Real Number|real number]]. +Then there exists a [[Definition:Natural Numbers|natural number]] greater than $x$. +:$\forall x \in \R: \exists n \in \N: n > x$ +That is, the [[Definition:Natural Numbers|set of natural numbers]] is [[Definition:Bounded Above Set|unbounded above]]. +\end{theorem} + +\begin{proof} +Let $x \in \R$. +Let $S$ be the set of all [[Definition:Natural Numbers|natural numbers]] less than or equal to $x$: +:$S = \set {a \in \N: a \le x}$ +It is possible that $S = \O$. +Suppose $0 \le x$. +Then by definition, $0 \in S$. +But $S = \O$, so this is a [[Proof by Contradiction|contradiction]]. +From the [[Trichotomy Law for Real Numbers]] it follows that $0 > x$. +Thus we have the element $0 \in \N$ such that $0 > x$. +Now suppose $S \ne \O$. +Then $S$ is [[Definition:Bounded Above Set|bounded above]] (by $x$, for example). +Thus by the [[Continuum Property]] of $\R$, $S$ has a [[Definition:Supremum of Set|supremum]] in $\R$. +Let $s = \map \sup S$. +Now consider the number $s - 1$. +Since $s$ is the [[Definition:Supremum of Set|supremum]] of $S$, $s - 1$ cannot be an [[Definition:Upper Bound of Set|upper bound]] of $S$ by definition. +So $\exists m \in S: m > s - 1 \implies m + 1 > s$. +But as $m \in \N$, it follows that $m + 1 \in \N$. +Because $m + 1 > s$, it follows that $m + 1 \notin S$ and so $m + 1 > x$. +\end{proof}<|endoftext|> +\section{Real Number is between Floor Functions} +Tags: Floor Function + +\begin{theorem} +:$\forall x \in \R: \floor x \le x < \floor {x + 1}$ +\end{theorem} + +\begin{proof} +$\floor x$ is defined as: +:$\floor x = \sup \set {m \in \Z: m \le x}$ +So $\floor x \le x$ by definition. +From [[Floor plus One]]: +:$\floor {x + 1} > \floor x$ +Hence by the definition of the [[Definition:Supremum of Set|supremum]]: +:$\floor {x + 1} > x$ +The result follows. +{{qed}} +[[Category:Floor Function]] +akn94fzyqk42jhnqcxjm9xqmvzfon8w +\end{proof}<|endoftext|> +\section{Real Number is between Ceiling Functions} +Tags: Ceiling Function + +\begin{theorem} +:$\forall x \in \R: \left \lceil {x - 1} \right \rceil \le x < \left \lceil {x} \right \rceil$ +\end{theorem} + +\begin{proof} +$\left \lceil {x} \right \rceil$ is defined as: +:$\left \lceil {x} \right \rceil = \inf \left({\left\{{m \in \Z: m \ge x}\right\}}\right)$ +So $\left \lceil {x} \right \rceil \ge x$ by definition. +Now $\left \lceil {x - 1} \right \rceil < \left \lceil {x} \right \rceil$, so by the definition of the [[Definition:Infimum of Set|infimum]]: +:$\left \lceil {x - 1} \right \rceil > x$ +The result follows. +{{qed}} +[[Category:Ceiling Function]] +d8pg5fzw8p22asnvcvmbkt769zo7dcs +\end{proof}<|endoftext|> +\section{Real Number minus Floor} +Tags: Floor Function + +\begin{theorem} +:$x - \floor x \in \hointr 0 1$ +\end{theorem} + +\begin{proof} +{{begin-eqn}} +{{eqn | l = \floor x + | o = \le + | m = x + | mo= < + | r = \floor x + 1 + | c = {{Defof|Floor Function}} +}} +{{eqn | ll= \implies + | l = \floor x - \floor x + | o = \le + | m = x - \floor x + | mo= < + | r = \floor x + 1 - \floor x + | c = subtracting $\floor x$ from all parts +}} +{{eqn | ll= \implies + | l = 0 + | o = \le + | m = x - \floor x + | mo= < + | r = 1 + | c = +}} +{{eqn | ll= \implies + | l = x - \floor x + | o = \in + | m = \hointr 0 1 + | c = as required +}} +{{end-eqn}} +{{qed}} +\end{proof}<|endoftext|> +\section{Ceiling minus Real Number} +Tags: Ceiling Function + +\begin{theorem} +:$\forall x \in \R: \left \lceil {x} \right \rceil - x \in \left[{0 \,.\,.\, 1}\right)$ +\end{theorem} + +\begin{proof} +{{begin-eqn}} +{{eqn | l = \left \lceil {x} \right \rceil - 1 + | o = < + | r = x \le \left \lceil {x} \right \rceil + | c = [[Real Number is between Ceiling Functions]] +}} +{{eqn | ll= \implies + | l = \left \lceil {x} \right \rceil - 1 - \left \lceil {x} \right \rceil + | o = < + | r = x - \left \lceil {x} \right \rceil \le \left \lceil {x} \right \rceil - \left \lceil {x} \right \rceil + | c = +}} +{{eqn | ll= \implies + | l = -1 + | o = < + | r = x - \left \lceil {x} \right \rceil \le 0 + | c = +}} +{{eqn | ll= \implies + | l = 1 + | o = > + | r = \left \lceil {x} \right \rceil - x \ge 0 + | c = +}} +{{eqn | ll= \implies + | l = \left \lceil {x} \right \rceil - x + | o = \in + | r = \left[{0 \,.\,.\, 1}\right) + | c = +}} +{{end-eqn}} +{{qed}} +[[Category:Ceiling Function]] +f6xkmvsorteylqbus2ipuydihvfmd4a +\end{proof}<|endoftext|> +\section{Real Number is Floor plus Difference} +Tags: Floor Function + +\begin{theorem} +:There exists an [[Definition:Integer|integer]] $n \in \Z$ such that for some $t \in \hointr 0 1$: +::$x = n + t$ +{{iff}}: +:$n = \floor x$ +\end{theorem} + +\begin{proof} +=== Sufficient Condition === +Let there exist $n \in \Z$ such that $x = n + t$, where $t \in \hointr 0 1$. +We have that $1 - t > 0$. +Thus: +:$0 \le x - n < 1$ +Thus: +: $n \le x < n + 1$ +That is, $n$ is the [[Definition:Floor Function|floor]] of $x$. +{{qed|lemma}} +=== Necessary Condition === +Let $n = \floor x$. +Let $t = x - \floor x$. +Then $x = n + t$. +From [[Real Number minus Floor]]: +:$t = x - \floor x \in \hointr 0 1$ +and so: +:$x = n + t: t \in \hointr 0 1$ +{{qed}} +\end{proof}<|endoftext|> +\section{Floor plus One} +Tags: Floor Function + +\begin{theorem} +Let $x \in \R$. +Then: +:$\left \lfloor {x + 1} \right \rfloor = \left \lfloor {x} \right \rfloor + 1$ +where $\left \lfloor {x} \right \rfloor$ is the [[Definition:Floor Function|floor function]] of $x$. +\end{theorem} + +\begin{proof} +{{begin-eqn}} +{{eqn | l = \left \lfloor {x + 1} \right \rfloor + | r = n + | c = +}} +{{eqn | ll= \implies + | l = n + | o = \le + | r = x + 1 < n + 1 + | c = Definition of [[Definition:Floor Function|Floor Function]] +}} +{{eqn | ll= \implies + | l = n - 1 + | o = \le + | r = x < n + | c = +}} +{{eqn | ll= \implies + | l = \left \lfloor {x} \right \rfloor + | r = n - 1 + | c = Definition of [[Definition:Floor Function|Floor Function]] +}} +{{eqn | ll= \implies + | l = \left \lfloor {x + 1} \right \rfloor + | r = \left \lfloor {x} \right \rfloor + 1 + | c = +}} +{{end-eqn}} +{{qed}} +\end{proof}<|endoftext|> +\section{Real Number is Integer iff equals Floor} +Tags: Floor Function + +\begin{theorem} +:$x = \floor x \iff x \in \Z$ +\end{theorem} + +\begin{proof} +Let $x = \floor x$. +As $\floor x \in \Z$, then so must $x$ be. +Now let $x \in \Z$. +We have: +:$\floor x = \sup \set {m \in \Z: m \le x}$ +As $x \in \sup \set {m \in \Z: m \le x}$, and there can be no greater $n \in \Z$ such that $n \in \sup \set {m \in \Z: m \le x}$, it follows that: +:$x = \floor x$ +{{qed}} +\end{proof}<|endoftext|> +\section{Sum of Floor and Floor of Negative} +Tags: Floor Function + +\begin{theorem} +Let $x \in \R$. Then: +:$\floor x + \floor {-x} = \begin{cases} 0 & : x \in \Z \\ -1 & : x \notin \Z \end{cases}$ +where $\floor x$ denotes the [[Definition:Floor Function|floor]] of $x$. +\end{theorem} + +\begin{proof} +Let $x \in \Z$. +Then from [[Real Number is Integer iff equals Floor]]: +:$x = \floor x$ +Now $x \in \Z \implies -x \in \Z$, so: +:$\floor {-x} = -x$ +Thus: +:$\floor x + \floor {-x} = x + \paren {-x} = x - x = 0$ +Now let $x \notin \Z$. +From [[Real Number is Floor plus Difference]]: +:$x = n + t$ +where $n = \floor x$ and $t \in \hointr 0 1$. +Thus: +:$-x = -\paren {n + t} = -n - t = -n - 1 + \paren {1 - t}$ +As $t \in \hointr 0 1$, we have: +:$1 - t \in \hointr 0 1$ +Thus: +:$\floor {-x} = -n - 1$ +So: +:$\floor x + \floor {-x} = n + \paren {-n - 1} = n - n - 1 = -1$ +{{qed}} +\end{proof}<|endoftext|> +\section{Floor defines Equivalence Relation} +Tags: Floor Function, Equivalence Relations + +\begin{theorem} +Let $x \in \R$ be a [[Definition:Real Number|real number]]. +Let $\floor x$ denote the [[Definition:Floor Function|floor function]] of $x$. +Let $\RR$ be the [[Definition:Relation|relation]] defined on $\R$ such that: +:$\forall x, y, \in \R: \tuple {x, y} \in \RR \iff \floor x = \floor y$ +Then $\RR$ is an [[Definition:Equivalence Relation|equivalence]], and $\forall n \in \Z$, the [[Definition:Equivalence Class|$\RR$-class]] of $n$ is the [[Definition:Half-Open Real Interval|half-open interval]] $\hointr n {n + 1}$. +\end{theorem} + +\begin{proof} +Checking in turn each of the critera for [[Definition:Equivalence Relation|equivalence]]: +=== Reflexivity === +:$\forall x \in \R: \floor x = \floor x$ +Thus the [[Definition:Floor Function|floor function]] is [[Definition:Reflexive Relation|reflexive]]. +{{qed|lemma}} +=== Symmetry === +:$\forall x, y \in \R: \floor x = \floor y \implies \floor y = \floor x$ +Thus the [[Definition:Floor Function|floor function]] is [[Definition:Symmetric Relation|symmetric]]. +{{qed|lemma}} +=== Transitivity === +Let $\floor x = \floor y$ and $\floor y = \floor z$. +Let $n = \floor x = \floor y = \floor z$, which follows from transitivity of $=$. +Thus from [[Real Number is Floor plus Difference]]: +:$x = n + t_x, y = n + t_y, z = n + t_z: t_x, t_y, t_z \in \hointr 0 1$ +Thus: +:$x = n + t_x, z = n + t_z$ +and: +:$\floor x = \floor z$ +Thus the [[Definition:Floor Function|floor function]] is [[Definition:Transitive Relation|transitive]]. +{{qed|lemma}} +Thus we have shown that $\RR$ is an [[Definition:Equivalence Relation|equivalence relation]]. +{{qed|lemma}} +Now we show that the [[Definition:Equivalence Class|$\RR$-class]] of $n$ is the [[Definition:Half-Open Real Interval|interval]] $\hointr n {n + 1}$. +Defining $\RR$ as above, with $n \in \Z$: +{{begin-eqn}} +{{eqn | l = x + | o = \in + | r = \eqclass n \RR + | c = +}} +{{eqn | ll= \leadsto + | l = \floor x + | r = \floor n + | c = +}} +{{eqn | r = n + | c = +}} +{{eqn | ll= \leadsto + | l = \exists t \in \hointr 0 1: x + | r = n + t + | c = +}} +{{eqn | ll= \leadsto + | l = x + | o = \in + | r = \hointr n {n + 1} + | c = +}} +{{end-eqn}} +{{qed}} +\end{proof}<|endoftext|> +\section{Ceiling defines Equivalence Relation} +Tags: Ceiling Function, Equivalence Relations + +\begin{theorem} +Let $\mathcal R$ be the [[Definition:Relation|relation]] defined on $\R$ such that: +:$\forall x, y, \in \R: \left({x, y}\right) \in \mathcal R \iff \left \lceil {x}\right \rceil = \left \lceil {y}\right \rceil$ +where $\left \lceil {x}\right \rceil$ is the [[Definition:Ceiling Function|ceiling]] of $x$. +Then $\mathcal R$ is an [[Definition:Equivalence Relation|equivalence]], and $\forall n \in \Z$, the [[Definition:Equivalence Class|$\mathcal R$-class]] of $n$ is the [[Definition:Half-Open Real Interval|half-open interval]] $\left({n - 1 \,.\,.\, n}\right]$. +\end{theorem} + +\begin{proof} +Checking in turn each of the critera for [[Definition:Equivalence Relation|equivalence]]: +=== Reflexivity === +:$\forall x \in \R: \left \lceil {x}\right \rceil = \left \lceil {x}\right \rceil$ +Thus the [[Definition:Ceiling Function|ceiling function]] is [[Definition:Reflexive Relation|reflexive]]. +{{qed|lemma}} +=== Symmetry === +:$\forall x, y \in \R: \left \lceil {x}\right \rceil = \left \lceil {y}\right \rceil \implies \left \lceil {y}\right \rceil = \left \lceil {x}\right \rceil$ +Thus the [[Definition:Ceiling Function|ceiling function]] is [[Definition:Symmetric Relation|symmetric]]. +{{qed|lemma}} +=== Transitivity === +Let: +:$\left \lceil {x}\right \rceil = \left \lceil {y}\right \rceil$ +:$\left \lceil {y}\right \rceil = \left \lceil {z}\right \rceil$ +Let: +:$n = \left \lceil {x}\right \rceil = \left \lceil {y}\right \rceil = \left \lceil {z}\right \rceil$ +which follows from transitivity of $=$. +Thus: +:$x = n - t_x, y = n - t_y, z = n - t_z: t_x, t_y, t_z \in \left[{0 \,.\,.\, 1}\right)$ +from [[Real Number is Ceiling minus Difference]]. +So: +:$x = n - t_x$ +and: +:$z = n - t_z$ +and so: +:$\left \lceil {x}\right \rceil = \left \lceil {z}\right \rceil$ +Thus the [[Definition:Ceiling Function|ceiling function]] is [[Definition:Transitive Relation|transitive]]. +{{qed|lemma}} +Thus we have shown that $\mathcal R$ is an [[Definition:Equivalence Relation|equivalence]]. +Now we show that the [[Definition:Equivalence Class|$\mathcal R$-class]] of $n$ is the [[Definition:Half-Open Real Interval|interval]] $\left({n - 1 \,.\,.\, n}\right]$. +Defining $\mathcal R$ as above, with $n \in \Z$: +{{begin-eqn}} +{{eqn | l = x + | o = \in + | r = \left[\!\left[{n}\right]\!\right]_\mathcal R + | c = +}} +{{eqn | ll= \implies + | l = \left \lceil {x}\right \rceil + | r = \left \lceil {n}\right \rceil = n + | c = +}} +{{eqn | ll= \implies + | l = \exists t \in \left({0 \,.\,.\, 1}\right]: x + | r = n - t + | c = +}} +{{eqn | ll= \implies + | l = x + | o = \in + | r = \left({n - 1 \,.\,.\, n}\right] + | c = +}} +{{end-eqn}} +{{qed}} +[[Category:Ceiling Function]] +[[Category:Equivalence Relations]] +kktdzbepmt2otz9fgu475n43d93twsn +\end{proof}<|endoftext|> +\section{Real Number is Ceiling minus Difference} +Tags: Ceiling Function + +\begin{theorem} +Let $n$ be a [[Definition:Integer|integer]]. +{{TFAE}} +:$(1): \quad$ There exists $t \in \hointr 0 1$ such that $x = n - t$ +:$(2): \quad n = \ceiling x$ +\end{theorem} + +\begin{proof} +=== 1 implies 2 === +Let $x = n - t$, where $t \in \hointr 0 1$. +Because $0 \le t < 1$, we have: +:$0 \leq n - x < 1$ +Thus: +:$n - 1 < x \le n$ +That is, $n$ is the [[Definition:Ceiling Function|ceiling]] of $x$. +{{qed|lemma}} +=== 2 implies 1 === +Now let $n = \ceiling x$. +Let $t = \ceiling x - x$. +Then $x = n - t$. +From [[Ceiling minus Real Number]], $t = \ceiling x - x \in \hointr 0 1$ +{{qed}} +\end{proof}<|endoftext|> +\section{Cauchy's Inequality} +Tags: Analysis, Inequalities, Cauchy's Inequality, Cauchy-Bunyakovsky-Schwarz Inequality + +\begin{theorem} +:$\displaystyle \sum {r_i^2} \sum {s_i^2} \ge \left({\sum {r_i s_i}}\right)^2$ +\end{theorem} + +\begin{proof} +For any $\lambda \in \R$, we define $f: \R \to \R$ as the [[Definition:Real Function|function]]: +:$\displaystyle \map f \lambda = \sum {\paren {r_i + \lambda s_i}^2}$ +Now: +:$\map f \lambda \ge 0$ +because it is the sum of [[Definition:Square Function|squares]] of [[Definition:Real Number|real numbers]]. +Hence: +{{begin-eqn}} +{{eqn | l = \forall \lambda \in \R: \map f \lambda + | o = \equiv + | r = \sum {\paren {r_i^2 + 2 \lambda r_i s_i + \lambda^2 s_i^2} } \ge 0 + | c = +}} +{{eqn | o = \equiv + | r = \sum {r_i^2} + 2 \lambda \sum {r_i s_i} + \lambda^2 \sum {s_i^2} \ge 0 + | c = +}} +{{end-eqn}} +This is a [[Definition:Quadratic Equation|quadratic equation]] in $\lambda$. +From [[Solution to Quadratic Equation]]: +:$\displaystyle a \lambda^2 + b \lambda + c = 0: a = \sum {s_i^2}, b = 2 \sum {r_i s_i}, c = \sum {r_i^2}$ +The [[Definition:Discriminant of Quadratic Equation|discriminant]] of this equation (that is $b^2 - 4 a c$) is: +:$D := \displaystyle 4 \paren {\sum {r_i s_i} }^2 - 4 \sum {r_i^2} \sum {s_i^2}$ +{{AimForCont}} $D$ is [[Definition:Strictly Positive Real Number|(strictly) positive]]. +Then $\map f \lambda = 0$ has two [[Definition:Distinct Elements|distinct]] [[Definition:Real Number|real]] [[Definition:Root of Polynomial|roots]], $\lambda_1 < \lambda_2$, say. +From [[Sign of Quadratic Function Between Roots]], it follows that $f$ is [[Definition:Strictly Negative Real Number|(strictly) negative]] somewhere between $\lambda_1$ and $\lambda_2$. +But we have: +:$\forall \lambda \in \R: \map f \lambda \ge 0$ +From this [[Definition:Contradiction|contradiction]] it follows that: +:$D \le 0$ +which is the same thing as saying: +:$\displaystyle \sum {r_i^2} \sum {s_i^2} \ge \paren {\sum {r_i s_i} }^2$ +{{qed}} +\end{proof} + +\begin{proof} +From the [[Cauchy-Schwarz Inequality/Complex Numbers|Complex Number form of the Cauchy-Schwarz Inequality]], we have: +:$\displaystyle \sum \left|{w_i}\right|^2 \left|{z_i}\right|^2 \ge \left|{\sum w_i z_i}\right|^2$ +where all of $w_i, z_i \in \C$. +As elements of $\R$ are also elements of $\C$, it follows that: +:$\displaystyle \sum \left|{r_i}\right|^2 \left|{s_i}\right|^2 \ge \left|{\sum r_i s_i}\right|^2$ +where all of $r_i, s_i \in \R$. +But from the [[Definition:Modulus of Complex Number|definition of modulus]], it follows that: +:$\displaystyle \forall r_i \in \R: \left|{r_i}\right|^2 = r_i^2$ +Thus: +:$\displaystyle \sum {r_i^2} \sum {s_i^2} \ge \left({\sum {r_i s_i}}\right)^2$ +where all of $r_i, s_i \in \R$. +{{qed}} +\end{proof}<|endoftext|> +\section{Cancellable Finite Semigroup is Group} +Tags: Semigroups, Finite Groups, Cancellability + +\begin{theorem} +Let $\struct {S, \circ}$ be a [[Definition:Non-Empty Set|non-empty]] [[Definition:Finite Semigroup|finite semigroup]] in which all [[Definition:Element|elements]] are [[Definition:Cancellable Element|cancellable]]. +Then $\struct {S, \circ}$ is a [[Definition:Group|group]]. +\end{theorem} + +\begin{proof} +As $\struct {S, \circ}$ is a [[Definition:Semigroup|semigroup]], it is already [[Definition:Closed Algebraic Structure|closed]] and [[Definition:Associative|associative]]. +It remains to be shown that it has an [[Definition:Identity Element|identity]] and that every [[Definition:Element|element]] of $S$ has an [[Definition:Inverse Element|inverse]] in $S$. +First we show that $\struct {S, \circ}$ has an [[Definition:Identity Element|identity]]. +Choose $a \in S$. +Let the [[Definition:Mapping|mapping]] $\lambda_a: S \to S$ be the [[Definition:Left Regular Representation|left regular representation]] of $\struct {S, \circ}$ with respect to $a$. +[[Definition:By Hypothesis|By hypothesis]]: +:all [[Definition:Element|elements]] of $S$ are [[Definition:Cancellable Element|cancellable]] +:$S$ is [[Definition:Finite Semigroup|finite]]. +By [[Regular Representation wrt Cancellable Element on Finite Semigroup is Bijection]], $\lambda_a$ is a [[Definition:Bijection|bijection]]. +Hence $a \circ e = a$ for some $e \in S$. +Let $x \in S$. +Then because of [[Definition:Cancellable Element|cancellability]]: +{{begin-eqn}} +{{eqn | l = a \circ e \circ x + | r = a \circ x + | c = +}} +{{eqn | ll= \leadsto + | l = e \circ x + | r = x + | c = +}} +{{eqn | ll= \leadsto + | l = x \circ e \circ x + | r = x \circ x + | c = +}} +{{eqn | ll= \leadsto + | l = x \circ e + | r = x + | c = +}} +{{end-eqn}} +Thus $e$ is the [[Definition:Identity Element|identity]]. +The existence of [[Definition:Inverse Element|inverses]] comes from the [[Definition:Surjection|surjectivity]] of $\lambda_a$. +As $\lambda_a$ is [[Definition:Surjection|surjective]]: +:$\exists y \in S: \map {\lambda_a} y = e$ +That is: +:$a \circ y = e$ +So we see that $y$ acts as a [[Definition:Right Inverse Element|right inverse]] for $a$. +This is the case for any $a \in S$: all of them have some [[Definition:Right Inverse Element|right inverse]]. +So, from [[Right Inverse for All is Left Inverse]], each of these [[Definition:Element|elements]] is also a [[Definition:Left Inverse Element|left inverse]], and therefore an [[Definition:Inverse Element|inverse]]. +Thus $S$ is [[Definition:Closed Algebraic Structure|closed]], [[Definition:Associative|associative]], has an [[Definition:Identity Element|identity]] and every element has an [[Definition:Inverse Element|inverse]]. +So, by definition, $\struct {S, \circ}$ is a [[Definition:Group|group]]. +{{qed}} +\end{proof}<|endoftext|> +\section{Finite Semigroup Equal Elements for Different Powers} +Tags: Semigroups + +\begin{theorem} +Let $\left({S, \circ}\right)$ be a [[Definition:Finite|finite]] [[Definition:Semigroup|semigroup]]. +Then: +: $\forall x \in S: \exists m, n \in \N: m \ne n: x^m = x^n$ +\end{theorem} + +\begin{proof} +List the [[Definition:Power of Element of Semigroup|positive powers]] $x, x^2, x^3, \ldots$ of any element $x$ of a finite semigroup $\left({S, \circ}\right)$. +Since all are elements of $S$, and the [[Definition:Semigroup|semigroup]] has a [[Definition:Finite|finite]] number of elements, it follows from the [[Pigeonhole Principle]] this list must contain repetitions. +So there must be at least one instance where $x^m = x^n$ for some $m, n \in \N$. +{{Qed}} +[[Category:Semigroups]] +1xgxx01rhazfn21f659gs2m7pt8z620 +\end{proof}<|endoftext|> +\section{Element has Idempotent Power in Finite Semigroup} +Tags: Semigroups, Idempotence + +\begin{theorem} +Let $\struct {S, \circ}$ be a [[Definition:Finite Semigroup|finite semigroup]]. +For every [[Definition:Element|element]] in $\struct {S, \circ}$, there is a [[Definition:Power of Element of Semigroup|power]] of that [[Definition:Element|element]] which is [[Definition:Idempotent Element|idempotent]]. +That is: +:$\forall x \in S: \exists i \in \N: x^i = x^i \circ x^i$ +\end{theorem} + +\begin{proof} +From [[Finite Semigroup Equal Elements for Different Powers]], we have: +:$\forall x \in S: \exists m, n \in \N: m \ne n: x^m = x^n$ +Let $m > n$. +Let $n = k, m = k + l$. +Then: +: $\forall x \in S: \exists k, l \in \N: x^k = x^{k + l}$ +Now we show that: +:$x^k = x^{k + l} \implies x^{k l} = x^{k l} \circ x^{k l}$ +That is, that $x^{k l}$ is [[Definition:Idempotent Element|idempotent]]. +First: +{{begin-eqn}} +{{eqn | l = x^k + | r = x^{k + l} + | c = +}} +{{eqn | ll= \leadsto + | l = x^k \circ x^l + | r = x^{k + l} \circ x^l + | c = both sides ${} \circ x^l$ +}} +{{eqn | ll= \leadsto + | l = x^{k + l} + | r = x^{k + 2 l} + | c = [[Index Laws for Semigroup/Sum of Indices|Index Laws for Semigroup: Sum of Indices]] +}} +{{eqn | ll= \leadsto + | l = x^k + | r = x^{k + 2 l} + | c = as $x^k = x^{k + l}$ +}} +{{end-eqn}} +From here we can easily prove by [[Principle of Mathematical Induction|induction]] that: +: $\forall n \in \N: x^k = x^{k + n l}$ +In particular: +: $x^k = x^{k + k l} = x^{k \paren {l + 1} }$ +There are two cases to consider: +:$(1): \quad$ If $l = 1$, then $x^k = x^{k \paren {l + 1} } = x^{2 k} = x^k \circ x^k$, and $x^{k l} = x^k$ is [[Definition:Idempotent Element|idempotent]]. +:$(2): \quad$ If $l > 1$, then: +{{begin-eqn}} +{{eqn | l = x^k + | r = x^{k + k l} + | c = +}} +{{eqn | r = x^{k \paren {l + 1} } + | c = +}} +{{eqn | ll= \leadsto + | l = x^k \circ x^{k \paren {l - 1} } + | r = x^{k \paren {l + 1} } \circ x^{k \paren {l - 1} } + | c = both sides ${} \circ x^{k \paren {l - 1} }$ +}} +{{eqn | ll= \leadsto + | l = x^{k + k l - k} + | r = x^{kl + k + k l - k} + | c = [[Index Laws for Semigroup/Sum of Indices|Index Laws for Semigroup: Sum of Indices]] +}} +{{eqn | ll= \leadsto + | l = x^{k l} + | r = x^{k l + k l} + | c = +}} +{{eqn | r = x^{k l} \circ x^{k l} + | c = +}} +{{end-eqn}} +and again, $x^{k l}$ is [[Definition:Idempotent Element|idempotent]]. +{{Qed}} +\end{proof}<|endoftext|> +\section{Powers of Ring Elements} +Tags: Ring Theory, Proofs by Induction + +\begin{theorem} +Let $\struct {R, +, \circ}$ be a [[Definition:Ring (Abstract Algebra)|ring]] whose [[Definition:Ring Zero|zero]] is $0_R$. +Let $n \cdot x$ be an [[Definition:Integral Multiple|integral multiple]] of $x$: +:$n \cdot x = \begin {cases} +0_R & : n = 0 \\ +x & : n = 1 \\ +\paren {n - 1} \cdot x + x & : n > 1 +\end {cases}$ +that is $n \cdot x = x + x + \cdots \paren n \cdots x$. +For $n < 0$ we use: +:$-n \cdot x = n \cdot \paren {-x}$ +Then: +:$\forall n \in \Z: \forall x \in R: \paren {n \cdot x} \circ x = n \cdot \paren {x \circ x} = x \circ \paren {n \cdot x}$ +\end{theorem} + +\begin{proof} +Proof by [[Principle of Mathematical Induction|induction]]: +For all $n \in \N$, let $\map P n$ be the [[Definition:Proposition|proposition]]: +:$\paren {n \cdot x} \circ x = n \cdot \paren {x \circ x} = x \circ \paren {n \cdot x}$ +First we verify $\map P 0$. +When $n = 0$, we have: +{{begin-eqn}} +{{eqn | l = \paren {0 \cdot x} \circ x + | r = 0_R \circ x + | c = +}} +{{eqn | r = 0_R + | c = +}} +{{eqn | r = 0 \cdot \paren {x \circ x} + | c = +}} +{{eqn | r = x \circ 0_R + | c = +}} +{{eqn | r = x \circ \paren {0 \cdot x} + | c = +}} +{{end-eqn}} +So $\map P 0$ holds. +=== Basis for the Induction === +Now we verify $\map P 1$: +{{begin-eqn}} +{{eqn | l = \paren {1 \cdot x} \circ x + | r = x \circ x + | c = +}} +{{eqn | r = 1 \cdot \paren {x \circ x} + | c = +}} +{{eqn | r = x \circ \paren {1 \cdot x} + | c = +}} +{{end-eqn}} +So $\map P 1$ holds. +This is our [[Definition:Basis for the Induction|basis for the induction]]. +=== Induction Hypothesis === +Now we need to show that, if $\map P k$ is true, where $k \ge 1$, then it logically follows that $\map P {k + 1}$ is true. +So this is our [[Definition:Induction Hypothesis|induction hypothesis]]: +:$\paren {k \cdot x} \circ x = k \cdot \paren {x \circ x} = x \circ \paren {k \cdot x}$ +Then we need to show: +:$\paren {\paren {k + 1} \cdot x} \circ x = \paren {k + 1} \cdot \paren {x \circ x} = x \circ \paren {\paren {k + 1} \cdot x}$ +=== Induction Step === +This is our [[Definition:Induction Step|induction step]]: +{{begin-eqn}} +{{eqn | l = \paren {\paren {k + 1} \cdot x} \circ x + | r = \paren {x + \paren {k \cdot x} } \circ x + | c = +}} +{{eqn | r = x \circ x + \paren {k \cdot x} \circ x + | c = {{Ring-axiom|D}} +}} +{{eqn | r = x \circ x + k \cdot \paren {x \circ x} + | c = [[Powers of Ring Elements#Induction Hypothesis|Induction Hypothesis]] +}} +{{eqn | r = \paren {k + 1} \cdot \paren {x \circ x} + | c = +}} +{{end-eqn}} +A similar construction shows that $\paren {k + 1} \cdot \paren {x \circ x} = x \circ \paren {\paren {k + 1} \cdot x}$. +So $\map P k \implies \map P {k + 1}$ and the result follows by the [[Principle of Mathematical Induction]]. +Therefore: +:$\forall n \in \N: \paren {n \cdot x} \circ x = n \cdot \paren {x \circ x} = x \circ \paren {n \cdot x}$ +{{qed|lemma}} +The result for $n < 0$ follows directly from [[Powers of Group Elements]]. +{{qed}} +[[Category:Ring Theory]] +[[Category:Proofs by Induction]] +8hv2385mggjxed224pcgpzdu9xdduss +\end{proof}<|endoftext|> +\section{Power of Conjugate equals Conjugate of Power} +Tags: Conjugacy + +\begin{theorem} +Let $\struct {G, \circ}$ be a [[Definition:Group|group]] whose [[Definition:Identity Element|identity]] is $e$. +Let $x, y \in G$ such that $\exists a \in G: x \circ a = a \circ y$. +That is, let $x$ and $y$ be [[Definition:Conjugate of Group Element|conjugate]]. +Then: +: $\forall n \in \Z: y^n = \paren {a^{-1} \circ x \circ a}^n = a^{-1} \circ x^n \circ a$ +It follows directly that: +: $\exists b \in G: \forall n \in \Z: y^n = b \circ x^n \circ b^{-1}$ +In particular: +: $y^{-1} = \paren {a^{-1} \circ x \circ a}^{-1} = a^{-1} \circ x^{-1} \circ a$ +\end{theorem} + +\begin{proof} +Proof by [[Principle of Mathematical Induction|induction]]: +For all $n \in \N$, let $\map P n$ be the [[Definition:Proposition|proposition]] $y^n = a^{-1} \circ x^n \circ a$. +$\map P 0$ is true, as this just says $e = a^{-1} \circ e \circ a$. +=== Basis for the Induction === +$\map P 1$ is the case $y = a^{-1} \circ x \circ a$, which is how [[Definition:Conjugate of Group Element|conjugacy]] is defined for $x$ and $y$. +This is our [[Principle of Mathematical Induction#Basis for the Induction|basis for the induction]]. +=== Induction Hypothesis === +Now we need to show that, if $\map P k$ is true, where $k \ge 1$, then it logically follows that $\map P {k + 1}$ is true. +So this is our [[Principle of Mathematical Induction#Induction Hypothesis|induction hypothesis]]: +:$y^k = a^{-1} \circ x^k \circ a$ +Then we need to show: +:$y^{k + 1} = a^{-1} \circ x^{k + 1} \circ a$ +=== Induction Step === +This is our [[Principle of Mathematical Induction#Induction Step|induction step]]: +{{begin-eqn}} +{{eqn | l = y^{k + 1} + | r = \paren {a^{-1} \circ x \circ a}^{k + 1} + | c = +}} +{{eqn | r = \paren {a^{-1} \circ x \circ a}^k \circ \paren {a^{-1} \circ x \circ a} + | c = +}} +{{eqn | r = \paren {a^{-1} \circ x^k \circ a} \circ \paren {a^{-1} \circ x \circ a} + | c = [[Power of Conjugate equals Conjugate of Power#Induction Hypothesis|Induction Hypothesis]] +}} +{{eqn | r = a^{-1} \circ x^k \circ x \circ a + | c = +}} +{{eqn | r = a^{-1} \circ x^{k + 1} \circ a + | c = +}} +{{end-eqn}} +So $\map P k \implies \map P {k + 1}$ and the result follows by the [[Principle of Mathematical Induction]]. +Therefore: +:$\forall n \in \N: y^n = a^{-1} \circ x^n \circ a$ +Now we need to show that if $\map P n$ holds, then $\map P {-n}$ holds. +That is: +:$y^{-n} = a^{-1} \circ x^{-n} \circ a$ +Let $n \in \N$. +Then: +{{begin-eqn}} +{{eqn | l = y^{-n} + | r = \paren {a^{-1} \circ x \circ a}^{-n} + | c = +}} +{{eqn | r = \paren {\paren {a^{-1} \circ x \circ a}^n}^{-1} + | c = [[Index Laws for Monoids/Negative Index|Index Laws for Monoids: Negative Index]] +}} +{{eqn | r = \paren {a^{-1} \circ x^n \circ a}^{-1} + | c = +}} +{{eqn | r = a^{-1} \circ \paren {x^n}^{-1} \circ \paren {a^{-1} }^{-1} + | c = [[Inverse of Group Product/General Result|Inverse of Group Product]] +}} +{{eqn | r = a^{-1} \circ x^{-n} \circ a + | c = +}} +{{end-eqn}} +Thus $\map P n$ has been shown to hold for all $n \in \Z$. +Hence the result. +{{qed}} +\end{proof}<|endoftext|> +\section{Product of Conjugates equals Conjugate of Products} +Tags: Conjugacy + +\begin{theorem} +Let $\struct {G, \circ}$ be a [[Definition:Group|group]]. +Then: +:$\forall a, x, y \in G: \paren {a \circ x \circ a^{-1} } \circ \paren {a \circ y \circ a^{-1} } = a \circ \paren {x \circ y} \circ a^{-1}$ +That is, the [[Definition:Product Element|product]] of [[Definition:Conjugate of Group Element|conjugates]] is equal to the [[Definition:Conjugate of Group Element|conjugate]] of the [[Definition:Product Element|product]]. +\end{theorem} + +\begin{proof} +Follows directly from the [[Definition:Group Axioms|group axioms]]. +{{Qed}} +\end{proof}<|endoftext|> +\section{Power of Product with Inverse} +Tags: Group Theory + +\begin{theorem} +Let $G$ be a [[Definition:Group|group]] whose [[Definition:Identity (Abstract Algebra)|identity]] is $e$. +Let $a, b \in G: a b = b a^{-1}$. +Then: +: $\forall n \in \Z: a^n b = b a^{-n}$ +\end{theorem} + +\begin{proof} +Proof by [[Principle of Mathematical Induction|induction]]: +For all $n \in \Z$, let $P \left({n}\right)$ be the [[Definition:Proposition|proposition]] $a^n b = b a^{-n}$. +$P(0)$ is trivially true, as $a^0 b = e b = b = b e = b a^{-0}$. +=== Basis for the Induction === +$P(1)$ is true, as this is the given relation between $a$ and $b$: +:$a b = b a^{-1}$ +This is our [[Principle of Mathematical Induction#Basis for the Induction|basis for the induction]]. +=== Induction Hypothesis === +Now we need to show that, if $P \left({k}\right)$ is true, where $k \ge 1$, then it logically follows that $P \left({k+1}\right)$ is true. +So this is our [[Principle of Mathematical Induction#Induction Hypothesis|induction hypothesis]]: +:$a^k b = b a^{-k}$ +Then we need to show: +:$a^{k+1} b = b a^{-\left({k+1}\right)}$ +=== Induction Step === +This is our [[Principle of Mathematical Induction#Induction Step|induction step]]: +{{begin-eqn}} +{{eqn | l=a^{k+1} b + | r=a \left({a^k b}\right) + | c= +}} +{{eqn | r=a \left({b a^{-k} }\right) + | c=[[Power of Product with Inverse#Induction Hypothesis|Induction Hypothesis]] +}} +{{eqn | r=\left({a b}\right) a^{-k} + | c= +}} +{{eqn | r=\left({b a^{-1} }\right) a^{-k} + | c=[[Power of Product with Inverse#Basis for the Induction|Basis for the Induction]] +}} +{{eqn | r=b \left({a^{-1} a^{-k} }\right) + | c= +}} +{{eqn | r=b a^{-\left({k+1}\right)} + | c= +}} +{{end-eqn}} +So $P \left({k}\right) \implies P \left({k+1}\right)$ and the result follows by the [[Principle of Mathematical Induction]]. +Therefore $\forall n \in \N: a^n b = b a^{-n}$. +Now we show that $P \left({-1}\right)$ holds, i.e. that $a^{-1} b = b a$. +{{begin-eqn}} +{{eqn | l=a b + | r=b a^{-1} + | c= +}} +{{eqn | ll=\iff + | l=a b a + | r=b + | c= +}} +{{eqn | ll=\iff + | l=b a + | r=a^{-1} b + | c= +}} +{{end-eqn}} +... thus showing that $P \left({-1}\right)$ holds. +The proof that $P \left({n}\right)$ holds for all $n \in \Z: n < 0$ then follows by [[Principle of Mathematical Induction|induction]], similarly to the proof for $n > 0$. +{{qed}} +[[Category:Group Theory]] +fghfsm379raiteyy6tp8lhpqp8i5qbl +\end{proof}<|endoftext|> +\section{Powers of Elements in Group Direct Product} +Tags: Group Theory, Direct Products + +\begin{theorem} +Let $\left({G, \circ_1}\right)$ and $\left({H, \circ_2}\right)$ be [[Definition:Group|group]] whose [[Definition:Identity (Abstract Algebra)|identities]] are $e_G$ and $e_H$. +Let $\left({G \times H, \circ}\right)$ be the group direct product (either [[Definition:External Direct Product|external]] or [[Definition:Internal Group Direct Product|internal]]) of $G$ and $H$. +Then: +: $\forall n \in \Z: \forall g \in G, h \in H: \left({g, h}\right)^n = \left({g^n, h^n}\right)$ +\end{theorem} + +\begin{proof} +Proof by [[Principle of Mathematical Induction|induction]]: +For all $n \in \N$, let $P \left({n}\right)$ be the [[Definition:Proposition|proposition]] $\forall g \in G, h \in H: \left({g, h}\right)^n = \left({g^n, h^n}\right)$. +=== Basis for the Induction === +$P(0)$ is true, as this says: +: $\left({g, h}\right)^0 = \left({e_G, e_H}\right)$ +$P(1)$ is true, as this says: +: $\left({g, h}\right) = \left({g, h}\right)$ +This is our [[Principle of Mathematical Induction#Basis for the Induction|basis for the induction]]. +=== Induction Hypothesis === +Now we need to show that, if $P \left({k}\right)$ is true, where $k \ge 1$, then it logically follows that $P \left({k+1}\right)$ is true. +So this is our [[Principle of Mathematical Induction#Induction Hypothesis|induction hypothesis]]: +:$\left({g, h}\right)^k = \left({g^k, h^k}\right)$ +Then we need to show: +:$\left({g, h}\right)^{k+1} = \left({g^{k+1}, h^{k+1}}\right)$ +=== Induction Step === +This is our [[Principle of Mathematical Induction#Induction Step|induction step]]: +{{begin-eqn}} +{{eqn | l=\left({g, h}\right)^{k+1} + | r=\left({g, h}\right)^k \circ \left({g, h}\right) + | c= +}} +{{eqn | r=\left({g^k, h^k}\right) \circ \left({g, h}\right) + | c=[[#Induction Hypothesis|Induction Hypothesis]] +}} +{{eqn | r=\left({g^k \circ_1 g, h^k \circ_2 h}\right) + | c=by definition of [[Definition:External Direct Product|external]] or [[Definition:Internal Group Direct Product|internal]] group product +}} +{{eqn | r=\left({g^{k+1}, h^{k+1} }\right) + | c= +}} +{{end-eqn}} +So $P \left({k}\right) \implies P \left({k+1}\right)$ and the result follows by the [[Principle of Mathematical Induction]]. +Therefore: +:$\forall n \in \N: \forall g \in G, h \in H: \left({g, h}\right)^n = \left({g^n, h^n}\right)$ +So we have shown the result holds true for all $n \ge 0$. +The result for $n < 0$ follows directly from [[Powers of Group Elements/Negative Index|Powers of Group Elements for negative indices]]. +{{qed}} +[[Category:Group Theory]] +[[Category:Direct Products]] +hsrdqb9vr937yff7dzku2el3m4ojkc6 +\end{proof}<|endoftext|> +\section{Powers of Commutative Elements in Semigroups} +Tags: Semigroups, Commutativity + +\begin{theorem} +Let $\left ({S, \circ}\right)$ be a [[Definition:Semigroup|semigroup]]. +Let $a, b \in S$ both be [[Definition:Cancellable Element|cancellable elements]] of $S$. +Then the following results hold: +\end{theorem}<|endoftext|> +\section{Powers of Commutative Elements in Monoids} +Tags: Monoids + +\begin{theorem} +These results are an extension of the results in [[Powers of Commutative Elements in Semigroups]] in which the [[Definition:Domain of Mapping|domain]] of the indices is extended to include all [[Definition:Integer|integers]]. +Let $\left ({S, \circ}\right)$ be a [[Definition:Monoid|monoid]] whose [[Definition:Identity Element|identity]] is $e_S$. +Let $a, b \in S$ be [[Definition:Invertible Element|invertible elements]] for $\circ$ that also [[Definition:Commute|commute]]. +Then the following results hold. +\end{theorem}<|endoftext|> +\section{Powers of Commutative Elements in Groups} +Tags: Group Theory, Commutativity + +\begin{theorem} +Let $\struct {G, \circ}$ be a [[Definition:Group|group]]. +Let $a, b \in G$ such that $a$ and $b$ [[Definition:Commuting Elements|commute]]. +Then the following results hold: +\end{theorem}<|endoftext|> +\section{General Morphism Property for Semigroups} +Tags: Semigroups, Homomorphisms + +\begin{theorem} +Let $\struct {S, \circ}$ and $\struct {T, *}$ be [[Definition:Semigroup|semigroups]]. +Let $\phi: S \to T$ be a [[Definition:Semigroup Homomorphism|homomorphism]]. +Then: +:$\forall s_k \in S: \map \phi {s_1 \circ s_2 \circ \cdots \circ s_n} = \map \phi {s_1} * \map \phi {s_2} * \cdots * \map \phi {s_n}$ +Hence it follows that: +:$\forall n \in \N_{>0}: \forall s \in S: \map \phi {s^n} = \paren {\map \phi s}^n$ +\end{theorem} + +\begin{proof} +$\forall s_k \in S: \map \phi {s_1 \circ s_2 \circ \cdots \circ s_n} = \map \phi {s_1} * \map \phi {s_2} * \cdots * \map \phi {s_n}$ can be proved by [[Principle of Mathematical Induction|induction]]. +For all $n \in \N_{>0}$, let $\map P n$ be the [[Definition:Proposition|proposition]]: +:$\map \phi {s_1 \circ s_2 \circ \cdots \circ s_n} = \map \phi {s_1} * \map \phi {s_2} * \cdots * \map \phi {s_n}$ +$P(1)$ is true, as this just says: +:$\map \phi {s_1} = \map \phi {s_1}$. +=== Basis for the Induction === +$P(2)$ is the case: +:$\map \phi {s_1 \circ s_2} = \map \phi {s_1} * \map \phi {s_2}$ +This follows from the fact that $\phi$ is a [[Definition:Semigroup Homomorphism|homomorphism]]. +This is our [[Principle of Mathematical Induction#Basis for the Induction|basis for the induction]]. +=== Induction Hypothesis === +Now we need to show that, if $\map P k$ is true, where $k \ge 2$, then it logically follows that $\map P {k + 1}$ is true. +So this is our [[Principle of Mathematical Induction#Induction Hypothesis|induction hypothesis]]: +:$\map \phi {s_1 \circ s_2 \circ \cdots \circ s_k} = \map \phi {s_1} * \map \phi {s_2} * \cdots * \map \phi {s_k}$ +Then we need to show: +:$\map \phi {s_1 \circ s_2 \circ \cdots \circ s_k \circ s_{k + 1} } = \map \phi {s_1} * \map \phi {s_2} * \cdots * \map \phi {s_k} * \map \phi {s_{k + 1} }$ +=== Induction Step === +This is our [[Principle of Mathematical Induction#Induction Step|induction step]]: +{{begin-eqn}} +{{eqn | l = \map \phi {s_1 \circ s_2 \circ \cdots \circ s_k \circ s_{k + 1} } + | r = \map \phi {s_1 \circ s_2 \circ \cdots \circ s_k} * \map \phi {s_{k + 1} } + | c = [[General Morphism Property for Semigroups#Basis for the Induction|Basis for the Induction]] +}} +{{eqn | r = \map \phi {s_1} * \map \phi {s_2} * \cdots * \map \phi {s_k} * \map \phi {s_{k + 1} } + | c = [[General Morphism Property for Semigroups#Induction Hypothesis|Induction Hypothesis]] +}} +{{end-eqn}} +So $\map P k \implies \map P {k + 1}$ and the result follows by the [[Principle of Mathematical Induction]]. +Therefore: +:$\map \phi {s_1 \circ s_2 \circ \cdots \circ s_n} = \map \phi {s_1} * \map \phi {s_2} * \cdots * \map \phi {s_n}$ +The result for $n \in \N_{>0}$ follows directly from the above, by replacing each occurrence of $s_k$ with $s$. +{{Qed}} +\end{proof}<|endoftext|> +\section{Homomorphism of Power of Group Element} +Tags: Group Homomorphisms + +\begin{theorem} +Let $\struct {G, \circ}$ and $\struct {H, \ast}$ be [[Definition:Group|groups]]. +Let $\phi: S \to T$ be a [[Definition:Group Homomorphism|group homomorphism]]. +Then: +: $\forall n \in \Z: \forall g \in G: \map \phi {g^n} = \paren {\map \phi g}^n$ +\end{theorem} + +\begin{proof} +The result for $n \in \N_{>0}$ follows directly from [[General Morphism Property for Semigroups]]. +For $n = 0$, we use [[Homomorphism with Cancellable Codomain Preserves Identity]]. +For $n < 0$, we use [[Homomorphism with Identity Preserves Inverses]], along with [[Index Laws for Monoids/Negative Index|Index Laws for Monoids: Negative Index]]. +{{qed}} +\end{proof}<|endoftext|> +\section{Existence of Unique Subsemigroup Generated by Subset} +Tags: Subsemigroups + +\begin{theorem} +Let $\struct {S, \circ}$ be a [[Definition:Semigroup|semigroup]]. +Let $\O \subset X \subseteq S$. +Let $\struct {T, \circ}$ be the [[Definition:Generator of Semigroup|subsemigroup generated by $X$]]. +Then $T = \gen X$ exists and is [[Definition:Unique|unique]]. +\end{theorem} + +\begin{proof} +=== Existence === +First, we prove that such a [[Definition:Subsemigroup|subsemigroup]] exists. +Let $\mathbb S$ be the [[Definition:Set|set]] of all [[Definition:Subsemigroup|subsemigroups]] of $S$ which contain $X$. +$\mathbb S \ne \O$ because $S$ is itself a [[Definition:Subsemigroup|subsemigroup]] of $S$ (trivial), and thus $S \in \mathbb S$. +Let $T$ be the [[Definition:Set Intersection|intersection]] of all the [[Definition:Element|elements]] of $\mathbb S$. +By [[Intersection of Subsemigroups]], $T$ is the largest element of $\mathbb S$ contained in each element of $\mathbb S$. +Thus $T$ is a [[Definition:Subsemigroup|subsemigroup]] of $S$. +Since $\forall x \in \mathbb S: X \subseteq x$, we see that $X \subseteq T$, so $T \in \mathbb S$. +=== Smallest === +Now to show that $T$ is the smallest such [[Definition:Subsemigroup|subsemigroup]]. +If any $K \le S: X \subseteq K$, then $K \in \mathbb S$ and therefore $T \subseteq K$. +So $T$ is the smallest [[Definition:Subsemigroup|subsemigroup]] of $S$ containing $X$. +=== Uniqueness === +Now we show that $T$ is unique. +Suppose $\exists T_1, T_2 \in \mathbb S$ such that $T_1$ and $T_2$ were two such smallest [[Definition:Subsemigroup|subsemigroups]] containing $X$. +Then, by the definition of '''smallest''', each would be equal in size. +{{WLOG}}, suppose that $T_1$ is not a [[Definition:Subset|subset]] of $T_2$. +Then their [[Definition:Set Intersection|intersection]] (by definition containing $X$) would be a [[Definition:Subsemigroup|subsemigroup]] smaller than $T_1$. +Hence $T_1$ would not be the smallest. +Therefore each must be the [[Definition:Subset|subset]] of each other. +By definition of [[Definition:Set Equality/Definition 2|set equality]], they must be the same [[Definition:Set|set]]. +So the smallest subsemigroup, whose existence we have proved above, is [[Definition:Unique|unique]]. +{{Qed}} +\end{proof}<|endoftext|> +\section{Finite Subgroup Test} +Tags: Subgroups, Finite Subgroup Test + +\begin{theorem} +Let $\struct {G, \circ}$ be a [[Definition:Group|group]]. +Let $H$ be a [[Definition:Non-Empty Set|non-empty]] [[Definition:Finite Set|finite]] [[Definition:Subset|subset]] of $G$. +Then: +:$H$ is a [[Definition:Subgroup|subgroup]] of $G$ +{{iff}}: +:$\forall a, b \in H: a \circ b \in H$ +That is, a [[Definition:Non-Empty Set|non-empty]] [[Definition:Finite Set|finite]] [[Definition:Subset|subset]] of $G$ is a [[Definition:Subgroup|subgroup]] {{iff}} it is [[Definition:Closed Algebraic Structure|closed]]. +\end{theorem} + +\begin{proof} +Let $H$ be a [[Definition:Finite Set|finite]] [[Definition:Subset|subset]] of $G$ such that $a, b \in H \implies a \circ b \in H$. +From the [[Two-Step Subgroup Test]], it follows that we only need to show that $a \in H \implies a^{-1} \in H$. +So, let $a \in H$. +First it is straightforward to show by [[Principle of Mathematical Induction|induction]] that $\set {x \in G: x = a^n: n \in \N} \subseteq H$. +That is, $a \in H \implies \forall n \in \N: a^n \in H$. +Now, since $H$ is [[Definition:Finite Set|finite]], we have that [[Order of Element Divides Order of Finite Group|the order of $a$ is finite]]. +Let the [[Definition:Order of Group Element|order of $a$]] be $m$. +From [[Inverse Element is Power of Order Less 1]] we have that $a^{m-1} = a^{-1}$. +As $a^{m-1} \in H$ (from above) the result follows. +{{qed}} +\end{proof} + +\begin{proof} +=== Sufficient Condition === +Let $H$ be a [[Definition:Subgroup|subgroup]] of $G$. +Then: +:$\forall a, b \in H: a \circ b \in H$ +by definition of [[Definition:Subgroup|subgroup]]. +{{qed|lemma}} +=== Necessary Condition === +Let $H$ be a [[Definition:Non-Empty Set|non-empty]] [[Definition:Finite Set|finite]] [[Definition:Subset|subset]] of $G$ such that: +:$\forall a, b \in H: a \circ b \in H$ +Let $x \in H$. +We have [[Definition:By Hypothesis|by hypothesis]] that $H$ is [[Definition:Closed Algebraic Structure|closed]] under $\circ$. +Thus all [[Definition:Element|elements]] of $\set {x, x^2, x^3, \ldots}$ are in $H$. +But $H$ is [[Definition:Finite Set|finite]]. +Therefore it must be the case that: +:$\exists r, s \in \N: x^r = x^s$ +for $r < s$. +So we can write: +{{begin-eqn}} +{{eqn | l = x^r + | r = x^s + | c = +}} +{{eqn | ll= \leadsto + | l = x^r \circ e + | r = x^r \circ x^{s - r} + | c = {{Defof|Identity Element}}, [[Powers of Group Elements/Sum of Indices|Powers of Group Elements: Sum of Indices]] +}} +{{eqn | n = 1 + | ll= \leadsto + | l = e + | r = x^{s - r} + | c = [[Cancellation Laws]] +}} +{{eqn | ll= \leadsto + | l = e + | o = \in + | r = H + | c = as $H$ is [[Definition:Closed Algebraic Structure|closed]] under $\circ$ +}} +{{end-eqn}} +Then we have: +{{begin-eqn}} +{{eqn | l = e + | r = x^{s - r} + | c = which is $(1)$ +}} +{{eqn | ll= \leadsto + | l = e + | r = x \circ x^{s - r - 1} + | c = as $H$ is [[Definition:Closed Algebraic Structure|closed]] under $\circ$ +}} +{{eqn | ll= \leadsto + | l = x^{-1} \circ e + | r = x^{-1} \circ x \circ x^{s - r - 1} + | c = +}} +{{eqn | n = 2 + | ll= \leadsto + | l = x^{-1} + | r = x^{s - r - 1} + | c = {{Defof|Inverse Element}}, {{Defof|Identity Element}} +}} +{{end-eqn}} +But we have that: +{{begin-eqn}} +{{eqn | l = r + | o = < + | r = s + | c = [[Definition:By Hypothesis|by hypothesis]] +}} +{{eqn | ll= \leadsto + | l = s - r + | o = < + | r = 0 +}} +{{eqn | ll= \leadsto + | l = s - r - 1 + | o = \le + | r = 0 +}} +{{eqn | ll= \leadsto + | l = x^{s - r - 1} + | o = \in + | r = \set {e, x, x^2, x^3, \ldots} + | c = +}} +{{eqn | ll= \leadsto + | l = x^{s - r - 1} + | o = \in + | r = H + | c = as all [[Definition:Element|elements]] of $\set {e, x, x^2, x^3, \ldots}$ are in $H$ +}} +{{end-eqn}} +So from $(2)$: +:$x^{-1} = x^{s - r - 1}$ +it follows that: +: $x^{-1} \in H$ +and from the [[Two-Step Subgroup Test]] it follows that $H$ is a [[Definition:Subgroup|subgroup]] of $G$. +{{qed}} +\end{proof}<|endoftext|> +\section{Powers of Element form Subgroup} +Tags: Subgroups + +\begin{theorem} +Let $\struct {G, \circ}$ be a [[Definition:Group|group]]. +Then: +:$\forall a \in G: H = \set {a^n: n \in \Z} \le G$ +That is, the [[Definition:Subset|subset]] of $G$ comprising all [[Definition:Element|elements]] possible as powers of $a \in G$ is a [[Definition:Subgroup|subgroup]] of $G$. +\end{theorem} + +\begin{proof} +Clearly $a \in H$, so $H \ne \O$. +Let $x, y \in H$. +{{begin-eqn}} +{{eqn | o = + | r = x, y \in H + | c = +}} +{{eqn | o = \leadsto + | r = \exists m, n \in \Z: x = a^m, y = a^n + | c = +}} +{{eqn | o = \leadsto + | r = x^{-1} y = \paren {a^m}^{-1} a^n + | c = +}} +{{eqn | o = \leadsto + | r = x^{-1} y = a^{-m} a^n = a^{n-m} + | c = +}} +{{eqn | o = \leadsto + | r = x^{-1} y \in H + | c = +}} +{{end-eqn}} +Thus by the [[One-Step Subgroup Test]]: +: $H \le G$ +{{qed}} +\end{proof}<|endoftext|> +\section{Existence of Unique Subgroup Generated by Subset} +Tags: Group Theory + +\begin{theorem} +Let $\struct {G, \circ}$ be a [[Definition:Group|group]]. +Let $\O \subset S \subseteq G$. +Let $\struct {H, \circ}$ be the [[Definition:Generator of Subgroup|subgroup generated by $S$]]. +Then $H = \gen S$ exists and is [[Definition:Unique|unique]]. +Also, $\struct {H, \circ}$ is the [[Definition:Set Intersection|intersection]] of all of the [[Definition:Subgroup|subgroups]] of $G$ which contain the set $S$: +:$\displaystyle \gen S = \bigcap_i {H_i}: S \subseteq H_i \le G$ +\end{theorem} + +\begin{proof} +=== Existence === +First, we prove that such a [[Definition:Subgroup|subgroup]] exists. +Let $\mathbb S$ be the [[Definition:Set|set]] of all [[Definition:Subgroup|subgroups]] of $G$ which contain $S$. +$\mathbb S \ne \O$ because [[Group is Subgroup of Itself|$G$ is itself a subgroup]] of $G$, and thus $G \in \mathbb S$. +Let $H$ be the [[Definition:Set Intersection|intersection]] of all the [[Definition:Element|elements]] of $\mathbb S$. +By [[Intersection of Subgroups is Subgroup]], $H$ is the largest element of $\mathbb S$ contained in each element of $\mathbb S$. +Thus $H$ is a [[Definition:Subgroup|subgroup]] of $G$. +Since $\forall x \in \mathbb S: S \subseteq x$, we see that $S \subseteq H$, so $H \in \mathbb S$. +=== Smallest === +Now to show that $H$ is the smallest such [[Definition:Subgroup|subgroup]]. +If any $K \le G: S \subseteq K$, then $K \in \mathbb S$ and therefore $H \subseteq K$. +So $H$ is the smallest [[Definition:Subgroup|subgroup]] of $G$ containing $S$. +=== Uniqueness === +Now we show that $H$ is unique. +Suppose $\exists H_1, H_2 \in \mathbb S$ such that $H_1$ and $H_2$ were two such smallest [[Definition:Subgroup|subgroups]] containing $S$. +Then, by the definition of "smallest", each would be equal in size. +If one is not a subset of the other, then their intersection (by definition containing $S$) would be a smaller subgroup and hence neither $H_1$ nor $H_2$ would be the smallest. +Hence one must be a [[Definition:Subset|subset]] of the other. +By definition of [[Definition:Set Equality/Definition 2|set equality]], that means they must be the same set. +So the smallest subgroup, whose existence we have proved above, is unique. +{{Qed}} +\end{proof}<|endoftext|> +\section{Homomorphism of Generated Group} +Tags: Group Homomorphisms + +\begin{theorem} +Let $\left({G, \circ}\right)$ and $\left({H, \circ}\right)$ be [[Definition:Group|groups]]. +Let $\phi: G \to H$ and $\psi: G \to H$ be [[Definition:Group Homomorphism|homomorphisms]]. +Let $\left \langle {S} \right \rangle = G$ be the [[Definition:Generator of Group|group generated by $S$]]. +Let $\forall x \in S: \phi \left({x}\right) = \psi \left({x}\right)$ +Then $\phi = \psi$. +\end{theorem} + +\begin{proof} +Let $H = \left\{{x \in G: \phi \left({x}\right) = \psi \left({x}\right)}\right\}$. +From [[Elements of Group with Equal Images under Homomorphisms form Subgroup]], $H$ is a [[Definition:Subgroup|subgroup]] of $G$. +But from the definition of the [[Definition:Generator of Group|group generated by $S$]], the smallest subgroup that contains $S$ is $G$ itself. +Thus $G = \left\{{x \in G: \phi \left({x}\right) = \psi \left({x}\right)}\right\}$ and so $\phi = \psi$. +{{Qed}} +\end{proof}<|endoftext|> +\section{Set of Words Generates Group} +Tags: Group Theory + +\begin{theorem} +Let $S \subseteq G$ where $G$ is a [[Definition:Group|group]]. +Let $\hat S$ be defined as $S \cup S^{-1}$, where $S^{-1}$ is the [[Definition:Inverse of Subset of Group|set of all the inverses]] of all the elements of $S$. +Then $\gen S = \map W {\hat S}$, where $\map W {\hat S}$ is the [[Definition:Word (Abstract Algebra)|set of words]] of $\hat S$. +\end{theorem} + +\begin{proof} +Let $H = \gen S$ where $S \subseteq G$. +$H$ must certainly include $\hat S$, because any [[Definition:Group|group]] containing $s \in S$ must also contain $s^{-1}$. +Thus $\hat S \subseteq H$. +By the [[Definition:Group Axioms|closure axiom]], $H$ must also contain all [[Definition:Product Element|products]] of a [[Definition:Finite Set|finite number]] of [[Definition:Element|elements]] of $\hat S$. +Thus $\map W {\hat S} \subseteq H$. +Now we prove that $\map W {\hat S} \le G$. +By the [[Two-Step Subgroup Test]]: +Let $x, y \in \map W {\hat S}$. + +As $x$ and $y$ are both [[Definition:Product Element|products]] of a [[Definition:Finite Set|finite number]] of [[Definition:Element|elements]] of $\hat S$, it follows that so is their [[Definition:Product Element|product]] $x y$. +Thus $x y \in \map W {\hat S}$ and {{GroupAxiom|0}} is satisfied. +Let $x = s_1 s_2 \ldots s_n \in \map W {\hat S}$. +Then $x^{-1} = s_n^{-1} \ldots s_2^{-1} s_1^{-1} \in \map W {\hat S}$. +Thus the conditions of the [[Two-Step Subgroup Test]] are fulfilled, and $\map W {\hat S} \le G$. +Thus $\map W {\hat S}$ is the [[Definition:Subgroup|subgroup]] of $G$ [[Definition:Generator of Group|generated by $S$]]. +{{qed}} +\end{proof}<|endoftext|> +\section{Subset Product is Subset of Generator} +Tags: Group Theory, Subset Products + +\begin{theorem} +Let $\struct {G, \circ}$ be a [[Definition:Group|group]]. +Let $X, Y \subseteq \struct {G, \circ}$. +Then $X \circ Y \subseteq \gen {X, Y}$ where: +:$X \circ Y$ is the [[Definition:Subset Product|Subset Product]] of $X$ and $Y$ in $G$. +:$\gen {X, Y}$ is the [[Definition:Generator of Subgroup|subgroup of $G$ generated by $X$ and $Y$]]. +\end{theorem} + +\begin{proof} +It is clear from [[Set of Words Generates Group]] that $\map W {\hat X \cup \hat Y} = \gen {X, Y}$. +It is equally clear that $X \circ Y \subseteq \map W {\hat X \cup \hat Y}$. +{{qed}} +[[Category:Group Theory]] +[[Category:Subset Products]] +3cekksmrfte9xtmhqnc5h4td9k4wcc6 +\end{proof}<|endoftext|> +\section{Order of Subset Product with Singleton} +Tags: Subset Products, Singletons, Order of Subset Product with Singleton + +\begin{theorem} +Let $\struct {G, \circ}$ be a [[Definition:Group|group]]. +Let $X, Y \subseteq \struct {G, \circ}$ such that $X$ is a [[Definition:Singleton|singleton]]: +:$X = \set x$ +Then: +:$\order {X \circ Y} = \order Y = \order {Y \circ X}$ +where $\order S$ is defined as the [[Definition:Order of Structure|order of $S$]]. +\end{theorem} + +\begin{proof} +From [[Regular Representations of Subset Product]], we have that the [[Definition:Left Regular Representation|left regular representation]] of $\struct {S, \circ}$ with respect to $a$ is: +:$\lambda_x \sqbrk S = \set x \circ S = x \circ S$ +The result then follows directly from [[Regular Representation of Invertible Element is Permutation]]. +{{qed}} +\end{proof}<|endoftext|> +\section{Product of Subset with Intersection} +Tags: Subset Products, Set Intersection, Product of Subset with Intersection + +\begin{theorem} +Let $\struct {G, \circ}$ be an [[Definition:Algebraic Structure|algebraic structure]]. +Let $X, Y, Z \subseteq G$. +Then: +:$X \circ \paren {Y \cap Z} \subseteq \paren {X \circ Y} \cap \paren {X \circ Z}$ +:$\paren {Y \cap Z} \circ X \subseteq \paren {Y \circ X} \cap \paren {Z \circ X}$ +where $X \circ Y$ denotes the [[Definition:Subset Product|subset product]] of $X$ and $Y$. +\end{theorem} + +\begin{proof} +Consider the [[Definition:Relation|relation]] $\RR \subseteq G \times G$ defined as: +:$\forall g, h \in G: \tuple {g, h} \in \RR \iff \exists g \in X$ +Then: +:$\forall S \subseteq G: X \circ S = \RR \sqbrk S$ +Then: +{{begin-eqn}} +{{eqn | l = X \circ \paren {Y \cap Z} + | r = \RR \sqbrk {Y \cap Z} + | c = +}} +{{eqn | o = \subseteq + | r = \RR \sqbrk Y \cap \RR \sqbrk Z + | c = [[Image of Intersection under Relation]] +}} +{{eqn | r = \paren {X \circ Y} \cap \paren {X \circ Z} + | c = +}} +{{end-eqn}} +Next, consider the [[Definition:Relation|relation]] $\RR \subseteq G \times G$ defined as: +:$\forall g, h \in G: \tuple {g, h} \in \RR \iff \exists h \in X$ +Then: +:$\forall S \subseteq G: S \circ X = \RR \sqbrk S$ +Then: +{{begin-eqn}} +{{eqn | l = \paren {Y \cap Z} \circ X + | r = \RR \sqbrk {Y \cap Z} + | c = +}} +{{eqn | o = \subseteq + | r = \RR \sqbrk Y \cap \RR \sqbrk Z + | c = [[Image of Intersection under Relation]] +}} +{{eqn | r = \paren {Y \circ X} \cap \paren {Z \circ X} + | c = +}} +{{end-eqn}} +{{qed}} +\end{proof} + +\begin{proof} +Let $x \in X, t \in Y \cap Z$. +By the definition of [[Definition:Set Intersection|intersection]], $t \in Y$ and $t \in Z$. +Consider $X \circ \paren {Y \cap Z}$. +We have $x \circ t \in X \circ \paren {Y \cap Z}$ by definition of [[Definition:Subset Product|subset product]]. +As $t \in Y$ and $t \in Z$, we also have $x \circ t \in X \circ Y$ and $x \circ t \in X \circ Z$. +The result follows. +Similarly, consider $\paren {Y \cap Z} \circ X$. +Then we have $t \circ x \in \paren {Y \cap Z} \circ X$ by definition of [[Definition:Subset Product|subset product]]. +As $t \in Y$ and $t \in Z$, we also have $t \circ x \in Y \circ X$ and $t \circ x \in Z \circ X$. +Again, the result follows. +{{Qed}} +\end{proof}<|endoftext|> +\section{Order of Subgroup Product} +Tags: Subgroups, Subset Products, Order of Subgroup Product + +\begin{theorem} +Let $G$ be a [[Definition:Group|group]]. +Let $H$ and $K$ be [[Definition:Subgroup|subgroups]] of $G$. +Then: +:$\order {H K} = \dfrac {\order H \order K} {\order {H \cap K} }$ +where: +:$H K$ denotes [[Definition:Subset Product|subset product]] +:$\order H$ denotes the [[Definition:Order of Structure|order of $H$]]. +\end{theorem} + +\begin{proof} +From [[Intersection of Subgroups is Subgroup]], we have that $H \cap K \le H$. +Let the number of [[Definition:Left Coset|left cosets]] of $H \cap K$ in $H$ be $r$. +Then the [[Definition:Left Coset Space|left coset space]] of $H \cap K$ in $H$ is: +: $\set {x_1 \paren {H \cap K}, x_2 \paren {H \cap K}, \ldots, x_r \paren {H \cap K} }$ +So each element of $H$ is in $x_i \paren {H \cap K}$ for some $1 \le i \le r$. +Also, if $i \ne j$, we have: +:$x_j^{-1} x_i \notin H \cap K$ +Let $h k \in H K$. +We can write $h = x_i g$ for some $1 \le i \le r$ and some $g \in K$. +Thus: +:$h k = x_i \paren {g k}$ +Since $g, k \in K$, this shows $h k \in x_i K$. +{{AimForCont}} the [[Definition:Left Coset|left cosets]] $x_i K$ are not all [[Definition:Disjoint Sets|disjoint]]. +Then by [[Left Coset Space forms Partition]]: +:$x_i K = x_j K$ for some $i, j$. +So by [[Left Congruence Class Modulo Subgroup is Left Coset]]: +:$x_j^{-1} x_i \in K$ +Since $x_i, x_j \in H$, we have: +:$x_j^{-1} x_i \in H \cap K$ +which [[Definition:Contradiction|contradicts]] the definition. +Therefore the [[Definition:Left Coset|left cosets]] $x_i K$ are [[Definition:Disjoint Sets|disjoint]] for $1 \le i \le r$. +This leads us to: +:$\dfrac {\order H} {\order {H \cap K} } = \dfrac {\order {H K} } {\order K} = r$ +whence the result. +{{qed}} +\end{proof} + +\begin{proof} +=== [[Order of Subgroup Product/Lemma|Lemma]] === +{{:Order of Subgroup Product/Lemma}}{{qed|lemma}} +We have that $H K$ is the [[Definition:Set Union|union]] of all [[Definition:Left Coset|left cosets]] $h K$ with $h \in H$: +:$\displaystyle H K = \bigcup_{h \mathop \in H} h K$ +From [[Left Coset Space forms Partition]], unequal $h K$ are [[Definition:Disjoint Sets|disjoint]]. +From [[Cosets are Equivalent]], each $h K$ contains $\order K$ [[Definition:Element|elements]]. +From the [[Order of Subgroup Product/Lemma|Lemma]], the number of different such [[Definition:Left Coset|left cosets]] is: +:$\index H {H \cap K}$ +where $\index H {H \cap K}$ denotes the [[Definition:Index of Subgroup|index]] of $H \cap K$ in $H$. +From [[Lagrange's Theorem (Group Theory)|Lagrange's Theorem]]: +$\index H {H \cap K} = \dfrac {\order H} {\order {H \cap K} }$ +Hence: +:$\order {H K} = \dfrac {\order H \order K} {\order {H \cap K} }$ +{{qed}} +\end{proof} + +\begin{proof} +The number of [[Definition:Product Element|product elements]] $h k$ that can be formed where $h \in H$ and $k \in K$ is $\order H \order K$, although this may include duplication: +:$h_1 k_1 = h_2 k_2$ may be possible for $h_1, h_2 \in H, k_1, k_2 \in K$. +So, consider the [[Definition:Cartesian Product|Cartesian product]] $H \times K$. +From [[Cardinality of Cartesian Product]]: +:$\size {H \times K} = \order H \times \order K$ +Let us define a [[Definition:Relation|relation]] $\sim$ on $H \times K$ as: +:$\tuple {h_1, k_1} \sim \tuple {h_2, k_2} \iff h_1 k_1 = h_2 k_2$ +As $\sim$ is based on the [[Definition:Equals|equality relation]] it is seen that $\sim$ is an [[Definition:Equivalence Relation|equivalence relation]]: +:[[Definition:Reflexive Relation|Reflexivity]]: $h_1 k_1 = h_1 k_1$ +:[[Definition:Symmetric Relation|Symmetry]]: $h_1 k_1 = h_2 k_2 \implies h_2 k_2 = h_1 k_1$ +:[[Definition:Transitive Relation|Transitivity]]: $h_1 k_1 = h_2 k_2, h_2 k_2 = h_3 k_3 \implies h_1 k_1 = h_3 k_3$ +Each [[Definition:Equivalence Class|equivalence class]] of $\sim$ corresponds to a particular [[Definition:Element|element]] of $H K$. +Hence $\size {H K}$ is the number of [[Definition:Equivalence Class|equivalence classes]] of $\sim$. +It remains to be shown that each of these [[Definition:Equivalence Class|equivalence classes]] contains exactly $\order {H \cap K}$ [[Definition:Element|elements]]. +Let $E$ be the [[Definition:Equivalence Class|equivalence class]] of $\tuple {h k}$. +We aim to prove that: +:$(1): \quad E = \set {\tuple {h x^{-1}, x k}: x \in H \cap K}$ +Let $x \in H \cap K$. +Then: +:$a x^{-1} \in H$ +and: +:$x k \in K$ +so: +:$\tuple {h x^{-1}, x k} \in H \times K$ +[[Definition:Converse|Conversely]]: +:$\tuple {h, k} \sim \tuple {h_1, k_1} \implies h k = h_1 k_1$ +and so: +:$x = h_1^{-1} h = k_1 k^{-1} \in A \cap B$ +Thus: +:$h_1 = h x^{-1}$ +and: +:$k_1 = x k$ +and $(1)$ is seen to hold. +Finally: +:$\tuple {h x^{-1}, x k} = \tuple {h y^{-1}, y k} \implies x = y$ +Thus $E$ has exactly $\order {H \cap K}$ [[Definition:Element|elements]] and the proof is complete. +{{qed}} +\end{proof}<|endoftext|> +\section{Index of Intersection of Subgroups} +Tags: Subgroups, Index of Subgroups, Set Intersection, Index of Intersection of Subgroups + +\begin{theorem} +Let $G$ be a [[Definition:Group|group]]. +Let $H, K$ be [[Definition:Subgroup|subgroups]] of [[Definition:Finite Index|finite index]] of $G$. +Then: +:$\index G {H \cap K} \le \index G H \index G K$ +where $\index G H$ denotes the [[Definition:Index of Subgroup|index of $H$ in $G$]]. +Note that here the symbol $\le$ is being used with its meaning '''less than or equal to'''. +Equality holds {{iff}} $H K = \set {h k: h \in H, k \in K} = G$. +\end{theorem} + +\begin{proof} +Note that $H \cap K$ is a [[Definition:Subgroup|subgroup]] of $H$. +From [[Tower Law for Subgroups]], we have: +:$\index G {H \cap K} = \index G H \index H {H \cap K}$ +From [[Index in Subgroup]], also: +:$\index H {H \cap K} \le \index G K$ +Combining these results yields the desired inequality. +Again from [[Index in Subgroup]], it follows that: +:$\index H {H \cap K} = \index G K$ +{{iff}} $H K = G$. +{{qed}} +\end{proof}<|endoftext|> +\section{Intersection of Subgroups of Prime Order} +Tags: Prime Groups, Subgroups + +\begin{theorem} +Let $G$ be a [[Definition:Group|group]] whose [[Definition:Identity Element|identity]] is $e$. +Let $H$ and $K$ be [[Definition:Subset|subsets]] of $G$ such that: +:$\order H = \order K = p$ +:$H \ne K$ +:$p$ is [[Definition:Prime Number|prime]]. +Then: +: $H \cap K = \set e$ +That is, the [[Definition:Set Intersection|intersection]] of two unequal [[Definition:Subgroup|subgroups]] of a [[Definition:Group|group]], both of whose [[Definition:Order of Structure|order]] is the same [[Definition:Prime Number|prime]], consists solely of the [[Definition:Identity Element|identity]]. +\end{theorem} + +\begin{proof} +From [[Intersection of Subgroups is Subgroup]]: +:$H \cap K \le G$ +and: +:$H \cap K \le H$ +where $\le$ denotes [[Definition:Subgroup|subgrouphood]]. +So: +{{begin-eqn}} +{{eqn | l = H \cap K + | o = \le + | r = H + | c = [[Intersection of Subgroups is Subgroup]] +}} +{{eqn | ll= \leadsto + | l = \order {H \cap K} + | o = \divides + | r = \order H + | c = [[Lagrange's Theorem (Group Theory)|Lagrange's Theorem]] +}} +{{eqn | ll= \leadsto + | l = \order {H \cap K} + | o = \divides + | r = p + | c = +}} +{{eqn | ll= \leadsto + | l = \order {H \cap K} + | r = 1 \text{ or } p + | c = $p$ is [[Definition:Prime Number|prime]] +}} +{{end-eqn}} +Because $H \ne K$ and $\order H = \order k$, it follows that $H \nsubseteq K$. +So: +{{begin-eqn}} +{{eqn | l = H \cap K + | o = \ne + | r = H + | c = [[Intersection with Subset is Subset]] +}} +{{eqn | ll= \leadsto + | l = H \cap K + | o = \subset + | r = H + | c = +}} +{{eqn | ll= \leadsto + | l = \order {H \cap K} + | o = < + | r = \order H = p + | c = +}} +{{eqn | ll= \leadsto + | l = \order {H \cap K} + | r = 1 + | c = +}} +{{eqn | ll= \leadsto + | l = H \cap K + | r = \set e + | c = {{Defof|Trivial Group}} +}} +{{end-eqn}} +{{Qed}} +\end{proof}<|endoftext|> +\section{Tower Law for Subgroups} +Tags: Subgroups, Tower Law for Subgroups, Named Theorems, Index of Subgroups + +\begin{theorem} +Let $\struct {G, \circ}$ be a [[Definition:Group|group]]. +Let $H$ be a [[Definition:Subgroup|subgroup]] of $G$ with [[Definition:Finite Index|finite index]]. +Let $K$ be a [[Definition:Subgroup|subgroup]] of $H$. +Then: +:$\index G K = \index G H \index H K$ +where $\index G H$ denotes the [[Definition:Index of Subgroup|index of $H$ in $G$]]. +\end{theorem} + +\begin{proof} +Let $p = \index G H$, $q = \index H K$. +By hypothesis these numbers are [[Definition:Finite|finite]]. +Therefore, there exist $g_1, \ldots, g_p \in G$ such that $G$ is a [[Definition:Disjoint Union (Set Theory)|disjoint union]]: +$\displaystyle G = \bigsqcup_{i \mathop = 1}^p g_i H$ +Similarly, there exist $h_1,\ldots,h_q \in H$ such that $H$ is a [[Definition:Disjoint Union (Set Theory)|disjoint union]]: +$\displaystyle H = \bigsqcup_{j \mathop = 1}^q h_j K$ +Thus: +{{begin-eqn}} +{{eqn | l = G + | r = \bigsqcup_{i \mathop = 1}^p g_i H +}} +{{eqn | r = \bigsqcup_{i \mathop = 1}^p g_i \bigsqcup_{j \mathop = 1}^q h_j K +}} +{{eqn | r = \bigsqcup_{i \mathop = 1}^p \bigsqcup_{j \mathop = 1}^q g_i \paren {h_j K} + | c = [[Product of Subset with Union]] +}} +{{eqn | r = \bigsqcup_{i \mathop = 1}^p \bigsqcup_{j \mathop = 1}^q \paren {g_i h_j} K + | c = [[Subset Product within Semigroup is Associative/Corollary|Subset Product within Semigroup is Associative: Corollary]] +}} +{{end-eqn}} +This expression for $G$ is the [[Definition:Disjoint Union (Set Theory)|disjoint union]] of $p q$ [[Definition:Coset|cosets]]. +Therefore the number of elements of the [[Definition:Coset Space|coset space]] is: +:$\index G K = p q = \index G H \index H K$ +{{qed}} +\end{proof} + +\begin{proof} +Assume $G$ is [[Definition:Finite Group|finite]]. +Then: +{{begin-eqn}} +{{eqn | l = \index G H + | r = \frac {\order G} {\order H} + | c = [[Lagrange's Theorem (Group Theory)|Lagrange's Theorem]] +}} +{{eqn | l = \index G K + | r = \frac {\order G} {\order K} + | c = [[Lagrange's Theorem (Group Theory)|Lagrange's Theorem]] +}} +{{eqn | ll= \leadsto + | l = \index G K + | r = \frac {\order H} {\order K} \times \index G H + | c = +}} +{{end-eqn}} +Since $K \le H$, from [[Lagrange's Theorem (Group Theory)|Lagrange's Theorem]] we have that $\dfrac {\order H} {\order K} = \index H K$. +Hence the result. +{{qed}} +\end{proof}<|endoftext|> +\section{Morphism from Integers to Group} +Tags: Group Isomorphisms, Group Epimorphisms, Integers, Ideal Theory + +\begin{theorem} +Let $G$ be a [[Definition:Group|group]] whose [[Definition:Identity Element|identity]] is $e$. +Let $g \in G$. +Let $\phi: \Z \to G$ be the [[Definition:Mapping|mapping]] defined as: +:$\forall n \in \Z: \map \phi n = g^n$. +Then: +: If $g$ has [[Definition:Infinite Order Element|infinite order]], then $\phi$ is a [[Definition:Group Isomorphism|group isomorphism]] from $\struct {\Z, +}$ to $\gen g$. +: If $g$ has [[Definition:Finite Order Element|finite order]] such that $\order g = m$, then $\phi$ is a [[Definition:Group Epimorphism|group epimorphism]] from $\struct {\Z, +}$ to $\gen g$ whose [[Definition:Kernel of Group Homomorphism|kernel]] is the [[Definition:Principal Ideal of Ring|principal ideal]] $\paren m$. +:Thus $\gen g$ is [[Definition:Group Isomorphism|isomorphic]] to $\struct {\Z, +}$, and $m$ is the smallest [[Definition:Strictly Positive Integer|(strictly) positive integer]] such that $g^m = e$. +\end{theorem} + +\begin{proof} +By [[Epimorphism from Integers to Cyclic Group]], $\phi$ is an [[Definition:Group Epimorphism|epimorphism]] from $\struct {\Z, +}$ onto $\gen g$. +By [[Kernel of Group Homomorphism is Subgroup]], the [[Definition:Kernel of Group Homomorphism|kernel]] $K$ of $G$ is a [[Definition: Subgroup|subgroup]] of $\struct {\Z, +}$. +Therefore by [[Subgroup of Integers is Ideal]] and [[Principal Ideals of Integers]], $\exists m \in \N_{>0}: K = \paren m$. +Thus $\gen g \rangle \cong \struct {\Z, +}$. +By [[Canonical Epimorphism from Integers by Principal Ideal]]: +:$\forall m \in \N_{>0}: \order {\Z_m} = m$ +So, if $\gen g$ is [[Definition:Finite|finite]], and if $\gen g \cong \struct {\Z, +}$, then $m = \order g$. +Furthermore, $m$ is the smallest [[Definition:Strictly Positive Integer|(strictly) positive integer]] such that $g^m = e$, since $m$ is the smallest (strictly) positive integer in $\paren m$ from [[Principal Ideals of Integers]]. +If $\gen g$ is [[Definition:Infinite|infinite]], then $m = 0$ and so $\phi$ is a [[Definition:Group Isomorphism|(group) isomorphism]] from $\struct {\Z, +}$ onto $\gen g$. +{{qed}} +\end{proof}<|endoftext|> +\section{Identity is Only Group Element of Order 1} +Tags: Identity Elements, Order of Group Elements + +\begin{theorem} +In every [[Definition:Group|group]], the [[Definition:Identity Element|identity]], and only the [[Definition:Identity Element|identity]], has [[Definition:Order of Group Element|order]] $1$. +\end{theorem} + +\begin{proof} +Let $G$ be a [[Definition:Group|group]] with [[Definition:Identity Element|identity]] $e$. +Then: +:$e^1 = e$ +and: +:$\forall a \in G: a \ne e: a^1 = a \ne e$. +Hence the result. +{{qed}} +\end{proof}<|endoftext|> +\section{Group Element is Self-Inverse iff Order 2} +Tags: Order of Group Elements + +\begin{theorem} +Let $\struct {S, \circ}$ be a [[Definition:Group|group]] whose [[Definition:Identity Element|identity]] is $e$. +An [[Definition:Element|element]] $x \in \struct {S, \circ}$ is [[Definition:Self-Inverse Element|self-inverse]] {{iff}}: +:$\order x = 2$ +\end{theorem} + +\begin{proof} +Let $x \in G: x \ne e$. +{{begin-eqn}} +{{eqn | l = \order x + | r = 2 + | c = +}} +{{eqn | ll= \leadstoandfrom + | l = x \circ x + | r = e + | c = {{Defof|Order of Group Element}} +}} +{{eqn | ll= \leadstoandfrom + | l = x + | r = x^{-1} + | c = [[Equivalence of Definitions of Self-Inverse]] +}} +{{end-eqn}} +{{qed}} +{{expand|Free generalisation to monoids}} +\end{proof}<|endoftext|> +\section{Powers of Infinite Order Element} +Tags: Order of Group Elements + +\begin{theorem} +Let $G$ be a [[Definition:Group|group]] whose [[Definition:Identity Element|identity]] is $e$. +Let $a \in G$ have [[Definition:Infinite Order Element|infinite order]] in $G$. +Then: +:$\forall m, n \in \Z: m \ne n \implies a^m \ne a^n$ +\end{theorem} + +\begin{proof} +Let $m, n \in \Z$. +Then: +{{begin-eqn}} +{{eqn | l = a^m + | r = a^n + | c = +}} +{{eqn | ll= \leadsto + | l = a^m \paren {a^n}^{-1} + | r = e + | c = +}} +{{eqn | lo= \land + | l = a^n \paren {a^m}^{-1} + | r = e + | c = +}} +{{eqn | ll= \leadsto + | l = a^{m - n} + | r = a^{n - m} = e + | c = +}} +{{eqn | ll= \leadsto + | l = a^{\size {m - n} } + | r = e + | c = +}} +{{eqn | ll= \leadsto + | l = \size {m - n} + | r = 0 + | c = {{Defof|Infinite Order Element}}: there exists no $x \in \Z_{>0}$ such that $a^x = e$ +}} +{{eqn | ll= \leadsto + | l = m + | r = n + | c = +}} +{{end-eqn}} +The result follows from [[Rule of Transposition|transposition]]. +{{qed}} +\end{proof}<|endoftext|> +\section{Element of Finite Group is of Finite Order} +Tags: Finite Groups, Order of Group Elements, Element of Finite Group is of Finite Order + +\begin{theorem} +In any [[Definition:Finite Group|finite group]], each [[Definition:Element|element]] has [[Definition:Finite Order Element|finite order]]. +\end{theorem} + +\begin{proof} +Follows as a direct [[Definition:Corollary|corollary]] to the result [[Powers of Infinite Order Element]]. +{{qed}} +\end{proof} + +\begin{proof} +Let $G$ be a [[Definition:Group|group]] whose [[Definition:Identity Element|identity]] is $e$. +From [[Element has Idempotent Power in Finite Semigroup]], for every [[Definition:Element|element]] in a ''finite'' [[Definition:Semigroup|semigroup]], there is a [[Definition:Power of Group Element|power]] of that [[Definition:Element|element]] which is [[Definition:Idempotent Element|idempotent]]. +As $G$, being a [[Definition:Group|group]], is also a [[Definition:Semigroup|semigroup]], the same applies to $G$. +That is: +:$\forall x \in G: \exists n \in \N_{>0}: x^n \circ x^n = x^n$ +From [[Identity is only Idempotent Element in Group]], it follows that: +:$x^n \circ x^n = x^n \implies x^n = e$ +So $x$ has [[Definition:Finite Order Element|finite order]]. +{{Qed}} +\end{proof}<|endoftext|> +\section{Inverse Element is Power of Order Less 1} +Tags: Inverse Elements, Order of Group Elements + +\begin{theorem} +Let $G$ be a [[Definition:Group|group]] whose [[Definition:Identity Element|identity]] is $e$. +Let $g \in G$ be of [[Definition:Finite Order Element|finite order]]. +Then: +: $\order g = n \implies g^{n - 1} = g^{-1}$ +\end{theorem} + +\begin{proof} +{{begin-eqn}} +{{eqn | l = \order g + | r = n + | c = +}} +{{eqn | ll= \leadsto + | l = g^n + | r = e + | c = +}} +{{eqn | ll= \leadsto + | l = g^n g^{-1} + | r = e g^{-1} + | c = +}} +{{eqn | ll= \leadsto + | l = g^{n - 1} + | r = g^{-1} + | c = +}} +{{end-eqn}} +{{qed}} +\end{proof}<|endoftext|> +\section{Equal Powers of Finite Order Element} +Tags: Order of Group Elements + +\begin{theorem} +:$g^r = g^s \iff k \divides \paren {r - s}$ +\end{theorem} + +\begin{proof} +=== Necessary Condition === +Suppose that $k \divides \paren {r - s}$. +From the definition of [[Definition:Divisor of Integer|divisor]]: +:$k \divides \left({r - s}\right) \implies \exists t \in \Z: r - s = k t$ +So: +:$g^{r - s} = g^{k t}$ +Thus: +{{begin-eqn}} +{{eqn | l = g^r + | r = g^{s + k t} + | c = +}} +{{eqn | r = g^s g^{k t} + | c = +}} +{{eqn | r = g^s \paren {g^k}^t + | c = +}} +{{eqn | r = g^s \paren e^t + | c = +}} +{{eqn | r = g^s + | c = +}} +{{end-eqn}} +{{qed|lemma}} +=== Sufficient Condition === +Let $g^r = g^s$. +Then: +: $g^{r - s} = g^r g^{-s} = g^s g^{-s} = e$ + +By the [[Division Theorem]]: +: $r - s = q k + t$ +for some $q \in \Z, 0 \le t < k$. +Thus: +:$e = g^{r - s} = g^{k q + t} = \paren {g^k}^q g^t = e^q g^t = g^t$ +So by the definition of $k$: +:$\paren {t < k} \land \paren {e = g^t} \implies t = 0$ +So: +:$r - s = q k + 0 = q k \implies k \divides \paren {r - s}$ +{{qed}} +\end{proof}<|endoftext|> +\section{Order of Element Divides Order of Finite Group} +Tags: Finite Groups, Order of Group Elements + +\begin{theorem} +In a [[Definition:Finite Group|finite group]], the [[Definition:Order of Group Element|order of a group element]] [[Definition:Divisor of Integer|divides]] the [[Definition:Order of Group|order of its group]]: +:$\forall x \in G: \order x \divides \order G$ +\end{theorem} + +\begin{proof} +Let $G$ be a [[Definition:Group|group]]. +Let $x \in G$. +By definition, the [[Definition:Order of Group Element/Definition 2|order of $x$]] is the [[Definition:Order of Group|order]] of the [[Definition:Generated Subgroup|subgroup generated by $x$]]. +Therefore, by [[Lagrange's Theorem (Group Theory)|Lagrange's Theorem]], $\order x$ is a [[Definition:Divisor of Integer|divisor]] of $\order G$. +{{qed}} +\end{proof}<|endoftext|> +\section{Recurrence Relation for Number of Derangements on Finite Set} +Tags: Counting Arguments, Number of Derangements on Finite Set + +\begin{theorem} +The number of [[Definition:Derangement|derangements]] $D_n$ on a [[Definition:Finite Set|finite set]] $S$ of [[Definition:Cardinality|cardinality]] $n$ is: +:$D_n = \paren {n - 1} \paren {D_{n - 1} + D_{n - 2} }$ +where $D_1 = 0$, $D_2 = 1$. +\end{theorem} + +\begin{proof} +Let $\card S = 1$ such that $S = \set s$, say. +Then $\map f s = s$ is the only [[Definition:Permutation|permutation]] on $S$. +This by definition is not a [[Definition:Derangement|derangement]]. +Thus: +:$D_1 = 0$ +Let $\card S = 2$. +Then $S = \set {s, t}$, say. +There are two [[Definition:Permutation|permutations]] on $S$: +:$f = \set {\tuple {s, s}, \tuple {t, t} }$ +and: +:$g = \set {\tuple {s, t}, \tuple {t, s} }$ +and only the latter is a [[Definition:Derangement|derangement]]. +So: +:$D_2 = 1$ +Let $\card S > 2$. +Let $f: S \to S$ be a [[Definition:Derangement|derangement]]. +We aim to count the total number of such $f$. +{{WLOG}}, we can take: +:$S = \set {1, 2, \ldots, n + 1}$ +Now, consider an arbitrary $s \in S$ such that $s \ne 1$. +Let $\map f s = 1$. +By the [[Sum Rule for Counting]], the total number of $f$ will be: +:$\paren {\text {the number of $f$ where } \map f 1 \ne s} + \paren {\text {the number of $f$ where} \map f 1 = s}$ +=== Case 1 === +:$\map f 1 \ne s$ +Take: +:$T_1 = \set {1, 2, \dotsc, s - 1, s + 1, \dotsc, n + 1} = S \setminus \set s$ +Define the [[Definition:Derangement|derangement]] $g_1: T_1 \to T_1$ by: +:$\forall t \in T_1: \map {g_1} t = \map f t$ +Then $g_1$ is a [[Definition:Derangement|derangement]] on a set of [[Definition:Cardinality|cardinality]] $n$. +Thus there are $D_n$ such $g_1$. +Note that: +:$f = g_1 \cup \map f s$ +We have that $s$ can be chosen in $n$ ways. +By the [[Product Rule for Counting]] there are in total $n D_n$ such $f$ where $\map f s \ne 1$. +=== Case 2 === +:$\map f 1 = s$ +Take: +:$T_2 = \set {2, 3, \dotsc, s - 1, s + 1, \dotsc, n + 1} = S \setminus \set {1, s}$ +Define the [[Definition:Derangement|derangement]] $g_2: T_2 \to T_2$ by: +:$\forall t \in T_2: \map {g_2} t = \map f t$ +Then $g_2$ is a [[Definition:Derangement|derangement]] on a [[Definition:Set|set]] of [[Definition:Cardinality|cardinality]] $n - 1$. +So there are $D_{n - 1}$ such $g_2$. +Note that: +:$f = g_2 \cup \set s \cup \map f s$ +We have that $s$ can be chosen in $n$ ways +By the [[Product Rule for Counting]] there are in total $n D_{n - 1}$ such $f$ where $\map f s = 1$. +Summing the results from both cases, we get the total number of [[Definition:Derangement|derangements]] $f$ on a [[Definition:Set|set]] of [[Definition:Cardinality|cardinality]] $n + 1$ is: +:$D_{n + 1} = n D_n + n D_{n - 1} = n \paren {D_n + D_{n - 1} }$ +as was to be proved. +{{qed}} +[[Category:Counting Arguments]] +[[Category:Number of Derangements on Finite Set]] +r2mrcuj4x7ja1j53ganx2y98kpw4oic +\end{proof}<|endoftext|> +\section{Element to Power of Group Order is Identity} +Tags: Group Theory + +\begin{theorem} +Let $G$ be a [[Definition:Group|group]] whose [[Definition:Identity Element|identity]] is $e$ and whose [[Definition:Order of Structure|order]] is $n$. +Then: +:$\forall g \in G: g^n = e$ +\end{theorem} + +\begin{proof} +Let $G$ be a [[Definition:Group|group]] such that $\order G = n$. +Let $g \in G$ and let $\order G = k$. +From [[Order of Element Divides Order of Finite Group]]: +:$k \divides n$ +So: +:$\exists m \in \Z_{>0}: k m = n$ +Thus: +{{begin-eqn}} +{{eqn | l = g^n + | r = \paren {g^k}^m + | c = [[Powers of Group Elements/Product of Indices|Powers of Group Elements: Product of Indices]] +}} +{{eqn | r = e^m + | c = {{Defof|Order of Group Element}}: $g^k = e$ +}} +{{eqn | r = e + | c = [[Power of Identity is Identity]] +}} +{{end-eqn}} +{{qed}} +\end{proof}<|endoftext|> +\section{Boolean Group is Abelian} +Tags: Abelian Groups, Boolean Groups, Boolean Group is Abelian + +\begin{theorem} +Let $G$ be a [[Definition:Boolean Group|Boolean group]]. +Then $G$ is [[Definition:Abelian Group|abelian]]. +\end{theorem} + +\begin{proof} +By definition of [[Definition:Boolean Group|Boolean group]], all [[Definition:Element|elements]] of $G$, other than the [[Definition:Identity Element|identity]], have [[Definition:Order of Group Element|order]] $2$. +By [[Group Element is Self-Inverse iff Order 2]] and [[Identity is Self-Inverse]], all elements of $G$ are [[Definition:Self-Inverse Element|self-inverse]]. +The result follows directly from [[All Elements Self-Inverse then Abelian]]. +{{qed}} +\end{proof} + +\begin{proof} +Let $ a, b \in G$. +By definition of [[Definition:Boolean Group|Boolean group]]: +:$\forall x \in G: x^2 = e$ +where $e$ is the [[Definition:Identity Element|identity]] of $G$. +Then: +{{begin-eqn}} +{{eqn | l = a b + | r = a e b + | c = [[Definition:Group Axioms|Group Axiom $G2$]]: Properties of [[Definition:Identity Element|Identity]] +}} +{{eqn | r = a \left({a b}\right)^2 b + | c = as $\forall x \in G: x^2 = e$ +}} +{{eqn | r = a \left({a b}\right) \left({a b}\right) b + | c = +}} +{{eqn | r = \left({a a}\right) \left({b a}\right) \left({b b}\right) + | c = [[Definition:Group Axioms|Group Axiom $G1$]]: [[Definition:Associative|Associativity]] +}} +{{eqn | r = a^2 \left({b a}\right) b^2 + | c = +}} +{{eqn | r = e \left({b a}\right) e + | c = as $\forall x \in G: x^2 = e$ +}} +{{eqn | r = b a + | c = +}} +{{end-eqn}} +Thus $a b = b a$ and therefore $G$ is [[Definition:Abelian Group|abelian]]. +{{qed}} +\end{proof}<|endoftext|> +\section{Order of Group Element equals Order of Inverse} +Tags: Order of Group Elements, Inverse Elements + +\begin{theorem} +Let $G$ be a [[Definition:Group|group]] whose [[Definition:Identity Element|identity]] is $e$. +Then: +: $\forall x \in G: \order x = \order {x^{-1} }$ +where $\order x$ denotes the [[Definition:Order of Group Element|order]] of $x$. +\end{theorem} + +\begin{proof} +By [[Powers of Group Elements/Negative Index|Powers of Group Elements: Negative Index]]: +: $\paren {x^k}^{-1} = x^{-k} = \paren {x^{-1} }^k$ +Hence: +{{begin-eqn}} +{{eqn | l = x^k + | r = e + | c = +}} +{{eqn | ll= \leadsto + | l = \paren {x^{-1} }^k + | r = e^{-1} + | c = +}} +{{eqn | r = e + | c = +}} +{{eqn | ll= \leadsto + | l = \order {x^{-1} } + | o = \le + | r = \order x + | c = {{Defof|Order of Group Element}} +}} +{{end-eqn}} +Similarly: +{{begin-eqn}} +{{eqn | l = \paren {x^{-1} }^k + | r = e + | c = +}} +{{eqn | ll= \leadsto + | l = \paren {\paren {x^{-1} }^{-1} }^k + | r = e^{-1} + | c = +}} +{{eqn | ll= \leadsto + | l = x^k + | r = e^{-1} + | c = +}} +{{eqn | r = e + | c = +}} +{{eqn | ll= \leadsto + | l = \order x + | o = \le + | r = \order {x^{-1} } + | c = {{Defof|Order of Group Element}} +}} +{{end-eqn}} +A similar argument shows that if $x$ is of [[Definition:Infinite Order Element|infinite order]], then so must $x^{-1}$ be. +Hence the result. +{{Qed}} +\end{proof}<|endoftext|> +\section{Even Order Group has Order 2 Element} +Tags: Finite Groups, Order of Group Elements, Even Order Group has Order 2 Element + +\begin{theorem} +Let $G$ be a [[Definition:Group|group]] whose [[Definition:Identity Element|identity]] is $e$. +Let $G$ be of even [[Definition:Order of Structure|order]]. +Then: +:$\exists x \in G: \order x = 2$ +That is: +:$\exists x \in G: x \ne e: x^2 = e$ +\end{theorem} + +\begin{proof} +In any [[Definition:Group|group]] $G$, the [[Identity is Self-Inverse|identity element $e$ is self-inverse]] with [[Identity is Only Group Element of Order 1]], and is the only such. +That leaves an [[Definition:Odd Integer|odd number]] of [[Definition:Element|elements]]. +Each element in $x \in G: \order x > 2$ can be paired off with its [[Definition:Inverse Element|inverse]], as $\order {x^{-1} } = \order x > 2$ from [[Order of Group Element equals Order of Inverse]]. +Hence there must be at least one [[Definition:Element|element]] which has not been paired off with any of the others which is therefore [[Definition:Self-Inverse Element|self-inverse]]. +The result follows from [[Group Element is Self-Inverse iff Order 2]]. +{{qed}} +\end{proof}<|endoftext|> +\section{Order of Conjugate Element equals Order of Element} +Tags: Order of Group Elements, Conjugacy + +\begin{theorem} +Let $\struct {G, \circ}$ be a [[Definition:Group|group]] whose [[Definition:Identity Element|identity]] is $e$. +Then +:$\forall a, x \in \struct {G, \circ}: \order {x \circ a \circ x^{-1} } = \order a$ +where $\order a$ denotes the [[Definition:Order of Group Element|order of $a$ in $G$]]. +\end{theorem} + +\begin{proof} +Let $\order a = k$. +Then $a^k = e$, and: +: $\forall n \in \N_{>0}: n < k \implies a^n \ne e$ +by definition of the [[Definition:Order of Group Element|order of $a$ in $G$]] +We have: +{{begin-eqn}} +{{eqn | l = \paren {x \circ a \circ x^{-1} }^k + | r = x \circ a^k \circ x^{-1} + | c = [[Power of Conjugate equals Conjugate of Power]] +}} +{{eqn | r = x \circ e \circ x^{-1} + | c = +}} +{{eqn | r = x \circ x^{-1} + | c = +}} +{{eqn | r = e + | c = +}} +{{end-eqn}} +Thus $\order {x \circ a \circ x^{-1} } \le \order a$. +Now suppose $a^n = y, y \ne e$. +Then: +: $x \circ a^n \circ x^{-1} = x \circ y \circ x^{-1}$ +If $x \circ y = e$, then: +: $x \circ a^n \circ x^{-1} = x^{-1}$ +If $y \circ x^{-1} = e$, then: +: $x \circ a^n \circ x^{-1} = x$ +So: +: $a^n \ne e \implies x \circ a^n \circ x^{-1} = \paren {x \circ a \circ x^{-1} }^n \ne e$ +Thus: +: $\order {x \circ a \circ x^{-1} } \ge \order a$ +and the result follows. +{{qed}} +\end{proof}<|endoftext|> +\section{Order of Homomorphic Image of Group Element} +Tags: Group Homomorphisms + +\begin{theorem} +Let $G$ and $H$ be [[Definition:Group|groups]] whose [[Definition:Identity Element|identities]] are $e_G$ and $e_H$ respectively. +Let $\phi: G \to H$ be a [[Definition:Group Homomorphism|homomorphism]]. +Let $g \in G$ be of [[Definition:Order of Group Element|finite order]]. +Then: +:$\forall g \in G: \order {\map \phi g} \divides \order g$ +where $\divides$ denotes [[Definition:Divisor of Integer|divisibility]]. +\end{theorem} + +\begin{proof} +Let $\phi: G \to H$ be a [[Definition:Group Homomorphism|homomorphism]]. +Let $\order g = n, \order {\map \phi g} = m$. +{{begin-eqn}} +{{eqn | l = \paren {\map \phi g}^n + | r = \map \phi {g^n} + | c = [[Homomorphism of Power of Group Element]] +}} +{{eqn | r = \map \phi {e_G} + | c = +}} +{{eqn | r = e_H + | c = [[Homomorphism to Group Preserves Identity]] +}} +{{end-eqn}} +It follows from [[Element to Power of Multiple of Order is Identity]] that $m \divides n$. +{{Qed}} +\end{proof}<|endoftext|> +\section{No Group has Two Order 2 Elements} +Tags: Order of Group Elements + +\begin{theorem} +A [[Definition:Group|group]] can not contain exactly two [[Definition:Element|elements]] of [[Definition:Order of Group Element|order]] $2$. +\end{theorem} + +\begin{proof} +Let $\struct {G, \circ}$ be a [[Definition:Group|group]] whose [[Definition:Identity Element|identity]] is $e$. +Suppose: +: $s, t \in \struct {G, \circ}\: s \ne t, \order s = \order t = 2$ +That is. they are [[Definition:Self-Inverse Element|self-inverse]]: +:$s^2 = e = t^2$ +The [[Definition:Identity Element|identity]] is of [[Definition:Order of Group Element|order]] $1$. +Hence $s$ nor $t$ is the [[Definition:Identity Element|identity]] +Hence, as $s \ne t$, then $s \circ t \in G$ is [[Definition:Distinct Elements|distinct]] from both $s$ and $t$. +Also $s \circ t \ne e$ because $s \ne t^{-1}$. +Suppose $s$ and $t$ [[Definition:Commute|commute]]. +Then $\paren {s \circ t}^2 = e$ from [[Self-Inverse Elements Commute iff Product is Self-Inverse]]. +Thus there is a third element (at least) in $G$ which is of [[Definition:Order of Group Element|order]] $2$. +Now suppose $s$ and $t$ do ''not'' [[Definition:Commute|commute]]. +Then from [[Commutation Property in Group]], $s \circ t \circ s^{-1}$ is another [[Definition:Element|element]] of $G$ [[Definition:Distinct Elements|distinct]] from both $s$ and $t$. +But from [[Order of Conjugate Element equals Order of Element]]: +:$\order {s \circ t \circ s^{-1} } = \order t$ +and thus $s \circ t \circ s^{-1}$ is another [[Definition:Element|element]] of [[Definition:Order of Group Element|order]] $2$. +Thus there is a third [[Definition:Element|element]] (at least) in $G$ which is of [[Definition:Order of Group Element|order]] $2$. +{{Qed}} +\end{proof}<|endoftext|> +\section{Odd Order Group Element is Square} +Tags: Order of Group Elements + +\begin{theorem} +Let $\struct {G, \circ}$ be a [[Definition:Group|group]] whose [[Definition:Identity Element|identity]] is $e$. +Let $x \in G$. +Then: +:$\exists y \in G: y^2 = x$ +{{iff}}: +:the [[Definition:Order of Group Element|order]] $\order x$ is [[Definition:Odd Integer|odd]] +\end{theorem} + +\begin{proof} +Let $\order x$ be [[Definition:Odd Integer|odd]]. +Then: +:$\exists n \in \Z_{> 0}: x^{2 n - 1} = e$ +from the definition of the [[Definition:Order of Group Element|order of an element]]. +Conversely, suppose that +:$\exists n \in \Z_{> 0}: x^{2 n - 1} = e$ +Then $\order x$ is a [[Definition:Divisor of Integer|divisor]] of $2 n - 1$ from [[Element to Power of Multiple of Order is Identity]]. +Hence $\order x$ is [[Definition:Odd Integer|odd]]. +So $\order x$ is [[Definition:Odd Integer|odd]] {{iff}} $\exists n \in \Z_{> 0}: x^{2 n - 1} = e$. +Then: +{{begin-eqn}} +{{eqn | l = \order x + | r = 2 n - 1 + | c = +}} +{{eqn | ll= \leadstoandfrom + | l = x^{2 n - 1} + | r = e + | c = +}} +{{eqn | ll= \leadstoandfrom + | l = x^{2 n} + | r = x + | c = +}} +{{eqn | ll= \leadstoandfrom + | l = \paren {x^n}^2 + | r = x + | c = +}} +{{end-eqn}} +Hence the result. +{{qed}} +\end{proof}<|endoftext|> +\section{Order of Isomorphic Image of Group Element} +Tags: Group Isomorphisms + +\begin{theorem} +Let $G$ and $H$ be [[Definition:Group|groups]] whose [[Definition:Identity Element|identities]] are $e_G$ and $e_H$. +Let $\phi: G \to H$ be a [[Definition:Group Isomorphism|group isomorphism]]. +Then: +:$a \in G \implies \order {\map \phi a} = \order a$ +\end{theorem} + +\begin{proof} +First, suppose $a$ is of [[Definition:Finite Order Element|finite order]]. +By definition, $\phi$ is [[Definition:Bijection|bijective]], therefore [[Definition:Injection|injective]]. +The result then follows from [[Order of Homomorphic Image of Group Element]]. +{{qed|lemma}} +Now suppose $a$ is of [[Definition:Infinite Order Element|infinite order]]. +Suppose $\map \phi a$ is of [[Definition:Finite Order Element|finite order]]. +Consider the mapping $\phi^{-1}: H \to G$. +Let $b = \map \phi a$. +Let $\order b = m$. +Then: +:$\order a = \order {\map {\phi^{-1} } b} = m$ +and that would mean $a$ was of [[Definition:Finite Order Element|finite order]]. +The result follows by [[Rule of Transposition|transposition]]. +{{qed}} +\end{proof}<|endoftext|> +\section{Non-Trivial Group has Non-Trivial Cyclic Subgroup} +Tags: Cyclic Groups + +\begin{theorem} +Let $G$ be a [[Definition:Group|group]] whose [[Definition:Identity Element|identity element]] is $e$. +Let $g \in G$. +If $g$ has [[Definition:Infinite Order Element|infinite order]], then $\gen g$ is an [[Definition:Infinite Cyclic Group|infinite cyclic group]]. +If $\order g = n$, then $\gen g$ is a [[Definition:Cyclic Group|cyclic group]] with $n$ [[Definition:Element|elements]]. +Thus, every [[Definition:Group|group]] which is [[Definition:Non-Trivial Group|non-trivial]] has at least one [[Definition:Cyclic Group|cyclic]] [[Definition:Subgroup|subgroup]] which is also [[Definition:Non-Trivial Group|non-trivial]]. +In the case that $G$ is itself [[Definition:Cyclic Group|cyclic]], that [[Definition:Cyclic Group|cyclic]] [[Definition:Subgroup|subgroup]] may of course be itself. +\end{theorem} + +\begin{proof} +=== Infinite Order === +Suppose that $g$ has [[Definition:Infinite Order Element|infinite order]]. +We have that $\gen g$ consists of all possible [[Powers of Group Elements|powers of $g$]]. +So $\gen g$ can contain a [[Definition:Finite Set|finite number of elements]] only if some of these were equal. +Then we would have: +:$\exists i, j \in \Z, i < j: g^i = g^j$ +and so: +:$g^{j - i} = g^{i - i} = g^0 = e$ +which would mean that $g$ was of [[Definition:Finite Order Element|finite order]]. +This contradiction leads to the conclusion that $\gen g$ must be an [[Definition:Infinite Cyclic Group|infinite cyclic group]]. +{{qed|lemma}} +=== Finite Order === +If $\order g = n$, we have from [[Equal Powers of Finite Order Element]] that there are exactly $n$ different [[Definition:Element|elements]] of $G$ of the form $g^i$. +Hence $\gen g$ is a [[Definition:Cyclic Group|cyclic group]] with $n$ elements. +{{qed}} +\end{proof}<|endoftext|> +\section{Epimorphism from Integers to Cyclic Group} +Tags: Integers, Group Epimorphisms, Cyclic Groups + +\begin{theorem} +Let $\gen a = \struct {G, \circ}$ be a [[Definition:Cyclic Group|cyclic group]]. +Let $f: \Z \to G$ be a [[Definition:Mapping|mapping]] defined as: +$\forall n \in \Z: \map f n = a^n$. +Then $f$ is a [[Definition:Group Epimorphism|(group) epimorphism]] from $\struct {\Z, +}$ onto $\gen a$. +\end{theorem} + +\begin{proof} +By [[Powers of Element form Subgroup]]: +:$\forall n \in \N: a^n \in \gen a$ +Hence by the [[Index Laws for Monoids/Negative Index|Index Law for Monoids: Negative Index]]: +:$\forall n \in \Z: a^n \in \gen a$ +Also, by [[Index Laws for Monoids/Sum of Indices|Index Law for Monoids: Sum of Indices]], $f$ is a [[Definition:Group Homomorphism|homomorphism]] from $\struct {\Z, +}$ into $\struct {G, \circ}$. +By [[Homomorphism Preserves Subsemigroups]], its [[Definition:Codomain of Mapping|codomain]] $f \sqbrk \Z$ is therefore a [[Definition:Subgroup|subgroup]] of $\gen a$ containing $a$. +By [[Existence of Unique Subgroup Generated by Subset/Singleton Generator|Existence of Unique Subgroup Generated by Subset: Singleton Generator]], $\gen a$ is the smallest subgroup of $G$ containing $a$. +So: +:$f \sqbrk \Z = \gen a$ +Therefore $f$ is an [[Definition:Group Epimorphism|epimorphism]] from $\struct {\Z, +}$ onto $\struct {G, \circ}$. +{{Qed}} +\end{proof}<|endoftext|> +\section{Cyclic Group is Abelian} +Tags: Cyclic Groups, Abelian Groups, Cyclic Group is Abelian + +\begin{theorem} +Let $G$ be a [[Definition:Cyclic Group|cyclic group]]. +Then $G$ is [[Definition:Abelian Group|abelian]]. +\end{theorem} + +\begin{proof} +We have that [[Integers under Addition form Abelian Group]]. +The result then follows from combining: +: [[Epimorphism from Integers to Cyclic Group]] +: [[Epimorphism Preserves Commutativity]]. +{{qed}} +\end{proof} + +\begin{proof} +Let $G$ be a [[Definition:Cyclic Group|cyclic group]]. +All elements of $G$ are of the form $a^n$, where $n \in \Z$. +Let $x, y \in G: x = a^p, y = a^q$. +From [[Powers of Group Elements/Sum of Indices|Powers of Group Elements: Sum of Indices]]: +: $x y = a^p a^q = a^{p + q} = a^{q + p} = a^q a^p = y x$ +Thus: +: $\forall x, y \in G: x y = y x$ +and $G$ is by definition [[Definition:Abelian Group|abelian]]. +{{qed}} +\end{proof}<|endoftext|> +\section{Cyclic Groups of Same Order are Isomorphic} +Tags: Cyclic Groups, Group Isomorphisms + +\begin{theorem} +Two [[Definition:Cyclic Group|cyclic groups]] of the same [[Definition:Order of Structure|order]] are [[Definition:Group Isomorphism|isomorphic]] to each other. +\end{theorem} + +\begin{proof} +Let $G_1$ and $G_2$ be [[Definition:Cyclic Group|cyclic groups]], both of [[Definition:Finite Group|finite order]] $k$. +Let $G_1 = \gen a, G_2 = \gen b$. +Then, by the definition of a [[Definition:Cyclic Group|cyclic group]]: +:$\order a = \order b = k$ +Also, by definition: +:$G_1 = \set {a^0, a^1, \ldots, a^{k - 1} }$ +and: +:$G_2 = \set {b^0, b^1, \ldots, b^{k - 1} }$ +Let us set up the obvious [[Definition:Bijection|bijection]]: +:$\phi: G_1 \to G_2: \map \phi {a^n} = b^n$ +The next task is to show that $\phi$ is an [[Definition:Group Isomorphism|isomorphism]]. +Note that $\map \phi {a^n} = b^n$ holds for all $n \in \Z$, not just where $0 \le n < k$, as follows: +Let $n \in \Z: n = q k + r, 0 \le r < k$, by the [[Division Theorem]]. +Then, by [[Element to Power of Remainder]]: +:$a^n = a^r, b^n = b^r$ +Thus: +:$\map \phi {a^n} = \map \phi {a^r} = b^r = b^n$ +Now let $x, y \in G_1$. +Since $G_1 = \gen a$, it follows that: +:$\exists s, t \in \Z: x = a^s, y = a^t$ +Thus: +{{begin-eqn}} +{{eqn | l = \map \phi {x y} + | r = \map \phi {a^s a^t} + | c = +}} +{{eqn | r = \map \phi {a^{s + t} } + | c = +}} +{{eqn | r = b^{s + t} + | c = +}} +{{eqn | r = b^s b^t + | c = +}} +{{eqn | r = \map \phi {a^s} \map \phi {a^t} + | c = +}} +{{eqn | r = \map \phi x \map \phi y + | c = +}} +{{end-eqn}} +So $\phi$ is a [[Definition:Group Homomorphism|homomorphism]]. +As $\phi$ is [[Definition:Bijection|bijective]], $\phi$ is an [[Definition:Group Isomorphism|isomorphism]] from $G_1$ to $G_2$. +Thus $G_1 \cong G_2$ and the result is proved. +{{Qed}} +\end{proof}<|endoftext|> +\section{Order of Subgroup of Cyclic Group} +Tags: Cyclic Groups + +\begin{theorem} +Let $C_n = \gen g$ be the [[Definition:Cyclic Group|cyclic group]] of [[Definition:Order of Structure|order]] $n$ which is [[Definition:Generator of Cyclic Group|generated]] by $g$ whose [[Definition:Identity Element|identity]] is $e$. +Let $a \in C_n: a = g^i$. +Let $H = \gen a$. +Then: +:$\order H = \dfrac n {\gcd \set {n, i} }$ +where: +:$\order H$ denotes the [[Definition:Order of Structure|order]] of $H$ +:$\gcd \set {n, i}$ denotes the [[Definition:Greatest Common Divisor|greatest common divisor]] of $n$ and $i$. +\end{theorem} + +\begin{proof} +The fact that $H$ is [[Definition:Cyclic Group|cyclic]] follows from [[Subgroup of Cyclic Group is Cyclic]]. +We need to show that $H$ has $\dfrac n d$ elements. +Let $\order H = k$. +By [[Non-Trivial Group has Non-Trivial Cyclic Subgroup]]: +:$k = \order a$ +where $\order a$ denotes the [[Definition:Order of Group Element|order]] of $a$. +That is: +:$a^k = e$ +We have that $a = g^i$. +So: +{{begin-eqn}} +{{eqn | l = a^k + | r = e + | c = +}} +{{eqn | ll= \leadstoandfrom + | l = \paren {g^i}^k + | r = e = g^n + | c = +}} +{{eqn | ll= \leadstoandfrom + | l = n + | o = \divides + | r = i k + | c = {{Defof|Order of Group Element}} +}} +{{end-eqn}} +We now need to calculate the smallest $k$ such that: +: $n \divides i k$ +where $\divides$ denotes [[Definition:Divisor of Integer|divisibility]]. +That is, the smallest $t \in \N$ such that $n t = i k$. +Let $d = \gcd \set {n, i}$. +Thus: +:$t = \dfrac {k \paren {i / d} } {n / d}$ +From [[Integers Divided by GCD are Coprime]], $\dfrac n d$ and $\dfrac i d$ are [[Definition:Coprime Integers|coprime]]. +Thus from [[Euclid's Lemma]]: +:$\dfrac n d \divides k$ +As $a \divides b \implies a \le b$, the smallest value of $k$ such that $\dfrac k {\paren {n / d} } \in \Z$ is $\dfrac n d$. +Hence the result. +{{Qed}} +\end{proof}<|endoftext|> +\section{Number of Powers of Cyclic Group Element} +Tags: Cyclic Groups + +\begin{theorem} +Let $G$ be a [[Definition:Cyclic Group|cyclic group]] of [[Definition:Order of Structure|order]] $n$, [[Definition:Generator of Cyclic Group|generated]] by $g$. +Let $d \divides n$. +Then the element $g^{n/d}$ has $d$ distinct [[Definition:Power of Group Element|powers]]. +\end{theorem} + +\begin{proof} +Follows directly from [[Order of Subgroup of Cyclic Group]]: +:$\order {\gen {g^{n/d} } } = \dfrac n {\gcd \set {n, n/d} } = d$ +Thus from [[List of Elements in Finite Cyclic Group]]: +:$\gen {g^{n/d} } = \set {e, g^{n/d}, \paren {g^{n/d} }^2, \ldots, \paren {g^{n/d} }^{d - 1} }$ +and the result follows. +{{qed}} +\end{proof}<|endoftext|> +\section{Subgroup of Finite Cyclic Group is Determined by Order} +Tags: Cyclic Groups + +\begin{theorem} +Let $G = \gen g$ be a [[Definition:Cyclic Group|cyclic group]] whose [[Definition:Order of Structure|order]] is $n$ and whose [[Definition:Identity Element|identity]] is $e$. +Let $d \divides n$, where $\divides$ denotes [[Definition:Divisor of Integer|divisibility]]. +Then there exists exactly one [[Definition:Subgroup|subgroup]] $G_d = \gen {g^{n / d} }$ of $G$ with $d$ [[Definition:Element|elements]]. +\end{theorem} + +\begin{proof} +Let $G$ be [[Definition:Generated Subgroup|generated]] by $g$, such that $\order g = n$. +From [[Number of Powers of Cyclic Group Element]], $g^{n/d}$ has $d$ distinct [[Definition:Power of Group Element|powers]]. +Thus $\gen {g^{n / d} }$ has $d$ [[Definition:Element|elements]]. +Now suppose $H$ is another [[Definition:Subgroup|subgroup]] of $G$ of [[Definition:Order of Structure|order]] $d$. +Then by [[Subgroup of Cyclic Group is Cyclic]], $H$ is [[Definition:Cyclic Group|cyclic]]. +Let $H = \gen y$ where $y \in G$. +Thus $\order y = d$. +Thus $\exists r \in \Z: y = g^r$. +Since $\order y = d$, it follows that $y^d = g^{r d} = e$. +From [[Equal Powers of Finite Order Element]]: +: $n \divides r d$ +Thus: +{{begin-eqn}} +{{eqn | lo= \exists k \in \N: + | l = k n + | r = r d + | c = +}} +{{eqn | r = k \paren {\dfrac n d} d + | c = +}} +{{eqn | ll= \leadsto + | l = r + | r = k \paren {\dfrac n d} + | c = +}} +{{eqn | ll= \leadsto + | l = \dfrac n d + | o = \divides + | r = r + | c = +}} +{{end-eqn}} +Thus $y$ is a power of $g^{n / d}$. +Hence $H$ is a [[Definition:Subgroup|subgroup]] of $\gen {g^{n / d} }$. +Since both $H$ and $\gen {g^{n / d} }$ have [[Definition:Order of Structure|order]] $d$, they must be equal. +{{qed}} +\end{proof}<|endoftext|> +\section{Cyclic Group Elements whose Powers equal Identity} +Tags: Cyclic Groups + +\begin{theorem} +Let $G$ be a [[Definition:Cyclic Group|cyclic group]] whose [[Definition:Identity Element|identity]] is $e$ and whose [[Definition:Order of Structure|order]] is $n$. +Let $d \divides n$. +Then there exist exactly $d$ elements $x \in G$ satisfying the equation $x^d = e$. +These are the elements of the group $G_d$ [[Definition:Generator of Cyclic Group|generated by]] $g^{n / d}$: +:$G_d = \gen {g^{n / d} }$ +\end{theorem} + +\begin{proof} +From the argument in [[Subgroup of Finite Cyclic Group is Determined by Order]], it follows that $x$ satisfies the equation $x^d = e$ {{iff}} $x$ is a [[Definition:Power of Group Element|power]] of $g^{n/d}$. +Thus there are $d$ solutions to this equation. +{{qed}} +\end{proof}<|endoftext|> +\section{Prime Group is Cyclic} +Tags: Cyclic Groups, Prime Groups + +\begin{theorem} +Let $p$ be a [[Definition:Prime Number|prime number]]. +Let $G$ be a [[Definition:Group|group]] whose [[Definition:Order of Group|order]] is $p$. +Then $G$ is [[Definition:Cyclic Group|cyclic]]. +\end{theorem} + +\begin{proof} +Let $a \in G: a \ne e$ where $e$ is the [[Definition:Identity Element|identity]] of $G$. +From [[Group of Prime Order p has p-1 Elements of Order p]], $a$ has [[Definition:Order of Group|order]] $p$. +Hence by definition, $a$ [[Definition:Generator of Cyclic Group|generates]] $G$. +Hence also by definition, $G$ is [[Definition:Cyclic Group|cyclic]]. +{{qed}} +\end{proof}<|endoftext|> +\section{Group of Order less than 6 is Abelian} +Tags: Abelian Groups + +\begin{theorem} +All [[Definition:Group|groups]] with less than $6$ [[Definition:Element|elements]] are [[Definition:Abelian Group|abelian]]. +\end{theorem} + +\begin{proof} +Let $G$ be a non-[[Definition:Abelian Group|abelian group]]. +From [[Non-Abelian Group has Order Greater than 4]], the [[Definition:Order of Structure|order]] of $G$ must be at least $5$. +But $5$ is a [[Definition:Prime Number|prime number]]. +By [[Prime Group is Cyclic]] it follows that a [[Definition:Group|group]] of [[Definition:Order of Group|order]] $5$ is [[Definition:Cyclic Group|cyclic]]. +By [[Cyclic Group is Abelian]] this group is [[Definition:Abelian Group|abelian]]. +Hence the result. +{{qed}} +\end{proof}<|endoftext|> +\section{Power of Generator of Cyclic Group is Generator iff Power is Coprime with Order} +Tags: Cyclic Groups, Coprime Integers + +\begin{theorem} +Let $C_n$ be the [[Definition:Cyclic Group|cyclic group]] of [[Definition:Order of Structure|order]] $n$. +Let $C_n = \gen a$, that is, that $C_n$ is [[Definition:Generator of Cyclic Group|generated]] by $a$. +Then: +:$C_n = \gen {a^k} \iff k \perp n$ +That is, $C_n$ is also [[Definition:Generator of Cyclic Group|generated]] by $a^k$ {{iff}} $k$ is [[Definition:Coprime Integers|coprime]] to $n$. +\end{theorem} + +\begin{proof} +=== Necessary Condition === +Let $k \perp n$. +Then by [[Integer Combination of Coprime Integers]]: +:$\exists u, v \in \Z: 1 = u k + v n$ +So $\forall m \in \Z$, we have: +{{begin-eqn}} +{{eqn | l =a^m + | r = a^{m u k + m v n} + | c = +}} +{{eqn | r = a^{m u k} + | c = as $a^{m v n} = e$ +}} +{{eqn | r = \paren {a^k}^{m u} + | c = +}} +{{end-eqn}} +Thus $a^k$ [[Definition:Generator of Cyclic Group|generate]] $C_n$. +{{qed|lemma}} +=== Sufficient Condition === +Let $C_n = \gen {a^k}$. +That is, let $a^k$ [[Definition:Generator of Cyclic Group|generate]] $C_n$. +{{begin-eqn}} +{{eqn | lo= \exists u \in \Z: + | l = a + | r = \paren {a^k}^u + | c = as $a$ is an [[Definition:Element|element]] of the [[Definition:Generator of Cyclic Group|group generated]] by $a^k$ +}} +{{eqn | ll= \leadsto + | l = u k + | o = \equiv + | r = 1 + | rr= \pmod n + | c = +}} +{{eqn | ll= \leadsto + | lo= \exists u, v \in \Z: + | l = 1 + | r = u k + v n + | c = +}} +{{eqn | ll= \leadsto + | l = k + | o = \perp + | r = n + | c = [[Integer Combination of Coprime Integers]] +}} +{{end-eqn}} +{{qed}} +\end{proof}<|endoftext|> +\section{Order of Conjugate of Subgroup} +Tags: Conjugacy + +\begin{theorem} +Let $G$ be a [[Definition:Group|group]]. +Let $H$ be a [[Definition:Subgroup|subgroup]] of $G$ such that $H$ is of [[Definition:Order of Structure|finite order]]. +Then $\order {H^a} = \order H$. +\end{theorem} + +\begin{proof} +From the definition of [[Definition:Conjugate of Group Subset|Conjugate of Group Subet]] we have $H^a = a H a^{-1}$. +From [[Set Equivalence of Regular Representations]]: +:$\order {a H a^{-1} } = \order {a H} = \order H$ +{{qed}} +\end{proof}<|endoftext|> +\section{Subgroup of Index 2 is Normal} +Tags: Normal Subgroups + +\begin{theorem} +A [[Definition:Subgroup|subgroup]] of [[Definition:Index of Subgroup|index $2$]] is always [[Definition:Normal Subgroup|normal]]. +\end{theorem} + +\begin{proof} +Suppose $H \le G$ such that $\index G H = 2$. +Thus $H$ has two [[Definition:Left Coset|left cosets]] (and two [[Definition:Right Coset|right cosets]]) in $G$. +If $g \in H$, then $g H = H = H g$. +If $g \notin H$, then $g H = G \setminus H$ as there are only two [[Definition:Coset|cosets]] and the [[Congruence Class Modulo Subgroup is Coset|cosets partition $G$]]. +For the same reason, $g \notin H \implies H g = G \setminus H$. +That is, $g H = H g$. +The result follows from the definition of [[Definition:Normal Subgroup|normal subgroup]]. +{{qed}} +\end{proof}<|endoftext|> +\section{Intersection of Normal Subgroups is Normal} +Tags: Normal Subgroups + +\begin{theorem} +Let $G$ be a [[Definition:Group|group]]. +Let $I$ be an [[Definition:Indexing Set|indexing set]]. +Let $\family {N_i}_{i \mathop \in I}$ be a [[Definition:Non-Empty Set|non-empty]] [[Definition:Indexed Family|indexed family]] of [[Definition:Normal Subgroup|normal subgroups]] of $G$. +Then $\displaystyle \bigcap_{i \mathop \in I} N_i$ is a [[Definition:Normal Subgroup|normal subgroup]] of $G$. +\end{theorem} + +\begin{proof} +Let $\displaystyle N = \bigcap_{i \mathop \in I} N_i$. +From [[Intersection of Subgroups is Subgroup]], $N \le G$. +Suppose $H \in \set {N_i: i \in I}$. +We have that $N \subseteq H$. +Thus from [[Subgroup is Superset of Conjugate iff Normal]]: +:$a N a^{-1} \subseteq a H a^{-1} \subseteq H$ +Thus $a N a^{-1}$ is a [[Definition:Subset|subset]] of each one of the [[Definition:Subgroup|subgroups]] in $\set {N_i: i \in I}$, and hence in their [[Definition:Intersection of Family|intersection]] $N$. +That is, $a N a^{-1} \subseteq N$. +The result follows by [[Subgroup is Superset of Conjugate iff Normal]]. +{{qed}} +\end{proof}<|endoftext|> +\section{Union of Conjugacy Classes is Normal} +Tags: Normal Subgroups + +\begin{theorem} +Let $G$ be a [[Definition:Group|group]]. +Let $H \le G$. +Then $H$ is [[Definition:Normal Subgroup|normal]] in $G$ {{iff}} $H$ is a [[Definition:Set Union|union]] of [[Definition:Conjugacy Class|conjugacy classes]] of $G$. +\end{theorem} + +\begin{proof} +{{begin-eqn}} +{{eqn | l = H + | o = \lhd + | r = G + | c = where $\lhd$ denotes that $H$ is [[Definition:Normal Subgroup|normal]] in $G$ +}} +{{eqn | ll= \leadstoandfrom + | l = \forall g \in G: g H g^{-1} + | o = \subseteq + | r = H + | c = {{Defof|Normal Subgroup}} +}} +{{eqn | ll= \leadstoandfrom + | l = \forall x \in H: \forall g \in G: g x g^{-1} + | o = \in + | r = H + | c = +}} +{{eqn | ll= \leadstoandfrom + | l = \forall x \in H: \conjclass x + | o = \subseteq + | r = H + | c = where $\conjclass x$ is the [[Definition:Conjugacy Class|conjugacy class]] of $x \in G$ +}} +{{eqn | ll= \leadstoandfrom + | l = H + | r = \bigcup_{x \mathop \in H} \conjclass x + | c = +}} +{{end-eqn}} +Hence the result. +{{qed}} +\end{proof}<|endoftext|> +\section{Unique Subgroup of a Given Order is Normal} +Tags: Normal Subgroups + +\begin{theorem} +Let a [[Definition:Group|group]] $G$ have only one [[Definition:Subgroup|subgroup]] of a given [[Definition:Order of Structure|order]]. +Then that [[Definition:Subgroup|subgroup]] is [[Definition:Normal Subgroup|normal]]. +\end{theorem} + +\begin{proof} +Let $H \le G$, where $\le$ denotes that $H$ is a [[Definition:Subgroup|subgroup]] of $G$. +Let $H$ be the only [[Definition:Subgroup|subgroup]] of $G$ whose [[Definition:Order of Structure|order]] is $\order H$. +Let $g \in G$. +From [[Conjugate of Subgroup is Subgroup]]: +:$g H g^{-1} \le G$ +From [[Order of Conjugate of Subgroup]]: +: $\order {g H g^{-1} } = \order H$ +But $H$ is the only [[Definition:Subgroup|subgroup]] of $G$ of [[Definition:Order of Structure|order]] $\order H$. +Hence any [[Definition:Subgroup|subgroup]] whose [[Definition:Order of Structure|order]] is $\order H$ must in fact ''be'' $H$. +That is, $g H g^{-1} = H$. +The result follows from [[Subgroup equals Conjugate iff Normal]]. +{{Qed}} +\end{proof}<|endoftext|> +\section{Group Generated by Normal Intersection is Normal} +Tags: Normal Subgroups + +\begin{theorem} +Let $I$ be an [[Definition:Indexing Set|indexing set]], and $\left\{{N_i: i \in I}\right\}$ be a set of [[Definition:Normal Subgroup|normal subgroups]] of the [[Definition:Group|group]] $G$. +Then $\left \langle {N_i: i \in I} \right \rangle$ is a [[Definition:Normal Subgroup|normal subgroup]] of $G$. +\end{theorem} + +\begin{proof} +By definition, $\left \langle {N_i: i \in I} \right \rangle$ is the [[Definition:Set Intersection|intersection]] of all the [[Definition:Subgroup|subgroups]] of $G$ which contain every $N_i$. +For each $H \le G$, the [[Definition:Conjugate of Group Subset|conjugate]] $g H g^{-1}$ contains each $g N_i g^{-1}$. +Since each $N_i \lhd G$, it follows that: +: $N_i \subseteq g H g^{-1}$ +Thus it follows that: +: $\left \langle {N_i: i \in I} \right \rangle \lhd G$ +{{qed}} +[[Category:Normal Subgroups]] +bae4k1vl8whwigmiqi2pq8hjl1rthii +\end{proof}<|endoftext|> +\section{Smallest Normal Subgroup containing Set} +Tags: Normal Subgroups + +\begin{theorem} +Let $S \subseteq G$ where $G$ is a [[Definition:Group|group]]. +Then there exists a unique smallest [[Definition:Normal Subgroup|normal subgroup]] of $G$ which contains $S$. +\end{theorem} + +\begin{proof} +Let $\Bbb S$ be the [[Definition:Set|set]] of all [[Definition:Normal Subgroup|normal subgroups]] of $G$ that contain $S$. +$\Bbb S \ne \varnothing$, since $S \subseteq G \lhd G$. +Let $N = \bigcap H: H \in \Bbb S$, that is, the [[Definition:Set Intersection|intersection]] of all [[Definition:Element|elements]] of $\Bbb S$. +By [[Intersection of Normal Subgroups is Normal]], $N \lhd G$, and by the definition of [[Definition:Set Intersection|intersection]], $S \subseteq N$. +By the method of construction, $N$ is the smallest such subgroup. +By the definition of "smallest", $N$ is unique. +{{qed}} +[[Category:Normal Subgroups]] +syea0j00dnpyhtp2qtjk5ukvyhh4kzb +\end{proof}<|endoftext|> +\section{Conjugate of Set with Inverse Closed for Inverses} +Tags: Conjugacy + +\begin{theorem} +Let $G$ be a [[Definition:Group|group]]. +Let $S \subseteq G$. +Let $\hat S = S \cup S^{-1}$, where $S^{-1}$ is the [[Definition:Inverse of Subset of Group|set of all the inverses]] of all the elements of $S$. +Let $\tilde S = \set {a s a^{-1}: s \in \hat S, a \in G}$. +That is, $\tilde S$ is the [[Definition:Set|set]] containing all the [[Definition:Conjugate of Group Element|conjugates]] of the [[Definition:Element|elements]] of $S$ and all their [[Definition:Inverse Element|inverses]]. +Then: +: $\forall x \in \tilde S: x^{-1} \in \tilde S$ +\end{theorem} + +\begin{proof} +Let $x \in \tilde S$. +That is: +:$\exists s \in \hat S: x = a s a^{-1}$ +Then: +{{begin-eqn}} +{{eqn | l = x^{-1} + | r = \paren {a s a^{-1} }^{-1} + | c = +}} +{{eqn | r = a s^{-1} a^{-1} + | c = [[Power of Conjugate equals Conjugate of Power]] +}} +{{end-eqn}} +Since $s^{-1} \in \hat S$, it follows that $x^{-1} \in \tilde S$. +{{Qed}} +[[Category:Conjugacy]] +o56izk8q21jzbzwty9uvh72whozxfbh +\end{proof}<|endoftext|> +\section{Conjugate of Set with Inverse is Closed} +Tags: Conjugacy + +\begin{theorem} +Let $G$ be a [[Definition:Group|group]]. +Let $S \subseteq G$. +Let $\hat S = S \cup S$. +Let $\tilde S = \left\{{a s a^{-1}: s \in \hat S, a \in G}\right\}$. +Let $W \left({\tilde S}\right)$ be the [[Definition:Word (Abstract Algebra)|set of words]] of $\tilde S$. +Then $\forall w \in W \left({\tilde S}\right): \forall a \in G: a w a^{-1} \in W \left({\tilde S}\right)$. +\end{theorem} + +\begin{proof} +Let $w \in W \left({\tilde S}\right)$. +From the [[Definition:Word (Abstract Algebra)|definition]] of $W \left({\tilde S}\right)$, we have: +$w = \left({a_1 s_1 a_1^{-1}}\right) \left({a_2 s_2 a_2^{-1}}\right) \cdots \left({a_n s_n a_n^{-1}}\right), n \in \N^*, a_i \in G, s_1 \in \hat S, 1 \le i \le n$. +Thus: +{{begin-eqn}} +{{eqn | l=a w a^{-1} + | r=a \left({a_1 s_1 a_1^{-1} }\right) \left({a_2 s_2 a_2^{-1} }\right) \cdots \left({a_n s_n a_n^{-1} }\right) a^{-1} + | c= +}} +{{eqn | r=a \left({a_1 s_1 a_1^{-1} }\right) a^{-1} a \left({a_2 s_2 a_2^{-1} }\right) a^{-1} \cdots a \left({a_n s_n a_n^{-1} }\right) a^{-1} + | c= +}} +{{eqn | r=\left({\left({a a_1}\right) s_1 \left({a a_1}\right)^{-1} }\right) \left({\left({a a_2}\right) s_2 \left({a a_2}\right)^{-1} }\right) \cdots \left({\left({a a_n}\right) s_n \left({a a_n}\right)^{-1} }\right) + | c= +}} +{{end-eqn}} +As $G$ is a group, all of the $a a_i \in G$. +The result follows. +{{qed}} +[[Category:Conjugacy]] +jompv56uogazp88n2mpvhk944cn1tqz +\end{proof}<|endoftext|> +\section{Generator of Normal Subgroup} +Tags: Normal Subgroups + +\begin{theorem} +Let $G$ be a [[Definition:Group|group]]. +Let $S \subseteq G$. +Let $\hat S = S \cup S^{-1}$, where $S^{-1}$ is the [[Definition:Inverse of Subset of Group|set of all the inverses]] of all the elements of $S$. +Let $\tilde S = \left\{{a s a^{-1}: s \in \hat S, a \in G}\right\}$. +Let $W \left({\tilde S}\right)$ be the [[Definition:Word (Abstract Algebra)|set of words]] of $\tilde S$. +Let $N$ be the smallest [[Definition:Normal Subgroup|normal subgroup]] of $G$ that contains $S$. +Then $N = \left \langle {S} \right \rangle = W \left({\tilde S}\right)$. +\end{theorem} + +\begin{proof} +Let $N$ be the smallest [[Definition:Normal Subgroup|normal subgroup]] of $G$ that contains $S$, where $S \subseteq G$. +$N$ must certainly include $\hat S$, because any group containing $s \in S$ must also contain $s^{-1}$. +Therefore, $N$ must be the smallest normal subgroup containing $\hat S$. +Since $N \lhd G$, it follows that $\forall a \in G: \forall s \in \hat S: a s a^{-1} \in N$. +Thus, $\forall x \in \tilde S: x \in N$. +Thus $\tilde S \subseteq N$. +By the [[Definition:Group Axioms|closure axiom]], $N$ must also contain all products of any finite number of elements of $\tilde S$. +Thus $W \left({\tilde S}\right) \subseteq N$. +Now we prove that $W \left({\tilde S}\right) \lhd G$. +$\tilde S \ne \varnothing$, as $e \left({s s^{-1}}\right) e^{-1} = e \in \tilde S$. +By [[Conjugate of Set with Inverse Closed for Inverses]], $\tilde S$ is closed under taking inverses. +So from [[Set of Words Generates Group/Corollary|Set of Words Generates Group: Corollary]]: +: $W \left({\tilde S}\right) \le G$ +From [[Conjugate of Set with Inverse is Closed]]: +: $\forall w \in W \left({\tilde S}\right): \forall a \in G: a w a^{-1} \in W \left({\tilde S}\right)$ +From [[Subgroup is Normal iff Contains Conjugate Elements]]: +: $W \left({\tilde S}\right) \lhd G$. +The result follows by the definition of the minimality of $N$. +{{qed}} +[[Category:Normal Subgroups]] +tmf3it242yg20gtqnpz2aoa5sswhrjt +\end{proof}<|endoftext|> +\section{Subset Product with Normal Subgroup as Generator} +Tags: Normal Subgroups, Subset Products + +\begin{theorem} +Let $G$ be a [[Definition:Group|group]] whose [[Definition:Identity Element|identity]] is $e$. +Let: +:$H$ be a [[Definition:Subgroup|subgroup]] of $G$ +:$N$ be a [[Definition:Normal Subgroup|normal subgroup]] of $G$. +Then: +:$N \lhd \gen {N, H} = N H = H N \le G$ +where: +:$\le$ denotes [[Definition:Subgroup|subgroup]] +:$\lhd$ denotes [[Definition:Normal Subgroup|normal subgroup]] +:$\gen {N, H}$ denotes a [[Definition:Generator of Subgroup|subgroup generator]] +:$N H$ denotes [[Definition:Subset Product|subset product]]. +\end{theorem} + +\begin{proof} +From [[Subset Product is Subset of Generator]]: +:$N H \subseteq \gen {N, H}$ +From [[Subset Product with Normal Subgroup is Subgroup]]: +:$N H = H N \le G$ +Then by the definition of a [[Definition:Generator of Subgroup|subgroup generator]], $\gen {N, H}$ is the smallest [[Definition:Subgroup|subgroup]] containing $N H$ and so: +:$\gen {N, H} = N H = H N \le G$ +From [[Normal Subgroup of Subset Product of Subgroups]] we have that: +:$N \lhd N H$ +Hence the result. +{{qed}} +\end{proof}<|endoftext|> +\section{Subset Product of Normal Subgroups is Normal} +Tags: Normal Subgroups, Subset Products + +\begin{theorem} +Let $\struct {G, \circ}$ be a [[Definition:Group|group]]. +Let $N$ and $N'$ be [[Definition:Normal Subgroup|normal subgroups]] of $G$. +Then $N N'$ is also a [[Definition:Normal Subgroup|normal subgroup]] of $G$. +\end{theorem} + +\begin{proof} +From [[Subset Product with Normal Subgroup is Subgroup]], we already have that $N N'$ is a [[Definition:Subgroup|subgroup]] of $G$. +Let $n n' \in N N'$, so that $n \in N, n' \in N'$. +Let $g \in G$. +From [[Subgroup is Normal iff Contains Conjugate Elements]]: +:$g n g^{-1}\in N, g n' g^{-1}\in N'$ +So: +:$\paren {g n g^{-1} } \paren {g n' g^{-1} } = g n n' g^{-1} \in N N'$ +So $N N'$ is [[Definition:Normal Subgroup|normal]]. +{{qed}} +\end{proof}<|endoftext|> +\section{Prime Group is Simple} +Tags: Simple Groups, Prime Groups + +\begin{theorem} +[[Definition:Group|Groups]] of [[Definition:Prime Number|prime]] [[Definition:Order of Structure|order]] are [[Definition:Simple Group|simple]]. +\end{theorem} + +\begin{proof} +Follows directly from [[Prime Group has no Proper Subgroups]]: a group of prime order has only itself and the [[Definition:Trivial Group|trivial group]] as [[Definition:Subgroup|subgroups]]. +From [[Trivial Subgroup and Group Itself are Normal]], these subgroups are normal. +{{qed}} +\end{proof}<|endoftext|> +\section{Prime Group has no Proper Subgroups} +Tags: Subgroups, Prime Groups + +\begin{theorem} +A [[Definition:Non-Trivial Group|nontrivial group]] $G$ has no [[Definition:Proper Subgroup|proper subgroups]] except the [[Definition:Trivial Group|trivial group]] {{iff}} $G$ is [[Definition:Finite Group|finite]] and its [[Definition:Order of Structure|order]] is [[Definition:Prime Number|prime]]. +\end{theorem} + +\begin{proof} +=== Sufficient Condition === +Suppose $G$ is [[Definition:Finite Group|finite]] and of [[Definition:Prime Number|prime]] [[Definition:Order of Structure|order]] $p$. +From [[Lagrange's Theorem (Group Theory)|Lagrange's Theorem]], the [[Definition:Order of Structure|order]] of any [[Definition:Subgroup|subgroup]] of $G$ must [[Definition:Divisor of Integer|divide]] the [[Definition:Order of Structure|order]] $p$ of $G$. +From the definition of [[Definition:Prime Number|prime]], any [[Definition:Subgroup|subgroups]] of $p$ can therefore only have [[Definition:Order of Structure|order]] $1$ or $p$. +Hence $G$ can have only itself and the [[Definition:Trivial Group|trivial group]] as [[Definition:Subgroup|subgroups]]. +{{qed|lemma}} +=== Necessary Condition === +Suppose $G$ is ''not'' [[Definition:Finite Group|finite]] and [[Definition:Prime Group|prime]]. +Let the [[Definition:Identity Element|identity]] of $G$ be $e$. +Let $h \in G$ be an [[Definition:Element|element]] of $G$ such that $h \ne e$. +Then $H = \gen h$ is a [[Definition:Cyclic Group|cyclic subgroup]] of $G$. +If $H \ne G$ then $H$ is a [[Definition:Non-Trivial Group|non-trivial]] [[Definition:Proper Subgroup|proper subgroup]] of $G$, and the proof is complete. +Otherwise, $H = G$ is a [[Definition:Cyclic Group|cyclic group]] and there are two possibilities: +:$(1): \quad G$ is [[Definition:Infinite Cyclic Group|infinite]] +:$(2): \quad G$ is [[Definition:Finite Cyclic Group|finite]] (and of non-[[Definition:Prime Group|prime]] [[Definition:Order of Structure|order]]). +First, suppose $G$ is [[Definition:Infinite Cyclic Group|infinite]]. +From [[Infinite Cyclic Group is Isomorphic to Integers]], $G$ is [[Definition:Group Isomorphism|isomorphic]] to $\struct {\Z, +}$. +From [[Subgroups of Additive Group of Integers]], $\struct {\Z, +}$ has [[Definition:Proper Subgroup|proper subgroups]], for example: $\gen 2$. +Because $G \cong \struct {\Z, +}$, then so does $G$ have [[Definition:Proper Subgroup|proper subgroups]], and the proof is complete. +Suppose $G$ is [[Definition:Finite Group|finite]], and of [[Definition:Order of Structure|order]] $n$ where $n$ is not [[Definition:Prime Group|prime]] . +Then: +: $\exists d \in \N: d \divides n, 1 < d < n$ +From [[Subgroup of Finite Cyclic Group is Determined by Order]], $G$ has a [[Definition:Proper Subgroup|proper subgroup]] of [[Definition:Order of Structure|order]] $d$ and again, the proof is complete. +{{qed}} +\end{proof}<|endoftext|> +\section{Quotient Group of Cyclic Group} +Tags: Cyclic Groups, Quotient Groups, Quotient Group of Cyclic Group + +\begin{theorem} +Let $G$ be a [[Definition:Cyclic Group|cyclic group]] which is [[Definition:Generator of Cyclic Group|generated by $g$]]. +Let $H$ be a [[Definition:Subgroup|subgroup]] of $G$. +Then $g H$ [[Definition:Generator of Cyclic Group|generates]] $G / H$. +\end{theorem} + +\begin{proof} +Let $G$ be a [[Definition:Cyclic Group|cyclic group]] [[Definition:Generator of Cyclic Group|generated by $g$]]. +Let $H \le G$. +We need to show that every element of $G / H$ is of the form $\left({g H}\right)^k$ for some $k \in \Z$. +Suppose $x H \in G / H$. +Then, since $G$ is generated by $g$, $x = g^k$ for some $k \in \Z$. +But $\left({g H}\right)^k = \left({g^k}\right) H = x H$. +So $g H$ [[Definition:Generator of Cyclic Group|generates]] $G / H$. +{{Qed}} +\end{proof} + +\begin{proof} +Let $H$ be a [[Definition:Subgroup|subgroup]] of the [[Definition:Cyclic Group|cyclic group]] $G = \left \langle {g} \right \rangle$. +Then by [[Homomorphism of Powers/Integers|Homomorphism of Powers for Integers]]: +:$\forall n \in \Z: q_H \left({g^n}\right) = \left({q_H \left({g}\right)}\right)^n = \left({g H}\right)^n$ +As $G = \left\{{g^n: n \in \Z}\right\}$, we conclude that: +:$G / H = q_H \left({G}\right) = \left\{{\left({g H}\right)^n: n \in \Z}\right\}$ +Thus, by [[Epimorphism from Integers to Cyclic Group]], $g H$ generates $G / H$. +{{Qed}} +\end{proof}<|endoftext|> +\section{Order of Element in Quotient Group} +Tags: Quotient Groups + +\begin{theorem} +Let $G$ be a [[Definition:Group|group]], and let $H$ be a [[Definition:Normal Subgroup|normal subgroup]] of $G$. +Let $G / H$ be the [[Definition:Quotient Group|quotient group]] of $G$ by $H$. +The [[Definition:Order of Group Element|order]] of $a H \in G / H$ [[Definition:Divisor of Integer|divides]] the [[Definition:Order of Group Element|order]] of $a \in G$. +\end{theorem} + +\begin{proof} +Let $G$ be a [[Definition:Group|group]] with [[Definition:Normal Subgroup|normal subgroup]] $H$. +Let $G / H$ be the [[Definition:Quotient Group|quotient of $G$ by $H$]]. +From [[Quotient Group Epimorphism is Epimorphism]], $G / H$ is a [[Definition:Homomorphic Image|homomorphic image]] of $G$. +Let $q_H: G \to G / H$ given by $\map f a = a H$ be that [[Definition:Quotient Epimorphism|quotient epimorphism]]. +Let $a \in G$ such that $a^n = e$ for some [[Definition:Integer|integer]] $n$. +Then, by the [[Definition:Morphism Property|morphism property]] of $q_H$: +{{begin-eqn}} +{{eqn | l = \map {q_H} {a^n} + | r = \paren {\map {q_H} a}^n + | c = +}} +{{eqn | r = \paren {a H}^n + | c = +}} +{{eqn | r = a^n H + | c = {{Defof|Coset Product}} +}} +{{eqn | r = e H + | c = [[Definition:By Hypothesis|by hypothesis]] +}} +{{eqn | r = H + | c = +}} +{{end-eqn}} +Hence $\order H$ [[Definition:Divisor of Integer|divides]] $n$. +{{qed}} +\end{proof}<|endoftext|> +\section{Abelian Quotient Group} +Tags: Abelian Groups, Quotient Groups + +\begin{theorem} +Let $G$ be a [[Definition:Group|group]]. +Let $H$ be a [[Definition:Normal Subgroup|normal subgroup]] of $G$. +Let $G / H$ denote the [[Definition:Quotient Group|quotient group]] of $G$ by $H$. +Then $G / H$ is [[Definition:Abelian Group|abelian]] {{iff}} $H$ contains every element of $G$ of the form $a b a^{-1} b^{-1}$ where $a, b \in G$. +\end{theorem} + +\begin{proof} +Let $G / H$ be [[Definition:Abelian Group|abelian]]. +Then: +{{begin-eqn}} +{{eqn | lo= \forall a H, b H \in G / H: + | l = a H b H + | r = b H a H + | c = +}} +{{eqn | ll= \leadsto + | l = a b H + | r = b a H + | c = +}} +{{eqn | ll= \leadsto + | l = a b \paren {b a}^{-1} + | o = \in + | r = H + | c = +}} +{{eqn | ll= \leadsto + | l = a b a^{-1} b^{-1} + | o = \in + | r = H + | c = +}} +{{end-eqn}} +The argument reverses. +Suppose that $\forall a, b \in G: a b a^{-1} b^{-1} \in H$. +{{begin-eqn}} +{{eqn | lo= \forall a, b \in G: + | l = a b a^{-1} b^{-1} + | o = \in + | r = H + | c = +}} +{{eqn | ll= \leadsto + | l = a b \paren {b a}^{-1} + | o = \in + | r = H + | c = +}} +{{eqn | ll= \leadsto + | l = a b H + | r = b a H + | c = +}} +{{eqn | ll= \leadsto + | l = a H b H + | r = b H a H + | c = +}} +{{end-eqn}} +{{Qed}} +\end{proof}<|endoftext|> +\section{Intersection of Quotient Groups} +Tags: Quotient Groups + +\begin{theorem} +Let $N \lhd G$ be a [[Definition:Normal Subgroup|normal subgroup]] of $G$. +Let: +:$N \le A \le G$ +:$N \le B \le G$ +For a [[Definition:Subgroup|subgroup]] $H$ of $G$, let $\alpha$ be the [[Definition:Bijection|bijection]] defined as: +:$\map \alpha H = \set {h N: h \in H}$ +Then: +:$\map \alpha {A \cap B} = \map \alpha A \cap \map \alpha B$ +\end{theorem} + +\begin{proof} +From the proof of the [[Correspondence Theorem (Group Theory)|Correspondence Theorem]]: +:$\map \alpha H \subseteq G / N$ +Then: +{{begin-eqn}} +{{eqn | l = \map \alpha A \cap \map \alpha B + | r = \set {g N: g \in A \cap B} + | c = +}} +{{eqn | r = \paren {A \cap B} / N + | c = +}} +{{eqn | r = \map \alpha {A \cap B} + | c = +}} +{{end-eqn}} +{{qed}} +\end{proof}<|endoftext|> +\section{Generator of Quotient Groups} +Tags: Quotient Groups, Generated Subgroups + +\begin{theorem} +Let $N \lhd G$ be a [[Definition:Normal Subgroup|normal subgroup]] of $G$. +Let: +:$N \le A \le G$ +:$N \le B \le G$ +For a [[Definition:Subgroup|subgroup]] $H$ of $G$, let $\alpha$ be the [[Definition:Bijection|bijection]] defined as: +:$\map \alpha H = \set {h N: h \in H}$ +Then: +:$\map \alpha {\gen {A, B} } = \gen {\map \alpha A, \map \alpha B}$ +where $\gen {A, B}$ denotes the [[Definition:Generated Subgroup|subgroup of $G$ generated]] by $\set {A, B}$. +\end{theorem} + +\begin{proof} +From the proof of the [[Correspondence Theorem (Group Theory)|Correspondence Theorem]]: +:$\map \alpha H \subseteq G / N$ +Then: +{{begin-eqn}} +{{eqn | l = \map \alpha {\gen {A, B} } + | r = \set {h N: h \in \gen {A, B} } + | c = +}} +{{eqn | r = \set {h N \in \gen {A / N, B / N} } + | c = +}} +{{eqn | r = \gen {\map \alpha A, \map \alpha B} + | c = +}} +{{end-eqn}} +{{qed}} +\end{proof}<|endoftext|> +\section{Quotient of Group by Center Cyclic implies Abelian} +Tags: Quotient Groups, Cyclic Groups, Abelian Groups, Centers of Groups + +\begin{theorem} +Let $G$ be a [[Definition:Group|group]]. +Let $\map Z G$ be the [[Definition:Center of Group|center]] of $G$. +Let $G / \map Z G$ be the [[Definition:Quotient Group|quotient group]] of $G$ by $\map Z G$. +Let $G / \map Z G$ be [[Definition:Cyclic Group|cyclic]]. +Then $G$ is [[Definition:Abelian Group|abelian]], so $G = \map Z G$. +That is, the group $G / \map Z G$ cannot be a [[Definition:Cyclic Group|cyclic group]] which is [[Definition:Non-Trivial Group|non-trivial]]. +\end{theorem} + +\begin{proof} +Suppose $G / \map Z G$ is [[Definition:Cyclic Group|cyclic]]. +Then by definition: +:$\exists \tau \in G / \map Z G: G / \map Z G = \gen \tau$ +Since $\tau$ is a [[Definition:Coset|coset]] by $\map Z G$: +:$\exists t \in G: \tau = t \map Z G$ +Thus each [[Definition:Coset|coset]] of $\map Z G$ in $G$ is equal to $\paren {t \map Z G}^i = t^i \map Z G$ for some $i \in \Z$. +Now let $x, y \in G$. +Suppose $x \in t^m \map Z G, y \in t^n \map Z G$. +Then $x = t^m z_1, y = t^n z_2$ for some $z_1, z_2 \in \map Z G$. +Thus: +{{begin-eqn}} +{{eqn | l = x y + | r = t^m z_1 t^n z_2 + | c = +}} +{{eqn | r = t^m t^n z_1 z_2 + | c = $z_1$ [[Definition:Commute|commutes]] with all $t \in G$ since it is in the [[Definition:Center of Group|center]] +}} +{{eqn | r = t^{m + n} z_1 z_2 + | c = +}} +{{eqn | r = t^{n + m} z_2 z_1 + | c = +}} +{{eqn | r = t^n t^m z_2 z_1 + | c = +}} +{{eqn | r = t^n z_2 t^m z_1 + | c = $z_2$ [[Definition:Commute|commutes]] with all $t \in G$ since it is in the [[Definition:Center of Group|center]] +}} +{{eqn | r = y x + | c = +}} +{{end-eqn}} +This holds for all $x, y \in G$, and thus $G$ is [[Definition:Abelian Group|abelian]]. +Thus by [[Group equals Center iff Abelian]] $\map Z G = G$. +Therefore [[Quotient of Group by Itself]] it follows that $G / \map Z G$ is the [[Definition:Trivial Group|trivial group]]. +{{qed}} +\end{proof}<|endoftext|> +\section{Centralizer is Normal Subgroup of Normalizer} +Tags: Normal Subgroups, Normalizers, Quotient Groups + +\begin{theorem} +Let $G$ be a [[Definition:Group|group]]. +Let $H \le G$ be a [[Definition:Subgroup|subgroup]] of $G$. +Let $\map {C_G} H$ be the [[Definition:Centralizer of Subgroup|centralizer]] of $H$ in $G$. +Let $\map {N_G} H$ be the [[Definition:Normalizer|normalizer]] of $H$ in $G$. +Let $\Aut H$ be the [[Definition:Automorphism Group of Group|automorphism group]] of $H$. +Then: +:$(1): \quad \map {C_G} H \lhd \map {N_G} H$ +:$(2): \quad \map {N_G} H / \map {C_G} H \cong K$ +where: +:$\map {N_G} H / \map {C_G} H$ is the [[Definition:Quotient Group|quotient group]] of $\map {N_G} H$ by $\map {C_G} H$ +:$K$ is a [[Definition:Subgroup|subgroup]] of $\Aut H$. +\end{theorem} + +\begin{proof} +In order to invoke the [[First Isomorphism Theorem for Groups]], we must construct a [[Definition:Group Homomorphism|group homomorphism]] $\phi: \map {N_G} H \to \Aut H$. +Consider the mapping $\phi: x \mapsto \paren {g \mapsto x g x^{-1}}$. +From [[Inner Automorphism is Automorphism]], $g \mapsto x g x^{-1}$ is an [[Definition:Group Automorphism|automorphism]] of $G$, so $\phi$ is well-defined. +To see that $\phi$ is a [[Definition:Group Homomorphism|homomorphism]], notice that for any $x, y \in \map {N_G} H$: +{{begin-eqn}} +{{eqn | l = \map \phi x \map \phi y + | r = \paren {g \mapsto x g x^{-1} } \circ \paren {g \mapsto y g y^{-1} } + | c = where $\circ$ denote [[Definition:Composition of Mappings|composition of maps]] +}} +{{eqn | r = g \mapsto x \paren {y g y^{-1} } x^{-1} +}} +{{eqn | r = g \mapsto \paren {x y} g \paren {x y}^{-1} + | c = [[Inverse of Group Product]] +}} +{{eqn | r = \map \phi {x y} +}} +{{end-eqn}} +Hence $\phi$ is a [[Definition:Group Homomorphism|homomorphism]]. +Now we prove that $\ker \phi = \map {C_G} H$. +Note that for $x \in \map {N_G} H$: +{{begin-eqn}} +{{eqn | l = x + | o = \in + | r = \ker \phi +}} +{{eqn | ll = \leadstoandfrom + | l = g + | r = x g x^{-1} + | rr = \forall g \in H + | c = $g \mapsto g$ is the [[Definition:Identity Element of Group|identity]] of $\Aut H$ +}} +{{eqn | ll = \leadstoandfrom + | l = g x + | r = x g + | rr = \forall g \in H +}} +{{eqn | ll = \leadstoandfrom + | l = x + | o = \in + | r = \map {C_G} H + | c = {{Defof|Centralizer of Subgroup}} +}} +{{end-eqn}} +Hence $\ker \phi = \map {C_G} H$. +By [[Kernel is Normal Subgroup of Domain]]: +:$\map {C_G} H \lhd \map {N_G} H$ +By [[First Isomorphism Theorem for Groups]]: +:$\map {N_G} H / \map {C_G} H \cong \Img \phi$ +By [[Image of Group Homomorphism is Subgroup]]: +:$\Img \phi \le \Aut H$ +Hence the result. +{{qed}} +\end{proof}<|endoftext|> +\section{Number of Distinct Conjugate Subsets is Index of Normalizer} +Tags: Normalizers, Conjugacy, Number of Distinct Conjugate Subsets is Index of Normalizer + +\begin{theorem} +Let $G$ be a [[Definition:Group|group]]. +Let $S$ be a [[Definition:Subset|subset]] of $G$. +Let $\map {N_G} S$ be the [[Definition:Normalizer|normalizer]] of $S$ in $G$. +Let $\index G {\map {N_G} S}$ be the [[Definition:Index of Subgroup|index of $\map {N_G} S$ in $G$]]. +The number of distinct [[Definition:Subset|subsets]] of $G$ which are [[Definition:Conjugate of Group Subset|conjugates]] of $S \subseteq G$ is $\index G {\map {N_G} S}$. +\end{theorem} + +\begin{proof} +Let $G$ [[Definition:Group Action|act on]] its [[Definition:Power Set|power set]] $\powerset G$ by the rule: +:$\forall g \in G, S \in \powerset G: g * S := S^{g^{-1} } = \set {x \in G: g^{-1} x g \in S}$ +That is, the [[Definition:Conjugacy Action on Subsets|conjugacy action on subsets]]. +From [[Conjugacy Action on Subsets is Group Action]], $*$ is a [[Definition:Group Action|group action]]. +The [[Definition:Orbit (Group Theory)|orbit]] of $S \in \powerset G$ is the [[Definition:Conjugacy Class|conjugacy class]] of $S$. +That is, $\Orb S$ is the [[Definition:Set|set]] of [[Definition:Distinct|distinct]] [[Definition:Subset|subsets]] of $G$ which are [[Definition:Conjugate of Group Subset|conjugate]] to $S$ +The [[Definition:Stabilizer|stabilizer]] of $S$ is its [[Definition:Normalizer|normalizer]] $\map {N_G} S$. +From the [[Orbit-Stabilizer Theorem]]: +:$\order {\Orb x} = \index G {\Stab x} = \dfrac {\order G} {\order {\Stab x} }$ +from which the result follows directly. +{{Qed}} +\end{proof}<|endoftext|> +\section{Subset has 2 Conjugates then Normal Subgroup} +Tags: Conjugacy, Normal Subgroups + +\begin{theorem} +Let $G$ be a [[Definition:Group|group]]. +Let $S$ be a [[Definition:Subset|subset]] of $G$. +Let $S$ have exactly two [[Definition:Conjugate of Group Subset|conjugates]] in $G$. +Then $G$ has a [[Definition:Proper Subgroup|proper]] [[Definition:Non-Trivial Group|non-trivial]] [[Definition:Normal Subgroup|normal subgroup]]. +\end{theorem} + +\begin{proof} +{{MissingLinks}} +Consider the [[Definition:Centralizer|centralizer]] $\map {C_G} S$ of $S$ in $G$. +From [[Centralizer of Group Subset is Subgroup]], $\map {C_G} S$ is a [[Definition:Subgroup|subgroup]] of $G$. +If $\map {C_G} S = G$, then $S$ has no [[Definition:Conjugate of Group Subset|conjugate]] but itself. +{{explain|Link to that result.}} +So, in order for $S$ to have exactly two [[Definition:Conjugate of Group Subset|conjugates]] in $G$, it is necessary for $\map {C_G} S$ to be a [[Definition:Proper Subgroup|proper subgroup]]. +Let $e$ be the [[Definition:Identity Element|identity]] of $G$. +If $\map {C_G} S = \set e$, then for there to be exactly two [[Definition:Conjugate of Group Subset|conjugates]] of $S$: +:$\forall a \ne b \in G \setminus \set e: b x b^{-1} = a x a^{-1}$ +But: +{{begin-eqn}} +{{eqn | l = b x b^{-1} + | r = a x a^{-1} + | c = +}} +{{eqn | ll= \leadsto + | l = \paren {a^{-1} b} x b^{-1} + | r = x a^{-1} + | c = +}} +{{eqn | ll= \leadsto + | l = \paren {a^{-1} b} x \paren {a^{-1} b}^{-1} + | r = x + | c = +}} +{{eqn | ll= \leadsto + | l = a^{-1} b + | o = \in + | r = \map {C_G} S + | c = +}} +{{end-eqn}} +This implies either that $\map {C_G} S$ is actually [[Definition:Non-Trivial Group|nontrivial]], or that $a^{-1}b = e \iff a = b$, a [[Definition:Contradiction|contradiction]]. +Thus $\map {C_G} S$ is a [[Definition:Non-Trivial Group|nontrivial]] [[Definition:Proper Subgroup|proper subgroup]] of $G$. +We have that there are exactly $2$ [[Definition:Conjugacy Class|conjugacy classes]] of $S$. +These are in [[Definition:One-to-One Correspondence|one-to-one correspondence]] with [[Definition:Coset|cosets]] of $S$. +Thus the [[Definition:Index of Subgroup|index]] $\index G {\map {C_G} S}$ of the [[Definition:Centralizer of Group Subset|centralizer]] is: +:$\index G {\map {C_G} S} = 2$ +From [[Subgroup of Index 2 is Normal]]: +:$\map {C_G} S$ is a [[Definition:Proper Subgroup|proper]] [[Definition:Non-Trivial Group|nontrivial]] [[Definition:Normal Subgroup|normal subgroup]] of $G$. +{{qed}} +\end{proof}<|endoftext|> +\section{Element of Group Not Conjugate to Proper Subgroup} +Tags: Conjugacy + +\begin{theorem} +Let $G$ be a [[Definition:Finite Group|finite group]]. +Let $H$ be a [[Definition:Proper Subgroup|proper subgroup]] of $G$. +Then there is at least one [[Definition:Element|element]] of $G$ not contained in $H$ or in any of its [[Definition:Conjugate of Group Subset|conjugates]]. +\end{theorem} + +\begin{proof} +Let $S \subseteq G$ be defined by: +:$S := \set {g \in G: \exists h \in H, a \in G: g = a h a^{-1} }$ +It is required to show that $S \ne G$. +Let $\map {N_G} H$ be the [[Definition:Normalizer|normalizer]] of $H$ in $G$. +By [[Subgroup is Subgroup of Normalizer]], $H \le \map {N_G} H$. +Therefore, by definition of [[Definition:Index of Subgroup|index]]: +:$\index G {\map {N_G} H} \le \index G H$ +Each of the [[Definition:Conjugate of Group Subset|conjugates]] of $H$ has $\order H$ elements. +By [[Number of Distinct Conjugate Subsets is Index of Normalizer]], there are $\index G {\map {N_G} H}$ [[Definition:Conjugate of Group Subset|conjugates]]. +Therefore: +:$\card S \le \order H \index G {\map {N_G} H}$ +with equality {{iff}} all [[Definition:Conjugate of Group Subset|conjugates]] of $H$ are [[Definition:Disjoint Sets|disjoint]]. +Combining this with the earlier inequality, we find that: +:$\card S \le \order H \index G H = \order G$ +Now if $H$ is [[Definition:Normal Subgroup|normal]] in $G$, then by [[Normalizer of Subgroup is Largest Subgroup containing that Subgroup as Normal Subgroup]]: +:$\index G {\map {N_G} H} < \index G H$ +for $\map {N_G} H = G$ in that case, and $H$ is a [[Definition:Proper Subgroup|proper subgroup]]. +If $H$ is not [[Definition:Normal Subgroup|normal]] in $G$, then it has multiple [[Definition:Conjugate of Group Subset|conjugates]]. +By [[Conjugate of Subgroup is Subgroup]], they all are [[Definition:Subgroup|subgroups]] of $G$. +In particular, then, the [[Definition:Identity Element|identity]] of $G$ is common to all of them. +Hence they are not [[Definition:Disjoint Sets|disjoint]], and: +:$\card S < \order H \index G {\map {N_G} H}$ +In either case, it follows that: +:$\card S < \order G$ +as desired. +{{qed}} +\end{proof}<|endoftext|> +\section{Second Isomorphism Theorem} +Tags: Named Theorems, Isomorphisms, Isomorphism Theorems + +\begin{theorem} +=== [[Second Isomorphism Theorem/Groups|Groups]] === +{{:Second Isomorphism Theorem/Groups}} +=== [[Second Isomorphism Theorem/Rings|Rings]] === +{{:Second Isomorphism Theorem/Rings}} +This result is also referred to by some sources as the '''first isomorphism theorem'''. +\end{theorem}<|endoftext|> +\section{Third Isomorphism Theorem} +Tags: Isomorphisms, Named Theorems, Isomorphism Theorems + +\begin{theorem} +=== [[Third Isomorphism Theorem/Groups|Groups]] === +{{:Third Isomorphism Theorem/Groups}} +=== [[Third Isomorphism Theorem/Rings|Rings]] === +{{:Third Isomorphism Theorem/Rings}} +\end{theorem}<|endoftext|> +\section{Parity Group is Group} +Tags: Parity Group, Groups of Order 2 + +\begin{theorem} +The [[Definition:Parity Group|parity group]] is in fact a [[Definition:Group|group]]. +\end{theorem} + +\begin{proof} +We can completely describe the parity group by showing its [[Definition:Cayley Table|Cayley table]]: +:$\begin{array}{r|rr} +\struct {\set {1, -1}, \times} & 1 & -1\\ +\hline +1 & 1 & -1 \\ +-1 & -1 & 1 \\ +\end{array} \qquad +\begin{array}{r|rr} +\struct {\Z_2, +_2} & \eqclass 0 2 & \eqclass 1 2 \\ +\hline +\eqclass 0 2 & \eqclass 0 2 & \eqclass 1 2 \\ +\eqclass 1 2 & \eqclass 1 2 & \eqclass 0 2 \\ +\end{array}$ +From [[Prime Group is Cyclic]], there is only one [[Definition:Group|group]] of [[Definition:Order of Group|order $2$]], up to [[Definition:Group Isomorphism|isomorphism]]. +Thus both instantiations of the [[Definition:Parity Group|parity group]] are [[Definition:Group Isomorphism|isomorphic]] to $C_2$, the [[Definition:Cyclic Group|cyclic group]] of [[Definition:Order of Group|order $2$]]. +{{qed}} +\end{proof}<|endoftext|> +\section{Inverse of Inner Automorphism} +Tags: Inner Automorphisms + +\begin{theorem} +Let $G$ be a [[Definition:Group|group]]. +Let $x \in G$. +Let $\kappa_x$ be the [[Definition:Inner Automorphism|inner automorphism of $G$ given by $x$]]. +Then: +: $\paren {\kappa_x}^{-1} = \kappa_{x^{-1} }$ +\end{theorem} + +\begin{proof} +Let $G$ be a [[Definition:Group|group]] whose [[Definition:Identity Element|identity]] is $e$. +Let $x \in G$. +Let $\kappa_x \in \Inn G$. +Then from the definition of [[Definition:Inner Automorphism|inner automorphism]]: +:$\forall g \in G: \map {\kappa_x} g = x g x^{-1}$ +As $G$ is a [[Definition:Group|group]]: +:$x \in G \implies x^{-1} \in G$ +So: +:$\kappa_{x^{-1} } \in \Inn G$ +and is defined as: +:$\forall g \in G: \map {\kappa_{x^{-1} } } g = x^{-1} g \paren {x^{-1} }^{-1} = x^{-1} g x$ +Now we need to show that: +:$\kappa_x \circ \kappa_{x^{-1} } = I_G = \kappa_{x^{-1} } \circ \kappa_x$ +where $I_G: G \to G$ is the [[Definition:Identity Mapping|identity mapping]]. +So: +{{begin-eqn}} +{{eqn | ll= \forall g \in G: + | l = \map {\kappa_x \circ \kappa_{x^{-1} } } g + | r = \map {\kappa_x} {\map {\kappa_{x^{-1} } } g} + | c = {{Defof|Composition of Mappings}} +}} +{{eqn | r = \map {\kappa_x } {x^{-1} g x} + | c = Definition of $\kappa_{x^{-1} }$ +}} +{{eqn | r = x \paren {x^{-1} g x} x^{-1} + | c = Definition of $\kappa_x$ +}} +{{eqn | r = g + | c = [[Definition:Group|Group Properties]] +}} +{{eqn | r = x^{-1} \paren {x g x^{-1} } x + | c = [[Definition:Group|Group Properties]] +}} +{{eqn | r = \map {\kappa_{x^{-1} } } {x g x^{-1} } + | c = Definition of $\kappa_{x^{-1} }$ +}} +{{eqn | r = \map {\kappa_{x^{-1} } } {\map {\kappa_x} g} + | c = Definition of $\kappa_x$ +}} +{{eqn | r = \map {\kappa_{x^{-1} } \circ \kappa_x} g + | c = {{Defof|Composition of Mappings}} +}} +{{end-eqn}} +Thus: +:$\forall g \in G: \map {\kappa_x \circ \kappa_{x^{-1} } } g = \map {I_G} g = \map {\kappa_{x^{-1} } \circ \kappa_x} g$ +Hence the result. +{{qed}} +[[Category:Inner Automorphisms]] +se1c1eh10u9lnbhu4qlivuoylagn5kf +\end{proof}<|endoftext|> +\section{Inner Automorphism Group is Isomorphic to Quotient Group with Center} +Tags: Quotient Groups, Inner Automorphisms, Group Isomorphisms + +\begin{theorem} +Let $G$ be a [[Definition:Group|group]]. +Let $\Inn G$ be the [[Definition:Inner Automorphism Group|inner automorphism group]] of $G$. +Let $\map Z G$ be the [[Definition:Center of Group|center]] of $G$. +Let $G / \map Z G$ be the [[Definition:Quotient Group|quotient group]] of $G$ by $\map Z G$. +Then $G / \map Z G \cong \Inn G$. +\end{theorem} + +\begin{proof} +Let $G$ be a [[Definition:Group|group]]. +Let the [[Definition:Mapping|mapping]] $\kappa: G \to \Inn G$ be defined as: +:$\map \kappa a = \kappa_a$ +where $\kappa_a$ is the [[Definition:Inner Automorphism|inner automorphism]] of $G$ given by $a$. +From [[Kernel of Inner Automorphism Group is Center]]: +:$\map \ker \kappa = \map Z G$ +and also that: +:$\Img \kappa = \Inn G$ +From the [[First Isomorphism Theorem for Groups|First Isomorphism Theorem]]: +:$\Img \kappa \cong G / \map \ker \kappa$ +Thus, as $\map \ker \kappa = \map Z G$ and $\Img \kappa = \Inn G$: +:$G / \map Z G \cong \Inn G$ +{{qed}} +\end{proof}<|endoftext|> +\section{Conjugates of Elements in Centralizer} +Tags: Conjugacy, Centralizers, Cosets + +\begin{theorem} +Let $G$ be a [[Definition:Group|group]]. +Let $\map {C_G} a$ be the [[Definition:Centralizer of Group Element|centralizer]] of $a$ in $G$. +Then $\forall g, h \in G: g a g^{-1} = h a h^{-1}$ {{iff}} $g$ and $h$ belong to the same [[Definition:Left Coset|left coset]] of $\map {C_G} a$. +\end{theorem} + +\begin{proof} +The [[Definition:Centralizer of Group Element|centralizer]] of $a$ in $G$ is defined as: +:$\map {C_G} a = \set {x \in G: x \circ a = a \circ x}$ +Let $g, h \in G$. +Then: +{{begin-eqn}} +{{eqn | l = g a g^{-1} + | r = h a h^{-1} + | c = +}} +{{eqn | ll= \leadstoandfrom + | l = g^{-1} \paren {g a g^{-1} } h + | r = g^{-1} \paren {h a h^{-1} } h + | c = +}} +{{eqn | ll= \leadstoandfrom + | l = a g^{-1} h + | r = g^{-1} h a + | c = +}} +{{eqn | ll= \leadstoandfrom + | l = g^{-1} h + | o = \in + | r = \map {C_G} a + | c = {{Defof|Centralizer of Group Element}} +}} +{{end-eqn}} +By [[Elements in Same Left Coset iff Product with Inverse in Subgroup]]: +:$g$ and $h$ belong to the same [[Definition:Left Coset|left coset]] of $\map {C_G} a$ {{iff}} $g^{-1} h \in \map {C_G} a$. +The result follows. +{{Qed}} +\end{proof}<|endoftext|> +\section{Number of Conjugates is Number of Cosets of Centralizer} +Tags: Conjugacy, Centralizers, Cosets + +\begin{theorem} +Let $G$ be a [[Definition:Group|group]]. +Let $\map {C_G} a$ be the [[Definition:Centralizer of Group Element|centralizer]] of $a$ in $G$. +Then the number of different [[Definition:Conjugate of Group Element|conjugates]] of $a$ in $G$ equals the number of different [[Definition:Left Coset|(left) cosets]] of $\map {C_G} a$: +:$\card {\conjclass a} = \index G {\map {C_G} a}$ +where: +:$\conjclass a$ is the [[Definition:Conjugacy Class|conjugacy class]] of $a$ in $G$ +:$\index G {\map {C_G} a}$ is the [[Definition:Index of Subgroup|index of $\map {C_G} a$ in $G$]]. +Consequently: +:$\card {\conjclass a} \divides \order G$ +\end{theorem} + +\begin{proof} +Let $x, y \in \conjclass a$. +By definition of $\conjclass a$: +:$x a x^{-1} = y a y^{-1}$ +By [[Conjugates of Elements in Centralizer]], this is the case {{iff}} $x$ and $y$ belong to the same [[Definition:Left Coset|left coset]] of $\map {C_G} a$. +Hence: +:$\card {\conjclass a} = \index G {\map {C_G} a}$ +It follows from [[Lagrange's Theorem (Group Theory)|Lagrange's Theorem]] that: +:$\card {\conjclass a} \divides \order G$ +{{qed}} +\end{proof}<|endoftext|> +\section{Size of Conjugacy Class is Index of Normalizer} +Tags: Conjugacy Classes, Normalizers + +\begin{theorem} +Let $G$ be a [[Definition:Group|group]]. +Let $x \in G$. +Let $\conjclass x$ be the [[Definition:Conjugacy Class|conjugacy class]] of $x$ in $G$. +Let $\map {N_G} x$ be the [[Definition:Normalizer|normalizer of $x$ in $G$]]. +Let $\index G {\map {N_G} x}$ be the [[Definition:Index of Subgroup|index of $\map {N_G} x$ in $G$]]. +The number of elements in $\conjclass x$ is $\index G {\map {N_G} x}$. +\end{theorem} + +\begin{proof} +The number of elements in $\conjclass x$ is the number of [[Definition:Conjugate of Group Subset|conjugates]] of the set $\set x$. +From [[Number of Distinct Conjugate Subsets is Index of Normalizer]], the number of distinct [[Definition:Subset|subsets]] of a $G$ which are [[Definition:Conjugate of Group Subset|conjugates]] of $S \subseteq G$ is $\index G {\map {N_G} S}$. +The result follows. +{{qed}} +\end{proof}<|endoftext|> +\section{Conjugacy Class of Element of Center is Singleton} +Tags: Conjugacy Classes, Centers of Groups, Singletons + +\begin{theorem} +Let $G$ be a [[Definition:Group|group]]. +Let $\map Z G$ denote the [[Definition:Center of Group|center]] of $G$. +The [[Definition:Element|elements]] of $\map Z G$ form [[Definition:Singleton|singleton]] [[Definition:Conjugacy Class|conjugacy classes]], and the elements of $G \setminus \map Z G$ belong to multi-element [[Definition:Conjugacy Class|conjugacy classes]]. +\end{theorem} + +\begin{proof} +Let $\conjclass a$ be the [[Definition:Conjugacy Class|conjugacy class]] of $a$ in $G$. +{{begin-eqn}} +{{eqn | l = a + | o = \in + | r = \map Z G + | c = +}} +{{eqn | ll= \leadstoandfrom + | l = \forall x \in G: x a + | r = a x + | c = +}} +{{eqn | ll= \leadstoandfrom + | l = \forall x \in G: x a x^{-1} + | r = a + | c = +}} +{{eqn | ll= \leadstoandfrom + | l = \conjclass a + | r = \set a + | c = +}} +{{end-eqn}} +{{qed}} +\end{proof}<|endoftext|> +\section{Conjugacy Class Equation} +Tags: Conjugacy, Normal Subgroups, Named Theorems, Normalizers, Conjugacy Class Equation, Conjugacy Classes + +\begin{theorem} +Let $G$ be a [[Definition:Group|group]]. +Let $\order G$ denote the [[Definition:Order of Structure|order]] of $G$. +Let $\map Z G$ denote the [[Definition:Center of Group|center]] of $G$. +Let $x \in G$. +Let $\map {N_G} x$ denote the [[Definition:Normalizer|normalizer of $x$ in $G$]]. +Let $\index G {\map {N_G} x}$ denote the [[Definition:Index of Subgroup|index of $\map {N_G} x$ in $G$]]. +Let $m$ be the number of [[Definition:Singleton|non-singleton]] [[Definition:Conjugacy Class|conjugacy classes]] of $G$. +Let $x_j: j \in \set {1, 2, \ldots, m}$ be arbitrary [[Definition:Element|elements]] of those [[Definition:Conjugacy Class|conjugacy classes]]. +Then: +:$\displaystyle \order G = \order {\map Z G} + \sum_{j \mathop = 1}^m \index G {\map {N_G} {x_j} }$ +\end{theorem} + +\begin{proof} +From [[Conjugacy Class of Element of Center is Singleton]], all [[Definition:Element|elements]] of $\map Z G$ form their own [[Definition:Singleton|singleton]] [[Definition:Conjugacy Class|conjugacy classes]]. +=== Abelian Group === +Suppose $G$ is [[Definition:Abelian Group|abelian]]. +Then from [[Group equals Center iff Abelian]] we have: +:$\map Z G = G$ +So there are as many [[Definition:Conjugacy Class|conjugacy classes]] as there are [[Definition:Element|elements]] in $\map Z G$ and hence in $G$. +So in this case the result certainly holds. +{{qed|lemma}} +=== Non-Abelian Group === +Now suppose $G$ is non-abelian. +Thus: +:$\map Z G \ne G$ +and therefore: +:$G \setminus \map Z G \ne \O$ +From [[Conjugacy Class of Element of Center is Singleton]], all the non-[[Definition:Singleton|singleton]] [[Definition:Conjugacy Class|conjugacy classes]] of $G$ are in $G \setminus \map Z G$. +From the way the theorem has been worded, there are $m$ of them. +Let us choose one element from each of the non-[[Definition:Singleton|singleton]] [[Definition:Conjugacy Class|conjugacy classes]] and call them $x_1, x_2, \ldots, x_m$. +Thus, these [[Definition:Conjugacy Class|conjugacy classes]] can be written: +:$\conjclass {x_1}, \conjclass {x_2}, \ldots, \conjclass {x_m}$ +So: +:$\displaystyle \order {G \setminus \map Z G} = \sum_{j \mathop = 1}^m \order {\conjclass {x_j} }$ +or: +:$\displaystyle \order G - \order {\map Z G} = \sum_{j \mathop = 1}^m \order {\conjclass {x_j} }$ +From [[Size of Conjugacy Class is Index of Normalizer]]: +:$\order {\conjclass {x_j} } = \index G {\map {N_G} {x_j} }$ +and the result follows. +{{Qed}} +\end{proof} + +\begin{proof} +Let the distinct [[Definition:Orbit (Group Theory)|orbits]] of $G$ under the [[Definition:Conjugacy Action|conjugacy action]] be: +:$\Orb {x_1}, \Orb {x_2}, \ldots, \Orb {x_s}$ +Then from the [[Partition Equation]]: +:$\order G = \order {\Orb {x_1} } + \order {\Orb {x_2} } + \cdots + \order {\Orb {x_s} }$ +From the [[Orbit-Stabilizer Theorem]]: +:$\order {\Orb {x_i} } \divides \order G, i = 1, \ldots, s$ +The result follows from the definition of the [[Definition:Conjugacy Action|conjugacy action]]. +{{Qed}} +\end{proof}<|endoftext|> +\section{Group of Order Prime Squared is Abelian} +Tags: Order of Groups, Abelian Groups, P-Groups, Groups of Order p^2 + +\begin{theorem} +A [[Definition:Group|group]] whose [[Definition:Order of Structure|order]] is the [[Definition:Square Number|square]] of a [[Definition:Prime Number|prime]] is [[Definition:Abelian Group|abelian]]. +\end{theorem} + +\begin{proof} +Let $G$ be a [[Definition:Group|group]] of order $p^2$, where $p$ is [[Definition:Prime Number|prime]]. +Let $\map Z G$ be the [[Definition:Center of Group|center]] of $G$. +By [[Center of Group is Subgroup]], $\map Z G$ is a [[Definition:Subgroup|subgroup]] of $G$. +By [[Lagrange's Theorem (Group Theory)|Lagrange's Theorem]]: +:$\order {\map Z G} \divides \order G$ +It follows that $\order {\map Z G} = 1, p$ or $p^2$. +By [[Center of Group of Prime Power Order is Non-Trivial]]: +:$\order {\map Z G} \ne 1$ +Suppose $\order {\map Z G} = p$. +Then: +{{begin-eqn}} +{{eqn | l = \order {\map Z G} + | r = \index G {\map Z G} + | c = {{Defof|Index of Subgroup}} +}} +{{eqn | r = \order G / \order {\map Z G} + | c = [[Lagrange's Theorem (Group Theory)|Lagrange's Theorem]] +}} +{{eqn | r = p^2 / p + | c = +}} +{{eqn | r = p + | c = +}} +{{end-eqn}} +So $G / \map Z G$ is [[Definition:Non-Trivial Group|non-trivial]], and of [[Definition:Prime Number|prime]] [[Definition:Order of Structure|order]]. +From [[Prime Group is Cyclic]], $G / \map Z G$ is a [[Definition:Cyclic Group|cyclic group]]. +But by [[Quotient of Group by Center Cyclic implies Abelian]], that cannot be the case. +Therefore $\order {\map Z G} = p^2$ and therefore $\map Z G = G$. +Therefore $G$ is [[Definition:Abelian Group|abelian]]. +{{qed}} +\end{proof}<|endoftext|> +\section{Center of Group of Prime Power Order is Non-Trivial} +Tags: Group Theory, P-Groups, Centers of Groups + +\begin{theorem} +Let $G$ be a [[Definition:Group|group]] whose [[Definition:Order of Structure|order]] is the [[Definition:Prime Power|power of a prime]]. +Then the [[Definition:Center of Group|center]] of $G$ is [[Definition:Trivial Group|non-trivial]]: +:$\forall G: \order G = p^r: p \in \mathbb P, r \in \N_{>0}: \map Z G \ne \set e$ +\end{theorem} + +\begin{proof} +Suppose $G$ is [[Definition:Abelian Group|abelian]]. +From [[Group equals Center iff Abelian]]: +: $\map Z G = G$ +and the result is seen to be true as $G$ is itself [[Definition:Non-Trivial Group|non-trivial]]. +From [[Prime Group is Cyclic]] and [[Cyclic Group is Abelian]], this will always be the case for $r = 1$. +So, suppose $G$ is non-[[Definition:Abelian Group|abelian]]. +Thus $\map Z G \ne G$ and therefore $G \setminus \map Z G \ne \O$. +Let $\conjclass {x_1}, \conjclass {x_2}, \ldots, \conjclass {x_m}$ be the [[Definition:Conjugacy Class|conjugacy classes]] into which $G \setminus \map Z G$ is [[Definition:Partition (Set Theory)|partitioned]]. +From [[Conjugacy Class of Element of Center is Singleton]], all of these will have more than one element. +From the [[Conjugacy Class Equation]]: +:$\displaystyle \order {\map Z G} = \order G - \sum_{j \mathop = 1}^m \order {\conjclass {x_j} }$ +From [[Number of Conjugates is Number of Cosets of Centralizer]]: +:$\order {\conjclass {x_j} } \divides \order G$ +Let $\map {N_G} x$ be the [[Definition:Normalizer|normalizer of $x$ in $G$]]. +Then: +:$\forall j: 1 \le j \le m: \index G {\map {N_G} {x_j} } > 1 \implies p \divides \index G {\map {N_G} {x_j} }$ +Since $p \divides \order G$, it follows that: +: $p \divides \order {\map Z G}$ +and the result follows. +{{Qed}} +\end{proof}<|endoftext|> +\section{Center of Group of Order Prime Cubed} +Tags: Normal Subgroups, Centers of Groups, P-Groups + +\begin{theorem} +Let $G$ be a [[Definition:Group|group]] of [[Definition:Order of Structure|order]] $p^3$, where $p$ is a [[Definition:Prime Number|prime]]. +Let $\map Z G$ be the [[Definition:Center of Group|center]] of $G$. +Then $\order {\map Z G} \ne p^2$. +\end{theorem} + +\begin{proof} +{{AimForCont}} $\order {\map Z G} = p^2$. +Then $\order {G / \map Z G} = p$. +From [[Prime Group is Cyclic]] it follows that $G / \map Z G$ is [[Definition:Cyclic Group|cyclic]]. +From [[Quotient of Group by Center Cyclic implies Abelian]] it follows that $G$ is [[Definition:Abelian Group|abelian]]. +From [[Group equals Center iff Abelian]] it follows that $\order {\map Z G} = p^3$. +The result follows by [[Proof by Contradiction]]. +{{qed}} +{{expand|This results generalizes naturally to a group of order $p^n$: its center is not of order $p^{n-1}$.}} +[[Category:Normal Subgroups]] +[[Category:Centers of Groups]] +[[Category:P-Groups]] +37p385xpkzyb8mfwohstvgjm9u679z5 +\end{proof}<|endoftext|> +\section{Prime Power Group has Non-Trivial Proper Normal Subgroup} +Tags: Normal Subgroups, P-Groups + +\begin{theorem} +Let $G$ be a [[Definition:Group|group]], whose [[Definition:Identity Element|identity]] is $e$, such that $\order G = p^n: n > 1, p \in \mathbb P$. +Then $G$ has a [[Definition:Proper Subgroup|proper]] [[Definition:Normal Subgroup|normal subgroup]] which is [[Definition:Non-Trivial Group|non-trivial]]. +\end{theorem} + +\begin{proof} +From [[Center of Group is Normal Subgroup]], $\map Z G \lhd G$. +By [[Center of Group of Prime Power Order is Non-Trivial]], $\map Z G$ is [[Definition:Non-Trivial Group|non-trivial]]. +If $\map Z G$ is a [[Definition:Proper Subgroup|proper subgroup]], the proof is finished. +Otherwise, $\map Z G = G$. +Then $G$ is [[Definition:Abelian Group|abelian]] by [[Group equals Center iff Abelian]]. +However, then any $a \in G: a \ne e$ generates a [[Definition:Non-Trivial Group|non-trivial]] [[Definition:Normal Subgroup|normal subgroup]] $\gen a$, as [[Subgroup of Abelian Group is Normal]]. +If $\order a = \order {\gen a} < \order G$, the proof is complete. +Otherwise: +:$\order a = \order G = p^n$ +Then $\order {a^p} = p^{n - 1}$ from [[Subgroup of Finite Cyclic Group is Determined by Order]], so $a^p$ generates that [[Definition:Non-Trivial Group|non-trivial]] [[Definition:Normal Subgroup|normal subgroup]]. +{{qed}} +\end{proof}<|endoftext|> +\section{Composition Series of Group of Prime Power Order} +Tags: Cyclic Groups, Normal Series, P-Groups + +\begin{theorem} +Let $G$ be a [[Definition:Group|group]] whose [[Definition:Identity Element|identity]] is $e$, and whose [[Definition:Order of Structure|order]] is a [[Definition:Prime Power|prime power]]: +:$\order G = p^n, p \in \mathbb P, n \ge 1$ +Then $G$ has a [[Definition:Composition Series|composition series]]: +:$\set e = G_0 \subset G_1 \subset \ldots \subset G_n = G$ +such that $\order {G_k} = p^k$, $G_k \lhd G_{k + 1}$ and $G_{k + 1} / G_k$ is [[Definition:Cyclic Group|cyclic]] and of [[Definition:Order of Structure|order]] $p$. +\end{theorem} + +\begin{proof} +To be proved by [[Second Principle of Mathematical Induction|induction]] on $n$. +Let $P_n$ be the [[Definition:Proposition|proposition]] for $\order G = p^n$. +=== Basis for the Induction === +$P_1$ is trivially true because: +:$\set e = G_0 \subset G_1 = G$ +From [[Prime Group is Cyclic]], a [[Definition:Group|group]] whose [[Definition:Order of Structure|order]] is [[Definition:Prime Number|prime]] is [[Definition:Cyclic Group|cyclic]]. +=== Induction Hypothesis === +Suppose $P_k$ is true for all [[Definition:Group|groups]] of order $p^k$ for all $k < n$. +We need to show that this implies $P_{k + 1}$ is true. +=== Induction Step === +Let $G$ be a [[Definition:Group|group]] of [[Definition:Order of Structure|order]] $p^n$. +By [[Prime Power Group has Non-Trivial Proper Normal Subgroup]], $G$ has a [[Definition:Proper Subgroup|proper]] [[Definition:Trivial Subgroup|non-trivial]] [[Definition:Normal Subgroup|normal subgroup]]. +There will be a [[Definition:Finite Set|finite number]] of these, so we are free to pick one of maximal [[Definition:Order of Structure|order]]. +We call this $H$, such that $\order H = p^t, t < n$. +We need to show that $t = n - 1$. +Suppose $t < n - 1$. +Then $G / H$ is a [[Definition:Group|group]] of [[Definition:Order of Structure|order]] $p^{n - t} \ge p^2$. +Again by [[Prime Power Group has Non-Trivial Proper Normal Subgroup]], $G / H$ has a [[Definition:Proper Subgroup|proper]] [[Definition:Trivial Subgroup|non-trivial]] [[Definition:Normal Subgroup|normal subgroup]], which we will call $N$. +Let $H' = \set {g \in G: g H \in N}$. +We now show that $H \lhd G$. +Let $g, g' \in H'$. +Then $g H, g' H \in N$. +Since $N < G / H$: +: $\paren {g H} \paren {g' H} = g g' H \in N$ +and so $g g' \in N$. +If $g \in H$, then $g H \in N$. +Since $N < G / H$: +:$\paren {g H}^{-1} = g^{-1} H \in N$ +and so $g^{-1} \in H'$. +Next: +{{begin-eqn}} +{{eqn | l = \paren {H'}^a + | r = \set {g \in G: a g a^{-1} \in H'} + | c = +}} +{{eqn | r = \set {g \in G: a g a^{-1} H \in N} + | c = +}} +{{eqn | r = \set {g \in G: \paren {a H} \paren {g H} \paren {a H}^{-1} \in N} + | c = +}} +{{eqn | r = \set {g \in G: \paren {g H} \in N^{a H} } + | c = {{Defof|Conjugate of Group Subset}} +}} +{{eqn | r = \set {g \in G: \paren {g H} \in N} + | c = as $N$ is [[Definition:Normal Subgroup|normal]] +}} +{{eqn | r = H' + | c = +}} +{{end-eqn}} +So clearly $H' / H = N$, therefore: +:$\dfrac {\order {H'} } {\order H} = \index {H'} N = \order N \ge p$ +So: +:$\order {H'} \ge p \order H$ +contradicting the maximality of $\order H$. +It follows that $t = n - 1$. +Finally, we set $G_{n - 1} = H$ and use induction to show that $P_{n - 1}$ holds. +Since $G / H = G / G_{n - 1}$ is a [[Definition:Group|group]] of [[Definition:Order of Structure|order]] $p$, then it is automatically [[Definition:Cyclic Group|cyclic]]. +{{qed}} +\end{proof}<|endoftext|> +\section{Sum Rule for Counting} +Tags: Combinatorics, Counting Arguments + +\begin{theorem} +Let there be: +: $r_1$ different [[Definition:Object|objects]] in the [[Definition:Set|set]] $S_1$ +: $r_2$ different [[Definition:Object|objects]] in the set $S_2$ +: $\ldots$ +: $r_m$ different [[Definition:Object|objects]] in the set $S_m$. +Let $\displaystyle \bigcap_{i \mathop = 1}^m S_i = \varnothing$. +Then the number of ways to select an object from one of the $m$ sets is $\displaystyle \sum_{i \mathop = 1}^m r_i$. +\end{theorem} + +\begin{proof} +A direct application of [[Cardinality of Set Union]]. +{{qed}} +\end{proof}<|endoftext|> +\section{Product Rule for Counting} +Tags: Combinatorics, Counting Arguments, Product Rule for Counting + +\begin{theorem} +Let it be possible to choose an [[Definition:Element|element]] $\alpha$ from a given [[Definition:Set|set]] $S$ in $m$ different ways. +Let it be possible to choose an [[Definition:Element|element]] $\beta$ from a given [[Definition:Set|set]] $T$ in $n$ different ways. +Then the [[Definition:Ordered Pair|ordered pair]] $\tuple {\alpha, \beta}$ can be chosen from the [[Definition:Cartesian Product|cartesian product]] $S \times T$ in $m n$ different ways. +\end{theorem} + +\begin{proof} +{{handwaving}} +The validity of this rule follows directly from the definition of [[Definition:Integer Multiplication|multiplication of integers]]. +The product $a b$ (for $a, b \in \N_{>0}$) is the number of [[Definition:Sequence|sequences]] $\sequence {A, B}$, where $A$ can be any one of $a$ items and $B$ can be any one of $b$ items. +{{qed}} +\end{proof}<|endoftext|> +\section{Commutativity of Group Direct Product} +Tags: Group Direct Products + +\begin{theorem} +Let $\struct {G, \circ_g}$ and $\struct {H, \circ_h}$ be [[Definition:Group|groups]]. +Let $\struct {G \times H, \circ}$ be the [[Definition:Group Direct Product|group direct product]] of $\struct {G, \circ_g}$ and $\struct {H, \circ_h}$, where the operation $\circ$ is defined as: +:$\tuple {g_1, h_1} \circ \tuple {g_2, h_2} = \tuple {g_1 \circ_g g_2, h_1 \circ_h h_2}$ +Let $\struct {H \times G, \star}$ be the [[Definition:Group Direct Product|group direct product]] of $\struct {H, \circ_h}$ and $\struct {G, \circ_g}$, where the operation $\star$ is defined as: +:$\tuple {h_1, g_1} \star \tuple {h_2, g_2} = \tuple {h_1 \circ_h h_2, g_1 \circ_g g_2}$ +The [[Definition:Group Direct Product|group direct product]] $\struct {G \times H, \circ}$ is [[Definition:Group Isomorphism|isomorphic]] to the $\struct {H \times G, \star}$. +\end{theorem} + +\begin{proof} +The [[Definition:Mapping|mapping]] $\theta: G \times H \to H \times G$ defined as: +:$\forall g \in G, h \in H: \map \theta {g, h} = \tuple {h, g}$ +is to be shown to be a [[Definition:Group Homomorphism|group homomorphism]], and that $\theta$ is [[Definition:Bijection|bijective]], as follows: +=== Injective === +{{begin-eqn}} +{{eqn | l = \map \theta {g_1, h_1} + | r = \map \theta {g_2, h_2} + | c = +}} +{{eqn | ll= \leadsto + | l = \tuple {h_1, g_1} + | r = \tuple {h_2, g_2} + | c = Definition of $\theta$ +}} +{{eqn | ll= \leadsto + | l = \tuple {g_1, h_1} + | r = \tuple {g_2, h_2} + | c = [[Equality of Ordered Pairs]] +}} +{{end-eqn}} +Thus $\theta$ is [[Definition:Injection|injective]] by definition. +{{qed|lemma}} +=== Surjective === +{{begin-eqn}} +{{eqn | l = \tuple {h, g} + | o = \in + | r = H \times G + | c = +}} +{{eqn | ll= \leadsto + | l = \tuple {g, h} + | o = \in + | r = G \times H + | c = +}} +{{eqn | ll= \leadsto + | lo= \exists \tuple {g, h} \in G \times H: + | l = \tuple {h, g} + | r = \map \theta {g, h} + | c = +}} +{{end-eqn}} +Thus $\theta$ is [[Definition:Surjection|surjective]] by definition. +{{qed|lemma}} +=== Group Homomorphism === +Let $\tuple {g_1, h_1}, \tuple {g_2, h_2} \in G \times H$. +Then: +{{begin-eqn}} +{{eqn | l = \map \theta {\tuple {g_1, h_1} \circ \tuple {g_2, h_2} } + | r = \map \theta {g_1 \circ_g g_2, h_1 \circ_h h_2} + | c = Definition of $\circ$ +}} +{{eqn | r = \tuple {h_1 \circ_h h_2, g_1 \circ_g g_2} + | c = Definition of $\theta$ +}} +{{eqn | r = \tuple {h_1, g_1} \star \tuple {h_2, g_2} + | c = Definition of $\star$ +}} +{{eqn | r = \map \theta {g_1, h_1} \star \map \theta {g_2, h_2} + | c = Definition of $\theta$ +}} +{{end-eqn}} +thus proving that $\theta$ is a [[Definition:Group Homomorphism|homomorphism]]. +{{Qed}} +\end{proof}<|endoftext|> +\section{External Direct Product of Abelian Groups is Abelian Group} +Tags: Group Direct Products, Abelian Groups + +\begin{theorem} +Let $\struct {G, \circ_1}$ and $\struct {H, \circ_2}$ be [[Definition:Group|groups]]. +Then the [[Definition:Group Direct Product|group direct product]] $\struct {G \times H, \circ}$ is [[Definition:Abelian Group|abelian]] {{iff}} both $\struct {G, \circ_1}$ and $\struct {H, \circ_2}$ are [[Definition:Abelian Group|abelian]]. +\end{theorem} + +\begin{proof} +Let $\struct {G, \circ_1}$ and $\struct {H, \circ_2}$ be [[Definition:Group|groups]] whose [[Definition:Identity Element|identities]] are $e_G$ and $e_H$ respectively. +Suppose $\struct {G, \circ_1}$ and $\struct {H, \circ_2}$ are both [[Definition:Abelian Group|abelian]]. +Then from [[External Direct Product Commutativity]], $\struct {G \times H, \circ}$ is also [[Definition:Abelian Group|abelian]]. +Now suppose that $\struct {G \times H, \circ}$ is [[Definition:Abelian Group|abelian]]. +Then: +{{begin-eqn}} +{{eqn | l = \tuple {g_1 \circ_1 g_2, e_H} + | r = \tuple {g_1 \circ_1 g_2, e_H \circ_2 e_H} + | c = Definition of $e_H$ +}} +{{eqn | r = \tuple {g_1, e_H} \circ \tuple {g_2, e_H} + | c = {{Defof|Group Direct Product}} +}} +{{eqn | r = \tuple {g_2, e_H} \circ \tuple {g_1, e_H} + | c = as $\struct {G \times H, \circ}$ is [[Definition:Abelian Group|abelian]] +}} +{{eqn | r = \tuple {g_2 \circ_1 g_1, e_H \circ_2 e_H} + | c = {{Defof|Group Direct Product}} +}} +{{eqn | r = \tuple {g_2 \circ_1 g_1, e_H} + | c = Definition of $e_H$ +}} +{{end-eqn}} +Thus: +: $g_1 \circ_1 g_2 = g_2 \circ_1 g_1$ +and $\struct {G, \circ_1}$ is seen to be [[Definition:Abelian Group|abelian]]. +The same argument holds for $\struct {H, \circ_2}$. +{{qed}} +\end{proof}<|endoftext|> +\section{Associativity of Group Direct Product} +Tags: Group Direct Products, Associativity + +\begin{theorem} +The [[Definition:Group Direct Product|group direct product]] $G \times \paren {H \times K}$ is [[Definition:Group Isomorphism|(group) isomorphic]] to $\paren {G \times H} \times K$. +\end{theorem} + +\begin{proof} +Let $G, H, K$ be [[Definition:Group|groups]]. +The [[Definition:Mapping|mapping]] $\theta: G \times \paren {H \times K} \to \paren {G \times H} \times K$ defined as: +:$\forall g \in G, h \in H, k \in K: \map \theta {\tuple {g, \tuple {h, k} } } = \tuple {\tuple {g, h}, k}$ +is to be shown to be a [[Definition:Group Homomorphism|group homomorphism]], and that $\theta$ is [[Definition:Bijection|bijective]], as follows: +=== Injective === +Let $\map \theta {\tuple {g_1, \tuple {h_1, k_1} } } = \map \theta {\tuple {g_2, \tuple {h_2, k_2} } }$. +By the definition of $\theta$: +:$\tuple {\tuple {g_1, h_1}, k_1} = \tuple {\tuple {g_2, h_2}, k_2}$ +By [[Equality of Ordered Pairs]]: +:$\tuple {g_1, h_1} = \tuple {g_2, h_2}$ +:$k_1 = k_2$ +and consequently: +:$g_1 = g_2, h_1 = h_2$ +Thus: +:$\tuple {g_1, \tuple {h_1, k_1} } = \tuple {g_2, \tuple {h_2, k_2} }$ +and so $\theta$ is seen to be [[Definition:Injection|injective]]. +=== Surjective === +If $\tuple {\tuple {g, h}, k} \in \paren {G \times H} \times K$, then $g \in G, h \in H, k \in K$. +Thus: +:$\tuple {g, \tuple {h, k} } = \in G \times \paren {H \times K}$ +and: +:$\map \theta {\tuple {g, \tuple {h, k} } } = \tuple {\tuple {g, h}, k}$ +and so $\theta$ is seen to be [[Definition:Surjection|surjective]]. +=== Group Homomorphism === +Now let $\tuple {g_1, \tuple {h_1, k_1} }, \tuple {g_2, \tuple {h_2, k_2} } \in G \times \tuple {H \times K}$. +Then: +{{begin-eqn}} +{{eqn | l = \map \theta {\tuple {g_1, \tuple {h_1, k_1} } \tuple {g_2, \tuple {h_2, k_2} } } + | r = \map \theta {\tuple {g_1 g_2, \tuple {h_1, k_1} \tuple {h_2, k_2} } } + | c = +}} +{{eqn | r = \map \theta {\tuple {g_1 g_2, \tuple {h_1 h_2, k_1 k_2} } } + | c = +}} +{{eqn | r = \tuple {\tuple {g_1 g_2, h_1 h_2}, k_1 k_2} + | c = +}} +{{eqn | r = \tuple {\tuple {g_1, h_1} \tuple {g_2, h_2}, k_1 k_2} + | c = +}} +{{eqn | r = \tuple {\tuple {g_1, h_1}, k_1} \tuple {\tuple {g_2, h_2}, k_2} + | c = +}} +{{eqn | r = \map \theta {\tuple {g_1, \tuple {h_1, k_1} } } \map \theta {\tuple {g_2, \tuple {h_2, k_2} } } + | c = +}} +{{end-eqn}} +showing that $\theta$ is a [[Definition:Group Homomorphism|(group) homomorphism]]. +The result follows. +{{qed}} +[[Category:Group Direct Products]] +[[Category:Associativity]] +0e07ovfdbn3d1ksqoxy8c7rugsa4xz5 +\end{proof}<|endoftext|> +\section{Group Direct Product of Cyclic Groups} +Tags: Group Direct Products, Cyclic Groups, Group Direct Product of Cyclic Groups + +\begin{theorem} +Let $G$ and $H$ both be [[Definition:Finite Group|finite]] [[Definition:Cyclic Group|cyclic groups]] with [[Definition:Order of Group|orders]] $n = \order G$ and $m = \order H$ respectively. +Then their [[Definition:Group Direct Product|group direct product]] $G \times H$ is [[Definition:Cyclic Group|cyclic]] {{iff}} $g$ and $h$ are [[Definition:Coprime Integers|coprime]], that is, $g \perp h$. +\end{theorem} + +\begin{proof} +Let $G$ and $H$ be [[Definition:Group|groups]] whose [[Definition:Identity Element|identities]] are $e_G$ and $e_H$ respectively. +=== Necessary condition === +Suppose: +: $(1): \quad \order G = n, G = \gen x$ +: $(2): \quad \order H = m, H = \gen y$ +: $(3): \quad m \perp n$ +Then: +{{begin-eqn}} +{{eqn | l = \order {\tuple {x, y} } + | r = k + | c = +}} +{{eqn | ll= \leadsto + | l = \tuple {x, y}^k + | r = e_{G \times H} = \tuple {x^k, y^k} + | c = [[Powers of Elements in Group Direct Product]] +}} +{{eqn | ll= \leadsto + | l = x^k + | r = e_G, y^k = e_H + | c = +}} +{{eqn | ll= \leadsto + | l = n + | o = \divides + | r = k, m \divides k + | c = +}} +{{eqn | ll= \leadsto + | l = \lcm \set {n, m} + | o = \divides + | r = k + | c = +}} +{{eqn | ll= \leadsto + | l = n m + | o = \divides + | r = k + | c = as $n \perp m$ +}} +{{end-eqn}} +But then: +:$\tuple {x, y}^{n m} = e_{G \times H} = \tuple {x^{n m}, y^{n m} }$ +Thus: +:$k \divides n m$ +So: +:$\order {\tuple {x, y} } = n m \implies \gen {\tuple {x, y} } = G \times H$ +{{qed|lemma}} +=== Sufficient condition === +Suppose that $G \times H$ is [[Definition:Cyclic Group|cyclic]]. +Let $\tuple {x, y}$ be a [[Definition:Generator of Group|generator]] of $G \times H$. +By [[Cardinality of Cartesian Product]] the [[Definition:Order of Group|order]] of $G \times H$ is: +:$\order G \cdot \order H = g h$ +Therefore by [[Order of Cyclic Group equals Order of Generator]]: +:$\order {\tuple {x, y} } = g h$ +On the other hand, by [[Order of Group Element in Group Direct Product]] we have: +:$\order {\tuple {x, y} } = \lcm \set {\tuple {\order x, \order y} }$ +Next we claim that $x$ generates $G$. +Let $x' \in G$. +Then: +:$\tuple {x', e_H} \in G \times H$ +so there exists $k \in \N$ such that: +:$\tuple {x, y}^k = \tuple {x^k, y^k} = \tuple {x', e_H}$ +and therefore $x^k = x'$. +Thus the powers of $x$ generate the whole group $G$. +In the same way, it is seen that $y$ generates $H$. +Therefore by [[Order of Cyclic Group equals Order of Generator]]: +:$\order x = g$ +:$\order y = h$ +Thus we have that: +:$g h = \order {\tuple {x, y} } = \lcm \set {g, h}$ +Moreover by [[Product of GCD and LCM]] we have that: +:$\lcm \set {g, h} = \dfrac {g h} {\gcd \set {g, h} }$ +These two equalities imply that: +:$\gcd \set {g, h} = 1$ +That is, $g$ and $h$ are [[Definition:Coprime Integers|coprime]]. +{{qed}} +\end{proof}<|endoftext|> +\section{Group Direct Product of Infinite Cyclic Groups} +Tags: Group Direct Products, Cyclic Groups + +\begin{theorem} +The [[Definition:Group Direct Product|group direct product]] of two [[Definition:Infinite Cyclic Group|infinite cyclic groups]] is not [[Definition:Cyclic Group|cyclic]]. +\end{theorem} + +\begin{proof} +Let $G_1 = \struct {G_1, \circ_1}$ and $G_2 = \struct {G_2, \circ_2}$ be [[Definition:Infinite Cyclic Group|infinite cyclic groups]]. +Let $G = \struct {G, \circ} = G_1 \times G_2$. +Let $G_1 = \gen {g_1}, G_2 = \gen {g_2}$. +From [[Generators of Infinite Cyclic Group]]: +:$g_1$ and $g_1^{-1}$ are the only [[Definition:Generator of Cyclic Group|generators]] of $G_1$ +:$g_2$ and $g_2^{-1}$ are the only [[Definition:Generator of Cyclic Group|generators]] of $G_2$. +So a [[Definition:Generator of Group|generator]] of $G$ must be of the form $\tuple { g_1^{\pm 1}, g_2^{\pm 1} }$ to have any hope to generate all of $G$. +{{WLOG}}, suppose that $G = \gen {\tuple {g_1, g_2} }$. +Let $e_1$ be the [[Definition:Identity Element|identity element]] of $G_1$. +Let $x_2 \in G_2$. +From the definition of an [[Definition:Infinite Cyclic Group|infinite cyclic group]], both $g_1$ and $g_2$ are of [[Definition:Order of Group Element|infinite order]]. +Suppose now $\tuple {e_1, x_2} \in \gen {\tuple {g_1, g_2} }$. +Then there is an $i \in \Z$ such that we have: +:$\tuple {g_1, g_2}^i = \tuple {g_1^i, g_2^i} = \tuple {e_1, x_2}$ +However, as $g_1$ is of [[Definition:Order of Group Element|infinite order]], it must be that $i = 0$. +Hence $x_2 = g_2^0 = e_2$, where $e_2$ is the [[Definition:Identity Element|identity element]] of $G_2$. +It follows that $\tuple {e_1, g_2} \notin \gen {\tuple {g_1, g_2} }$. +Therefore: +:$G \ne \gen {\tuple {g_1, g_2} }$ +It follows that $G$ cannot be generated by one [[Definition:Element|element]]. +{{qed}} +\end{proof}<|endoftext|> +\section{Order of Group Element in Group Direct Product} +Tags: Group Direct Products + +\begin{theorem} +Let $G$ and $H$ be [[Definition:Finite Group|finite groups]]. +Let $g \in G: \order g = m, h \in H: \order h = n$. +Then $\order {\tuple {g, h} }$ in $G \times H$ is $\lcm \set {m, n}$. +\end{theorem} + +\begin{proof} +Let $G$ and $H$ be a [[Definition:Group|groups]] whose [[Definition:Identity Element|identities]] are $e_G$ and $e_H$. +Let $l = \lcm \set {m, n}$. +From the definition of [[Definition:Lowest Common Multiple of Integers|lowest common multiple]]: +:$\exists x, y \in \Z: l = m x = n y$ +From the definition of [[Definition:Order of Group Element|order of an element]]: +:$g^m = e_G, h^n = e_H$ +Thus: +{{begin-eqn}} +{{eqn | l = \tuple {g, h}^l + | r = \tuple {g^l, h^l} + | c = +}} +{{eqn | r = \tuple {\paren {g^m}^x, \paren {h^n}^y} + | c = +}} +{{eqn | r = \tuple {e_G^x, e_H^y} + | c = +}} +{{eqn | r = \tuple {e_G, e_H} + | c = +}} +{{end-eqn}} +Now suppose $\exists k \in \Z: \tuple {g, h}^k = \tuple {e_G, e_H}$. +It follows that: +:$\tuple {g, h}^k = \tuple {g^k, h^k} = \tuple {e_G, e_H}$ +Thus: +:$g^k = e_G, h^k = e_H$ +It follows from $g^k = e_G$ and $\order g = m$, by [[Absolute Value of Integer is not less than Divisors]], that: +:$m \divides k$ +Similarly it follows that: +:$n \divides k$ +So $k$ is a [[Definition:Positive Integer|positive]] [[Definition:Common Multiple|common multiple]] of both $m$ and $n$. +Since $l$ is the [[Definition:Lowest Common Multiple of Integers|lowest common multiple]]: +:$l \le k$ +Therefore $l$ is the smallest such that $\tuple {g, h}^l = \tuple {e_G, e_H}$. +The result follows by definition of the [[Definition:Order of Group Element|order of an element]]. +{{qed}} +[[Category:Group Direct Products]] +qn3x5oeiex39ck218cbpsj3tjzn6b74 +\end{proof}<|endoftext|> +\section{Subgroup Product is Internal Group Direct Product iff Surjective} +Tags: Internal Group Direct Products + +\begin{theorem} +Let $G$ be a [[Definition:Group|group]]. +Let $\sequence {H_n}$ be a [[Definition:Sequence|sequence]] of [[Definition:Subgroup|subgroups]] of $G$. +Let $\displaystyle \phi: \prod_{k \mathop = 1}^n H_k \to G$ be a [[Definition:Mapping|mapping]] defined by: +:$\displaystyle \map \phi {h_1, h_2, \ldots, h_n} = \prod_{k \mathop = 1}^n h_k$ +Then $\phi$ is [[Definition:Surjection|surjective]] {{iff}}: +: $\displaystyle G = \prod_{k \mathop = 1}^n H_k$ +That is, {{iff}} $G$ is the [[Definition:Internal Group Direct Product|internal group direct product]] of $H_1, H_2, \ldots, H_n$. +\end{theorem} + +\begin{proof} +=== Necessary Condition === +Let $\phi$ be a [[Definition:Surjection|surjection]]. +Then $\Img \phi$ consists of all the products $\displaystyle \prod_{k \mathop = 1}^n h_k$ such that: +:$\forall k \in \closedint 1 n: h_k \in H_k$ +Thus, as $\phi$ is [[Definition:Surjection|surjective]], every [[Definition:Element|element]] of $G$ must be representable in this form. +Using the notation: +:$\displaystyle \prod_{k \mathop = 1}^n H_k = \set {\prod_{k \mathop = 1}^n h_k: \forall k \in \closedint 1 n: h_k \in H_k}$ +the result follows immediately. +Thus every [[Definition:Element|element]] of $G$ is the product of an [[Definition:Element|element]] of each of $H_k$. +{{qed|lemma}} +=== Sufficient Condition === +Let $\displaystyle G = \prod_{k \mathop = 1}^n H_k$. +If this is the case, then $g$ can be written as: +:$\displaystyle g = \prod_{k \mathop = 1}^n h_k: \forall k \in \closedint 1 n: h_k \in H_k$ +Thus: +:$g = \map \phi {h_1, h_2, \ldots, h_n}$ +and $\phi$ is [[Definition:Surjection|surjective]]. +{{qed}} +[[Category:Internal Group Direct Products]] +19kysqrglzqel7vkr389ikpxd1jxabb +\end{proof}<|endoftext|> +\section{Internal Group Direct Product is Injective} +Tags: Internal Group Direct Products, Injections + +\begin{theorem} +Let $G$ be a [[Definition:Group|group]] whose [[Definition:Identity Element|identity]] is $e$. +Let $H_1, H_2$ be [[Definition:Subgroup|subgroups]] of $G$. +Let $\phi: H_1 \times H_2 \to G$ be a [[Definition:Mapping|mapping]] defined by: +:$\map \phi {h_1, h_2} = h_1 h_2$ +Then $\phi$ is [[Definition:Injection|injective]] {{iff}}: +:$H_1 \cap H_2 = \set e$ +\end{theorem} + +\begin{proof} +=== Necessary Condition === +Let $\phi$ be an [[Definition:Injection|injection]]. +Let $\map \phi {h_1, h_2} = \map \phi {k_1, k_2}$. +As $\phi$ is [[Definition:Injection|injective]], this means that: +:$\tuple {h_1, h_2} = \tuple {k_1, k_2}$ +and thus: +:$h_1 = k_1, h_2 = k_2$ +From the definition of $\phi$, this means: +:$h_1 h_2 = k_1 k_2$ +Thus, each element of $G$ that can be expressed as a product of the form $h_1 h_2$ can be thus expressed ''uniquely''. +Now, suppose $h \in H_1 \cap H_2$. +We have: +{{begin-eqn}} +{{eqn | l = h = h e + | o = : + | r = h \in H_1, e \in H_2 + | c = +}} +{{eqn | l = h = e h + | o = : + | r = e \in H_1, h \in H_2 + | c = +}} +{{end-eqn}} +Thus we see that: +{{begin-eqn}} +{{eqn | l = \map \phi {h, e} + | r = \map \phi {e, h} + | c = +}} +{{eqn | ll= \leadsto + | l = \tuple {h, e} + | r = \tuple {e, h} + | c = +}} +{{eqn | ll= \leadsto + | l = h + | r = e + | c = +}} +{{end-eqn}} +Thus: +:$H_1 \cap H_2 = \set e$ +{{qed|lemma}} +=== Sufficient Condition === +Let $H_1 \cap H_2 = \set e$. +Let: +:$\map \phi {h_1, h_2} = \map \phi {k_1, k_2}$ +Then: +:$h_1 h_2 = k_1 k_2: h_1, k_1 \in H_1, h_2, k_2 \in H_2$ +Thus: +:$k_1^{-1} h_1 = k_2 h_2^{-1}$ +But: +:$k_1^{-1} h_1 \in H_1$ and $k_2 h_2^{-1} \in H_2$ +As they are equal, we have: +:$k_1^{-1} h_1 = k_2 h_2^{-1} \in H_1 \cap H_2 = \set e$ +It follows that: +:$h_1 = k_1, h_2 = k_2$ +and thus: +:$\tuple {h_1, h_2} = \tuple {k_1, k_2}$ +Thus $\phi$ is [[Definition:Injection|injective]] and the result follows. +{{qed}} +[[Category:Internal Group Direct Products]] +[[Category:Injections]] +8dkzl8zwmhesfwrzsqdn1xfvrontyp8 +\end{proof}<|endoftext|> +\section{Internal Group Direct Product Isomorphism} +Tags: Internal Group Direct Products, Group Isomorphisms, Normal Subgroups + +\begin{theorem} +Let $G$ be a [[Definition:Group|group]]. +Let $H_1, H_2$ be [[Definition:Subgroup|subgroups]] of $G$. +Let $\phi: H_1 \times H_2 \to G$ be the [[Definition:Mapping|mapping]] defined by $\map \phi {h_1, h_2} := h_1 h_2$. +If $\phi$ is a [[Definition:Group Isomorphism|(group) isomorphism]], then both $H_1$ and $H_2$ are [[Definition:Normal Subgroup|normal subgroups]] of $G$. +\end{theorem} + +\begin{proof} +$\phi$ is an [[Definition:Group Isomorphism|isomorphism]], so in particular a [[Definition:Group Homomorphism|(group) homomorphism]]. +Thus by [[Induced Group Product is Homomorphism iff Commutative]], every [[Definition:Element|element]] of $H_1$ [[Definition:Commuting Elements|commutes]] with every [[Definition:Element|element]] of $H_2$. +Now suppose $a \in G$. +As $\phi$ is an [[Definition:Group Isomorphism|isomorphism]], it follows that $\phi$ is [[Definition:Surjection|surjective]]. +Thus by [[Subgroup Product is Internal Group Direct Product iff Surjective]]: +:$\exists h_1 \in H_1, h_2 \in H_2: a = h_1 h_2$ +Now any [[Definition:Element|element]] of $a H_1 a^{-1}$ is in the form $a h a^{-1}$ for some $h \in H_1$. +Thus: +{{begin-eqn}} +{{eqn | l = a h a^{-1} + | r = \paren {h_1 h_2} h \paren {h_1 h_2}^{-1} + | c = +}} +{{eqn | r = h_1 h_2 h h_2^{-1} h_1^{-1} + | c = +}} +{{eqn | r = h_1 h h_2 h_2^{-1} h_1^{-1} + | c = $h \in H_1$ and $h_2 \in H_2$ [[Definition:Commuting Elements|commute]] +}} +{{eqn | r = h_1 h h_1^{-1} \in H_1 + | c = +}} +{{end-eqn}} +Thus $a H_1 a^{-1} \subseteq H_1$, and $H_1$ is therefore a [[Definition:Normal Subgroup|normal subgroup]] of $G$. +Similarly, any [[Definition:Element|element]] of $a H_2 a^{-1}$ is in the form $a h a^{-1}$ for some $h \in H_2$. +Thus: +{{begin-eqn}} +{{eqn | l = a h a^{-1} + | r = \paren {h_1 h_2} h \paren {h_1 h_2}^{-1} + | c = +}} +{{eqn | r = h_1 h_2 h h_2^{-1} h_1^{-1} + | c = +}} +{{eqn | r = h_1 h' h_1^{-1} + | c = as $h' = h_2 h h_2^{-1} \in H_2$ +}} +{{eqn | r = h' h_1 h_1^{-1} + | c = $h_1 \in H_1$ and $h' \in H_2$ [[Definition:Commuting Elements|commute]] +}} +{{eqn | r = h' \in H_2 + | c = +}} +{{end-eqn}} +Thus $a H_2 a^{-1} \subseteq H_2$ and $H_2$ is [[Definition:Normal Subgroup|normal]] as well. +{{qed}} +[[Category:Internal Group Direct Products]] +[[Category:Group Isomorphisms]] +[[Category:Normal Subgroups]] +tal7o18xi8em9ebt61g1ghj7qd3spdp +\end{proof}<|endoftext|> +\section{Internal Group Direct Product of Normal Subgroups} +Tags: Internal Group Direct Products, Normal Subgroups, Group Homomorphisms + +\begin{theorem} +Let $G$ be a [[Definition:Group|group]] whose [[Definition:Identity Element|identity]] is $e$. +Let $H_1, H_2$ be [[Definition:Subgroup|subgroups]] of $G$. +Let $\phi: H_1 \times H_2 \to G$ be a [[Definition:Mapping|mapping]] defined by $\map \phi {h_1, h_2} = h_1 h_2$. +Let $H_1$ and $H_2$ be [[Definition:Normal Subgroup|normal subgroups]] of $G$, and let $H_1 \cap H_2 = \set e$. +Then $\phi$ is a [[Definition:Group Homomorphism|(group) homomorphism]]. +\end{theorem} + +\begin{proof} +Let $H_1$ and $H_2$ be [[Definition:Normal Subgroup|normal subgroups]] of $G$. +Let $h_1 \in H_1, h_2 \in H_2$. +Consider $x \in G: x = h_1 h_2 h_1^{-1} h_2^{-1}$. +{{begin-eqn}} +{{eqn | l = x + | r = h_1 h_2 h_1^{-1} h_2^{-1} + | c = +}} +{{eqn | r = \paren {h_1 h_2 h_1^{-1} } h_2^{-1} + | c = +}} +{{end-eqn}} +As $H_2$ is [[Definition:Normal Subgroup|normal]], we have $h_1 h_2 h_1^{-1} \in H_2$ and thus $x \in H_2$. +Similarly, we can show that $x \in H_1$ and so $x \in H_1 \cap H_2$ and thus $x = e$. +From [[Product of Commuting Elements with Inverses]], $h_1 h_2 h_1^{-1} h_2^{-1} = e$ {{iff}} $h_1$ and $h_2$ [[Definition:Commuting Elements|commute]]. +Thus $h_1 h_2 = h_2 h_1$. +As $h_1$ and $h_2$ are arbitrary [[Definition:Element|elements]] of $H_1$ and $H_2$, it follows that every [[Definition:Element|element]] of $H_1$ [[Definition:Commuting Elements|commutes]] with every [[Definition:Element|element]] of $H_2$. +Thus from [[Induced Group Product is Homomorphism iff Commutative]], $\phi$ is a homomorphism. +{{qed}} +[[Category:Internal Group Direct Products]] +[[Category:Normal Subgroups]] +[[Category:Group Homomorphisms]] +dr1zduk92t4fsu9xcfdmjshzwap7ogp +\end{proof}<|endoftext|> +\section{Internal Direct Product Theorem} +Tags: Internal Group Direct Products, Normal Subgroups, Named Theorems, Internal Direct Product Theorem + +\begin{theorem} +Let $\struct {G, \circ}$ be a [[Definition:Group|group]] whose [[Definition:Identity Element|identity]] is $e$. +Let $H_1, H_2 \le G$. +Then $G$ is the [[Definition:Internal Group Direct Product|internal group direct product]] of $H_1$ and $H_2$ {{iff}}: +:$(1): \quad G = H_1 \circ H_2$ +:$(2): \quad H_1 \cap H_2 = \set e$ +:$(3): \quad H_1, H_2 \lhd G$ +where $H_1 \lhd G$ denotes that $H_1$ is a [[Definition:Normal Subgroup|normal subgroup]] of $G$. +\end{theorem} + +\begin{proof} +=== Sufficient Condition === +Let $G$ be the [[Definition:Internal Group Direct Product|internal group direct product]] of $H_1$ and $H_2$. +:$(1): \quad$ From [[Subgroup Product is Internal Group Direct Product iff Surjective]], $G = H_1 \circ H_2$. +:$(2): \quad$ From [[Internal Group Direct Product is Injective]], $H_1$ and $H_2$ are [[Definition:Independent Subgroups|independent subgroups]] of $G$. +:$(3): \quad$ From [[Internal Group Direct Product Isomorphism]], $H_1 \lhd G$ and $H_2 \lhd G$. +{{qed|lemma}} +=== Necessary Condition === +Let $C: H_1 \times H_2 \to G$ be the [[Definition:Mapping|mapping]] defined as: +:$\forall \tuple {h_1, h_2} \in H_1 \times H_2: \map C {h_1, h_2} = h_1 \circ h_2$ +Suppose the three conditions hold. +:$(1): \quad$ From [[Subgroup Product is Internal Group Direct Product iff Surjective]], $C$ is [[Definition:Surjection|surjective]]. +:$(2): \quad$ From [[Internal Group Direct Product is Injective]], $C$ is [[Definition:Injection|injective]]. +:$(3): \quad$ From [[Internal Group Direct Product of Normal Subgroups]], $C$ is a [[Definition:Group Homomorphism|group homomorphism]]. +Putting these together, we see that $C$ is a [[Definition:Bijection|bijective]] [[Definition:Group Homomorphism|homomorphism]], and therefore an [[Definition:Group Isomorphism|isomorphism]]. +So by definition, $G$ is the [[Definition:Internal Group Direct Product|internal group direct product]] of $H_1$ and $H_2$. +{{qed}} +\end{proof} + +\begin{proof} +A specific instance of the [[Internal Direct Product Theorem/General Result|general result]], with $n = 2$. +{{qed}} +\end{proof}<|endoftext|> +\section{Inclusion Mapping is Surjection iff Identity} +Tags: Surjections, Inclusion Mappings, Identity Mappings + +\begin{theorem} +Let $T$ be a [[Definition:Set|set]]. +Let $S\subseteq T$ be a [[Definition:Subset|subset]]. +Let $i_S: S \to T$ be the [[Definition:Inclusion Mapping|inclusion mapping]]. +Then: +:$i_S: S \to T$ is [[Definition:Surjection|surjective]] {{iff}} $i_S: S \to T = I_S: S \to S$ +where $I_S: S \to S$ denotes the [[Definition:Identity Mapping|identity mapping]] on $S$. +Alternatively, this theorem can be worded as: +:$i_S: S \to S = I_S: S \to S$ +It follows directly that from [[Surjection by Restriction of Codomain]], the [[Definition:Surjective Restriction|surjective restriction]] of $i_S: S \to T$ to $i_S: S \to \Img {i_S}$ is itself the [[Definition:Identity Mapping|identity mapping]]. +\end{theorem} + +\begin{proof} +It is apparent from the definitions of both the [[Definition:Inclusion Mapping|inclusion mapping]] and the [[Definition:Identity Mapping|identity mapping]] that: +:$(1): \quad \Dom {i_S} = S = \Dom {I_S}$ +:$(2): \quad \forall s \in S: \map {i_S} s = s = \map {I_S} s$ +=== Necessary Condition === +Let $i_S: S \to T = I_S: S \to S$. +From [[Equality of Mappings]], we have that the [[Definition:Codomain of Mapping|codomain]] of $i_S$ equals the [[Definition:Codomain of Mapping|codomain]] of $I_S$. +Thus the [[Definition:Codomain of Mapping|codomain]] of $i_S$ equals the [[Definition:Codomain of Mapping|codomain]] of $I_S$ equals $S$ and thus $T = S$. +So $\forall s \in S: s = \map {i_S} s$ and so $i_S$ is [[Definition:Surjection|surjective]]. +{{qed|lemma}} +=== Sufficient Condition === +Now let $i_S: S \to T$ be a [[Definition:Surjection|surjection]]. +Then: +:$\forall s \in T: s = \map {i_S} s \implies s \in S$ +and therefore: +:$T \subseteq S$ +Thus: +:$T = S$ +and so the [[Definition:Codomain of Mapping|codomain]] of $i_S$ equals the [[Definition:Codomain of Mapping|codomain]] of $I_S$ which equals $S$. +Thus $i_S: S \to T = I_S: S \to S$. +{{qed}} +\end{proof}<|endoftext|> +\section{Cyclic Group of Order 6} +Tags: Examples of Cyclic Groups, Groups of Order 6 + +\begin{theorem} +Let $C_n$ be the [[Definition:Cyclic Group|cyclic group]] of [[Definition:Order of Structure|order]] $n$. +Then: +: $C_2 \times C_3 \cong C_6$ +: $C_6$ is the [[Definition:Internal Group Direct Product|internal group direct product]] of $C_2$ and $C_3$. +\end{theorem} + +\begin{proof} +From [[Group Direct Product of Cyclic Groups]] noting that $2 \perp 3$: +:$C_2 \times C_3 \cong C_6$ +* $C_6$ is the [[Definition:Internal Group Direct Product|internal group direct product]] of $C_2$ and $C_3$: +Let $\left({C_6, \circ}\right) = \left \langle {x} \right \rangle$, let $\left({C_2, \circ}\right) = \left \langle {x^3} \right \rangle$, and let $\left({C_3, \circ}\right) = \left \langle {x^2} \right \rangle$. +From [[Subgroup of Abelian Group is Normal]], $C_2 \lhd C_6$ and $C_3 \lhd C_6$. +We can factorise the elements of $C_6$ thus: +:$e = e \circ e, x = x^3 \circ \left({x^2}\right)^2, x^2 = e \circ x^2, x^3 = x^3 \circ e, x^4 = e \circ \left({x^2}\right)^2, x^5 = x^2 \circ x^3$ +Hence the result. +{{qed}} +[[Category:Examples of Cyclic Groups]] +[[Category:Groups of Order 6]] +krip2kmx54xcemcyjmzixdpxw4ne1rd +\end{proof}<|endoftext|> +\section{Internal Direct Product Generated by Subgroups} +Tags: Internal Group Direct Products + +\begin{theorem} +Let $G$ be a [[Definition:Group|group]] whose [[Definition:Identity Element|identity]] is $e$. +Let $\sequence {H_n}$ be a [[Definition:Sequence|sequence]] of [[Definition:Subgroup|subgroups]] of $G$. +Then: +:the [[Definition:Generator of Subgroup|subgroup generated by $\displaystyle \bigcup_{k \mathop = 1}^n H_k$]] is the [[Definition:Internal Group Direct Product|internal group direct product]] of $\sequence {H_n}$ +{{iff}}: +:$\sequence {H_n}$ is an [[Definition:Independent Subgroups|independent sequence of subgroups]] such that every element of $H_i$ [[Definition:Commute|commutes]] with every element of $H_j$ whenever $1 \le i < j \le n$. +\end{theorem} + +\begin{proof} +In the following, the notation $\closedint m n$ is to be understood to mean a [[Definition:Integer Interval|(closed) integer interval]]: +:$\closedint m n := \set {x \in \Z: m \le x \le n}$ +for $m, n \in \Z$. +For each $k \in \closedint 1 n$, let $\displaystyle L_k = \prod_{j \mathop = 1}^k H_j$ be the [[Definition:Finite Cartesian Product|cartesian product]] of the subgroups $H_1, H_2, \ldots, H_k$ of $G$. +Let $\displaystyle C_k: L_k \to G: \map {C_k} {x_1, x_2, \ldots, x_k} = \prod_{j \mathop = 1}^k x_j$. +=== Necessary Condition === +Suppose that the [[Definition:Generator of Subgroup|subgroup generated by $\displaystyle \bigcup_{k \mathop = 1}^n H_k$]] is the [[Definition:Internal Group Direct Product/General Definition|internal group direct product]] of $\sequence {H_n}$. +We have that [[Internal Group Direct Product is Injective]]. +Hence by definition $C_k$ is a [[Definition:Group Monomorphism|monomorphism]]. +It follows from [[Kernel is Trivial iff Monomorphism]] that the [[Definition:Kernel of Group Homomorphism|kernel]] of $C_n$ is $\set {\tuple {e, e, \ldots, e} }$. +Therefore $\sequence {H_n}$ is an [[Definition:Independent Subgroups|independent sequence]]. +Let $x \in H_i$ and $y \in H_j$ where $1 \le i < j \le n$. +For each $k \in \closedint 1 n$, let $x_k$ and $y_k$ be defined as: +:$x_k = \begin{cases} +e & : k \ne i \\ +x & : k = i +\end{cases} +\qquad +y_k = \begin{cases} +e & : k \ne j \\ +y & : k = j +\end{cases}$ +Then: +{{begin-eqn}} +{{eqn | l = x y + | r = \paren {y_i x_i} \paren {y_j x_j} + | c = +}} +{{eqn | r = \prod_{k \mathop = 1}^n y_k x_k + | c = +}} +{{eqn | r = \map {C_n} {y_1 x_1, \ldots, y_n x_n} + | c = +}} +{{eqn | r = \map {C_n} {\tuple {y_1, \ldots, y_n} \tuple {x_1, \ldots, x_n} } + | c = +}} +{{eqn | r = \map {C_n} {y_1, \ldots, y_n} \map {C_n} {x_1, \ldots, x_n} + | c = +}} +{{eqn | r = y x + | c = +}} +{{end-eqn}} +{{qed|lemma}} +=== Sufficient Condition === +Suppose that $\sequence {H_n}$ is an [[Definition:Independent Subgroups|independent sequence of subgroups]] such that every element of $H_i$ [[Definition:Commute|commutes]] with every element of $H_j$ whenever $1 \le i < j \le n$. +Let $S$ be the set of all $k \in \closedint 1 n$ such that $C_k: L_k \to G$ is a [[Definition:Group Homomorphism|(group) homomorphism]]. +Clearly $1 \in S$. +Now let $k \in S$ such that $k < n$. +Let $\tuple {x_1, \ldots, x_k, x_{k + 1} }, \tuple {y_1, \ldots, y_k, y_{k + 1} } \in L_k$. +By the [[General Associativity Theorem]] and [[Element Commutes with Product of Commuting Elements]]: +{{begin-eqn}} +{{eqn | l = \map {C_{k + 1} } {\tuple {x_1, \ldots, x_k, x_{k + 1} } \tuple {y_1, \ldots, y_k, y_{k + 1} } } + | r = \map {C_{k + 1} } {x_1 y_1, \ldots, x_k y_k, x_{k + 1} y_{k + 1} } + | c = +}} +{{eqn | r = \paren {x_1 y_1} \cdots \paren {x_k y_k} \paren {x_{k + 1} y_{k + 1} } + | c = +}} +{{eqn | r = \map {C_k} {\paren {x_1 y_1} \cdots \paren {x_k y_k} } \paren {x_{k + 1} y_{k + 1} } + | c = +}} +{{eqn | r = \map {C_k} {\tuple {x_1, \ldots, x_k} \tuple {y_1, \ldots, y_k} } \paren {x_{k + 1} y_{k + 1} } + | c = +}} +{{eqn | r = \map {C_k} {x_1, \ldots, x_k} \map {C_k} {y_1, \ldots, y_k} \paren {x_{k + 1} y_{k + 1} } + | c = +}} +{{eqn | r = \paren {\prod_{k \mathop = 1}^n{x_j} } \paren {\prod_{k \mathop = 1}^n{y_j} } x_{k + 1} y_{k + 1} + | c = +}} +{{eqn | r = \paren {\paren {\prod_{k \mathop = 1}^n x_j} x_{k + 1} } \paren {\paren {\prod_{k \mathop = 1}^n y_j} y_{k + 1} } + | c = +}} +{{eqn | r = \map {C_{k + 1} } {x_1, \ldots, x_k, x_{k + 1} } \map {C_{k + 1} } {y_1, \ldots, y_k, y_{k + 1} } + | c = +}} +{{end-eqn}} +Thus $C_{k + 1}: L_{k + 1} \to G$ is a [[Definition:Group Homomorphism|homomorphism]], so $k + 1 \in S$. +So by [[Principle of Mathematical Induction|induction]]: +:$S = \closedint 1 n$ +In particular, $C_n: L_n \to G$ is a [[Definition:Group Homomorphism|homomorphism]]. +By [[Morphism Property Preserves Closure]] and [[Homomorphism Preserves Subsemigroups]], the [[Definition:Codomain of Mapping|codomain]] $\displaystyle \prod_{k \mathop = 1}^n H_k$ of $C_n$ is therefore a [[Definition:Subgroup|subgroup]] of $G$ containing $\displaystyle \bigcup_{k \mathop = 1}^n H_k$. +However, any [[Definition:Subgroup|subgroup]] containing $\displaystyle \bigcup_{k \mathop = 1}^n H_k$ must clearly contain $\displaystyle \prod_{k \mathop = 1}^n H_k$. +Therefore, $\displaystyle \prod_{k \mathop = 1}^n H_k$ is the [[Definition:Subgroup|subgroup]] of $G$ generated by $\displaystyle \bigcup_{k \mathop = 1}^n H_k$. +As $\sequence {H_n}$ is an [[Definition:Independent Subgroups|independent sequence of subgroups]], the [[Definition:Kernel of Group Homomorphism|kernel]] of $C_n$ is $\set {\tuple {e, \ldots, e} }$. +Hence by the [[Quotient Theorem for Group Epimorphisms]], $C_n$ from $L_n$ to the subgroup of $G$ generated by $\displaystyle \bigcup_{k \mathop = 1}^n H_k$ is an [[Definition:Group Isomorphism|isomorphism]]. +{{qed}} +\end{proof}<|endoftext|> +\section{Internal Group Direct Product Commutativity} +Tags: Internal Group Direct Products, Internal Direct Products, Commutativity, Internal Group Direct Product Commutativity + +\begin{theorem} +Let $\struct {G, \circ}$ be a [[Definition:Group|group]] whose [[Definition:Identity Element|identity]] is $e$. +Let $H_1, H_2 \le G$. +Let $\struct {G, \circ}$ be the [[Definition:Internal Group Direct Product|internal group direct product]] of $H_1$ and $H_2$. +Then: +:$\forall h_1 \in H_1, h_2 \in H_2: h_1 \circ h_2 = h_2 \circ h_1$ +\end{theorem} + +\begin{proof} +Let $\sqbrk {x, y}$ denote the [[Definition:Commutator of Group Elements|commutator]] of $x, y \in G$: +:$\sqbrk {x, y} := x^{-1} y^{-1} x y$ +We have that: +{{begin-eqn}} +{{eqn | n = 1 + | l = y x \sqbrk {x, y} + | r = y x x^{-1} y^{-1} x y + | c = {{Defof|Commutator of Group Elements}} +}} +{{eqn | r = y y^{-1} x y + | c = {{GroupAxiom|3}} +}} +{{eqn | r = x y + | c = {{GroupAxiom|3}} +}} +{{end-eqn}} +Let $h_1 \in H_1$, $h_2 \in H_2$. +We have: +{{begin-eqn}} +{{eqn | l = \sqbrk {h_1, h_2} + | r = {h_1}^{-1} {h_2}^{-1} h_1 h_2 + | c = +}} +{{eqn | r = {h_1}^{-1} \paren { {h_2}^{-1} h_1 h_2} + | c = {{GroupAxiom|1}} +}} +{{eqn | o = \in + | r = {h_1}^{-1} H_1 + | c = {{Defof|Normal Subgroup}} +}} +{{eqn | ll= \leadsto + | l = \sqbrk {h_1, h_2} + | o = \in + | r = H_1 + | c = as ${h_1}^{-1} H_1 = H_1$ +}} +{{end-eqn}} +and: +{{begin-eqn}} +{{eqn | l = \sqbrk {h_1, h_2} + | r = {h_1}^{-1} {h_2}^{-1} h_1 h_2 + | c = +}} +{{eqn | r = \paren { {h_1}^{-1} {h_2}^{-1} h_1} h_2 + | c = {{GroupAxiom|1}} +}} +{{eqn | o = \in + | r = H_2 h_2 + | c = {{Defof|Normal Subgroup}} +}} +{{eqn | ll= \leadsto + | l = \sqbrk {h_1, h_2} + | o = \in + | r = H_2 + | c = as $H_2 h_2 = H_2$ +}} +{{end-eqn}} +Thus: +:$\sqbrk {h_1, h_2} \in H_1 \cap H_2$ +But as $H_1 \cap H_2 = \set e$, it follows that: +:$\sqbrk {h_1, h_2} = e$ +It follows from [[Commutator is Identity iff Elements Commute]] that: +:$h_1 h_2 = h_2 h_1$ +and the result follows. +{{qed}} +\end{proof}<|endoftext|> +\section{Internal and External Group Direct Products are Isomorphic} +Tags: Group Direct Products, Internal Group Direct Products, Group Isomorphisms + +\begin{theorem} +Let $G$ be a [[Definition:Group|group]] whose [[Definition:Identity Element|identity]] is $e$. +Then $G$ is the [[Definition:Group Direct Product/Finite Product|(external) group direct product]] of $G_1, G_2, \ldots, G_n$ {{iff}} $G$ is the [[Definition:Internal Group Direct Product/General Definition|internal group direct product]] of $N_1, N_2, \ldots, N_n$ such that: +:$\forall i \in \N_n: N_i \cong G_i$ +where: +:$\cong$ denotes [[Definition:Group Isomorphism|(group) isomorphism]] +:$\N_n$ denotes $\set {1, 2, \ldots, n}$ +\end{theorem} + +\begin{proof} +=== Necessary Condition === +Let $G$ be the [[Definition:Group Direct Product/Finite Product|external direct product]] of [[Definition:Group|groups]] $G_1, G_2, \ldots, G_n$. +For all $i \in \N_n$, let $N_i$ be defined as the set: +:$N_i = \set e \times \cdots \times \set e \times G_i \times \set e \times \cdots \times \set e$ +of elements which have entry $e$ everywhere except possibly in the $i$th co-ordinate. +It remains to be checked that: +:$(1): \quad N_i$ is [[Definition:Group Isomorphism|isomorphic]] to $G_i$ +:$(2): \quad N_i$ is a [[Definition:Normal Subgroup|normal subgroup]] of $G$ +:$(3): \quad$ Every element of $G$ has a unique expression: +:$\tuple {g_1, \ldots, g_n} = \tuple {g_1, e, \ldots, e} \tuple {e, g_2, e \ldots e} \ldots \tuple {e, \ldots, g_n}$ +as a product of elements of $N_1, \ldots, N_n$. +==== $N_i$ is [[Definition:Group Isomorphism|isomorphic]] to $G_i$ ==== +{{finish}} +==== $N_i$ is a [[Definition:Normal Subgroup|normal subgroup]] of $G$ ==== +{{finish}} +==== Every element of $G$ has a unique expression ==== +{{finish}} +=== Sufficient Condition === +Now let $G$ be the [[Definition:Internal Group Direct Product|internal group direct product]] of $N_1, N_2, \ldots, N_n$. +We define a [[Definition:Mapping|mapping]] $\theta: G \to N_1 \times N_2 \times \cdots \times N_n$ by: +:$\map \theta {g_1 g_2 \ldots g_n} = \tuple {g_1, g_2, \ldots, g_n}$ +The fact that $\theta$ is a [[Definition:Bijection|bijection]] follows from the definitions. +It remains to be shown that $\theta$ is a [[Definition:Group Homomorphism|homomorphism]]. +The proof proceeds by [[Principle of Mathematical Induction|induction]]. +For all $n \in \Z_{\ge 2}$, let $\map P n$ be the [[Definition:Proposition|proposition]]: +:$\map \theta {g_1 g_2 \ldots g_n} \map \theta {h_1 h_2 \ldots h_n} = \map \theta {g_1 g_2 \ldots g_n h_1 h_2 \ldots h_n}$ +==== Basis for the Induction ==== +$\map P 2$ is the case: +{{begin-eqn}} +{{eqn | l = \map \theta {g_1 g_2} \map \theta {h_1 h_2} + | r = \tuple {g_1, g_2} \tuple {h_1, h_2} + | c = Definition of $\theta$ +}} +{{eqn | r = \tuple {g_1 h_1, g_2 h_2} + | c = +}} +{{eqn | r = \map \theta {g_1 h_1 g_2 h_2} + | c = Definition of $\theta$ +}} +{{eqn | r = \map \theta {g_1 g_2 h_1 h_2} + | c = [[Internal Group Direct Product Commutativity]]: $h_1 g_2 = g_2 h_1$ +}} +{{end-eqn}} +Thus $\map P 2$ is seen to hold. +This is the [[Principle of Mathematical Induction#Basis for the Induction|basis for the induction]]. +==== Induction Hypothesis ==== +Now it needs to be shown that, if $\map P k$ is true, where $k \ge 1$, then it logically follows that $\map P {k + 1}$ is true. +So this is the [[Principle of Mathematical Induction#Induction Hypothesis|induction hypothesis]]: +:$\map \theta {g_1 g_2 \ldots g_k} \map \theta {h_1 h_2 \ldots h_k} = \map \theta {g_1 g_2 \ldots g_k h_1 h_2 \ldots h_k}$ +from which it is to be shown that: +:$\map \theta {g_1 g_2 \ldots g_{k + 1} } \map \theta {h_1 h_2 \ldots h_{k + 1} } = \map \theta {g_1 g_2 \ldots g_{k + 1} h_1 h_2 \ldots h_{k + 1} }$ +==== Induction Step ==== +This is the [[Principle of Mathematical Induction#Induction Step|induction step]]: +{{begin-eqn}} +{{eqn | o = + | r = \map \theta {g_1 g_2 \ldots g_{k + 1} } \map \theta {h_1 h_2 \ldots h_{k + 1} } + | c = +}} +{{eqn | r = \map \theta {\paren {g_1 g_2 \ldots g_k} g_{k + 1} } \map \theta {\paren {h_1 h_2 \ldots h_k} h_{k + 1} } + | c = {{GroupAxiom|1}} +}} +{{eqn | r = \map \theta {\paren {g_1 g_2 \ldots g_k} g_{k + 1} \paren {h_1 h_2 \ldots h_k} h_{k + 1} } + | c = [[Internal and External Group Direct Products are Isomorphic#Basis for the Induction|Basis for the Induction]] +}} +{{eqn | r = \map \theta {g_1 g_2 \ldots g_k g_{k + 1} h_1 h_2 \ldots h_k h_{k + 1} } + | c = {{GroupAxiom|1}} +}} +{{end-eqn}} +So $\map P k \implies \map P {k + 1}$ and the result follows by the [[Principle of Mathematical Induction]]. +Therefore: +:$\forall n \in \Z_{>1}: \map \theta {g_1 g_2 \ldots g_n} \map \theta {h_1 h_2 \ldots h_n} = \map \theta {g_1 g_2 \ldots g_n h_1 h_2 \ldots h_n}$ +So $\theta$ is a [[Definition:Group Homomorphism|homomorphism]]. +{{qed}} +\end{proof}<|endoftext|> +\section{Direct Product of Central Subgroup with Inverse Isomorphism is Central Subgroup} +Tags: Quotient Groups, Isomorphisms, Central Subgroups + +\begin{theorem} +Let $G$ and $H$ be [[Definition:Group|groups]]. +Let $\map Z G$ denote the [[Definition:Center of Group|center]] of $G$. +Let $Z$ and $W$ be [[Definition:Central Subgroup|central subgroups]] of $G$ and $H$ respectively. +Let: +:$Z \cong W$ +where $\cong$ denotes [[Definition:Group Isomorphism|isomorphism]]. +Let such a [[Definition:Group Isomorphism|group isomorphism]] be $\theta: Z \to W$. +Let $X$ be the [[Definition:Set|set]] defined as: +:$X = \set {\tuple {x, \map \theta x^{-1} }: x \in Z}$ +Then $X$ is a [[Definition:Central Subgroup|central subgroup]] of $G \times H$. +\end{theorem} + +\begin{proof} +First note that by [[Group Homomorphism Preserves Inverses]]: +:$\forall x \in G: \map \phi {x^{-1} } = \paren {\map \phi x}^{-1} = \map \phi x^{-1}$ +and so there is no [[Definition:Amphiboly|amphiboly]] in the notation used. +It is established that $\tuple {x, \map \theta x^{-1} } \in G \times H$: +:$x \in G$ +:$\map \theta x \in H$ and so $\map \theta x^{-1} \in H$. +Let: +:$e_G$ be the [[Definition:Identity Element|identity element]] of $G$ +:$e_H$ be the [[Definition:Identity Element|identity element]] of $H$ +We have that $e_G \in Z$, and so: +{{begin-eqn}} +{{eqn | l = \tuple {e_G, \map \theta {e_G}^{-1} } + | r = \tuple {e_G, \map \theta {e_G} } + | c = as $e_G^{-1} = e_G$ +}} +{{eqn | r = \tuple {e_G, e_H} + | c = [[Group Homomorphism Preserves Identity]] +}} +{{end-eqn}} +Thus $\tuple {e_G, e_H} \in X$ and so $X \ne \O$. +Then we note that: +{{begin-eqn}} +{{eqn | l = \tuple {x, \map \theta x^{-1} } \tuple {x^{-1}, \map \theta x} + | r = \tuple {x x^{-1}, \map \theta x^{-1} \map \theta x} + | c = {{Defof|Group Direct Product}} +}} +{{eqn | r = \tuple {e_G, e_H} + | c = as $\theta$ is a [[Definition:Group Isomorphism|group isomorphism]] +}} +{{eqn | r = \tuple {x^{-1} x, \map \theta x \map \theta x^{-1} } + | c = as $\theta$ is a [[Definition:Group Isomorphism|group isomorphism]] +}} +{{eqn | r = \tuple {x^{-1}, \map \theta x} \tuple {x, \map \theta x^{-1} } + | c = {{Defof|Group Direct Product}} +}} +{{end-eqn}} +and so $\tuple {x^{-1}, \map \theta x}$ is the [[Definition:Inverse Element|inverse]] of $\tuple {x, \map \theta x^{-1} }$ in $H$. +Let $x, y \in G$. +Then $x^{-1} \in G$ from {{GroupAxiom|0}}. +Hence if: +:$\tuple {x, \map \theta x^{-1} } \in X$ +it follows that: +:$\tuple {x^{-1}, \map \theta x} \in X$ +Let $x, y^{-1} \in G$. +Then: +:$\tuple {x, \map \theta x^{-1} } \in X$ +and: +:$\tuple {y^{-1}, \map \theta y} \in X$ +Thus: +{{begin-eqn}} +{{eqn | l = \tuple {x, \map \theta x^{-1} } \tuple {y^{-1}, \map \theta y} + | r = \tuple {x y^{-1}, \map \theta x^{-1} \map \theta y} + | c = {{Defof|Group Direct Product}} +}} +{{eqn | r = \tuple {x y^{-1}, \map \theta {x^{-1} y} } + | c = as $\theta$ is a [[Definition:Group Isomorphism|group isomorphism]] +}} +{{eqn | r = \tuple {x y^{-1}, \map \theta {\paren {y^{-1} x}^{-1} } } + | c = [[Inverse of Group Product]] +}} +{{eqn | r = \tuple {x y^{-1}, \map \theta {\paren {x y^{-1} }^{-1} } } + | c = $x$ and $y$ are in the [[Definition:Center of Group|center]] of $G$, so [[Definition:Commuting Elements|commute]] +}} +{{eqn | r = \tuple {x y^{-1}, \map \theta {x y^{-1} }^{-1} } + | c = +}} +{{end-eqn}} +So: +:$\tuple {x, \map \theta x^{-1} } \tuple {y^{-1}, \map \theta y} \in X$ +and it follows from the [[One-Step Subgroup Test]] that $X$ is a [[Definition:Subgroup|subgroup]] of $G \times H$. +{{qed|lemma}} +We have that $Z$ is a [[Definition:Central Subgroup|central subgroup]] of $G$. +We also have that the [[Definition:Image of Mapping|image]] of $\theta$ is $W$. +From [[Direct Product of Central Subgroups]], $Z \times W$ is a [[Definition:Central Subgroup|central subgroup]] of $G \times H$. +It follows that $X$ is a [[Definition:Central Subgroup|central subgroup]] of $G \times H$. +{{qed}} +\end{proof}<|endoftext|> +\section{Pullback of Quotient Group Isomorphism is Subgroup} +Tags: Pullbacks of Quotient Group Isomorphisms, Quotient Groups, Group Isomorphisms, Subgroups + +\begin{theorem} +Let $\struct {G, \circ}$ be a [[Definition:Group|group]] whose [[Definition:Identity Element|identity element]] is $e_G$. +Let $\struct {H, *}$ be a [[Definition:Group|group]] whose [[Definition:Identity Element|identity element]] is $e_H$. +Let $N \lhd G, K \lhd H$ be [[Definition:Normal Subgroup|normal subgroups]] of $G$ and $H$ respectively. +Let: +:$G / N \cong H / K$ +where: +:$G / N$ denotes the [[Definition:Quotient Group|quotient]] of $G$ by $N$ +:$\cong$ denotes [[Definition:Group Isomorphism|group isomorphism]]. +Let $\theta: G / N \to H / K$ be such a [[Definition:Group Isomorphism|group isomorphism]]. +Let $G \times^\theta H$ be the [[Definition:Pullback of Quotient Group Isomorphism|pullback of $G$ and $H$ via $\theta$]]. +Then $G \times^\theta H$ is a [[Definition:Subgroup|subgroup]] of $G \times H$. +\end{theorem} + +\begin{proof} +This result is proved by an application of the [[Two-Step Subgroup Test]]: +=== Condition $(1)$ === +From the definition of [[Definition:Pullback of Quotient Group Isomorphism|pullback]]: +:$\tuple {e_G, e_H} \in G \times^\theta H$ +{{iff}}: +:$\map \theta {e_G \circ N} = e_H * K$ +By [[Coset by Identity]], $e_G \circ N, e_H * K$ are the [[Definition:Identity Element|identities]] of $G / N$ and $H / K$ +From [[Group Homomorphism Preserves Identity]]: +:$\map \theta {e_G \circ N} = e_H * K$ +So $\tuple {e_G, e_H} \in G \times^\theta H$ +Thus $G \times^\theta H$ is [[Definition:Non-Empty Set|non-empty]]. +{{qed|lemma}} +=== Condition $(2)$ === +Let $\tuple {g, h}$ and $\tuple {g', h'}$ be [[Definition:Element|elements]] of $G \times^\theta H$. +It follows by definition of $\theta$ that: +:$\map \theta {g \circ N} = h * K$ +and: +:$\map \theta {g' \circ N} = h' * K$ +By the [[Definition:Morphism Property|morphism property]]: +:$\map \theta {g \circ N} * \map \theta {g' \circ N} = \map \theta {g \circ N \circ g' \circ N} = \map \theta {\paren {g \circ g'} \circ N}$ +Hence: +{{begin-eqn}} +{{eqn | l = \paren {h * h'} * K + | r = h * K * h' * K +}} +{{eqn | l = + | r = \map \theta {g \circ N} * \map \theta {g' \circ N} +}} +{{eqn | l = + | r = \map \theta {\paren {g \circ g'} \circ N} +}} +{{end-eqn}} +Thus: +:$\tuple {g \circ g', h * h'} \in G \times^\theta H$ +Hence $G \times^\theta H$ is closed under the operation. +{{qed|lemma}} +=== Condition $(3)$ === +Let $\tuple {g, h} \in G \times^\theta H$. +Then: +{{begin-eqn}} +{{eqn | l = \map \theta {g \circ N} + | r = h * K + | c = +}} +{{eqn | ll= \leadsto + | l = \map \theta {g \circ N}^{-1} + | r = h^{-1} * K + | c = +}} +{{end-eqn}} +Then: +{{begin-eqn}} +{{eqn | l = \map \theta {g \circ N}^{-1} + | r = \map \theta {g^{-1} \circ N} + | c = [[Group Homomorphism Preserves Inverses]] +}} +{{eqn | ll= \leadsto + | l = \map \theta {g^{-1} \circ N} + | r = h^{-1} * K + | c = +}} +{{eqn | ll= \leadsto + | l = \tuple {g^{-1}, h^{-1} } + | o = \in + | r = G \times^\theta H + | c = +}} +{{end-eqn}} +Thus $G \times^\theta H$ is closed under inverses. +{{qed|lemma}} +Therefore by the [[Two-Step Subgroup Test]]: +:$G \times^\theta H \le G \times H$ +{{qed}} +\end{proof}<|endoftext|> +\section{Group/Examples/x+y over 1+xy} +Tags: Examples of Groups, Examples of Groups: x+y over 1+xy + +\begin{theorem} +Let $G = \set {x \in \R: -1 < x < 1}$ be the [[Definition:Set|set]] of all [[Definition:Real Number|real numbers]] whose [[Definition:Absolute Value|absolute value]] is less than $1$. +Let $\circ: G \times G \to \R$ be the [[Definition:Binary Operation|binary operation]] defined as: +:$\forall x, y \in G: x \circ y = \dfrac {x + y} {1 + x y}$ +The [[Definition:Algebraic Structure|algebraic structure]] $\struct {G, \circ}$ is a [[Definition:Group|group]]. +\end{theorem} + +\begin{proof} +Let $-1 < x, y, z < 1$. +We check the [[Definition:Group Axioms|group axioms]] in turn: +=== $\text G 1$: Associativity === +{{begin-eqn}} +{{eqn | l = \paren {x \circ y} \circ z + | r = \frac {\frac {x + y} {1 + x y} + z} {1 + \frac {x + y} {1 + xy} z} + | c = +}} +{{eqn | r = \frac {x + y + z + x y z} {1 + x y + x z + y z} + | c = +}} +{{eqn | l = x \circ \paren {y \circ z} + | r = \frac {x + \frac {y + z} {1 + y z} } {1 + x \frac {y + z} {1 + y z} } + | c = +}} +{{eqn | r = \frac {x + y + z + x y z} {1 + x y + x z + y z} + | c = +}} +{{eqn | r = \paren {x \circ y} \circ z + | c = +}} +{{end-eqn}} +Thus $\circ$ has been shown to be [[Definition:Associative|associative]]. +{{qed|lemma}} +=== $\text G 2$: Identity === +{{begin-eqn}} +{{eqn | l = x + | r = 0 + | c = +}} +{{eqn | ll= \leadsto + | l = \frac {x + y} {1 + x y} + | r = \frac {0 + y} {1 + 0 y} + | c = +}} +{{eqn | r = \frac y 1 + | c = +}} +{{eqn | r = y + | c = +}} +{{end-eqn}} +Similarly, putting $y = 0$ we find $x \circ y = x$. +So $0$ is the [[Definition:Identity Element|identity]]. +{{qed|lemma}} +=== $\text G 3$: Inverses === +{{begin-eqn}} +{{eqn | l = x \circ -x + | r = \frac {x + \paren {-x} } {1 + x \paren {-x} } + | c = +}} +{{eqn | r = 0 + | c = +}} +{{end-eqn}} +Similarly, putting $x = -y$ gives us $\paren {-y} \circ y = 0$. +So each $x$ has an [[Definition:Inverse Element|inverse]] $-x$. +{{qed|lemma}} +=== $\text G 0$: Closure === +First note that: +:$-1 < x, y < 1 \implies x y > -1 \implies 1 + x y > 0$ +Next: +{{begin-eqn}} +{{eqn | l = -1 + | o = < + | r = x, y < 1 + | c = +}} +{{eqn | ll= \leadsto + | l = 0 + | o = < + | r = \paren {1 - x} \paren {1 - y} + | c = +}} +{{eqn | ll= \leadsto + | l = 0 + | o = < + | r = 1 + x y - \paren {x + y} + | c = +}} +{{eqn | ll= \leadsto + | l = 0 + | o = < + | r = \frac {1 + x y - \paren {x + y} } {1 + x y} + | c = +}} +{{eqn | ll= \leadsto + | l = 0 + | o = < + | r = \frac {1 + x y} {1 + x y} - \frac {x + y} {1 + x y} + | c = +}} +{{eqn | ll= \leadsto + | l = 0 + | o = < + | r = 1 - \frac {x + y} {1 + x y} + | c = +}} +{{eqn | ll= \leadsto + | l = \frac {x + y} {1 + x y} + | o = < + | r = 1 + | c = +}} +{{end-eqn}} +Finally: +{{begin-eqn}} +{{eqn | l = -1 + | o = < + | r = x, y < 1 + | c = +}} +{{eqn | ll= \leadsto + | l = 0 + | o = < + | r = \paren {1 + x} \paren {1 + y} + | c = +}} +{{eqn | ll= \leadsto + | l = 0 + | o = < + | r = 1 + x y + \paren {x + y} + | c = +}} +{{eqn | ll= \leadsto + | l = 0 + | o = < + | r = \frac {1 + x y + \paren {x + y} } {1 + x y} + | c = +}} +{{eqn | ll= \leadsto + | l = 0 + | o = < + | r = \frac {1 + x y} {1 + x y} + \frac {x + y} {1 + x y} + | c = +}} +{{eqn | ll=\leadsto + | l = 0 + | o = < + | r = 1 + \frac {x + y} {1 + x y} + | c = +}} +{{eqn | ll= \leadsto + | l = -1 + | o = < + | r = \frac {x + y} {1 + x y} + | c = +}} +{{end-eqn}} +Thus: +:$-1 < x, y < 1 \implies -1 < x \circ y < 1$ +and we see that in this range, $\circ$ is [[Definition:Closed Algebraic Structure|closed]]. +{{qed|lemma}} +Thus the given [[Definition:Set|set]] and [[Definition:Binary Operation|operation]] form a [[Definition:Group|group]]. +{{Qed}} +\end{proof}<|endoftext|> +\section{Group/Examples/inv x = 1 - x} +Tags: Examples of Groups, Group: Examples: inv x = 1 - x + +\begin{theorem} +Let $S = \set {x \in \R: 0 < x < 1}$. +Then an operation $\circ$ can be found such that $\struct {S, \circ}$ is a [[Definition:Group|group]] such that the [[Definition:Inverse Element|inverse]] of $x \in S$ is $1 - x$. +\end{theorem} + +\begin{proof} +Define $f: \openint 0 1 \to \R$ by: +:$\map f x := \map \ln {\dfrac {1 - x} x}$ +Let us show that $f$ is a [[Definition:Bijection|bijection]] by constructing an [[Definition:Inverse Mapping|inverse mapping]] $g: \R \to \openint 0 1$: +:$\map g z := \dfrac 1 {1 + \exp z}$ +=== [[Group/Examples/inv x = 1 - x/Lemma 1|Lemma 1]] === +{{:Group/Examples/inv x = 1 - x/Lemma 1}} +=== [[Group/Examples/inv x = 1 - x/Lemma 2|Lemma 2]] === +{{:Group/Examples/inv x = 1 - x/Lemma 2}} +{{finish|Might also be necessary to demonstrate the domain of $f$ is the image set of $g$ and vice versa.}} +Thus $f$ is a [[Definition:Bijection|bijection]]. +Let $\struct {\R, +}$ be the [[Definition:Additive Group of Real Numbers|additive group on $\R$]]. +Now define $\circ := +_f$ to be the [[Definition:Operation Induced by Injection|operation induced on $\openint 0 1$ by $f$ and $+$]]: +:$x \circ y := \map {f^{-1} } {\map f x + \map f y}$ +Let us determine the behaviour of $\circ$ more explicitly: +{{begin-eqn}} +{{eqn | l = x \circ y + | r = \map g {\map f x + \map f y} +}} +{{eqn | r = \frac 1 {1 + \map \exp {\map \log {\frac {1 - x} x} + \map \log {\frac {1 - y} y} } } + | c = +}} +{{eqn | r = \frac 1 {1 + \map \exp {\map \log {\frac {1 - x} x} } \map \exp {\map \log {\frac {1 - y} y} } } + | c = [[Exponential of Sum]] +}} +{{eqn | r = \frac 1 {1 + \paren {\frac {1 - x} x} \paren {\frac {1 - y} y} } + | c = [[Exponential of Natural Logarithm]] +}} +{{end-eqn}} +{{MissingLinks}} +We see that $\circ$ is [[Definition:Commutative Operation|commutative]]. +Let $x = \dfrac 1 2$, $\dfrac {1 - x} x = 1$ so that: +{{begin-eqn}} +{{eqn | l = \dfrac 1 2 \circ y + | r = \dfrac 1 {1 + \paren {\frac {1 - y} y} } +}} +{{eqn | r = \frac 1 {\frac y y + \frac {1 - y} y} +}} +{{eqn | r = \frac 1 {\frac 1 y} +}} +{{eqn | r = y +}} +{{end-eqn}} +so that $\dfrac 1 2$ is the [[Definition:Identity Element|identity element]] for $\circ$. +Furthermore, putting $y = 1 - x$, the following obtains: +{{begin-eqn}} +{{eqn | l = \frac 1 {1 + \paren {\frac {1 - x} x} \paren {\frac x {1 - x} } } + | r = \frac 1 {1 + 1} +}} +{{eqn | r = \frac 1 2 +}} +{{end-eqn}} +establishing $1 - x$ to be the [[Definition:Inverse Element|inverse]] of $x$, as desired. +That $\circ$ in fact determines a [[Definition:Group|group]] on $\openint 0 1$ follows from [[Pullback of Group is Group]]. +{{qed}} +\end{proof}<|endoftext|> +\section{Group/Examples/Self-Inverse and Cancellable Elements} +Tags: Examples of Groups + +\begin{theorem} +Let $S$ be a [[Definition:Set|set]] with an [[Definition:Binary Operation|operation]] which assigns to each $\tuple {a, b} \in S \times S$ an element $a \ast b \in S$ such that: +: $(1): \quad \exists e \in S: a \ast b = e \iff a = b$ +: $(2): \quad \forall a, b, c \in S: \paren {a \ast c} \ast \paren {b \ast c} = a \ast b$ +Then $\struct {S, \circ}$ is a [[Definition:Group|group]], where $\circ$ is defined as $a \circ b = a \ast \paren {e \ast b}$. +\end{theorem} + +\begin{proof} +We verify the [[Definition:Group Axioms|group axioms]], in the following order (for convenience): +=== G0: Closure === +Let $a, b \in S$. Then from the definition of $\ast$, we have $a \ast b \in S$, and hence also $a \circ b = a \ast \paren {e \ast b} \in S$. +This proves [[Definition:Closed Algebraic Structure|closure]] of $\circ$. +{{qed|lemma}} +=== G2: Identity === +We assert that $e$ is the identity with respect to $\circ$. +We verify this as follows (let $a \in S$): +{{begin-eqn}} +{{eqn | l = a \circ e + | r = a \ast \paren {e \ast e} + | c = Definition of $\circ$ +}} +{{eqn | r = a \ast e + | c = Property $(1)$ of $\ast$ +}} +{{eqn | l = e \circ a + | r = e \ast \paren {e \ast a} + | c = Definition of $\circ$ +}} +{{eqn | r = \paren {a \ast a} \ast \paren {e \ast a} + | c = Property $(1)$ of $\ast$ +}} +{{eqn | r = a \ast e + | c = Property $(2)$ of $\ast$ +}} +{{end-eqn}} +It follows that $a \circ e = e \circ a = a \ast e$. +It remains to prove the following identity: +: $(3): \quad a \ast e = a$ +By property $(1)$, it suffices to prove $a \ast \paren {a \ast e} = e$: +{{begin-eqn}} +{{eqn | l = a \ast \paren {a \ast e} + | r = \paren {a \ast e} \ast \paren {\paren {a \ast e} \ast e} + | c = Property $(2)$ of $\ast$ +}} +{{eqn | r = \paren {a \ast e} \ast \paren {\paren {a \ast e} \ast \paren {e \ast e} } + | c = Property $(1)$ of $\ast$ +}} +{{eqn | r = \paren {a \ast e} \ast \paren {a \ast e} + | c = Property $(2)$ of $\ast$ +}} +{{eqn | r = e + | c = Property $(1)$ of $\ast$ +}} +{{end-eqn}} +So indeed $e$ is the [[Definition:Identity Element|identity]] for $\circ$. +{{qed|lemma}} +=== G3: Inverses === +For any element $a \in S$, we claim that $e \ast a$ is the [[Definition:Inverse Element|inverse]] for $a$. +This is verified as follows: +{{begin-eqn}} +{{eqn | l = \paren {e \ast a} \circ a + | r = \paren {e \ast a} \ast \paren {e \ast a} + | c = Definition of $\circ$ +}} +{{eqn | r = e + | c = Property $(1)$ of $\ast$ +}} +{{eqn | l = a \circ \paren {e \ast a} + | r = a \ast \paren {e \ast \paren {e \ast a} } + | c = Definition of $\circ$ +}} +{{eqn | r = a \ast \paren {e \circ a} + | c = Definition of $\circ$ +}} +{{eqn | r = a \ast a + | c = $e$ is the identity element for $\circ$ +}} +{{eqn | r = e + | c = Property $(1)$ of $\ast$ +}} +{{end-eqn}} +We conclude that $e \ast a$ is indeed the [[Definition:Inverse Element|inverse]] for $a$. +{{qed|lemma}} +=== G1: Associativity === +Lastly, we prove that $\circ$ is [[Definition:Associative|associative]], i.e.: +:$a \circ \paren {b \circ c} = \paren {a \circ b} \circ c$ +To this end, we observe the following property of $\ast$: +{{begin-eqn}} +{{eqn | n = 4 + | l = e \ast \paren {a \ast b} + | r = \paren {b \ast b} \ast \paren {a \ast b} + | c = Property $(1)$ of $\ast$ +}} +{{eqn | r = b \ast a + | c = Property $(2)$ of $\ast$ +}} +{{end-eqn}} +Subsequently, we compute: +{{begin-eqn}} +{{eqn | l = \paren {a \circ b} \circ c + | r = \paren {a \ast \paren {e \ast b} } \ast \paren {e \ast c} + | c = Definition of $\circ$ +}} +{{eqn | r = \paren {a \ast \paren {e \ast b} } \ast \paren {\paren {e \ast c} \ast e} + | c = Property $(3)$ of $\ast$ +}} +{{eqn | r = \paren {a \ast \paren {e \ast b} } \ast \paren {\paren {\paren {e \ast c} \ast b} \ast \paren {e \ast b} } + | c = Property $(2)$ of $\ast$ +}} +{{eqn | r = a \ast \paren {\paren {e \ast c} \ast b} + | c = Property $(2)$ of $\ast$ +}} +{{eqn | r = a \ast \paren {e \ast \paren {b \ast \paren {e \ast c} } } + | c = Property $(4)$ of $\ast$ +}} +{{eqn | r = a \circ \paren {b \circ c} + | c = Definition of $\circ$ +}} +{{end-eqn}} +This somewhat cumbersome calculation shows that we indeed have [[Definition:Associative|associativity]]. +{{qed|lemma}} +It follows that $\struct {S, \circ}$ is indeed a [[Definition:Group|group]]. +{{qed}} +\end{proof}<|endoftext|> +\section{Complex Numbers under Addition form Abelian Group} +Tags: Additive Group of Complex Numbers, Complex Addition, Examples of Abelian Groups, Examples of Infinite Groups + +\begin{theorem} +Let $\C$ be the set of [[Definition:Complex Number|complex numbers]]. +The [[Definition:Algebraic Structure|structure]] $\struct {\C, +}$ is an [[Definition:Infinite Group|infinite]] [[Definition:Abelian Group|abelian group]]. +\end{theorem} + +\begin{proof} +Taking the [[Definition:Group Axioms|group axioms]] in turn: +=== $\text G 0$: Closure === +[[Complex Addition is Closed]]. +{{qed|lemma}} +=== $\text G 1$: Associativity === +[[Complex Addition is Associative]]. +{{qed|lemma}} +=== $\text G 2$: Identity === +From [[Complex Addition Identity is Zero]], we have that the [[Definition:Identity Element|identity element]] of $\struct {\C, +}$ is the [[Definition:Complex Number|complex number]] $0 + 0 i$: +:$\paren {x + i y} + \paren {0 + 0 i} = \paren {x + 0} + i \paren {y + 0} = x + i y$ +and similarly for $\paren {0 + 0 i} + \paren {x + i y}$. +{{qed|lemma}} +=== $\text G 3$: Inverses === +From [[Inverse for Complex Addition]], the [[Definition:Inverse Element|inverse]] of $x + i y \in \struct {\C, +}$ is $-x - i y$. +{{qed|lemma}} +=== $\text C$: Commutativity === + +[[Complex Addition is Commutative]]. +{{qed|lemma}} +=== Infinite === +[[Complex Numbers are Uncountable]]. +{{qed}} +\end{proof}<|endoftext|> +\section{Non-Zero Complex Numbers under Multiplication form Abelian Group} +Tags: Complex Multiplication, Examples of Abelian Groups, Examples of Infinite Groups + +\begin{theorem} +Let $\C_{\ne 0}$ be the set of [[Definition:Complex Number|complex numbers]] without [[Definition:Zero (Number)|zero]], that is: +:$\C_{\ne 0} = \C \setminus \set 0$ +The [[Definition:Algebraic Structure|structure]] $\struct {\C_{\ne 0}, \times}$ is an [[Definition:Infinite Group|infinite]] [[Definition:Abelian Group|abelian group]]. +\end{theorem} + +\begin{proof} +Taking the [[Definition:Group Axioms|group axioms]] in turn: +=== $\text G 0$: Closure === +[[Non-Zero Complex Numbers Closed under Multiplication]]. +{{qed|lemma}} +=== $\text G 1$: Associativity === +[[Complex Multiplication is Associative]]. +{{qed|lemma}} +=== $\text G 2$: Identity === +From [[Complex Multiplication Identity is One]], the identity element of $\struct {\C_{\ne 0}, \times}$ is the [[Definition:Complex Number|complex number]] $1 + 0 i$. +{{qed|lemma}} +=== $\text G 3$: Inverses === +From [[Inverse for Complex Multiplication]], the inverse of $x + i y \in \struct {\C_{\ne 0}, \times}$ is: +:$\dfrac 1 z = \dfrac {x - i y} {x^2 + y^2} = \dfrac {\overline z} {z \overline z}$ +where $\overline z$ is the [[Definition:Complex Conjugate|complex conjugate]] of $z$. +{{qed|lemma}} +=== $\text C$: Commutativity === + +[[Complex Multiplication is Commutative]]. +{{qed|lemma}} +=== Infinite === +[[Complex Numbers are Uncountable]]. +{{qed}} +\end{proof}<|endoftext|> +\section{Real Numbers under Addition form Abelian Group} +Tags: Additive Group of Real Numbers + +\begin{theorem} +Let $\R$ be the set of [[Definition:Real Number|real numbers]]. +The [[Definition:Algebraic Structure|structure]] $\struct {\R, +}$ is an [[Definition:Infinite Group|infinite]] [[Definition:Abelian Group|abelian group]]. +\end{theorem} + +\begin{proof} +Taking the [[Definition:Group Axioms|group axioms]] in turn: +=== $\text G 0$: Closure === +[[Real Addition is Closed]]. +{{qed|lemma}} +=== $\text G 1$: Associativity === +[[Real Addition is Associative]]. +{{qed|lemma}} +=== $\text G 2$: Identity === +From [[Real Addition Identity is Zero]], we have that the [[Definition:Identity Element|identity element]] of $\struct {\R, +}$ is the real number $0$. +{{qed|lemma}} +=== $\text G 3$: Inverses === +From [[Inverses for Real Addition]], we have that the [[Definition:Inverse Element|inverse]] of $x \in \struct {\R, +}$ is $-x$. +{{qed|lemma}} +=== $\text C$: Commutativity === + +[[Real Addition is Commutative]]. +{{qed|lemma}} +=== Infinite === +[[Real Numbers are Uncountable|Real Numbers are Uncountably Infinite]]. +{{qed}} +\end{proof}<|endoftext|> +\section{Non-Zero Real Numbers under Multiplication form Abelian Group} +Tags: Real Multiplication, Examples of Abelian Groups, Examples of Infinite Groups, Non-Zero Real Numbers under Multiplication form Abelian Group + +\begin{theorem} +Let $\R_{\ne 0}$ be the set of [[Definition:Real Number|real numbers]] without [[Definition:Zero (Number)|zero]]: +:$\R_{\ne 0} = \R \setminus \set 0$ +The [[Definition:Algebraic Structure|structure]] $\struct {\R_{\ne 0}, \times}$ is an [[Definition:Uncountable Group|uncountable]] [[Definition:Abelian Group|abelian group]]. +\end{theorem} + +\begin{proof} +Taking the [[Definition:Group Axioms|group axioms]] in turn: +=== $\text G 0$: Closure === +From [[Non-Zero Real Numbers Closed under Multiplication/Proof 2|Non-Zero Real Numbers Closed under Multiplication: Proof 2]], $\R_{\ne 0}$ is [[Definition:Closed Algebraic Structure|closed]] under [[Definition:Real Multiplication|multiplication]]. +Note that [[Non-Zero Real Numbers Closed under Multiplication/Proof 2|proof 2]] needs to be used specifically here, as [[Non-Zero Real Numbers Closed under Multiplication/Proof 1|proof 1]] rests on this result. +{{qed|lemma}} +=== $\text G 1$: Associativity === +[[Real Multiplication is Associative]]. +{{qed|lemma}} +=== $\text G 2$: [[Real Multiplication Identity is One|Identity]] === +{{:Real Multiplication Identity is One}} +{{qed|lemma}} +=== $\text G 3$: [[Inverses for Real Multiplication|Inverses]] === +{{:Inverses for Real Multiplication}} +{{qed|lemma}} +=== $\text C$: Commutativity === + +[[Real Multiplication is Commutative]]. +{{qed|lemma}} +=== Infinite === +[[Real Numbers are Uncountable|Real Numbers are Uncountably Infinite]]. +{{qed}} +\end{proof} + +\begin{proof} +We have [[Real Numbers under Multiplication form Monoid]]. +From [[Inverses for Real Multiplication]], the non-[[Definition:Zero (Number)|zero]] [[Definition:Number|numbers]] are exactly the [[Definition:Invertible Element|invertible elements]] of [[Definition:Real Multiplication|real multiplication]]. +Thus from [[Invertible Elements of Monoid form Subgroup of Cancellable Elements]], the non-[[Definition:Zero (Number)|zero]] [[Definition:Real Number|real numbers]] under [[Definition:Real Multiplication|multiplication]] form a [[Definition:Group|group]]. +From: +:[[Real Multiplication is Commutative]] +:[[Subset Product within Commutative Structure is Commutative]] +it follows that this [[Definition:Group|group]] is also [[Definition:Abelian Group|Abelian]]. +{{qed}} +\end{proof} + +\begin{proof} +From [[Non-Zero Real Numbers under Multiplication form Group]], $\struct {\R_{\ne 0}, \times}$ forms a [[Definition:Group|group]]. +{{qed|lemma}} +From [[Real Multiplication is Commutative]] it follows that $\struct {\R_{\ne 0}, \times}$ is [[Definition:Abelian Group|abelian]]. +{{qed|lemma}} +From [[Real Numbers are Uncountable|Real Numbers are Uncountably Infinite]] it follows that $\struct {\R_{\ne 0}, \times}$ is an [[Definition:Uncountable Group|uncountable]] [[Definition:Abelian Group|abelian group]]. +{{qed}} +\end{proof}<|endoftext|> +\section{Rational Numbers under Addition form Abelian Group} +Tags: Rational Addition, Examples of Abelian Groups, Examples of Infinite Groups + +\begin{theorem} +Let $\Q$ be the set of [[Definition:Rational Number|rational numbers]]. +The [[Definition:Algebraic Structure|structure]] $\struct {\Q, +}$ is a [[Definition:Countably Infinite|countably infinite]] [[Definition:Abelian Group|abelian group]]. +\end{theorem} + +\begin{proof} +The [[Definition:Rational Number|rational numbers]] are, by definition, the [[Definition:Field of Quotients|field of quotients]] of the [[Definition:Integral Domain|integral domain]] $\struct {\Z, +, \times}$ of [[Definition:Integer|integers]]. +Hence by definition, $\struct {\Q, +, \times}$ is a [[Definition:Field (Abstract Algebra)|field]]. +The fact that $\struct {\Q, +}$ forms an [[Definition:Abelian Group|abelian group]] follows directly from the definition of a [[Definition:Field (Abstract Algebra)|field]]. +From [[Rational Numbers are Countably Infinite]], we have that $\struct {\Q, +}$ is [[Definition:Countably Infinite|countably infinite]]. +{{Qed}} +\end{proof}<|endoftext|> +\section{Non-Zero Rational Numbers under Multiplication form Abelian Group} +Tags: Rational Multiplication, Examples of Abelian Groups, Examples of Infinite Groups + +\begin{theorem} +Let $\Q_{\ne 0}$ be the [[Definition:Set|set]] of non-[[Definition:Zero (Number)|zero]] [[Definition:Rational Number|rational numbers]]: +:$\Q_{\ne 0} = \Q \setminus \set 0$ +The [[Definition:Algebraic Structure|structure]] $\struct {\Q_{\ne 0}, \times}$ is a [[Definition:Countably Infinite Group|countably infinite]] [[Definition:Abelian Group|abelian group]]. +\end{theorem} + +\begin{proof} +From the [[Definition:Rational Number|definition of rational numbers]], the [[Definition:Algebraic Structure|structure]] $\struct {\Q, + \times}$ is constructed as the [[Definition:Field of Quotients|field of quotients]] of the [[Definition:Integral Domain|integral domain]] $\struct {\Z, +, \times}$ of [[Definition:Integer|integers]]. +Hence from [[Multiplicative Group of Field is Abelian Group]], $\struct {\Q_{\ne 0}, \times}$ is an [[Definition:Abelian Group|abelian group]]. +From [[Rational Numbers are Countably Infinite]], we have that $\struct {\Q_{\ne 0}, \times}$ is a [[Definition:Countably Infinite Group|countably infinite group]]. +{{qed}} +\end{proof}<|endoftext|> +\section{Integers under Multiplication form Countably Infinite Commutative Monoid} +Tags: Integer Multiplication + +\begin{theorem} +The [[Definition:Set|set]] of [[Definition:Integer|integers]] under [[Definition:Integer Multiplication|multiplication]] $\struct {\Z, \times}$ is a [[Definition:Countably Infinite Set|countably infinite]] [[Definition:Commutative Monoid|commutative monoid]]. +\end{theorem} + +\begin{proof} +First we note that [[Integers under Multiplication form Monoid]]. +{{qed|lemma}} +Then we have: +=== Commutativity === +[[Integer Multiplication is Commutative]]. +{{qed|lemma}} +=== Infinite === +[[Integers are Countably Infinite]]. +{{Qed}} +\end{proof}<|endoftext|> +\section{Additive Group of Rationals is Subgroup of Reals} +Tags: Rational Addition, Additive Group of Real Numbers, Normal Subgroups, Additive Group of Rational Numbers + +\begin{theorem} +Let $\struct {\Q, +}$ be the [[Definition:Additive Group of Rational Numbers|additive group of rational numbers]]. +Let $\struct {\R, +}$ be the [[Definition:Additive Group of Real Numbers|additive group of real numbers]]. +Then $\struct {\Q, +}$ is a [[Definition:Normal Subgroup|normal subgroup]] of $\struct {\R, +}$. +\end{theorem} + +\begin{proof} +From the definition of [[Definition:Real Number|real numbers]], $\Q$ is a [[Definition:Subset|subset]] of $\R$. +As $\struct {\R, +}$ is a [[Definition:Group|group]], and $\struct {\Q, +}$ is a [[Definition:Group|group]], it follows from the definition of [[Definition:Subgroup|subgroup]] that $\struct {\Q, +}$ is a subgroup of $\struct {\R, +}$. +As $\struct {\R, +}$ is [[Definition:Abelian Group|abelian]], it follows from [[Subgroup of Abelian Group is Normal]] that $\struct {\Q, +}$ is [[Definition:Normal Subgroup|normal]] in $\struct {\R, +}$. +{{Qed}} +\end{proof}<|endoftext|> +\section{Additive Group of Integers is Subgroup of Rationals} +Tags: Additive Group of Integers, Additive Group of Rational Numbers, Normal Subgroups + +\begin{theorem} +Let $\struct {\Z, +}$ be the [[Definition:Additive Group of Integers|additive group of integers]]. +Let $\struct {\Q, +}$ be the [[Definition:Additive Group of Rational Numbers|additive group of rational numbers]]. +Then $\struct {\Z, +}$ is a [[Definition:Subgroup|subgroup]] of $\struct {\Q, +}$. +\end{theorem} + +\begin{proof} +Recall that [[Integers form Integral Domain]]. +The set $\Q$ of [[Definition:Rational Number|rational numbers]] is defined as the [[Definition:Rational Number|field of quotients of the integers]]. +The fact that the [[Definition:Integer|integers]] are a [[Definition:Subgroup|subgroup]] of the [[Definition:Rational Number|rationals]] follows from the work done in proving the [[Existence of Field of Quotients]] from an [[Definition:Integral Domain|integral domain]]. +{{qed}} +\end{proof}<|endoftext|> +\section{Additive Group of Reals is Normal Subgroup of Complex} +Tags: Additive Group of Real Numbers, Additive Group of Complex Numbers, Examples of Normal Subgroups + +\begin{theorem} +Let $\struct {\R, +}$ be the [[Definition:Additive Group of Real Numbers|additive group of real numbers]]. +Let $\struct {\C, +}$ be the [[Definition:Additive Group of Complex Numbers|additive group of complex numbers]]. +Then $\struct {\R, +}$ is a [[Definition:Normal Subgroup|normal subgroup]] of $\struct {\C, +}$. +\end{theorem} + +\begin{proof} +From [[Additive Group of Reals is Subgroup of Complex]], $\struct {\R, +}$ is a [[Definition:Subgroup|subgroup]] of $\struct {\C, +}$. +Then from [[Complex Numbers under Addition form Abelian Group]], $\struct {\C, +}$ is [[Definition:Abelian Group|abelian]]. +The result follows from [[Subgroup of Abelian Group is Normal]]. +{{Qed}} +[[Category:Additive Group of Real Numbers]] +[[Category:Additive Group of Complex Numbers]] +[[Category:Examples of Normal Subgroups]] +1u96lp0tx8g2ml9xdr96nzf90lau0jp +\end{proof}<|endoftext|> +\section{Multiplicative Group of Reals is Subgroup of Complex} +Tags: Multiplicative Group of Real Numbers, Multiplicative Group of Complex Numbers, Normal Subgroups + +\begin{theorem} +Let $\struct {\R_{\ne 0}, \times}$ be the [[Definition:Multiplicative Group of Real Numbers|multiplicative group of real numbers]]. +Let $\struct {\C_{\ne 0}, \times}$ be the [[Definition:Multiplicative Group of Complex Numbers|multiplicative group of complex numbers]]. +Then $\struct {\R_{\ne 0}, \times}$ is a [[Definition:Normal Subgroup|normal subgroup]] of $\struct {\C_{\ne 0}, \times}$. +\end{theorem} + +\begin{proof} +Let $x, y \in \C_{\ne 0}$ such that $x = x_1 + 0 i, y = y_1 + 0 i$. +As $x$ and $y$ are [[Definition:Wholly Real|wholly real]], we have that $x, y \in \R_{\ne 0}$. +Then $x + y = x_1 y_1 - 0 \cdot 0 + \left({0 \cdot y_1 + x_1 \cdot 0}\right) i = x_1 y_1 + 0i$ which is also [[Definition:Wholly Real|wholly real]]. +Also, the inverse of $x$ is $\dfrac 1 x = \dfrac 1 {x_1} + 0 i$ which is also [[Definition:Wholly Real|wholly real]]. +Thus by the [[Two-Step Subgroup Test]], $\struct {\R_{\ne 0}, \times}$ is a [[Definition:Subgroup|subgroup]] of $\struct {\C_{\ne 0}, \times}$. +From [[Non-Zero Complex Numbers under Multiplication form Abelian Group]], $\struct {\C_{\ne 0}, \times}$ is [[Definition:Abelian Group|abelian]]. +The result follows from [[Subgroup of Abelian Group is Normal]]. +{{Qed}} +\end{proof}<|endoftext|> +\section{Multiplicative Group of Rationals is Subgroup of Reals} +Tags: Normal Subgroups, Multiplicative Group of Rational Numbers, Multiplicative Group of Real Numbers + +\begin{theorem} +Let $\struct {\Q_{\ne 0}, \times}$ be the [[Definition:Multiplicative Group of Rational Numbers|multiplicative group of rational numbers]]. +Let $\struct {\R_{\ne 0}, \times}$ be the [[Definition:Multiplicative Group of Real Numbers|multiplicative group of real numbers]]. +Then $\struct {\Q_{\ne 0}, \times}$ is a [[Definition:Normal Subgroup|normal subgroup]] of $\left({\R_{\ne 0}, \times}\right)$. +\end{theorem} + +\begin{proof} +From the definition of [[Definition:Real Number|real numbers]], it follows that $\Q_{\ne 0}$ is a [[Definition:Subset|subset]] of $\R_{\ne 0}$. +As $\struct {\R_{\ne 0}, \times}$ is a [[Definition:Group|group]], and $\struct {\Q_{\ne 0}, \times}$ is a [[Definition:Group|group]], it follows from the definition of [[Definition:Subgroup|subgroup]] that $\struct {\Q_{\ne 0}, \times}$ is a subgroup of $\struct {\R_{\ne 0}, \times}$. +As $\struct {\R_{\ne 0}, \times}$ is [[Definition:Abelian Group|abelian]], it follows from [[Subgroup of Abelian Group is Normal]] that $\struct {\Q_{\ne 0}, \times}$ is [[Definition:Normal Subgroup|normal]] in $\struct {\R_{\ne 0}, \times}$. +{{qed}} +\end{proof}<|endoftext|> +\section{Circle Group is Infinite Abelian Group} +Tags: Circle Group, Complex Numbers, Circle Group is Infinite Abelian Group + +\begin{theorem} +The [[Definition:Circle Group|circle group]] $\struct {K, \times}$ is an [[Definition:Uncountable Set|uncountably infinite]] [[Definition:Abelian Group|abelian group]] under the operation of [[Definition:Complex Multiplication|complex multiplication]]. +\end{theorem} + +\begin{proof} +We note that $K \ne \varnothing$ as the [[Complex Multiplication Identity is One|identity element $1 + 0 i \in K$]]. +Since all $z \in K$ have [[Definition:Complex Modulus|modulus]] $1$, they have, for some $\theta \in \hointr 0 {2 \pi}$, the [[Definition:Polar Form of Complex Number|polar form]]: +:$z = \map \exp {i \theta} = \cos \theta + i \sin \theta$ +Conversely, if a [[Definition:Complex Number|complex number]] has such a [[Definition:Polar Form of Complex Number|polar form]], it has [[Definition:Complex Modulus|modulus]] $1$. +Observe the following property of the [[Definition:Complex Exponential Function|complex exponential function]]: +:$\forall a, b \in \C: \map \exp {a + b} = \map \exp a \map \exp b$ +We must show that if $x, y \in K$ then $x \cdot y^{-1} \in K$. +Let $x, y \in K$ be arbitrary. +Choose suitable $s, t \in \hointr 0 {2 \pi}$ such that: +:$x = \map \exp {i s}$ +:$y = \map \exp {i t}$ +We compute: +{{begin-eqn}} +{{eqn | l = \map \exp {i t} \map \exp {-i t} + | r = \map \exp {i \paren {t - t} } +}} +{{eqn | r = \map \exp 0 +}} +{{eqn | r = 1 +}} +{{end-eqn}} +So $y^{-1} = \map \exp {-i t}$. +We note that this lies in $K$. +Furthermore, we have: +{{begin-eqn}} +{{eqn | l = x y + | r = \map \exp {i s} \map \exp {-2 \pi i t} +}} +{{eqn | r = \map \exp {i \paren {s - t} } +}} +{{end-eqn}} +We conclude that $x y \in K$. +By the [[Two-Step Subgroup Test]], $K$ is a subgroup of $\C$ under [[Definition:Complex Multiplication|complex multiplication]]. +{{handwaving|actually, instead of $\hointr 0 {2 \pi}$ we should use $\R / 2 \pi \Z$ for formal correctness, but I haven't thought of a nice way to do so. Also the property of $\exp$ is only established for the reals so far.}} +That the operation $\times$ on $K$ is [[Definition:Commutative Operation|commutative]] follows from [[Complex Multiplication is Commutative]] and [[Restriction of Commutative Operation is Commutative]]. +That is, $\times$ is [[Definition:Commutative Operation|commutative]] on $K$ because it is already [[Definition:Commutative Operation|commutative]] on $\C$. +Finally we have that the [[Circle Group is Uncountably Infinite]]. +{{qed}} +\end{proof} + +\begin{proof} +First we note that $K \subseteq \C$. +So to show that $K$ is a [[Definition:Group|group]] it is sufficient to show that $K$ is a [[Definition:Subgroup|subgroup]] of the [[Definition:Multiplicative Group of Complex Numbers|multiplicative group of complex numbers]] $\struct {\C_{\ne 0}, \times}$. +From [[Complex Multiplication Identity is One]], the [[Definition:Identity Element|identity element]] $1 + 0 i$ is in $K$. +Thus $K \ne \O$. +We now show that $z, w \in K \implies z w \in K$: +{{begin-eqn}} +{{eqn | l = z, w + | o = \in + | r = K + | c = +}} +{{eqn | ll= \leadsto + | l = \cmod z + | r = 1 = \cmod w + | c = +}} +{{eqn | ll= \leadsto + | l = \cmod {z w} + | r = \cmod z \cmod w + | c = +}} +{{eqn | ll= \leadsto + | l = z w + | o = \in + | r = K + | c = +}} +{{end-eqn}} +Next we see that $z \in K \implies z^{-1} \in K$: +:$\cmod z = 1 \implies \cmod {\dfrac 1 z} = 1$ +Thus by the [[Two-Step Subgroup Test]]: +:$K \le \C_{\ne 0}$ +Thus $K$ is a [[Definition:Group|group]]. +From [[Complex Multiplication is Commutative]] it also follows from [[Subgroup of Abelian Group is Abelian]] that $K$ is an [[Definition:Abelian Group|abelian group]]. +Finally we have that the [[Circle Group is Uncountably Infinite]]. +{{qed}} +\end{proof} + +\begin{proof} +Taking the [[Definition:Group Axioms|group axioms]] in turn: +=== $\text G 0$: Closure === +{{begin-eqn}} +{{eqn | l = z, w + | o = \in + | r = K + | c = +}} +{{eqn | ll= \leadsto + | l = \cmod z + | r = 1 = \cmod w + | c = +}} +{{eqn | ll= \leadsto + | l = \cmod {z w} + | r = \cmod z \cmod w + | c = +}} +{{eqn | ll= \leadsto + | l = z w + | o = \in + | r = K + | c = +}} +{{end-eqn}} +So $\struct {\mathbb S, \cdot}$ is [[Definition:Closed Algebraic Structure|closed]]. +{{qed|lemma}} +=== $\text G 1$: Associativity === +[[Complex Multiplication is Associative]]. +{{qed|lemma}} +=== $\text G 2$: Identity === +From [[Complex Multiplication Identity is One]] we have that the [[Definition:Identity Element|identity element]] of $K$ is $1 + 0 i$. +{{qed|lemma}} +=== $\text G 3$: Inverses === +We have that: +:$\cmod z = 1 \implies \dfrac 1 {\cmod z} = \cmod {\dfrac 1 z} = 1$ +But: +:$z \times \dfrac 1 z = 1 + 0 i$ +So the [[Definition:Inverse Element|inverse]] of $z$ is $\dfrac 1 z$. +{{qed|lemma}} +=== $\text C$: Commutative === +We have that [[Complex Multiplication is Commutative]]. +We also have from [[Restriction of Commutative Operation is Commutative]] that $\times$ is likewise [[Definition:Commutative Operation|commutative]] on $K$. +{{qed|lemma}} +=== Uncountably Infinite === +[[Circle Group is Uncountably Infinite]]. +{{Qed|lemma}} +All the criteria are satisfied, and the result follows. +{{qed}} +\end{proof} + +\begin{proof} +Consider the [[Definition:Complex Modulus|complex modulus]] function $\left\vert{\cdot}\right\vert: \C \to \R, z \mapsto \left\vert{z}\right\vert$. +By [[Complex Modulus is Norm]], we have that $\left\vert{z}\right\vert \ge 0$ for all $z \in \C$, and: +:$\left\vert{z}\right\vert = 0 \iff z = 0$ +From [[Group of Units of Field]] and [[Complex Numbers form Field]], we have $\C^\times = \C \setminus \left\{{0}\right\}$. +By above observation, the [[Definition:Complex Modulus|modulus]] has a [[Definition:Restriction of Mapping|restriction]] to $\C^\times$: +:$\left\vert{\cdot}\right\vert: \C^\times \to \R_{>0}, z \mapsto \left\vert{z}\right\vert$ +From $\left\vert{1}\right\vert = 1$ and [[Modulus of Product]], it follows that it is in fact a [[Definition:Group Homomorphism|group homomorphism]]: +:$\phi: \left({C^\times, \cdot}\right) \to \left({\R_{>0}, \cdot}\right), z \mapsto \left\vert{z}\right\vert$ +Now $K$ is by definition the [[Definition:Kernel of Group Homomorphism|kernel]] of $\phi$. +Hence, by [[Kernel of Group Homomorphism is Subgroup]], $\left({K, \times}\right)$ is a [[Definition:Subgroup|subgroup]] of $\left({\C^\times, \cdot}\right)$. +By [[Subgroup of Abelian Group is Abelian]] and [[Complex Multiplication is Commutative]], it is also [[Definition:Abelian Group|abelian]]. +Finally, note that $\left({K, \times}\right)$ is [[Definition:Uncountable Set|uncountable]] from [[Circle Group is Uncountably Infinite]]. +{{qed}} +\end{proof}<|endoftext|> +\section{Homomorphism from Reals to Circle Group} +Tags: Circle Group, Examples of Group Homomorphisms + +\begin{theorem} +Let $\struct {\R, +}$ be the [[Definition:Additive Group of Real Numbers|additive group of real numbers]]. +Let $\struct {K, \times}$ be the [[Definition:Circle Group|circle group]]. +Let $\phi: \struct {\R, +} \to \struct {K, \times}$ be the [[Definition:Mapping|mapping]] defined as: +:$\forall x \in \R: \map \phi x = e^{i x}$ +Then $\phi$ is a [[Definition:Group Homomorphism|(group) homomorphism]]. +\end{theorem} + +\begin{proof} +Let $x, y \in \R$. +Then: +{{begin-eqn}} +{{eqn | l = \map \phi x \times \map \phi y + | r = e^{i x} e^{i y} + | c = +}} +{{eqn | r = e^{i \paren {x + y} } + | c = +}} +{{eqn | r = \map \phi {x + y} + | c = +}} +{{end-eqn}} +{{qed}} +\end{proof}<|endoftext|> +\section{Integers under Addition form Infinite Cyclic Group} +Tags: Examples of Cyclic Groups, Integer Addition, Additive Group of Integers + +\begin{theorem} +The [[Definition:Additive Group of Integers|additive group of integers]] $\struct {\Z, +}$ is an [[Definition:Infinite Cyclic Group|infinite cyclic group]] which is [[Definition:Generator of Cyclic Group|generated by]] the element $1 \in \Z$. +\end{theorem} + +\begin{proof} +By [[Epimorphism from Integers to Cyclic Group]] and [[Definition:Integer Multiplication|integer multiplication]]: +:$\forall n \in \Z: n = \map {+^n} 1 \in \gen 1$ +Thus: +:$\struct {\Z, +} = \gen 1$ +and thus, by the definition of a [[Definition:Cyclic Group|cyclic group]], is [[Definition:Cyclic Group|cyclic]]. +{{qed}} +\end{proof}<|endoftext|> +\section{Generators of Additive Group of Integers} +Tags: Integer Addition, Cyclic Groups, Additive Group of Integers + +\begin{theorem} +The only [[Definition:Generator of Cyclic Group|generators]] of the [[Definition:Additive Group of Integers|additive group of integers]] $\struct {\Z, +}$ are $1$ and $-1$. +\end{theorem} + +\begin{proof} +From [[Integers under Addition form Infinite Cyclic Group]], $\struct {\Z, +}$ is an [[Definition:Infinite Cyclic Group|infinite cyclic group]] generated by $1$. +From [[Generators of Infinite Cyclic Group]], there is only one other [[Definition:Generator of Cyclic Group|generator]] of such a group, and that is the inverse of that generator. +The result follows. +{{qed}} +\end{proof}<|endoftext|> +\section{Inverse of Generator of Cyclic Group is Generator} +Tags: Cyclic Groups, Inverse of Generator of Cyclic Group is Generator + +\begin{theorem} +Let $\gen g = G$ be a [[Definition:Cyclic Group|cyclic group]]. +Then: +:$G = \gen {g^{-1} }$ +where $g^{-1}$ denotes the [[Definition:Inverse Element|inverse]] of $g$. +Thus, in general, a [[Definition:Generator of Cyclic Group|generator]] of a [[Definition:Cyclic Group|cyclic group]] is not [[Definition:Unique|unique]]. +\end{theorem} + +\begin{proof} +Let $\gen g = G$. +Then from [[Set of Words Generates Group]]: +:$\map W {\set {g, g^{-1} } } = G$ +But: +:$\gen {g^{-1} } = \map W {\set {g, g^{-1} } }$ +and the result follows. +{{qed}} +\end{proof} + +\begin{proof} +Let $C_n = \gen g$ be the [[Definition:Cyclic Group|cyclic group]] of [[Definition:Order of Structure|order $n$]]. +By definition, $g^n = e$. +We have that $n - 1$ is [[Definition:Coprime Integers|coprime]] to $n$. +So it follows from that [[Power of Generator of Cyclic Group is Generator iff Power is Coprime with Order]] that: +: $C_n = \gen {g^{n - 1} }$ +But from [[Inverse Element is Power of Order Less 1]]: +:$g^{n - 1} = g^{-1}$ +{{Qed}} +\end{proof}<|endoftext|> +\section{Generators of Infinite Cyclic Group} +Tags: Cyclic Groups, Infinite Cyclic Group + +\begin{theorem} +Let $\gen g = G$ be an [[Definition:Infinite Cyclic Group|infinite cyclic group]]. +Then the only other [[Definition:Generator of Cyclic Group|generator]] of $G$ is $g^{-1}$. +Thus an [[Definition:Infinite Cyclic Group|infinite cyclic group]] has exactly $2$ [[Definition:Generator of Cyclic Group|generators]]. +\end{theorem} + +\begin{proof} +By definition, the [[Definition:Infinite Cyclic Group|infinite cyclic group]] with [[Definition:Generator of Cyclic Group|generator]] $g$ is: +:$\gen g = \set {\ldots, g^{-2}, g^{-1}, e, g, g^2, \ldots}$ +where $e$ denotes the [[Definition:Identity Element|identity]] $e = g^0$. +The fact that $g^{-1}$ [[Definition:Generator of Cyclic Group|generates]] $G$ is shown by [[Inverse of Generator of Cyclic Group is Generator]]. +Futhermore: +:$\gen e = \set e \ne G$ +By definition of [[Definition:Infinite Cyclic Group|infinite cyclic group]]: +:$g^i \ne g^j$ for all $i \ne j$ +Let $n \in Z \setminus \set {-1, 0, 1}$. +Then: +:$\gen {g^n} = \set {\ldots, g^{-2 n}, g^{-n}, e, g^n, g^{2 n}, \ldots}$ +But since $\order n > 1$, none of these elements is equal to $g$, because $1 \notin n \Z$. +So: +:$g \notin \gen {g^n} \implies \gen {g^n} \ne \gen g$ +{{qed}} +\end{proof}<|endoftext|> +\section{Subgroup of Integers is Ideal} +Tags: Subgroups, Integers, Ideal Theory + +\begin{theorem} +Let $\struct {\Z, +}$ be the [[Definition:Additive Group of Integers|additive group of integers]]. +Every [[Definition:Subgroup|subgroup]] of $\struct {\Z, +}$ is an [[Definition:Ideal of Ring|ideal]] of the [[Definition:Ring (Abstract Algebra)|ring]] $\struct {\Z, +, \times}$. +\end{theorem} + +\begin{proof} +Let $H$ be a subgroup of $\struct {\Z, +}$. +Let $n \in \Z, h \in H$. +Then from the definition of [[Definition:Cyclic Group|cyclic group]] and [[Index Laws for Monoids/Negative Index|Negative Index Law for Monoids]]: +:$n h = n \cdot h \in \gen h \subseteq H$ +The result follows. +{{Qed}} +\end{proof}<|endoftext|> +\section{Additive Group of Integers is Subgroup of Reals} +Tags: Additive Group of Integers, Additive Group of Real Numbers + +\begin{theorem} +Let $\struct {\Z, +}$ be the [[Definition:Additive Group of Integers|additive group of integers]]. +Let $\struct {\R, +}$ be the [[Definition:Additive Group of Real Numbers|additive group of real numbers]]. +Then $\struct {\Z, +}$ is a [[Definition:Subgroup|subgroup]] of $\struct {\R, +}$. +\end{theorem} + +\begin{proof} +From [[Additive Group of Integers is Subgroup of Rationals]], $\struct {\Z, +}$ is a [[Definition:Subgroup|subgroup]] of $\struct {\Q, +}$. +From [[Additive Group of Rationals is Subgroup of Reals]], $\struct {\Q, +}$ is a [[Definition:Subgroup|subgroup]] of $\struct {\R, +}$. +Thus $\struct {\Z, +}$ is a [[Definition:Subgroup|subgroup]] of $\struct {\R, +}$. +{{qed}} +\end{proof}<|endoftext|> +\section{Quotient Group of Reals by Integers is Circle Group} +Tags: Integers, Real Numbers, Complex Numbers, Quotient Groups, Circle Group + +\begin{theorem} +Let $\struct {\Z, +}$ be the [[Definition:Additive Group of Integers|additive group of integers]]. +Let $\struct {\R, +}$ be the [[Definition:Additive Group of Real Numbers|additive group of real numbers]]. +Let $K$ be the [[Definition:Circle Group|circle group]]. +Then the [[Definition:Quotient Group|quotient group]] of $\struct {\R, +}$ by $\struct {\Z, +}$ is [[Definition:Isomorphism (Abstract Algebra)|isomorphic]] to $K$. +\end{theorem} + +\begin{proof} +Define $\phi: \R / \Z \to K$ by: +:$\map \phi {x + \Z} = \map \exp {2 \pi i x}$ +Then $\phi$ is [[Definition:Well-Defined Mapping|well-defined]]. +For, if $x + \Z = y + \Z$, then $y = x + n$ for some $n \in \Z$, and: +:$\map \exp {2 \pi i \paren {x + n} } = \map \exp {2 \pi i x}$ +by [[Complex Exponential Function has Imaginary Period]]. +Moreover, by [[Exponential of Sum/Complex Numbers|Exponential of Sum]]: +:$\map \exp {x + y + \Z} = \map \exp {2 \pi i \paren {x + y} } = \map \exp {2 \pi i x} \, \map \exp {2 \pi i y}$ +meaning $\phi$ is a [[Definition:Group Homomorphism|group homomorphism]]. +By [[Euler's Formula]]: +:$\map \exp {2 \pi i x} = \map \cos {2 \pi i x} + i \, \map \sin {2 \pi i x}$ +so that, by [[Sine and Cosine are Periodic on Reals]]: +:$\map \phi x = 1$ {{iff}} $x + \Z = 0 + \Z$ +Hence, by [[Kernel is Trivial iff Monomorphism/Group|Kernel is Trivial iff Monomorphism]], $\phi$ is a [[Definition:Group Monomorphism|monomorphism]]. +From [[Definition:Polar Form of Complex Number|polar form for complex numbers]], it follows that all $z \in \C$ with $\cmod z = 1$ are of the form: +:$z = \map \exp {2 \pi i x}$ +for some $x \in \R$. +Hence $\phi$ is also an [[Definition:Group Epimorphism|epimorphism]]. +Thus $\phi: \R / \Z \to K$ is a [[Definition:Group Isomorphism|group isomorphism]]. +{{qed}} +\end{proof}<|endoftext|> +\section{Integers Modulo m under Addition form Cyclic Group} +Tags: Additive Groups of Integers Modulo m, Modulo Addition + +\begin{theorem} +Let $\Z_m$ be the set of [[Definition:Integers Modulo m|integers modulo $m$]]. +Let $+_m$ be the [[Definition:Binary Operation|operation]] of [[Definition:Modulo Addition|addition modulo $m$]]. +Let $\struct {\Z_m, +_m}$ denote the [[Definition:Additive Group of Integers Modulo m|additive group of integers modulo $m$]]. +Then $\struct {\Z_m, +_m}$ is a [[Definition:Cyclic Group|cyclic group]] of [[Definition:Order of Structure|order]] $m$, [[Definition:Generator of Cyclic Group|generated]] by the element $\eqclass 1 m \in \Z_m$. +\end{theorem} + +\begin{proof} +From the definition of [[Definition:Integers Modulo m|integers modulo $m$]], we have: +:$\Z_m = \dfrac \Z {\RR_m} = \set {\eqclass 0 m, \eqclass 1 m, \ldots, \eqclass {m - 1} m}$ +It is established that [[Modulo Addition is Well-Defined]]: +:$\eqclass a m +_m \eqclass b m = \eqclass {a + b} m$ +The [[Definition:Group Axioms|group axioms]] are fulfilled: +: '''{{GroupAxiom|0}}''': [[Modulo Addition is Closed|Addition modulo $m$ is closed]]. +: '''{{GroupAxiom|1}}''': [[Modulo Addition is Associative|Addition modulo $m$ is associative]]. +: '''{{GroupAxiom|2}}''': The [[Modulo Addition has Identity|identity element of $\struct {\Z_m, +_m}$]] is $\eqclass 0 m$. +: '''{{GroupAxiom|3}}''': The [[Modulo Addition has Inverses|inverse of $\eqclass k m \in \Z_m$]] is $-\eqclass k m = \eqclass {-k} m = \eqclass {n - k} m$. +: Commutativity: [[Modulo Addition is Commutative|Addition modulo $m$ is commutative]]. +From [[Integers under Addition form Infinite Cyclic Group]] and [[Quotient Group of Cyclic Group]], $\struct {\dfrac \Z {\RR_m}, +_m}$ is [[Definition:Cyclic Group|cyclic]] [[Definition:Order of Group|order]] $m$. +{{Qed}} +\end{proof}<|endoftext|> +\section{Integers Modulo m under Multiplication form Commutative Monoid} +Tags: Modulo Multiplication, Examples of Monoids + +\begin{theorem} +The [[Definition:Algebraic Structure|structure]]: +:$\struct {\Z_m, \times}$ +(where $\Z_m$ is the set of [[Definition:Integers Modulo m|integers modulo $m$]]) is a [[Definition:Commutative Monoid|commutative monoid]]. +\end{theorem} + +\begin{proof} +[[Modulo Multiplication is Closed|Multiplication modulo $m$ is closed]]. +[[Modulo Multiplication is Associative|Multiplication modulo $m$ is associative]]. +[[Modulo Multiplication has Identity|Multiplication modulo $m$ has an identity]]: +:$\forall k \in \Z: \eqclass k m \eqclass 1 m = \eqclass k m = \eqclass 1 m \eqclass k m$ +This [[Identity of Monoid is Unique|identity is unique]]. +[[Modulo Multiplication is Commutative|Multiplication modulo $m$ is commutative]]. +Thus all the conditions are fulfilled for this to be a [[Definition:Commutative Monoid|commutative monoid]]. +{{qed}} +\end{proof}<|endoftext|> +\section{Multiplicative Inverse in Ring of Integers Modulo m} +Tags: Ring of Integers Modulo m, Multiplicative Inverse in Ring of Integers Modulo m + +\begin{theorem} +Let $\struct {\Z_m, +_m, \times_m}$ be the [[Definition:Ring of Integers Modulo m|ring of integers modulo $m$]]. +Then $\eqclass k m \in \Z_m$ has an [[Definition:Multiplicative Inverse|inverse]] in $\struct {\Z_m, \times_m}$ {{iff}} $k \perp m$. +\end{theorem} + +\begin{proof} +First, suppose $k \perp m$. +That is: +:$\gcd \set {k, m} = 1$ +Then, by [[Bézout's Identity]]: +:$\exists u, v \in \Z: u k + v m = 1$ +Thus: +:$\eqclass {u k + v m} m = \eqclass {u k} m = \eqclass u m \eqclass k m = \eqclass 1 m$ +Thus $\eqclass u m$ is an [[Definition:Multiplicative Inverse|inverse]] of $\eqclass k m$. +Suppose that: +:$\exists u \in \Z: \eqclass u m \eqclass k m = \eqclass {u k} m = 1$. +Then: +:$u k \equiv 1 \pmod m$ +and: +:$\exists v \in \Z: u k + v m = 1$ +Thus from [[Bézout's Identity]]: +:$k \perp m$ +{{qed}} +\end{proof} + +\begin{proof} +From [[Ring of Integers Modulo m is Ring]], $\left({\Z_m, +_m, \times_m}\right)$ is a [[Definition:Commutative and Unitary Ring|commutative ring with unity $\left[\!\left[{1}\right]\!\right]_m$]]. +Thus by definition $\left({\Z_m, \times_m}\right)$ is a [[Definition:Commutative Monoid|commutative monoid]]. +The result follows from [[Multiplicative Inverse in Monoid of Integers Modulo m]]. +{{qed}} +\end{proof}<|endoftext|> +\section{Reduced Residue System is Subset of Set of All Residue Classes} +Tags: Residue Classes, Reduced Residue Systems + +\begin{theorem} +Let $\Z_m$ be the set of [[Definition:Set of Residue Classes|set of residue classes modulo $m$]]. +Let $\Z'_m$ be the [[Definition:Reduced Residue System|reduced residue system modulo $m$]]. +Then: +:$\forall m \in \Z_{> 1}: \O \subset \Z'_m \subset \Z_m$ +\end{theorem} + +\begin{proof} +By definition of [[Definition:Reduced Residue System|reduced residue system modulo $m$]]: +:$\Z'_m = \set {x \in \Z_m: x \perp m}$ +From [[Subset of Set with Propositional Function]]: +:$\Z'_m \subseteq \Z_m$ +We have that: +:$\gcd \set {m, 0} = m$ +Thus it follows that: +:$m > 1 \implies \gcd \set {m, 0} \ne 1$ +So: +:$\eqclass 0 m \notin \Z'_m$ +However: +:$\eqclass 0 m \in \Z_m$ +so: +:$\Z'_m \ne \Z_m$ +Thus: +:$\Z'_m \subset \Z_m$ +Then: +:$\eqclass 1 m = 1 \implies 1 \perp m$ +So: +:$\forall m \in \Z: \eqclass 1 m \in \Z'_m$ +Thus: +:$\Z'_m \ne \O$ +and therefore: +:$\O \subset \Z'_m$ +{{qed}} +[[Category:Residue Classes]] +[[Category:Reduced Residue Systems]] +6472l2hgwcmn9kek2nctjmqg87ke7g6 +\end{proof}<|endoftext|> +\section{Reduced Residue System under Multiplication forms Abelian Group} +Tags: Reduced Residue Systems, Examples of Abelian Groups, Multiplicative Groups of Reduced Residues, Reduced Residue System under Multiplication forms Abelian Group + +\begin{theorem} +Let $\Z_m$ be the set of [[Definition:Set of Residue Classes|set of residue classes modulo $m$]]. +Let $\struct {\Z'_m, \times}$ denote the [[Definition:Multiplicative Group of Reduced Residues|multiplicative group of reduced residues modulo $m$]]. +Then $\struct {\Z'_m, \times}$ is an [[Definition:Abelian Group|abelian group]], precisely equal to the [[Definition:Group of Units|group of units]] of $\Z_m$. +\end{theorem} + +\begin{proof} +From [[Ring of Integers Modulo m is Ring]], $\struct {\Z_m, +, \times}$ forms a [[Definition:Commutative and Unitary Ring|(commutative) ring with unity]]. +Then we have that the [[Group of Units is Group|units of a ring with unity form a group]]. +By [[Multiplicative Inverse in Ring of Integers Modulo m]] we have that the elements of $\struct {\Z'_m, \times}$ are precisely those that have [[Definition:Inverse Element|inverses]], and are therefore the [[Definition:Unit of Ring|units]] of $\struct {\Z_m, +, \times}$. +The fact that $\struct {\Z'_m, \times}$ is [[Definition:Abelian Group|abelian]] follows from [[Restriction of Commutative Operation is Commutative]]. +{{qed}} +\end{proof} + +\begin{proof} +Taking the [[Definition:Group Axioms|group axioms]] in turn: +=== $\text G 0$: Closure === +From [[Modulo Multiplication on Reduced Residue System is Closed]]: +:$\struct {\Z'_m, \times}$ is [[Definition:Closed Algebraic Structure|closed]]. +{{qed|lemma}} +=== $\text G 1$: Associativity === +We have that [[Modulo Multiplication is Associative]]. +{{qed|lemma}} +=== $\text G 2$: Identity === +From [[Modulo Multiplication has Identity]], $\eqclass 1 m$ is the [[Definition:Identity Element|identity element]] of $\struct {\Z'_m, \times_m})$. +{{qed|lemma}} +=== $\text G 3$: Inverses === +From [[Multiplicative Inverse in Monoid of Integers Modulo m]], $\eqclass k m \in \Z_m$ has an [[Definition:Inverse Element|inverse]] in $\struct {\Z_m, \times_m}$ {{iff}} $k$ is [[Definition:Coprime Integers|coprime]] to $m$. +Thus every element of $\struct {\Z'_m, \times_m}$ has an [[Definition:Inverse Element|inverse]]. +{{qed|lemma}} +All the [[Definition:Group Axioms|group axioms]] are thus seen to be fulfilled, and so $\struct {\Z'_m, \times_m}$ is a [[Definition:Group|group]]. +{{qed}} +\end{proof} + +\begin{proof} +Taking the [[Definition:Finite Group Axioms|finite group axioms]] in turn: +=== $\text {FG} 0$: Closure === +From [[Modulo Multiplication on Reduced Residue System is Closed]]: +:$\struct {\Z'_m, \times}$ is [[Definition:Closed Algebraic Structure|closed]]. +{{qed|lemma}} +=== $\text {FG} 1$: Associativity === +We have that [[Modulo Multiplication is Associative]]. +{{qed|lemma}} +=== $\text {FG} 2$: Finiteness === +The order of $\struct {\Z'_m, \times}$ is $\map \phi n$ by definition, where $\map \phi n$ denotes the [[Definition:Euler Phi Function|Euler $\phi$ function]]. +As $\map \phi n < n$ it follows that $\struct {\Z'_m, \times}$ is of [[Definition:Finite Order (Structure)|finite order]]. +{{qed|lemma}} +=== $\text {FG} 3$: Cancellability === +We have that [[Modulo Multiplication on Reduced Residue System is Cancellable]]. +{{qed|lemma}} +Thus all the [[Definition:Finite Group Axioms|finite group axioms]] are fulfilled, and $\struct {\Z'_m, \times}$ is a [[Definition:Group|group]]. +It remains to note that [[Modulo Multiplication is Commutative]] to confirm that $\struct {\Z'_m, \times}$ is [[Definition:Abelian Group|abelian]]. +{{qed}} +\end{proof}<|endoftext|> +\section{Ring of Integers Modulo Prime is Field} +Tags: Ring of Integers Modulo m, Galois Fields, Ring of Integers Modulo Prime is Field + +\begin{theorem} +Let $m \in \Z: m \ge 2$. +Let $\struct {\Z_m, +, \times}$ be the [[Definition:Ring of Integers Modulo m|ring of integers modulo $m$]]. +Then: +:$m$ is [[Definition:Prime Number|prime]] +{{iff}}: +:$\struct {\Z_m, +, \times}$ is a [[Definition:Field (Abstract Algebra)|field]]. +\end{theorem} + +\begin{proof} +=== Prime Modulus === +$\struct {\Z_m, +, \times}$ is a [[Definition:Commutative Ring with Unity|commutative ring with unity]] by definition. +From [[Reduced Residue System under Multiplication forms Abelian Group]], $\struct {\Z'_m, \times}$ is an [[Definition:Abelian Group|abelian group]]. +$\Z'_m$ consists of all the elements of $\Z_m$ [[Definition:Coprime Integers|coprime]] to $m$. +Now when $m$ is [[Definition:Prime Number|prime]], we have, from [[Reduced Residue System Modulo Prime]]: +:$\Z'_m = \set {\eqclass 1 m, \eqclass 2 m, \ldots, \eqclass {m - 1} m}$ +That is: +:$\Z'_m = \Z_m \setminus \set {\eqclass 0 m}$ +where $\setminus$ denotes [[Definition:Set Difference|set difference]]. +Hence the result. +{{qed|lemma}} +=== Composite Modulus === +Now suppose $m \in \Z: m \ge 2$ is [[Definition:Composite Number|composite]]. +From [[Ring of Integers Modulo Composite is not Integral Domain]], $\struct {\Z_m, +, \times}$ is not an [[Definition:Integral Domain|integral domain]]. +From [[Field is Integral Domain]] $\struct {\Z_m, +, \times}$ is not a [[Definition:Field (Abstract Algebra)|field]]. +{{qed}} +\end{proof} + +\begin{proof} +Let $p$ be [[Definition:Prime Number|prime]]. +From [[Irreducible Elements of Ring of Integers]], we have that $p$ is [[Definition:Irreducible Element of Ring|irreducible]] in the [[Definition:Ring of Integers|ring of integers]] $\struct {\Z, +, \times}$. +From [[Ring of Integers is Principal Ideal Domain]], $\struct {\Z, +, \times}$ is a [[Definition:Principal Ideal Domain|principal ideal domain]]. +Thus by [[Principal Ideal of Principal Ideal Domain is of Irreducible Element iff Maximal]], $\ideal p$ is a [[Definition:Maximal Ideal of Ring|maximal ideal]] of $\struct {\Z, +, \times}$. +Hence by [[Maximal Ideal iff Quotient Ring is Field]], $\Z / \ideal p$ is a [[Definition:Field (Abstract Algebra)|field]]. +But $\Z / \ideal p$ is exactly $\struct {\Z_p, +, \times}$. +{{qed|lemma}} +Let $p$ be [[Definition:Composite Number|composite]]. +Then $p$ is not [[Definition:Irreducible Element of Ring|irreducible]] in $\struct {\Z, +, \times}$. +Thus by [[Principal Ideal of Principal Ideal Domain is of Irreducible Element iff Maximal]], $\ideal p$ is not a [[Definition:Maximal Ideal of Ring|maximal ideal]] of $\struct {\Z, +, \times}$. +Hence by [[Maximal Ideal iff Quotient Ring is Field]], $\Z / \ideal p$ is not a [[Definition:Field (Abstract Algebra)|field]]. +{{qed}} +\end{proof} + +\begin{proof} +Let $m$ be [[Definition:Prime Number|prime]]. +From [[Ring of Integers Modulo Prime is Integral Domain]], $\struct {\Z_m, +, \times}$ is an [[Definition:Integral Domain|integral domain]]. +From [[Finite Integral Domain is Galois Field]], $\struct {\Z_m, +, \times}$ is a [[Definition:Field (Abstract Algebra)|field]]. +{{qed|lemma}} +Now suppose $m \in \Z: m \ge 2$ is [[Definition:Composite Number|composite]]. +From [[Ring of Integers Modulo Composite is not Integral Domain]], $\struct {\Z_m, +, \times}$ is not an [[Definition:Integral Domain|integral domain]]. +From [[Field is Integral Domain]] $\struct {\Z_m, +, \times}$ is not a [[Definition:Field (Abstract Algebra)|field]]. +{{qed}} +\end{proof}<|endoftext|> +\section{Subgroups of Additive Group of Integers} +Tags: Subgroups, Additive Group of Integers, Additive Groups of Integer Multiples, Subgroups of Additive Group of Integers + +\begin{theorem} +Let $\struct {\Z, +}$ be the [[Definition:Additive Group of Integers|additive group of integers]]. +Let $n \Z$ be the [[Definition:Additive Group of Integer Multiples|additive group of integer multiples]] of $n$. +Every [[Definition:Non-Trivial Group|non-trivial]] [[Definition:Subgroup|subgroup]] of $\struct {\Z, +}$ has the form $n \Z$. +\end{theorem} + +\begin{proof} +First we note that, from [[Integer Multiples under Addition form Infinite Cyclic Group]], $\struct {n \Z, +}$ is an [[Definition:Infinite Group|infinite]] [[Definition:Cyclic Group|cyclic group]]. +From [[Cyclic Group is Abelian]], it follows that $\struct {n \Z, +}$ is an [[Definition:Infinite Group|infinite]] [[Definition:Abelian Group|abelian group]]. +Let $H$ be a [[Definition:Non-Trivial Group|non-trivial]] [[Definition:Subgroup|subgroup]] of $\struct {\Z, +}$. +Because $H$ is [[Definition:Non-Trivial Group|non-trivial]]: +:$\exists m \in \Z: m \in H: m \ne 0$ +Because $H$ is itself a [[Definition:Group|group]]: +:$-m \in H$ +So either $m$ or $-m$ is [[Definition:Positive Integer|positive]] and therefore in $\Z_{>0}$. +Thus: +:$H \cap \Z_{>0} \ne \O$ +From the [[Well-Ordering Principle]], $H \cap \Z_{>0}$ has a [[Definition:Smallest Element|smallest element]], which we can call $n$. +It follows from [[Subgroup of Infinite Cyclic Group is Infinite Cyclic Group]] that: +: $\forall a \in \Z: a n \in H$ +Thus: +: $n \Z \subseteq H$ +{{AimForCont}}: +:$\exists m \in \Z: m \in H \setminus n \Z$ +Then $m \ne 0$, and also $-m \in H \setminus n \Z$. +Assume $m > 0$, otherwise we consider $-m$. +By the [[Division Theorem]]: +:$m = q n + r$ +If $r = 0$, then $m = q n \in n \Z$, so $0 \le r < n$. +Now this means $r = m - q n \in H$ and $0 \le r < n$. +This would mean $n$ was not the [[Definition:Smallest Element|smallest element]] of $H \cap \Z$. +Hence, by [[Proof by Contradiction]], there can be no such $m \in H \setminus n \Z$. +Thus: +:$H \setminus n \Z = \O$ +Thus from [[Set Difference with Superset is Empty Set]]: +:$H \subseteq n \Z$ +Thus we have $n \Z \subseteq H$ and $H \subseteq n \Z$. +Hence: +:$H = n \Z$ +{{qed}} +\end{proof}<|endoftext|> +\section{Integer Multiples under Addition form Infinite Cyclic Group} +Tags: Sets of Integer Multiples, Additive Groups of Integer Multiples, Examples of Cyclic Groups + +\begin{theorem} +Let $n \Z$ be the [[Definition:Set of Integer Multiples|set of integer multiples]] of $n$. +Then $\struct {n \Z, +}$ is a [[Definition:Countably Infinite|countably infinite]] [[Definition:Cyclic Group|cyclic group]]. +It is generated by $n$ and $-n$: +:$n \Z = \gen n$ +:$n \Z = \gen {-n}$ +Hence $\struct {n \Z, +}$ can be justifiably referred to as the [[Definition:Additive Group of Integer Multiples|additive group of integer multiples]]. +\end{theorem} + +\begin{proof} +From [[Integer Multiples under Addition form Subgroup of Integers]], $\struct {n \Z, +}$ is a [[Definition:Subgroup|subgroup]] of the [[Definition:Additive Group of Integers|additive group of integers]] $\struct {\Z, +}$. +From [[Integers under Addition form Infinite Cyclic Group]], $\struct {\Z, +}$ is a [[Definition:Cyclic Group|cyclic group]]. +So by [[Subgroup of Cyclic Group is Cyclic]], $\struct {n \Z, +}$ is a [[Definition:Cyclic Group|cyclic group]]. +The final assertions follow from [[Subgroup of Infinite Cyclic Group is Infinite Cyclic Group]]. +{{qed}} +\end{proof}<|endoftext|> +\section{Quotient Group of Integers by Multiples} +Tags: Quotient Groups, Modulo Arithmetic, Additive Group of Integers, Additive Groups of Integer Multiples, Additive Groups of Integers Modulo m + +\begin{theorem} +Let $\struct {\Z, +}$ be the [[Definition:Additive Group of Integers|additive group of integers]]. +Let $\struct {m \Z, +}$ be the [[Definition:Additive Group of Integer Multiples|additive group of integer multiples]] of $m$. +Let $\struct {\Z_m, +_m}$ be the [[Definition:Additive Group of Integers Modulo m|additive group of integers modulo $m$]]. +Then the [[Definition:Quotient Group|quotient group]] of $\struct {\Z, +}$ by $\struct {m \Z, +}$ is $\struct {\Z_m, +_m}$. +Thus: +:$\index \Z {m \Z} = m$ +\end{theorem} + +\begin{proof} +From [[Subgroups of Additive Group of Integers]], $\struct {m \Z, +}$ is a [[Definition:Subgroup|subgroup]] of $\struct {\Z, +}$. +From [[Subgroup of Abelian Group is Normal]], $\struct {m \Z, +}$ is [[Definition:Normal Subgroup|normal]] in $\struct {\Z, +}$. +Therefore the [[Definition:Quotient Group|quotient group]] $\dfrac {\struct {\Z, +} } {\struct {m \Z, +} }$ is defined. +Now $\Z$ modulo $m \Z$ is [[Definition:Congruence Modulo Subgroup|Congruence Modulo a Subgroup]]. +This is merely [[Definition:Congruence Modulo Integer|congruence modulo an integer]]. +Thus the [[Definition:Quotient Set|quotient set]] $\Z / m \Z$ is $\Z_m$. +The [[Definition:Left Coset|left coset]] of $k \in \Z$ is denoted $k + m \Z$, which is the same thing as $\eqclass k m$ from the definition of [[Definition:Residue Class|residue class]]. +So $\index \Z {m \Z} = m$ follows from the definition of [[Definition:Index of Subgroup|Subgroup Index]]. +{{qed}} +\end{proof}<|endoftext|> +\section{Euler's Theorem} +Tags: Number Theory + +\begin{theorem} +Let $a, m \in \Z$ be [[Definition:Coprime Integers|coprime integers]]: $a \perp m$. +Let $\map \phi m$ be the [[Definition:Euler Phi Function|Euler $\phi$ function]] of $m$. +Then: +:$a^{\map \phi m} \equiv 1 \pmod m$ +\end{theorem} + +\begin{proof} +Let $\eqclass a m$ denote the [[Definition:Residue Class|residue class modulo $m$ of $a$]]. +Since $a \perp m$, it follows by [[Reduced Residue System under Multiplication forms Abelian Group]] that $\eqclass a m$ belongs to the [[Definition:Abelian Group|abelian group]] $\struct {\Z'_m, \times}$. +Let $k = \order {\eqclass a m}$ where $\order {\, \cdot \,}$ denotes the [[Definition:Order of Group Element|order of a group element]]. +By [[Order of Element Divides Order of Finite Group]]: +:$k \divides \order {\Z'_m}$ +By the definition of the [[Definition:Euler Phi Function|Euler $\phi$ function]]: +:$\order {\Z'_m} = \map \phi m$ +Thus: +{{begin-eqn}} +{{eqn | l = \eqclass a m^k + | r = \eqclass a m + | c = {{Defof|Order of Group Element}} +}} +{{eqn | ll= \leadsto + | l = \eqclass a m^{\map \phi m} + | r = \eqclass {a^{\map \phi m} } m + | c = [[Congruence of Powers]] +}} +{{eqn | r = \eqclass 1 m + | c = +}} +{{eqn | ll= \leadsto + | l = a^{\map \phi m} + | o = \equiv + | r = 1 \pmod m + | c = {{Defof|Residue Class}} +}} +{{end-eqn}} +{{qed}} +{{Namedfor|Leonhard Paul Euler|cat = Euler}} +\end{proof}<|endoftext|> +\section{Symmetry Group is Group} +Tags: Symmetry Groups + +\begin{theorem} +Let $P$ be a [[Definition:Geometric Figure|geometric figure]]. +Let $S_P$ be the [[Definition:Set|set]] of all [[Definition:Symmetry Mapping|symmetries]] of $P$. +Let $\circ$ denote [[Definition:Composition of Mappings|composition of mappings]]. +The [[Definition:Symmetry Group|symmetry group]] $\struct {S_P, \circ}$ is indeed a [[Definition:Group|group]]. +\end{theorem} + +\begin{proof} +By definition, a [[Definition:Symmetry Mapping|symmetry mapping]] is a [[Definition:Bijection|bijection]], and hence a [[Definition:Permutation|permutation]]. +From [[Symmetric Group is Group]], the [[Definition:Set|set]] of all [[Definition:Permutation|permutations]] on $P$ form the [[Definition:Symmetric Group|symmetric group]] $\struct {\map \Gamma P, \circ}$ on $P$. +Thus $S_P$ is a [[Definition:Subset|subset]] of $\struct {\map \Gamma P, \circ}$. +Let $A$ and $B$ be [[Definition:Symmetry Mapping|symmetry mappings]] on $P$. +From [[Composition of Symmetries is Symmetry]], $A \circ B$ is also a [[Definition:Symmetry Mapping|symmetry mapping]] on $P$. +Also, by the definition of [[Definition:Symmetry Mapping|symmetry]], $A^{-1}$ is also a [[Definition:Symmetry Mapping|symmetry mapping]] on $P$. +Thus we have: +:$A, B \in S_P \implies A \circ B \in S_P$ +:$A \in S_P \implies A^{-1} \in \S_P$ +The result follows from the [[Two-Step Subgroup Test]]. +{{qed}} +\end{proof}<|endoftext|> +\section{Internal Angles of Square} +Tags: Squares + +\begin{theorem} +The [[Definition:Internal Angle|internal angles]] of a [[Definition:Square (Geometry)|square]] are [[Definition:Right Angle|right angles]]. +\end{theorem} + +\begin{proof} +By definition, a [[Definition:Square (Geometry)|square]] is a [[Definition:Regular Polygon|regular]] [[Definition:Quadrilateral|quadrilateral]]. +From [[Internal Angles of Regular Polygon]], the [[Definition:Internal Angle|internal angles]] of a [[Definition:Square (Geometry)|square]] measure $\dfrac {180 \degrees \paren {4 - 2} } 4 = 90 \degrees$. +The result follows from [[Measurement of Right Angle]]. +{{qed}} +[[Category:Squares]] +n7tjkuonqy6jbbedb9iv3obmyjyrq54 +\end{proof}<|endoftext|> +\section{Area of Square} +Tags: Areas of Quadrilaterals, Area of Square + +\begin{theorem} +A [[Definition:Square (Geometry)|square]] has an [[Definition:Area|area]] of $L^2$ where $L$ is the [[Definition:Length of Line|length]] of a [[Definition:Side of Polygon|side]] of the square. +Thus we have that the [[Definition:Area|area]] is a [[Definition:Real Function|function]] of the [[Definition:Length of Line|length]] of the [[Definition:Side of Polygon|side]]: +:$\forall L \in \R_{\ge 0}: \map \Area L = L^2$ +where it is noted that the [[Definition:Domain of Mapping|domain]] of $L$ is the [[Definition:Positive Real Number|set of non-negative real numbers]]. +\end{theorem} + +\begin{proof} +Let $\Box ABCD$ be a [[Definition:Square (Geometry)|square]] whose [[Definition:Side of Polygon|side]] $AB$ is of [[Definition:Linear Measure|length]] $L$. +Let $\Box EFGH$ be a [[Definition:Square (Geometry)|square]] whose [[Definition:Side of Polygon|side]] $EF$ is of [[Definition:Linear Measure|length]] $1$. +:[[File:AreaOfSquare.png|300px]] +From the [[Axiom:Area Axioms|Axioms of Area]], the [[Definition:Area|area]] of $\Box EFGH$ is $1$. +By definition, $AB : EF = L : 1$. +From [[Similar Polygons are composed of Similar Triangles]], the [[Definition:Ratio|ratio]] of the [[Definition:Area|areas]] of $\Box ABCD$ to $\Box EFGH$ is the [[Definition:Duplicate Ratio|duplicate ratio]] of the [[Definition:Ratio|ratio]] of $AB$ to $EF$. +Thus by definition of [[Definition:Duplicate Ratio|duplicate ratio]]: +: $\Box ABCD : \Box EFGH = \left({AB : EF}\right)^2$ +That is: +: $\dfrac {\Box ABCD} {\Box EFGH} = \left({\dfrac L 1}\right)^2 = L^2$ +That is, the [[Definition:Area|area]] of $\Box ABCD$ has $L^2$ as many units as $\Box EFGH$. +Hence the result. +{{qed}} +\end{proof} + +\begin{proof} +=== Integer Side Length === +In the case where $L = 1$, the statement follows from the [[Definition:Area|definition of area]]. +If $L \in \N, L > 1$, then we can divide the square into smaller squares, each of side length one. +Since there will be $L$ squares of side length one on each side, it follows that there will be $L \cdot L = L^2$ squares of side length one. +Thus, the area of the square of side length $L$ is $L^2 \cdot 1 = L^2$. +{{Qed|lemma}} +=== Rational Side Length === +Let $A$ be the [[Definition:Area|area]] of the [[Definition:Square (Geometry)|square]] $S$ with side length $L$. +If $L$ is a [[Definition:Rational Number|rational number]], then: +:$\exists p, q \in \N: L = \dfrac p q$ +Create a [[Definition:Square (Geometry)|square]] of [[Definition:Side of Polygon|side]] [[Definition:Length of Line|length]] $p$. +From the [[Area of Square/Proof 1#Integer Side Length|integer side length case]], its area equals $p^2$. +Divide the [[Definition:Side of Polygon|sides]] into $q$ equal parts. +Thus the [[Definition:Square (Geometry)|square]] of [[Definition:Side of Polygon|side]] [[Definition:Length of Line|length]] $p$ is divided into $q^2$ small [[Definition:Square (Geometry)|squares]]. +As they all have [[Definition:Side of Polygon|side]] [[Definition:Length of Line|length]] $\dfrac p q$, each of them equals $S$. +It follows arithmetically: +:$A = \dfrac {p^2} {q^2} = \paren {\dfrac p q}^2 = L^2$ +{{Qed|lemma}} +=== Irrational Side Length === +Let $L$ be an [[Definition:Irrational Number|irrational number]]. +Then from [[Rationals are Everywhere Dense in Topological Space of Reals]] we know that within an arbitrarily small distance $\epsilon$ from $L$, we can find a rational number less than $L$ and a rational number greater than $L$. +In formal terms, we have: +:$\forall \epsilon > 0: \exists A, B \in \Q_+: A < L < B: \left|{A - L}\right| < \epsilon, \left|{B - L}\right| < \epsilon$ +Thus: +: $\displaystyle \lim_{\epsilon \to 0^+} A = L$ +: $\displaystyle \lim_{\epsilon \to 0^+} B = L$ +Since a square of side length $B$ can contain a square of side length $L$, which can in turn contain a square of side length $A$, then: +: $\operatorname {area} \Box B \ge \operatorname {area} \Box L \ge \operatorname {area}\Box A$ +By the [[Area of Square/Proof 1#Rational Side Length|result for rational numbers]]: +: $\operatorname {area}\Box B = B^2$ +: $\operatorname {area}\Box A = A^2$ +We also note that: +: $\displaystyle \lim_{B \to L} B^2 = L^2 = \lim_{A \to L} A^2$ +Thus: +: $\displaystyle \lim_{B \to L} \operatorname {area} \Box B = \lim_{B \to L} B^2 = L^2$ +: $\displaystyle \lim_{A \to L} \operatorname {area} \Box A = \lim_{A \to L} A^2 = L^2$ +Finally: +: $L^2 \ge \operatorname {area}\Box L \ge L^2$ +so: +: $\operatorname {area}\Box L = L^2$ +{{Qed}} +\end{proof} + +\begin{proof} +Let a [[Definition:Square (Geometry)|square]] have a [[Definition:Side of Polygon|side length]] $a \in \R$. +This square is equivalent to the area under the graph of $\map f x = a$ from $0$ to $a$. +Thus from [[Definition:Definite Integral/Geometric Interpretation|the geometric interpretation of the definite integral]], the area of the square will be the integral: +:$\displaystyle A = \int_0^a a \rd l$ +Thus: +{{begin-eqn}} +{{eqn | l = A + | r = \int_0^a a \rd l +}} +{{eqn | r = \bigintlimits {l \cdot a} 0 a + | c = [[Integral of Constant]] +}} +{{eqn | r = a \cdot a - 0 \cdot a +}} +{{eqn | r = a^2 +}} +{{end-eqn}} +{{Qed}} +=== Warning === +'''This proof is circular.''' +The use of the [[Definition:Definite Integral|definite integral]] to represent [[Definition:Area|area]] is based on the fact that the [[Area of Rectangle|area of a rectangle]] is the product of the rectangle's [[Definition:Width|width]] and [[Definition:Height (Linear Measure)|height]]. +That fact is in turn derived from this one. +However, this demonstration neatly parallels the integration based proofs of the areas of other figures, for example [[Area of Circle]]. +\end{proof}<|endoftext|> +\section{Order of Symmetric Group} +Tags: Symmetric Groups + +\begin{theorem} +Let $S$ be a [[Definition:Finite Set|finite set]] of [[Definition:Cardinality|cardinality]] $n$. +Let $\struct {\map \Gamma S, \circ}$ be the [[Definition:Symmetric Group|symmetric group]] on $S$. +Then $\struct {\map \Gamma S, \circ}$ has $n!$ elements (see [[Definition:Factorial|factorial]]). +\end{theorem} + +\begin{proof} +A direct application of [[Cardinality of Set of Bijections]]. +{{qed}} +\end{proof}<|endoftext|> +\section{Powers of Permutation Element} +Tags: Symmetric Groups + +\begin{theorem} +Let $S_n$ denote the [[Definition:Symmetric Group on n Letters|symmetric group on $n$ letters]]. +Let $\pi \in S_n$, and let $i \in \N^*_n$. +Let $k \in \Z: k > 0$ be the smallest such that: +: $\pi^k \left({i}\right) \in \left\{{i, \pi \left({i}\right), \pi^2 \left({i}\right), \ldots, \pi^{k-1} \left({i}\right)}\right\}$ +Then $\pi^k \left({i}\right) = i$. +\end{theorem} + +\begin{proof} +Suppose $\pi^k \left({i}\right) = \pi^r \left({i}\right)$ for some $r > 0$. +Then, since $\pi$ has an [[Definition:Inverse Element|inverse]], $\pi^{k - r} \left({i}\right) = i$. +This contradicts the definition of $k$, so $r = 0$. +{{qed}} +[[Category:Symmetric Groups]] +s6busb2smrfp9a2bew0qxesh1oun8w9 +\end{proof}<|endoftext|> +\section{Fixed Elements form 1-Cycles} +Tags: Symmetric Groups + +\begin{theorem} +Let $S_n$ denote the [[Definition:Symmetric Group on n Letters|symmetric group on $n$ letters]]. +Let $\pi \in S_n$. +Let $\Fix \pi$ be the [[Definition:Fixed Element of Permutation|set of elements fixed by $\pi$]]. +For any $\pi \in S_n$, all the elements of $\Fix \pi$ form [[Definition:Cyclic Permutation|$1$-cycles]]. +\end{theorem} + +\begin{proof} +Let $\pi$ be a permutation, and let $x \in \Fix \pi$. +From the definition of a [[Definition:Fixed Element of Permutation|fixed element]], $\map \pi x = x$. +From the definition of a [[Definition:Cyclic Permutation|$k$-cycle]], we see that $1$ is the smallest $k \in \Z: k > 0$ such that $\map {\pi^k} x = x$. +The result follows. +{{qed}} +[[Category:Symmetric Groups]] +5th1obufnxiwimah16mkfvfzxd09v13 +\end{proof}<|endoftext|> +\section{Equality of Cycles} +Tags: Symmetric Groups + +\begin{theorem} +Let $S_n$ denote the [[Definition:Symmetric Group on n Letters|symmetric group on $n$ letters]], realised as the [[Definition:Permutation on n Letters|permutations of $\left\{{1, \ldots, n}\right\}$]]. +Let $\rho = \begin{bmatrix} a_0 & \cdots & a_{k-1} \end{bmatrix}$, $\sigma = \begin{bmatrix} b_0 & \cdots & b_{k-1} \end{bmatrix} \in S_n$ be [[Definition:Cyclic Permutation|$k$-cycles]] of $S_n$. +For any $d \in \Z$, by [[Integer is Congruent to Integer less than Modulus]] we can associate to $d$ a unique [[Definition:Integer|integer]] $\tilde d \in \left\{{0, \ldots, k-1}\right\}$ such that $d \equiv \tilde d \pmod k$. +Define $a_d$ and $b_d$ for any $d \in \Z$ by $a_d = a_{\tilde d}$ and $b_d = b_{\tilde d}$ +Choose $i, j \in \left\{{1, \ldots, k}\right\}$ such that: +:$\displaystyle a_i = \min\left\{ {a_0, \ldots, a_{k-1} }\right\}$ +:$\displaystyle b_j = \min\left\{ {b_0, \ldots, b_{k-1} }\right\}$ +Then $\rho = \sigma$ {{iff}} for all $d \in \Z$, $a_{i + d} = b_{j + d}$. +That is, $\rho = \sigma$ {{iff}} they are identical when written with the lowest element first. +\end{theorem} + +\begin{proof} +{{ProofWanted}} +[[Category:Symmetric Groups]] +p1lu4rvnq1iznqntvshmssa721dm2ur +\end{proof}<|endoftext|> +\section{Identity Permutation is Disjoint from All} +Tags: Symmetric Groups, Identity Mappings + +\begin{theorem} +Let $S_n$ denote the [[Definition:Symmetric Group on n Letters|symmetric group on $n$ letters]]. +Let $e \in S_n$ be the [[Definition:Identity Mapping|identity permutation]] on $S_n$. +Then $e$ is [[Definition:Disjoint Permutations|disjoint]] from every permutation $\pi$ on $S_n$ (including itself). +\end{theorem} + +\begin{proof} +By definition of the [[Definition:Identity Mapping|identity permutation]]: +: $\forall i \in \N_{>0}: e \left({i}\right) = i$ +Thus $e$ [[Definition:Fixed Element of Permutation|fixes]] all elements of $S_n$. +Thus each element [[Definition:Moved Element of Permutation|moved]] by a permutation $\pi$ is [[Definition:Fixed Element of Permutation|fixed]] by $e$. +The set of elements moved by $e$ is $\O$, so the converse is true vacuously. +{{qed}} +[[Category:Symmetric Groups]] +[[Category:Identity Mappings]] +ej17rkvuch2qhbeqa3wmxc2042jco6v +\end{proof}<|endoftext|> +\section{Disjoint Permutations Commute} +Tags: Symmetric Groups, Permutation Theory + +\begin{theorem} +Let $S_n$ denote the [[Definition:Symmetric Group on n Letters|symmetric group on $n$ letters]]. +Let $\rho, \sigma \in S_n$ such that $\rho$ and $\sigma$ are [[Definition:Disjoint Permutations|disjoint]]. +Then $\rho \sigma = \sigma \rho$. +\end{theorem} + +\begin{proof} +Let $\rho$ and $\sigma$ be [[Definition:Disjoint Permutations|disjoint permutations]]. +Let $i \in \Fix \rho$. +Then: +:$\map {\sigma \rho} i = \map \sigma i$ +whereas: +:$\map {\rho \sigma} i = \map \rho {\map \sigma i}$ +{{AimForCont}} $\map \sigma i \notin \Fix \rho$. +Then because $\sigma$ and $\rho$ are [[Definition:Disjoint Permutations|disjoint]] it follows that: +{{begin-eqn}} +{{eqn | l = \map \sigma i + | o = \in + | r = \Fix \sigma +}} +{{eqn | ll= \leadsto + | l = \map {\sigma^2} i + | r = \map \sigma i +}} +{{eqn | ll= \leadsto + | l = \map {\sigma^{-1} \sigma^2} i + | r = \map {\sigma^{-1} \sigma} i +}} +{{eqn | ll= \leadsto + | l = \map \sigma i + | r = i +}} +{{end-eqn}} +But it was previously established that $i \in \Fix \rho$. +This is a [[Definition:Contradiction|contradiction]]. +Therefore: +:$\map \sigma i \in \Fix \rho$ +and so: +:$\map {\rho \sigma} i = \map \sigma i = \map {\sigma \rho} i$ +Let $i \notin \Fix \rho$. +Then: +: $i \in \Fix \sigma$ +and the same proof can be performed with $\rho$ and $\sigma$ exchanged. +{{qed}} +\end{proof}<|endoftext|> +\section{Permutation Induces Equivalence Relation} +Tags: Symmetric Groups, Equivalence Relations + +\begin{theorem} +Let $S_n$ denote the [[Definition:Symmetric Group on n Letters|symmetric group on $n$ letters]]. +Let $\pi \in S_n$. +Let $\mathcal R_\pi$ be the [[Definition:Relation|relation]] defined by: +:$i \mathrel {\mathcal R_\pi} j \iff \exists k \in \Z: \map {\pi^k} i = j$ +Then $\mathcal R_\pi$ is an [[Definition:Equivalence Relation|equivalence relation]]. +\end{theorem} + +\begin{proof} +Let $\pi \in S_n$. +First we note that, from [[Element of Finite Group is of Finite Order]], every [[Definition:Element|element]] of a [[Definition:Finite Group|finite group]] has [[Definition:Finite Order Element|finite order]]. +Thus $\pi$ has [[Definition:Finite Order Element|finite order]], so: +:$\exists r \in \Z: \pi^r = e$ +Checking in turn each of the criteria for [[Definition:Equivalence Relation|equivalence]]: +=== Reflexive === +From above, $\exists r \in \Z: \pi^r = e$. +Therefore $\exists k \in \Z: \map {\pi^k} i = i$. +So $\mathcal R_\pi$ is [[Definition:Reflexive Relation|reflexive]]. +{{qed|lemma}} +=== Symmetric === +Let $\map {\pi^k} i = j$. +Because $\pi^r = e$, we have $\map {\pi^r} j = j$ (from above). +Thus $\map {\pi^{r - k} } j = i$. +So $\mathcal R_\pi$ is [[Definition:Symmetric Relation|symmetric]]. +{{qed|lemma}} +=== Transitive === +Let $\map {\pi^{s_1} } i = j, \map {\pi^{s_2} } j = k$. +Then $\map {\pi^{s_1 + s_2} } i = k$. +So $\mathcal R_\pi$ is [[Definition:Transitive Relation|transitive]]. +{{qed|lemma}} +All criteria are met, and so $\mathcal R_\pi$ is an [[Definition:Equivalence Relation|equivalence relation]]. +{{qed}} +[[Category:Symmetric Groups]] +[[Category:Equivalence Relations]] +rjo3omx45c5ldvuigd6uen9pr0jdghw +\end{proof}<|endoftext|> +\section{Existence and Uniqueness of Cycle Decomposition} +Tags: Symmetric Groups, Permutation Theory + +\begin{theorem} +Let $S_n$ denote the [[Definition:Symmetric Group on n Letters|symmetric group on $n$ letters]]. +Every [[Definition:Element|element]] of $S_n$ may be [[Definition:Unique|uniquely expressed]] as a [[Definition:Cycle Decomposition|cycle decomposition]], up to the order of factors. +\end{theorem} + +\begin{proof} +By definition, a [[Definition:Cycle Decomposition|cycle decomposition]] of an [[Definition:Element|element]] of $S_n$ is a product of [[Definition:Disjoint Permutations|disjoint]] [[Definition:Cyclic Permutation|cycles]]. +=== Construction of Disjoint Permutations === +Let $\sigma \in S_n$ be a [[Definition:Permutation|permutation]] on $S_n$. +Let $\mathcal R_\sigma$ be the [[Definition:Equivalence Relation|equivalence]] defined in [[Permutation Induces Equivalence Relation]]. +Let $\N_n$ be used to denote the [[Definition:Initial Segment of One-Based Natural Numbers|(one-based) initial segment of natural numbers]]: +:$\N_n = \closedint 1 n = \set {1, 2, 3, \ldots, n}$ +Let $\N_n / \mathcal R_\sigma = \set {E_1, E_2, \ldots, E_m}$ be the [[Definition:Quotient Set|quotient set]] of $\N_n$ determined by $\mathcal R_\sigma$. +By [[Equivalence Class of Element is Subset]]: +:$E \in \N_n / \mathcal R_\sigma \implies E \subseteq \N_n$ +For any $E_i \in \N_n / \mathcal R_\sigma$, let $\rho_i: \paren {\N_n \setminus E_i} \to \paren {\N_n \setminus E_i}$ be the [[Definition:Identity Mapping|identity mapping]] on $\N_n \setminus E_i$. +By [[Identity Mapping is Permutation]], $\rho_i$ is a [[Definition:Permutation|permutation]]. +Also, let $\phi_i = \tuple {E_i, E_i, R}$ be a [[Definition:Relation|relation]] where $R$ is defined as: +:$\forall x, y \in E_i: \tuple {x, y} \in R \iff \map \sigma x = y$ +It is easily seen that $\phi_i$ is [[Definition:Many-to-One Relation|many to one]]. +For all $x \in E_i$: +{{begin-eqn}} +{{eqn | l = x + | o = \mathcal R_\sigma + | r = \map \sigma x + | c = +}} +{{eqn | ll= \leadsto + | l = \map \sigma x + | o = \in + | r = E_i + | c = +}} +{{eqn | ll= \leadsto + | l = \sigma \sqbrk {E_i} + | o = \subseteq + | r = E_i + | c = +}} +{{end-eqn}} +which shows that $\phi_i$ is [[Definition:Left-Total Relation|left-total]]. +It then follows from the definition of a [[Definition:Mapping|mapping]] that $\phi_i: E_i \to E_i$ is a [[Definition:Mapping|mapping]] defined by: +:$\map {\phi_i} x = \map \sigma x$ +It is seen that $\phi_i$ is an [[Definition:Injection|injection]] because $\sigma$ is an [[Definition:Injection|injection]]. +So by [[Injection from Finite Set to Itself is Surjection/Corollary|Injection from Finite Set to Itself is Permutation]], $\phi_i$ is a [[Definition:Permutation|permutation]] on $E_i$. +By [[Intersection with Relative Complement is Empty]], $E_i$ and $\N_n \setminus E_i$ are [[Definition:Disjoint Sets|disjoint]]. +By [[Union with Relative Complement]]: +: $E_i \cup \paren {\N_n \setminus E_i} = \N_n$ +So by [[Union of Bijections with Disjoint Domains and Codomains is Bijection]], let the [[Definition:Permutation|permutation]] $\sigma_i \in S_n$ be defined by: +:$\map {\sigma_i} x = \map {\paren {\phi_i \cup \rho_i} } x = \begin{cases} +\map \sigma x & : x \in E_i \\ +x & : x \notin E_i +\end{cases}$ +By [[Equivalence Classes are Disjoint]], it follows that each of the $\sigma_i$ are [[Definition:Disjoint Permutations|disjoint]]. +{{qed|lemma}} +=== These Permutations are Cycles === +It is now to be shown that all of the $\sigma_i$ are [[Definition:Cyclic Permutation|cycles]]. +From [[Order of Element Divides Order of Finite Group]], there exists $\alpha \in \Z_{\gt 0}$ such that $\sigma_i^\alpha = e$, and so: +:$\map {\sigma_i^\alpha} x = \map e x = x$ +By the [[Well-Ordering Principle]], let $k = \min \set {\alpha \in \N_{\gt 0}: \map {\sigma_i^\alpha} x = x}$ +Because $\sigma_i$ [[Definition:Fixed Element of Permutation|fixes]] each $y \notin E_i$, it suffices to show that: +:$E_i = \set {x, \map {\sigma_i} x, \ldots, \map {\sigma_i^{k - 1} } x}$ +for some $x \in E_i$. +If $x \in E_i$, then for all $t \in \Z$: +:$x \mathrel {\mathcal R_\sigma} \map {\sigma_i^t} x \implies \map {\sigma_i^t} x \in E_i$ +It has been shown that: +:$(1) \quad \set {x, \map {\sigma_i} x, \ldots, \map {\sigma_i^{k - 1} } x} \subseteq E_i$ +Let $x, y \in E_i$. +Then: +{{begin-eqn}} +{{eqn | l = x + | o = \mathcal R_\sigma + | r = y +}} +{{eqn | ll= \leadsto + | l = \map {\sigma_i^t} x + | r = y + | c = for some $t \in \Z$, by [[Permutation Induces Equivalence Relation]] +}} +{{eqn | ll= \leadsto + | l = \map {\sigma_i^{k q + r} } x + | r = y + | c = for some $q \in \Z$, and $0 \le r \lt k$ by the [[Division Theorem]] +}} +{{eqn | ll= \leadsto + | l = \map {\sigma_i^r \sigma_i^{k q} } x + | r = y +}} +{{eqn | ll= \leadsto + | l = \map {\sigma_i^r} x + | r = y + | c = [[Fixed Point of Permutation is Fixed Point of Power]] +}} +{{end-eqn}} +It has been shown that: +:$(2) \quad E_i \subseteq \set {x, \map {\sigma_i} x, \ldots, \map {\sigma_i^{k - 1} } x}$ +Combining $(1)$ and $(2)$ yields: +:$E_i = \set {x, \map {\sigma_i} x, \ldots, \map {\sigma_i^{k - 1} } x}$ +{{qed|lemma}} +=== The Product of These Cycles form the Permutation === +Finally, it is now to be shown that $\sigma = \sigma_1 \sigma_2 \cdots \sigma_m$. +From the [[Fundamental Theorem on Equivalence Relations]]: +:$x \in \N_n \implies x \in E_j$ +for some $j \in \set {1, 2, \ldots, m}$. +Therefore: +{{begin-eqn}} +{{eqn | l = \map {\sigma_1 \sigma_2 \cdots \sigma_m} x + | r = \map {\sigma_1 \sigma_2 \cdots \sigma_j} x +}} +{{eqn | r = \map {\sigma_j} x + | c = because $\sigma_j \sqbrk {E_j} = E_j$ +}} +{{eqn | r = \map \sigma x + | c = Definition of $\sigma_i$ +}} +{{end-eqn}} +and so existence of a [[Definition:Cycle Decomposition|cycle decomposition]] has been shown. +{{qed|lemma}} +=== Uniqueness of Cycle Decomposition === +Take the [[Definition:Cycle Decomposition|cycle decomposition]] of $\sigma$, which is $\sigma_1 \sigma_2 \cdots \sigma_m$. +Let $\tau_1 \tau_2 \cdots \tau_s$ be some product of [[Definition:Disjoint Permutations|disjoint]] [[Definition:Cyclic Permutation|cycles]] such that $\sigma = \tau_1 \tau_2 \cdots \tau_s$. +It is assume that this product describes $\sigma$ completely and does not contain any duplicate [[Definition:Cyclic Permutation|$1$-cycles]]. +Let $x$ be a [[Definition:Moved Element of Permutation|moved element]] of $\sigma$. +Then there exists a $j \in \set {1, 2, \ldots, s}$ such that $\map {\tau_j} x \ne x$. +And so: +{{begin-eqn}} +{{eqn | l = \map \sigma x + | r = \map {\tau_1 \tau_2 \cdots \tau_j} x +}} +{{eqn | r = \map {\tau_j} x + | c = [[Power of Moved Element is Moved]] +}} +{{end-eqn}} +It has already been shown that $x \in E_i$ for some $i \in \set {1, 2, \ldots, m}$. +Therefore: +{{begin-eqn}} +{{eqn | l = \map {\sigma_i} x + | r = \map {\tau_j} x +}} +{{eqn | l = \map {\sigma_i^2} x + | r = \map {\tau_{j \prime} \tau_j} x + | c = by [[Power of Moved Element is Moved]] +}} +{{eqn | r = \map {\tau_j^2} x + | c = because $\map {\sigma_i^2} x \ne \map {\sigma_i} x$ and this product is [[Definition:Disjoint Permutations|disjoint]] +}} +{{eqn | l = \vdots + | o = \vdots + | r = \vdots +}} +{{eqn | l = \map {\sigma_i^{k - 1} } x + | r = \map {\tau_j^{k - 1} } x +}} +{{end-eqn}} +This effectively shows that $\sigma_i = \tau_j$. +Doing this for every $E_i$ implies that $m = s$ and that there exists a $\rho \in S_m$ such that: +:$\sigma_{\map \rho i} = \tau_i$ +In other words, $\tau_1 \tau_2 \cdots \tau_m$ is just a reordering of $\sigma_1 \sigma_2 \cdots \sigma_m$. +{{qed}} +\end{proof}<|endoftext|> +\section{Powers of Disjoint Permutations} +Tags: Symmetric Groups + +\begin{theorem} +Let $S_n$ denote the [[Definition:Symmetric Group on n Letters|symmetric group on $n$ letters]]. +Let $\rho, \sigma$ be [[Definition:Disjoint Permutations|disjoint permutations]]. +Then: +: $\forall k \in \Z: \paren {\sigma \rho}^k = \sigma^k \rho^k$ +\end{theorem} + +\begin{proof} +A direct application of [[Power of Product of Commutative Elements in Group]], and the fact that [[Disjoint Permutations Commute]]. +{{qed}} +\end{proof}<|endoftext|> +\section{Order of Product of Disjoint Permutations} +Tags: Permutation Theory, Order of Product of Disjoint Permutations + +\begin{theorem} +Let $S_n$ denote the [[Definition:Symmetric Group on n Letters|symmetric group on $n$ letters]]. +Let $\pi$ be a product of [[Definition:Disjoint Permutations|disjoint permutations]] of [[Definition:Order of Group Element|orders]] $k_1, k_2, \ldots, k_r$. +Then: +:$\order \pi = \lcm \set {k_1, k_2, \ldots, k_r}$ +where: +:$\order \pi$ denotes the [[Definition:Order of Group Element|order]] of $\pi$ in $S_n$ +:$\lcm$ denotes [[Definition:Lowest Common Multiple of Integers|lowest common multiple]]. +\end{theorem} + +\begin{proof} +Suppose $\pi$ is a [[Definition:Cyclic Permutation|cycle]]. +Then from [[Order of Cycle is Length of Cycle]], $\order \pi$ is its [[Definition:Length of Cyclic Permutation|length]]. +As the [[Definition:Lowest Common Multiple of Integers|LCM]] of $n \in \Z$ is $n$ itself, the result follows. +Let $\pi = \rho_1 \rho_2 \cdots \rho_r$ where: +: each $\rho_s$ is of [[Definition:Order of Group Element|order]] $k_s$ +: $\rho_1$ to $\rho_r$ are mutually [[Definition:Disjoint Permutations|disjoint permutations]]. +Let $t = \lcm \set {k_1, k_2, \ldots, k_r}$. +From [[Disjoint Permutations Commute]]: +:$\pi^t = \rho_1^t \rho_2^t \cdots \rho_r^t$ +We have that: +:$\forall s: 1 \le s \le r: \exists m \in \Z: t = m s$ +Hence: +:$\forall s: \rho_s^t = e$ +Thus $\pi^t = e$ and certainly $\order \pi \divides t$. +Let $\pi^u = e$ for some $u \in \Z_{>0}$. +We have that $\rho_1, \rho_2, \cdots, \rho_r$ are mutually [[Definition:Disjoint Permutations|disjoint permutations]]. +Thus $\rho_1^u, \rho_2^u, \cdots, \rho_r^u$ are also mutually [[Definition:Disjoint Permutations|disjoint permutations]]. +Then we have that each $\rho_s^u = e$. +Hence $k_s \divides u$. +Thus, if $\pi^u = e$, then: +:$t \divides u$ +Thus, by choosing $u = \order \pi$: +:$\order \pi = t = \lcm \set {k_1, k_2, \ldots, k_r}$ +{{qed}} +\end{proof}<|endoftext|> +\section{Group Action defines Permutation Representation} +Tags: Group Actions, Group Homomorphisms + +\begin{theorem} +Let $\map \Gamma X$ be the set of [[Definition:Permutation|permutations]] on a [[Definition:Set|set]] $X$. +Let $G$ be a [[Definition:Group|group]]. +Let $\phi: G \times X \to X$ be a [[Definition:Group Action|group action]]. +For $g \in G$, let $\phi_g: X \to X$ be the [[Definition:Mapping|mapping]] defined as: +:$\map {\phi_g} x = \map \phi {g, x}$ +Let $\tilde \phi: G \to \map \Gamma X$ be the [[Definition:Permutation Representation/Group Action|mapping associated to]] $\phi$, defined by: +:$\map {\tilde \phi} g := \phi_g$ +Then $\tilde \phi$ is a [[Definition:Group Homomorphism|group homomorphism]]. +\end{theorem} + +\begin{proof} +Note that, by [[Group Action determines Bijection|Group Action Determines Bijection]], $\phi_g \in \map \Gamma X$ for $g \in G$. +Let $g, h \in G$. +From the [[Definition:Group Action|definition of group action]]: +:$\forall \tuple {g, x} \in G \times X: \map \phi {g, x} \in X = g \wedge x \in X$ +First we show that for all $x \in X$: +:$\map {\phi_g \circ \phi_h} x = \map {\phi_{g h} } x$ +Thus: +{{begin-eqn}} +{{eqn | l = \map {\phi_g \circ \phi_h} x + | r = g \wedge \paren {h \wedge x} + | c = Definition of $\phi_g$, $\phi_h$ +}} +{{eqn | r = \paren {g h} \wedge x + | c = {{Defof|Group Action}} +}} +{{eqn | r = \map {\phi_{g h} } x + | c = Definition of $\phi_{g h}$ +}} +{{end-eqn}} +Also, we have: +: $e \wedge x = x \implies \map {\phi_e} x = x$ +where $e$ is the [[Definition:Identity Element|identity]] of $G$. +Therefore, we have shown that $\tilde \phi: G \to \map \Gamma X: g \mapsto \phi_g$ is a [[Definition:Group Homomorphism|group homomorphism]]. +{{qed}} +\end{proof}<|endoftext|> +\section{Group Action determines Bijection} +Tags: Group Actions + +\begin{theorem} +Let $*$ be a [[Definition:Group Action|group action]] of $G$ on $X$. +Then each $g \in G$ determines a [[Definition:Bijection|bijection]] $\phi_g: X \to X$ given by: +:$\map {\phi_g} x = g * x$ +Its [[Definition:Inverse Mapping|inverse]] is: +:$\phi_{g^{-1} }: X \to X$. +These [[Definition:Bijection|bijection]] are sometimes called '''transformations''' of $X$. +\end{theorem} + +\begin{proof} +=== Proof of Injectivity === +Let $x, y \in X$ +Then: +{{begin-eqn}} +{{eqn | l = \map {\phi_g} x + | r = \map {\phi_g} y + | c = +}} +{{eqn | ll= \leadsto + | l = g * x + | r = g * y + | c = +}} +{{eqn | ll= \leadsto + | l = g^{-1} * \paren {g * x} + | r = g^{-1} * \paren {g * y} + | c = +}} +{{eqn | ll= \leadsto + | l = \paren {g^{-1} g} * x + | r = \paren {g^{-1} g} * y + | c = +}} +{{eqn | ll= \leadsto + | l = e * x + | r = e * y + | c = +}} +{{eqn | ll= \leadsto + | l = x + | r = y + | c = +}} +{{end-eqn}} +Thus $\phi_g$ is an [[Definition:Injection|injection]]. +{{qed|lemma}} +=== Proof of Surjectivity === +Let $x \in X$. +Then: +{{begin-eqn}} +{{eqn | l = x + | r = e * x + | c = +}} +{{eqn | r = \paren {g g^{-1} } * x + | c = +}} +{{eqn | r = g * \paren {g^{-1} * x} + | c = +}} +{{eqn | r = \map {\phi_g} y + | c = where $y = g^{-1} * x \in X$ +}} +{{end-eqn}} +Thus a group action is a [[Definition:Surjection|surjection]]. +{{qed|lemma}} +So a group action is an [[Definition:Injection|injection]] and a [[Definition:Surjection|surjection]] and therefore a [[Definition:Bijection|bijection]]. +{{qed}} +\end{proof}<|endoftext|> +\section{Group Action Induces Equivalence Relation} +Tags: Group Actions, Equivalence Relations + +\begin{theorem} +Let $G$ be a [[Definition:Group|group]] whose [[Definition:Identity Element|identity]] is $e$. +Let $X$ be a [[Definition:Set|set]]. +Let $*: G \times S \to S$ be a [[Definition:Group Action|group action]]. +Let $\mathcal R_G$ be the [[Definition:Equivalence Relation Induced by Group Action|relation induced]] by $G$, that is: +:$x \mathrel {\mathcal R_G} y \iff y \in \Orb x$ +where: +:$\Orb x$ denotes the [[Definition:Orbit (Group Theory)|orbit]] of $x \in X$. +Then: +:$\mathcal R_G$ is an [[Definition:Equivalence Relation|equivalence relation]]. +:The [[Definition:Equivalence Class|equivalence class]] of an element is its [[Definition:Orbit (Group Theory)|orbit]]. +\end{theorem} + +\begin{proof} +Let $x \mathrel {\mathcal R_G} y \iff y \in \Orb x$. +Checking in turn each of the critera for [[Definition:Equivalence Relation|equivalence]]: +=== Reflexivity === +$x = e * x \implies x \in \Orb x$ from the definition of [[Definition:Group Action|group action]]. +Thus $\mathcal R_G$ is [[Definition:Reflexive Relation|reflexive]]. +{{qed|lemma}} +=== Symmetry === +{{begin-eqn}} +{{eqn | l = y + | o = \in + | r = \Orb x + | c = +}} +{{eqn | ll= \leadsto + | l = \exists g \in G: y + | r = g * x + | c = +}} +{{eqn | ll= \leadsto + | l = g^{-1} * \paren {g * x} + | r = g^{-1} * y + | c = +}} +{{eqn | ll= \leadsto + | l = x + | r = g^{-1} * y + | c = +}} +{{eqn | ll= \leadsto + | l = \exists g^{-1} \in G: x + | r = g^{-1} * y + | c = +}} +{{eqn | ll= \leadsto + | l = x + | o = \in + | r = \Orb y + | c = +}} +{{end-eqn}} +Thus $\mathcal R_G$ is [[Definition:Symmetric Relation|symmetric]]. +{{qed|lemma}} +=== Transitivity === +{{begin-eqn}} +{{eqn | l = y + | o = \in + | r = \Orb x, z \in \Orb y + | c = +}} +{{eqn | ll= \leadsto + | l = \exists g_1 \in G: y + | r = g_1 * x, \exists g_2 \in G: z = g_2 * y + | c = +}} +{{eqn | ll= \leadsto + | l = z + | r = g_2 * \paren {g_1 * x} + | c = +}} +{{eqn | ll= \leadsto + | l = z + | r = \paren {g_2 g_1} * x + | c = +}} +{{eqn | ll= \leadsto + | l = z + | o = \in + | r = \Orb x + | c = +}} +{{end-eqn}} +Thus $\mathcal R_G$ is [[Definition:Transitive Relation|transitive]]. +{{qed|lemma}} +So $\mathcal R_G$ has been shown to be an [[Definition:Equivalence Relation|equivalence relation]]. +Hence the result, by definition of an [[Definition:Equivalence Class|equivalence class]]. +{{Qed}} +\end{proof}<|endoftext|> +\section{Partition Equation} +Tags: Group Actions, Named Theorems + +\begin{theorem} +Let [[Definition:Group|group]] $G$ [[Definition:Group Action|act on]] a [[Definition:Finite Set|finite set]] $X$. +Let the distinct [[Definition:Orbit (Group Theory)|orbits]] of $X$ under the [[Definition:Group Action|action]] of $G$ be: +:$\Orb {x_1}, \Orb {x_2}, \ldots, \Orb {x_s}$ +Then: +:$\card X = \card {\Orb {x_1} } + \card {\Orb {x_2} } + \cdots + \card {\Orb {x_s} }$ +\end{theorem} + +\begin{proof} +Follows trivially from the fact that the [[Group Action Induces Equivalence Relation]]. +{{explain|cite a result about equiv rels and partitions, and one about cardinality and partitions}} +{{qed}} +[[Category:Group Actions]] +[[Category:Named Theorems]] +pwvgvk2c6p89r84hq9vudcd3u47firc +\end{proof}<|endoftext|> +\section{Definition:Stabilizer} +Tags: Definitions: Group Actions + +\begin{theorem} +Let $G$ be a [[Definition:Group|group]]. +Let $X$ be a [[Definition:Set|set]]. +Let $*: G \times X \to X$ be a [[Definition:Group Action|group action]]. +For each $x \in X$, the '''stabilizer of $x$ by $G$''' is defined as: +:$\Stab x := \set {g \in G: g * x = x}$ +where $*$ denotes the [[Definition:Group Action|group action]]. +\end{theorem}<|endoftext|> +\section{Stabilizer is Subgroup} +Tags: Group Actions, Stabilizers, Stabilizer is Subgroup + +\begin{theorem} +Let $\struct {G, \circ}$ be a [[Definition:Group|group]] which [[Definition:Group Action|acts on]] a [[Definition:Set|set]] $X$. +Let $\Stab x$ be the [[Definition:Stabilizer|stabilizer of $x$ by $G$]]. +Then for each $x \in X$, $\Stab x$ is a [[Definition:Subgroup|subgroup]] of $G$. +\end{theorem} + +\begin{proof} +From the {{GroupActionAxiom|2}}: +:$e * x = x \implies e \in \Stab x$ +and so $\Stab x$ cannot be [[Definition:Empty Set|empty]]. +Let $g, h \in \Stab x$. +{{begin-eqn}} +{{eqn | l = g, h + | o = \in + | r = \Stab x + | c = +}} +{{eqn | ll= \leadsto + | l = g * x + | r = x + | c = {{Defof|Stabilizer}} of $x$ by $G$ +}} +{{eqn | lo= \land + | l = h * x + | r = x + | c = {{Defof|Stabilizer}} of $x$ by $G$ +}} +{{eqn | ll= \leadsto + | l = g * \paren {h * x} + | r = x + | c = +}} +{{eqn | ll= \leadsto + | l = \paren {g \circ h} * x + | r = x + | c = {{GroupActionAxiom|1}} +}} +{{eqn | ll= \leadsto + | l = g \circ h + | o = \in + | r = \Stab x + | c = {{Defof|Stabilizer}} of $x$ by $G$ +}} +{{end-eqn}} +Let $g \in \Stab x$. +Then: +:$x = \paren {g^{-1} \circ g} * x = g^{-1} * \paren {g * x} = g^{-1} * x$ +Hence $g^{-1} \in \Stab x$. +Thus the conditions for the [[Two-Step Subgroup Test]] are fulfilled, and $\Stab x \le G$. +{{qed}} +\end{proof}<|endoftext|> +\section{Orbit-Stabilizer Theorem} +Tags: Group Actions, Stabilizers, Named Theorems, Orbit-Stabilizer Theorem + +\begin{theorem} +Let $G$ be a [[Definition:Group|group]] which [[Definition:Group Action|acts on]] a [[Definition:Finite Set|finite set]] $X$. +Let $x \in X$. +Let $\Orb x$ denote the [[Definition:Orbit (Group Theory)|orbit]] of $x$. +Let $\Stab x$ denote the [[Definition:Stabilizer|stabilizer of $x$ by $G$]]. +Let $\index G {\Stab x}$ denote the [[Definition:Index of Subgroup|index]] of $\Stab x$ in $G$. +Then: +:$\order {\Orb x} = \index G {\Stab x} = \dfrac {\order G} {\order {\Stab x} }$ +\end{theorem} + +\begin{proof} +Let us define the [[Definition:Mapping|mapping]]: +:$\phi: G \to \Orb x$ +such that: +:$\map \phi g = g * x$ +where $*$ denotes the [[Definition:Group Action|group action]]. +It is clear that $\phi$ is [[Definition:Surjection|surjective]], because from the definition $x$ was acted on by all the [[Definition:Element|elements]] of $G$. +Next, from [[Stabilizer is Subgroup/Corollary|Stabilizer is Subgroup: Corollary]]: +:$\map \phi g = \map \phi h \iff g^{-1} h \in \Stab x$ +This means: +:$g \equiv h \pmod {\Stab x}$ +Thus there is a well-defined [[Definition:Bijection|bijection]]: +:$G \mathbin / \Stab x \to \Orb x$ +given by: +:$g \, \Stab x \mapsto g * x$ +So $\Orb x$ has the same number of elements as $G \mathbin / \Stab x$. +That is: +:$\order {\Orb x} = \index G {\Stab x}$ +The result follows. +{{qed}} +\end{proof} + +\begin{proof} +Let $x \in X$. +Let $\phi: \Orb x \to G \mathbin / \Stab x$ be a [[Definition:Mapping|mapping]] from the [[Definition:Orbit (Group Theory)|orbit]] of $x$ to the [[Definition:Left Coset Space|left coset space]] of $\Stab x$ defined as: +:$\forall g \in G: \map \phi {g * x} = g \, \Stab x$ +where $*$ is the [[Definition:Group Action|group action]]. +Note: this is ''not'' a [[Definition:Group Homomorphism|homomorphism]] because $\Orb x$ is not a [[Definition:Group|group]]. +Suppose $g * x = h * x$ for some $g, h \in G$. +Then: +:$h^{-1} g * x = h^{-1} h * x$ +and so: +:$h^{-1} g * x = x$ +Thus: +:$h^{-1} g \in \Stab x$ +so by [[Left Coset Space forms Partition]]: +:$g \, \Stab x = h \, \Stab x$ +demonstrating that $\phi$ is [[Definition:Well-Defined Mapping|well-defined]]. +Let $\map \phi {g_1 * x} = \map \phi {g_2 * x}$ for some $g_1, g_2 \in G$. +Then: +:$g_1 \, \Stab x = g_2 \, \Stab x$ +and so by [[Left Coset Space forms Partition]]: +:$g_2^{-1} g_1 \in \Stab x$ +So by definition of $\Stab x$: +:$x = g_2^{-1} g_1 * x$ +Thus: +{{begin-eqn}} +{{eqn | l = g_2 * x + | r = g_2 * \paren {g_2^{-1} g_1 * x} + | c = applying $g_2$ to both sides +}} +{{eqn | r = \paren {g_2 g_2^{-1} g_1} x + | c = {{Defof|Group Action}} +}} +{{eqn | r = g_1 * x + | c = +}} +{{end-eqn}} +thus demonstrating that $\phi$ is [[Definition:Injection|injective]]. +As the [[Definition:Left Coset|left coset]] $g \, \Stab x$ is $\map \phi {g * x}$ by definition of $\phi$, it follows that $\phi$ is a [[Definition:Surjection|surjection]]. +The result follows. +{{qed}} +\end{proof}<|endoftext|> +\section{Group Acts on Itself} +Tags: Group Actions + +\begin{theorem} +Let $\struct {G, \circ}$ be a [[Definition:Group|group]] whose [[Definition:Identity Element|identity]] is $e$. +Then $\struct {G, \circ}$ [[Definition:Group Action|acts on]] itself by the rule: +:$\forall g, h \in G: g * h = g \circ h$ +\end{theorem} + +\begin{proof} +Follows directly from the [[Definition:Group Axioms|group axioms]] and the definition of a [[Definition:Group Action|group action]]. +{{qed}} +\end{proof}<|endoftext|> +\section{Action of Group on Coset Space is Group Action} +Tags: Cosets, Group Action on Coset Space + +\begin{theorem} +Let $G$ be a [[Definition:Group|group]] whose [[Definition:Identity Element|identity]] is $e$. +Let $H$ be a [[Definition:Subgroup|subgroup]] of $G$. +Let $*: G \times G / H \to G / H$ be the [[Definition:Group Action on Coset Space|action on the (left) coset space]]: +:$\forall g \in G, \forall g' H \in G / H: g * \paren {g' H} := \paren {g g'} H$ +Then $G$ is a [[Definition:Group Action|group action]]. +\end{theorem} + +\begin{proof} +{{begin-eqn}} +{{eqn | l = a * \paren {b * g' H} + | r = a * \paren {\paren {b g'} H} + | c = Definition of $*$ +}} +{{eqn | r = \paren {a \paren {b g'} } H + | c = Definition of $*$ +}} +{{eqn | r = \paren {a b} g' H + | c = {{GroupAxiom|1}} +}} +{{end-eqn}} +demonstrating that $*$ satisfies {{GroupActionAxiom|2}}. +Then: +{{begin-eqn}} +{{eqn | l = e * g' H + | r = \paren {e g'} H + | c = Definition of $*$ +}} +{{eqn | r = g'H + | c = {{GroupAxiom|2}} +}} +{{end-eqn}} +demonstrating that $*$ satisfies {{GroupActionAxiom|1}}. +{{qed}} +\end{proof}<|endoftext|> +\section{Group Action on Sets with k Elements} +Tags: Examples of Group Actions, Group Action on Sets with k Elements + +\begin{theorem} +Let $\struct {G, \circ}$ be a [[Definition:Finite Group|finite group]] whose [[Definition:Identity Element|identity]] is $e$. +Let $\Bbb S = \set {S \subseteq G: \card S = k}$, that is, the [[Definition:Set|set]] of all of [[Definition:Subset|subsets]] of $G$ which have exactly $k$ [[Definition:Element|elements]]. +Let $G$ act on $\Bbb S$ by the rule: +:$\forall S \in \Bbb S: g * S = g S = \set {x \in G: x = g s: s \in S}$ +This is a [[Definition:Group Action|group action]], and: +: $\forall S \in \Bbb S: \order {\Stab S} \divides \card S$ +where $\Stab S$ denotes the [[Definition:Stabilizer|stabilizer of $S$ by $G$]]. +\end{theorem} + +\begin{proof} +First it is necessary to prove that this is a [[Definition:Group Action|group action]]. +The [[Definition:Group Action|action]] is the same as the one defined in [[Group Action on Coset Space]], but this time we are limiting ourselves to the [[Definition:Subset|subsets]] of $G$ which have the same number of [[Definition:Element|elements]]. +From the result in [[Group Action on Coset Space]], we only have to prove that: +:$\card {g S} = \card S$ +This follows directly from [[Order of Subset Product with Singleton]]. +It remains to be shown that: +:$\order {\Stab S} \divides \card S$ +By definition of [[Definition:Stabilizer|stabilizer]]: +:$\Stab S = \set {g \in G: g S = S}$ +It follows that: +:$\forall s \in S: g s \in S$ +Then: +:$\Stab S s \subseteq S$ +Thus: +:$\card {\Stab S s} \le \card S$ +We have from [[Stabilizer is Subgroup/Corollary 1|corollary $1$ to Stabilizer is Subgroup]] that the [[Definition:Stabilizer|stabilizer]] contains the [[Definition:Identity Element|identity]]: +:$e \in \Stab S \le G$ +It therefore follows that: +:$s \in S \implies s \in \Stab S s$ +Thus: +:$\displaystyle S = \bigcup_{t \mathop \in S} \Stab S t$ +By [[Congruence Class Modulo Subgroup is Coset]], distinct [[Definition:Right Coset|right cosets]] are [[Definition:Disjoint Sets|disjoint]]. +Thus $S$ consists of a [[Definition:Set Union|union]] of [[Definition:Disjoint Sets|disjoint]] [[Definition:Right Coset|right cosets]] of $\Stab S$. +Thus $\displaystyle \bigcup_{t \mathop \in S} \Stab S t$ is a [[Definition:Set Partition|partition]] of $S$. +Therefore: +:$\forall s \in S: \card {\Stab S s} \divides \card S$ +As we have: +:$\forall s \in S: \card {\Stab S s} = \order {\Stab S}$ +the result follows: +:$\card {\Stab S} \divides \card S$ +{{qed}} +\end{proof}<|endoftext|> +\section{Group Action on Subgroup of Symmetric Group} +Tags: Group Actions, Symmetric Groups + +\begin{theorem} +Let $S_n$ be the [[Definition:Symmetric Group on n Letters|symmetric group of $n$ elements]]. +Let $H$ be a [[Definition:Subgroup|subgroup]] of $S_n$. +Let $X$ be any [[Definition:Set|set]] with $n$ [[Definition:Element|elements]]. +Then $H$ [[Definition:Group Action|acts on]] $X$ as a [[Group Action determines Bijection|group of transformations]] on $X$. +\end{theorem} + +\begin{proof} +The [[Definition:Identity Mapping|identity permutation]] takes each element of $X$ to itself, thus fulfilling {{GroupActionAxiom|1}}. +The [[Definition:Group Operation|group operation]] in $S_n$ ensures fulfilment of {{GroupActionAxiom|2}}. +{{qed}} +\end{proof}<|endoftext|> +\section{Kernel of Group Action is Normal Subgroup} +Tags: Group Actions + +\begin{theorem} +Let $G$ be a [[Definition:Group|group]] whose [[Definition:Identity Element|identity]] is $e$. +Let $X$ be a [[Definition:Set|set]]. +Let $\phi: G \times X \to X$ be a [[Definition:Group Action|group action]]. +Let $G_0$ denote the [[Definition:Kernel of Group Action|kernel of $\phi$]]. +Then $G_0$ is a [[Definition:Normal Subgroup|normal subgroup]] of $G$. +\end{theorem} + +\begin{proof} +Let $h \in G_0$. +{{begin-eqn}} +{{eqn | l = h + | o = \in + | r = G_0 + | c = +}} +{{eqn | ll= \leadstoandfrom + | l = h + | o = \in + | r = \set {g \in G: \forall x \in X: g \cdot x = x} + | c = {{Defof|Kernel of Group Action}} +}} +{{eqn | ll= \leadstoandfrom + | lo= \forall x \in X: + | l = h + | o = \in + | r = \set {g \in G: g \cdot x = x} + | c = +}} +{{eqn | ll= \leadstoandfrom + | lo= \forall x \in X: + | l = h + | o = \in + | r = \Stab x + | c = +}} +{{eqn | ll= \leadstoandfrom + | l = h + | o = \in + | r = \bigcap_{x \mathop \in X} \Stab x + | c = +}} +{{eqn | ll= \leadstoandfrom + | l = G_0 + | r = \bigcap_{x \mathop \in X} \Stab x + | c = +}} +{{end-eqn}} +From [[Stabilizer is Subgroup]]: +:$\Stab x \le G$ +Thus $G_0$ is the [[Definition:Set Intersection|intersection]] of [[Definition:Subgroup|subgroups]]. +By [[Intersection of Subgroups is Subgroup/General Result|Intersection of Subgroups is Subgroup]]: +:$G_0 \le G$ +To prove [[Definition:Normal Subgroup|normality]] it is sufficient to show: +:$\forall g \in G: g G_0 g^{-1} = G_0$ +Let $h \in G_0, g \in G$ be arbitrary. +Then: +{{begin-eqn}} +{{eqn | l = \paren {g h g^{-1} } \cdot x + | r = g \cdot \paren {h \cdot \paren { g^{-1} \cdot x } } + | c = [[Definition:Group Action|Group Action]] [[Definition:Associative Operation|Associates]] with [[Definition:Group Operation|Group Operation]]: {{GroupActionAxiom|1}} +}} +{{eqn | r = g \cdot \paren {g^{-1} \cdot x} + | c = because $h \in \Stab { g^{-1} \cdot x}$ +}} +{{eqn | r = \paren {g \cdot g^{-1} } \cdot x + | c = [[Definition:Group Action|Group Action]] [[Definition:Associative Operation|Associates]] with [[Definition:Group Operation|Group Operation]]: {{GroupActionAxiom|1}} +}} +{{eqn | r = x + | c = $e \cdot x = x$: {{GroupActionAxiom|2}} +}} +{{end-eqn}} +Therefore: +:$g h g^{-1} \in G_0$ +so: +:$g G_0 g^{-1} \subseteq G_0$ +Conversely suppose that $h \in G_0$. +Then by the above: +:$h' = g^{-1} h g \in G_0$ +Therefore: +:$h = g h' g^{-1} \in g G_0 g^{-1}$ +and so: +:$G_0 \subseteq g G_0 g^{-1}$ +This concludes the proof. +{{qed}} +\end{proof}<|endoftext|> +\section{Quotient of Transformation Group acts Effectively} +Tags: Group Actions, Quotient Groups + +\begin{theorem} +Let $G$ be a [[Definition:Transformation Group|transformation group]] (which may or may not be [[Definition:Effective Transformation Group|effective]]) acting on $X$. +Then the [[Definition:Quotient Group|quotient group]] $G / G_0$, where $G_0$ is the [[Definition:Kernel of Group Action|kernel]], ''does'' act [[Definition:Effective Transformation Group|effectively]] on $X$. +\end{theorem}<|endoftext|> +\section{Condition for Group to Act Effectively on Left Coset Space} +Tags: Group Action on Coset Space, Cosets + +\begin{theorem} +Let $G$ be a [[Definition:Group|group]] whose [[Definition:Identity Element|identity]] is $e$. +Let $H$ be a [[Definition:Subgroup|subgroup]] of $G$. +Then $G$ [[Definition:Effective Transformation Group|acts effectively]] on the [[Definition:Left Coset Space|left coset space]] $G / H$ {{iff}}: +:$\ds \bigcap_{a \mathop \in G} H^a = \set e$ +where $H^a$ denotes the [[Definition:Conjugate of Group Subset|conjugate]] of $H$ by $a$. +\end{theorem} + +\begin{proof} +$G$ [[Definition:Effective Transformation Group|acts effectively]] on the [[Definition:Left Coset Space|left coset space]] $G / H$ {{iff}} $a H \mapsto g a H$ is [[Definition:Faithful Group Action|faithful]], {{iff}}: +{{begin-eqn}} +{{eqn | lo= \forall g \in G: \forall a H \in G / H: + | l = g a H = a H + | o = \implies + | r = g = e + | c = {{Defof|Faithful Group Action}} +}} +{{eqn | ll= \leadstoandfrom + | lo= \forall g \in G: \forall a \in G: + | l = a^{-1} g a \in H + | o = \implies + | r = g = e + | c = [[Left Cosets are Equal iff Product with Inverse in Subgroup]] +}} +{{eqn | ll= \leadstoandfrom + | lo= \forall g \in G: \forall a \in G: + | l = g \in a H a^{-1} + | o = \implies + | r = g = e +}} +{{eqn | ll= \leadstoandfrom + | lo= \forall g \in G: \forall a \in G: + | l = g \in H^a + | o = \implies + | r = g = e + | c = {{Defof|Conjugate of Group Subset}} +}} +{{eqn | ll= \leadstoandfrom + | l = \bigcap_{a \mathop \in G} H^a + | r = \set e + | c = {{Defof|Intersection of Family}} +}} +{{end-eqn}} +{{qed}} +\end{proof}<|endoftext|> +\section{Set of Permutations is Largest Effective Transformation Group} +Tags: Group Actions + +\begin{theorem} +The set of [[Definition:Permutation|permutations]] of a [[Definition:Set|set]] $X$ forms the largest [[Definition:Effective Transformation Group|effective transformation group]] of $X$. +\end{theorem}<|endoftext|> +\section{Conjugacy Action is Group Action} +Tags: Conjugacy Action + +\begin{theorem} +Let $\left({G, \circ}\right)$ be a [[Definition:Group|group]] whose [[Definition:Identity Element|identity]] is $e$. +\end{theorem}<|endoftext|> +\section{Conjugacy Action on Identity} +Tags: Conjugacy Action + +\begin{theorem} +Let $G$ be a [[Definition:Group|group]] whose [[Definition:Identity Element|identity]] is $e$. +For the [[Definition:Conjugacy Action|conjugacy action]]: +:$\order {\Orb e} = 1$ +and thus: +:$\Stab e = G$ +\end{theorem} + +\begin{proof} +{{begin-eqn}} +{{eqn | l = g * e + | r = g e g^{-1} + | c = +}} +{{eqn | r = g g^{-1} + | c = +}} +{{eqn | r = e + | c = +}} +{{end-eqn}} +So the only [[Definition:Conjugate of Group Element|conjugate]] of $e$ is $e$ itself. +Thus: +:$\Orb e = \set e$ +and so: +:$\order {\Orb e} = 1$ +From the [[Orbit-Stabilizer Theorem]], it follows immediately that: +:$\Stab e = G$ +{{qed}} +\end{proof}<|endoftext|> +\section{Cauchy's Lemma (Group Theory)} +Tags: Group Theory, Cauchy's Lemma (Group Theory) + +\begin{theorem} +Let $\struct {G, \circ}$ be a [[Definition:Group|group]] of [[Definition:Order of Structure|finite order]] whose [[Definition:Identity Element|identity]] is $e$. +Let $p$ be a [[Definition:Prime Number|prime number]] which [[Definition:Divisor of Integer|divides]] the [[Definition:Order of Structure|order]] of $G$. +Then $\struct {G, \circ}$ has an [[Definition:Element|element]] of [[Definition:Order of Group Element|order]] $p$. +\end{theorem} + +\begin{proof} +Let $\order G = n$ such that $p \divides n$. +Let: +:$X = \set {\tuple {a_1, a_2, \ldots, a_p} \in G^p: a_1 a_2 \cdots a_p = e}$ +where $G^p$ is the [[Definition:Cartesian Space|cartesian product]] $\underbrace {G \times G \times \cdots \times G}_p$. +The first $p - 1$ [[Definition:Coordinate of Ordered Tuple|coordinates]] of an [[Definition:Element|element]] of $X$ can be chosen arbitrarily. +The last [[Definition:Coordinate of Ordered Tuple|coordinate]] is determined by the fact that: +:$a_1 a_2 \cdots a_{p - 1} = a_p^{-1}$ +So from the [[Product Rule for Counting]], it follows that: +:$\card X = n^{p - 1}$ +Let $C_p$ be a [[Definition:Cyclic Group|cyclic group]] of [[Definition:Order of Structure|order]] $p$ [[Definition:Generator of Cyclic Group|generated]] by the [[Definition:Element|element]] $c$. +Let $C_p$ [[Definition:Group Action|act on]] the set $X$ by the rule: +:$c * \tuple {a_1, a_2, \ldots, a_p} = \tuple {a_2, a_3, \ldots, a_p, a_1}$ +By the [[Orbit-Stabilizer Theorem]], the number of [[Definition:Element|elements]] in any [[Definition:Orbit (Group Theory)|orbit]] is a [[Definition:Divisor of Integer|divisor]] of the order of $C_p$, which is $p$. +As $p$ is [[Definition:Prime Number|prime]], an [[Definition:Orbit (Group Theory)|orbit]] has either $p$ [[Definition:Element|elements]] or $1$ [[Definition:Element|element]] by definition. +Let $r$ be the number of [[Definition:Orbit (Group Theory)|orbits]] with one [[Definition:Element|element]]. +Let $s$ be the number of [[Definition:Orbit (Group Theory)|orbits]] with $p$ [[Definition:Element|elements]]. +Then by the [[Partition Equation]]: +:$r + s p = n^{p - 1} = \card X$ +[[Definition:By Hypothesis|By hypothesis]], $p \divides n$, so: +:$r + s p = n^{p - 1} \implies p \divides r$ +We know that $r \ne 0$ because, for example, the [[Definition:Orbit (Group Theory)|orbit]] of $\tuple {e, e, \ldots, e} \in X$ has only one [[Definition:Element|element]]. +So there must be at least $p$ [[Definition:Orbit (Group Theory)|orbits]] with only one [[Definition:Element|element]]. +Each such [[Definition:Element|element]] has the form $\tuple {a, a, \ldots, a} \in X$ so $a^p = e$. +So $G$ contains at least $p$ [[Definition:Element|elements]] $x$ satisfying $x^p = e$. +So $G$ contains an [[Definition:Element|element]] $a \ne e$ such that $a^p = e$. +That is, $a$ must have [[Definition:Order of Group Element|order]] $p$. +{{qed}} +\end{proof} + +\begin{proof} +By the [[First Sylow Theorem/Corollary|corollary to the First Sylow Theorem]], $G$ has [[Definition:Subgroup|subgroups]] of [[Definition:Order of Group|order]] $p^r$ for all $r$ such that $p^r \divides \order G$. +Thus $G$ has at least one [[Definition:Subgroup|subgroup]] $H$ of [[Definition:Order of Group|order]] $p$. +As a [[Prime Group is Cyclic]], $H$ is a [[Definition:Cyclic Group|cyclic group]]. +Thus by definition $H$ has an [[Definition:Element|element]] of [[Definition:Order of Group Element|order]] $p$. +Hence the result. +{{qed}} +\end{proof}<|endoftext|> +\section{Transposition is Self-Inverse} +Tags: Symmetric Groups + +\begin{theorem} +All [[Definition:Transposition|transpositions]] are [[Definition:Self-Inverse Element|self-inverse]]. +\end{theorem} + +\begin{proof} +Let $\pi = \begin{bmatrix} k_1 & k_2 \end{bmatrix}$ be a [[Definition:Transposition|transposition]]. +Writing $\pi \pi$ in [[Definition:Permutation on n Letters/Cycle Notation|cycle notation]] gives: +:$\begin{bmatrix} k_1 & k_2 \end{bmatrix} \begin{bmatrix} k_1 & k_2 \end{bmatrix}$ +from which we see that $k_1 \to k_2 \to k_1$ and $k_2 \to k_1 \to k_2$. +The result follows from the definition of [[Definition:Self-Inverse Element|self-inverse]]. +{{qed}} +[[Category:Symmetric Groups]] +lehv6tztngn5ci5vftba2lh34nrl8ch +\end{proof}<|endoftext|> +\section{Conjugates of Transpositions} +Tags: Symmetric Groups + +\begin{theorem} +Let $k_1, k_2, k_3 \in \left\{{1, 2, \ldots, n}\right\}$. +Then: +: $(1): \quad \begin{bmatrix} k_1 & k_2 \end{bmatrix} = \begin{bmatrix} k_3 & k_2 \end{bmatrix} \begin{bmatrix} k_1 & k_3 \end{bmatrix} \begin{bmatrix} k_3 & k_2 \end{bmatrix}$ +: $(2): \quad \begin{bmatrix} k_1 & k_2 \end{bmatrix} = \begin{bmatrix} k_3 & k_1 \end{bmatrix} \begin{bmatrix} k_3 & k_2 \end{bmatrix} \begin{bmatrix} k_3 & k_1 \end{bmatrix}$ +\end{theorem} + +\begin{proof} +$(1):$ Calculating the product of $\begin{bmatrix} k_3 & k_2 \end{bmatrix} \begin{bmatrix} k_1 & k_3 \end{bmatrix} \begin{bmatrix} k_3 & k_2 \end{bmatrix}$: +:: $k_1 \to k_3 \to k_2$ +:: $k_2 \to k_3 \to k_1$ +:: $k_3 \to k_2 \to k_3$ +hence the result. +{{qed|lemma}} +$(2):$ Calculating the product of $\begin{bmatrix} k_3 & k_1 \end{bmatrix} \begin{bmatrix} k_3 & k_2 \end{bmatrix} \begin{bmatrix} k_3 & k_1 \end{bmatrix}$: +:: $k_1 \to k_3 \to k_2$ +:: $k_2 \to k_3 \to k_1$ +:: $k_3 \to k_1 \to k_3$ +hence the result. +{{qed}} +[[Category:Symmetric Groups]] +psvlecwx3uss76gc4u3nnod6mn6y4ch +\end{proof}<|endoftext|> +\section{K-Cycle can be Factored into Transpositions} +Tags: Cyclic Permutations + +\begin{theorem} +Every [[Definition:Cyclic Permutation|$k$-cycle]] can be factorised into the product of $k - 1$ [[Definition:Transposition|transpositions]]. +This factorisation is not unique. +\end{theorem} + +\begin{proof} +The cycle $\begin{pmatrix} 1 & 2 & \ldots & k \end{pmatrix}$ has the factorisation: +:$\begin{pmatrix} 1 & 2 & \ldots & k \end{pmatrix} = \begin{pmatrix} 1 & k \end{pmatrix} \ldots \begin{pmatrix} 1 & 3 \end{pmatrix} \begin{pmatrix} 1 & 2 \end{pmatrix}$ +Therefore, the general [[Definition:Cyclic Permutation|$k$-cycle]] $\begin{pmatrix} i_1 & i_2 & \ldots & i_k \end{pmatrix}$ has the factorisation: +:$\begin{pmatrix} i_1 & i_2 & \ldots & i_k \end{pmatrix} = \begin{pmatrix} i_1 & i_k \end{pmatrix} \ldots \begin{pmatrix} i_1 & i_3 \end{pmatrix} \begin{pmatrix} i_1 & i_2 \end{pmatrix}$ +The cycle $\begin{pmatrix} 1 & 2 & \ldots & k \end{pmatrix}$ also has the factorisation: +:$\begin{pmatrix} 1 & 2 & \ldots & k \end{pmatrix} = \begin{pmatrix} 1 & 2 \end{pmatrix} \begin{pmatrix} 2 & 3 \end{pmatrix} \ldots \begin{pmatrix} k - 1 & k \end{pmatrix}$ +Therefore, the general [[Definition:Cyclic Permutation|$k$-cycle]] $\begin{pmatrix} i_1 & i_2 & \ldots & i_k \end{pmatrix}$ also has the factorisation: +:$\begin{pmatrix} i_1 & i_2 & \ldots & i_k \end{pmatrix} = \begin{pmatrix} i_1 & i_2 \end{pmatrix} \begin{pmatrix} i_2 & i_3 \end{pmatrix} \ldots \begin{pmatrix} i_{k - 1} & i_k \end{pmatrix}$ +{{qed}} +\end{proof}<|endoftext|> +\section{Sign of Permutation is Plus or Minus Unity} +Tags: Symmetric Groups + +\begin{theorem} +Let $n \in \N$ be a [[Definition:Natural Number|natural number]]. +Let $\N_n$ denote the [[Definition:Set|set]] of [[Definition:Natural Numbers|natural numbers]] $\set {1, 2, \ldots, n}$. +Let $S_n$ denote the [[Definition:Symmetric Group on n Letters|symmetric group on $n$ letters]]. +Let $\sequence {x_k}_{k \mathop \in \N_n}$ be a [[Definition:Finite Sequence|finite]] [[Definition:Real Sequence|sequence in $\R$]]. +Let $\pi \in S_n$. +Let $\map {\Delta_n} {x_1, x_2, \ldots, x_n}$ be the [[Definition:Product of Differences|product of differences]] of $\tuple {x_1, x_2, \ldots, x_n}$. +Let $\map \sgn \pi$ be the [[Definition:Sign of Permutation on n Letters|sign of $\pi$]]. +Let $\pi \cdot \map {\Delta_n} {x_1, x_2, \ldots, x_n}$ be defined as: +:$\pi \cdot \map {\Delta_n} {x_1, x_2, \ldots, x_n} := \map {\Delta_n} {x_{\map \pi 1}, x_{\map \pi 2}, \ldots, x_{\map \pi n} }$ +Then either: +:$\pi \cdot \Delta_n = \Delta_n$ +or: +:$\pi \cdot \Delta_n = -\Delta_n$ +That is: +:$\map \sgn \pi = \begin{cases} +1 & :\pi \cdot \Delta_n = \Delta_n \\ +-1 & : \pi \cdot \Delta_n = -\Delta_n +\end{cases}$ +Thus: +:$\pi \cdot \Delta_n = \map \sgn \pi \Delta_n$ +\end{theorem} + +\begin{proof} +If $\exists i, j \in \N_n$ such that $x_i = x_j$, then $\map {\Delta_n} {x_1, x_2, \ldots, x_n} = 0$ and the result follows trivially. +So, suppose all the elements $x_k$ are [[Definition:Distinct|distinct]]. +Let us use $\Delta_n$ to denote $\map {\Delta_n} {x_1, x_2, \ldots, x_n}$. +Let $1 \le a < b \le n$. +Then $x_a - x_b$ is a [[Definition:Divisor of Integer|divisor]] of $\Delta_n$. +Then $x_{\map \pi a} - x_{\map \pi b}$ is a factor of $\pi \cdot \Delta_n$. +There are two possibilities for the ordering of $\map \pi a$ and $\map \pi b$: +Either $\map \pi a < \map \pi b$ or $\map \pi a > \map \pi b$. +If the former, then $x_{\map \pi a} - x_{\map \pi b}$ is a factor of $\Delta_n$. +If the latter, then $-\paren {x_{\map \pi a} - x_{\map \pi b} }$ is a factor of $\Delta_n$. +The same applies to all factors of $\Delta_n$. +Thus: +{{begin-eqn}} +{{eqn | l = \pi \cdot \Delta_n + | r = \pi \cdot \prod_{1 \mathop \le i \mathop < j \mathop \le n} \paren {x_i - x_j} + | c = +}} +{{eqn | r = \pm \prod_{1 \mathop \le i \mathop < j \mathop \le n} \paren {x_i - x_j} + | c = +}} +{{eqn | r = \pm \Delta_n + | c = +}} +{{end-eqn}} +{{Qed}} +\end{proof}<|endoftext|> +\section{Parity Function is Homomorphism} +Tags: Symmetric Groups + +\begin{theorem} +Let $S_n$ denote the [[Definition:Symmetric Group on n Letters|symmetric group on $n$ letters]]. +Let $\pi \in S_n$. +Let $\map \sgn \pi$ be the [[Definition:Sign of Permutation on n Letters|sign of $\pi$]]. +Let the [[Definition:Parity of Permutation|parity function]] of $\pi$ be defined as: +:Parity of $\pi = \begin{cases} +\mathrm {Even} & : \map \sgn \pi = 1 \\ +\mathrm {Odd} & : \map \sgn \pi = -1 +\end{cases}$ +The [[Definition:Mapping|mapping]] $\sgn: S_n \to C_2$, where $C_2$ is the [[Definition:Cyclic Group|cyclic group]] of [[Definition:Order of Structure|order 2]], is a [[Definition:Group Homomorphism|homomorphism]]. +\end{theorem} + +\begin{proof} +We need to show that: +:$\forall \pi, \rho \in S_n: \map \sgn \pi \, \map \sgn \rho = \map \sgn {\pi \rho}$ +Let $\Delta_n$ be an arbitrary [[Definition:Product of Differences|product of differences]]. +{{begin-eqn}} +{{eqn | l = \map \sgn {\pi \rho} \Delta_n + | r = \pi \rho \cdot \Delta_n + | c = {{Defof|Sign of Permutation}} +}} +{{eqn | r = \pi \cdot \paren {\rho \cdot \Delta_n} + | c = [[Permutation on Polynomial is Group Action]] +}} +{{eqn | r = \pi \cdot \paren {\map \sgn \rho \cdot \Delta_n} + | c = {{Defof|Sign of Permutation}} +}} +{{eqn | r = \pi \, \map \sgn \rho \cdot \Delta_n + | c = [[Permutation on Polynomial is Group Action]] +}} +{{eqn | r = \map \sgn \pi \, \map \sgn \rho \Delta_n + | c = {{Defof|Sign of Permutation}} +}} +{{end-eqn}} +As $\struct {\set {1, -1}, \times}$ is the [[Definition:Parity Group|parity group]], the result follows immediately. +{{Qed}} +\end{proof}<|endoftext|> +\section{Permutation on Polynomial is Group Action} +Tags: Group Actions, Permutation Theory + +\begin{theorem} +Let $n \in \Z: n > 0$. +Let $F_n$ be the set of all [[Definition:Polynomial|polynomials]] in $n$ variables $x_1, x_2, \ldots, x_n$: +:$F = \set {\map f {x_1, x_2, \ldots, x_n}: f \text{ is a polynomial in $n$ variables} }$ +Let $S_n$ denote the [[Definition:Symmetric Group on n Letters|symmetric group on $n$ letters]]. +Let $*: S_n \times F \to F$ be the [[Definition:Mapping|mapping]] defined as: +:$\forall \pi \in S_n, f \in F: \pi * \map f {x_1, x_2, \ldots, x_n} = \map f {x_{\map \pi 1}, x_{\map \pi 2}, \ldots, x_{\map \pi n} }$ +Then $*$ is a [[Definition:Group Action|group action]]. +\end{theorem} + +\begin{proof} +Let $\pi, \rho \in S_n$. +Let $\pi * f$ be the [[Definition:Permutation on Polynomial|permutation on the polynomial]] $f$ by $\pi$. +Let $e \in S_n$ be the [[Definition:Identity Element|identity]] of $S_n$. +From [[Symmetric Group is Group]]: +:$e * f = f$ +thus fulfilling {{GroupActionAxiom|1}}. +Then we have that: +{{begin-eqn}} +{{eqn | l = \paren {\pi \circ \rho} * f + | r = \map \pi {\rho * f} + | c = +}} +{{eqn | r = \pi * \paren {\rho * f} + | c = +}} +{{end-eqn}} +thus fulfilling {{GroupActionAxiom|2}} +{{qed}} +\end{proof}<|endoftext|> +\section{Parity of Inverse of Permutation} +Tags: Symmetric Groups + +\begin{theorem} +Let $S_n$ denote the [[Definition:Symmetric Group on n Letters|symmetric group on $n$ letters]]. +Then: +:$\forall \pi \in S_n: \map \sgn \pi = \map \sgn {\pi^{-1} }$ +\end{theorem} + +\begin{proof} +From [[Parity Function is Homomorphism]]: +:$\map \sgn {I_{S_n} } = 1$ +Thus: +:$\pi \pi^{-1} = I_{S_n} \implies \map \sgn \pi \, \map \sgn {\pi^{-1} } = 1$ +The result follows immediately. +{{qed}} +\end{proof}<|endoftext|> +\section{Parity of Conjugate of Permutation} +Tags: Symmetric Groups + +\begin{theorem} +Let $S_n$ denote the [[Definition:Symmetric Group on n Letters|symmetric group on $n$ letters]]. +Then: +:$\forall \pi, \rho \in S_n: \map \sgn {\pi \rho \pi^{-1} } = \map \sgn \rho$ +where $\map \sgn \pi$ is the [[Definition:Sign of Permutation|sign of $\pi$]]. +\end{theorem} + +\begin{proof} +As $\map \sgn \pi = \pm 1$ for any $\pi \in S_n$, we can apply the laws of [[Definition:Commutative Operation|commutativity]] and [[Definition:Associative|associativity]]: +{{begin-eqn}} +{{eqn | l = \map \sgn \pi \, \map \sgn \rho + | r = \map \sgn \rho \, \map \sgn \pi + | c = +}} +{{eqn | ll= \leadsto + | l = \map \sgn \pi \, \map \sgn \rho \, \map \sgn {\pi^{-1} } + | r = \map \sgn \rho \, \map \sgn \pi \, \map \sgn {\pi^{-1} } + | c = +}} +{{eqn | ll= \leadsto + | l = \map \sgn {\pi \rho \pi^{-1} } + | r = \map \sgn \rho + | c = [[Parity Function is Homomorphism]] +}} +{{end-eqn}} +{{Qed}} +\end{proof}<|endoftext|> +\section{Parity of K-Cycle} +Tags: Cyclic Permutations + +\begin{theorem} +Let $\pi$ be a [[Definition:Cyclic Permutation|$k$-cycle]]. +Then: +:$\map \sgn \pi = \begin{cases} +1 & : k \ \text {odd} \\ +-1 & : k \ \text {even} +\end{cases}$ +Thus: +:$\map \sgn \pi = \paren {-1}^{k - 1}$ +or equivalently: +:$\map \sgn \pi = \paren {-1}^{k + 1}$ +\end{theorem} + +\begin{proof} +From [[Transposition is of Odd Parity]], any [[Definition:Transposition|transposition]] is of [[Definition:Parity (Permutation)|odd parity]]. +From [[K-Cycle can be Factored into Transpositions]], we see that a [[Definition:Cyclic Permutation|$k$-cycle]] is the product of $k - 1$ [[Definition:Transposition|transposition]]s. +Thus $\pi$ is even {{iff}} $k$ is [[Definition:Odd Integer|odd]]. +{{qed}} +\end{proof}<|endoftext|> +\section{Alternating Group is Normal Subgroup of Symmetric Group} +Tags: Symmetric Groups, Alternating Groups + +\begin{theorem} +Let $n \ge 2$ be a [[Definition:Natural Number|natural number]]. +Let $S_n$ denote the [[Definition:Symmetric Group on n Letters|symmetric group on $n$ letters]]. +Let $A_n$ be the [[Definition:Alternating Group|alternating group on $n$ letters]]. +Then $A_n$ is a [[Definition:Normal Subgroup|normal subgroup]] of $S_n$ whose [[Definition:Index of Subgroup|index]] is $2$. +\end{theorem} + +\begin{proof} +We have that $\map \sgn {S_n}$ is [[Definition:Surjection|onto]] $C_2$. +Thus from the [[First Isomorphism Theorem for Groups|First Isomorphism Theorem]], $A_n$ consists of the set of [[Definition:Even Permutation|even permutations]] of $S_n$. +The result follows from [[Subgroup of Index 2 is Normal]]. +{{Qed}} +\end{proof}<|endoftext|> +\section{Group of Permutations either All or Half Even} +Tags: Symmetric Groups + +\begin{theorem} +Let $G$ be a [[Definition:Group|group]] of [[Definition:Permutation|permutations]]. +Then either ''exactly half'' of the permutations in $G$ are [[Definition:Even Permutation|even]], or they are ''all'' [[Definition:Even Permutation|even]]. +\end{theorem} + +\begin{proof} +From [[Parity Function is Homomorphism]], the [[Definition:Mapping|mapping]]: +:$\sgn : G \to \Z_2$ +is a [[Definition:Group Homomorphism|homomorphism]]. +Then: +:$G / \map \ker \sgn \cong \Img \sgn$ +from the [[First Isomorphism Theorem for Groups|First Isomorphism Theorem]]. +The only possibilities for $\Img \sgn$ are $\set 0$ or $\Z_2$. +So either: +:$\order {G / \map \ker \sgn} = 1$ +in which case all the [[Definition:Permutation|permutations]] of $G$ are [[Definition:Even Permutation|even]], or: +:$\order {G / \map \ker \sgn} = 2$ +in which case exactly half of them are. +{{qed}} +[[Category:Symmetric Groups]] +jktrlc08nm231gxlx8dba3n056hbeqi +\end{proof}<|endoftext|> +\section{Cycle Decomposition of Conjugate} +Tags: Symmetric Groups, Conjugacy, Permutation Theory + +\begin{theorem} +Let $S_n$ denote the [[Definition:Symmetric Group on n Letters|symmetric group on $n$ letters]]. +Let $\pi, \rho \in S_n$. +The [[Definition:Cycle Decomposition|cycle decomposition]] of the [[Definition:Permutation|permutation]] $\pi \rho \pi^{-1}$ can be obtained from that of $\rho$ by replacing each $i$ in the [[Definition:Cycle Decomposition|cycle decomposition]] of $\rho$ with $\map \pi i$. +\end{theorem} + +\begin{proof} +Consider the effect of $\pi \rho \pi^{-1}$ on $\map \pi i$: +:$\map {\pi \rho \pi^{-1} } {\map \pi i} = \map \pi {\map \rho i}$ +That is: +:$\pi \rho \pi^{-1}$ maps $\map \pi i$ to $\map \pi {\map \rho i}$ +In the [[Definition:Cycle Decomposition|cycle decomposition]] of $\pi \rho \pi^{-1}$, $\map \pi i$ lies to the left of $\map \pi {\map \rho i}$, whereas in the [[Definition:Cycle Decomposition|cycle decomposition]] of $\rho$, $i$ lies to the left of $\map \rho i$. +{{qed}} +\end{proof}<|endoftext|> +\section{Conjugate Permutations have Same Cycle Type} +Tags: Conjugacy, Permutation Theory + +\begin{theorem} +Let $n \ge 1$ be a [[Definition:Natural Number|natural number]]. +Let $G$ be a [[Definition:Subgroup|subgroup]] of the [[Definition:Symmetric Group on n Letters|symmetric group on $n$ letters]] $S_n$. +Let $\sigma, \rho \in G$. +Then $\sigma$ and $\rho$ are [[Definition:Conjugate of Group Element|conjugate]] {{iff}} they have the same [[Definition:Cycle Type|cycle type]]. +\end{theorem} + +\begin{proof} +Let $\sigma \in G$ have [[Definition:Cycle Type|cycle type]] $\tuple {k_1, k_2, \ldots, k_n}$. +From [[Existence and Uniqueness of Cycle Decomposition]], $\sigma$ can be expressed uniquely as the [[Definition:Product Element|product]] of [[Definition:Disjoint Permutations|disjoint]] [[Definition:Cyclic Permutation|cycles]]: +:$\sigma = \alpha_1 \alpha_2 \dotsm \alpha_l$ +where $\alpha_i$ is a [[Definition:Cyclic Permutation|$k_i$-cycle]]. +Let $\tau \in G$ such that $\rho = \tau \sigma \tau^{-1}$. +Then: +{{begin-eqn}} +{{eqn | l = \tau \sigma \tau^{-1} + | r = \tau \alpha_1 \alpha_2 \dotsm \alpha_l \tau^{-1} + | c = +}} +{{eqn | n = 1 + | r = \tau \alpha_1 \tau^{-1} \tau \alpha_2 \tau^{-1} \dotsm \tau \alpha_l \tau^{-1} + | c = [[Product of Conjugates equals Conjugate of Products]] +}} +{{eqn | r = \alpha_{\map \tau 1} \alpha_{\map \tau 2} \dotsm \alpha_{\map \tau l} + | c = [[Cycle Decomposition of Conjugate]] +}} +{{end-eqn}} +We have that for $i, j \in \set {1, 2, \ldots, l}$, $\alpha_i$ and $\alpha_j$ are [[Definition:Disjoint Permutations|disjoint]]. +We also have that $\tau$ is a [[Definition:Bijection|bijection]], and so an [[Definition:Injection|injection]]. +It follows that $\alpha_{\map \tau i}$ and $\alpha_{\map \tau j}$ are also [[Definition:Disjoint Permutations|disjoint]] for all $i, j \in \set {1, 2, \ldots, l}$. +Thus: +:the [[Definition:Product Element|product]] in $(1)$ is $\rho$ written as the [[Definition:Product Element|product]] of [[Definition:Disjoint Permutations|disjoint]] [[Definition:Cyclic Permutation|cycles]] +where: +:$\tau \alpha_i \tau^{-1}$ is a [[Definition:Cyclic Permutation|$k_i$-cycle]]. +Thus $\rho$ has the same [[Definition:Cycle Type|cycle type]] as $\sigma$. +{{qed}} +\end{proof}<|endoftext|> +\section{Transpositions of Adjacent Elements generate Symmetric Group} +Tags: Symmetric Groups + +\begin{theorem} +Let $n \in \Z: n > 1$. +Let $S_n$ denote the [[Definition:Symmetric Group on n Letters|symmetric group on $n$ letters]]. +Then the [[Definition:Transposition|transpositions]] $a_k = \begin{pmatrix} k & k + 1 \end{pmatrix}$ for $1 \le k < n$ are a [[Definition:Generator of Group|set of generators]] for $S_n$. +They satisfy the relations: +{{begin-eqn}} +{{eqn | l = a_k^2 + | r = e + | c = (for $1 \le k < n$) +}} +{{eqn | l = \paren {a_k a_{k + 1} }^3 + | r = e + | c = (for $1 \le k < n - 1$) +}} +{{eqn | l = \paren {a_i a_j}^2 + | r = e + | c = (for $1 \le i, j < n, \size {i - j} > 1$) +}} +{{end-eqn}} +\end{theorem}<|endoftext|> +\section{Hermitian Matrix has Real Eigenvalues} +Tags: Hermitian Matrices, Hermitian Matrix has Real Eigenvalues + +\begin{theorem} +Every [[Definition:Hermitian Matrix|Hermitian matrix]] has [[Definition:Eigenvalue|eigenvalues]] which are all [[Definition:Real Number|real numbers]]. +\end{theorem} + +\begin{proof} +Let $\mathbf A$ be a [[Definition:Hermitian Matrix|Hermitian matrix]]. +Then, by definition: +: $\mathbf A = \mathbf A^*$ +where $\mathbf A^*$ denotes the [[Definition:Conjugate Transpose of Matrix|conjugate transpose]] of $\mathbf A$. +Let $\lambda$ be an [[Definition:Eigenvalue|eigenvalue]] of $\mathbf A$. +Let $\mathbf v$ be an [[Definition:Eigenvector|eigenvector]] corresponding to the [[Definition:Eigenvalue|eigenvalue]] $\lambda$. +By definition of [[Definition:Eigenvector|eigenvector]]: +: $\mathbf{A v} = \lambda \mathbf v$ +[[Definition:Matrix Product (Conventional)|Left-multiplying]] both sides by $\mathbf v^*$, we obtain: +:$(1):\quad \mathbf v^* \mathbf{A v} = \mathbf v^* \lambda \mathbf v = \lambda \mathbf v^* \mathbf v$ +Firstly, note that both $\mathbf v^* \mathbf{A v}$ and $\mathbf v^* \mathbf v$ are $1 \times 1$-matrices. +{{explain|Links to the proof of this fact}} +Now observe that, using [[Conjugate Transpose of Matrix Product/General Case|Conjugate Transpose of Matrix Product: General Case]]: +:$\left({\mathbf v^* \mathbf{A v}}\right)^* = \mathbf v^* \mathbf A^* \left({\mathbf v^*}\right)^*$ +As $\mathbf A$ is [[Definition:Hermitian Matrix|Hermitian]], and $\left({\mathbf v^*}\right)^* = \mathbf v$ by [[Conjugate Transpose is Involution]], it follows that: +:$\mathbf v^* \mathbf A^* \left({\mathbf v^*}\right)^* = \mathbf v^* \mathbf{A v}$ +That is, $\mathbf v^* \mathbf{A v}$ is also [[Definition:Hermitian Matrix|Hermitian]]. +By [[Product with Conjugate Transpose Matrix is Hermitian]], $\mathbf v^* \mathbf v$ is [[Definition:Hermitian Matrix|Hermitian]]. +So both $\mathbf v^* \mathbf{A v}$ and $\mathbf v^* \mathbf v$ are [[Definition:Hermitian Matrix|Hermitian]] $1 \times 1$ matrices. +Now suppose that we have for some $a,b \in \C$: +:$\mathbf v^* \mathbf{A v} = \begin{bmatrix}a\end{bmatrix}$ +:$\mathbf v^* \mathbf v = \begin{bmatrix}b\end{bmatrix}$ +Note that $b \ne 0$ as an [[Definition:Eigenvector|eigenvector]] is by definition [[Definition:Zero Vector|non-zero]]. +By definition of a [[Definition:Hermitian Matrix|Hermitian matrix]]: +: $a = \bar a$ and $b = \bar b$ +where $\bar a$ denotes the [[Definition:Complex Conjugate|complex conjugate]] of $a$. +By [[Complex Number equals Conjugate iff Wholly Real]], it follows that $a, b \in \R$, that is, are [[Definition:Real Number|real]]. +From equation $(1)$, it follows that $\begin{bmatrix}a\end{bmatrix} = \lambda \begin{bmatrix}b\end{bmatrix}$. +Thus, $a = \lambda b$, i.e. $\lambda = \dfrac a b$ (recall that $b \ne 0$). +Hence $\lambda$, being a [[Definition:Quotient (Algebra)|quotient]] of [[Definition:Real Number|real numbers]], is [[Definition:Real Number|real]]. +{{qed}} +\end{proof} + +\begin{proof} +Let $\mathbf A$ be a [[Definition:Hermitian Matrix|Hermitian matrix]]. +Then, by definition, $\mathbf A = \mathbf A^\dagger$, where $^\dagger$ designates the [[Definition:Conjugate Transpose of Matrix|conjugate transpose]]. +Let $\lambda$ be an [[Definition:Eigenvalue|eigenvalue]] of $\mathbf A$. +Let $\mathbf v$ be an [[Definition:Eigenvector|eigenvector]] corresponding to the [[Definition:Eigenvalue|eigenvalue]] $\lambda$ of $\mathbf A$. +Denote with $\left\langle{\cdot, \cdot}\right\rangle$ the [[Definition:Inner Product|inner product]] on $\C$. +{{begin-eqn}} +{{eqn | l = \lambda * \left\langle v, v\right\rangle + | r = \left\langle \lambda*v, v\right\rangle + | c = [[Properties of Complex Inner Product]] +}} +{{eqn | r = \left\langle \mathbf A*v, v\right\rangle + | c = {{Defof|Eigenvector}}: $\lambda*v = \mathbf A*v$ +}} +{{eqn | r = \left\langle v, \mathbf A^\dagger * v\right\rangle + | c = [[Properties of Adjugate]] +}} +{{eqn | r = \left\langle v, \mathbf A*v\right\rangle + | c = $\mathbf A$ is [[Definition:Hermitian Matrix|Hermitian]], so $\mathbf A^\dagger = \mathbf A$ +}} +{{eqn | r = \left\langle v, \lambda*v\right\rangle + | c = {{Defof|Eigenvector}}: $\lambda*v = \mathbf A*v$ +}} +{{eqn | r = \overline{\lambda}*\left\langle v, v\right\rangle + | c = [[Properties of Complex Inner Product]] +}} +{{end-eqn}} +We have that $v \ne 0$, and because of the [[Definition:Positive Definite|positive definiteness]], it must be that: +: $\left\langle v, v\right\rangle \ne 0$ +{{explain|A link is needed to whatever it is ($\left\langle v, v\right\rangle$, presumably) is [[Definition:Positive Definite|positive definite]].}} +Thus: +: $\left\langle v, v\right\rangle \ne 0$ +So we can divide both sides by $\left\langle v, v\right\rangle$. +Thus $\lambda = \overline{\lambda}$. +By [[Complex Number equals Conjugate iff Wholly Real]], $\lambda$ is a [[Definition:Real Number|real number]]. +$\lambda$ was arbitrary, so it follows that every [[Definition:Eigenvalue|eigenvalue]] is a [[Definition:Real Number|real number]]. +Hence the result. +{{qed}} +\end{proof}<|endoftext|> +\section{Permutation of Cosets} +Tags: Symmetric Groups, Cosets, Permutation Theory + +\begin{theorem} +Let $G$ be a [[Definition:Group|group]] and let $H \le G$. +Let $\mathbb S$ be the set of all distinct [[Definition:Left Coset|left cosets]] of $H$ in $G$. +Then: +:$(1): \quad$ For any $g \in G$, the [[Definition:Mapping|mapping]] $\theta_g: \mathbb S \to \mathbb S$ defined by $\map {\theta_g} {x H} = g x H$ is a [[Definition:Permutation|permutation]] of $\mathbb S$. +:$(2): \quad$ The mapping $\theta$ defined by $\map \theta g = \theta_g$ is a [[Definition:Group Homomorphism|homomorphism]] from $G$ into the [[Definition:Symmetric Group|symmetric group]] on $\mathbb S$. +:$(3): \quad$ The [[Definition:Kernel of Group Homomorphism|kernel]] of $\theta$ is the [[Definition:Subgroup|subgroup]] $\displaystyle \bigcap_{x \mathop \in G} x H x^{-1}$. +\end{theorem} + +\begin{proof} +First we need to show that $\theta_g$ is [[Definition:Well-Defined Mapping|well-defined]] and [[Definition:Injection|injective]]. +{{begin-eqn}} +{{eqn | l = x H + | r = y H + | c = +}} +{{eqn | ll= \leadstoandfrom + | l = y^{-1} x + | o = \in + | r = H + | c = +}} +{{eqn | ll= \leadstoandfrom + | l = \paren {g y}^{-1} g x + | r = y^{-1} x \in H + | c = +}} +{{eqn | ll= \leadstoandfrom + | l = \map {\theta_g} {y H} + | r = \map {\theta_g} {x H} + | c = +}} +{{end-eqn}} +Thus $\theta_g$ is [[Definition:Well-Defined Mapping|well-defined]] and [[Definition:Injection|injective]]. +Then we see that $\forall x H \in \mathbb S: \map {\theta_g} {g^{-1} x H} = x H$, so $\theta_g$ is [[Definition:Surjection|surjective]]. +Thus $\theta_g$ is a [[Definition:Well-Defined Mapping|well-defined]] [[Definition:Bijection|bijection]] on $\mathbb S$, and therefore a [[Definition:Permutation|permutation]] on $\mathbb S$. +Next we see: +{{begin-eqn}} +{{eqn | l = \map {\theta_{u v} } {x H} + | r = u v x H + | c = +}} +{{eqn | r = \map {\theta_u} {v x H} + | c = +}} +{{eqn | r = \map {\theta_u} {\map {\theta_v }{x H} }= + | c = +}} +{{end-eqn}} +This shows that $\theta_{u v} = \theta_u \theta_v$, and thus: +:$\map \theta {u v} = \map \theta u \, \map \theta v$ +Thus $\theta$ is a [[Definition:Group Homomorphism|homomorphism]]. +Now to calculate $\map \ker \theta$: +{{begin-eqn}} +{{eqn | l = \map \ker \theta + | r = \set {g \in G: \theta_g = I_\mathbb S} + | c = +}} +{{eqn | r = \set {g \in G: \forall x \in G: \map {\theta_g} {x H} = x H} + | c = +}} +{{eqn | r = \set {g \in G: \forall x \in G: g x h = x H} + | c = +}} +{{eqn | r = \set {g \in G: \forall x \in G: x^{-1} g x h = H} + | c = +}} +{{eqn | r = \set {g \in G: \forall x \in G: x^{-1} g x \in H} + | c = +}} +{{eqn | r = \set {g \in G: \forall x \in G: g \in x H x^{-1} } + | c = +}} +{{eqn | r = \bigcap_{x \mathop \in G} x H x^{-1} + | c = +}} +{{end-eqn}} +as required. +{{Qed}} +\end{proof}<|endoftext|> +\section{Cayley's Representation Theorem} +Tags: Symmetric Groups, Group Theory, Subgroups, Cayley's Representation Theorem + +\begin{theorem} +Let $S_n$ denote the [[Definition:Symmetric Group on n Letters|symmetric group on $n$ letters]]. +Every finite [[Definition:Group|group]] is [[Definition:Group Isomorphism|isomorphic]] to a [[Definition:Subgroup|subgroup]] of $S_n$ for some $n \in \Z$. +\end{theorem} + +\begin{proof} +Let $H = \set e$. +We can apply [[Permutation of Cosets]] to $H$ so that: +:$\mathbb S = G$ +and: +:$\map \ker \theta = \set e$ +The result follows by the [[First Isomorphism Theorem for Groups]]. +{{Qed}} +\end{proof} + +\begin{proof} +Let $G$ be any arbitrary [[Definition:Finite Group|finite group]] whose [[Definition:Identity Element|identity]] is $e_G$. +Let $S$ be the [[Definition:Symmetric Group|symmetric group]] on the elements of $G$, where $e_S$ is the [[Definition:Identity Element|identity]] of $S$. +For any $x \in G$, let $\lambda_x$ be the [[Definition:Left Regular Representation|left regular representation of $G$ with respect to $x$]]. +From [[Regular Representations in Group are Permutations]], $\forall x \in G: \lambda_x \in S$. +So, we can define a [[Definition:Mapping|mapping]] $\theta: G \to S$ as: +:$\forall x \in G: \map \theta x = \lambda_x$ +From [[Composition of Regular Representations]], we have: +:$\forall x, y \in G: \lambda_x \circ \lambda_y = \lambda_{x y}$ +where in this context $\circ$ denotes [[Definition:Composition of Mappings|composition of mappings]]. +Thus by definition of $\theta$: +:$\map \theta x \circ \map \theta y = \map \theta {x y}$ +demonstrating that $\theta$ is a [[Definition:Group Homomorphism|homomorphism]]. +Having established that fact, we can now consider $\map \ker \theta$, where $\ker$ denotes the [[Definition:Kernel of Group Homomorphism|kernel]] of $\theta$. +Let $x \in G$. +We have that: +{{begin-eqn}} +{{eqn | l = x + | o = \in + | r = \map \ker \theta + | c = +}} +{{eqn | ll= \leadsto + | l = \map \theta x + | r = e_S + | c = {{Defof|Kernel of Group Homomorphism}} +}} +{{eqn | ll= \leadsto + | l = \lambda_x + | r = I_G + | c = where $I_G$ is the [[Definition:Identity Mapping|identity mapping]] of $G$ +}} +{{eqn | ll= \leadsto + | l = \map {\lambda_x} {e_G} + | r = \map {I_G} {e_G} + | c = {{Defof|Identity Mapping}} +}} +{{eqn | ll= \leadsto + | l = x + | r = e_G + | c = Definitions of $\lambda_x$ and $I_G$ +}} +{{end-eqn}} +So $\map \ker \theta$ can contain no element other than $e_G$. +So, since clearly $e_G \in \map \ker \theta$, it follows that: +:$\map \ker \theta = \set {e_G}$ +By [[Kernel is Trivial iff Monomorphism]], it follows that $\theta$ is a [[Definition:Group Monomorphism|monomorphism]]. +By [[Monomorphism Image is Isomorphic to Domain]], we have that: +:$\theta \sqbrk G \cong \Img \theta$ +that is, $\theta$ is [[Definition:Group Isomorphism|isomorphic]] to its image. +Hence the result. +{{qed}} +\end{proof}<|endoftext|> +\section{Basic Results about Modules} +Tags: Module Theory + +\begin{theorem} +Let $\struct {G, +_G}$ be an [[Definition:Abelian Group|abelian group]] whose [[Definition:Identity Element|identity]] is $e$. +Let $\struct {R, +_R, \times_R}$ be a [[Definition:Ring (Abstract Algebra)|ring]] whose [[Definition:Ring Zero|zero]] is $0_R$. +Let $\struct {G, +_G, \circ}_R$ be an [[Definition:Module|$R$-module]]. +Let $x \in G, \lambda \in R, n \in \Z$. +Let $\sequence {x_m}$ be a [[Definition:Sequence|sequence of elements of $G$]]. +Let $\sequence {\lambda_m}$ be a [[Definition:Sequence|sequence of elements of $R$]] that is, [[Definition:Scalar (Module)|scalars]]. +Then: +\end{theorem}<|endoftext|> +\section{Basic Results about Unitary Modules} +Tags: Unitary Modules + +\begin{theorem} +Let $\struct {G, +_G}$ be an [[Definition:Abelian Group|abelian group]] whose [[Definition:Identity Element|identity]] is $e$. +Let $\struct {R, +_R, \times_R}$ be a [[Definition:Ring with Unity|ring with unity]] whose [[Definition:Ring Zero|zero]] is $0_R$ and whose [[Definition:Unity of Ring|unity]] is $1_R$. +Let $\struct {G, +_G, \circ}_R$ be an [[Definition:Unitary Module|unitary $R$-module]]. +Let $x \in G, n \in \Z$. +Then: +\end{theorem}<|endoftext|> +\section{Epimorphism preserves Modules} +Tags: Epimorphisms, Module Theory + +\begin{theorem} +Let $\left({G, +_G, \circ}\right)_R$ be an [[Definition:Module|$R$-module]]. +Let $\left({H, +_H, \circ}\right)_R$ be an [[Definition:R-Algebraic Structure|$R$-algebraic structure]]. +Let $\phi: G \to H$ be an [[Definition:R-Algebraic Structure Epimorphism|epimorphism]]. +Then $H$ is an [[Definition:Module|$R$-module]]. +\end{theorem} + +\begin{proof} +If $\left({G, +_G: \circ}\right)_R$ is an [[Definition:Module|$R$-module]], then: +$\forall x, y, \in G, \forall \lambda, \mu \in R$: +: $(1): \quad \lambda \circ \left({x +_G y}\right) = \left({\lambda \circ x}\right) +_G \left({\lambda \circ y}\right)$ +: $(2): \quad \left({\lambda +_R \mu}\right) \circ x = \left({\lambda \circ x}\right) +_G \left({\mu \circ x}\right)$ +: $(3): \quad \left({\lambda \times_R \mu}\right) \circ x = \lambda \circ \left({\mu \circ x}\right)$ +If $\phi: G \to H$ is an [[Definition:R-Algebraic Structure Epimorphism|epimorphism]], then: +: $\forall x, y \in G: \phi \left({x +_G y}\right) = \phi \left({x}\right) +_H \phi \left({y}\right)$ +: $\forall x \in S: \forall \lambda \in R: \phi \left({\lambda \circ x}\right) = \lambda \circ \phi \left({x}\right)$ +: $\forall y \in H: \exists x \in G: y = \phi \left({x}\right)$ +As $\phi$ is an epimorphism, we can accurately specify the behaviour of all elements of $H$, as they are the images of elements of $G$. If $\phi$ were not an epimorphism, i.e. not [[Definition:Surjection|surjective]], we would have no way of knowing the behaviour of elements of $H$ outside of the [[Definition:Image of Subset under Mapping|image]] of $G$. Hence the specification that $\phi$ needs to be an epimorphism. +Now we check the [[Definition:Module|criteria for $H$ being a module]], in turn. +=== Module $(1)$ === +{{begin-eqn}} +{{eqn | l=\lambda \circ \left({\phi \left({x}\right) +_H \phi \left({y}\right)}\right) + | r=\lambda \circ \left({\phi \left({x +_G y}\right)}\right) + | c= +}} +{{eqn | r=\phi \left({\lambda \circ \left({x +_G y}\right)}\right) + | c= +}} +{{eqn | r=\phi \left({\left({\lambda \circ x}\right) +_G \left({\lambda \circ y}\right)}\right) + | c= +}} +{{eqn | r=\phi \left({\lambda \circ x}\right) +_H \phi \left({\lambda \circ y}\right) + | c= +}} +{{end-eqn}} +Thus $(1)$ is shown to hold for $H$. +=== Module $(2)$ === +{{begin-eqn}} +{{eqn | l=\left({\lambda +_R \mu}\right) \circ \phi \left({x}\right) + | r=\phi \left({\left({\lambda +_R \mu}\right) \circ x}\right) + | c= +}} +{{eqn | r=\phi \left({\left({\lambda \circ x}\right) +_G \left({\mu \circ x}\right)}\right) + | c= +}} +{{eqn | r=\phi \left({\lambda \circ x}\right) +_H \phi \left({\mu \circ x}\right) + | c= +}} +{{eqn | r=\lambda \circ \phi \left({x}\right) +_H \mu \circ \phi \left({x}\right) + | c= +}} +{{end-eqn}} +Thus $(2)$ is shown to hold for $H$. +=== Module $(3)$ === +{{begin-eqn}} +{{eqn | l=\left({\lambda \times_R \mu}\right) \circ \phi \left({x}\right) + | r=\phi \left({\left({\lambda \times_R \mu}\right) \circ x}\right) + | c= +}} +{{eqn | r=\phi \left({\lambda \circ \left({\mu \circ x}\right)}\right) + | c= +}} +{{eqn | r=\lambda \circ \left({\phi \left({\mu \circ x}\right)}\right) + | c= +}} +{{eqn | r=\lambda \circ \left({\mu \circ \phi \left({x}\right)}\right) + | c= +}} +{{end-eqn}} +Thus $(3)$ is shown to hold for $H$. +So all the criteria for $H$ to be an [[Definition:Module|$R$-module]] are fulfilled. +{{qed}} +\end{proof}<|endoftext|> +\section{Condition for Linear Transformation} +Tags: Linear Transformations + +\begin{theorem} +Let $G$ be a [[Definition:Unitary Module|unitary $R$-module]], and let $H$ be an [[Definition:Module|$R$-module]]. +Let $\phi: G \to H$ be a [[Definition:Mapping|mapping]]. +Then $\phi$ is a [[Definition:Linear Transformation|linear transformation]] {{iff}}: +:$\forall x, y \in G: \forall \lambda, \mu \in R: \map \phi {\lambda x + \mu y} = \lambda \map \phi x + \mu \map \phi y$ +\end{theorem} + +\begin{proof} +Any [[Definition:Linear Transformation|linear transformation]] clearly satisfies the condition. +Let $\phi$ be such that the condition is satisfied. +Let $\lambda = \mu = 1_R$. +Then $\map \phi {x + y} = \map \phi x + \map \phi y$. +Now let $\mu = 0_R$. +Then $\map \phi {\lambda x} = \lambda \map \phi x$. +Thus the conditions are fulfilled for $\phi$ to be a [[Definition:R-Algebraic Structure Homomorphism|homomorphism]], that is, a [[Definition:Linear Transformation|linear transformation]]. +{{Qed}} +\end{proof}<|endoftext|> +\section{Module of All Mappings is Module} +Tags: Examples of Modules + +\begin{theorem} +Let $\struct {R, +_R, \times_R}$ be a [[Definition:Ring (Abstract Algebra)|ring]]. +Let $\struct {G, +_G, \circ}_R$ be an [[Definition:Module|$R$-module]]. +Let $S$ be a [[Definition:Set|set]]. +Let $\struct {G^S, +_G', \circ}_R$ be the [[Definition:Module of All Mappings|module of all mappings]] from $S$ to $G$. +Then $\struct {G^S, +_G', \circ}_R$ is an [[Definition:Module|$R$-module]]. +\end{theorem} + +\begin{proof} +To show that $\struct {G^S, +_G', \circ}_R$ is an [[Definition:Module|$R$-module]], we verify the following: +$\forall f, g, \in G^S, \forall \lambda, \mu \in R$: +:$(1): \quad \lambda \circ \paren {f +_G' g} = \paren {\lambda \circ f} +_G' \paren {\lambda \circ g}$ +:$(2): \quad \paren {\lambda +_R \mu} \circ f = \paren {\lambda \circ f} +_G \paren {\mu \circ f}$ +:$(3): \quad \paren {\lambda \times_R \mu} \circ f = \lambda \circ \paren {\mu \circ f}$ +=== Criterion 1 === +:$(1): \quad \lambda \circ \paren {f +_G' g} = \paren {\lambda \circ f} +_G' \paren {\lambda \circ g}$ +Let $x \in S$. +Then: +{{begin-eqn}} +{{eqn | l = \lambda \circ \paren {\map {\paren {f +_G' g} } x} + | r = \lambda \circ \paren {\map f x +_G \map g x} + | c = +}} +{{eqn | r = \paren {\lambda \circ \map f x} +_G \paren {\lambda \circ \map g x} + | c = {{Defof|Module}} +}} +{{eqn | r = \paren {\map {\paren {\lambda \circ f} } x} +_G \paren {\map {\paren {\lambda \circ g} } x} + | c = +}} +{{eqn | r = \map {\paren {\paren {\lambda \circ f} +_G' \paren {\lambda \circ g} } } x + | c = +}} +{{end-eqn}} +Thus $(1)$ holds. +{{qed|lemma}} +=== Criterion 2 === +: $(2): \quad \paren {\lambda +_R \mu} \circ f = \paren {\lambda \circ f} +_G \paren {\mu \circ f}$ +Let $x \in S$. +{{begin-eqn}} +{{eqn | l = \map {\paren {\paren {\lambda +_R \mu} \circ f} } x + | r = \paren {\lambda +_R \mu} \circ \paren {\map f x} + | c = +}} +{{eqn | r = \paren {\lambda \circ \map f x} +_G \paren {\mu \circ \map f x} + | c = +}} +{{eqn | r = \map {\paren {\lambda \circ f} } x +_G \map {\paren {\mu \circ f} } x + | c = +}} +{{end-eqn}} +Thus $(2)$ holds. +{{qed|lemma}} +=== Criterion 3 === +:$(3): \quad \paren {\lambda \times_R \mu} \circ f = \lambda \circ \paren {\mu \circ f}$ +{{begin-eqn}} +{{eqn | l = \map {\paren {\paren {\lambda \times_R \mu} \circ f} } x + | r = \paren {\lambda \times_R \mu} \circ \paren {\map f x} + | c = +}} +{{eqn | r = \lambda \circ \paren {\mu \circ \map f x} + | c = +}} +{{eqn | r = \lambda \circ \paren {\map {\paren {\mu \circ f} } x} + | c = +}} +{{eqn | r = \map {\paren {\lambda \circ \paren {\mu \circ f} } } x + | c = +}} +{{end-eqn}} +Thus $(3)$ holds. +{{qed|lemma}} +Hence the result. +{{qed}} +\end{proof}<|endoftext|> +\section{Kuratowski's Theorem} +Tags: Graph Theory + +\begin{theorem} +The following conditions on a [[Definition:Graph (Graph Theory)|graph]] $\Gamma$ are equivalent: +:$(1): \quad \Gamma$ is [[Definition:Planar Graph|planar]] +:$(2): \quad \Gamma$ contains no [[Definition:Graph Subdivision|subdivision]] of either the [[Definition:Complete Graph|complete graph $K_5$]] or the [[Definition:Complete Bipartite Graph|complete bipartite graph $K_{3, 3}$]]. +\end{theorem} + +\begin{proof} +Let $\Gamma$ be a [[Definition:Graph (Graph Theory)|graph]] with $V$ [[Definition:Vertex of Graph|vertices]], $E$ [[Definition:Edge of Graph|edges]] and $F$ [[Definition:Face of Graph|faces]]. +The proof proceeds in two parts: $(1) \implies (2)$ and $(2) \implies (1)$. +=== $(1)$ implies $(2)$ === +First we consider $K_5$. +$K_5$ has $5$ [[Definition:Vertex of Graph|vertices]] and $10$ [[Definition:Edge of Graph|edge]]s. +By the [[Euler Polyhedron Formula]], a [[Definition:Planar Embedding|planar embedding]] would have $7$ [[Definition:Face of Graph|faces]]. +But each [[Definition:Face of Graph|face]] has at least $3$ [[Definition:Edge of Graph|edge]]s, while each [[Definition:Edge of Graph|edge]] bounds at most two [[Definition:Face of Graph|face]]s. +Suppose we count the incident [[Definition:Edge of Graph|edge]]-[[Definition:Face of Graph|face]] pairs. +Then the number of [[Definition:Face of Graph|face]]s is at most: +:$E \dfrac 2 3 = 6 \dfrac 2 3$ +which [[Definition:Contradiction|contradicts]] the deduction that $K_5$, if [[Definition:Planar Graph|planar]], should have $7$ [[Definition:Face of Graph|face]]s. +Hence $K_5$ is [[Definition:Non-Planar Graph|non-planar]]. +Now consider $K_{3, 3}$. +$K_{3, 3}$ has $6$ [[Definition:Vertex of Graph|vertices]] and $9$ [[Definition:Edge of Graph|edge]]s. +Hence $K_{3, 3}$ has $5$ [[Definition:Face of Graph|face]]s if it is [[Definition:Planar Graph|planar]]. +Since $K_{3, 3}$ is [[Definition:Bipartite Graph|bipartite]], from [[Graph is Bipartite iff No Odd Cycles]], each [[Definition:Cycle (Graph Theory)|cycle]] in $K_{3,3}$ has an [[Definition:Even Integer|even number]] of [[Definition:Edge of Graph|edge]]s. +Hence any [[Definition:Face of Graph|face]] must have at least $4$ [[Definition:Edge of Graph|edge]]s. +So the number of [[Definition:Face of Graph|face]]s is at most +:$E \dfrac 2 4 = 4 \dfrac 1 2$ +which [[Definition:Contradiction|contradicts]] the deduction that $K_{3, 3}$, if [[Definition:Planar Graph|planar]], should have $5$ [[Definition:Face of Graph|face]]s. +Hence $K_{3, 3}$ is [[Definition:Non-Planar Graph|non-planar]]. +Now consider a [[Definition:Graph Subdivision|subdivision]] of either graph. + +The arguments as above continue to work as before, since both $V$ and $E$ have increased by one in the [[Euler Polyhedron Formula|Euler formula]]. +So the expected value of $F$ is unchanged. +Hence if a [[Definition:Graph (Graph Theory)|graph]] $\Gamma$ contains a [[Definition:Graph Subdivision|subdivision]] of $K_5$ or $K_{3,3}$, then $\Gamma$ is not [[Definition:Planar Graph|planar]]. +Taking the [[Definition:Contrapositive Statement|contrapositive]]: +:$\Gamma$ is a [[Definition:Planar Graph|planar graph]] $\implies \Gamma$ does not contain a [[Definition:Graph Subdivision|subdivision]] of either $K_5$ or $K_{3, 3}$. +=== $(2)$ implies $(1)$ === +{{ProofWanted}} +{{namedfor|Kazimierz Kuratowski|cat = Kuratowski}} +[[Category:Graph Theory]] +gedzpex6s6czckd0wgmg0gqzs6smt6d +\end{proof}<|endoftext|> +\section{Fundamental Theorem of Finite Abelian Groups} +Tags: Fundamental Theorems, Abelian Groups, Proofs by Induction + +\begin{theorem} +Every [[Definition:Finite Group|finite]] [[Definition:Abelian Group|abelian group]] is an [[Definition:Internal Group Direct Product|internal group direct product]] of [[Definition:Cyclic Group|cyclic groups]] whose [[Definition:Order of Group|orders]] are [[Definition:Prime Power|prime powers]]. +The number of terms in the [[Definition:Internal Group Direct Product|product]] and the [[Definition:Order of Group|orders]] of the [[Definition:Cyclic Group|cyclic groups]] are [[Definition:Unique|uniquely determined]] by the group. +\end{theorem} + +\begin{proof} +Let $G$ be a [[Definition:Finite Group|finite]] [[Definition:Abelian Group|abelian group]]. +By means of [[Abelian Group is Product of Prime-power Order Groups]], we factor it uniquely into groups of [[Definition:Prime Power|prime-power]] [[Definition:Order of Group|order]]. +Then, [[Abelian Group of Prime-power Order is Product of Cyclic Groups]] applies to each of these factors. +Hence we conclude $G$ factors into [[Definition:Prime Power|prime-power]] [[Definition:Order of Group|order]] [[Definition:Cyclic Group|cyclic groups]]. +The factorisation of $G$ into [[Definition:Prime Power|prime-power]] [[Definition:Order of Group|order]] factors is already [[Definition:Unique|unique]]. +Therefore, a demonstration of the uniqueness of the secondary factorisation suffices: +Suppose $\order G = p^k$ with $p$ a [[Definition:Prime Number|prime]]. +Let $G$ have the following two factorisations: +:$G = H_1 \times H_2 \times \cdots \times H_m = K_1 \times K_2 \times \dotsb \times K_n$ +where the $H_i$'s and $K_i$'s are [[Definition:Non-Trivial Group|non-trivial]] [[Definition:Cyclic Group|cyclic]] [[Definition:Subgroup|subgroups]] with +:$\order {H_1} \ge \order {H_2} \ge \dotsb \ge \order {H_m}|$ +:$\order {K_1} \ge \order {K_2} \ge \dotsb \ge \order {K_n}$ +We proceed through [[Second Principle of Mathematical Induction|complete induction]] on $k$. +=== Basis for the induction === +For $k = 1$, the statement follows from [[Prime Group is Cyclic]]. +=== Induction Hypothesis === +Now we assume the theorem is true for all [[Definition:Abelian Group|abelian groups]] of [[Definition:Order of Structure|order]] $p^l$, where $l < k$. +=== Induction Step === +By [[Power of Elements is Subgroup]], $G^p = \set {x^p: x \in G}$ is a [[Definition:Proper Subgroup|proper subgroup]] of $G$. +It follows that: +:$G^p = H_1^p \times \cdots \times H_{m'}^p = K_1^p \times \cdots \times K_{n'}^p$ +where: +:$m'$ is the largest integer $i$ such that $\order {H_i} > p$ +:$n'$ is the largest integer $j$ such that $\order {K_j} > p$ +This $m'$ and $n'$ ensure that the [[Definition:Internal Group Direct Product|direct products]] above do not have trivial factors. +Also, by [[Cauchy's Group Theorem]], we have: +:$\order {G^p} < \order G$ +This means that we can apply the induction hypothesis. +It follows that: +:$m' = n'$ +and: +:$\order {H_i^p} = \order {K_i^p}$ +for $i = 1, 2, \ldots, m'$. +We know that $\order {H_i} = p \order {H_i^p}$, and the same for $K_i$ and $K_i^p$. +It follows that $\order {H_i} = \order {K_i}$ for all $i = 1, 2, \ldots, m'$. +Now by construction of $m'$: +:$i > m' \implies \order {H_i} = p = \order {K_i}$ +It follows that we have: +:$\order {H_1} \order {H_2} \dotsm \order {H_{m'} } p^{m - m'} = \order G = \order {K_1} \order {K_2} \dotsm \order {K_{n'} } p^{n - n'}$ +Therefore: +:$m - m' = n - n'$ +and as $m' = n'$, it must follow that $m = n$. +The result follows by [[Second Principle of Mathematical Induction|induction]]. +{{qed}} +\end{proof}<|endoftext|> +\section{Finite Direct Product of Modules is Module} +Tags: Module Theory, Direct Products, Finite Direct Product of Modules is Module + +\begin{theorem} +Let $\struct {R, +_R, \times_R}$ be a [[Definition:Ring (Abstract Algebra)|ring]]. +Let $\struct {G_1, +_1, \circ_1}_R, \struct {G_2, +_2, \circ_2}_R, \ldots, \struct {G_n, +_n, \circ_n}_R$ be [[Definition:Module|$R$-modules]]. +Let: +:$\ds G = \prod_{k \mathop = 1}^n G_k$ +be their [[Definition:Module Direct Product|direct product]]. +Then $G$ is a [[Definition:Module|module]]. +\end{theorem} + +\begin{proof} +This is a special case of [[Direct Product of Modules is Module]]. +\end{proof} + +\begin{proof} +We need to show that: +$\forall x, y, \in G, \forall \lambda, \mu \in R$: +:$(1): \quad \lambda \circ \paren {x + y} = \paren {\lambda \circ x} + \paren {\lambda \circ y}$ +:$(2): \quad \paren {\lambda +_R \mu} \circ x = \paren {\lambda \circ x} + \paren {\mu \circ x}$ +:$(3): \quad \paren {\lambda \times_R \mu} \circ x = \lambda \circ \paren {\mu \circ x}$ +Checking the criteria in order: +=== Criterion 1 === +:$(1): \quad \lambda \circ \paren {x + y} = \paren {\lambda \circ x} + \paren {\lambda \circ y}$ +Let $x = \tuple {x_1, x_2, \ldots, x_n}, y = \tuple {y_1, y_2, \ldots, y_n} \in G$. +{{begin-eqn}} +{{eqn | l = \lambda \circ \paren {x + y} + | r = \lambda \circ \paren {\tuple {x_1, x_2, \ldots, x_n} + \tuple {y_1, y_2, \ldots, y_n} } + | c = +}} +{{eqn | r = \lambda \circ \tuple {x_1 +_1 y_1, x_2 +_2 y_2, \ldots, x_n +_n y_n} + | c = +}} +{{eqn | r = \tuple {\lambda \circ_1 \paren {x_1 + y_1}, \lambda \circ_2 \paren {x_2 + y_2}, \ldots, \lambda \circ_n \paren {x_n + y_n} } + | c = +}} +{{eqn | r = \tuple {\lambda \circ_1 x_1, \lambda \circ_2 x_2, \ldots, \lambda \circ_n x_n} + \tuple {\lambda \circ_1 y_1, \lambda \circ_2 y_2, \ldots, \lambda \circ_n y_n} + | c = +}} +{{eqn | r = \paren {\lambda \circ \tuple {x_1, x_2, \ldots, x_n} } + \paren {\lambda \circ \tuple {y_1, y_2, \ldots, y_n} } + | c = +}} +{{eqn | r = \paren {\lambda \circ x} + \paren {\lambda \circ y} + | c = +}} +{{end-eqn}} +So $(1)$ holds. +{{qed|lemma}} +=== Criterion 2 === +:$(2): \quad \paren {\lambda +_R \mu} \circ x = \paren {\lambda \circ x} + \paren {\mu \circ x}$ +Let $x = \tuple {x_1, x_2, \ldots, x_n} \in G$. +{{begin-eqn}} +{{eqn | l = \paren {\lambda +_R \mu} \circ x + | r = \paren {\lambda +_R \mu} \circ \tuple {x_1, x_2, \ldots, x_n} + | c = +}} +{{eqn | r = \tuple {\paren {\lambda +_R \mu} \circ_1 x_1, \paren {\lambda +_R \mu} \circ_2 x_2, \ldots, \paren {\lambda +_R \mu} \circ_n x_n} + | c = +}} +{{eqn | r = \tuple {\paren {\lambda \circ_1 x_1} +_1 \paren {\mu \circ_1 x_1}, \paren {\lambda \circ_2 x_2} +_2 \paren {\mu \circ_2 x_2}, \ldots, \paren {\lambda \circ_n x_n} +_n \paren {\mu \circ_n x_n} } + | c = +}} +{{eqn | r = \tuple {\lambda \circ_1 x_1, \lambda \circ_2 x_2, \lambda \circ_n x_n} + \tuple {\mu \circ_1 x_1, \mu \circ_2 x_2, \ldots, \mu \circ_n x_n} + | c = +}} +{{eqn | r = \paren {\lambda \circ_1 \tuple {x_1, x_2, \ldots, x_n} } + \paren {\mu \circ_1 \tuple {x_1, x_2, \ldots, x_n} } + | c = +}} +{{eqn | r = \paren {\lambda \circ x} + \paren {\mu \circ x} + | c = +}} +{{end-eqn}} +So $(2)$ holds. +{{qed|lemma}} +=== Criterion 3 === +:$(3): \quad \paren {\lambda \times_R \mu} \circ x = \lambda \circ \paren {\mu \circ x}$ +Let $x = \tuple {x_1, x_2, \ldots, x_n} \in G$. +{{begin-eqn}} +{{eqn | l = \paren {\lambda \times_R \mu} \circ x + | r = \paren {\lambda \times_R \mu} \circ \tuple {x_1, x_2, \ldots, x_n} + | c = +}} +{{eqn | r = \tuple {\paren {\lambda \times_R \mu} \circ_1 x_1, \paren {\lambda \times_R \mu} \circ_2 x_2, \ldots, \paren {\lambda \times_R \mu} \circ_n x_n} + | c = +}} +{{eqn | r = \tuple {\lambda \circ_1 \paren {\mu \circ_1 x_1}, \lambda \circ_2 \paren {\mu \circ_2 x_2}, \ldots, \lambda \circ_n \paren {\mu \circ_n x_n} } + | c = +}} +{{eqn | r = \lambda \circ \tuple {\mu \circ_1 x_1, \mu \circ_2 x_2, \ldots, \mu \circ_n x_n} + | c = +}} +{{eqn | r = \lambda \circ \paren {\mu \circ \tuple {x_1, x_2, \ldots, x_n} } + | c = +}} +{{eqn | r = \lambda \circ \paren {\mu \circ x} + | c = +}} +{{end-eqn}} +So $(3)$ holds. +{{qed|lemma}} +Hence the result. +{{qed}} +\end{proof}<|endoftext|> +\section{Subring Module} +Tags: Subrings, Module Theory + +\begin{theorem} +Let $\struct {R, +, \times}$ be a [[Definition:Ring (Abstract Algebra)|ring]]. +Let $\struct {S, +_S, \times_S}$ be a [[Definition:Subring|subring]] of $R$. +Let $\struct {G, +_G, \circ}_R$ be an [[Definition:Module|$R$-module]]. +Let $\circ_S$ be the [[Definition:Restriction of Operation|restriction]] of $\circ$ to $S \times G$. +Then $\struct {G, +_G, \circ_S}_S$ is an [[Definition:Module|$S$-module]]. +The module $\struct {G, +_G, \circ_S}_S$ is called the '''$S$-module obtained from $\struct {G, +_G, \circ}_R$ by restricting scalar multiplication'''. +{{refactor|Extract the below into its own page}} +If $\struct {G, +_G, \circ}_R$ is a [[Definition:Unitary Module|unitary $R$-module]] and $1_R \in S$, then $\struct{G, +_G, \circ_S}_S$ is also [[Definition:Unitary Module|unitary]]. +\end{theorem} + +\begin{proof} +We have that: +:$\forall a, b \in S: a +_S b = a + b$ +:$\forall a, b \in S: a \times_S b = a \times b$ +:$\forall a \in S: \forall x \in G = a \circ_S x = a \circ x$ +as $+_S$, $\times_S$ and $\circ_S$ are [[Definition:Restriction of Operation|restrictions]]. +Let us verify the [[Definition:Module Axioms|module axioms]]. +=== {{Module-axiom|1}} === +We need to show that: +:$\forall a \in S: \forall x, y \in G: a \circ_S \paren {x +_G y} = a \circ_S x +_G a \circ_S y$ +We have: +{{begin-eqn}} +{{eqn | l = a \circ_S \paren {x +_G y} + | r = a \circ \paren {x +_G y} +}} +{{eqn | r = a \circ x +_G a \circ y + | c = {{Module-axiom|1}} on $\struct {G, +_G, \circ}_R$ +}} +{{eqn | r = a \circ_S x +_G a \circ_S y +}} +{{end-eqn}} +{{qed|lemma}} +=== {{Module-axiom|2}} === +We need to show that: +:$\forall a,b \in S: \forall x \in G: \paren {a +_S b} \circ_S x = a \circ_S x +_G b \circ_S y$ +We have: +{{begin-eqn}} +{{eqn | l = \paren {a +_S b} \circ_S x + | r = \paren {a + b} \circ x +}} +{{eqn | r = a \circ x + b \circ x + | c = {{Module-axiom|2}} on $\struct {G, +_G, \circ}_R$ +}} +{{eqn | r = a \circ_S x +_G b \circ_S x +}} +{{end-eqn}} +{{qed|lemma}} +=== {{Module-axiom|3}} === +We need to show that: +:$\forall a, b \in S: \forall x \in G: \paren {a \times_S b} \circ_S x = a \circ_S \paren {b \circ_S x}$ +We have: +{{begin-eqn}} +{{eqn | l = \paren {a \times_S b} \circ_S x + | r = \paren {a \times b} \circ x +}} +{{eqn | r = a \circ \paren {b \circ x} + | c = {{Module-axiom|3}} on $\struct {G, +_G, \circ}_R$ +}} +{{eqn | r = a \circ_S \paren {b \circ_S x} +}} +{{end-eqn}} +{{qed|lemma}} +Thus $\struct {G, +_G, \circ_S}_S$ is an [[Definition:Module|$S$-module]]. +{{qed|lemma}} +It remains to prove the final statement: +If $\struct {G, +_G, \circ}_R$ is a [[Definition:Unitary Module|unitary $R$-module]] and $1_R \in S$, then $\struct {G, +_G, \circ_S}_S$ is also [[Definition:Unitary Module|unitary]]. +To show that $\struct {G, +_G, \circ_S}_S$ is [[Definition:Unitary Module|unitary]], we must prove that: +:$\forall x \in G: 1_R \circ_S x = x$ +Since $1_R \in S$ by assumption, the [[Product Element|product]] $1_R \circ_S x$ is defined. +We now have: +{{begin-eqn}} +{{eqn | l = 1_R \circ_S x + | r = 1_R \circ x +}} +{{eqn | r = x + | c = [[Definition:Unitary Module Axioms|Unitary Module Axiom $(4)$]] on $\struct {G, +_G, \circ}_R$ +}} +{{end-eqn}} +{{qed}} +\end{proof}<|endoftext|> +\section{Z-Module Associated with Abelian Group is Unitary Z-Module} +Tags: Examples of Unitary Modules, Z-Module Associated with Abelian Group + +\begin{theorem} +Let $\struct {G, *}$ be an [[Definition:Abelian Group|abelian group]] with [[Definition:Identity Element|identity]] $e$. +Let $\struct {G, *, \circ}_\Z$ be the [[Definition:Z-Module Associated with Abelian Group|$Z$-module associated with $G$]]. +Then $\struct {G, *, \circ}_\Z$ is a [[Definition:Unitary Module|unitary $Z$-module]]. +\end{theorem} + +\begin{proof} +The notation $*^n x$ can be written as $x^n$. +Let us verify that $\struct {G, *, \circ}_\Z$ is a [[Definition:Unitary Module|unitary $\Z$-module]] by verifying the axioms in turn. +=== Axiom $(\text M 1)$ === +We need to show that $n \circ \paren {x * y} = \paren {n \circ x} * \paren {n \circ y}$. +From the definition, $n \circ x = x^n$ and so $n \circ \paren {x * y} = \paren {x * y}^n$ +From [[Power of Product in Abelian Group]]: +:$\paren {x * y}^n = x^n * y^n = \paren {n \circ x} * \paren {n \circ y}$ +{{qed|lemma}} +=== Axiom $(\text M 2)$ === +We need to show that $\paren {n + m} \circ x = \paren {n \circ x} * \paren {m \circ x}$. +That is, that $x^{n + m} = x^n * x^m$. +This is an instance of [[Powers of Group Elements/Sum of Indices|Powers of Group Elements: Sum of Indices]]. +{{qed|lemma}} +=== Axiom $(\text M 3)$ === +We need to show that $\paren {n \times m} \circ x = n \circ \paren {m \circ x}$. +That is, that $x^{n m} = \paren {x^m}^n$. +This follows directly from [[Powers of Group Elements/Product of Indices|Powers of Group Elements: Product of Indices]]. +{{qed|lemma}} +=== Axiom $(\text M 4)$ === +We need to show that $\forall x \in G: 1 \circ x = x$. +That is, that $x^1 = x$. +This follows from the definition of [[Definition:Power of Group Element|Power of Group Element]]. +{{qed|lemma}} +Having verified all four axioms, we have shown that $\struct {G, *, \circ}_\Z$ is a [[Definition:Unitary Module|unitary $\Z$-module]]. +{{qed}} +\end{proof}<|endoftext|> +\section{Projection on Cartesian Product of Modules} +Tags: Module Theory, Cartesian Product, Epimorphisms, Projections + +\begin{theorem} +Let $G$ be the [[Definition:Cartesian Product|cartesian product]] of a [[Definition:Sequence|sequence]] $\sequence {G_n}$ of [[Definition:Module|$R$-modules]]. +Then for each $j \in \closedint 1 n$, the [[Definition:Projection (Mapping Theory)|projection]] $\pr_j$ on the $j$th co-ordinate is an [[Definition:R-Algebraic Structure Epimorphism|epimorphism]] from $G$ onto $G_j$. +\end{theorem}<|endoftext|> +\section{Canonical Injection into Cartesian Product of Modules} +Tags: Module Theory, Cartesian Product, Monomorphisms + +\begin{theorem} +Let $G$ be the [[Definition:Cartesian Product|cartesian product]] of a [[Definition:Sequence|sequence]] $\sequence {G_n}$ of [[Definition:Module|$R$-modules]]. +Then for each $j \in \closedint 1 n$, the [[Definition:Canonical Injection (Abstract Algebra)|canonical injection]] $\inj_j$ from $G_j$ into $G$ is a [[Definition:R-Algebraic Structure Monomorphism|monomorphism]]. +\end{theorem} + +\begin{proof} +$G$ can be seen as functions: +:$\displaystyle f: A \to \bigcup_{a \mathop \in A} G_a$ +{{explain|Clarify the above sentence.}} +Let $a \in A$. +Let $x, y \in G_a$. +Let $r \in R$. +So both $x + y \in G_a$ and $r x \in G_a$. +Let $b \in A$. +=== Case 1 === +{{explain|Explain and clarify the notation}} +Let $b = a$. +Then: +:$\map {\map {\inj_a} {x + y} } b = x + y = \map {\map {\inj_a} x} b + \map {\map {\inj_a} y} b$ +and: +:$\map {\map {\inj_a} {r x} } b = r x = r \, \map {\map {\inj_a} x} b$ +{{qed|lemma}} +=== Case 2 === +Let $b \ne a$. +Then: +:$\map {\map {\inj_a} {x + y} } b = 0 + 0 = \map {\map {\inj_a} x} b + \map {\map {\inj_a} y} b$ +and: +:$\map {\map {\inj_a} {r x} } b = 0 = \map r 0 = r \, \map {\map {\inj_a} x} b$ +Therefore: +:$\map {\inj_a} {x + y} = \map {\inj_a} x + \map {\inj_a} y$ +and: +:$\map {\inj_a} {r x} = r \, \map {\inj_a} x$ +{{qed|lemma}} +So $\inj_a$ is a [[Definition:R-Algebraic Structure Homomorphism|homomorphism]]. +Combined with [[Canonical Injection is Injection]] gives that $\inj_a$ is a [[Definition:R-Algebraic Structure Monomorphism|monomorphism]]. +{{qed}} +\end{proof}<|endoftext|> +\section{Submodule Test} +Tags: Submodules + +\begin{theorem} +Let $\left({G, +, \circ}\right)_R$ be a [[Definition:Unitary Module|unitary $R$-module]]. +Let $H$ be a [[Definition:Empty Set|non-empty]] [[Definition:Subset|subset]] of $G$. +Then $\left({H, +, \circ}\right)_R$ is a [[Definition:Submodule|submodule]] of $G$ iff: +:$\forall x, y \in H: \forall \lambda \in R: x + y \in H, \lambda \circ x \in H$ +\end{theorem} + +\begin{proof} +If the conditions are fulfilled, then: +:$x \in H \implies -x = \left({- 1_R}\right) \circ x \in H$ +Thus $H$ is a [[Definition:Subgroup|subgroup]] of $\left({G, +}\right)$ by the [[Two-Step Subgroup Test]], and hence a [[Definition:Submodule|submodule]]. +{{qed}} +\end{proof}<|endoftext|> +\section{Module is Submodule of Itself} +Tags: Submodules + +\begin{theorem} +Let $\left({G, +_G, \circ}\right)_R$ be an [[Definition:Module|$R$-module]]. +Then $\left({G, +_G, \circ}\right)_R$ is a [[Definition:Submodule|submodule]] of itself. +\end{theorem} + +\begin{proof} +Follows directly from the fact that a [[Group is Subgroup of Itself|group is a subgroup of itself]]. +{{qed}} +\end{proof}<|endoftext|> +\section{Null Module Submodule of All} +Tags: Submodules, Module Theory + +\begin{theorem} +Let $\left({G, +_G, \circ}\right)_R$ be an [[Definition:Module|$R$-module]]. +Then the [[Definition:Null Module|null module]]: +:$\left({\left\{{e_G}\right\}, +_G, \circ}\right)_R$ +is a [[Definition:Submodule|submodule]] of $\left({G, +_G, \circ}\right)_R$. +\end{theorem} + +\begin{proof} +Follows directly from the fact that the [[Trivial Subgroup is Subgroup|trivial subgroup is a subgroup of $\left({G, +_G}\right)$]]. +{{qed}} +\end{proof}<|endoftext|> +\section{First Sylow Theorem} +Tags: Sylow Theorems, First Sylow Theorem + +\begin{theorem} +Let $p$ be a [[Definition:Prime Number|prime number]]. +Let $G$ be a [[Definition:Group|group]] such that: +:$\order G = k p^n$ +where: +:$\order G$ denotes the [[Definition:Order of Structure|order]] of $G$ +:$p$ is not a [[Definition:Divisor of Integer|divisor]] of $k$. +Then $G$ has at least one [[Definition:Sylow p-Subgroup|Sylow $p$-subgroup]]. +\end{theorem} + +\begin{proof} +Let $\order G = k p^n$ such that $p \nmid k$. +Let $\mathbb S = \set {S \subseteq G: \order S = p^n}$, that is, the set of all of [[Definition:Subset|subsets]] of $G$ which have exactly $p^n$ [[Definition:Element|elements]]. +Let $N = \order {\mathbb S}$. +Now $N$ is the number of ways $p^n$ [[Definition:Element|elements]] can be chosen from a set containing $p^n k$ [[Definition:Element|elements]]. + +From [[Cardinality of Set of Subsets]], this is given by: +:$N = \dbinom {p^n k} {p^n} = \dfrac {\paren {p^n k} \paren {p^n k - 1} \cdots \paren {p^n k - i} \cdots \paren {p^n k - p^n + 1} } {\paren {p^n} \paren {p^n - 1} \cdots \paren {p^n - i} \cdots \paren 1}$ +From [[Binomial Coefficient involving Power of Prime]]: +:$\dbinom {p^n k} {p^n} \equiv k \pmod p$ +Thus: +:$N \equiv k \pmod p$ +Now let $G$ [[Definition:Group Action|act]] on $\mathbb S$ by the rule: +:$\forall S \in \mathbb S: g * S = g S = \set {x \in G: x = g s: s \in S}$ +That is, $g * S$ is the [[Definition:Left Coset|left coset]] of $S$ by $g$. +From [[Group Action on Sets with k Elements]], this is a [[Definition:Group Action|group action]]. +Now, let $\mathbb S$ have $r$ [[Definition:Orbit (Group Theory)|orbits]] under this [[Definition:Group Action|action]]. +From [[Set of Orbits forms Partition]], the [[Definition:Orbit (Group Theory)|orbits]] [[Definition:Set Partition|partition]] $\mathbb S$. +Let these [[Definition:Orbit (Group Theory)|orbits]] be represented by $\set {S_1, S_2, \ldots, S_r}$, so that: +{{begin-eqn}} +{{eqn | l = \mathbb S + | r = \Orb {S_1} \cup \Orb {S_2} \cup \ldots \cup \Orb {S_r} + | c = +}} +{{eqn | l = \size {\mathbb S} + | r = \size {\Orb {S_1} } + \card {\Orb {S_2} } + \ldots + \size {\Orb {S_r} } + | c = +}} +{{end-eqn}} +If each [[Definition:Orbit (Group Theory)|orbit]] had [[Definition:Length of Orbit|length]] [[Definition:Divisor of Integer|divisible]] by $p$, then $p \divides N$. +But this can not be the case, because, as we have seen: +:$N \equiv k \pmod p$ +So at least one [[Definition:Orbit (Group Theory)|orbit]] has [[Definition:Length of Orbit|length]] which is ''not'' divisible by $p$. +Let $S \in \set {S_1, S_2, \ldots, S_r}$ be such that $\size {\Orb S)} = m: p \nmid m$. +Let $s \in S$. +It follows from [[Group Action on Prime Power Order Subset]] that: +:$\Stab S s = S$ +and so: +:$\size {\Stab S} = \size S = p^n$ +From [[Stabilizer is Subgroup]]: +:$\Stab S \le G$ +Thus $\Stab S$ is the [[Definition:Subgroup|subgroup]] of $G$ with $p^n$ [[Definition:Element|elements]] of which we wanted to prove the existence. +{{Qed}} +\end{proof} + +\begin{proof} +Let $\Bbb S = \set {S \subseteq G: \order S = p^n}$, that is, the set of all of [[Definition:Subset|subsets]] of $G$ which have exactly $p^n$ [[Definition:Element|elements]]. +Now let $G$ [[Definition:Group Action|act]] on $\Bbb S$ by the rule: +:$\forall S \in \Bbb S: g * S = g S = \set {x \in G: x = g s: s \in S}$ +From [[Set of Orbits forms Partition]], the [[Definition:Orbit (Group Theory)|orbits]] [[Definition:Set Partition|partition]] $\mathbb S$. +Let these [[Definition:Orbit (Group Theory)|orbits]] be represented by $\set {S_1, S_2, \ldots, S_n}$, so that: +:$(1): \quad \size {\mathbb S} = \size {\Orb {S_1} } + \card {\Orb {S_2} } + \ldots + \size {\Orb {S_r} }$ +Thus each $\Orb {S_i}$ is the [[Definition:Orbit (Group Theory)|orbit]] under $*$ of some $S_i$ whose [[Definition:Order of Group|order]] is $p^n$. +By the [[Orbit-Stabilizer Theorem]]: +:$(2): \quad \order {\Orb {S_i} } = \dfrac {\order G} {\order {\Stab {S_i} } }$ for all $i \in \set {1, 2, \ldots, n}$ +where $\Stab {S_i}$ is the [[Definition:Stabilizer|stabilizer]] of $S_i$ under $*$. +Let $s \in S_i$ and $x \in \Stab {S_i}$. +Then $s x \in S_i$ because $S_i x = S_i$. +Let $s$ be fixed while $x$ runs over $\Stab {S_i}$. +Then: +:$\forall s \in S_i: s \Stab {S_i} \subseteq S_i$ +Therefore $S_i$ is the [[Definition:Set Union|union]] of all the [[Definition:Left Coset|left cosets]] $s \Stab {S_i}$. +These [[Definition:Left Coset|left cosets]] may not all be [[Definition:Distinct Elements|distinct]]. +Let $r_i$ be the number of [[Definition:Distinct Elements|distinct]] [[Definition:Left Coset|left cosets]] of $s \Stab {S_i}$. +Then: +:$\card {S_i} = r_i \card {\Stab {S_i} }$ +because: +: [[Definition:Distinct Elements|distinct]] [[Definition:Left Coset|left cosets]] are [[Definition:Disjoint Sets|disjoint]] +: all have the same [[Definition:Cardinality|cardinality]] $\card {\Stab {S_i} }$. +Thus: +:$(3): \quad \card {\Stab {S_i} } \divides \card {S_i} = p^n$ +Because the only [[Definition:Divisor of Integer|divisors]] of $p^n$ are smaller [[Definition:Integer Power|powers]] of $p$: +:$\card {\Stab {S_i} } = p^{n_i}$ +for some $n_i \le n$, for $i = 1, 2, \ldots, r$. +We can write: +:$g = \order G = k p^n$ +where $p$ does not [[Definition:Divisor of Integer|divide]] $k$. +From $(2)$ and $(3)$: +:$\card {S_i} = k p^{d_i}$ +where $d_i = n - n_i$ for $i = 1, 2, \ldots, r$. +From [[Cardinality of Set of Subsets]]: +:$\card {\Bbb S} = \dbinom {k p^n} {p_n}$ +We have from [[Binomial Coefficient involving Power of Prime]] that: +:$\dbinom {k p^n} {p_n} \equiv k \pmod p$ +But $p$ does not [[Definition:Divisor of Integer|divide]] $k$. +Hence $p$ does not [[Definition:Divisor of Integer|divide]] $\card {\Bbb S}$. +Hence by $(1)$ and $(4)$: +:$p \nmid k \paren {p^{d_1} + p^{d_2} + \cdots p^{d_r} }$ +If $d_i > 0$ for all $i = 1, 2, \ldots, r$, then $p$ would [[Definition:Divisor of Integer|divide]] this [[Definition:Integer|integer]]. +So there must be some $i$ such that $d_i = 0$. +Then, for this $i$: +:$n_i = n$ +That is: +:$\card {\Stab {S_i} } = p^n$ +Thus $\Stab {S_i}$ is the [[Definition:Subgroup|subgroup]] of $G$ of [[Definition:Order of Group|order]] $p_n$ that was asserted. +{{qed}} +\end{proof}<|endoftext|> +\section{H-Cobordism Theorem} +Tags: Topology + +\begin{theorem} +Let $X^n, Y^n$ be two [[Definition:Simply Connected|simply connected]] manifolds. +Let $n \in \N: n \ge 5$ and $\exists W$ such that $W$ is an [[Definition:Cobordism|h-cobordism]] between $X$ and $Y$. +Then $\exists \psi: W \to X \times \closedint 0 1$ such that $\psi$ is a [[Definition:Diffeomorphism|diffeomorphism]]. +In particular, $X$ and $Y$ are [[Definition:Diffeomorphism|diffeomorphic]]. +\end{theorem} + +\begin{proof} +{{ProofWanted|In progress}} +[[Category:Topology]] +2guptkt0e7yhebs6rugk4vfd5rl5e1l +\end{proof}<|endoftext|> +\section{Group has Subgroups of All Prime Power Factors} +Tags: P-Groups, Subgroups + +\begin{theorem} +Let $p$ be a [[Definition:Prime Number|prime]]. +Let $G$ be a [[Definition:Finite Group|finite group]] of [[Definition:Order of Structure|order]] $n$. +If $p^k \divides n$ then $G$ has at least one [[Definition:Subgroup|subgroup]] of [[Definition:Order of Structure|order]] $p^k$. +\end{theorem} + +\begin{proof} +From [[Composition Series of Group of Prime Power Order]], a [[Definition:P-Group|$p$-group]] has [[Definition:Subgroup|subgroups]] corresponding to every [[Definition:Divisor of Integer|divisor]] of its [[Definition:Order of Structure|order]]. +Thus, taken with the [[First Sylow Theorem]], a [[Definition:Finite Group|finite group]] has a [[Definition:Subgroup|subgroup]] corresponding to every [[Definition:Prime Power|prime power]] [[Definition:Divisor of Integer|divisor]] of its [[Definition:Order of Structure|order]]. +{{qed}} +\end{proof}<|endoftext|> +\section{Normalizer of Sylow p-Subgroup} +Tags: P-Groups, Subgroups, Normalizers, Sylow p-Subgroups + +\begin{theorem} +Let $P$ be a [[Definition:Sylow p-Subgroup|Sylow $p$-subgroup]] of a [[Definition:Finite Group|finite group]] $G$. +Let $\map {N_G} P$ be the [[Definition:Normalizer|normalizer]] of $P$. +Then any [[Definition:P-Subgroup|$p$-subgroup]] of $\map {N_G} P$ is contained in $P$. +In particular, $P$ is the unique [[Definition:Sylow p-Subgroup|Sylow $p$-subgroup]] of $\map {N_G} P$. +\end{theorem} + +\begin{proof} +Let $Q$ be a [[Definition:P-Subgroup|$p$-subgroup]] of $N = \map {N_G} P$. +Let $\order Q = p^m, \order P = p^n$. +By [[Normalizer of Subgroup is Largest Subgroup containing that Subgroup as Normal Subgroup]]: +:$P \lhd \map {N_G} P$ +thus by [[Subset Product with Normal Subgroup as Generator]]: +:$\gen {P, Q} = P Q$ +Thus by [[Order of Subgroup Product]]: +:$P Q \le G: \order {P Q} = p^{n + m - s}$ +where $\order {P \cap Q} = p^s$. +Since $n$ is the highest power of $p$ dividing $\order G$, this is possible only when $m \le s$. +Since $P \cap Q \le Q, s \le m$ thus we conclude that $m = s$ and therefore $P \cap Q = Q$. +Thus from [[Intersection with Subset is Subset]]: +:$Q \subseteq P$ +In particular, if $Q$ is a [[Definition:Sylow p-Subgroup|Sylow $p$-subgroup]] of $\map {N_G} P$, then $Q = P$. +{{qed}} +\end{proof}<|endoftext|> +\section{Second Sylow Theorem} +Tags: Sylow Theorems + +\begin{theorem} +Let $P$ be a [[Definition:Sylow p-Subgroup|Sylow $p$-subgroup]] of the [[Definition:Finite Group|finite group]] $G$. +Let $Q$ be any [[Definition:P-Subgroup|$p$-subgroup]] of $G$. +Then $Q$ is a [[Definition:Subset|subset]] of a [[Definition:Conjugate of Group Subset|conjugate]] of $P$. +\end{theorem} + +\begin{proof} +Let $P$ be a [[Definition:Sylow p-Subgroup|Sylow $p$-subgroup]] of $G$. +Let $\mathbb S$ be the [[Definition:Set|set]] of all [[Definition:Distinct|distinct]] [[Definition:Conjugate of Group Subset|$G$-conjugates]] of $P$: +:$\mathbb S = \set {g P g^{-1}: g \in G}$ +Let $h * S$ be the [[Definition:Conjugacy Action|conjugacy action]]: +:$\forall h \in P, S \in \mathbb S: h * S = h S h^{-1}$ +From [[Conjugacy Action on Subgroups is Group Action]], this is a [[Definition:Group Action|group action]] for $S \le G$. +To show it is [[Definition:Closed Algebraic Structure|closed]] for $S \in \mathbb S$: +{{begin-eqn}} +{{eqn | l = S + | o = \in + | r = \mathbb S + | c = +}} +{{eqn | ll= \leadsto + | l = \exists g \in G: S + | r = g P g^{-1} + | c = +}} +{{eqn | ll= \leadsto + | l = h * S + | r = h \paren {g P g^{-1} } h^{-1} + | c = +}} +{{eqn | r = \paren {h g} P \paren {h g}^{-1} + | c = +}} +{{eqn | ll= \leadsto + | l = h * S + | o = \in + | r = \mathbb S + | c = +}} +{{end-eqn}} +So, consider the [[Definition:Orbit (Group Theory)|orbits]] and [[Definition:Stabilizer|stabilizers]] of $\mathbb S$ under this [[Definition:Group Action|group action]]. +Since $\forall S \in \mathbb S: \Stab S \le P$, we have that: +:$\size {\Stab S} \divides \order P$ +Therefore, by the [[Orbit-Stabilizer Theorem]], these [[Definition:Length of Orbit|orbit lengths]] are all [[Definition:Congruence Modulo Integer|congruent]] to either $0$ or $1$ modulo $p$, since $P$ is a [[Definition:Sylow p-Subgroup|Sylow $p$-subgroup]] of $G$. +Note that this will imply, as we shall mark later on: +:$\size {\mathbb S} \equiv 1 \pmod p$ +Now, $h * P = h P h^{-1} = P$, so: +:$\Orb P = \set P$ +We now show that $P$ is the only [[Definition:Element|element]] of $\mathbb S$ such that $\size {\Orb S} = 1$. +If $g P g^{-1}$ has one [[Definition:Element|element]] in its [[Definition:Orbit (Group Theory)|orbit]], then: +:$\forall x \in P: x \paren {g P g^{-1} } x^{-1} = g P g^{-1}$ +Thus $\forall x \in P$ we have that: +:$g^{-1} x g \in \map {N_G} P$ +From [[Order of Conjugate Element equals Order of Element]], we have that: +:$\order {g^{-1} x g} = \order x$ +Thus $P_1 = g^{-1} P g$ is a [[Definition:P-Subgroup|$p$-subgroup]] of $\map {N_G} P$. +As $P$ and $P_1$ have the same number of [[Definition:Element|element]]s, $P_1$ is a [[Definition:Sylow p-Subgroup|Sylow $p$-subgroup]] of $\map {N_G} P$. +Hence $P_1 = P$ by [[Normalizer of Sylow p-Subgroup]], so $g P g^{-1} = P$. +Thus $P$ is the only [[Definition:Element|element]] of $\mathbb S$ whose [[Definition:Orbit (Group Theory)|orbit]] has [[Definition:Length of Orbit|length]] $1$. +From [[Stabilizer of Coset Action on Set of Subgroups]], $P = \map {N_G} P$. +Thus, for any $g \notin P$, $\size {\Orb {g P g^{-1} } }$ under conjugation by [[Definition:Element|element]]s of $P$ has [[Definition:Orbit (Group Theory)|orbit]] greater than $1$. +Hence: +: $\size {\mathbb S} \equiv 1 \pmod p$ +as promised. +Next we consider [[Definition:Orbit (Group Theory)|orbit]]s of $\mathbb S$ under conjugation by [[Definition:Element|element]]s of $Q$. +Since every [[Definition:Orbit (Group Theory)|orbit]] has [[Definition:Length of Orbit|length]] a power of $p$, the above conclusion shows there is at least one [[Definition:Orbit (Group Theory)|orbit]] of [[Definition:Length of Orbit|length]] $1$. +So there is an [[Definition:Element|element]] $g$ such that: +:$\forall x \in Q: x \paren {g P g^{-1} } x^{-1} = g P g^{-1}$ +As previously: +:$g^{-1} Q g \subseteq \map {N_G} P$ +So by [[Normalizer of Sylow p-Subgroup]]: +:$g^{-1} Q g \subseteq P$ +Thus $Q \subseteq g P g^{-1}$ as required. +{{qed}} +\end{proof}<|endoftext|> +\section{Third Sylow Theorem} +Tags: Sylow Theorems, Third Sylow Theorem + +\begin{theorem} +All the [[Definition:Sylow p-Subgroup|Sylow $p$-subgroups]] of a [[Definition:Finite Group|finite group]] are [[Definition:Conjugate of Group Subset|conjugate]]. +\end{theorem} + +\begin{proof} +Suppose $P$ and $Q$ are [[Definition:Sylow p-Subgroup|Sylow $p$-subgroups]] of $G$. +By the [[Second Sylow Theorem]], $Q$ is a [[Definition:Subset|subset]] of a [[Definition:Conjugate of Group Subset|conjugate]] of $P$. +But since $\order P = \order Q$, it follows that $Q$ must equal a [[Definition:Conjugate of Group Subset|conjugate]] of $P$. +{{qed}} +\end{proof} + +\begin{proof} +Let $G$ be a [[Definition:Finite Group|finite group]] of [[Definition:Order of Group|order]] $p^n m$, where $p \nmid m$ and $n > 0$. +Let $H$ be a [[Definition:Sylow p-Subgroup|Sylow $p$-subgroup]] of $G$. +We have that: +:$\order H = p^n$ +:$\index G H = m$ +Let $S_1, S_2, \ldots, S_m$ denote the [[Definition:Left Coset|left cosets]] of $G \pmod H$. +We have that $G$ [[Definition:Group Action|acts on]] $G / H$ by the rule: +:$g * S_i = g S_i$. +Let $H_i$ denote the [[Definition:Stabilizer|stabilizer]] of $S_i$. +By the [[Orbit-Stabilizer Theorem]]: +:$\order {H_i} = p^n$ +while: +:$S_i = g H \implies g H g^{-1} \subseteq H_i$ +Because $\order {g H g^{-1} } = \order H = \order {H_i}$, we have: +:$g H g^{-1} \subseteq H_i$ +Let $H'$ be a second [[Definition:Sylow p-Subgroup|Sylow $p$-subgroup]] of $G$. +Then $H'$ acts on $G / H$ by the same rule as $G$. +Since $p \nmid m$, there exists at least one [[Definition:Orbit (Group Theory)|orbit]] under $H'$ whose [[Definition:Cardinality|cardinality]] is not [[Definition:Divisor of Integer|divisible]] by $p$. +Suppose that $S_1, S_2, \ldots, S_r$ are the [[Definition:Element|elements]] of an [[Definition:Orbit (Group Theory)|orbit]] where $p \nmid r$. +Let $K = H' \cap H_1$. +Then $K$ is the [[Definition:Stabilizer|stabilizer]] of $S_1$ under the action of $H'$. +Therefore: +:$\index {H'} K = r$ +However: +:$\order {H'} = p^n$ +and: +:$p \nmid r$ +from which it follows that: +:$r = 1$ +and: +:$K = H'$ +Therefore: +:$\order K = \order {H'} = \order {H_1} = p^n$ +and: +:$H' = K = H_1$ +Thus $H'$ and $H$ are [[Definition:Conjugate of Group Subset|conjugates]]. +\end{proof}<|endoftext|> +\section{Fourth Sylow Theorem} +Tags: Sylow Theorems, Fourth Sylow Theorem + +\begin{theorem} +The number of [[Definition:Sylow p-Subgroup|Sylow $p$-subgroups]] of a [[Definition:Finite Group|finite group]] is [[Definition:Congruence (Number Theory)|congruent]] to $1 \pmod p$. +\end{theorem} + +\begin{proof} +Let $G$ be a [[Definition:Finite Group|finite group]] of [[Definition:Order of Group|order]] $p^n m$, where $p \nmid m$ and $n > 0$. +Let $r$ be the number of [[Definition:Sylow p-Subgroup|Sylow $p$-subgroups]] of $G$. +Let $H$ be a [[Definition:Sylow p-Subgroup|Sylow $p$-subgroup]] of $G$. +We have that: +:$\order H = p^n$ +:$\index G H = m$ +Let $S_1, S_2, \ldots, S_m$ denote the [[Definition:Element|elements]] of the [[Definition:Left Coset Space|left coset space]] of $G / H$. +We have that $H$ [[Definition:Group Action|acts on]] $G / H$ by the rule: +:$g * S_i = g S_i$ +for $S_i \in G / H$. +Unless $H = G$ and $r = 1$, there is more than $1$ [[Definition:Orbit (Group Theory)|orbit]]. +{{explain|Why? See [[Orbits of Group Action on Sets with Power of Prime Size]]}} +We have that $H$ is the [[Definition:Stabilizer|stabilizer]] of the [[Definition:Left Coset|coset]] $H$, which must be one of $S_1, S_2, \ldots, S_m$. +Let $S_1, S_2, \ldots, S_k$ be the [[Definition:Element|elements]] of $G / H$ whose [[Definition:Stabilizer|stabilizer]] is $H$. +From the [[Orbit-Stabilizer Theorem]] and from $\order H = p^n$ we see there are $2$ cases: +:$(1): \quad$ The [[Definition:Orbit (Group Theory)|orbit]] of $S_i$ contains $p^t$ [[Definition:Element|elements]] where $0 < t < n$ +:$(2): \quad$ The [[Definition:Orbit (Group Theory)|orbit]] of $S_i$ contains only the [[Definition:Element|element]] $S_i$. +$(2)$ occurs {{iff}} $S_i$ is one of the [[Definition:Left Coset|cosets]] $S_1, S_2, \ldots, S_k$ whose [[Definition:Stabilizer|stabilizer]] is $H$. +So counting the [[Definition:Element|elements]] of $G / H$, we see that: +:$m = k + u p$ +or: +:$m \equiv k \pmod p$ +From the [[Fifth Sylow Theorem]], we have: +:$m \equiv k r \pmod p$ +and so: +:$k r \equiv k \pmod p$ +from which it follows: +:$r \equiv 1 \pmod p$ +because $k \not \equiv 0 \pmod p$. +Hence the result. +{{qed}} +\end{proof}<|endoftext|> +\section{Sylow p-Subgroup is Unique iff Normal} +Tags: Normal Subgroups, Sylow p-Subgroups + +\begin{theorem} +A [[Definition:Group|group]] $G$ has exactly one [[Definition:Sylow p-Subgroup|Sylow $p$-subgroup]] $P$ {{iff}} $P$ is [[Definition:Normal Subgroup|normal]]. +\end{theorem} + +\begin{proof} +If $G$ has precisely one [[Definition:Sylow p-Subgroup|Sylow $p$-subgroup]], it must be [[Definition:Normal Subgroup|normal]] from [[Unique Subgroup of a Given Order is Normal]]. +Suppose a [[Definition:Sylow p-Subgroup|Sylow $p$-subgroup]] $P$ is [[Definition:Normal Subgroup|normal]]. +Then it equals its [[Definition:Conjugate of Group Subset|conjugates]]. +Thus, by the [[Third Sylow Theorem]], there can be only one such [[Definition:Sylow p-Subgroup|Sylow $p$-subgroup]]. +{{qed}} +\end{proof}<|endoftext|> +\section{Intersection of Normal Subgroup with Sylow P-Subgroup} +Tags: Normal Subgroups, Set Intersection, Sylow p-Subgroups + +\begin{theorem} +Let $P$ be a [[Definition:Sylow p-Subgroup|Sylow $p$-subgroup]] of a [[Definition:Finite Group|finite group]] $G$. +Let $N$ be a [[Definition:Normal Subgroup|normal subgroup]] of $G$. +Then $P \cap N$ is a [[Definition:Sylow p-Subgroup|Sylow $p$-subgroup]] of $N$. +\end{theorem} + +\begin{proof} +Since $N \lhd G$, we see that: +:$\gen {P, N} = P N$ +from [[Subset Product with Normal Subgroup as Generator]]. +Since $P \cap N \le P$, it follows that: +:$\order {P \cap N} = p^k$ +where $k > 0$. +By [[Order of Subgroup Product]]: +:$\order {P N} \order {P \cap N} = \order P \order N$ +Hence from [[Lagrange's Theorem (Group Theory)|Lagrange's Theorem]]: +:$\index N {P \cap N} = \index {P N} P$ +By [[Tower Law for Subgroups]]: +:$\index G P = \index G {P N} \index {P N} P$ +Thus: +:$\index {P N} P \divides \index G P$ +where $\divides$ denotes [[Definition:Divisor of Integer|divisibility]]. +Thus: +:$p \nmid \index {P N} P$ +so: +:$p \nmid \index N {P \cap N}$ +Thus $P \cap N$ is a [[Definition:Sylow p-Subgroup|Sylow $p$-subgroup]] of $N$. +{{qed}} +\end{proof}<|endoftext|> +\section{Quotient of Sylow P-Subgroup} +Tags: Quotient Groups, Sylow p-Subgroups + +\begin{theorem} +Let $P$ be a [[Definition:Sylow p-Subgroup|Sylow $p$-subgroup]] of a [[Definition:Finite Group|finite group]] $G$. +Let $N$ be a [[Definition:Normal Subgroup|normal subgroup]] of $G$. +Then $P N / N$ is a [[Definition:Sylow p-Subgroup|Sylow $p$-subgroup]] of $G / N$. +\end{theorem} + +\begin{proof} +We have that $P \le G$ and $ N \lhd G$. +So by the [[Second Isomorphism Theorem for Groups]]: +:$P N / N \cong P / \paren {P \cap N}$ +We have that: +:$P N / N = \set {p N : p \in P}$ +and so every [[Definition:Element|element]] of $P N / N$ has [[Definition:Order of Group Element|order]] a [[Definition:Power of Group Element|power]] of $p$. +Hence $P N / N$ is a [[Definition:P-Subgroup|$p$-subgroup]] of $G / N$. +From [[Intersection of Normal Subgroup with Sylow P-Subgroup]], we have that: +:$p \nmid \index G {P N}$ +So $P N / N$ is a [[Definition:Sylow p-Subgroup|Sylow $p$-subgroup]] of $G / N$. +{{qed}} +\end{proof}<|endoftext|> +\section{Fifth Sylow Theorem} +Tags: Sylow Theorems, Fifth Sylow Theorem + +\begin{theorem} +The number of [[Definition:Sylow p-Subgroup|Sylow $p$-subgroups]] of a [[Definition:Finite Group|finite group]] is a [[Definition:Divisor of Integer|divisor]] of their common [[Definition:Index of Subgroup|index]]. +\end{theorem} + +\begin{proof} +Let $G$ be a [[Definition:Finite Group|finite group]] of [[Definition:Order of Group|order]] $p^n m$, where $p \nmid m$ and $n > 0$. +Let $r$ be the number of [[Definition:Sylow p-Subgroup|Sylow $p$-subgroups]] of $G$. +It is to be shown that $r \divides m$. +Let $H$ be a [[Definition:Sylow p-Subgroup|Sylow $p$-subgroup]] of $G$. +We have that: +:$\order H = p^n$ +:$\index G H = m$ +Let $S_1, S_2, \ldots, S_m$ denote the [[Definition:Element|elements]] of the [[Definition:Left Coset Space|left coset space]] of $G / H$. +We have that $G$ [[Definition:Group Action|acts on]] $G / H$ by the rule: +:$g * S_i = g S_i$ +for $S_i \in G / H$. +There is only one [[Definition:Orbit (Group Theory)|orbit]] under this action, namely the whole of $G / H$. +{{explain|Why? See [[Orbits of Group Action on Sets with Power of Prime Size]]}} +Therefore the [[Definition:Stabilizer|stabilizer]] of each $S_i$ is a [[Definition:Subgroup|subgroup]] of $G$ of [[Definition:Index of Subgroup|index]] $m$ and [[Definition:Order of Group|order]] $p^n$. +In other words, each $S_i$ has a [[Definition:Sylow p-Subgroup|Sylow $p$-subgroup]] as a [[Definition:Stabilizer|stabilizer]]. +Now it is shown that each [[Definition:Sylow p-Subgroup|Sylow $p$-subgroup]] is the [[Definition:Stabilizer|stabilizer]] of one or more of the [[Definition:Left Coset|cosets]] $S_1, S_2, \ldots, S_m$. +We have that $H$ is the [[Definition:Stabilizer|stabilizer]] of the [[Definition:Left Coset|coset]] $H$, which must be one of $S_1, S_2, \ldots, S_m$. +Let $S_1, S_2, \ldots, S_k$ be the [[Definition:Element|elements]] of $G / H$ whose [[Definition:Stabilizer|stabilizer]] is $H$. +By the [[Third Sylow Theorem]], any other [[Definition:Sylow p-Subgroup|Sylow $p$-subgroup]] of $H$ is a [[Definition:Conjugate of Group Subset|conjugates]] $g H g^{-1}$ of $H$. +Thus it is seen that $g H g^{-1}$ is a [[Definition:Stabilizer|stabilizer]] of the [[Definition:Left Coset|cosets]] $g S_1, g S_2, \ldots, g S_k$. +So each of the $r$ distinct [[Definition:Sylow p-Subgroup|Sylow $p$-subgroups]] of $G$ is the [[Definition:Stabilizer|stabilizer]] of exactly $k$ [[Definition:Element|elements]] of $G / H$. +Thus: +:$m = k r$ +and so: +:$r \divides m$ +as required. +{{qed}} +\end{proof}<|endoftext|> +\section{Finite Submodule of Function Space} +Tags: Submodules, Function Spaces + +\begin{theorem} +Let $\left({G, +}\right)$ be a [[Definition:Group|group]] whose [[Definition:Identity Element|identity]] is $e$. +Let $R$ be a [[Definition:Ring (Abstract Algebra)|ring]]. +Let $\left({G, +, \circ}\right)_R$ be an [[Definition:Module|$R$-module]]. +Let $S$ be a set. +Let $G^S$ the set of all [[Definition:Mapping|mappings]] $f: S \to G$. +Let $G^{\left({S}\right)}$ be the set of all [[Definition:Mapping|mappings]] $f: S \to G$ such that $f \left({x}\right) = e$ for all but [[Definition:Finite|finitely]] many elements $x$ of $S$. +Then: +: $\left({G^{\left({S}\right)}, +', \circ}\right)_R$ is a [[Definition:Submodule|submodule]] of $\left({G^S, +, \circ}\right)_R$ +where $+'$ is the [[Definition:Induced Structure|operation induced]] on $G^{\left({S}\right)}$ by $+$. +\end{theorem} + +\begin{proof} +Let $\left({G, +, \circ}\right)_R$ be an [[Definition:Module|$R$-module]] and $S$ be a [[Definition:Set|set]]. +We need to show that $\left({G^{\left({S}\right)}, +'}\right)$ is a [[Definition:Group|group]]. +Let $f, g \in G^{\left({S}\right)}$. +Let: +: $F = \left\{{x \in S: f \left({x}\right) \ne e}\right\}$ +: $G = \left\{{x \in S: g \left({x}\right) \ne e}\right\}$ +From the definition of $f$ and $g$, both $F$ and $G$ are [[Definition:Finite|finite]]. +Since $e + e = e$ by definition of [[Definition:Identity Element|identity element]], it follows that if: +: $f \left({x}\right) + g \left({x}\right) \ne e$ +then necessarily $f \left({x}\right) \ne e$ or $g \left({x}\right) \ne e$. +That is: +: $\left({f +' g}\right) \left({x}\right) = f \left({x}\right) + g \left({x}\right) \ne e \implies x \in F \cup G$ +But as $F$ and $G$ are both [[Definition:Finite|finite]], it follows that $F \cup G$ is also [[Definition:Finite|finite]]. +Hence $f +' g \in G^{\left({S}\right)}$ and $\left({G^{\left({S}\right)}, +'}\right)$ is [[Definition:Closed Algebraic Structure|closed]]. +Now let $f \in G^{\left({S}\right)}$. +Let $f^*$ be the [[Induced Structure Inverse]] of $f$. +Thus $f^* \left({x}\right) = - \left({f \left({x}\right)}\right)$. +Again, let $F = \left\{{x \in S: f \left({x}\right) \ne e}\right\}$. +It follows directly that $x \in S \setminus F \implies f^* \left({x}\right) = e$. +Hence $f^* \left({x}\right) \ne e \implies x \in F$ and hence $f^* \in G^{\left({S}\right)}$. +So by the [[Two-Step Subgroup Test]], it follows that $\left({G^{\left({S}\right)}, +'}\right)$ is a [[Definition:Subgroup|subgroup]] of $\left({G^S, +}\right)$. +Hence $\left({G^{\left({S}\right)}, +', \circ}\right)_R$ is an [[Definition:Module|$R$-module]]. +The result follows. +{{qed}} +\end{proof}<|endoftext|> +\section{Polynomial Functions form Submodule of All Functions} +Tags: Module Theory, Polynomial Theory + +\begin{theorem} +Let $K$ be a [[Definition:Commutative and Unitary Ring|commutative ring with unity]]. +Let $K^K$ be the [[Definition:Module of All Mappings|$K$-module mappings]] $f: K \to K$. +Let $P \left({K}\right) \subseteq K^K$ be the [[Definition:Set|set]] of all [[Definition:Polynomial Function (Abstract Algebra)|polynomial functions]] on $K$. +Then $P \left({K}\right)$ is a $K$-[[Definition:Submodule|submodule]] of $K^K$. +\end{theorem}<|endoftext|> +\section{Intersection and Sum of Submodules} +Tags: Submodules + +\begin{theorem} +Let $\left({G, +, \circ}\right)_R$ be an [[Definition:Module|$R$-module]]. +Let $H$ and $K$ be [[Definition:Submodule|submodules]] of $G$. +Then $H + K$ and $H \cap K$ are also [[Definition:Submodule|submodules]] of $G$. +The [[Definition:Set Intersection|intersection]] of any [[Definition:Set|set]] of submodules of $G$ is a [[Definition:Submodule|submodule]]. +Thus if $S \subseteq G$, the [[Definition:Set Intersection|intersection]] of all submodules of $G$ containing $S$ is the smallest submodule of $G$ containing $S$. +\end{theorem}<|endoftext|> +\section{Linear Transformation of Submodule} +Tags: Linear Transformations + +\begin{theorem} +Let $G$ and $H$ be [[Definition:Module|$R$-modules]]. +Let $\phi: G \to H$ be a [[Definition:Linear Transformation|linear transformation]]. +Then: +:$(1): \quad$ If $M$ is a [[Definition:Submodule|submodule]] of $G$, $\phi \sqbrk M$ is a [[Definition:Submodule|submodule]] of $H$ +:$(2): \quad$ If $N$ is a [[Definition:Submodule|submodule]] of $H$, $\phi^{-1} \sqbrk N$ is a [[Definition:Submodule|submodule]] of $G$ +:$(3): \quad$ The [[Definition:Codomain of Mapping|codomain]] of $\phi$ is a [[Definition:Submodule|submodule]] of $H$ +:$(4): \quad$ The [[Definition:Kernel of Linear Transformation|kernel]] of $\phi$ is a [[Definition:Submodule|submodule]] of $G$. +\end{theorem} + +\begin{proof} +Since a [[Definition:Linear Transformation|linear transformation]] $\phi: G \to H$ is, in particular, a [[Definition:Group Homomorphism|homomorphism]] from the [[Definition:Group|group]] $G$ to the [[Definition:Group|group]] $H$, it follows that: +:$(1): \quad$ By [[Homomorphism with Cancellable Codomain Preserves Identity]], $\map \phi {e_G} = e_H$ +:$(2): \quad$ By [[Homomorphism with Identity Preserves Inverses]], $\map \phi {-x} = -\map \phi x$. +From [[Epimorphism preserves Modules]] and definition of [[Definition:Surjection|surjection]], it follows that as $M$ is a [[Definition:Submodule|submodule]] of $G$, then $\phi \sqbrk M$ is a [[Definition:Submodule|submodule]] of $H$. +The result follows ... +{{stub}} +\end{proof}<|endoftext|> +\section{Elements of Module with Equal Images under Linear Transformations form Submodule} +Tags: Linear Transformations + +\begin{theorem} +Let $G$ and $H$ be [[Definition:Module|$R$-modules]]. +Let $\phi$ and $\psi$ be [[Definition:Linear Transformation|linear transformations]] from $G$ into $H$. +Then the set $S = \set {x \in G: \map \phi x = \map \psi x}$ is a [[Definition:Submodule|submodule]] of $G$. +\end{theorem} + +\begin{proof} +Let $x, y \in S$. +Let $\lambda \in R$. +Then: +{{begin-eqn}} +{{eqn | l = \map \phi {x + y} + | r = \map \phi x + \map \phi y + | c = {{Defof|Linear Transformation}} +}} +{{eqn | r = \map \psi x + \map \psi y + | c = $x, y \in S$ +}} +{{eqn | r = \map \psi {x + y} + | c = {{Defof|Linear Transformation}} +}} +{{eqn | l = \map \phi {\lambda \circ x} + | r = \lambda \circ \map \phi x + | c = {{Defof|Linear Transformation}} +}} +{{eqn | r = \lambda \circ \map \psi x + | c = $x \in S$ +}} +{{eqn | r = \map \psi {\lambda \circ x} + | c = {{Defof|Linear Transformation}} +}} +{{end-eqn}} +Hence $x + y, \lambda \circ x \in S$. +By [[Submodule Test]], $S$ is a [[Definition:Submodule|submodule]] of $G$. +{{qed}} +\end{proof}<|endoftext|> +\section{Poincaré Conjecture} +Tags: Topology, Millennium Problems + +\begin{theorem} +Let $\Sigma^m$ be a smooth $m$-manifold. +Let $\Sigma^m$ satisfy: +:$H_0 \struct {\Sigma; \Z} = 0$ +and: +:$H_m \struct {\Sigma; \Z} = \Z$ +{{explain|Definition of the notation $H_0 \struct {\Sigma; \Z}$, nature of $H_0$ and $H_m$}} +Then $\Sigma^m$ is homeomorphic to the $m$-sphere $\Bbb S^m$. +\end{theorem} + +\begin{proof} +The proof proceeds on several dimensional-cases. Note that the case $m = 3$ is incredibly intricate, and that a full proof would be impractical to produce here. An outline of the $m = 3$ case will be given instead. +=== Dimension $m = 1$ === +Follows from the [[Classification of Compact One-Manifolds]]. +{{qed|lemma}} +=== Dimension $m = 2$ === +Follows from the [[Classification of Compact Two-Manifolds]]. +{{qed|lemma}} +=== Dimension $m = 3$ === +Follows from [[Thurston's Geometrization Conjecture]], proved by {{AuthorRef|Grigori Perelman}}. +{{ProofWanted}} +{{qed|lemma}} +=== Dimension $m = 4$ === +Follows from $4$-dimensional [[Topological h-Cobordism Theorem]] of {{AuthorRef|Andrew John Casson}} and {{AuthorRef|Michael Hartley Freedman}}. +{{ProofWanted}} +{{qed|lemma}} +=== Dimension $m = 5$ === +{{ProofWanted}} +Summary: +Any $\Sigma^5$ bounds a contractible $6$-manifold $Z$. +Let $\Bbb D^6$ be a $6$-disk (AKA 6-ball). +{{explain|Open or closed disk / ball?}} +Then $Z - \Bbb D^6$ is an $h$-cobordism between $\Sigma$ and $\partial \Bbb D^6 = \Bbb S^5$. +Hence $\Sigma$ is differomorphic to $\Bbb S^5$ by the [[h-Cobordism Theorem]]. +{{qed|lemma}} +=== Dimension $m \ge 6$ === +{{ProofWanted}} +We can cut two small $m$-disks $D', D''$ from $\Sigma$. +The remaining manifold, $\Sigma \setminus \paren {D' \cup D''}$ is an h-cobordism between $\partial D'$ and $\partial D''$. +These are just two copies of $\Bbb S^{m-1}$. +By the $h$-cobordism theorem, there exists a diffeomorphism: +:$\phi: \Sigma \setminus \paren {D' \cup D''} \to \Bbb S^{m - 1} \times \closedint 0 1$ +which can be chosen to restrict to the identity on one of the $\Bbb S^{m - 1}$. +Let $\Xi$ denote this $\Bbb S^{m - 1}$ such that $\phi$ restricts to the identity. +Since $\psi \vert_\Xi = Id$, we can extend $\psi$ across $D''$, the interior of $\Xi$ to obtain a diffeomorphism $\phi': \Sigma \setminus D'' \to \Bbb S^{m - 1} \cup D'$. +{{explain|What does $\psi \vert_\Xi$ mean?}} +Let $\Bbb D^m$ denote this latter manifold, which is merely an $m$-disk. +Our diffeomorphism $\phi': \Sigma \setminus D'' \to \Bbb D^m$ induces a diffeomorphism on the boundary spheres $\Bbb S^{m - 1}$. +Any diffeomorphism of the boundary sphere $\Bbb S^{m - 1}$ can be extended radially to the whole disk: +:$\map {\operatorname {int} } {\Bbb S^{m - 1} } = D''$ +but only as a homeomorphism of D''. +Hence the extended function $\phi'': \Sigma \to \Bbb S^m$ is a homeomorphism. +{{qed}} +{{Namedfor|Jules Henri Poincaré|cat = Poincaré}} +\end{proof}<|endoftext|> +\section{Linear Transformation of Generated Module} +Tags: Linear Transformations + +\begin{theorem} +Let $G$ and $H$ be [[Definition:Module|$R$-modules]]. +Let $\phi$ and $\psi$ be [[Definition:Linear Transformation|linear transformations]] $G$ into $H$. +Let $S$ be a [[Definition:Generator of Module|generator]] for $G$. +Suppose that $\forall x \in S: \map \phi x = \map \psi x$. +Then $\phi = \psi$. +\end{theorem}<|endoftext|> +\section{Generated Submodule is Linear Combinations} +Tags: Unitary Modules, Linear Algebra + +\begin{theorem} +Let $G$ be a [[Definition:Unitary Module|unitary $R$-module]]. +Let $S \subseteq G$. +Then the [[Definition:Generator of Module|submodule $H$ generated by $S$]] is the [[Definition:Set|set]] of all [[Definition:Linear Combination|linear combinations]] of $S$. +\end{theorem} + +\begin{proof} +First the extreme case: +The smallest submodule of $G$ containing $\O$ is $\set {e_G}$. +[[Definition:Linear Combination of Empty Set|By definition]], $\set {e_G}$ is the set of all linear combinations of $\varnothing$. +Now the general case: +Let $\O \subset S \subseteq G$. +Let $L$ be the set of all [[Definition:Linear Combination of Subset|linear combinations]] of $S$. +Since $G$ is a [[Definition:Unitary Module|unitary $R$-module]], every element $x \in S$ is the linear combination $1_R x$, so $S \subseteq L$. +But $L$ is closed for addition and scalar multiplication, so [[Submodule Test|is a submodule]]. +Thus $H \subseteq L$. +But as every linear combination of $S$ clearly belongs to any submodule of $G$ which contains $S$, we also have $L \subseteq H$. +\end{proof}<|endoftext|> +\section{Empty Set is Linearly Independent} +Tags: Empty Set, Linear Algebra + +\begin{theorem} +The [[Definition:Empty Set|empty set]] is a [[Definition:Linearly Independent Set|linearly independent set]]. +\end{theorem} + +\begin{proof} +There are no [[Definition:Sequence|sequences]] at all of $n$ terms of the [[Definition:Empty Set|empty set]] for any $n > 0$. +Hence the result holds [[Definition:Vacuous Truth|vacuously]]. +{{qed}} +\end{proof}<|endoftext|> +\section{Subset of Module Containing Identity is Linearly Dependent} +Tags: Module Theory, Linear Algebra + +\begin{theorem} +Let $G$ be a [[Definition:Group|group]] whose [[Definition:Identity Element|identity]] is $e$. +Let $\struct {R, +, \circ}$ be a [[Definition:Ring (Abstract Algebra)|ring]] whose [[Definition:Ring Zero|zero]] is $0_R$. +Let $\struct {G, +_G, \circ}_R$ be an [[Definition:Module|$R$-module]]. +Let $H \subseteq G$ such that $e \in H$. +Then $H$ is a [[Definition:Linearly Dependent Set|linearly dependent set]]. +\end{theorem} + +\begin{proof} +From [[Scalar Product with Identity]], $\forall \lambda: \lambda \circ e = e$. +Let $H \subseteq G$ such that $e \in H$. +Consider any [[Definition:Sequence|sequence]] $\sequence {a_k}_{1 \mathop \le k \mathop \le n}$ in $H$ which includes $e$. +So, let $a_j = e$ for some $j \in \closedint 1 n$. +Let $c \in R \ne 0_R$. +Consider the [[Definition:Sequence|sequence]] $\sequence {\lambda_k}_{1 \mathop \le k \mathop \le n}$ of [[Definition:Element|elements]] of $R$ defined as: +:$\lambda_k = \begin{cases} +c & : k \ne j \\ +0_R & : k= j +\end{cases}$ +Then: +{{begin-eqn}} +{{eqn | l = \sum_{k \mathop = 1}^n \lambda_k \circ a_k + | r = \lambda_1 \circ a_1 + \lambda_2 \circ a_2 + \cdots + \lambda_j \circ a_j + \cdots + \lambda_n \circ a_n + | c = +}} +{{eqn | r = 0_R \circ a_1 + 0_R \circ a_2 + \cdots + c \circ e + \cdots + 0_R \circ a_n + | c = +}} +{{eqn | r = e + e + \cdots + e + \cdots + e + | c = +}} +{{eqn | r = e + | c = +}} +{{end-eqn}} +Thus there exists a [[Definition:Sequence|sequence]] $\sequence {\lambda_k}_{1 \mathop \le k \mathop \le n}$ in which not all $\lambda_k = 0_R$ such that: +:$\displaystyle \sum_{k \mathop = 1}^n \lambda_k \circ a_k = e$ +Hence the result. +{{qed}} +\end{proof}<|endoftext|> +\section{Subset of Linearly Independent Set is Linearly Independent} +Tags: Unitary Modules, Linear Algebra + +\begin{theorem} +A [[Definition:Subset|subset]] of a [[Definition:Linearly Independent Set|linearly independent set]] is also [[Definition:Linearly Independent Set|linearly independent]]. +\end{theorem} + +\begin{proof} +Let $G$ be an [[Definition:Unitary Module|unitary $R$-module]]. +Then $\sequence {a_n}$ is a [[Definition:Linearly Independent Sequence|linearly independent sequence]] {{iff}} $\set {a_1, a_2, \ldots, a_n}$ is a [[Definition:Linearly Independent Set|linearly independent set]] of $G$. +So suppose that $\set {a_1, a_2, \ldots, a_n}$ is a [[Definition:Linearly Independent Set|linearly independent set]] of $G$. +Then clearly $\sequence {a_n}$ is a [[Definition:Linearly Independent Sequence|linearly independent sequence]] of $G$. +Conversely, let $\sequence {a_n}$ be a [[Definition:Linearly Independent Sequence|linearly independent sequence]] of $G$. +Let $\sequence {b_m}$ be a [[Definition:Sequence of Distinct Terms|sequence of distinct terms]] of $\set {a_1, a_2, \ldots, a_n}$. +Let $\sequence {\mu_m}$ be a sequence of [[Definition:Scalar (Vector Space)|scalars]] such that $\displaystyle \sum_{j \mathop = 1}^m \mu_j b_j = 0$. +For each $k \in \closedint 1 n$, let: +:$\lambda_k = \begin{cases} +\mu_j & : j \text { is the unique index such that } a_k = b_j \\ +0 & : a_k \notin \set {b_1, b_2, \ldots, b_m} +\end{cases}$ +Then: +:$\displaystyle 0 = \sum_{j \mathop = 1}^m \mu_j b_j = \sum_{k \mathop = 1}^n \lambda_k a_k$ +Thus: +:$\forall k \in \closedint 1 n: \lambda_k = 0$ +As $\set {\mu_1, \ldots, \mu_m} \subseteq \set {\lambda_1, \ldots, \lambda_n}$, it follows that: +:$\forall j \in \closedint 1 m: \mu_j = 0$ +and so $\sequence {b_m}$ has been shown to be a [[Definition:Linearly Independent Sequence|linearly independent sequence]]. +Hence the result. +{{Qed}} +\end{proof}<|endoftext|> +\section{Superset of Linearly Dependent Set} +Tags: Unitary Modules, Linear Algebra + +\begin{theorem} +Any [[Definition:Set|set]] containing a [[Definition:Linearly Dependent Set|linearly dependent set]] is also [[Definition:Linearly Dependent Set|linearly dependent]]. +\end{theorem} + +\begin{proof} +Suppose $\left\{{a_1, a_2, \ldots, a_n}\right\}$ is a [[Definition:Linearly Independent Set|linearly independent set]]. +Then by [[Subset of Linearly Independent Set]], any [[Definition:Subset|subset]] $\left\{{b_1, b_2, \ldots, b_m}\right\}$ of $\left\{{a_1, a_2, \ldots, a_n}\right\}$ must itself be [[Definition:Linearly Independent Set|linearly independent]]. +Thus if $\left\{{b_1, b_2, \ldots, b_m}\right\}$ is [[Definition:Linearly Dependent Set|linearly dependent]], then so must $\left\{{a_1, a_2, \ldots, a_n}\right\}$ be. +{{qed}} +\end{proof}<|endoftext|> +\section{Number of Ordered Bases from a Basis} +Tags: Module Theory + +\begin{theorem} +Each [[Definition:Basis (Linear Algebra)|basis]] of $n$ elements determines $n!$ [[Definition:Ordered Basis|ordered bases]]. +\end{theorem} + +\begin{proof} +Follows directly from [[Cardinality of Set of Bijections]]. +{{qed}} +\end{proof}<|endoftext|> +\section{Classification of Compact Two-Manifolds} +Tags: Algebraic Topology + +\begin{theorem} +Any [[Definition:Smooth Manifold|smooth]], [[Definition:Compact Space|compact]], [[Definition:Path-Connected|path-connected]] [[Definition:Topological Manifold|manifold]] of [[Definition:Dimension (Topology)|dimension $2$]] is [[Definition:Diffeomorphism|diffeomorphic]] to [[Definition:Sphere (Topology)|the sphere $\mathbb S^2$]], a [[Definition:Connected Sum|connected sum]] of [[Definition:Torus (Topology)|tori $\mathbb T^2$]], or a [[Definition:Connected Sum|connected sum]] of [[Definition:Projective Space|projective spaces $\mathbb{RP}^2$]]. +Any such [[Definition:Topological Manifold|$2$-manifold]] with [[Definition:Boundary of Manifold|boundary]] is [[Definition:Diffeomorphism|diffeomorphic]] to [[Definition:Sphere (Topology)|the sphere $\mathbb S^2$]], a [[Definition:Connected Sum|connected sum]] of [[Definition:Torus (Topology)|tori $\mathbb T^2$]], or a [[Definition:Connected Sum|connected sum]] of [[Definition:Projective Space|projective spaces $\mathbb{RP}^2$]], with a number of [[Definition:Open Disk|open disks]] removed. +The [[Definition:Euler Characteristic|Euler characteristic]], [[Definition:Orientability|orientability]], and number of [[Definition:Boundary Curve|boundary curves]] suffice to describe a [[Definition:Topological Surface|surface]]. +\end{theorem} + +\begin{proof} +It is known that the [[Definition:Connected Sum|connected sum]] of $g$ [[Definition:Torus (Topology)|tori]], $\mathbb T_1^2 \# \mathbb T_2^2 \# \ldots \# \mathbb T_g^2$, which we denote $g \mathbb T^2$, is [[Definition:Orientable|orientable]] and has [[Definition:Euler Characteristic|Euler characteristic]] $2 - 2 g - b$, where $b$ is the number of [[Definition:Boundary Curve|boundary curves]]. +It is also known that the [[Definition:Connected Sum|connected sum]] of $p$ [[Definition:Projective Space|projective spaces]] $p \mathbb{RP}^2$ is [[Definition:Non-Orientable|non-orientable]] and has [[Definition:Euler Characteristic|Euler characteristic]] $2 - p - b$. +Thus the [[Definition:Euler Characteristic|Euler characteristic]], number of [[Definition:Boundary Curve|boundary curves]] and [[Definition:Orientability|orientability]] distinguish any closed, [[Definition:Path-Connected Space|path-connected]] [[Definition:Topological Manifold|$2$-manifold]]. +=== Lemma === +A [[Definition:Compact Space|compact]], [[Definition:Boundaryless Manifold|boundaryless]] [[Definition:Topological Manifold|$2$-manifold]] $S$ is [[Definition:Diffeomorphism|diffeomorphic]] to a [[Definition:Polyhedral Disk|polyhedral disk]] $P$ with [[Definition:Edge of Polyhedron|edges]] identified pairwise. +That is, for any [[Definition:Closed Set (Topology)|closed]], [[Definition:Connected Topological Space|connected]] [[Definition:Topological Manifold|$2$-manifold]], $\exists$ a [[Definition:Polyhedral Disk|polyhedral disk]] $P$ and an [[Definition:Equivalence Relation|equivalence relation]] $\sim$ such that $S \cong P \setminus \sim$. +=== Proof of Lemma === +{{ProofWanted}} +{{qed|lemma}} +With this lemma, the classification can be completed. +Throughout the proof, the term [[Definition:Topological Surface|surface]] is used in its [[Definition:Topology (Mathematical Branch)|topological]] meaning of [[Definition:Smooth Manifold|smooth]] [[Definition:Topological Manifold|$2$-manifold]]. +Consider the [[Definition:Polygonal Representation|polygonal representation]] guaranteed to exist by the lemma above. +Suppose there is only one pair of [[Definition:Edge of Polyhedron|edges]] on $P$. +Then they are either identified in an [[Definition:Orientable|orientable]] or [[Definition:Non-Orientable|non-orientable]] manner. +Thus they yield either $\mathbb S^2$ or $\mathbb{RP}^2$, respectively. +Now suppose there is more than one pair of [[Definition:Edge of Polyhedron|edges]] in $P$ identified by $\sim$. +If it can be shown that such a [[Definition:Topological Surface|surface]] can always be [[Definition:Decomposable Set|decomposed]] into the [[Definition:Connected Sum|connected sum]] of: +: either a $\mathbb T^2$ a $\mathbb{RP}^2$ +and: +: a [[Definition:Topological Surface|surface]] described by a [[Definition:Polyhedral Disk|polyhedral disk]] with fewer [[Definition:Edge of Polyhedron|edges]] than $P$ +the classification of [[Definition:Path-Connected|path-connected]] [[Definition:Topological Surface|surface]] will be complete. +There are $5$ cases to be examined: +=== Case 1 === +There are two adjacent [[Definition:Edge of Polyhedron|edges]] identified with opposite [[Definition:Orientation (Topology)|orientations]]. +Then the identification can be performed to obtain a new [[Definition:Polyhedral Disk|polyhedral disk]] with one less pair of [[Definition:Edge of Polyhedron|edges]]. +{{qed|lemma}} +=== Case 2 === +There are two adjacent [[Definition:Edge of Polyhedron|edges]] identified with the same [[Definition:Orientation (Topology)|orientations]]. +Then there is a [[Definition:Curve (Topology)|curve]] in $P$ from the unshared points of these two [[Definition:Edge of Polyhedron|edges]] which, under $\sim$, becomes a [[Definition:Simple Closed Curve|simple closed curve]] in $S$. +{{explain|Is "curve" in this context synonymous with [[Definition:Arc (Topology)]]?}} +The [[Definition:Polyhedral Disk|triangular disk]] $\Delta$ created by this [[Definition:Curve (Topology)|curve]] and the two [[Definition:Edge of Polyhedron|edges]] is just two [[Definition:Edge of Polyhedron|edges]] identified with the same [[Definition:Orientation (Topology)|orientations]], with a [[Definition:Boundary of Manifold|boundary]]. +Hence: +:$\Delta \cong \mathbb{RP}^2 - \mathbb D_\Delta^2$ +and therefore if we remove the two [[Definition:Edge of Polyhedron|edges]] in question from $P$, we construct a [[Definition:Polygonal Disk|polygonal disk]] $P - \Delta$ such that: +:$\left({\left({P - \Delta}\right) \setminus \sim}\right) \# \mathbb{RP}^2 \cong S$ +{{Explain|"polygonal disk" or "polyhedral disk"? Terminology is confused here.}} +{{qed|lemma}} +=== Case 3 === +There are two non-adjacent [[Definition:Edge of Polyhedron|edges]] in $P$, identified with the same [[Definition:Orientation (Topology)|orientations]]. +There exists a [[Definition:Curve (Topology)|curve]] $C$ in $P$ from the endpoint of one of these [[Definition:Edge of Polyhedron|edges]] to the identified point in the other. +A new [[Definition:Polygonal Disk|polygonal disk]], and a new [[Definition:Equivalence Relation|equivalence relation]], can be defined as follows: +Let $P'$ be the identification of the [[Definition:Edge of Polyhedron|edges]] in question and the separation of the disk along $C$. +Let $\sim'$ be all the equivalences of $\sim$ together with the new equivalence taking one of the copies of $C$ in $P'$ to the other, with appropriate [[Definition:Orientation (Topology)|orientation]]. +$P'$ and $\sim'$ are constructed so that $P' \setminus \sim' \cong S$. +But now $P'$ satisfies [[Classification of Compact Two-Manifolds#Case 2|Case 2]] because of the [[Definition:Orientation (Topology)|orientations]] imposed on the copies of $C$. +{{qed|lemma}} +=== Case 4 === +There are two non-adjacent edges in $P$, identified with opposite [[Definition:Orientation (Topology)|orientations]], such that neither edge is between any other pair of identified edges on the perimeter of $P$. +If we identify the two edges in question, a cylinder $Y$ is obtained. +There is a curve $C$ in $Y$ such that if $Y$ is separated along $C$, two cylinders are obtained. +If the separation is maintained, but the remaining identifications are performed, then two [[Definition:Topological Surface|surfaces]], each with [[Definition:Boundary of Manifold|boundary]] $C$, are obtained. +So the original [[Definition:Topological Surface|surface]] $S$ was the [[Definition:Connected Sum|connected sum]] of two [[Definition:Topological Surface|surfaces]], each with fewer edges in their respective polyhedral disks than $S$ had. +{{qed|lemma}} +=== Case 5 === +There are two non-adjacent edges identified with opposing [[Definition:Orientation (Topology)|orientations]], such that some other pair of identified edges are interlaced with the edges in question on the perimeter of $P$. +By the preceding cases, it is possible to decompose this [[Definition:Topological Surface|surface]] through the removal of all edges with the same [[Definition:Orientation (Topology)|orientation]], and forming a [[Definition:Connected Sum|connected sum]] with a number of projective planes. +Hence we regard $\sim$ as identifying any two edges with opposing [[Definition:Orientation (Topology)|orientations]] only. +Let the pairs of identified edges that are interlaced be $a_1, a_2, b_1, b_2$, such that $a_1 \sim a_2$ and $b_1 \sim b_2$. +By performing the identification on $a$, a cylinder is obtained. +By further performing the identification on $b$, the remaining edges of $P$ form the [[Definition:Boundary of Manifold|boundary]] of a disk on a [[Definition:Torus (Topology)|torus]] $\mathbb T^2$. +Hence the original [[Definition:Topological Surface|surface]] was the [[Definition:Connected Sum|connected sum]]: +:$\mathbb T^2 \# \left({P - a_1 - a_2 - b_1 - b_2}\right) \setminus \sim$ +{{qed|lemma}} +Any polyhedral disk with more than one pair of identified edges must satisfy at least one of the above $5$ cases. +So it can be decomposed into the [[Definition:Connected Sum|connected sum]] of either a projective plane or a [[Definition:Torus (Topology)|torus]] with a [[Definition:Topological Surface|surface]] described by a polyhedral disk with fewer pairs of identified edges. +Since the polyhedral disk with only one pair of identified edges is either a sphere or a projective plane, every [[Definition:Topological Surface|surface]] without [[Definition:Boundary of Manifold|boundary]] $S$ is either a sphere or the [[Definition:Connected Sum|connected sum]] of a collection of [[Definition:Torus (Topology)|tori]] and projective planes. +Since: +:$\mathbb T^2 \# \mathbb{RP}^2 = \mathbb{RP}^2 \# \mathbb{RP}^2 \# \mathbb{RP}^2$ +any [[Definition:Compact Space|compact]] [[Definition:Topological Surface|surface]] without [[Definition:Boundary of Manifold|boundary]] is [[Definition:Diffeomorphism|diffeomorphic]] to either $\mathbb S^2, g \mathbb T^2$, or $p \mathbb{RP}^2$. +The case for [[Definition:Topological Surface|surfaces]] with [[Definition:Boundary of Manifold|boundary]] is obtained through the removal and insertion of $\mathbb D^2$s to the [[Definition:Topological Surface|surface]]. +{{qed}} +[[Category:Algebraic Topology]] +piqrmyefe4oz6jpyvzf2snkoitr6rys +\end{proof}<|endoftext|> +\section{Standard Ordered Basis is Basis} +Tags: Module Theory, Linear Algebra + +\begin{theorem} +Let $\struct {R, +, \circ}$ be a [[Definition:Ring with Unity|ring with unity]] whose [[Definition:Ring Zero|zero]] is $0_R$ and whose [[Definition:Unity of Ring|unity]] is $1_R$. +Let $n$ be a [[Definition:Positive Integer|positive integer]]. +For each $j \in \closedint 1 n$, let $e_j$ be the [[Definition:Ordered Tuple|ordered $n$-tuple]] of [[Definition:Element|elements]] of $R$ whose $j$th entry is $1_R$ and all of whose other entries is $0_R$. +Then $\sequence {e_n}$ is an [[Definition:Ordered Basis|ordered basis]] of the [[Definition:Module on Cartesian Product|$R$-module $R^n$]]. +This [[Definition:Ordered Basis|ordered basis]] is called the '''[[Definition:Standard Ordered Basis|standard ordered basis of $R^n$]]'''. +The corresponding [[Definition:Set|set]] $\set {e_1, e_2, \ldots, e_n}$ is called the '''[[Definition:Standard Basis|standard basis of $R^n$]]'''. +\end{theorem} + +\begin{proof} +{{begin-eqn}} +{{eqn | l = \sum_{k \mathop = 1}^n \lambda_k e_k + | r = \lambda_1 \tuple {1_R, 0_R, 0_R, \ldots, 0_R} + | c = +}} +{{eqn | o = + + | r = \lambda_2 \tuple {0_R, 1_R, 0_R, \ldots, 0_R} + | c = +}} +{{eqn | o = + + | r = \ldots + | c = +}} +{{eqn | o = + + | r = \lambda_n \tuple {0_R, 0_R, 0_R, \ldots, 1_R} + | c = +}} +{{eqn | r = \tuple {\lambda_1, \lambda_2, \lambda_3, \ldots, \lambda_n} + | c = +}} +{{end-eqn}} +{{qed}} +\end{proof}<|endoftext|> +\section{Basis for Finite Submodule of Function Space} +Tags: Submodules, Function Spaces, Linear Algebra + +\begin{theorem} +Let $\struct {R, +, \circ}$ be a [[Definition:Ring with Unity|ring with unity]] whose [[Definition:Ring Zero|zero]] is $0_R$ and whose [[Definition:Unity of Ring|unity]] is $1_R$. +Let $A$ be a [[Definition:Set|set]]. +For each $a \in A$, let $f_a: A \to R$ be defined as: +:$\forall x \in A: \map {f_a} x = \begin{cases} +1 & : x = a \\ +0 & : x \ne a +\end{cases}$ +Then $B = \set {f_a: a \in A}$ is a [[Definition:Basis (Linear Algebra)|basis]] of the [[Finite Submodule of Function Space]] $R^{\paren A}$. +\end{theorem} + +\begin{proof} +Let $\sequence {a_k}_{1 \mathop \le k \mathop \le n}$ be a [[Definition:Sequence of Distinct Terms|sequence of distinct terms]] of $A$. +Let $\sequence {\lambda_k}_{1 \mathop \le k \mathop \le n}$ a [[Definition:Sequence|sequence]] of [[Definition:Scalar (Module)|scalars]]. +Then: $\displaystyle \sum_{k \mathop = 1}^n \lambda_k f_{a_k}$ is the [[Definition:Mapping|mapping]] whose value at $a_k$ is $\lambda_k$ and whose value at any $x$ not in $\set {a_1, a_2, \ldots, a_n}$ is zero. +Hence $B$ is a [[Definition:Generator of Module|generator]] of $R^{\left({A}\right)}$ which is [[Definition:Linearly Independent Set|linearly independent]]. +Thus, by definition, $B$ is a [[Definition:Basis (Linear Algebra)|basis]] of $R^{\paren A}$. +If $A = \closedint 1 n$, then $B$ is the [[Definition:Standard Basis|standard basis]] of $R^n$. +{{Qed}} +\end{proof}<|endoftext|> +\section{Unique Representation by Ordered Basis} +Tags: Unitary Modules, Linear Algebra + +\begin{theorem} +Let $G$ be a [[Definition:Unitary Module|unitary $R$-module]]. +Then $\sequence {a_n}$ is an [[Definition:Ordered Basis|ordered basis]] of $G$ {{Iff}}: +:For every $x \in G$ there exists [[Definition:Exactly One|one and only one]] [[Definition:Sequence|sequence]] $\sequence {\lambda_n}$ of [[Definition:Scalar (Module)|scalars]] such that $\displaystyle x = \sum_{k \mathop = 1}^n \lambda_k a_k$. +\end{theorem} + +\begin{proof} +=== Necessary Condition === +Let $\sequence {a_n}$ be an [[Definition:Ordered Basis|ordered basis]] of $G$. +Then every element of $G$ is a [[Definition:Linear Combination|linear combination]] of $\set {a_1, \ldots, a_n}$, which is a [[Definition:Generator of Module|generator]] of $G$, by [[Generated Submodule is Linear Combinations]]. +Thus there exists at least one such sequence of [[Definition:Scalar (Module)|scalar]]. +Now suppose there were two such sequences of [[Definition:Scalar (Module)|scalars]]: $\sequence {\lambda_n}$ and $\sequence {\mu_n}$. +That is, suppose $\displaystyle \sum_{k \mathop = 1}^n \lambda_k a_k = \sum_{k \mathop = 1}^n \mu_k a_k$. +Then: +{{begin-eqn}} +{{eqn | l =\ sum_{k \mathop = 1}^n \paren {\lambda_k - \mu_k} a_k + | r = \sum_{k \mathop = 1}^n \paren {\lambda_k a_k - \mu_k a_k} + | c = +}} +{{eqn | r = \sum_{k \mathop = 1}^n \lambda_k a_k - \sum_{k \mathop = 1}^n \mu_k a_k + | c = +}} +{{eqn | r = 0 + | c = +}} +{{end-eqn}} +So $\lambda_k = \mu_k$ for all $k \in \closedint 1 n$ as $\sequence {a_n}$ is a [[Definition:Linearly Independent Sequence|linearly independent sequence]]. +{{qed|lemma}} +=== Sufficient Condition === +Now suppose there is [[Definition:Exactly One|one and only one]] [[Definition:Sequence|sequence]] $\sequence {\lambda_n}$ such that the condition holds. +It is clear that $\set {a_1, \ldots, a_n}$ [[Definition:Generator of Module|generates]] $G$. +Suppose $\displaystyle \sum_{k \mathop = 1}^n \lambda_k a_k = 0$. +Then, since also $\displaystyle \sum_{k \mathop = 1}^n 0 a_k = 0$, we have, by hypothesis: +:$\forall k \in \closedint 1 n: \lambda_k = 0$ +Therefore $\sequence {a_n}$ is a [[Definition:Linearly Independent Sequence|linearly independent sequence]]. +{{qed}} +\end{proof}<|endoftext|> +\section{Isomorphism from R^n via n-Term Sequence} +Tags: Module Theory, Unitary Modules, Isomorphisms + +\begin{theorem} +Let $G$ be a [[Definition:Unitary Module|unitary $R$-module]]. +Let $\sequence {a_k}_{1 \mathop \le k \mathop \le n}$ be an [[Definition:Ordered Basis|ordered basis]] of $G$. +Let $R^n$ be the [[Definition:Module on Cartesian Product|$R$-module $R^n$]]. +Let $\psi: R^n \to G$ be defined as: +:$\displaystyle \map \psi {\sequence {\lambda_k}_{1 \mathop \le k \mathop \le n} } = \sum_{k \mathop = 1}^n \lambda_k a_k$ +Then $\psi$ is an [[Definition:R-Algebraic Structure Isomorphism|isomorphism]]. +\end{theorem} + +\begin{proof} +By [[Unique Representation by Ordered Basis]], $\psi$ is a [[Definition:Bijection|bijection]]. +We have: +{{begin-eqn}} +{{eqn | l = \sum_{k \mathop = 1}^n \lambda_k a_k + \sum_{k \mathop = 1}^n \mu_k a_k + | r = \sum_{k \mathop = 1}^n \paren {\lambda_k a_k + \mu_k a_k} + | c = +}} +{{eqn | r = \sum_{k \mathop = 1}^n \paren {\lambda_k + \mu_k} a_k + | c = +}} +{{end-eqn}} +and we have: +{{begin-eqn}} +{{eqn | l = \beta \sum_{k \mathop = 1}^n \lambda_k a_k + | r = \sum_{k \mathop = 1}^n \beta \paren {\lambda_k a_k} + | c = +}} +{{eqn | r = \sum_{k \mathop = 1}^n \paren {\beta \lambda_k} a_k + | c = +}} +{{end-eqn}} +thus proving that $\psi$ is also a [[Definition:R-Algebraic Structure Homomorphism|homomorphism]]. +{{Qed}} +\end{proof}<|endoftext|> +\section{Unitary R-Modules with n-Element Bases Isomorphic} +Tags: Unitary Modules + +\begin{theorem} +Any two [[Definition:Unitary Module|unitary $R$-modules]] having [[Definition:Basis (Linear Algebra)|bases]] of $n$ [[Definition:Element|elements]] are [[Definition:R-Algebraic Structure Isomorphism |isomorphic]]. +\end{theorem} + +\begin{proof} +From [[Isomorphism from R^n via n-Term Sequence]], they are both [[Definition:R-Algebraic Structure Isomorphism|isomorphic]] to the [[Definition:Module on Cartesian Product|$R$-module $R^n$]]. +{{qed}} +\end{proof}<|endoftext|> +\section{R-Module R^n is n-Dimensional} +Tags: Module on Cartesian Product + +\begin{theorem} +The [[Definition:Module on Cartesian Product|$R$-module $R^n$]] is [[Definition:Dimension (Linear Algebra)|$n$-dimensional]]. +\end{theorem} + +\begin{proof} +Follows directly from [[Standard Ordered Basis is Basis]]. +{{qed}} +[[Category:Module on Cartesian Product]] +4e2u0vlwjpxp8kx7zj4kipknso5ie65 +\end{proof}<|endoftext|> +\section{Unique Linear Transformation Between Modules} +Tags: Linear Transformations + +\begin{theorem} +Let $G$ and $H$ be [[Definition:Unitary Module|unitary $R$-modules]]. +Let $\left \langle {a_n} \right \rangle$ be an [[Definition:Ordered Basis|ordered basis]] of $G$. +Let $\left \langle {b_n} \right \rangle$ be a [[Definition:Sequence|sequence]] of elements of $H$. +Then there is a unique [[Definition:Linear Transformation|linear transformation]] $\phi: G \to H$ satisfying $\forall k \in \left[{1 \,.\,.\, n}\right]: \phi \left({a_k}\right) = b_k$ +\end{theorem} + +\begin{proof} +By [[Isomorphism from R^n via n-Term Sequence]], the mapping $\phi: G \to H$ defined as: +: $\displaystyle \phi \left({\sum_{k \mathop = 1}^n \lambda_k a_k}\right) = \sum_{k \mathop = 1}^n \lambda_k b_k$ +is well-defined. +Thus: +: $\forall k \in \left[{1 \,.\,.\, n}\right]: \phi \left({a_k}\right) = b_k$ +{{wtd|Verification that $\phi$ is linear needs to be done.}} +By [[Linear Transformation of Generated Module]], $\phi$ is the only [[Definition:Linear Transformation|linear transformation]] whose value at $a_k$ is $b_k$ for all $k \in \left[{1 \,.\,.\, n}\right]$. +{{Qed}} +\end{proof}<|endoftext|> +\section{Linear Transformation from Ordered Basis less Kernel} +Tags: Linear Transformations + +\begin{theorem} +Let $G$ and $H$ be [[Definition:Unitary Module|unitary $R$-modules]]. +Let $\phi: G \to H$ be a non-zero [[Definition:Linear Transformation|linear transformation]]. +Let $G$ be [[Definition:Dimension of Module|$n$-dimensional]]. +Let $\left \langle {a_n} \right \rangle$ be any [[Definition:Ordered Basis|ordered basis]] of $G$ such that $\left\{{a_k: r + 1 \le k \le n}\right\}$ is the [[Definition:Basis (Linear Algebra)|basis]] of the [[Definition:Kernel of Linear Transformation|kernel]] of $\phi$. +Then $\left \langle {\phi \left({a_r}\right)} \right \rangle$ is an [[Definition:Ordered Basis|ordered basis]] of the [[Definition:Image of Mapping|image]] of $\phi$. +\end{theorem} + +\begin{proof} +Suppose: +:$\displaystyle \sum_{k \mathop = 1}^r \lambda_k \phi \left({a_k}\right) = 0$ +Then: +:$\displaystyle \phi \left({\sum_{k \mathop = 1}^r \lambda_k a_k}\right) = 0$ +So $\displaystyle \sum_{k \mathop = 1}^r \lambda_k \phi \left({a_k}\right)$ belongs to the [[Definition:Kernel of Linear Transformation|kernel]] of $\phi$ and hence is also a [[Definition:Linear Combination|linear combination]] of $\left\{{a_k: r + 1 \le k \le n}\right\}$. +Thus $\forall k \in \left[{1 \,.\,.\, r}\right]: \lambda_k = 0$ since $\left \langle {a_n} \right \rangle$ is [[Definition:Linearly Independent Sequence|linearly independent]]. +Thus the sequence $\left \langle {\phi \left({a_r}\right)} \right \rangle$ is linearly independent. +We have $\forall k \in \left[{r + 1 \,.\,.\, n}\right]: \phi \left({a_k}\right) = 0$. +So let $x \in G$. +Let $\displaystyle x = \sum_{k \mathop = 1}^n \mu_k a_k$. +Then: +: $\displaystyle \phi \left({x}\right) = \sum_{k \mathop = 1}^n \mu_k \phi \left({a_k}\right) = \sum_{k \mathop = 1}^r \mu_k \phi \left({a_k}\right)$ +Therefore $\left \langle {\phi \left({a_r}\right)} \right \rangle$ is an [[Definition:Ordered Basis|ordered basis]] of the [[Definition:Image of Mapping|image]] of $\phi \left({G}\right)$. +{{Qed}} +\end{proof}<|endoftext|> +\section{Addition of Linear Transformations} +Tags: Linear Transformations + +\begin{theorem} +Let $\left({G, +_G, \circ}\right)_R$ and $\left({H, +_H, \circ}\right)_R$ be [[Definition:Module|$R$-modules]]. +Let $\phi: G \to H$ and $\psi: G \to H$ be [[Definition:Linear Transformation|linear transformations]]. +Let $\phi +_H \psi$ be the operation on $H^G$ induced by $+_H$ as defined in [[Definition:Induced Structure|Induced Structure]]. +Then $\phi +_H \psi: G \to H$ is a [[Definition:Linear Transformation|linear transformation]]. +\end{theorem}<|endoftext|> +\section{Negative Linear Transformation} +Tags: Linear Transformations + +\begin{theorem} +Let $\left({G, +_G, \circ}\right)_R$ and $\left({H, +_H, \circ}\right)_R$ be [[Definition:Module|$R$-modules]]. +Let $\phi: G \to H$ be a [[Definition:Linear Transformation|linear transformation]]. +Let $- \phi$ be the negative of $\phi$ as defined in [[Induced Structure Inverse]]. +Then $- \phi: G \to H$ is also a [[Definition:Linear Transformation|linear transformation]]. +\end{theorem} + +\begin{proof} +From the definition of a [[Definition:Module|module]], the [[Definition:Group|group]] $\left({H, +_H}\right)$ is [[Definition:Abelian Group|abelian]]. +Therefore we can apply [[Inverse Mapping in Induced Structure]] to show that $- \phi: G \to H$ is a [[Definition:R-Algebraic Structure Homomorphism|homomorphism]]. +Then: +{{begin-eqn}} +{{eqn | l=\left({- \phi}\right) \left({\lambda \circ x}\right) + | r=-\phi \left({\lambda \circ x}\right) + | c= +}} +{{eqn | r=- \lambda \circ \phi \left({x}\right) + | c= +}} +{{eqn | r=\lambda \circ \left({- \phi \left({x}\right)}\right) + | c= +}} +{{eqn | r=\lambda \circ \left({- \phi}\right) \left({x}\right) + | c= +}} +{{end-eqn}} +{{qed}} +\end{proof}<|endoftext|> +\section{Linear Transformations form Abelian Group} +Tags: Linear Transformations, Abelian Groups + +\begin{theorem} +Let $\struct {G, +_G}$ and $\struct {H, +_H}$ be [[Definition:Group|groups]]. +Let $\struct {R, +_R, \times_R}$ be a [[Definition:Ring (Abstract Algebra)|ring]]. +Let $\struct {G, +_G, \circ}_R$ and $\struct {H, +_H, \circ}_R$ be [[Definition:Module|$R$-modules]]. +Let $\map {\mathcal L_R} {G, H}$ be [[Definition:Set of All Linear Transformations|the set of all linear transformations]] from $G$ to $H$. +Let $\oplus_H$ be the operation on $H^G$ as defined in [[Addition of Linear Transformations]]. +Then $\struct {\map {\LL_R} {G, H}, \oplus_H}$ is an [[Definition:Abelian Group|abelian group]]. +\end{theorem} + +\begin{proof} +From [[Structure Induced by Group Operation is Group]], $\struct {H^G, \oplus_H}$ is a [[Definition:Group|group]] +Let $\phi, \psi \in \map {\LL_R} {G, H}$. +From [[Addition of Linear Transformations]], $\phi \oplus_H \psi \in \map {\LL_R} {G, H}$. +From [[Negative Linear Transformation]], $-\phi \in \map {\LL_R} {G, H}$. +Thus, from the [[Two-Step Subgroup Test]], $\struct {\map {\LL_R} {G, H}, \oplus_H}$ is a [[Definition:Subgroup|subgroup]] of $\struct {H^G, \oplus_H}$. +{{finish|Still need to show that it is abelian}} +\end{proof}<|endoftext|> +\section{Morse-Sard Theorem} +Tags: Topology + +\begin{theorem} +Let $f: X \to Y$ be any [[Definition:Smooth Mapping|smooth map]] of [[Definition:Topological Manifold|manifolds]]. +Then [[Definition:Almost Every|almost every]] point in $Y$ is a [[Definition:Regular Value|regular value]] of $f$. +\end{theorem} + +\begin{proof} +{{ProofWanted|In progress}} +\end{proof}<|endoftext|> +\section{Linear Transformation from Center of Scalar Ring} +Tags: Linear Transformations + +\begin{theorem} +Let $\struct {G, +_G, \circ}_R$ and $\struct {H, +_H, \circ}_R$ be [[Definition:Module|$R$-modules]]. +Let $\phi: G \to H$ be a [[Definition:Linear Transformation|linear transformation]]. +Let $\map Z R$ be the [[Definition:Center of Ring|center]] of the [[Definition:Scalar Ring of Module|scalar ring]] $R$. +Let $\lambda \in \map Z R$. +Then $\lambda \circ \phi$ is a [[Definition:Linear Transformation|linear transformation]]. +\end{theorem} + +\begin{proof} +Let $\lambda \in \lambda \in \map Z R$. +Then: +{{begin-eqn}} +{{eqn | l = \map {\paren {\lambda \circ \phi} } {x +_G y} + | r = \lambda \circ \map \phi {x +_G y} + | c = +}} +{{eqn | r = \lambda \circ \paren {\map \phi x +_H \map \phi y} + | c = +}} +{{eqn | r = \lambda \circ \map \phi x +_H \lambda \circ \map \phi y + | c = +}} +{{eqn | r = \map {\paren {\lambda \circ \phi} } x +_H \map {\paren {\lambda \circ \phi} } y + | c = +}} +{{end-eqn}} +Because $\lambda \in \map Z R$, $\lambda$ [[Definition:Commute|commutes]] with all elements of $R$. +So $\forall \mu \in R: \lambda \circ \mu = \mu \circ \lambda$. +Thus: +{{begin-eqn}} +{{eqn | l = \map {\paren {\lambda \circ \phi} } {\mu \circ x} + | r = \lambda \circ \map \phi {\mu \circ x} + | c = +}} +{{eqn | r = \lambda \circ \mu \circ \map \phi x + | c = +}} +{{eqn | r = \mu \circ \lambda \circ \map \phi x + | c = as $\lambda \in \map Z R$ +}} +{{eqn | r = \mu \circ \map {\paren {\lambda \circ \phi} } x + | c = +}} +{{end-eqn}} +{{qed}} +\end{proof}<|endoftext|> +\section{Linear Transformations of Commutative Scalar Ring} +Tags: Linear Transformations + +\begin{theorem} +Let $R$ be a [[Definition:Commutative Ring|commutative ring]]. +Let $\left({G, +_G, \circ}\right)_R$ and $\left({H, +_H, \circ}\right)_R$ be [[Definition:Module|$R$-modules]]. +Let $\mathcal L_R \left({G, H}\right)$ be [[Definition:Set of All Linear Transformations|the set of all linear transformations]] from $G$ to $H$. +Then $\mathcal L_R \left({G, H}\right)$ is a [[Definition:Submodule|submodule]] of the [[Definition:Module of All Mappings|$R$-module $H^G$]]. +If $H$ is a [[Definition:Unitary Module|unitary module]], then so is $\mathcal L_R \left({G, H}\right)$. +\end{theorem} + +\begin{proof} +From [[Group equals Center iff Abelian]], the [[Definition:Center of Ring|center]] of a [[Definition:Commutative Ring|commutative ring]] is the entire ring. +The result follows from [[Linear Transformation from Center of Scalar Ring]]. +{{qed}} +\end{proof}<|endoftext|> +\section{Product of Linear Transformations} +Tags: Linear Transformations + +\begin{theorem} +Let $R$ be a [[Definition:Commutative and Unitary Ring|commutative ring with unity]] whose [[Definition:Ring Zero|zero]] is $0_R$ and whose [[Definition:Unity of Ring|unity]] is $1_R$. +Let $\struct {G, +_G, \circ}_R$ be a [[Definition:Unitary Module|unitary $R$-module]] such that $\map \dim G = n$. +Let $\struct {H, +_H, \circ}_R$ be a [[Definition:Unitary Module|unitary $R$-module]] such that $\map \dim H = m$. +Let $\map {\LL_R} {G, H}$ be [[Definition:Set of All Linear Transformations|the set of all linear transformations]] from $G$ to $H$. +Then: +:$\map \dim {\map {\LL_R} {G, H} } = n m$ +Let $\sequence {a_n}$ be an [[Definition:Ordered Basis|ordered basis]] for $G$. +Let $\sequence {b_m}$ be an [[Definition:Ordered Basis|ordered basis]] for $H$. +Let $\phi_{i j}: G \to H$ be the unique [[Definition:Linear Transformation|linear transformation]] defined for each $i \in \closedint 1 n, j \in \closedint 1 m$ which satisfies: +:$\forall k \in \closedint 1 n: \map {\phi_{i j} } {a_k} = \delta_{i k} b_j$ +where $\delta$ is the [[Definition:Kronecker Delta|Kronecker delta]]. +Then: +:$\set {\phi_{i j}: i \in \closedint 1 n, j \in \closedint 1 m}$ +is a [[Definition:Basis (Linear Algebra)|basis]] for $\map \dim {\map {\LL_R} {G, H} }$. +\end{theorem} + +\begin{proof} +Let $B = \set {\phi_{i j}: i \in \closedint 1 n, j \in \closedint 1 m}$. +Let $\displaystyle \sum_{j \mathop = 1}^m \sum_{i \mathop = 1}^n \lambda_{i j} \phi_{i j} = 0$. +Then: +:$\displaystyle \forall k \in \closedint 1 n: 0 = \sum_{j \mathop = 1}^m \sum_{i \mathop = 1}^n \lambda_{i j} \map {\phi_{i j} } {a_k} = \sum_{j \mathop = 1}^m \lambda_{k j} b_j$ +So: +:$\forall j \in \closedint 1 n: \lambda_{k j} = 0$ +Hence $B$ is [[Definition:Linearly Independent Set|linearly independent]]. +Now let $\phi \in \map {\LL_R} {G, H}$. +Let $\tuple {\alpha_{i 1}, \alpha_{i 2}, \ldots, \alpha_{i m} }$ be the [[Definition:Sequence|sequence]] of [[Definition:Scalar (Module)|scalars]] that satisfies: +:$\displaystyle \forall i \in \closedint 1 n: \map \phi {a_i} = \sum_{j \mathop = 1}^m \alpha_{i j} b_j$ +Then: +:$\displaystyle \forall k \in \closedint 1 n: \map \phi {a_k} = \map {\paren {\sum_{j \mathop = 1}^m \sum_{i \mathop = 1}^n \alpha_{i j} u_{i j} } } {a_k}$ +by a calculation similar to the preceding. +So, by [[Linear Transformation of Generated Module]]: +:$\displaystyle u = \sum_{j \mathop = 1}^m \sum_{i \mathop = 1}^n \alpha_{i j} u_{i j}$ +Thus $B$ is a [[Definition:Generator of Module|generator]] for $\phi \in \map {\LL_R} {G, H}$. +{{Qed}} +\end{proof}<|endoftext|> +\section{Ring of Linear Operators} +Tags: Linear Operators + +\begin{theorem} +Let $\map {\LL_R} G$ be [[Definition:Set of All Linear Transformations|the set of all linear operators]] on $G$. +{{explain|the precise nature of $G$}} +Let $\phi \circ \psi$ denote the [[Definition:Composition of Mappings|composition]] of the two [[Definition:Linear Operator|linear operators]] $\phi$ and $\psi$. +Then $\struct {\map {\LL_R} G, +, \circ}$ is a [[Definition:Ring (Abstract Algebra)|ring]]. +\end{theorem} + +\begin{proof} +Follows from [[Composite of Homomorphisms is Homomorphism/R-Algebraic Structure|Composite of R-Algebraic Structure Homomorphisms is Homomorphism]], as it is a [[Definition:Subring|subring]] of the ring of all endomorphisms of the [[Definition:Abelian Group|abelian group]] $\struct {G, +}$. +{{ProofWanted}} +[[Category:Linear Operators]] +eda5k9ojbrvu9i00azjlutffjtyrjib +\end{proof}<|endoftext|> +\section{Dimension of Algebraic Dual} +Tags: Linear Transformations + +\begin{theorem} +Let $G$ be an [[Definition:Dimension (Linear Algebra)|$n$-dimensional]] [[Definition:Module|$R$-module]]. +Let $G^*$ be the [[Definition:Algebraic Dual|algebraic dual]] of $G$. +Let $G^{**}$ be the [[Definition:Algebraic Dual|algebraic dual]] of $G^*$. +Then $G^*$ and $G^{**}$ are also [[Definition:Dimension (Linear Algebra)|$n$-dimensional]]. +\end{theorem} + +\begin{proof} +Follows directly from [[Product of Linear Transformations]]. +{{Qed}} +\end{proof}<|endoftext|> +\section{Annihilator is Submodule of Algebraic Dual} +Tags: Linear Transformations + +\begin{theorem} +Let $R$ be a [[Definition:Commutative Ring|commutative ring]]. +Let $G$ be a [[Definition:Module|module]] over $R$. +Let $M$ be a [[Definition:Submodule|submodule]] of $G$. +Let $G^*$ be the [[Definition:Algebraic Dual|algebraic dual]] of $G$. +Then the [[Definition:Annihilator on Algebraic Dual|annihilator]] $M^\circ$ of $M$ is a [[Definition:Submodule|submodule]] of $G^*$. +Similarly, let $N$ be a [[Definition:Submodule|submodule]] of $G^*$. +Let $G^{**}$ be the [[Definition:Algebraic Dual|algebraic dual]] of $G^*$. +Then the [[Definition:Annihilator on Algebraic Dual|annihilator]] $N^\circ$ of $N$ is a [[Definition:Submodule|submodule]] of $G^{**}$. +\end{theorem}<|endoftext|> +\section{Inverse Evaluation Isomorphism of Annihilator} +Tags: Linear Transformations + +\begin{theorem} +Let $R$ be a [[Definition:Commutative Ring|commutative ring]]. +Let $G$ be a [[Definition:Module|module]] over $R$ whose [[Definition:Dimension of Module|dimension]] is [[Definition:Finite|finite]]. +Let $G^*$ be the [[Definition:Algebraic Dual|algebraic dual]] of $G$. +Let $G^{**}$ be the [[Definition:Algebraic Dual|algebraic dual]] of $G^*$. +Let $N$ be a [[Definition:Submodule|submodule]] of $G^*$. +Let $J$ be the [[Definition:Evaluation Isomorphism|evaluation isomorphism]] from $G$ onto $G^{**}$. +Let $N^\circ$ be the [[Definition:Annihilator on Algebraic Dual|annihilator]] of $N$. +Then: +:$J^{-1} \left({N^\circ}\right) = \left\{{x \in G: \forall t' \in N: t' \left({x}\right) = 0}\right\}$ +\end{theorem} + +\begin{proof} +As $G$ is finite-dimensional, then by [[Evaluation Isomorphism is Isomorphism]] $J: G \to G^{**}$ is an [[Definition:Module Isomorphism|isomorphism]], and therefore a [[Definition:Surjection|surjection]]. +Thus: +:$N^\circ = \left\{{x^\wedge \in G^{**}: \forall t' \in N: x^\wedge \left({t'}\right) = 0}\right\}$ +where $x^\wedge$ is as defined in the definition of the [[Definition:Evaluation Linear Transformation|evaluation linear transformation]]. +The result follows. +{{qed}} +\end{proof}<|endoftext|> +\section{Properties of Evaluation Linear Transformation} +Tags: Linear Transformations + +\begin{theorem} +Let $R$ be a [[Definition:Commutative Ring|commutative ring]]. +Let $G$ be an [[Definition:Module|$R$-module]]. +Let $G^*$ be the [[Definition:Algebraic Dual|algebraic dual]] of $G$. +Let $\left \langle {x, t'} \right \rangle$ be the [[Definition:Evaluation Linear Transformation|evaluation linear transformation]] from $G$ to $G^{**}$. +Then the [[Definition:Mapping|mapping]] $\phi: G \times G^* \to R$ defined as $\forall \left({x, t'}\right) \in G \times G^*: \phi \left({x, t'}\right) = \left \langle {x, t'} \right \rangle$ satisfies the following properties: +: $(1): \quad \forall x, y \in G: \forall t' \in G^*: \left \langle {x + y, t'} \right \rangle = \left \langle {x, t'} \right \rangle + \left \langle {y, t'} \right \rangle$ +: $(2): \quad \forall x \in G: \forall s', t' \in G^*: \left \langle {x, s' + t'} \right \rangle = \left \langle {x, s'} \right \rangle + \left \langle {x, t'} \right \rangle$ +: $(3): \quad \forall x \in G: \forall s', t' \in G^*: \forall \lambda \in R: \left \langle {\lambda x, t'} \right \rangle = \lambda \left \langle {x, t'} \right \rangle = \left \langle {x, \lambda t'} \right \rangle$ +\end{theorem}<|endoftext|> +\section{Zero Vector Space Product iff Factor is Zero} +Tags: Linear Algebra, Vector Algebra, Zero Vector Space Product iff Factor is Zero, Zero Vectors + +\begin{theorem} +Let $F$ be a [[Definition:Field (Abstract Algebra)|field]] whose [[Definition:Field Zero|zero]] is $0_F$ and whose [[Definition:Unity of Field|unity]] is $1_F$. +Let $\struct {\mathbf V, +, \circ}_F$ be a [[Definition:Vector Space|vector space]] over $F$, as defined by the [[Definition:Vector Space Axioms|vector space axioms]]. +Let $\mathbf v \in \mathbf V, \lambda \in F$. +Then: +:$\lambda \circ \mathbf v = \bszero \iff \paren {\lambda = 0_F \lor x = \bszero}$ +\end{theorem} + +\begin{proof} +A [[Definition:Vector Space|vector space]] is a [[Definition:Module|module]], so all results about modules also apply to vector spaces. +So from [[Scalar Product with Identity]] it follows directly that: +:$\lambda = 0_F \lor \mathbf v = e \implies \lambda \circ \mathbf v = \bszero$ +Next, suppose $\lambda \circ \mathbf v = \bszero$ but $\lambda \ne 0_F$. +Then: +{{begin-eqn}} +{{eqn | l = \bszero + | r = \lambda^{-1} \circ \bszero + | c = [[Zero Vector Scaled is Zero Vector]] +}} +{{eqn | r = \lambda^{-1} \circ \paren {\lambda \circ \mathbf v} + | c = as $\lambda \circ \mathbf v = \bszero$ +}} +{{eqn | r = \paren {\lambda^{-1} \circ \lambda} \circ \mathbf v + | c = {{Vector-space-axiom|7}} +}} +{{eqn | r = 1 \circ \mathbf v + | c = {{Field-axiom|M4}} +}} +{{eqn | r = \mathbf v + | c = {{Vector-space-axiom|8}} +}} +{{end-eqn}} +{{Qed}} +\end{proof} + +\begin{proof} +The [[Definition:Sufficient Condition|sufficient condition]] is proved in [[Vector Scaled by Zero is Zero Vector]], and in [[Zero Vector Scaled is Zero Vector]]. +The [[Definition:Necessary Condition|necessary condition]] is proved in [[Vector Product is Zero only if Factor is Zero]]. +{{qed}} +\end{proof}<|endoftext|> +\section{Homomorphic Image of Vector Space} +Tags: Homomorphisms, Linear Algebra + +\begin{theorem} +Let $\struct {K, +_K, \times_K}$ be a [[Definition:Division Ring|division ring]]. +Let $\struct {V, +_V, \circ_V}_K$ be a [[Definition:Vector Space|$K$-vector space]]. +Let $\struct {W, +_W, \circ_W}_K$ be a [[Definition:R-Algebraic Structure|$K$-algebraic structure]]. +Let $\phi: V \to W$ be a [[Definition:R-Algebraic Structure Homomorphism|homomorphism]], i.e. a [[Definition:Linear Transformation on Vector Space|linear transformation]]. +Then the [[Definition:Homomorphic Image|homomorphic image]] of $\phi$ is a [[Definition:Vector Space|$K$-vector space]]. +\end{theorem} + +\begin{proof} +Let us write $\phi \sqbrk V$ for the [[Definition:Homomorphic Image|homomorphic image]] of $\phi$. +From [[Homomorphic Image of R-Module is R-Module]], $\phi \sqbrk V$ is a [[Definition:Module|$K$-module]]. +It thus suffices to show that $\phi \sqbrk V$ is [[Definition:Unitary Module|unitary]], since then it will be a [[Definition:Vector Space|$K$-vector space]]. +To this end, let $1_K$ be the [[Definition:Unity of Ring|unity]] of $K$. +Then for any $\map \phi {\mathbf v} \in \phi \sqbrk V$, compute: +{{begin-eqn}} +{{eqn | l = 1_K \circ_W \map \phi {\mathbf v} + | r = \map \phi {1_K \circ_V \mathbf v} + | c = $\phi$ is a [[Definition:Linear Transformation on Vector Space|linear transformation]] +}} +{{eqn | r = \map \phi {\mathbf v} + | c = $V$ is a [[Definition:Vector Space|$K$-vector space]] +}} +{{end-eqn}} +Hence $\phi \sqbrk V$ is [[Definition:Unitary Module|unitary]], and so a [[Definition:Vector Space|$K$-vector space]]. +{{qed}} +\end{proof}<|endoftext|> +\section{Direct Product of Vector Spaces is Vector Space} +Tags: Linear Algebra + +\begin{theorem} +Let $K$ be a [[Definition:Field (Abstract Algebra)|field]]. +Let $V_1, V_2, \ldots, V_n$ be [[Definition:Vector Space|$K$-vector spaces]]. +Let $\struct {V, + , \circ}_K$ be their [[Definition:Direct Product of Vector Spaces|direct product]]. +Then $\struct {V, + , \circ}_K$ is a [[Definition:Vector Space|$K$-vector space]]. +\end{theorem} + +\begin{proof} +This follows directly from [[Finite Direct Product of Modules is Module]] and the definition of [[Definition:Vector Space|vector space]]. +\end{proof}<|endoftext|> +\section{Definition:Vector Space on Cartesian Product} +Tags: Definitions: Examples of Vector Spaces + +\begin{theorem} +Let $\struct {K, +, \circ}$ be a [[Definition:Division Ring|division ring]]. +Let $n \in \N_{>0}$. +Let $+: K^n \times K^n \to K^n$ be defined as: +:$\tuple {\alpha_1, \ldots, \alpha_n} + \tuple {\beta_1, \ldots, \beta_n} = \tuple {\alpha_1 +_R \beta_1, \ldots, \alpha_n +_R \beta_n}$ +Let $\times: K \times K^n \to K^n$ be defined as: +:$\lambda \times \tuple {\alpha_1, \ldots, \alpha_n} = \tuple {\lambda \times_R \alpha_1, \ldots, \lambda \times_R \alpha_n}$ +Then $\struct {K^n, +, \times}_K$ is '''the $K$-vector space $K^n$'''. +\end{theorem}<|endoftext|> +\section{Definition:Vector Space over Division Subring} +Tags: Definitions: Examples of Vector Spaces + +\begin{theorem} +Let $\struct {R, +, \circ}$ be a [[Definition:Ring with Unity|ring with unity]] whose [[Definition:Unity of Ring|unity]] is $1_R$. +Let $S$ be a [[Definition:Division Subring|division subring]] of $R$, such that $1_R \in S$. +Then $\struct {R, +, \circ_S}_S$, where $\circ_S$ is the [[Definition:Restriction of Operation|restriction]] of $\circ$ to $S \times R$, is the '''vector space on $R$ over the [[Definition:Division Subring|division subring]] $S$'''. +\end{theorem}<|endoftext|> +\section{Complex Numbers form Vector Space over Reals} +Tags: Linear Algebra, Complex Analysis, Examples of Vector Spaces + +\begin{theorem} +Let $\R$ be the set of [[Definition:Real Number|real numbers]]. +Let $\C$ be the set of [[Definition:Complex Number|complex numbers]]. +Then the [[Definition:Module|$\R$-module]] $\C$ is a [[Definition:Vector Space|vector space]]. +\end{theorem} + +\begin{proof} +Recall that [[Real Numbers form Field]]. +Thus by definition, $\R$ is also a [[Definition:Division Ring|division ring]]. +Thus we only need to show that [[Definition:Module|$\R$-module]] $\C$ is a [[Definition:Unitary Module|unitary module]], by demonstrating the module properties: +$\forall x, y, \in \C, \forall \lambda, \mu \in \R$: +:$(1): \quad \lambda \paren {x + y} = \paren {\lambda x} + \paren {\lambda y}$ +:$(2): \quad \paren {\lambda + \mu} x = \paren {\lambda x} + \paren {\mu x}$ +:$(3): \quad \paren {\lambda \mu} x = \lambda \paren {\mu x}$ +:$(4): \quad 1 x = x$ +As $\lambda, \mu \in \R$ it follows that $\lambda, \mu \in \C$. +Thus from [[Complex Multiplication Distributes over Addition]], $(1)$ and $(2)$ immediately follow. +$(3)$ follows from [[Complex Multiplication is Associative]]. +$(4)$ follows from [[Complex Multiplication Identity is One]], as $1 + 0 i$ is the [[Definition:Unity of Field|unity]] of $\C$. +{{qed}} +\end{proof}<|endoftext|> +\section{Division Ring is Vector Space over Prime Subfield} +Tags: Division Rings, Subfields, Linear Algebra + +\begin{theorem} +Let $\struct {K, +, \times}$ be a [[Definition:Division Ring|division ring]]. +Let $\struct {S, +, \times}$ be the [[Definition:Prime Subfield|prime subfield]] of $K$ +Then $\struct {K, +, \times_S}_S$ is an [[Definition:Vector Space|$S$-vector space]], where $\times_S$ is the [[Definition:Restriction of Operation|restriction]] of $\times$ to $S \times K$. +\end{theorem}<|endoftext|> +\section{Vector Space over Subring} +Tags: Linear Algebra + +\begin{theorem} +Let $K$ be a [[Definition:Division Subring|division subring]] of the [[Definition:Division Ring|division ring]] $\struct {L, +_L, \times_L}$. +Let $\struct {G, +_G, \circ}_L$ be a [[Definition:Vector Space|$L$-vector space]]. +Then $\struct {G, +_G, \circ_K}_K$ is a [[Definition:Vector Space|$K$-vector space]], where $\circ_K$ is the [[Definition:Restriction of Operation|restriction]] of $\circ$ to $K \times G$. +The [[Definition:Vector Space|$K$-vector space]] $\struct {G, +_G, \circ_K}_K$ is called the '''$K$-vector space obtained from $\struct {L, +_L, \times_L}$ by restricting scalar multiplication'''. +\end{theorem}<|endoftext|> +\section{Subspaces of Dimension 2 Real Vector Space} +Tags: Linear Algebra, Subspaces of Dimension 2 Real Vector Space + +\begin{theorem} +Take the [[Definition:Real Vector Space|$\R$-vector space]] $\left({\R^2, +, \times}\right)_\R$. +Let $S$ be a [[Definition:Vector Subspace|subspace]] of $\left({\R^2, +, \times}\right)_\R$. +Then $S$ is one of: +: $(1): \quad \left({\R^2, +, \times}\right)_\R$ +: $(2): \quad \left\{{0}\right\}$ +: $(3): \quad$ A line through the origin. +\end{theorem} + +\begin{proof} +Let $S$ be a non-zero [[Definition:Vector Subspace|subspace]] of $\left({\R^2, +, \times}\right)_\R$. +Then $S$ contains a [[Definition:Zero Vector|non-zero vector]] $\left({\alpha_1, \alpha_2}\right)$. +Hence $S$ also contains $\left\{{\lambda \times \left({\alpha_1, \alpha_2}\right), \lambda \in \R}\right\}$. +From [[Equation of Straight Line in Plane]], this set may be described as a line through the origin. +Suppose $S$ also contains a [[Definition:Zero Vector|non-zero vector]] $\left({\beta_1, \beta_2}\right)$ which is not on that line. +Then $\alpha_1 \times \beta_2 - \alpha_2 \times \beta_1 \ne 0$. +Otherwise $\left({\beta_1, \beta_2}\right)$ would be $\zeta \times \left({\alpha_1, \alpha_2}\right)$, where either $\zeta = \beta_1 / \alpha_1$ or $\zeta = \beta_2 / \alpha_2$ according to whether $\alpha_1 \ne 0$ or $\alpha_2 \ne 0$. +But then $S = \left({\R^2, +, \times}\right)_\R$. +Because, if $\left({\gamma_1, \gamma_2}\right)$ is any vector at all, then: +: $\left({\gamma_1, \gamma_2}\right) = \lambda \times \left({\alpha_1, \alpha_2}\right) + \mu \times \left({\beta_1, \beta_2}\right)$ +where $\lambda = \dfrac {\gamma_1 \times \beta_2 - \gamma_2 \times \beta_1} {\alpha_1 \times \beta_2 - \alpha_2 \times \beta_1}, \mu = \dfrac {\alpha_1 \times \gamma_2 - \alpha_2 \times \gamma_1} {\alpha_1 \times \beta_2 - \alpha_2 \times \beta_1}$ +which we get by solving the simultaneous eqns: +{{begin-eqn}} +{{eqn | l=\alpha_1 \times \lambda + \beta_1 \times \mu + | r=0 + | c= +}} +{{eqn | l=\alpha_2 \times \lambda + \beta_2 \times \mu + | r=0 + | c= +}} +{{end-eqn}} +The result follows. +{{qed}} +\end{proof} + +\begin{proof} +Follows directly from [[Dimension of Proper Subspace is Less Than its Superspace]]. +{{qed}} +\end{proof}<|endoftext|> +\section{Subspace of Real Continuous Functions} +Tags: Vector Subspaces, Analysis + +\begin{theorem} +Let $\mathbb J = \set {x \in \R: a \le x \le b}$ be a [[Definition:Closed Real Interval|closed interval]] of the [[Definition:Real Number Line|real number line]] $\R$. +Let $\map \CC {\mathbb J}$ be the set of all [[Definition:Continuous Real Function|continuous real functions]] on $\mathbb J$. +Then $\struct {\map \CC {\mathbb J}, +, \times}_\R$ is a [[Definition:Vector Subspace|subspace]] of the [[Definition:Vector Space|$\R$-vector space]] $\struct {\R^{\mathbb J}, +, \times}_\R$. +\end{theorem} + +\begin{proof} +By definition, $\map \CC {\mathbb J} \subseteq \R^{\mathbb J}$. +Let $f, g \in \map \CC {\mathbb J}$. +By [[Two-Step Vector Subspace Test]], it needs to be shown that: +:$\paren 1: \quad f + g \in \map \CC {\mathbb J}$ +:$\paren 2: \quad \lambda f \in \map \CC {\mathbb J}$ for any $\lambda \in \R$ +$\paren 1$ follows by [[Sum Rule for Continuous Functions]]. +$\paren 2$ follows by [[Multiple Rule for Continuous Functions]]. +Hence $\struct {\map \CC {\mathbb J}, +, \times}_\R$ is a [[Definition:Vector Subspace|subspace]] of the [[Definition:Vector Space|$\R$-vector space]] $\struct {\R^{\mathbb J}, +, \times}_\R$. +{{qed}} +\end{proof}<|endoftext|> +\section{Singleton is Linearly Independent} +Tags: Linear Algebra + +\begin{theorem} +Let $K$ be a [[Definition:Division Ring|division ring]]. +Let $\struct {G, +_G}$ be a [[Definition:Group|group]] whose [[Definition:Identity Element|identity]] is $e$. +Let $\struct {G, +_G, \circ}_K$ be a [[Definition:Vector Space|$K$-vector space]] whose [[Definition:Zero Scalar|zero]] is $0_K$. +Let $x \in G: x \ne e$. +Then $\set x$ is a [[Definition:Linearly Independent Set|linearly independent subset]] of $G$. +\end{theorem} + +\begin{proof} +The only [[Definition:Sequence of Distinct Terms|sequence of distinct terms]] in $\set x$ is the one that goes: $x$. +Suppose $\exists \lambda \in K: \lambda \circ x = e$. +From [[Zero Vector Space Product iff Factor is Zero]] it follows that $\lambda = 0$. +Hence the result from definition of [[Definition:Linearly Independent Set|linearly independent set]]. +{{qed}} +\end{proof}<|endoftext|> +\section{Linearly Dependent Sequence of Vector Space} +Tags: Linear Algebra + +\begin{theorem} +Let $\struct {G, +}$ be a [[Definition:Group|group]] whose [[Definition:Identity Element|identity]] is $\mathbf 0$. +Let $\struct {G, +, \circ}_K$ be a [[Definition:Vector Space|$K$-vector space]]. +Let $\sequence {a_k}_{1 \mathop \le k \mathop \le n}$ be a [[Definition:Sequence|sequence]] of distinct [[Definition:Zero Vector|non-zero vectors]] of $G$. +Then $\sequence {a_k}_{1 \mathop \le k \mathop \le n}$ is [[Definition:Linearly Dependent Sequence|linearly dependent]] {{iff}}: +:$\exists p \in \closedint 2 n: a_p$ is a [[Definition:Linear Combination|linear combination]] of $\sequence {a_k}_{1 \mathop \le k \mathop \le p - 1}$ +\end{theorem} + +\begin{proof} +=== Necessary Condition === +Suppose $\sequence {a_k}_{1 \mathop \le k \mathop \le n}$ is [[Definition:Linearly Dependent Sequence|linearly dependent]]. +[[Definition:By Hypothesis|By hypothesis]], the [[Definition:Set|set]] of all [[Definition:Integer|integers]] $r \in \closedint 1 n$ such that $\sequence {a_k}_{1 \mathop \le k \mathop \le r}$ is [[Definition:Linearly Independent Sequence|linearly dependent]] is not [[Definition:Empty Set|empty]]. +Let $p$ be its [[Definition:Smallest Element|smallest element]]. +Then from [[Singleton is Linearly Independent]], $p \ge 2$, as $a_1 \ne \mathbf 0$ and hence $\set {a_1}$ is [[Definition:Linearly Independent Sequence|linearly independent]]. +Also, there exist [[Definition:Scalar (Vector Space)|scalars]] $\lambda_1, \ldots, \lambda_p$, not all of which are [[Definition:Zero Scalar|zero]], such that $\displaystyle \sum_{k \mathop = 1}^p \lambda_k \circ a_k = \mathbf 0$. +Suppose $\lambda_p = 0$. +Then not all of $\lambda_1, \ldots, \lambda_{p-1}$ can be [[Definition:Zero Scalar|zero]]. +Then $\sequence {a_k}_{1 \mathop \le k \mathop \le p-1}$ is [[Definition:Linearly Independent Sequence|linearly dependent]]. +That contradicts the definition of $p$, so $\lambda_p \ne 0$. +So, because: +:$\displaystyle \lambda_p \circ a_p = - \sum_{k \mathop = 1}^{p - 1} \lambda_k \circ a_k$ +we must have: +:$\displaystyle a_p = \sum_{k \mathop = 1}^{p - 1} \paren {-{\lambda_p}^{-1} \lambda_k} \circ a_k$ +and thus $a_p$ is a [[Definition:Linear Combination|linear combination]] of $\sequence {a_k}_{1 \mathop \le k \mathop \le p - 1}$. +{{qed|lemma}} +=== Sufficient Condition === +Now suppose that $a_p$ is a [[Definition:Linear Combination|linear combination]] of $\sequence {a_k}_{1 \mathop \le k \mathop \le p - 1}$. +Then: +:$\displaystyle a_p = \sum_{k \mathop = 1}^{p - 1} \mu_k \circ a_k$ +So we can assign values to $\lambda_k$ as follows: +:$\forall k \in \closedint 1 n: \lambda_k = \begin{cases} +\mu_k & : k < p \\ +-1 & : k = p \\ +0 & : k > p \\ +\end{cases}$ +Then: +:$\displaystyle \sum_{k \mathop = 1}^n \lambda_k \circ a_k = \mathbf 0$ +Hence the result. +{{qed}} +\end{proof}<|endoftext|> +\section{Vector Space has Basis Between Linearly Independent Set and Finite Spanning Set} +Tags: Bases of Vector Spaces, Generators of Vector Spaces + +\begin{theorem} +Let $K$ be a [[Definition:Division Ring|division ring]]. +Let $G$ be a [[Definition:Finitely Generated Module|finitely generated]] [[Definition:Vector Space|$K$-vector space]]. +Let $H$ be a [[Definition:Linearly Independent Set|linearly independent subset]] of $G$. +Let $F$ be a [[Definition:Finite Set|finite]] [[Definition:Generator of Vector Space|generator]] for $G$ such that $H \subseteq F$. +Then there is a [[Definition:Basis of Vector Space|basis]] $B$ for $G$ such that $H \subseteq B \subseteq F$. +\end{theorem} + +\begin{proof} +Let $\mathbb S$ be the [[Definition:Set|set]] of all $S \subseteq G$ such that $S$ is a [[Definition:Generator of Vector Space|generator]] for $G$ and that $H \subseteq S \subseteq F$. +Because $F \in \mathbb S$, it follows that $\mathbb S \ne \O$. +Because $F$ is [[Definition:Finite Set|finite]], then so is every [[Definition:Element|element]] of $\mathbb S$. +Let $R = \set {r \in \Z: r = \card S \in \mathbb S}$. +That is, $R$ is the set of all the integers which are the number of [[Definition:Element|elements]] in [[Definition:Generator of Vector Space|generators]] for $G$ that are subsets of $F$. +Let $n$ be the smallest [[Definition:Element|element]] of $R$. +Let $B$ be an [[Definition:Element|element]] of $\mathbb S$ such that $\card B = n$. +We note that as $H$ is a [[Definition:Linearly Independent Set|linearly independent set]], it does not contain $0$ by [[Subset of Module Containing Identity is Linearly Dependent]]. +Then $0 \notin B$, or $B \setminus \set 0$ would be a [[Definition:Generator of Vector Space|generator]] for $G$ with $n - 1$ [[Definition:Element|elements]]. +This would contradict the definition of $n$. +Let $m = \card H$. +Let $\sequence {a_n}$ be a [[Definition:Sequence of Distinct Terms|sequence of distinct vectors]] such that $H = \set {a_1, \ldots, a_m}$ and $B = \set {a_1, \ldots, a_n}$. +Suppose $B$ were [[Definition:Linearly Dependent Set|linearly dependent]]. +By [[Linearly Dependent Sequence of Vector Space]], there would exist $p \in \closedint 2 n$ and [[Definition:Scalar (Vector Space)|scalars]] $\mu_1, \ldots, \mu_{p - 1}$ such that $\displaystyle a_p = \sum_{k \mathop = 1}^{p - 1} \mu_k a_k$. +As $H$ is [[Definition:Linearly Independent Set|linearly independent]], $p > m$ and therefore $B' = B \setminus \set {a_p}$ would contain $H$. +{{explain|why $H$ being L.I. implies $p > m$}} +Now if $\displaystyle x = \sum_{k \mathop = 1}^n \lambda_k a_k$, then: +:$\displaystyle x = \sum_{k \mathop = 1}^{p - 1} \paren {\lambda_k + \lambda_p \mu_k} a_k + \sum_{k \mathop = p + 1}^n \lambda_k a_k$ +Hence $B'$ would be a [[Definition:Generator of Vector Space|generator]] for $G$ containing $n - 1$ [[Definition:Element|elements]], which contradicts the definition of $n$. +Thus $B$ must be [[Definition:Linearly Independent Set|linearly independent]] and hence is a [[Definition:Basis of Vector Space|basis]]. +{{qed}} +{{Proofread}} +\end{proof}<|endoftext|> +\section{Finitely Generated Vector Space has Basis} +Tags: Bases of Vector Spaces + +\begin{theorem} +Let $K$ be a [[Definition:Division Ring|division ring]]. +Let $V$ be a [[Definition:Finitely Generated Module|finitely generated]] [[Definition:Vector Space|vector space]] over $K$. +Then $V$ has a [[Definition:Finite Set|finite]] [[Definition:Basis of Vector Space|basis]]. +\end{theorem} + +\begin{proof} +This follows from [[Vector Space has Basis Between Linearly Independent Set and Finite Spanning Set]]. +It suffices to find: +:A [[Definition:Linearly Independent Set|linearly independent]] [[Definition:Subset|subset]] $L\subset V$ +:A [[Definition:Finite Set|finite]] [[Definition:Generator of Vector Space|generator]] $S\subset V$ +with $L\subset S$. +By [[Empty Set is Linearly Independent]], we make take $L = \O$ and $S$ any [[Definition:Finite Set|finite]] [[Definition:Generator of Vector Space|generator]], which exists because $V$ is [[Definition:Finitely Generated Module|finitely generated]]. +{{qed}} +\end{proof}<|endoftext|> +\section{Bases of Finitely Generated Vector Space have Equal Cardinality} +Tags: Bases of Vector Spaces + +\begin{theorem} +Let $K$ be a [[Definition:Division Ring|division ring]]. +Let $G$ be a [[Definition:Finitely Generated Module|finitely generated]] [[Definition:Vector Space|$K$-vector space]]. +Then any two [[Definition:Basis of Vector Space|bases]] of $G$ are [[Definition:Finite Set|finite]] and [[Definition:Set Equivalence|equivalent]]. +{{improve|I still think there's a simpler way of saying "they have the same number of elements" than bringing in all that top-heavy technical set theoretic language of "set equivalence" and "cardinality" and so on. Since we are talking about a finite set, the complexities which arise with regard to transfinites do not arise, so there is no direct need to go into such depth.}} +\end{theorem} + +\begin{proof} +Since a [[Definition:BasisBasis of Vector Space|basis]] is, by definition, both [[Definition:Linearly Independent Set|linearly independent]] and a [[Definition:Generator of Vector Space|generator]], the result follows directly from [[Size of Linearly Independent Subset is at Most Size of Finite Generator]]. +{{qed}} +\end{proof}<|endoftext|> +\section{Linearly Independent Subset also Independent in Generated Subspace} +Tags: Linear Algebra + +\begin{theorem} +Let $G$ be a [[Definition:Finitely Generated Module|finitely generated]] [[Definition:Vector Space|$K$-vector space]]. +Let $S$ be a [[Definition:Linearly Independent Set|linearly independent subset]] of $G$. +Let $M$ be the [[Definition:Vector Subspace|subspace]] of $G$ [[Definition:Generator of Module|generated]] by $S$. +If $M \ne G$, then $\forall b \in G: b \notin M$, the set $S \cup \set b$ is [[Definition:Linearly Independent Set|linearly independent]]. +\end{theorem} + +\begin{proof} +Suppose that: +:$\displaystyle \sum_{k \mathop = 1}^n \lambda_k x_k + \lambda b = 0$ +where $\sequence {x_n}$ is a [[Definition:Sequence of Distinct Terms|sequence of distinct vectors]] of $S$. +If $\displaystyle \lambda \ne 0$, then $\displaystyle b = -\lambda^{-1} \paren {\sum_{k \mathop = 1}^n \lambda_k x_k} \in M$ which contradicts the definition of $b$. +Hence $\lambda = 0$, and so: +:$\displaystyle \sum_{k \mathop = 1}^n \lambda_k x_k = 0$ +Therefore $\lambda_1 = \lambda_2 = \cdots = \lambda_n = \lambda = 0$ as $S$ is [[Definition:Linearly Independent Set|linearly independent]]. +{{qed}} +\end{proof}<|endoftext|> +\section{Sufficient Conditions for Basis of Finite Dimensional Vector Space} +Tags: Generators of Vector Spaces, Bases of Vector Spaces, Sufficient Conditions for Basis of Finite Dimensional Vector Space + +\begin{theorem} +Let $K$ be a [[Definition:Division Ring|division ring]]. +Let $n \ge 0$ be a [[Definition:Natural Number|natural number]]. +Let $E$ be an [[Definition:Dimension of Vector Space|$n$-dimensional]] [[Definition:Vector Space|vector space]] over $K$. +Let $B \subseteq E$ be a [[Definition:Subset|subset]] such that $\card B = n$. +{{TFAE}} +: $(1): \quad$ $B$ is a [[Definition:Basis of Vector Space|basis]] of $E$. +: $(2): \quad$ $B$ is [[Definition:Linearly Independent Set|linearly independent]]. +: $(3): \quad$ $B$ is a [[Definition:Generator of Module|generator]] for $E$. +\end{theorem} + +\begin{proof} +=== 1 implies 2 and 3 === +Let $B$ be a [[Definition:Basis of Vector Space|basis]] of $E$. +Then conditions $(2)$ and $(3)$ follow directly by the definition of [[Definition:Basis of Vector Space|basis]]. +{{qed|lemma}} +=== 2 implies 1 === +Let $B$ be [[Definition:Linearly Independent Set|linearly independent]]. +Suppose $B$ does not [[Definition:Generator of Vector Space|generate]] $E$. +Then, because $\card B = n$, by [[Linearly Independent Subset also Independent in Generated Subspace]] there would be a [[Definition:Linearly Independent Set|linearly independent subset]] of $n + 1$ [[Definition:Vector (Linear Algebra)|vectors]] of $E$. +But this would contradict [[Size of Linearly Independent Subset is at Most Size of Finite Generator]]. +Thus condition $(2)$ implies $(1)$. +{{qed|lemma}} +=== 3 implies 1 === +Let $B$ be a [[Definition:Generator of Vector Space|generator]] for $E$. +By [[Vector Space has Basis Between Linearly Independent Set and Finite Spanning Set]], $B$ contains a [[Definition:Basis of Vector Space|basis]] $B'$ of $E$. +But $B'$ has $n$ elements and hence $B' = B$ by [[Bases of Finitely Generated Vector Space have Equal Cardinality]]. +Thus condition $(3)$ implies $(1)$. +{{Qed}} +\end{proof} \ No newline at end of file